You are on page 1of 438

Legal Aptitude

and
Legal Reasoning
(for the CLAT and LLB Entrance Tests)

A. P. BHARDWAJ

ChandigarhyDelhiyChennai

CLAT-Prelim Pages.indd 1 13/09/2010 10:55:35


The aim of this publication is to supply information taken from sources believed to be valid
and reliable. This is not an attempt to render any type of professional advice or analysis, nor
is it to be treated as such. While much care has been taken to ensure the veracity and
accuracy of the information presented within, neither the publisher nor its authors bear any
responsibility for any damage arising from inadvertent omissions, negligence or inaccuracies
(typographical or factual) that may have found their way into this book.

Copyright © 2011 Dorling Kindersley (India) Pvt. Ltd


Licensees of Pearson Education in South Asia

No part of this eBook may be used or reproduced in any manner whatsoever without the
publisher’s prior written consent.

This eBook may or may not include all assets that were part of the print version. The
publisher reserves the right to remove any material present in this eBook at any time.

ISBN 9788131755204
eISBN 9789332510777

Head Office: A-8(A), Sector 62, Knowledge Boulevard, 7th Floor, NOIDA 201 309, India
Registered Office: 11 Local Shopping Centre, Panchsheel Park, New Delhi 110 017, India

CLAT-Prelim Pages.indd 2 13/09/2010 10:55:36


With love for my daughter

Anaadi

as a gift on her first birthday

CLAT-Prelim Pages.indd 3 13/09/2010 10:55:36


This page is intentionally left blank.

CLAT-Prelim Pages.indd 4 13/09/2010 10:55:36


Contents

Preface to the Second Edition vii


Acknowledgements ix
About the Author x

Chapter 1
Legal Terms and Maxims 1–16
Multiple-Choice Questions 17–30

Chapter 2
Criminal Law 31–51
Multiple-Choice Questions 52–82

Chapter 3
Civil Law 83–85
Multiple-Choice Questions 86–113

Chapter 4
Constitutional Law 114–183
Multiple-Choice Questions 184–248

Chapter 5
Mock Tests 249–318

Chapter 6
Previous Years’ Papers 319–427

CLAT-Prelim Pages.indd 5 13/09/2010 10:55:36


This page is intentionally left blank.

CLAT-Prelim Pages.indd 6 13/09/2010 10:55:36


Preface to the
Second Edition

It gives me immense pleasure to present the 2nd Edition of this book. It is the outcome of
the students’ liking and demand for Legal Aptitude and Legal Reasoning for the CLAT and
LLB Entrance Tests.
In order to facilitate the students to get the feel of the examination, I am incorporating
the original solved papers of previous years of CLAT, Panjab University, University of
Delhi and Symbiosis. It will give you an idea that how much labour goes in to hit the bulls
eye.
On the basis of the feedback from the students, I am also incorporating ten practice
papers for the students to check their level of competence and understanding, and to help
improve it.
I will be happy to receive suggestions. I wish you all the best.

A. P. BHARDWAJ

CLAT-Prelim Pages.indd 7 13/09/2010 10:55:36


Preface

The increasing demand from students preparing for entrance tests for national law
universities and the lack of any good book on this subject inspired me to write this book.
Legal Reasoning and Legal Aptitude for the CLAT has been written keeping in view
the significant role that legal aptitude plays in the entrance tests. The topics covered in this
book range from introductory concepts on the subject to advanced levels of legal niceties,
technicalities and case laws.
The book begins with a preliminary understanding of legal terminology and maxims
and gradually switches over to include civil, criminal and constitutional law. It provides a
comprehensive coverage of all relevant areas of law in the pithiest and profoundest manner.
The MCQs have been meticulously designed to make students understand the practical
application of all the legal concepts. The theoretical portion has been drafted with special
reference to and in the light of bare provisions of law.
I hope this book meets the student’s requirements. I will appreciate any feedback and
suggestion. All the best.

A. P. BHARDWAJ

CLAT-Prelim Pages.indd 8 13/09/2010 10:55:37


Acknowledgements

I graciously acknowledge Pearson and Showick Thorpe ‘in particular’ for inspiring me to
write this book.
I thank my wife Upasana whose contribution, motivation and selfless service has
greatly enabled me to accomplish this gargantuan task.
I am thankful to Suraj Chopra, who typed the script for me and whose relentless service
and expertise helped me to finish this work on time.
I am grateful to all my near and dear ones for their love, affection and blessings that
has been a constant source of encouragement to me.
Finally, I acknowledge God who blessed me with a galaxy of friends whose knowledge
and support has profusely contributed to this work.

A. P. BHARDWAJ

CLAT-Prelim Pages.indd 9 13/09/2010 10:55:37


About the Author

A. P. Bhardwaj is Director, Innovation (an institute for


advanced learning established in 2003) and Director,
Solutions (an institute for UGC–NET coaching). He
is a postgraduate in political science from Panjab
University, Chandigarh, and has a degree in law from
the same university. He has 10 years of teaching
experience.
Bhardwaj has single-handedly trained and tutored
more than 1,000 students for admission into national
law universities and other universities such as Panjab University, Delhi
University, Kurukshetra University and Himachal Pradesh University. He has
enabled more than 100 students to join civil and judicial services. He is one
of the patrons of the law colleges of Punjab, Haryana and Himachal Pradesh.
As an educationist, motivator and counselor, he delivers lectures across
Punjab, Haryana, Himachal Pradesh and Chandigarh. He can be contacted at
apb.innovation@gmail.com.

CLAT-Prelim Pages.indd 10 13/09/2010 10:55:37


Chapter 1
LEGAL TERMS AND MAXIMS
List of Legal Terms
Battery
A Beneficiary
Abduction Bigamy
Ab initio Bond
Abortion Burden of proof
Absconding
Accomplice C
Account of profits Capacity to contract
Accusare nemo se debet Capital punishment
A coeli unsque and centum Causation
Acquittal Caveat
Act of God Caveat actor
Actus reus Caveat emptor
Adjournment Caveat subscriptor
Actus reus non facit reum nisi mens sit rea Caveat venditor
Adjudication Compromis d’arbitrage
Admissibility of evidence Condominium
Admission Conjugal rights
Admonition Consensus ad idem
Adverse possession. Consent
Affidavit Conspiracy
Affray
Contempt of court
Aid and abet
(Criminal contempt)
Alienation
Contributory negligence
Alimony
Allegation D
Amicus curiae
Amnesty De facto
Animus Defamation
Antecedents Defendant
A posteriori De jure
A priori Delegated legislation
Arbitration Delegation
Arrest Delegates non potest delegare
Arson Discharge of contract
Asylum Doli capax
Audi alteram partem Double jeopardy
Aut punier aut dedere Dum casta vixerit
Autrefois convict Duress

B E
Bail Embargo
Bailiff Embezzlement
Bailment Equality is equity
Bankruptcy Estoppel

CLAT-1-Legal Terms.indd 1 01/09/2010 17:29:42


2 LEGAL TERMS AND MAXIMS

Ex gratia Moot
Ex nudo pacto non oritur action Mortgage
Ex officio Motive
Ex post facto
Extradition N

F Natural justice
Natural rights
Fiduciary Negligence
Flagrante delicto Negotiable instrument
Nemo debet bis vexari
G Nuisance
Genocide Nullity of marriage

H O
Habeas corpus Obiter dictum
Hijacking Offer
Homicide
Hostage P
Hostile witness Parole
Partnership
I Performance of contract
Ignorantia juris non excusat Per incuriam
Inchoate Perjury
Incriminate Persona non grata
Indemnity Piracy
Indictment Possession
Innuendo Privity
Insanity Privity of contract
Interlocutory Pro bono publico
In terrorem Provocation
Inter vivos Public interest immunity
Intra vires Putative father

J Q
Judgment Qualified privilege
Judicial immunity Quasi judicial
Juris et de jure
R
L
Ratio decidendi
Libel Rejoinder
Lien Remoteness of damage
Liquidator Res gestae
Litigant Res ipsa loquitur
Litigation Res judicata
Respondeat superior
M Restitution in integrum
Maintenance Restitution
Mala fide Restraint of marriage
Malice Restraint of trade
Mensrea Retrospective legislation
Mitigation Revocation of offer

CLAT.indb 2 31/03/2009 11:41:35


LEGAL TERMS AND MAXIMS 3

S U
Sabotage Uberrimae fidei
Simpliciter Ubi jus ibi remedium
Sine die
Slander Ultra vires
Stare decisis Utilitarianism
Subjudice
Sui generic V
Sui juris
Vandalism
T Vicarious liability
Tenancy Void
Tenancy at will Voidable
Tenancy by estoppel Voidable contract
Tortfeasor Volenti non fit injuria
Tortious

Legal Terminology
accusare nemo se debet
A No one is bound to accuse himself or herself.
abduction a coeli unsque ad centum
Wrongfully taking away or detaining another [from the heavens to the centre of the earth]
person, usually by force or fraud. A phrase describing the vertical extent of
ab initio the owner’s right in land.
From the beginning. It includes also the proprietor’s right to use
the property as he may think fit and to prevent
If a contract is void (say for mistake) ab
any other person encroaching thereon, whether
initio, it has the consequence that no innocent
above or below the surface.
third parties can acquire rights under any
subsequent contract. acquittal
A decision by a court that a defendant accused
abortion of a crime is not guilty.
The termination of a pregnancy: a miscarriage
act of God
or the premature expulsion of a foetus from the
An event due to natural causes (storms,
womb before the normal period of gestation is
earthquakes, floods, etc.) so exceptionally
complete. severe that no one could reasonably be expected
It is an offence to induce or attempt to induce to anticipate or guard against it.
an abortion.
actus reus
absconding [a guilty act]
The failure of a person to surrender to the custody The essential conduct or element of a crime that
of a court in order to avoid legal proceedings. must be proved to secure a conviction (compare
accomplice mensrea).
One who is a party to a crime, either as a In most cases the actus reus will simply be
principal or as an accessory. an act (e.g., appropriation of property is the act of
theft) accompanied by specified circumstances
account of profits (e.g., that the property belongs to another).
A remedy that a claimant can claim as an Sometimes, the actus reus may be an
alternative to damages in certain circumstances, omission to act (e.g., failure to prevent death
e.g., in an action for breach of copyright. may be the actus reus of manslaughter) or it may
A successful claimant is entitled to a sum include a specified consequence (death resulting
equal to the monetary gain the defendant has being the consequence required for the actus
made through deceiving the claimant. reus of murder or manslaughter).

CLAT.indb 3 31/03/2009 11:41:35


4 LEGAL TERMS AND MAXIMS

In certain cases, the actus reus may simply has title with the intention of possessing it as
be a state of affairs rather than an act (e.g., being one’s own.
unfit to drive through drink or drugs when in The adverse possessor must occupy the
charge of a motor vehicle on a road). land as if he were entitled to it to the exclusion
actus reus non facit reum nisi mens sit rea of all others. And he must intend to occupy it as
[an act does not make a person guilty of his his own.
crime unless his mind be also guilty]. affidavit
The maxim that forms the basis for defining A sworn written statement of evidence used
actus reus and mensrea as the two elements that mainly to support certain applications and,
must be proved before a person can be convicted in some circumstances, as evidence in court
of a crime. proceedings.
adjournment The person who makes the affidavit must
The postponement or suspension of the hearing/ swear or affirm that the contents are true before
session. a person authorized to take oaths in respect of
particular kind of affidavit.
adjudication
affray
The formal judgment or decision of a court or
The offence of using or threatening, other than
tribunal.
by words alone, unlawful violence. The conduct
admissibility of evidence must be such as would have caused a reasonable
The principles determining whether or not person to fear for his safety, though no such
particular items of evidence may be received by person need be present.
the court. The central principle of admissibility The defendant must intend to use or threaten
is relevance. All evidence that is sufficiently violence or, alternatively, must be aware that his
relevant is admissible and all that is not conduct may be violent or threaten violence.
sufficiently relevant is inadmissible. The offence is found in the Public Order Act
admission 1986, though it can be committed in private as
In civil proceedings, a statement by a party to well as in public places.
litigation or by his duly authorized agent that aid and abet
is adverse to the party’s case. Admission may To assist in the performance of a crime either
be informal (i.e., in a document or by word before or during (but not after) its commission.
of mouth) or formal (i.e., made in a statement Aiding usually refers to material assistance
of case or in reply to a request for further (e.g., providing the tools for the crime).
information). And abetting to lesser assistance (e.g.,
In criminal proceedings, a statement by the acting as a look out or driving a car to the scene
defendant admitting an offence or a fact. of the crime).
admissions may be informal or formal Aiders and abettors are liable to be tried as
An informal admission is called a confession. accessories.
A formal admission may be made either Mere presence at the scene of a crime is
before or at the hearing. But if not made in court, not aiding and abetting. The prosecution must
it must be in writing and signed by the defendant prove that the defendant had knowledge that he
or his legal adviser. was assisting the principal in the commission of
A formal admission may be made in respect the crime.
of any fact about which oral evidence could alienation
be given and is conclusive although it may be The transfer of property (particularly real
withdrawn at any stage with the permission of property) from one person to another.
the court. A plea of guilty to a charge read out in alimony
court is a formal admission. Formerly, financial provision made by a husband
admonition to his wife when they are living apart. Alimony
A reprimand from a judge to a defendant who is now known as maintenance or financial
has been discharged from the further prosecution provision.
of an offence. allegation
adverse possession Any statement of fact in a statement of case,
The occupation of land to which another person affidavit, or indictment.

CLAT.indb 4 31/03/2009 11:41:35


LEGAL TERMS AND MAXIMS 5

amicus curiae rant of arrest signed by a magistrate, which must


[a friend of the court or tribunal] be shown to the accused (though not necessarily
A non-party who gives evidence before the at the time of arrest).
court so as to assist it with research, argument, However, a warrant is not required for
or submissions. indictable offences and the Serious Organized
amnesty Crime and Police Act 2005 effectively gives the
An act erasing from legal memory some aspect police the power to arrest any person where they
of criminal conduct by an offender. It is most consider this to be necessary for any of the wide
frequently granted to groups of people in range of reasons. When an arrest is made, the
respect of political offences and is wider than accused must be told that he is being arrested
a pardon, which merely relieves an offender of and given the ground for his arrest.
punishment. Anyone making or assisting in an arrest
may use as much force as reasonable in the
animus circumstances. Resisting lawful arrest may
[Intention] constitute the crime of assault or obstructing a
The term is often used in combination; for police officer. A person who believes he has been
example, animus furandi—the intention to steal; wrongfully arrested may petition for habeas
animus manendi—the intention to remain in one corpus and may sue the person who arrested him
place (for the purposes of the law relating to for false imprisonment.
domicile).
arson
antecedents The intentional or reckless destruction or
An accused or convicted person’s previous damaging of property by fire without a lawful
convictions or history of bad character. excuse.
a posteriori asylum
[from the later (i.e., from effect to cause)] Refuge granted to an individual whose extradi-
Describing or relating to reasoning based on tion is sought by a foreign government or who is
deductions from observation or known facts, fleeing persecution in his native state.
i.e., inductive reasoning. This can include refuge in the territory of
a priori a foreign country (territorial asylum) or in a
[from the previous (i.e., from cause to effect)] foreign embassy (diplomatic asylum).
Describing or relating to reasoning that is audi alteram partem
based on abstract ideas, anticipates the effects [hear the other side]
of particular causes, or (more loosely) makes No one should be condemned unheard.
a presumption that is true as far as known. i.e.,
aut punier aut dedere
deductive reasoning.
[either punish or surrender]
arbitration In extradition law, the doctrine is that offenders
The determination of a dispute by one or more must be either punished by the state of refuge or
independent third parties (the arbitrators) rather surrendered to the state that can and will punish
than by a court. Arbitrators are appointed by them.
the parties in accordance with the terms of the
autrefois convict
arbitration agreement or in default by a court. An
[previously convicted]
arbitrator is bound to apply the law accurately
A special plea claiming that the defendant has
but may in general adopt whatever procedure he
previously been convicted by a court of com-
chooses and is not bound by the exclusionary
petent jurisdiction of the same (or substantially
rules of the law of evidence; he must, however,
the same) offence as that with which he is now
conform to the rules of natural justice.
charged or that he could have been convicted on
arrest an earlier indictment of the same (or substan-
The apprehension of a person suspected of tially the same) offence.
criminal activities. When this plea is entered, the judge
Most arrests are made by police officers, determines the issue. If the plea is successful
although anybody may, under prescribed condi- it bars further proceedings on the indictment.
tions, effect an arrest. The plea may be combined with one of not
In some cases, the officer will have a war- guilty. The Criminal Justice Act, 2003, allows

CLAT.indb 5 31/03/2009 11:41:35


6 LEGAL TERMS AND MAXIMS

for retrial in a limited range of offences where There is also a special defence if the
there is new and compelling evidence that the accused’s spouse has been absent for at least
acquitted person is guilty of the offence and a seven years, and is therefore presumed by the
retrial is in the interests of justice. accused to be dead, even if he does not have
positive proof of the death.
B Even though a person is found not guilty
of the crime of bigamy, the bigamous marriage
bail will still be void if that person had a spouse
The release by the police, magistrate or court of living at the time that the second marriage was
a person held in legal custody while awaiting celebrated.
trial or appealing against a criminal conviction.
bond
Conditions may be imposed on a person
A deed by which one person (the obligor)
released on bail by the police.
commits himself to another (the obligee) to do
A person granted bail undertakes to pay a
something or refrain from doing something.
specified sum to the court if he fails to appear on
the date set by the court. burden of proof
The duty of a party to litigation to prove a fact
bailiff
or facts in issue, generally the burden of proof
An officer of a court concerned with the service
falls upon the party who substantially asserts the
of the court processes and the enforcement of truth of a particular fact.
its orders, especially warrants of execution The burden of proof lies on prosecution or
authorizing the seizure of the goods of a debtor. claimant or plaintiff.
bailment
The transfer of the possession of goods by the C
owner (the bailor) to another (the bailee) for
a particular purpose. Examples of bailments capacity to contract
are the hiring of goods, the loan of goods. The Competence to enter into a legally binding
pledge of goods, and the delivery of goods for agreement.
carriage, safe custody, or repair or bailment of The capacity to contract is one of the essen-
luggage at the cloak room at a railway station. tial ingredient to enter into a valid contract.
The minors or the people of unsound mind
bankruptcy
have no capacity to enter into a contract.
The state of a person who has been adjudged by
The contract with such people are void ab
a court to be insolvent.
initio.
battery
capital punishment
The intentional or reckless application of
Death imposed as a punishment for crime.
physical force to another person.
causation
beneficiary
The relationship between an act and the
[One who benefits from a will]
consequences it produces.
A person entitled to benefit from a trust.
It is one of the elements that must be
The beneficiary holds a beneficial interest in proved before an accused can be convicted of a
the property of which a trustee holds the legal crime in which the effect of the act is part of the
interest. A beneficiary was formerly known as definition of the crime (e.g., murder).
the cestui que trust. Usually, it is sufficient to prove that the
bigamy accused had mensrea (intention or recklessness)
The act of going through a marriage ceremony in relation to the consequences; the burden of
with someone when one is already lawfully proof is on the prosecution.
married to someone else. In tort, it must be established that the defen-
Bigamy is a crime, punishable by up to dant’s tortuous conduct caused or contributed to
seven years’ imprisonment; however, there is a the damage to the claimant before the defendant
defence if the accused honestly and reasonably can be found liable for that damage.
believed that his or her first spouse was dead or Sometimes, a distinction is made between
that their previous marriage had been dissolved the effective or immediate cause (causa cau-
or annulled or was void. sans) of the damage and any other cause in the

CLAT.indb 6 31/03/2009 11:41:35


LEGAL TERMS AND MAXIMS 7

sequence of event leading up to it (causa sine include the right to the others consortium com-
qua non). pany, cohabitation (sexual intercourse), and
Simple causation problems are solved maintenance during the marriage.
by the “but for” test (would the damage have consensus ad idem
occurred but for the defendant’s tort), but this [Agreement on the same thing]
test is inadequate (tortfeasors would each have The agreement by contracting parties to identical
been sufficient to produce the damage). terms that is necessary for the formation of a
caveat legally binding contract.
[Let him beware] consent
A notice, usually in the form of an entry in a Agreement by choice, by one who has the free-
register, to the effect that no action of a certain dom and capacity to make that choice. Consent
kind may be taken without first informing the is essential in a number of circumstances. For
person who gave the notice (the caveator). example, contracts and marriages are invalid
For example, if there are doubts to the unless both parties give their consent. Consent
validity of a will, or who is entitled to administer must be given freely, without duress or decep-
an estate, or reservations about the executors, the tion, and with sufficient legal competence to
lodging of a caveat by any person at any Probate give it.
Registry assures that a grant of representation
is not issued without the caveator being notified conspiracy
of the application for a grant and being given An agreement between two or more people
chance to object. to behave in a manner that will automatically
constitute an offence by at least one of them
caveat actor
(e.g., two people agree that one of them shall
Let the doer be on his guard.
steal while the other waits in a getaway car).
caveat emptor
contempt of court
[Let the buyer be beware]
(Civil contempt)
A common law maxim warning a purchaser that
Disobedience to a court order or process, such
he could not claim that his purchases were de-
as breach of an injunction. If an injunction
fective unless he protected himself by obtaining
is served on a defendant with a penal notice
express guarantees from the vendor.
attached, breach of the injunction can result in
caveat subscriptor the defendant being imprisoned.
Let the person signing (e.g., a contract) be on (Criminal contempt)
his guard. Conduct that obstructs or tends to obstruct the
caveat venditor proper administration of justice.
Let the seller be on his guard. contributory negligence
compromis d’arbitrage A person’s carelessness for his own safety or
Agreements between states to submit disputes interests, which contributes materially to dam-
between them to an arbitration tribunal. age suffered by him as a result partly of his own
condominium fault and partly of the fault of another person or
Joint sovereignty over a territory by two or more persons.
states (the word is also used for the territory Thus, careless driving, knowingly traveling
subject to joint sovereignty). For example, with a drunken driver, and failure to wear a
the New Hebrides Islands in the South Pacific seat belt are common forms of contributory
were a Franco British condominium until 1980. negligence in highway accidents.
Sovereignty is joint, but each jointly governing
power has separate jurisdiction over its own D
subjects. de facto
Individual ownership of a part of a building [in fact]
(e.g., a flat in a block of flats) combined with Existing as a matter of fact rather than of right.
common ownership of the parts of the building As Prime Minister of India is the de facto
used in common. head of the government though the President
conjugal rights is the de jure or nominal or titular head of the
The rights of either spouse of a marriage, which state.

CLAT.indb 7 31/03/2009 11:41:35


8 LEGAL TERMS AND MAXIMS

defamation doli capax


The publication of an untrue statement about [Capable of wrong]
a person that tends to lower his reputation in A child under the age of 10 is deemed
the opinion of right thinking members of the incapable of committing any crime.
community or to make them shun or avoid him. Above the age of 10, children are doli capax
Defamation is usually in words, but pictures, and are treated as adults. Although they will
gestures, and other acts can be defamatory. usually be tried in special youth courts (with the
exception of homicide and certain other grave
defendant
offences) and subject to special punishments.
A person against whom court proceedings are
Formerly, there was a rebuttable presump-
brought. tion that a child between the ages of 10 and 14
de jure was also doli incapax (incapable of wrong). This
As a matter of legal right. presumption has now been abolished.
The President of India is the de jure head double jeopardy
of the state. No person may be twice put in jeopardy of life
delegated legislation or limb for the same offence. A defence to a
[subordinate legislation] prosecution for a crime, raising the claim that
Legislation made under powers conferred by the accused is being placed on trial for a second
an Act of Parliament (an enabling statute, often time for the same offence.
called the Parent Act). dum casta vixerit
The bulk of delegated legislation is As long as she lives chastely.
governmental: it consists mainly of Order in A clause sometimes inserted in a separation
Council and instruments of various names agreement, freeing the husband from the terms
(e.g., order, regulations, rules, directions, and of the agreement (e.g., maintenance obligations)
schemes) made by ministers. Its primary use is if his wife commits adultery.
to supplement Acts of parliament by prescribing duress
the detailed and technical rules required for their Pressure, especially actual or threatened physical
operation; unlike an Act, it has the advantage that force, put on a person to act in a particular way.
it can be made (and later amended if necessary) Acts carried out under duress usually have no
without taking up parliamentary time. legal effect.
Delegated legislation is also made by a For example, a contract obtained by duress
variety of bodies outside central government, is voidable.
examples being byelaws, the rules of the
Supreme Court, and the codes of conduct of E
certain professional bodies.
embargo
delegation The detention of ships in port: a type of reprisal.
The grant of authority to a person to act on behalf Ships of a delinquent state may be prevented
of one or more others, for agreed purposes. from leaving the ports of an injured state in
delegates non potest delegare order to compel the delinquent state to make
A person to whom something has been delegated reparation for the wrong done.
cannot delegate further, i.e., one to whom embezzlement
powers and duties have been entrusted cannot The dishonest appropriation by an employee of
entrust them to another. any money or property given to him on behalf
discharge of contract of his employer.
The termination of a contractual obligation. equality is equity
Discharge may take place by: [aequitas est quasi aequalitas]
1. performance of contract A maxim of equity stating that if there are no
2. express agreement, which may involve reasons for any other basis of division of property,
either bilateral discharge or unilateral those entitled to it shall share it equally.
discharge estoppel
3. breach of contract A rule of evidence or a rule of law that prevents
4. frustration of contract a person from denying the truth of a statement

CLAT.indb 8 31/03/2009 11:41:35


LEGAL TERMS AND MAXIMS 9

he has made or from denying the existence of Formerly, certain types of arrest could
facts that he has alleged to exist. The denial only be made when a person was in the act of
must have been acted upon (probably to his committing an offence. The phrase is most
disadvantage) by the person who wishes to take commonly applied to the situation in which
advantage of the estoppel or his position must a person finds his or her spouse in the act of
have been altered as a result. committing adultery. Someone who kills his or
ex gratia her spouse in this situation may have a defence
Done as a matter of favour. Na ex gratia payment of provocation.
is one not required to made by a legal duty.
G
ex nudo pacto non oritur action
[a right of action does not arise out of a naked genocide
agreement] Conduct aimed at the destruction of a national,
Where there is no consideration, there is no ethnic, racial, or religious group.
contract, no cause of action. Killing on a massive scale. For example,
A contract is a valid contract only and only genocide of the Jews at the hands of the Hitler.
if it fulfils the basic essentials of valid contract,
and then and only then it gives rise to cause of H
action.
ex officio habeas corpus
By virtue of holding an office. A prerogative writ used to challenge the
As the Vice-President of India is the ex- validity of a person’s detention, either in official
officio chairman of the Rajya Sabha. custody (e.g., when held pending deportation or
The Prime Minister of India is the ex-officio extradition) or in private hands.
chairman of the Planning Commission. hijacking
ex post facto Seizing or exercising control of an aircraft in
[by a subsequent act] flight by the use or threat of force.
Describing any legal act, such as a statute, that homicide
has retrospective effect. The act of killing a human being. Crimes of
As in the case of Indian Constitution homicide include murder, manslaughter and
Article 20 of the Indian Constitution deals with infanticide.
the ex post facto laws whereunder the civil laws hostage
can have retrospective application whereas A person who is held as a security.
the criminal laws cannot have retrospective
hostile witness
application rather they would have prospective
An adverse witness who willfully refuses to
application i.e., application from future date.
testify truthfully on behalf of the party who called
extradition him. A hostile witness may, with the permission
The surrender by one state to another of a person of the court, be cross-examined by that party
accused or convicted of committing an offence and have put to him a previous statement that is
in the territorial jurisdiction of the latter, which inconsistent with his present testimony. Under
being competent to try and punish him demands the Criminal Justice Act 2003, where the witness
his surrender. admits taking a previous inconsistent statement,
or his pervious inconsistent statement is proved
F against him, the statement is inadmissible as
fiduciary evidence of any matter stated of which oral
Fiduciary relationships are those relationships evidence by him would be admissible.
which are based on faith and trust. The The Zahira Sheikh example in Best Bakery
relationships between doctor and patient, lawyer Case related to the Godhara Riots is the glaring
and client, teacher and student, and husband and example of hostile witness.
wife are fiduciary relationships.
I
fagrante delicto
In the commission of an offence. ignorantia juris non excusat
Literally means to be caught red handed. Ignorance of the law is no excuse, i.e., no

CLAT.indb 9 31/03/2009 11:41:35


10 LEGAL TERMS AND MAXIMS

defence against criminal or other proceedings that is severe enough to prevent a defendant
arising from its breach. from knowing what he did (or what he did was
inchoate wrong).
[Incomplete] A person accused of a crime is presumed
Certain acts, although not constituting a sane and therefore responsible for his acts, but
complete offence, are nonetheless prohibited by he can rebut this presumption and escape a
the criminal law because they constitute steps conviction if he can prove that at the time of
towards the complete offence. These inchoate committing the crime he was insane.
offences include incitement, attempt and For purposes of this defence, insanity is
conspiracy. defined by the McNaghten Rules.
incriminate interlocutory
To charge with a criminal offence. During the course of proceedings.
To indicate involvement in the commission in terrorem
of a criminal offence. A witness in court need [Intimidating]
not answer a question if, in the judge opinion, The doctrine of in terrorem applies to condition
the answer might expose him to the danger of attached to gifts of personal property in wills
criminal prosecution. A witness does not have or elsewhere. Such conditions are in terrorem
this protection when his answer might lead only if it is appartent that the donor does not really
to civil action against him. intend the recipient to lose the gift, but is merely
indemnity making an idle threat; for example, when a
An agreement by one person (X) to pay to donor makes a gift subject to a condition against
another (Y) sums that are owed, or may become marriage without another person consent but
owed, to him by a third person (Z). does not make provision for the disposal of the
It is not conditional on the third person gift if the recipient does not comply with the
defaulting on the payment, i.e., Y can sue X condition. Such conditions are void.
without first demanding payment from Z. inter vivos
If it is conditional on the third person Between living people, if a trust is created inter
default (i.e., if Z remains the principal debtor vivos it is created during lifetime, as distinct
and must be sued for the money first) it is not an from upon death.
indemnity but a guarantee. Unlike a guarantee,
an indemnity need not be evidenced in writing. intra vires
[within powers]
indictment Describing an act carried out by a body (such as
A formal document accusing one or more a public authority or a company) that is within
persons of committing a specified indictable the limits of the powers conferred on it by statute
offence or offences. or some other constituting document (such as
It is read out to the accused at the trial. the memorandum and articles of association of
An indictment is in a particular form. a company).
It is headed with the name of the case and
the place of trial. J
There is then a statement of offence, stating
what crime has allegedly been committed, judgment
followed by particulars of the offence, with A decision made by a court in respect of the
such details as the date and place of the offence, matter before it. Judgments may be interim
property stolen, etc. (interlocutory), deciding a particular issue prior
innuendo to the trial of the case; or final, finally disposing
In an action for defamation, a statement in of the case. They may be in personam, imposing
which the words may be defamatory as a result a personal liability on a party (e.g., to pay
of a hidden meaning. The hidden meaning must damages); or in rem, determining some issue
be capable of being understood from the words of right, status, or property binding people
themselves by people to whom the statement is generally.
published. judicial immunity
insanity The exemption of a judge or magistrate from
[in criminal law] personal actions for damages arising from the
A defect of reasons, arising from mental disease, exercise of his judicial office. The immunity

CLAT.indb 10 31/03/2009 11:41:36


LEGAL TERMS AND MAXIMS 11

is absolute in respect of all words or actions of mensrea


the judge while acting with in his jurisdiction [a guilty mind]
and extends to acts done without jurisdiction The state of mind that the prosecution must prove
provided that they were done in good faith. a defendant to have had at the time of committing
juris et de jure a crime in order to secure a conviction.
Of law and from law; an irrebuttable presumption Mensrea varies from crime to crime; it is
either defined in the statute creating the crime or
is so described.
established by precedent. Common examples of
L mensrea are intention to bring about a particular
consequence. Recklessness as to whether such
libel consequences may come about, and (for a few
A defamatory statement made in permanent crimes) negligence.
form, such as writing, pictures, or film. Some crimes require knowledge of certain
circumstances as part of the mensrea (for
lien example, the crime of receiving stolen goods
The right of one person to retain possession of requires the knowledge that they were stolen).
goods owned by another until the possessor’s Some crimes require no mensrea; these
claim against the owner have been satisfied. are known as crime of strict liability. Whenever
The lien may be general, when the goods are mensrea is required the prosecution must prove
held as security for all outstanding debts of the that it existed at the same time as the sactus
owner, or particular, when only the claims of the reus of the crime (coincidence of actus reus and
possessor in respect of the goods held must be mensrea).
satisfied. A defendant cannot plead ignorance of the
Thus, an unpaid seller may in some con- law, nor is a good motive a defence. He may,
tracts be entitled to retain the goods until he re- however, bring evidence to show that they had
ceives the price. no mensrea for the crime he is charged with;
liquidator alternatively, he may admit that he had mensrea,
A person who conducts the winding up of a but raise a general defence (e.g., duress) or
company. Unless he is the official receiver, he a particular defence allowed in relation to the
must be a qualified insolvency practitioner. crime.
litigant mitigation
Reduction in the severity of some penalty.
A person who is a party to a court action (this
may include a company or corporation). A moot
litigant may present his case personally to the A mock trial, often held in university law
court. schools and at the Inns of Court, for student
as practice for future advocacy. A hypothetical
litigation case is presented to students for preparation
The taking of legal action by a litigant. and then argued before the judge(s) at the moot.
The field of law that is concerned with all This practice originates in the formal moots
contentious matters. held in the medieval Inns of Court, which were
considered an essential part of legal education.
M
mortgage
maintenance An interest in property created as a form of
The provision of food, clothing, and other basic security for a loan or payment of debt and
necessities of life. A husband or wife is obliged terminated on payment of the loan or debt.
to maintain his or her spouse. The borrower, who offers the security, is the
mortagagor; the lender, who provides the money,
mala fide is the mortgagee.
[bad faith]
Describing an act performed fraudulently or motive
dishonestly. The purpose behind a course of action.
malice N
[in criminal law]
A state of mind usually taken to be equivalent to natural justice
intention or recklessness: it does not require any [Rules of fair play]
hostile attitude. Originally developed by the courts of

CLAT.indb 11 31/03/2009 11:41:36


12 LEGAL TERMS AND MAXIMS

equity to control the decision of inferior courts notice of the defect. The most important classes
and then gradually extended (particularly in the of negotiable instruments are bills of exchange
20th Century) to apply equally to the decision (including cheques) and promissory notes.
of administrative and domestic tribunals and nemo debet bis vexari
of any authority exercising an administrative [no man ought to be twice vexed]
power that effects a person’s status, rights, or No person should be twice sued upon the same
liabilities. set of facts if there has been a final decision of a
Any decision reached in contravention of competent court.
natural justice is void as ultra vires. There are
nuisance
two principal rules. The first is the rule against
An activity or state of affairs that interferes with
bias, i.e., against departure from the standard
the use or enjoyment of land or rights over land
of even handed justice required of those who
(private nuisance) or with the health, safety, or
occupy judicial office—nemo judex in causa sua
comfort of the public at large (public nuisance).
(or in propria causa): no man may be a judge in
Private nuisance is a tort, protecting
his own cause.
occupies of land from damage to the land,
This means that any decision however fair
buildings, or vegetation or from unreasonable
it may seem, is invalid if made by a person with
interference with their comfort or convenience
any financial or other interest in the outcome
by excessive noise, dust, fumes, smells, etc.
or any known bias that might have affected his
An action is only available to persons who
impartiality.
have property rights (e.g., owners, lessees) or
natural rights exclusive occupation. Thus, for example, lodgers
The rights conferred on all individuals by the and family members of a property owner cannot
natural law. sue in private nuisance.
The fundamental rights found in civilized Public nuisance is a crime. At common
nations to which all men are entitled without law, it includes such activities as obstruction of
interference by the state. This concept of the highway, carrying on an offensive trade, and
natural law was particularly popular in the 18th selling food unfit for human consumption. The
century. Attorney General or a local authority may bring
It has had great influence in the legal history a civil action for an injunction on behalf of the
of the US, as seen, for example, in the Virginian public but a private citizen may obtain damages
Declaration of rights, in tort only if he can prove some special damage
“All men are by nature, equally free and over and above that suffered by the public at
independent and have certain inherent natural large.
rights of which when they enter a society they
nullity of marriage
cannot by any compact deprive or divest their
The invalidity of a marriage due to some defect
posterity”.
existing at the time the marriage was celebrated
negligence (or, sometimes, arising afterwards). A marriage
Carelessness amounting to the culpable breach may be null in the sense that it is void, i.e., it was
of a duty: failure to do or recognize something never in the eyes of the law a valid marriage (and
that a reasonable person (i.e., an average the “spouses” are legally merely cohabitants).
responsible citizen) would do or recognize, or
doing something that a reasonable person would O
not do. In cases of professional negligence,
involving someone with a special skill, that obiter dictum
person is expected to show the skill of an [a remark in passing]
average member of his profession. Something said by a judge while giving
negotiable instrument judgment that was not essential to the decision
A document that constitutes an obligation in the case.
to pay a sum of money and is transferable by It does not form a part of the ratio decidendi
delivery so that the holder for the time can sue of the case and therefore creates no binding
upon it in his own name. The transferee can precedent, but may be cited as persuasive
enforce the obligation even if the transferor’s authority in later cases.
title is defective, provided that he accepted the offer
document in good faith and for value and had no An indication of willingness to do or refrain

CLAT.indb 12 31/03/2009 11:41:36


LEGAL TERMS AND MAXIMS 13

from doing something that is capable of being result of their participation in some transaction
converted by acceptance into a legally binding or event; for example, privity of contract and
contract. It is made by an offeror to an offeree privity of estate.
and is capable of acceptance only by an offeree
who knows of its existence. privity of contract
The relationship that exists between the parties
P to a contract.
pro bono publico
parole
[for the public good]
The conditional release of a prisoner from
prison. Describing legal work that is carried out unpaid
for the good of the general community.
partnership
An association of two or more people formed provocation
for the purpose of carrying on a business with a Conduct or words causing someone to lose his
view to profit. self control.
performance of contract public interest immunity
The carrying out of obligations under a A doctrine that authorizes the non-disclosure of
contract. information or documents relevant to litigation
per incuriam on the basis that disclosure of such evidence is
[Through lack of care] against the public interest.
A decision of a court is made per incuriam if it
fails to apply a relevant statutory provision or putative father
ignores a binding precedent. A man alleged to be the father of an illegitimate
child. If the court accepts the mother allegations,
perjury
The offence of giving false evidence or evidence the man is declared the putative father and may
that one does not believe to be true (even if it is be ordered to make periodical payments for the
in fact the truth). maintenance of the child.
persona non grata
Q
[an unacceptable or unwelcome person]
A diplomatic agent who is unacceptable to the qualified privilege
receiving state. The sending state should recall
The defence that a statement cannot be made the
such an agent: if this fails to occur the host state
may ignore the presence of the agent or expel subject of an action for defamation because it
him from its territory. was made on a privileged occasion and was not
made maliciously, for an improper motive.
piracy
[piracy jure gentium] Qualified privilege covers statements made
Any illegal act of violence, detention, or robbery fairly in situations in which there is a legal or
committed on a private ship for personal gain moral obligation to give the information and
or revenge, against another ship, people, or the person to whom it is acting in defence of his
property on the high seas. Piracy may also be own property or reputation.
committed on or against an aircraft. Piracy also Qualified privilege also covers fair and
includes operating a pirate ship or aircraft and accurate reports of public meetings and various
inciting or assisting any other act of piracy. other public proceedings. The privilege attaching
possession to professional communications between
Actual control of property combined with the solicitor and client is probably qualified, rather
intention to use it, rightly or wrongly as one’s than absolute.
own. In the case of land, possession may be
actual, when the owner has entered into the quasi judicial
land, or possession in law, when he has the right Describing a function that resembles the judicial
to enter but has not yet done so. Possession function in that it involves deciding a dispute
includes receipt of rent and profits, or the right and ascertaining the facts and any relevant law,
to receive them. but differs in that it depends ultimately on the
privity exercise of an executive discretion rather than
The relationship that exists between people as a the application of law.

CLAT.indb 13 31/03/2009 11:41:36


14 LEGAL TERMS AND MAXIMS

under which statements forming part of the res


R gestae are admissible.
ratio decidendi res ipsa loquitur
[the reason for deciding] [the thing speak for itself]
The principle or principles of law on which the A principle often applied in the law of tort of
court reaches its decision. negligence. If an accident has occurred of a kind
The ratio of the case has to be deduced from that usually only happens if someone has been
its facts, the reasons the court gave for reaching negligent, and the state of affairs that produced
its decision, and the decision itself. the accident was under the control of the
It is said to be statement of law applied to defendant, it may be presumed in the absence
the material facts. of evidence that the accident was caused by the
Only the ratio of a case is binding on inferior defendant negligence.
courts, by reason of the doctrine of precedent. res judicata
rejoinder [a matter that has been decided]
Formerly, a pleading served by a defendant The principle that when a matter has been
in answer to the climants reply. Such a finally adjudicated upon by a court of competent
pleading could only be served with the courts jurisdiction it may not be reopened or challenged
permission. by the original parties or their successors in
remoteness of damage interest.
The extent to which a defendant is liable for the It is also known as action of estoppel. It
consequences of his wrongful act or omission. does not preclude an appeal or a challenge to the
In contract, the defendant compensates for jurisdiction of the court. Its justification is the
damage only if it was within his reasonable con- need for finality in litigation.
templation. He is presumed to have contemplat- respondeat superior
ed (and is, therefore, liable for) damage likely [let the principal answer]
to result from the breach according to the usual The doctrine by which an employer is responsible
course of events. for certain wrongs committed by his employee
Unusual damage resulting from special in the course of his employment.
circumstances is regarded as within his contem- restitution in integrum
plation only if a reasonable man, knowing what Restoration to original position.
he knew or ought to have known, would have it
liable to result. restitution
In tort, there is no single test to determine The return of property to the owner or person
whether or not damage is too remote. In actions entitled to possession.
for negligence and other forms of liability based If one person has unjustifiably received
on fault, the defendant is responsible only for either property or money from another, he has
damage of the type he should have foreseen, an obligation to restore it to the rightful owner
but if damage of that type is foreseeable, it is no in order that he should not be unjustly enriched
defence that the extent of the resulting damage or retain an unjustified advantage.
is greater than could have been expected. This obligation exists when, for example,
In torts of strict liability, the defendant goods or money have been transferred under
may be liable even for unforeseeable damage. compulsion.
Thus, the keeper of an animal belonging to a Under mistake or under a transaction that
dangerous species is liable for any damage it fails because of illegality, lack of formality, or
causes, whether foreseeable or not. for any other reason or when the person who has
res gestae taken the property has acquired a benefit through
[things done] his actions without justification.
The events with which the court is concerned or restraint of marriage
others contemporaneous with them. A condition in a contract or other disposition
In the law of evidence, res gestae denotes intended to prevent someone from marrying.
both a rule of relevance according to which Such conditions are usually (unless they are
events forming part of the res gestae are very limited) void, as they are considered to be
admissible and an exception to the hearsay rule against public policy.

CLAT.indb 14 31/03/2009 11:41:36


LEGAL TERMS AND MAXIMS 15

restraint of trade damage (actual financial or material loss), not


A contractual term that limits a person’s right to merely loss of reputation.
exercise his trade to carry on his business. An Proof of special damage is not necessary
example is a term in an employment contract or when the slander implies the commission of a
partnership agreement prohibiting a party from criminal offence punishable by imprisonment,
engaging in a similar business for a specified infection with a contagious disease, unchastity in
period after the relationship ends. Such a term a woman, or is calculated to disparage a person
is void unless the party relying on it shows that in his office, business, trade, or profession.
it does not offend public policy; it must also be stare decisis
reasonable as between the parties. [to stand by things decided]
retrospective legislation A maxim expressing the underlying basis of the
[retroactive legislation] doctrine of precedent , i.e., that it is necessary
Legislation that operates on matters taking to abide by former precedents when the same
points arise again in litigation.
place before its enactment, e.g., by penalising
conduct that was lawful when it occurred. There subjudice
is a presumption that statutes are not intended A rule limiting comment and disclosure relating
to have retroactive effect unless they merely to judicial proceedings, in order not to prejudge
change legal procedure. the issue or influence.
revocation of offer sui generic
The withdrawal of an offer by the offeror so that [of its own kind]
it can no longer be accepted. Revocation takes Forming a class of its own; unique.
effect as soon as it is known to the offeree (from sui juris
whatever source); offer can be revoked at any [of his own right]
time before acceptance unless they are coupled Describing the status of a person who is of full
with an option. age and capacity.

S T

sabotage tenancy
Damage to or destruction of property, especially Broadly, the interest of one who holds land by
the property of an employer during a strike or of any right or title. The term is often used in a more
the state for political reasons. Sabotage as such restricted sense, however, for the arrangement in
which the owner (the landlord) allows another
is not an offence, although it may be treated as
person (the tenant) to take possession of the land
criminal damage.
for an agreed period, usually in return for rent.
simpliciter
tenancy at will
Indicating that a word or phrase in a document
A tenancy that can be terminated by the landlord
is used absolutely, unconditionally, and free
or the tenant at any time.
from any shades of meaning given to it by
surrounding words or phrases. For example, if tenancy by estoppel
the word ‘yard’ is found in a document it means A tenancy that exists despite the fact that the
that the word is used in its most natural sense. person who granted it had no legal right to do
so.
Thus it is not a “stockyard”, which is a particular
type of yard. tortfeasor
One who commits a tort.
sine die
[without a date] tortious
To adjourn a case sine die is to adjourn it Having the nature of a tort; wrongful.
without setting a date for a future hearing.
U
slander
A defamatory statement made by such means as uberrimae fidei
spoken words or gestures, i.e., not in permanent [of the utmost good faith]
form. Describing a class of contracts in which one party
Generally, slander is only actionable on has a preliminary duty to disclose material facts
proof that its publication has caused special relevant to the subject matter to the other party.

CLAT.indb 15 31/03/2009 11:41:36


16 LEGAL TERMS AND MAXIMS

Nondisclosure makes the contract viodable. An employer is vicariously liable for


Example of this class are insurance contracts, torts committed by his employees when he has
In which knowledge of many material facts is authorized or ratified them or when the tort
confined to the party seeking insurance. was committed in the course of the employee’s
ubi jus ibi remedium work.
[where there is a wrong there is a remedy] Thus, negligent driving by someone em-
The principle that where one right is invaded or ployed as a driver is a tort committed in the
destroyed, the law gives a remedy to protect it or course of his employment, but if the driver were
damages for its loss. Further, where one right is to assault a passing pedestrian for motives of
denied the law affords the remedy of an action private revenge, the assault would not be con-
for its enforcement. nected with his job and his employer would not
ultra vires be liable.
[beyond the powers] The test is whether the tort is so closely
Describing an act by a public authority. connected with the employment that it would
Company, or other body that goes beyond the be fair and just to hold the employer vicariously
limits of the powers conferred on it. liable.
utilitarianism void
The approach to morality that regards pleasure Having no legal effect.
or the satisfaction of desire as the exclusive voidable
element in human good, and treats the morality Capable of being avoided (set aside).
of acts and rules as wholly dependent on the
voidable contract
consequences for human welfare. Its premise is
A contract that, though valid when made, is
the proposition that the fundamental objective
liable to be subsequently set aside. Voidable
of morality and justice is that happiness should
contracts may arise through misrepresentation,
be maximized. This philosophy was given by
some instance of mistake, nondisclosure, and
Jermey Bentham.
duress. Certain proprietary contracts entered into
V by minors are also viodable. The setting aside of
a viodable contract is effected by rescission.
vandalism volenti non fit injuria
[Defacing or damaging property] [no wrong is done to one who consents]
There is no offence of vandalism as such, but The defence that the claimant consented to the
it will usually constitute an offence of criminal injury or (more usually) to the risk of being
damage. injured. Knowledge of the risk of injury is not
vicarious liability sufficient; there must also be (even if only by
[vicarious responsibility] implication) full and free consent to bear the
Legal liability imposed on one person for risk. A claimant who has assumed the risk of
torts or crimes committed by another (usually injury has no action if the injury occurs. The
an employee but sometimes an independent scope of the defence is limited by statute in
contractor or agent), although the person made cases involving business liability and public and
vicariously liable is not personally at fault. private transport.

CLAT.indb 16 31/03/2009 11:41:36


LEGAL TERMS AND MAXIMS 17

MULTIPLE-CHOICE QUESTIONS
1. Audi aultrem paltrem means (c) The court an initiate any action against
(a) No one should be condemned un- the guilty person and convict him on
heard the basis of his action
(b) When the thing speaks for itself (d) None of these
(c) It is a principle of extradition law 7. The burden of proof lies on
(d) None of these (a) claimant
2. Res ispa loquitur means (b) prosecution
(a) No one should be condemned un- (c) both (a) and (b)
heard (d) defendant
(b) When the thing speaks for itself 8. Adverse possession means
(c) It is a principle of extradition law (a) lawful possession
(d) None of these (b) unlawful possession
3. Volenti non fit injuria means (c) neither (a) nor (b)
(a) No wrong is done to one who con- (d) custody under the judicial authority
sents 9. When a person aids and abets in the
(b) No one should be condemned un- performance of a crime either before or
heard during (but not after) its commission
(c) When the thing speaks for itself (a) the aiders and abettors are liable to be
(d) It is a principle of extradition law tried as accessories
4. Who is amicus curiae? (b) the aiders and abettors are not liable to
(a) A friend of the court or tribunal i.e., be tried as accessories
a non party who gives evidence be- (c) because aiding and abetting is no of-
fore the court so as to assist it with fence so no case can be made out
(d) None of these
research, argument or submissions.
10. Aut punier aut dedere means
(b) He is a party to the case.
(a) either punish or surrender. It is a doc-
(c) He is a public prosecutor otherwise
trine of extradition law, that offenders
designated as Advocate General.
must be either punished by the state of
(d) None of these
refuge or surrendered to the state that
5. When a contract is void ab initio?
can and will punish them.
(a) it is void from the beginning and if a
(b) it is a principle of natural justice that
contract is void (say for mistake) ab initio,
when someone takes refuge in anoth-
this has the consequence that no innocent er stage, he cannot be punished at all
third parties can acquire rights under any and he deserves to be protected at any
subsequent contract. cost
(b) Only the parties to the contract can (c) neither (a) nor (b)
claim their rights. (d) all of these
(c) It is a discretion of the court to make 11. Match the following:
its specific performance. A. Caveat 1. Let the seller be on his
(d) None of these guard
6. Actus reus non facit reum nisi mens sit rea B. Caveat actor 2. Let the person sign-
means ing (e.g., a contract)
(a) an act does not make a person guilty be on his guard
of his crime unless his mind be also C. Caveat emptor 3. Let the buyer be
guilty. The maxim that forms the basis beware
for defining actus reus and mensrea as D. Caveat 4. Let the doer be on his
the two elements that must be proved subscriptor guard
before a person can be convicted of a E. Caveat 5. Let him beware
crime. venditor
(b) It means that the action is sufficient to A B C D E
convict a person, the intention is irrel- (a) 5 4 3 2 1
evant (b) 1 2 3 4 5

CLAT.indb 17 31/03/2009 11:41:36


18 LEGAL TERMS AND MAXIMS

(c) 1 2 5 4 3 (c) neither (a) nor (b)


(d) 1 3 4 2 5 (d) both (a) and (b)
(e) 1 5 4 2 3 20. When any law is said to be intra vires
12. The contempt of court places (a) it means it is within the powers of leg-
(a) restrictions on the civil liberty islature and it does not go beyond the
(b) it curtails the right to life and liberty supreme and fundamental law of the
(c) it means the arbitrary misuse of pow- land
ers by the court (b) it means it is not within the powers
(d) None of these of legislature and it goes beyond the
13. The President of India is supreme and fundamental law of the
(a) defecto head land
(b) de jure head (c) neither (a) nor (b)
(c) neither (a) nor (b) (d) both (a) and (b)
(d) real head 21. Mensrea means
14. The Prime Minister of India is (a) guilty mind
(a) defecto head (b) innocent mind
(b) de jure head (c) illegal mind
(c) neither (a) nor (b) (d) None of these
(d) real head 22. When a person is declared persona non
15. Delegates non potest delegare means grata
(a) Delegated legislation cannot be fur- (a) it means a diplomatic agent is unac-
ther delegated ceptable to the receiving state and the
(b) Delegated legislation is meant to be sending state should recall such an
further delegated
agent
(c) neither (a) nor (b)
(b) it means a person is not of good repu-
(d) both (a) and (b)
tation
16. A person who is capable of wrong is
(c) it means a person is not of good char-
known as
acter
(a) doli capax
(d) None of these
(b) doli incapax
23. Obiter dictrum means
(c) neither (a) nor (b)
(a) A remark in the passing that is some-
(d) both (a) and (b)
thing said by a judge while giving
17. A person who is incapable of wrong is
known as judgement that was not essential to
(a) doli capax the decision in the case.
(b) doli incapax (b) It is a part of ratio decendie of the
(c) neither (a) nor (b) case and, therefore, creates binding
(d) both (a) and (b) precedent.
18. Double jeopardy means (c) These are the valid observations.
(a) No person can be punished twice for (d) None of these
the same offence 24. Ratio decendie means
(b) One who repeats the same offence can (a) observations passed by the court
be punished twice which are not binding
(c) neither (a) nor (b) (b) it is the ratio of the case that is bind-
(d) both (a) and (b) ing on inferior courts by reason of
19. When any law is said to be ultra vires doctrine of precedent
(a) it means it is within the powers of leg- (c) these are the technical terms to be
islature and it does not go beyond the decided by non judicial officers
supreme and fundamental law of the (d) None of these
land 25. Res judicata is known as
(b) it means it is not within the powers (a) action of estoppel
of legislature and it goes beyond the (b) it is same as ratio decendie
supreme and fundamental law of the (c) neither (a) nor (b)
land (d) both (a) and (b)

CLAT.indb 18 31/03/2009 11:41:36


LEGAL TERMS AND MAXIMS 19

26. When a legislation is said to be retrospec- Directions (Q. 33 to 60): Match the following:
tive legislation 33. A. Abduction 1. The failure of a per-
(a) it means it can be applied from back son to surrender to the
date custody of a court in
(b) it means it cannot be applied from order to avoid legal
back date proceedings.
(c) neither (a) nor (b) B. Abortion 2. The termination of a
(d) both (a) and (b) pregnancy, a miscar-
27. When a legislation is said to be prospective riage or the premature
legislation expulsion of a foetus
(a) it means it can be applied from future from the womb before
date the normal period of
gestation is complete.
(b) it means it cannot be applied from fu-
C. Absconding 3. Wrongfully taking
ture date
away or detaining an-
(c) neither (a) nor (b)
other person, usually
(d) both (a) and (b)
by force or fraud.
28. Ubi jus ibi remedium means A B C
(a) where there is a wrong there is a rem- (a) 3 2 1
edy (b) 1 3 2
(b) to every wrong there is no remedy (c) 1 2 3
(c) wrong and remedy have no relation (d) 2 3 1
(d) None of these 34. A. Accomplice 1. One who is a party
29. Vicarious liability to a crime, either as
(a) can hold employer/principle as liable a principal or as an
for the actions of his employee or accessory
agent B. Acquittal 2. A decision by a court
(b) cannot hold employer/principle as that a defendant
liable for the actions of his employee accused of a crime is
or agent not guilty
(c) neither (a) nor (b) C. Act of God 3. An event due to natu-
(d) both (a) and (b) ral causes (storms,
30. Tenancy at will earthquakes, floods,
(a) can be terminated by the landlord at etc.) so exceptionally
any time severe that no one
(b) can be terminated by the tenant at could reasonably be
anytime expected to anticipate
(c) can be terminated by the both at any- or guard against it.
time A B C
(d) None of these (a) 2 1 3
31. Stare decisis means (b) 3 1 2
(c) 1 2 3
(a) it is necessary to abide by former
(d) 1 3 2
precedents when the same points arise
35. A. Adjourn- 1. The postponement or
again in litigation.
ment suspensions of the
(b) it is not necessary to abide by former
hearing of a case until
precedents when the same points arise a future date
again in litigation. B. Adjudication 2. The formal judgement
(c) neither (a) nor (b) or decision of a court
(d) both (a) and (b) or tribunal
32. When a hearing is adjourned sine die C. Admissi- 3. The principles deter-
(a) it means it has been adjourned without bility of mining whether or
fixing a future date of hearing evidence not particular items
(b) it means it has been adjourned fixing a of evidence may be
future date of hearing received by the court.
(c) it means the court is abolished The central principle
(d) None of these of admissibility is rel-

CLAT.indb 19 31/03/2009 11:41:37


20 LEGAL TERMS AND MAXIMS

evance. All evidence B. Bailiff 2. Court’s officer


that is sufficiently C. Bailment 3. Release from police
relevant is admissible custody
and all that is not suf- A B C
ficiently relevant is (a) 1 3 2
inadmissible. (b) 2 1 3
A B C (c) 3 2 1
(a) 2 1 3 (d) 2 3 1
(b) 1 2 3 41. A. Bankruptcy 1. Insolvency
(c) 2 3 1 B. Battery 2. Actual use of force
(d) 1 3 2 C. Beneficiary 3. One who benefits
36. A. Admonition1. Offence of using or from will
A B C
threatening unlawful
(a) 1 2 3
violence.
(b) 2 1 3
B. Affidavit 2. A solemn declaration
(c) 3 2 1
true to the best of (d) 2 3 1
the knowledge of the 42. A. Bigamy 1. Second marriage
deponent. when first marriage is
C. Affray 3. A reprimand from a still subsisting
judge to a defendant. B. Capacity to 2. Competence to
A B C contract enter into a legal
(a) 2 3 1 contract
(b) 1 2 3 C. Capital 3. Death punishment
(c) 3 2 1 punishment
(d) 1 3 2 A B C
37. A. Alienation 1. The transfer of prop- (a) 3 2 1
erty (b) 1 2 3
B. Alimony 2. Written statement of (c) 3 1 2
allegation (d) 1 3 2
C. Allegation 3. Maintenance charges 43. A. Condo- 1. Joint sovereignty
to wife from husband minium
A B C B. Conjugal 2. Matrimonial rights
(a) 1 3 2 rights
(b) 2 1 3 C. Consent 3. Agreement by choice
(c) 3 2 1 A B C
(d) 3 1 2 (a) 3 2 1
38. A. Amnesty 1. Political pardon (b) 2 1 3
B. Animus 2. Intention (c) 1 2 3
(d) 2 3 1
C. Antecedents 3. Previous character
44. A. Discharge of 1. The termination
A B C
contract of contractual
(a) 1 2 3
obligation
(b) 2 1 3
B. Duress 2. Pressure/undue influ-
(c) 3 2 1 ence
(d) 2 3 1 C. Embargo 3. The detention of ships
39. A. A posteriori 1. From effect to cause in a port: a type of
B. A priori 2. From cause to effect reprisal
C. Arbitration 3. Mediation A B C
A B C (a) 3 2 1
(a) 1 2 3 (b) 1 2 3
(b) 2 1 3 (c) 3 1 2
(c) 3 2 1 (d) 2 1 3
(d) 2 3 1 45. A. Embezzle- 1. Criminal misappro-
40. A. Bail 1. Luggage at the rail- ment priation of public
way station property

CLAT.indb 20 31/03/2009 11:41:37


LEGAL TERMS AND MAXIMS 21

B. Ex gratia 2. Done as a matter of 51. A. Mitigation 1. Reduction in the


favour severity of some
C. Ex officio 3. By virtue of holding penalty
an office B. Moot 2. A mock trial
A B C C. Mortgage 3. An interest in the
(a) 3 2 1 property created as a
(b) 2 1 3 form of security for a
(c) 1 2 3 loan
(d) 2 3 1 A B C
46. A. Extadition 1. Red handed (a) 1 2 3
B. Fiduciary 2. Based on mutual in- (b) 2 1 3
terest and faith (c) 2 3 1
C. Flagrante 3. To extradite a (d) 1 3 2
delicto criminal 52. A. Motive 1. Cheque
A B C B. Negligence 2. Gross carelessness
(a) 3 2 1 C. Negotiable 3. Purpose behind
(b) 2 1 3 instrument action
(c) 3 1 2 A B C
(d) 2 1 3 (a) 1 2 3
47. A. Genocide 1. Massive killing (b) 3 2 1
B. Hijacking 2. Seizing an aircraft (c) 2 1 3
C. Homicide 3. Killing of human (d) 2 3 1
being 53. A. Nullity of 1. False evidence
marriage
A B C
B. Parole 2. Conditional release
(a) 3 2 1
C. Perjury 3. Invalid marriage
(b) 1 2 3
A B C
(c) 3 1 2
(a) 1 2 3
(d) 2 1 3
(b) 3 2 1
48. A. Hostage 1. To indict
(c) 2 1 3
B. Hostile 2. Adverse witness
(d) 2 3 1
witness
54. A. Provocation 1. Conduct or words
C. Incriminate 3. One held as a security causing someone to
A B C loose his self control
(a) 3 2 1 B. Putative 2. A man alleged to be
(b) 2 1 3 father the father of an ille-
(c) 3 1 2 gitimate child.
(d) 2 3 1 C. Quasi 3. A function that re-
49. A. Innenudo 1. Interim judicial sembles the judicial
B. Insanity 2. Indirect function
C. Interlocutory 3. Mental disorder A B C
A B C (a) 1 2 3
(a) 1 2 3 (b) 3 2 1
(b) 3 2 1 (c) 3 1 2
(c) 2 3 1 (d) 1 3 2
(d) 3 1 2 55. A. Restitution 1. Damage or destruc-
50. A. Malafide 1. Fraudulent or dishon- tion of property
est act B. Revocation 2. withdrawal offer
B. Mensrea 2. Hostile attitude of offer
C. Malice 3. Guilty mind C. Sabotage 3. return/restoration
A B C A B C
(a) 1 3 2 (a) 1 2 3
(b) 3 2 1 (b) 3 2 1
(c) 2 1 3 (c) 3 1 2
(d) 3 1 2 (d) 1 3 2

CLAT.indb 21 31/03/2009 11:41:37


22 LEGAL TERMS AND MAXIMS

56. A. Simpliciter 1. A word or a phrase in (c) A person to whom the promise is


a document is used made
absolutely/uncondi- (d) None of these
tionally 62. An invitation to tender is
B. Sine die 2. Without a date (a) A promise
C. Slander 3. A defamatory state- (b) Not a promise but is an invitation to
ment offer
A B C (c) A proposal
(a) 1 2 3 (d) None of these
(b) 3 2 1 63. The difference between an advertisement
(c) 2 1 3 for sale and a proposal is
(d) 1 3 2 (a) No difference at all
57. A. Stare decisis 1. Doctrine of precedent (b) That a proposal becomes a promise as
B. Sub judice 2. Pending matter soon as the party to whom it is made
C. Sui generis 3. Of its own kind accepts it but an advertisement does
A B C not
(a) 1 2 3 (c) Depending on the circumstances of
(b) 3 2 1 each case
(c) 1 3 2 (d) None of the above
(d) 3 1 2 64. Theft is offence against
58. A. Tortfeasor 1. One who commits (a) Movable property only
tort (b) Immovable property only
B. Vandalism 2. Damaging property (c) Movable and immovable both
C. Void 3. Having no legal ef- (d) None of these
fect 65. Among which of the following case
A B C the Supreme Court held that, there is no
(a) 1 2 3 moral, fundamental or legal rights or
(b) 1 3 2 equitable jurisdiction to go on strike by
(c) 3 2 1 the government employees?
(d) 3 1 2 (a) T. K. Rangarajan vs Government of
59. A. Voidable 1. Capable of being
Tamil Nadu
avoided
(b) Dwarka Prasad Agarwal vs B. D.
B. Ultra vires 2. Within powers
Agarwal
C. Intra vires 3. Beyond powers
(c) Union of India vs Rajiv Kumar,
A B C
A.I.T.U.C. vs Union of India
(a) 1 3 2
(d) A. K. Puriwar vs T. N. Muthy
(b) 3 2 1
66. In which case, Supreme Court held that
(c) 2 1 3
the Parliament has no power to amend
(d) 2 3 1
Fundamental Rights?
60. A. Arrest 1. Refuge granted to ab-
(a) Golaknath case
sconder
(b) Sajjan Singh case
B. Arson 2. Reckless destruction
of property by fire (c) Shankri Prasad case
C. Asylum 3. Apprehension of a (d) None of the above
person by police of- 67. In which case, Supreme Court held that the
ficer Preamble is a basic feature of Constitution
A B C and cannot be amended?
(a) 1 2 3 (a) Keshvanand Bharti case
(b) 3 2 1 (b) Golaknath case
(c) 2 1 3 (c) Minerva Mills case
(d) 1 3 2 (d) None of the above
61. A promisee is 68. ‘Ultra Vires’ is a term used for
(a) A person who makes a promise (a) A document corrupted by virus
(b) A person who monitors the statement (b) An act beyond the authority of a per-
of intention of the two parties son

CLAT.indb 22 31/03/2009 11:41:37


LEGAL TERMS AND MAXIMS 23

(c) An act authorized by law 77. In this famous case, the Supreme Court
(d) An illegal act said “the President means, for all practical
69. To betray a nation is an offence and purposes, the Prime Minister or council
punishable with death, that is of Ministers and his opinion, satisfaction
(a) Sedition or decision is constitutionally secured
(b) Treachery when Minister arrives at such opinion,
(c) Treason satisfaction or decision”.
(d) Anti-nationality (a) Ram Jawaya Kapur vs State of Pun-
70. The term res subjudice means jab
(a) A case has been decided by a court (b) Maneka Gandhi vs Union of India
(b) A case is pending before a court (c) Dr M. Ismail Faruqui vs Union of
(c) A case has been dismissed by a court India
(d) None of the above (d) None of the above
71. In which case the Supreme Court held that 78. After 86th Amendment to the Constitution
Article 21 includes right to die? Article 45 of the Constitution includes
(a) P. Rathinam/Nagbhusan Patnaik vs (a) provision for the right to education of
Union of India children
(b) Dr M. Ismail Faruqui vs Union of (b) provision for early childhood care and
India protection against any offence.
(c) K. Karunakaran vs State of UP (c) provision for early childhood care and
(d) None of the above education to children below the age of
72. Which section of Indian Penal Code was six years.
struck down by Supreme Court, while (d) provision for children for case and
making attempt to commit suicide non- education upto age of 18 years.
punishable (In P. Rathinam etc., vs Union 79. The Shah Bano case is related to
of India)? (a) Muslim wife’s right to maintenance
(a) S. 309 (b) S. 310 after divorce
(c) S. 498 (d) S. 51 (b) Muslim wife’s right to divorce
73. In which among the following cases (c) Muslim wife’s right to separation
the prisoners are entitled to rights under (d) Muslim wife’s right to husband’s
Articles 14, 19 and 21 of the Constitution property
of India and these articles operate within 80. When a person aids the commission of an
the prison? offence, he is called
(a) Sunil Batra’s case in 1978 (a) Abettor (b) Accessory
(b) Nandini Satpathy’s case in 1977 (c) Accomplice (d) Abeyer
(c) Maneka Gandhi’s case in 1978 81. If a person goes away secretly, or evades
(d) Indira Nehru Gandhi’s case in 1976 the jurisdiction of the court, he is
74. Public holidays are declared under (a) Absconding (b) Abrogating
(a) Negotiable Instruments Act, 1881 (c) Absenting (d) Abomining
(b) Contract Act, 1872 82. Match the following:
(c) Public Employees Act, 1967 I. Adulteration
(d) None of the above II. Adultery
75. Guidelines for protection of witnesses III. Alibi
were issued in IV. Alteration
(a) Naina Sahni Case A. Adding of substance to food which
(b) Raja Bhaiya case renders it dangerous to health
(c) Shivani Bhatnagar case B. Having sexual intercourse with mar-
(d) Neelam Katara case ried woman
76. Who is responsible for introduction of C. A dispute
Public Interest Litigation in India? D. An excuse
(a) Justice P. N. Bhagwati (a) I–D, II–C, III–B, IV–A
(b) Justice M. N. Venkatachaliah (b) I–A, II–B, III–D, IV–C
(c) Justice A. M. Ahmadi (c) I–A, II–C, III–D, IV–A
(d) Justice V. R. Krishna Iyer (d) I–B, II–C, III–A, IV–D

CLAT.indb 23 31/03/2009 11:41:37


24 LEGAL TERMS AND MAXIMS

83. A written statement in the name of a 93. A doctor has a ….. relationship with his
person by whom it is voluntarily signed patient
and sworn to is called (a) Fiduciary
(a) Advowson (b) Affidavit (b) Financial
(c) Affirmation (d) Affray (c) Personal
84. A crime of unlawfully damaging or (d) Impersonal
destroying property by fire is called 94. ‘Homicide’ is killing of a human being by
(a) Buggery (b) Affray another human being. It may be
(c) Arson (d) Rioting (a) Lawful
85. In civil action, the standard of proof is (b) Unlawful
(a) Beyond reasonable doubt (c) Both (a) and (b)
(b) On balance of probabilities (d) Never (a)
(c) Convincing to the judge 95. ‘Ignorantia juris nemineam exusal’ stands
(d) Logical and reasonable for
86. Prosecution : Accused :: ? : Defendant (a) Ignorance of law is no excuse
(a) Prosecution (b) Plaintiff (b) Ignorance of fact is no excuse
(c) Suer (d) Applicant (c) Laws are universally known
87. An insolvent person is (d) None of the above
(a) Not able to walk 96. Match the following:
(b) Infirm I. Inadvertence
(c) Financially unsound II. Incest
(d) Bankrupt III. Indemnity
88. What is the legal meaning of the word IV. Indictment
‘Battery’? A. Written accusation of crime
(a) Cells, as used in torch, tape recorder B. Exemption from penalty
etc. C. Sexual intercourse between members
(b) Battering a person to death of a family e.g., mother and son
(c) Actual or intended striking of another D. Carelessness
person (a) I–A, II–B, III–C, IV–D
(d) Assault resulting in, at least 6 months’ (b) I–D, II–C, III–B, IV–A
hospitalization (c) I–B, II–C, III–D, IV–A
89. Capital offences result in (d) I–C, II–D, III–A, IV–B
(a) Sentence to death 97. The killing of a new born child by its
(b) Sentence of life imprisonment parents
(c) Imprisonment over 10 years (a) Infanticide (b) Abortion
(d) Amputation (c) Foeticide (d) Malfeasance
90. What is a Caveat? 98. Innuendo hints at
(a) A warning (a) Insinuation (b) Defamation
(b) An injunction (c) Slander (d) Libel
(c) Certiorari 99. Match the following:
(d) Writ I. Interlocutory
91. What is a ‘cognizance”? II. Ultra vires
(a) Arousing judicial notice or knowl- III. Intra vires
edge IV. Ipso facto
(b) It is a crime A. By the fact
(c) It means custody without warrant B. Not final
(d) It means custody with warrant C. Beyond Powers
92. …..is an established defence when it is D. Within Powers
proved that the plaintiff failed to take (a) I–A, II–B, III–C, IV–D
reasonable care of himself and thus (b) I–A, II–C, III–D, IV–B
contributes to his injuries (c) I–B, II–C, III–D, IV–A
(a) Contributory negligence (d) I–C, II–D, III–B, IV–A
(b) Self injury 100. Mensrea is translated as
(c) Volenti non fit injuria (a) Guilty mind (b) Intention
(d) None of the above (c) Evil (d) Unlawful act

CLAT.indb 24 31/03/2009 11:41:37


LEGAL TERMS AND MAXIMS 25

101. The famous M’ Naghten Rules determine (a) Duress


(a) Insanity as defence (b) Dunress
(b) Law of sea (c) Compulsion
(c) Distinguish hurt from grievous hurt (d) None of the above
(d) Established seasoning in criminal cas- 111. PIL means:
es (a) Private interest litigation
102. A child born after father’s death (b) Public interest litigation
(a) Postliminits (b) Bastard (c) Publicity interest litigation
(c) Posthumous (d) Postea (d) Proactive interest litigation
103. The meaning of the term ‘quasi’ is 112. L. L.B. stands for
(a) As if it were (b) Null (a) Law est jury
(c) Void (d) Quashed (b) Legalite Lawyers Baccili
104. Who is a Recidivist? (c) Legum Baccalaureus
(a) Saint (d) None of the above
(b) Habitual criminal 113. Voting right comes under
(c) Rash person (a) Legal right
(d) Reserved person (b) Fundamental right
105. Match the following: (c) Constitutional right
I. Void (d) None of the above
II. Voidable 114. Whoever has sexual intercourse with a
III. Vis major person who and whom he knows or has
IV. Volenti non fit injuria reason to believe to be the wife of another
A. An act illegal from beginning man, without the consent or connivance of
B. An act until recinded, legal
that man, such sexual intercourse amounts
C. Consent is a defence
to
D. Irresistible force
(a) Rape
(a) I–A, II–B, III–C, IV–D
(b) Adultery
(b) I–C, II–D, III–B, IV–A
(c) Unnatural offence
(c) I–B, II–D, III–A, IV–C
(d) Abatement of rape
(d) I–A, II–B, III–D, IV–C
115. Cruelty to a women by husband or relative
106. Public Prosecutor is
of husband is defined under
(a) Counsel of the State in the case where
government is a party (a) Section 498A of Indian Penal Code
(b) Counsel of the State in a Criminal (b) Section 498 of Indian Penal Code
trial (c) Section 497 of Indian Penal Code
(c) Counsel of the State in a Civil trial (d) Section 496 of Indian Penal Code
(d) Counsel of the State in both Criminal 116. The first Indian Barrister was
and Civil trial. (a) Madhusudan Das
107. A lawyer appointed by court to represent a (b) Chitranjan Das
poor litigant is (c) Subhash Chandra Bose
(a) Amicus Curiae (d) Jnanendra Mohan Tagore
(b) Legal Aid man 117. The largest PIL (Public Interest Litigation)
(c) Lok Nyaya Pradayak winner
(d) None of the above (a) M. C. Mehta
108. ….is a general pardon (b) Common Cause a registered society
(a) Remission (b) Reprieve founded by H. D. Shourie.
(c) Amnesty (d) Suspension (c) Helpage India
109. Husband and wife have a right to each (d) M. K. Chawla
other’s company. This right is called 118. The maxim ‘Ubi jus’ibi remedium’ means
(a) Legal right (a) Justice should not only be done but
(b) Matrimonial right should be seen to be done.
(c) Consortium right (b) In law there is a remedy for every
(d) Conjugal right wrong.
110. An act done under threat or fear is done (c) Justice according to law
under (d) None of the above

CLAT.indb 25 31/03/2009 11:41:37


26 LEGAL TERMS AND MAXIMS

119. Contempt of court means (c) Dr Chanchal Goyal vs State of Rajas-


(a) Both civil contempt and criminal con- than
tempt (d) Radhe Mohanlal vs Rajasthan High
(b) Only criminal contempt Court
(c) Only civil contempt 125. In which one of the following states the
(d) None of the above Legislative Council was revived recently?
120. The doctrine (Rule) of Basic Structure (a) Tamil Nadu (b) Andhra Pradesh
was propounded by the Supreme Court in (c) Rajasthan (d) Punjab
(a) Sajjan Singh case 126. Who of the following is the Chairperson
(b) Golak Nath case of the National Commission for the
(c) Keshavanand Bharti case Protection of Child Rights (NCPCR)?
(d) A. K. Gopalan case (a) Shiela Dixit
121. In which among the cases the Supreme (b) Savitri Sinha
Court held that Legislative power of a State (c) Shanta Sinha
or Union is subject to the fundamental (d) None of the above
rights and the legislature cannot indirectly 127. The first Mobile Court of India was
take away or abridge fundamental rights launched in
which it could not do directly for granting (a) Haryana (b) Rajasthan
either recognition or aid to minority (c) Gujarat (d) Madhya Pradesh
institutions? 128. Vijay L. Kelkar, who was appointed as the
(a) TMA Pai Foundation vs State of Chairman of the 13th Finance Commission
Karnataka is a former
(b) TMA Pai foundation vs Union of (a) Comptroller and Auditor General
India (b) Finance Secretary
(c) TMA Pai Foundation vs State of (c) Chairman of Planning Commission
Rajasthan (d) None of the above
(d) None of the above 129. Which one the following High Court
122. It is the Fundamental rights of the consum- enjoys jurisdiction over largest number of
ers to know whether the food products, territories?
cosmetics and drugs are of non-vegetarian (a) Calcutta
or vegetarian origin, otherwise it will vio- (b) Punjab and Haryana
late their fundamental rights under Articles (c) Allahabad
19 (1) (a), 21 and 25 of the Constitution of (d) Guwahati
India. This observation related to 130. The salary and allowances of the High
(a) Ozair Hussain vs Union of India Court judges are paid out of
(b) Vikash Despandey vs Bar council of (a) the Consolidated Fund of India
India (b) the Public Accounts Fund
(c) TMA Pai foundation vs State of (c) Contingency Fund of India
Karnataka (d) Public Accounts Fund of the con-
(d) None of the above cerned states
123. Supreme Court of India recognized sexual 131. When were the High Courts of Bombay,
harassment as a human rights violation in Calcutta and Madras established?
India in case of (a) 1861 (b) 1935
(a) Vishaka vs State of Rajasthan (c) 1948 (d) 1950
(b) Birdhichand vs State of Maharashtra 132. Who of the following has been appointed
(c) Ahmed Khan vs Shah Bano Begum as the Chairman of the National Land
(d) Takaram vs State of Maharashtra Reform Council?
124. Among which of the cases the Supreme (a) Manmohan Singh
Court held that right of an Advocate to (b) Pranab Mukherjee
appear and conduct cases in Courts in (c) M. Veerappa Moily
still within the control and jurisdiction of (d) N. R. Madhav Menon
courts? 133. Who of the following is the Chairman of
(a) Ex-capt. Harish Uppal vs UOI the Thirteenth Finance Commission?
(b) Ram Narain Popli vs C.B.I. (a) K. C. Pant

CLAT.indb 26 31/03/2009 11:41:37


LEGAL TERMS AND MAXIMS 27

(b) Vijay Kelkar (c) It is passed by both the Houses


(c) C. Rangarajan (d) None of these
(d) None of the above 143. What is ‘cognizance’?
134. Who heads the new Commission on (a) Arousing judicial notice or knowl-
Centre–State relations appointed by the edge
government in April 2007? (b) It is a crime
(a) M. M. Punchhi (c) It means custody without warrant
(b) M. Veerappa Moily (d) It means custody with warrant
(c) K. C. Pant 144. ‘Damages’ are awarded except in one
(d) N. R. Madhav Menon case
135. Which one of the following was member (a) Tort
of Rajya Sabha at the time of appointment (b) Breach of contract
as Prime Minister? (c) Libel Suit
(a) P. V. Narsimha Rao (d) Criminal conduct
(b) Charan Singh 145. Deceit is
(c) Lal Bahadur Shastri (a) A tort
(d) None of the above (b) A crime
136. In 2006 President Abdul Kalam sent (c) A breach of contract
back a bill passed by Parliament for (d) Both (a) and (b)
reconsideration. The Bill related to 146. The word ‘due process of law’ indices
(a) reservations for the OBCs (a) As ordained by statute law
(b) 56 offices of profit (b) In course of time
(c) state funding of elections (c) In course through courts
(d) None of the above (d) Justice by tribunals
137. Which one of the following states is 147. Liability is
regarded as the pioneer of the Right to (a) Subjection to legal obligation
Information in India? (b) Responsibility
(a) Rajasthan (c) Duty
(b) West Bengal (d) None of the above
(c) Himachal Pradesh 148. Legal person is one
(d) Kerala (a) Who knows law
138. Under whose Prime Ministership was the (b) Capable of being right and duty bear-
anti-defection law passed? ing unit
(a) Rajiv Gandhi (b) Indira Gandhi (c) Who is involved in legal case
(c) V. P. Singh (d) A. B. Vajpayee (d) None of these
139. Which one of the following Prime Ministers 149. What is meant by Affidavit?
headed a minority Government? (a) Evidentiary Document
(a) V. P. Singh (b) Document
(b) I. K. Gujral (c) Letter of oath
(c) Chandra Shekhar (d) A request made to the court
(d) All the above 150. According to McNaghten rules, legal
140. Mid-term elections are held in India insanity is quite different from medical
(a) When circumstances so deemed insanity. It relates to the
(b) After three years (a) Total loss of conative faculties
(c) Every two and half years (b) Total loss of cognitive faculties
(d) Every 4th year (c) Congenital insanity
141. Autonomy means (d) None of these
(a) The power to manage funds 151. The doctrine of stare decisis is a feature of
(b) The power to manage law and order Common Law. This means that
(c) The power to manage internal affairs (a) The decision stares at the judge
(d) None of these (b) Precedents possess law quality in
142. A bill becomes an Act themselves and are binding
(a) When the Prime Minister assents to it (c) Decision of High Court is ‘Law’ even
(b) When it is passed by both the Houses through it is not binding on any Court
and assented to by the President except those subordinate to it

CLAT.indb 27 31/03/2009 11:41:37


28 LEGAL TERMS AND MAXIMS

(d) None of the above definitions are cor- (c) That which is recognized and prac-
rect ticed by the legal system
152. A ratio decidendi means (d) None of these
(a) The proportion of justice in a case 162. Legal person is one
(b) The principle upon which a case was (a) Who knows law
decided (b) Capable of being right and duty bear-
(c) The decisive facts of the case ing
(d) None of these (c) Who is involved in a legal case
153. The doctrine of res judicata means that (d) None of these
(a) Justice has resulted 163. Tort means
(b) Residual justice (a) A wrong
(c) The final judgement of a competent (b) A legal wrong
court may not be disputed by the par- (c) A legal civil wrong
ties or their heirs in any subsequent (d) None of these
legal proceedings 164. Unliquidated damages mean
(d) None of these (a) Damage to something mean
154. Res judicata (b) Damage caused by a firm which has
(a) Creates a binding precedent as to gone in liquidation
“law” (c) Damage to a firm in the hands of re-
(b) Does not create a binding precedent as ceivers
to “law” but makes the decision of the (d) Damage to be assessed by a Court as
competent court undisputable these are not pre-determined
(c) Does not bind the successors of the 165. Tort is a civil legal wrong
parties in dispute
(a) It is indistinguishable from contract
(d) Does not bind third parties in subse-
(b) It is partly distinguishable from con-
quent legal proceedings in the case
tract
155. ………. is an inquiry into cause of death
(c) It is other than mere breach of con-
(a) Coronoration (b) Inquest
tract
(c) Investigation (d) Subpoena
(d) It is none of these
156. Justice is the ……….
166. Crime is
(a) Means, end (b) Cause, effect
(a) Public wrong
(c) Effect, cause (d) End, means
157. Husband and wife have a right to each (b) Private wrong
other’s company. This right is called (c) Public as well as private wrong
(a) Legal Right (d) Neither public nor private wrong
(b) Matrimonial Right 167. Breach of contract is a civil wrong. It is
(c) Consortium Right redressible by an action for
(d) Conjugal Right (a) Liquidated damages
158. Legal relationships show that ‘immunity’ (b) Uniliquidated damages
denotes (c) Both liquidated as well as unliquidat-
(a) Freedom from infection ed damages
(b) Freedom from interference (d) Neither liquidated nor unliquidated
(c) Freedom from power of another damages
(d) None of the above 168. When force is used to cause bodily pain,
159. If a person evades the order of court he is disease or infirmity to any person, it is
called called
(a) Absconder (b) Abetter (a) Hurt
(c) Absentia (d) Abominator (b) Assault
160. What is an alibi? (c) Grievous assault
(a) Excuse (b) Dispute (d) None of these
(c) Evidence (d) Adulteration 169. Grievous hurt is defined in section
161. In law a right is (a) 319 (b) 320
(a) That which is ethically correct (c) 338 (d) 351
(b) That which inheres in every human 170. Permanent deprivation of the sight of
being either eye is an offence of

CLAT.indb 28 31/03/2009 11:41:37


LEGAL TERMS AND MAXIMS 29

(a) Grievous hurt (b) Assault (c) A breach of contract


(c) Hurt (d) None of these (d) Both (a) and (b)
171. Destruction or permanent impairing of 183. The word due process of law indicates
power of any member or joint is an offence (a) As ordained by statute law
of (b) In course of time
(a) Assault (b) Grievous hurt (c) In course through courts
(c) Hurt (d) None of these (d) Justice by tribunals
172. Permanent disfiguration on the head or 184. Liability is
face is an offence of (a) Subjection to legal obligation
(a) Hurt (b) Grievous hurt (b) Responsibility
(c) Murder (d) None of these (c) Duty
173. Culpable homicide is defined in the Indian (d) None of these
Penal Code, Section 185. For which institution there is indirect
(a) 299 (b) 300 election?
(c) 301 (d) 307 (a) Assembly
174. Murder is defined in the Indian Penal (b) Rajya Sabha
Code, Section (c) Lok Sabha
(a) 299 (b) 300 (d) Muncipal Corporation
(c) 301 (d) 307 186. What is the qualification for obtaining
175. Rape is provided as an offence under I.P.C. voting right in India?
Section (a) Age (b) Education
(a) 375 (b) 378 (c) Property (d) Caste
(c) 403 (d) 405 187. Publc bills are
176. Sexual intercourse by a man with his own (a) Introduced by Ministers
wife is a rape if she is below the age of (b) Introduced by Individuals
(a) 18 years (b) 15 years (c) Introduced by Parliament
(c) 16 years (d) 17 years (d) Introduced by Opposition Parties
177. Sexual intercourse by a man with a woman 188. Who appoints the District Judge?
even with her consent is a rape if she is (a) Governor
below the age of (b) State Minister for Law
(a) 16 years (b) 17 years (c) High Court Chief Justice
(c) 18 years (d) none of these (d) Chief Minister
178. Theft is defined in the I.P.C., Section 189. Section 84 of the I.P.C. was based on the
(a) 375 (b) 377 (a) Proximity rules
(c) 378 (d) 383 (b) McNaghten rules
179. Theft is offence against (c) Congential Insanity
(a) Movable property only (d) None of these
(b) Immovable property only 190. According to McNaghten rules, legal
(c) Movable and immovable both insanity is quite different from medical
(d) None of these insanity. It relates to the
180. What is a ‘congnizance’? (a) Total Loss of conative faculties
(a) Arousing judicial notice or knowl- (b) Total loss of congnitive faculties
edge (c) Congenital insanity
(b) It is a crime (d) None of these
(c) It means custody without warrant 191. The proof of insanity lies on the
(d) It means custody with warrant (a) Prosecution
181. Damages are awarded except in one case (b) Accused
(a) Tort (c) Parents
(b) Breach of contract (d) Doctor attending the accused.
(c) Libel suit 192. An unjust precedent is
(d) Criminal conduct (a) Law (b) Not Law
182. Deceit is (c) No precedent (d) Impossible
(a) A tort 193. The use of the word “Person” is
(b) A crime (a) purposive

CLAT.indb 29 31/03/2009 11:41:38


30 LEGAL TERMS AND MAXIMS

(b) descriptive (c) statutory provisions


(c) both of the above (d) no precise definition
(d) None of these 195. ….. is an inquiry into cause of death.
194. The definition of natural justice has (a) Coronoration (b) Inquest
evolved from (c) Investigation (d) Subpoena
196. Justice is the ………….
(a) religious tenets
(a) Means, end (b) Cause, effect
(b) rules of courts of justice (c) Effect, cause (d) End, means

ANSWERS

1. (a) 2. (b) 3. (a) 4. (a) 5. (a) 6. (a) 7. (c) 8. (b) 9. (a) 10. (a)
11. (a) 12. (a) 13. (b) 14. (a) 15. (a) 16. (a) 17. (b) 18. (a) 19. (b) 20. (a)
21. (a) 22. (a) 23. (a) 24. (b) 25. (a) 26. (a) 27. (a) 28. (a) 29. (a) 30. (c)
31. (a) 32. (a) 33. (a) 34. (c) 35. (b) 36. (c) 37. (a) 38. (a) 39. (a) 40. (c)
41. (a) 42. (b) 43. (c) 44. (b) 45. (c) 46. (a) 47. (b) 48. (a) 49. (c) 50. (a)
51. (a) 52. (b) 53. (b) 54. (a) 55. (b) 56. (a) 57. (a) 58. (a) 59. (a) 60. (b)
61. (a) 62. (b) 63. (b) 64. (a) 65. (a) 66. (d) 67. (a) 68. (b) 69. (c) 70. (b)
71. (a) 72. (a) 73. (a) 74. (a) 75. (d) 76. (a) 77. (a) 78. (c) 79. (a) 80. (a)
81. (a) 82. (b) 83. (b) 84. (c) 85. (a) 86. (b) 87. (d) 88. (c) 89. (a) 90. (a)
91. (a) 92. (a) 93. (a) 94. (b) 95. (a) 96. (b) 97. (a) 98. (b) 99. (c) 100. (a)
101. (a) 102. (c) 103. (a) 104. (b) 105. (a) 106. (a) 107. (a) 108. (c) 109. (d) 110. (a)
111. (b) 112. (c) 113. (c) 114. (b) 115. (a) 116. (b) 117. (a) 118. (b) 119. (a) 120. (c)
121. (a) 122. (a) 123. (a) 124. (a) 125. (b) 126. (c) 127. (a) 128. (b) 129. (d) 130. (a)
131. (a) 132. (a) 133. (b) 134. (a) 135. (a) 136. (b) 137. (b) 138. (a) 139. (d) 140. (a)
141. (d) 142. (b) 143. (a) 144. (d) 145. (d) 146. (c) 147. (a) 148. (b) 149. (c) 150. (b)
151. (b) 152. (b) 153. (c) 154. (a) 155. (c) 156. (d) 157. (d) 158. (d) 159. (a) 160. (a)
161. (c) 162. (b) 163. (c) 164. (d) 165. (a) 166. (a) 167. (b) 168. (c) 169. (b) 170. (a)
171. (b) 172. (b) 173. (a) 174. (b) 175. (a) 176. (c) 177. (a) 178. (c) 179. (a) 180. (a)
181. (d) 182. (a) 183. (c) 184. (a) 185. (b) 186. (a) 187. (c) 188. (a) 189. (b) 190. (b)
191. (b) 192. (b) 193. (a) 194. (d) 195. (c) 196. (d)

CLAT.indb 30 31/03/2009 11:41:38


Chapter 2
CRIMINAL LAW
CULPABALE HOMICIDE child in the mother’s womb is not homicide. But,
it may amount to culpable homicide to cause the
Section 299, Culpable Homicide. Whoever death of a living child, if any part of that child
causes death by doing an act with the intention has been brought forth, though the child may not
of causing death, or with the intention of causing have breathed or been completely born.
such bodily injury as is likely to cause death, or Sections 299 and 300, IPC define culpable
with the knowledge that he is likely by such act homicide, which is of two types:
to cause death, commits the offence of culpable 1. Culpable homicide amounting to murder
homicide. 2. Culpable homicide not amounting to
Illustrations murder.
The provisions relating to murder and culpable
(a) A lays sticks and turf over a pit, with the homicide are probably the most complicated in
intention of thereby causing death, or the IPC, and are so technical that very often they
with the knowledge that death is likely to lead to confusion.
be caused. Z believing the ground to be A murder is culpable homicide, but every
firm, treads on it, falls in and is killed. culpable homicide is not murder. Culpable
A has committed the offence of culpable homicide is the genus and murder, its species.
homicide. Section 299 defines culpable homicide
(b) A knows Z to be behind a bush. B does not simpliciter. Section 300 defines murder, which
know it. A, intending to cause, or knowing is also a culpable homicide with some special
it to be likely to cause Z’s death, induces characteristics, which are set out in clauses 1–4
B to fire at the bush. B fires and kills Z. of Section 300, subject to the exceptions given
Here B may be guilty of no offence, but in Section 300, then it will amount to murder.
A has committed the offence of culpable All other instances of culpable homicide, the
homicide. ones which may fall within the exceptions to
(b) A, by shooting at a fowl with intent to kill Section 300, will all be culpable homicide not
and steal it, kills B, who is behind a bush; amounting to murder.
A not knowing that he was there. Here, While Section 299 defines ‘culpable ho-
although A was doing an unlawful act, he micide’, it is not an exhaustive definition, It is
was not guilty of culpable homicide, as he important to remember that Section 300 also de-
did not intend to kill B, or to cause death fines culpable homicide, but which amounts to
by doing an act that he knew was likely to murder. Before going into further details about
distinctions between Section 299 and Section
cause death.
300, IPC, it is important to understand the sec-
Explanation 1. A person who causes bodily tions.
injury to another who is labouring under a The following are the essential ingredients
disorder, disease or bodily infirmity, and thereby of culpable homicide:
accelerates the death of that other, shall be (1) There must be a death of a person
deemed to have caused his death. (2) The death should have been caused by the
act of another person
Explanation 2. Where death is caused by bodily
(3) The act causing death should have been
injury, the person who causes such bodily injury
done
shall be deemed to have caused the death,
(a) with the intention of causing death;
although by resorting to proper remedies and
or
skilful treatment the death might have been (b) with the intention of causing such
prevented. bodily injury as is likely to cause
Explanation 3. The causing of the death of a death; or

CLAT-2-Criminal Law.indd 31 01/09/2010 17:29:23


32 CRIMINAL LAW

(c) with knowledge that such act is likely in the ordinary course of nature to cause death,
to cause death. or
The definition itself provides for three cir- Fourthly, if the person committing the
cumstances, wherein the presence or absence of act knows that it is so imminently dangerous
certain factors in causing death is nevertheless that it must, in all probability, cause death, or
treated as causing culpable homicide. These cir- such bodily injury as is likely to cause death,
cumstances are dealt with in explanations 1–3. and commits such act without any excuse for
incurring the risk of causing death or such injury
Explanation 1. Provides for a situation where the
as aforesaid.
injured person is suffering from some disorder,
disease or bodily infirmity, which quickened Illustrations
his death. The fact his death was quickened (a) A shoots Z with the intention of killing
or hastened by the disorders or disease he was him. Z dies in consequence. A commits
already suffering from, will not reduce the guilt murder.
or culpability of the person causing the injury. In (b) A, knowing that Z is labouring under
other words, the person who caused the injury such a disease that a blow is likely to
cannot escape criminal liability of culpable cause his death, strikes him with the
homicide by stating that if the person injured intention of causing bodily injury. Z dies
did not suffer from the said disease, he would in consequence of the blow. A is guilty
not have died. of murder, although the blow might have
Explanation 2. Provides for a situation wherein been sufficient in the ordinary course
a person who has been injured could have of nature to cause the death of a person
recovered and escaped death, if he had been in a sound state of health. But if A, not
given prompt and proper medical treatment. knowing that Z is labouring under any
In such situations too, the fact that the injured disease, gives him such a blow as would
person died because he could not avail of good not in the ordinary course of nature kill a
medical treatment, cannot be a ground for person in a sound state of health. Here A,
negating guilt or culpability of the person who although he may intend to cause bodily
inflicted the injury in the first place. injury, is not guilty of murder, if he did not
intent to cause death, or such bodily injury
Explanation 3. Is in respect to a slightly as in the ordinary cause of nature would
different situation. It takes into consideration cause death.
death caused to a child in the mother’s womb. (c) A intentionally gives Z a sword-cut or
The law states that if the death of the child club-wound sufficient to cause the death
is caused when still in the mother’s womb, it is of a man in the ordinary course of nature.
not culpable homicide. Z dies in consequence. Here, A is guilty
However, if any portion of the child, comes of murder, although he may not have
out of the mother’s womb, even if it is not fully intended to cause Z’s death.
born, and if death is caused to such child, then it (d) A without any excuses fires a loaded cannon
would amount to culpable homicide. into a crowd of persons and kills one of
them. A is guilty of murder, although he
MURDER may not have had a premeditated design
to kill any particular individual.
Section 300: Murder
Exception 1. When culpable homicide is not
Except in the cases hereinafter excepted, murder.
culpable homicide is murder, if the act by which Culpable homicide is not murder if the offender,
the death is caused is done with the intention of whilst deprived of the power of self-control
causing death, or by grave and sudden provocation, causes the
Secondly, if it is done with the intention of death of the person who gave the provocation
causing such bodily injury as the offender knows or accident.
to be likely to cause the death of the person to The above exception is subject to the
whom the harm is caused, or following provisions:
Thirdly, if it is done with the intention of Firstly, that the provocation is not sought
causing bodily injury to any person, and the or voluntarily provoked by the offender as an
bodily injury intended to be inflicted is sufficient excuse for killing or doing harm to any person.

CLAT.indb 32 31/03/2009 11:41:38


CRIMINAL LAW 33

Secondly, that the provocation is not given Exception 2. Culpable homicide is not murder
by anything done in obedience to the law, or by if the offender, in the exercise in good faith of the
a public servant in the lawful exercise of the right of private defence of person or property,
powers of such public servant. exceeds the power given to him by law and
Thirdly, that the provocation is not given causes the death of the person against whom
by anything done in the lawful exercise of the he is exercising such right of defence without
right of private defence. premeditation, and without any intention of
doing more harm than is necessary for the
Explanation. Whether the provocation was purpose of such defence.
grave and sudden enough to prevent the offence
from amounting to murder is a question of fact. Illustration
Illustrations Z attempts to horsewhip A, not in such a manner
as to cause grievous hurt to A. A draws out a
(a) A, under the influence of passion excited pistol. Z persists in the assault. A believing in
by a provocation given by Z, intentionally good faith that he can by no other means prevent
kills Y, Z’s child. This is murder, inasmuch himself from being horsewhipped, shoots Z
as the provocation was not given by the dead. A has not committed, murder, but only
child, and the death of the child was not culpable homicide.
caused by accident or misfortune in doing Exception 3. Culpable homicide is not murder
an act caused by a provocation. if the offender being a public servant or aiding
(b) Y gives grave and sudden provocation to a public servant acting for the advancement of
A. A, on this provocation, fires a pistol at public justice, exceeds the powers given to him
Y, neither intending nor knowing himself by law, and causes death by doing an act which
to be likely to kill Z, who is near him, but he, in good faith, believes to be lawful and
out of sight. A kills Z. Here A has not necessary for the due discharge of his duty as
committed murder, but merely culpable such public servant and without ill-will towards
homicide. the person whose death is caused.
(c) A is lawfully arrested by Z, a bailiff. A
Exception 4. Culpable homicide is not murder
is excited to sudden and violent passion
if it is committed without premeditation in a
by the arrest, and kills Z. This is murder, sudden fight in the heat of passion upon a sudden
inasmuch as the provocation was given quarrel and without the offenders having taken
by a, thing done by a public servant in the undue advantage or acted in a cruel or unusual
exercise of his powers. manner.
(d) A appears as a witness before Z, a
Explanation. It is immaterial in such cases
Magistrate. Z says that he does not believe
which party offers the provocation or commits
a word of A’s deposition, and that A has
the first assault.
perjured himself. A is moved to a sudden
passion by these words, and kills Z. This Exception 5. Culpable homicide is not murder
is murder. when the person whose death is caused, being
(e) A attempts to pull Z’s nose. Z, in the above the age of eighteen years, suffers death or
exercise of the right of private defence, takes the risk of death without his consent.
lays hold of A to prevent him from doing Illustration
so. A is moved to sudden and violent
A, by instigation, voluntarily causes Z, a person
passion in consequence, and kills Z. This
under eighteen years of age, to commit suicide.
is murder, inasmuch as the provocation
Here, on account of Z’s youth, he was incapable
was given by a thing done in the exercise of giving consent to his own death; A has
of the right of private defence. therefore abetted murder.
(f) Z strikes B. B is by this provocation
excited to violent rage. A, a bystander, Scope of Section 300
intending to take advantage of B’s rage, Section 300 defines murder with reference to
and to cause him to kill Z, puts a knife into culpable homicide defined in Section 299. If the
B’s hand for that purpose. B kills Z with special requirements provided in clauses 1–4 of
the knife. Here B may have committed a Section 300 are fulfilled, culpable homicide
only culpable homicide, but A is a guilty will then amount to murder, provided of course,
of murder. the act does not fall within any of the exception

CLAT.indb 33 31/03/2009 11:41:38


34 CRIMINAL LAW

provided in Section 300. If an act which falls Exception 4.


within clauses 1–4 of Section 300, also falls (a) A sudden fight without premeditation;
within one of the exception, then it will be (b) The offender should not take undue ad-
culpable homicide not amounting to murder. vantage or act in a cruel or unusual man-
It would probably have been more simple ner.
and less complicated if the Code had first de- Explanation. Who started the fight or quarrel
fined homicide and then defined separately cul- is immaterial.
pable homicide and murder. Since some clauses Exception 5. Death caused to a person above 18
in Sections 299 and 300 overlap, it has led to a years of age with his consent.
lot of discussions, debates and differences.
The following are the essential ingredients Death by Negligence
of Section 300: Section 304-A: Causing death by negligence
Culpable homicide is murder, if it is done Whoever causes the death of any person by
with: doing any rash or negligent act not amounting
(1) Intention to cause death; to culpable homicide, shall be punished with
(2) Intention to cause bodily injury, knowing imprisonment of either description for a term
that the injury caused, is likely to cause which may extend to two years, or with fine or
death; with both.
(3) Intention of causing bodily injury suffi- This section was added by an amendment
cient in the ordinary course of nature to of the Code 10—years after the IPC was enacted.
cause death; It does not create a new offence. This section is
(4) (a) knowledge that the act is so imminent- directed at offences, which fall outside the range
ly dangerous that in all probability it of Sections 229 and 300, where neither intention
will cause death or bodily injury likely nor knowledge to cause death is present.
to cause death; and In fact, if this section is also taken into
(b) such act should be without justifica- consideration, there are three types of homicide
tion. which are punishable under the code—culpable
Culpable homicide will not be murder, if homicide amounting to murder, culpable homi-
it is: cide not amounting to murder and the homicide
Exception 1. On grave and sudden provoca- by negligence.
tion; This section deals with homicide by negli-
Explanation: gence and covers that class of offences, where
(a) The provocation should not be voluntarily death is caused neither intentionally nor with the
sought or deliberately caused by the knowledge that the act of the offender is likely
accused; to cause death, but because of the rash negligent
(b) Should not be a result of act done by public act of the offender.
servant or in obedience to law; This clause limits itself to rash and negli-
(c) Not be in self-defence. gent acts which cause death, but falls short of
Whether a particular act is grave and culpable homicide of either description.
sudden is a question of fact. Rash or Negligent Act
Exception 2. In exercise of right of private
The doing of a rash or negligent act which
defence or person or property;
causes death, is the essence of this section.
(a) And without premeditation and without
Under Section 32, IPC, the act includes ‘illegal
intention.
omission’. Therefore, if an illegal omission
Exception 3. occurs as a result of negligence, which results in
(a) An act done by public servant or in aiding death, then this section will apply
a public servant; The term ‘negligence’ as used in this sec-
(b) Acting in advancement of public justice; tion does not mean mere carelessness. The rash-
(c) Such act of the public servant is in excess ness of negligence must be of such nature so as
of the powers conferred on him, but to be termed as a criminal act of negligence or
exercised in good faith; rashness.
(d) And such act is necessary to discharge Section 80 of the code provides ‘nothing
faith; is an offence which is done by accident of mis-
(e) And is without ill will. fortune and without any criminal knowledge or

CLAT.indb 34 31/03/2009 11:41:39


CRIMINAL LAW 35

intention in the doing of a lawful act in a lawful Suicide


manner by a lawful means and with proper care Section 305: Abetment of suicide of child or
and caution.’ It is absence of such proper care insane person
and caution, which is required of a reasonable
man in doing an act, which is made punishable If any person under eighteen years of age, any
under this section. insane person, any delirious person, any idiot,
It is the degree of negligence, which re- or any person in a state of intoxication, commits
ally determines whether a particular act would suicide, whoever abets the commission of
amount to a rash and negligent act is of such a such suicide, shall be punished with death or
degree that the risk run by the doer of the act as imprisonment for life, or imprisonment for a
defined under this section. It is only when the term not exceeding ten years, and shall also be
rash and negligent act is of such a degree that liable to fine.
the risk run by the doer of the act is very high Section 306: Abetment of suicide
or is done with such recklessness and with total If any person commits suicide, whoever abets the
disregard and indifference to the consequences commission of such suicide, shall be punished
of this act, the act can be constituted as a rash with imprisonment of either description for a
and negligent act under this section. term which may extend to ten years, and shall
Negligence is the gross and culpable also be liable to fine.
neglect or failure to exercise reasonable and Suicide has not been declared as a crime
proper care, and precaution to guard against, by the Code obviously because once a person
either to the public generally or to an individu- successfully commits suicide, that person is no
al in particular, which a reasonable man would longer alive to be prosecuted and the crime
have adopted. abates with him. However, an attempt to commit
Attempt to Commit Murder and suicide is punishable under Section 309 and an
Culpable Homicide abetment to commit suicide under Sections 305
and 306, IC.
Section 307: Attempt to murder This section is based on a reasonable pub-
Whoever does any act with such intention or lic policy to prevent other persons involvement,
knowledge, and under such circumstances that, instigation and aiding in terminating one’s life.
if he by that act caused death, he would be guilty It takes care of situation and threats imposed by
of murder, shall be punished with imprisonment death baiters.
of either description for a term which may extend To make out a case of abetment, there must
to ten years, and shall also be liable to fine; and, be instigation by the accused—provoking, incit-
hurt is caused to any person by such act, the ing or encouraging a person to do an act.
offender shall be liable either to imprisonment The offence of abetment must confirm to
for life, or to such punishment as is hereinbefore the definition of the term given in Section 107
mentioned. i.e., to say, there must be instigation, coopera-
Attempts by Life-Convicts tion or intentional assistance given to the would
be suicide. It is not necessary, nor indeed is it
When any person offending under this section is
a part of the definition, that the suicide should
under sentence of imprisonment for life, he may,
have been committed in consequence of the
if hurt is caused, be punished with death.
abetment.
Scope of Section 307 But, in order to render a person liable as an
abettor, it is necessary that the abettor should do
Section 307 provides that the definition as to something more than remaining a mute specta-
what amounts to an attempt to commit murder. tor.
In order or constitute an offence under this But, sometimes, it is conceivable that even
section, two elements are essential. the mere presence as spectator may encourage a
First, the intention or knowledge to commit person to do a deed, which she might otherwise
the murder. refrain from. In such cases, the question whether
Second, the actual act of trying to commit mere presence amounted to intentionally aiding
the murder. Thus it must have both the neces- another, will have to be decided.
sary mensrea and actus reus. In other words, for
offences under this section, all the elements of Section 309: Attempt to commit suicide
murder exist, except for the fact that death has Whoever attempts to commit suicide and does
not occurred. any act towards the commission of such offence,

CLAT.indb 35 31/03/2009 11:41:39


36 CRIMINAL LAW

shall be punished with simple imprisonment for for a period less than 20 days, it would be an
a term which may extend to one year or with offence of hurt.
fine, or with both. Section 321: Voluntarily Causing Hurt
Suicide is as such no crime under the Code.
“Whoever does any act with the intention
However, attempt to commit suicide is made
thereby causing hurt to any person, or with the
punishable under this section. Mens rea is one
knowledge that he is likely thereby to cause hurt
of the essential elements of this offence.
to any person, and does thereby cause hurt to
Section 319: Hurt any person, is said “voluntarily to cause hurt”.
“Whoever causes bodily pain, disease or infir- Section 322: Voluntarily Causing Grievous
mity to any person is said to cause hurt”. Hurt
There is nothing in this section to suggest “Whoever voluntarily causes hurt, if the hurt
that the hurt should be caused by direct physical which he intends to cause know himself to
contact between the accused and the victim. It be likely to cause is grievous hurt, and if the
may be caused by any means and includes ner- hurt which he causes is grievous hurt, is said
vous shock or mental derangement. “voluntarily to cause grievous hurt.”
‘Bodily pain’, except so slight a harm of Explanation. A person is not said voluntarily to
which no person of ordinary sense or temper cause grievous hurt except when he both causes
would complain of, is covered by the definition grievous hurt and intends or knows himself to
of hurt. Dragging by hair in aggressive manner be likely to cause grievous hurt. But he is said
and fisting in course of attack are not trivial acts voluntarily to cause grievous hurt, if intending or
and constitute offence of causing hurt. knowing himself to be likely to cause grievous
‘Infirmity’ has been defined as inability of hurt of one kind, he actually causes grievous
an organ to perform its normal function which hurt of another kind.
may either be temporary or permanent. A state
Section 359: Kidnapping
of temporary impairment or hysteria or terror
would constitute infirmity. Kidnapping is of two types: kidnapping
from India, and kidnapping from lawful
Section 320: Grievous Hurt
guardianship.
“The following kinds of hurt only are designated The literal meaning of kidnapping is ‘child
as “grievous”: stealing’. The two forms of kidnapping may
First—Emasculation. overlap each other.
Second—Permanent privation of the sight of
Section 360: Kidnapping from India
either eye.
Third—Permanent privation of the hearing of Whoever conveys any person beyond the limits
either ear. of India without the consent of that person, or
Fourth—Privation of any member or joint. of some person legally authorized to consent
Fifth—Destruction or permanent impairing of on behalf of that person, is said to kidnap that
the powers of any member or joint. person from India.
Sixth—Permanently disfiguration of the head For an offence under this section, it does
or face. not matter that the victim is a major or minor.
Seventh—Fracture or dislocation of a bone or Section 361: Kidnapping from Lawful Guard-
tooth. ianship
Eighth—Any hurt which endangers life or Whoever takes or entices any minor under sixteen
which causes the sufferer to be during the space years of age if a male, or under eighteen years of
of twenty days in severe bodily pain, or unable age if a female, or any person of unsound mind,
to follow his ordinary pursuits. out of the keeping of the lawful guardian of such
The mere fact that the injured remained in minor or person of unsound mind, without the
the hospital for 20 days would not be enough to consent of such guardian, is said to kidnap such
conclude that he was unable to follow his ordi- minor or person from lawful guardianship.
nary pursuits during that period. Explanation. The word “lawful guardian”
Continuance of severe bodily pain for 20 in this section include any person lawfully
days or disability to follow one’s avocation for entrusted with the care or custody of such minor
20 days constitutes grievous hurt; if it continues or other person.

CLAT.indb 36 31/03/2009 11:41:39


CRIMINAL LAW 37

Exception. This section does not extend to the marry her or arrange for her marriage.
act of any person who in good faith believes Where a girl was unwilling to follow the
himself to be the father of an illegitimate child, accused and accused compelled her by force
or who in good faith believes himself to be to follow, he would be guilty of abduction. But
entitled to the lawful custody of such child, where the accused entered a girl’s house and
unless such act is committed for an immoral or lifted her up in order to carry her away but on
unlawful purpose. raising of alarm by her, they dropped her and
Besides the four essential ingredients ran away, the offence of abduction was not com-
of this offence mentioned in Section 361, the mitted because the girl was not compelled to go
courts have formulated certain other guiding from her place. The accused were guilty of at-
principles: tempting to abduct under Sections 366 and 511.
(i) In the case of minor girls this section is
On being persuaded by the accused the
attracted irrespective of the question
victim when inside his house and came properly
whether she is married or unmarried.
dressed to accompany the accused. This will not
(ii) The consent of the minor is immaterial.
amount to ‘abduction’ within the meaning of
(iii) The motive or intention of the kidnapper
Section 362 as there was neither force nor de-
is also immaterial.
ceit which compelled the victim to go with the
(iv) If the kidnapped girl turns out to be under
accused.
18 years of age, the kidnapper must take
the consequences, even though the bona- Where the mother had assigned by a will
fide believed and had reasonable ground to R the guardianship of her minor daughter and
for believing that she was over eighteen. the duty of marrying her, and a parental relative
(v) The defence that the girl was of easy virtue of the daughter removed her by fraud and force
would not be sufficient to make accused for the purpose of getting her married to a per-
not liable. son other than the one selected by R, it was held
that such relative was guilty of kidnapping as
Section 362: Abduction
well as abduction.
“Whoever by force compels, or by any deceitful
means induces, any person to go from any place,
DISTINCTION BETWEEN
is said to abduct that person.”
KIDNAPPING AND ABDUCTION
The expression ‘deceitful means’ includes
a misleading statement. It is really speaking a Whereas a girl of 18 years of age or over can
matter of intention. The intention of the accused only be abducted and not kidnapped, a girl
is the basis and the gravity of the offence. The below 18 years of age can be kidnapped (if
volition, the intention and the conduct of the under a lawful guardian) as well as abducted (if
woman do not determine the offence. The of- with or without a guardian and use of force or
fence of abduction is complete if the accused deceitful means) provided the other statutory
has taken the women away by deceitful means conditions are satisfied. The differences in detail
intending to seduce her to sexual intercourse to are as follows:
KIDNAPPING ABDUCTION
(From Guardianship)
(1) It is committed only in respect of a minor (1) It is committed in respect of any person of
or a person of unsound mind. any age.
(2) The person kidnapped is removed from (2) Not necessary. The person abducted need not
lawful guardianship. A child without a be in the keeping of anybody.
guardian can’t be kidnapped.
(3) The minor or the person of unsound mind (3) Force, compulsion or deceitful means are
is simply taken away or enticed to go with employed.
the kidnapper. The means employed may
be innocent.
(4) Consent of the person enticed is (4) Consent of the person removed, if freely and
immaterial. voluntarily given condones the offence.

CLAT.indb 37 31/03/2009 11:41:39


38 CRIMINAL LAW

(5) The intent of the offender is irrelevant. (5) It is very important. Abduction must be with
certain intent.
(6) It is not a continuing offence. The (6) It is a continuing offence. A person is being
offence is completed as soon as the minor abducted both when she is first taken from
is removed from the custody of his or her any place and also when she is removed from
guardian. one place to another.
(7) It is a substantive offence, punishable (7) It is an auxiliary act, not punishable by itself
under Section 363, IPC. Thus, kidnapping is unless accompanied with some criminal
per se punishable. intent (specified in Secs. 364-366). A particular
purpose is necessary to punish the accused.
Section 375: Rape Amendments in Law of Rape
Section 375, IPC lays down that the sexual The Parliament in 1983 extensively amended the
intercourse must be under circumstances falling law of rape so as to make the law more realistic
under any of the six clauses. A man is said to and to safeguard the rights of innocent victims.
commit ‘rape’ who has sexual intercourse with Besides substantive law (I.P.C.), procedural
a women: provisions under Evidence Act and Criminal
(i) against her will; or Procedure code were also added to strengthen
(ii) without her consent; or the law. Some of the important changes brought
(iii) with her consent by putting her in fear of about by the Criminal Law (Amendment) Act,
death or of hurt; or 1983 are:
(iv) with her consent, when the man knows (i) Consent of a woman of unsound mind, etc.
that he is not her husband and that her A new clause ‘fifthly’ added to Section
consent is given because she believes that 375, IPC, negatives the consent of the
he is another man to whom she is lawfully women for the purpose of the offence of
married; or rape, if the woman is of unsound mind, or
(v) with her consent, when, at the time of is under the influence of intoxication at the
such consent, by reason of unsoundness of relevant time.
mind or intoxication or the administration (ii) Minimum punishment for rape: Section
by him personally or through another of
376, IPC has provided for a minimum of 7
any stupefying or unwholesome substance
years of imprisonment under cl. (1) and 10
she is unable to understand the nature and
years under cl. (2).
consequences of that to which she gives
(iii) Intercourse with wife during judicial
consent; or
separation without her consent prohibited:
(vi) with or without her consent when she is
Section 376 A, IPC, added.
under 16 years of age.
(iv) Custodial rape: Section 376 B to Section
Explanation. Penetration is sufficient to consti-
376D, IPC provided for a new category
tute sexual intercourse.
Exception. Sexual intercourse by a man with of offence, known as custodial rape. Such
his own wife is not rape, if the wife is above 15 cases do not amount to rape because the
years of age. victim’s consent is obtained under the
It may be noted that if the woman is under compelling circumstances. The offenders
16 years of age, it is immaterial that the act be occupy supervisory positions and take
done with her consent or even at the invitation undue advantage of their authority.
of woman herself (or that she had sex experi- (v) Burden of proof of innocence of accused:
ence already), for the policy of the law is to Section 114A inserted in Evidence Act,
protect children of such immature age against 1872, provides for a conclusive presump-
sexual intercourse. This is also known as ‘statu- tion as to the absencer of consent of the
tory rape’. woman under Section 376 (2) viz. custo-
A ‘man’ is defined by Section 10 of the code dial rape, rape on pregnant women and
as a male human being of any age. Thus, a boy gang rape.
above 12 years of age is capable of committing This has at least partially removed
rape under this section, whereas a boy below 12 the infirmity from the evidence of a
but above 7 years of age enjoys a qualified im- victim of rape that was hitherto unjustly
munity. attached to her testimony without taking

CLAT.indb 38 31/03/2009 11:41:39


CRIMINAL LAW 39

note of the fact that in India a disclosure Explanation 5. The consent mentioned in the
of this nature is likely to ruin the prospect definition may be express or implied, and may
of the girl’s rehabilitation in society for be given either by the person in possession, or
all times to come and unless her story by any person having for that purpose authority
was painfully true she would not have either express or implied.
taken such a grave risk merely to malign
the accurse. Moreover, in cases of rape, Illustrations
particularly custodial rape it is almost (a) A cuts down a tree on Z’s ground, with
impossible to get any other independent the intention of dishonestly taking the tree
evidence to corroborate the testimony of out of Z’s possession without Z’s consent.
the prosecutrix. Here, as soon as A has severed the tree in
(vi) Prohibition of disclosure of identity of the order to such taking, he has committed
victim in rape cases: Section 228A (1), theft.
IPC, added. (b) A puts a bait for dogs in his pockets, and
(vii) Trial in camera: Section 327, Cr.P.C, which thus induces Z’s dog to follow it. Here, if
confers the right of an open court trial has A’s intention be dishonestly to take the
been amended making the provisions for dog out of Z’s possession without Z’s
trial of rape in camera and prohibition of consent, A has committed theft as soon as
publication of trial proceedings in such Z’s dog has begun to follow A.
cases without the prior approval of the (c) A meets a bullock carrying a box of
court. treasure. He drives the bullock in a certain
Section 228A, IPC, and Section 327, direction, in order that he may dishonestly
Cr.P.C. not only protect the honour of take the treasure. As soon as the bullock
sexually victimized women but also make begins to move, A has committed theft of
it possible for them to depose in court the treasure.
without any fear of social ostracism. (d) A being Z’s servant, and entrusted by Z
Character assassination of prosecutrix with the care of Z’s plate, dishonestly runs
prohibited: A ‘proviso clause’ to Section 146 away with the plate, without Z’s consent.
Evidence Act, inserted via 2003 amendment A has committed theft.
Act, has disallowed to put questions about (e) Z, going on a journey, entrusts his plate to
prosecutrix character in cross-examination. A, the keeper of a warehouse, till Z shall
Section 378: Theft return. A carries the plate to a goldsmith
and sells it. Here, the plate was not in
Whoever, intending to take dishonestly any
Z’s possession. It could not therefore be
movable property out of the possession of any
taken out of Z’s possession, and A has
person without that person’s consent, moves
that property in order to such taking, is said to not committed theft, though he may have
commit theft. committed criminal breach of trust.
Explanation 1. A thing so long as it is attached (f) A finds a ring belonging to Z on a table in
to the earth, not being movable property, is not the house which Z occupies. Here, the ring
the subject to theft; but it becomes capable of is in Z’s possession, and if a dishonestly
being the subject of theft as soon as it is severed removed it, A commits theft.
from the earth. (g) A finds a ring lying on the highroad, not
Explanation 2. A moving effected by the same in the possession of any person. A, by
act which effects the severance may be a theft. taking it, commits no theft, though he
Explanation 3. A person is said to cause a may commit criminal misappropriation of
thing to move by removing an obstacle which property.
prevented it from moving or by separating it (h) A sees a ring belonging to Z lying on a
from any other thing, as well as by actually table in Z’s house. Not venturing to mis-
moving it. appropriate the ring immediately for fear
Explanation 4. A person, who by any means of search and detection, A hides the ring
causes an animal to move, is said to move in a place where it is highly improbable
that animal, and to move everything which, in that it will ever be found by Z, with the in-
consequence of the motion so caused, is caused, tention of taking the ring from the hiding
is moved by that animal. place and selling it when the loss is forgot-

CLAT.indb 39 31/03/2009 11:41:40


40 CRIMINAL LAW

ten. Here A, at the time of first moving the to be A’s own property, takes that property
ring, commits theft. out of B’s possession. Here, as A does
(i) A delivers his watch to Z, a jeweller, to be not take dishonestly, he does not commit
regulated. Z carries it to his shop. A, not theft.
owing to the jeweller any debt for which Section 383: Extortion
the jeweller might lawfully detain the
Whoever, intentionally puts any person in fear
watch as a security, enters the shop openly,
of any injury to that person, or to any other, and
takes his watch by force out of Z’s hand,
thereby dishonestly induces the person so put
and carried it away. Here A, though he
in fear to deliver to any person any property of
may have committed criminal trespass and
valuable security, or anything signed or sealed
assault, has not committed theft, inasmuch
which may be converted into a valuable security,
as what he did was not done dishonestly.
commits “extortion”.
(j) If A owes money to Z for repairing the
watch, and if Z retains the watch lawfully Illustrations
as a security for the debt, and A takes (a) A threatens to publish a defamatory libel
the watch out of Z’s possession, with the concerning Z unless Z gives him money.
intention of depriving Z of the property as He thus induces Z to give him money. A
a security for his debt, he commits theft, committed extortion.
inasmuch as he takes it dishonestly. (b) A threatens Z that he will keep Z’s child in
(k) Again, if A, having pawned his watch to Z, wronglful confinement unless Z will sign
takes it out of Z’s possession without Z’s and deliver to A a promissory note binding
consent, not having paid what he borrowed Z to pay certain money to A. Z signs
on the watch, he commits theft, though the and delivers the note. A has committed
watch is his own property inasmuch as he extortion.
takes it dishonestly. (c) A threatens to send club-men to plough
(l) A takes an article belonging to Z out of up Z’s field unless Z will sign and deliver
Z’s possession, without Z’s consent, to B a bond binding Z under a penalty to
with the intention of keeping it until he deliver certain produce to B, and thereby
obtains money from Z as a reward for its induces Z to sign and deliver the bond. A
restoration. Here A takes dishonestly; A has committed extortion.
has therefore committed theft. (d) A, by putting Z in fear of grievous hurt,
(m) A, being on friendly terms with Z, goes dishonestly induces Z to sign or affix his
into Z’s library in Z’s absence, and takes seal to a blank paper and deliver it to A.
away a book without Z’s express consent Z signs and delivers the paper to A. Here,
for the purpose merely of regarding it, and as the paper so signed may be converted
with the intention of returning it. Here, it into a valuable security, A has committed
is probable that A may have conceived extortion.
that he has Z’s implied consent to use Z’s
book. If this was A’s impression, A has CRIMINAL MISAPPROPRIATION
not committed theft.
AND CRIMINAL BREACH OF TRUST
(n) A asks charity from Z’s wife. She gives
A money, food and clothes, which A Section 403: Dishonest Misappropriation of
knows to belong to Z, her husband. Here Property
it is probable that A may conceive that Whoever dishonestly misappropriates or con-
Z’s wife is authorized to give away alms. verts to his own use any movable property,
If this was A’s impression, A has not shall be punished with imprisonment of either
committed theft. description for a term which may extend to two
(o) A is the paramour of Z’s wife. She gives years, or with fine, or with both.
a valuable property, which A knows to
belong to her husband Z, and to be such Illustrations
property as she has not authority from Z to (a) A takes property belonging to Z out of Z’s
give. If A takes the property dishonestly, possession, in good faith believing, at the
he commits theft. time when he takes it, that the property
(p) A, in good faith believing belonging to Z belongs to himself. A is not guilty of theft;

CLAT.indb 40 31/03/2009 11:41:40


CRIMINAL LAW 41

but if A, after discovering his mistake, Illustrations


dishonestly appropriates the property to (a) A finds a rupee on the high-road, not
his own use, he is guilty of an offence knowing to whom the rupee belongs.
under this section. A picks up the rupee. Here, A has not
(b) A, being on friendly terms with Z, goes committed the offence defined in this
into Z’s library, in Z’s absence, and takes section.
away a book without Z’s implied consent (b) A finds a letter on the road containing a
to take the book for the purpose of reading bank note. From the direction and contents
it, A has not committed theft. But, if A of the letter he learns to whom the note
afterwards sells the book for his own belongs. He appropriates the note. He is
benefit, he is guilty of an offence under guilty of an offence under this section.
this section. (c) A finds a cheque payable to bearer. He
(c) A and B, being joint owners of a horse, can form no conjecture as to the person
A takes the horse out of B’s possession, who has lost the cheque. But the name of
intending to use it. Here, as A has a right the person, who has drawn the cheque,
to use the horse, he does not dishonestly appears. A knows that this person can
misappropriate it. But, if A sells the horse direct him to the person in whose favour
the cheque was drawn. A appropriates the
and appropriates the whole proceeds to his
cheque without attempting to discover the
own use, he is guilty of an offence under
owner. He is guilty of an offence under
this section.
this section.
Explanation 1. A dishonest misappropriation (d) A sees Z drop his purse with money in it.
for a time only is a misappropriation within the A picks up the purse with the intention of
meaning of this section. restoring it to Z, but afterwards appropri-
Illustration ates it to his own use. A has committed an
offence under this section.
A finds a Government promissory note belonging (e) A finds a purse with money, not knowing
to Z, bearing a blank endorsement. A, knowing to whom it belongs; he afterwards discover
that the note belongs to Z, pledges it with a that it belongs to Z, and appropriates it
banker as a security for a loan, intending at a to his own use. A is guilty of an offence
future time to restore it to Z. A has committed under this section.
an offence under this section. (f) A finds a valuable ring, not knowing to
Explanation 2. A person who finds property whom it belongs. A sells it immediately
not in the possession of any other person, and without attempting to discover the owner.
takes such property for the purpose of protecting A is guilty of an offence under this
it for, or of restoring it to, the owner, does not section.
take or misappropriate it dishonestly, and is
not guilty of an offence; but he is guilty of the ESSENTIALS INGREDIENTS OF
offence above defined, if he appropriates it to CRIMINAL MISAPPROPRIATION
his own use, when he knows or has the means
of discovering the owner, or before he has used The offence of criminal misappropriation consists
reasonable means to discover and give notice to in dishonest misappropriation or conversion to
the owner and has kept the property a reasonable his own use any movable property.
It takes place when the possession has been
time to enable the owner to claim it.
innocently come by, but where, a subsequent
What are the reasonable means or what is
change of intention, or from the knowledge of
reasonable time in such a case is a question of
some new fact with which the party was not
fact. previously acquainted, the retaining becomes
It is not necessary that the finder should wrongful and fraudulent.
know who is the owner of the property, or that The two essential ingredients of this sec-
any particular person is the owner of it: it is tion are:
sufficient if, at the time of appropriating it, he (i) the accused misappropriated or converted
does not believe it to be his own property, or in to his own use another’s movable property,
good faith believes that the real owner cannot and,
be found. (ii) the accused did so dishonestly.

CLAT.indb 41 31/03/2009 11:41:40


42 CRIMINAL LAW

To ‘misappropriate’ means ‘improperly such trust is to be discharged, or of any legal


setting apart for one’s use to the exclusion of contract, express or implied, which he has made
the owner’. ‘Converts’ means appropriation and touching the discharge of such trust, or willfully
dealing with property of another without right suffers any other person so to do so, commits
as if it is his own property. The accused is not “criminal breach of trust”.
guilty when he merely retains or possesses such Explanation 1. A person, being an employer,
property. He is guilty only when he appropriates who deducts the employee’s contribution from
or converts to his own use such property. Fur- the wages payable to the employee for credit
ther, the property must be a movable property. to a Provident Fund or Family Pension Fund
A man is said to do a thing “dishonestly” established by any law for the time being in
when he does it with the intention of causing force, shall be deemed to have been entrusted
wrongful gain to one person or wrongful loss with the amount for the contribution so deducted
to another person. It follows that a mere misap- by him and if he makes default in the payment of
propriation or conversion to one’s use is not suf- such contribution to the said Fund in violation of
ficient for the completion of an offence, but that the said law, shall be deemed to have dishonestly
the element of dishonesty is essential, and dis- used the amount of the said contribution in
honesty comes into existence as soon as there is violation of a direction of law as aforesaid.
an intention of causing, wrongful gain or loss. Explanation 2. A person, being an employer,
Where a person found a purse and put it who deducts the employee’s contribution from
in his pocket, but was immediately afterwards the wages payable to the employee for credit to
arrested, he was not guilty of criminal misap- the Employee’s State Insurance Fund held and
propriation for it could not be assumed that by administered by the Employee’s State Insurance
the mere act of picking up the purse or putting it Corporation established under the Employee’s
in his pocket he intended to appropriate its con- State Insurance Act, 1948, shall be deemed to
tents to his own use. Mere retention of money have been entrusted with the amount of the
would not warrant a conviction under Section contribution so deducted by him and if makes
403 unless there is evidence that the accused default in the payment of such contribution to
used the money. the said Fund in violation of the said Act, shall be
Misappropriation or conversion need not deemed to have dishonestly used the amount of
be permanent, it may be for a time. The accused, the said contribution in violation of a deduction
a government servant, on duty received certain of law as aforesaid.
money in his possession for several months,
Illustrations
but fearing detection, had paid them into the
treasury, making a false entry at the time in his (a) A, being executor to the will of a deceased
books. He was held guilty of criminal misappro- person, dishonestly disobeys the law
priation. Similar would be the case where a ser- which directs him to divide the effects ac-
vant, authorized to collect money on behalf of cording to the will and appropriates them
his master, retained money on account of non- to his own use. A has committed criminal
payment of wages by his master. breach of trust.
The accused would not be guilty of the of- (b) A is a warehouse-keeper. Z, going on a
fence, where there was no information as to the journey, entrust his furniture to A, under
a contract that it shall be returned on pay-
circumstances under which the things were lost
ment of a stipulated sum for warehouse-
and the probability was that property was aban-
room. A dishonestly sells the goods. A has
doned by the original
committed criminal breach of trust.
(c) A, residing in Calcutta, is agent for Z,
CRIMINAL BREACH OF TRUST residing at Delhi. There is an express or
implied contract between A and Z that all
Section 405. Criminal Breach of Trust sums remitted by Z to A shall be invested
Whoever, being in any manner entrusted with by A, according to Z’s direction. Z remits
property, or with any dominion over property, a lakh of rupees to A, with directions to
dishonestly misappropriates or converts to his A to invest the same in Company’s paper.
own use that property, or dishonestly uses or A dishonestly disobeys the directions and
disposes of that property in violation of any employs the money in his own business. A
direction of law prescribing the mode in which has committed breach of trust.

CLAT.indb 42 31/03/2009 11:41:40


CRIMINAL LAW 43

(d) But if A, in the last illustration, not dis- article, intentionally deceives Z into be-
honestly but in good faith, believing that lieving that the article corresponds with
it will be more for Z’s advantage to hold the sample, and thereby dishonestly in-
shares in the Bank of Bengal, disobeys Z’s duces Z to buy and pay for the article. A
directions, and buys shares in the Bank of cheats.
Bengal, for Z, instead of buying Compa- (d) A, by tendering in payment for an article
ny’s paper, here, though Z should suffer a bill on a house with which A keeps no
loss, and should be entitled to bring a civil money, and by which A expects that the
action against A, on account of that loss, bill will be dishonoured, intentionally de-
yet A, not having acted dishonestly, has ceives Z, and thereby dishonestly induces
not committed criminal breach of trust. Z to deliver the article, intending not to
(e) A, a revenue-officer, is entrusted with pub- pay for it. A cheats.
lic money and is either directed by law, or (e) A, by pleading as diamonds articles which
bound by a contract, express or implied, he knows are not diamonds, intentionally
with the Government, to pay into a cer- deceives Z, and thereby dishonestly in-
tain treasury all the public money which duces Z to lend money. A cheats.
he holds. A dishonestly appropriates the (f) A intentionally deceives Z into a belief
money. A has committed criminal breach that A means to repay any money that Z
of trust. may lend to him and thereby, dishonestly
(f) A, a carrier, is entrusted by Z with prop- induces Z to lend him money, A not in-
erty to be carried by land or by water. A tending to repay it. A cheats.
dishonestly misappropriates the property. (g) A intentionally deceives Z into a belief
A has committed criminal breach of trust. that A means to deliver to Z a certain
quantity of indigo plant which he does not
CHEATING intend to deliver, and thereby dishonestly
induces Z to advance money upon the
Section 415: Cheating faith of such delivery. A cheats; but if A,
Whoever, by deceiving any person, fraudulently at the time of obtaining the money, intends
or dishonestly induces the person so deceived to to deliver the indigo plant, and afterwards
deliver any property to any person, or to consent breaks his contract and does not deliver it,
that any person shall retain any property, or he does not cheat, but is liable only to a
intentionally induces the person so deceived to civil action for breach of contract.
do or omit to do anything which he would not do (h) A intentionally deceives Z into a belief
or omit if he were not so deceived, and which act that A has performed A’s part of a contract
made with Z, which he has not performed,
or omission causes or is likely to cause damage
and thereby dishonestly includes Z to pay
or harm to that person in body, mind, reputation
money. A cheats.
or property, is said to “cheat”.
(i) A sells and conveys an estate to B. A,
Explanation. A dishonest concealment of
knowing that in consequence of such sale
facts is a deception within the meaning of this
he has no right to the property, sells or
section.
mortgages the same to Z, without disclos-
Illustrations ing the fact of the previous sale and con-
(a) A, by falsely pretending to be in the Civil veyance to B, and receives the purchase or
Service, intentionally deceives Z, and thus mortgage money from Z. A cheats.
dishonestly induces Z to let him have on Section 416: Cheating by Personation
credit goods for which he does not mean A person is said to “cheat by personation” if he
to pay. A cheats. cheats by pretending to be some other person,
(b) A, by putting a counterfeit mark on an ar- or by knowingly substituting one person for
ticle, intentionally deceives Z into a belief another, representing that he or any other person
that his article was made by a certain cele- is a person other than he or such other person
brated manufacturer, and thus dishonestly really is.
induces Z to buy and pay for the article. A Explanation. The offence is committed whether
cheats. the individual personated is a real or imaginary
(c) A, by exhibiting to Z a false sample of an person.

CLAT.indb 43 31/03/2009 11:41:40


44 CRIMINAL LAW

Illustrations whereby the deceived person is dishonestly


(a) A cheats by pretending to be a certain rich induced:
banker of the same name. A cheats by 1. to deliver any property
personation. 2. to make, alter or destroy
(b) A cheats by pretending to be B, a person (a) the whole or any part of a valuable
security, or,
who is deceased. A cheats by personation.
(b) anything which is signed or sealed and
Under this section, a person commits cheat-
which is capable of being converted
ing when he pretends to be some other person,
into a valuable security.
real or imaginary e.g., false representation as to
The difference between Section 415 and
caste, marital status, economic status, voter in
Section 420 is that where in pursuance of the
an election, etc. The person may pretend so by
deception, no property passes but inducement
word, act, sign, or dress. The offence is commit-
generated in the mind, the offence comes under
ted provided some gain has accrued or some loss
Section 415 (Simple cheating). But where, in
is incurred by either party.
pursuance of the deception property is delivered,
Where the accused representing himself
the offence is punishable under Section 420.
to be B at a University examination, got a hall- Section 415 deals with cheating, but Section
ticket under the examinee’s name and wrote pa- 420 deals with that species of cheating which
pers in B’s name, it was held that the accused involves delivery of property or destruction of
had committed cheating by personation and the valuable security. Punishment for offence under
offence of forgery. Where A personated as B at Section 415 is one year (Section 417), while un-
the examination, passed it and obtained the cer- der Section 420, up to 7 years’ imprisonment.
tificate in B’s name, B thereupon applied to have In every case when property is delivered by
his own name entered in the list of candidates a person cheated, there must always be a stage
for Government service. In this case, A is liable where the person makes up his mind to give the
for cheating under Section 416 and B is liable property on accepting the false representation
for attempting to cheat because he has applied made to him. It cannot be said that in such cases
for entering his name in the list of candidates. the offender can only be tried for the simple of-
Securing appointments from Government fence of cheating under Section 417 and cannot
officials by producing fake letters from Minis- be tried under Section 420 because the person
ters and also by posing to be the brother of a cheated parts with his property subsequent to
Minister, has been held to constitute an offence making up his mind to do so.
of cheating by personation, and of forgery.
Section 418. Cheating with knowledge MOCK OR INVALID MARRIAGES
that wrongful loss may ensue to person whose
interest offender is bound to protect—Whoev- Section 493. Cohabitation caused by a man
er cheats with the knowledge that he is likely deceitfully inducing a belief of lawful marriage.
thereby to cause wrongful loss to a person Every man who by deceit causes any woman
whose interest in the transaction to which the who is not lawfully married to him to believe that
cheating relates, he was bound, either by law, or she is lawfully married to him and to cohabit or
by legal contract, to protect, shall be punished have sexual intercourse with him in that belief,
with imprisonment of upto 3 years, or fine, or shall be punished with imprisonment of either
with both. description for a term which may extend to ten
This section deals with cheating a guard- years, and shall also be liable to fine.
ian, a trustee, a pleader, and agent, or manager Section 496. Marriage ceremony fraudulently
of a Hindu family or by directors or managers of gone through without lawful marriage. Whoever,
a bank in fraud of the shareholders and deposi- dishonestly or with a fraudulent intention,
tors. Thus, where a false balance sheet is shown goes through the ceremony of being married,
with an intent to secure or renew the deposits for knowing that he is not thereby lawfully married,
the company, the culprits are guilty under Sec- shall be punished with imprisonment of either
tion 418. description for a term which may extend to
Section 420: Cheating and Dishonestly seven years, and shall also be liable to fine.
Inducing Delivery of Property Section 494: Bigamy
This section deals with certain specified classes Marrying again during lifetime of husband or
of cheating. It deals with the cases of cheating wife. Whoever, having a husband or wife living,

CLAT.indb 44 31/03/2009 11:41:41


CRIMINAL LAW 45

marries in any case in which such marriage is being the husband or the relative of the husband
void by reason of its taking place during the life of a woman, subject such woman to cruelty
of such husband or wife, shall be punished with shall be punished with imprisonment for a term
imprisonment of either description for a term which may extend to three years and shall also
which may extend to seven years, and shall also be liable to fine.
be liable to fine. Explanation. For the purpose of this section,
Exception. This section does not extend to ‘cruelty’ means: (a) any wilful conduct which is
any person whose marriage with such husband of such a nature as is likely to drive the woman
or wife has been declared void by a court or to commit suicide or to cause grave injury or
competent jurisdiction, nor to any person who danger to life, limb or health (whether mental
contracts a marriage during the life of a framer or physical) of the woman; or (b) harassment
or wife, if such husband or wife, at the time of the woman where such harassment is with a
of the subsequent marriage, shall have been view to coercing her or any person related to her
continually absent from such person for the to meet any unlawful demand for any property
space of seven years, and shall not have been or valuable security or is on account of failure
heard of by such person as being alive within by her or any person related to her to meet such
that time provided the person contracting such demand.
subsequent marriage shall, before such marriage
takes place, inform the person with whom such GENERAL EXCEPTIONS
marriage is contracted of the real state of facts so
far as the same are within his or her knowledge. Mistake
Section 497: Adultery Section 76. Act done by a person bound, or by
Whoever has sexual intercourse with a person mistake of fact believing himself bound, by
who is and whom he knows or has reason to law—
believe to be wife of another man, without the Nothing is an offence which is done by a person
consent or connivance of that man, such sexual who is, or who by reason of a mistake of fact
intercourse not amounting to the offence of and not by reason of mistake of law, in good
rape, is guilty of the offence of adultery, and faith, believes himself to be bound by law to do
shall be punished with imprisonment of either it.
description for a term which may extend to five
years, or with fine, or with both. In such case the Illustrations
wife shall not be punishable as an abettor. (a) A, a soldier, fires on a mob by the order of
his superior officer, in conformity with the
CRIMINAL ELOPEMENT/ commands of the law. A has committed no
SEDUCTION offence.
(b) A, an office of a Court of Justice, being
Section 498. Enticing or taking away or ordered by that court to arrest Y, and after
detaining with criminal intent a married woman. due inquiry, believing Z to be Y, arrest Z.
Whoever takes or entices away any woman who A has committed no offence.
is and whom he knows or has reason to believe ‘Mistake’ is a slip made, not by design, but by
to be the wife of any other man, from that man, mischance. Even under English common law,
or from any person having the care of her on an honest and reasonable belief in the existence
behalf of that man, with intent that she may have of circumstances which would have made that
illicit intercourse with any person ,or conceals act an innocent act, has always been a valid
or detains with that any intent any such woman, defence.
shall be punished with imprisonment of either “When a person is ignorant of the exis-
description for a term which may extend to two tence of relevant facts, or mistaken as to them,
years, or with fine, or with both. his conduct may produce harmful results which
he neither intended nor foresaw. Mistake can be
CRUELTY BY HUSBAND OR admitted as a defence provided
RELATIVES OF HUSBAND (1) that the state of things believed to exist
would, if true, have justified the act done,
Section 498-A. Husband or relative of husband and
of a woman subjecting her to cruelty. Whoever, (2) the mistake must be reasonable, and

CLAT.indb 45 31/03/2009 11:41:41


46 CRIMINAL LAW

(3) that the mistake relates to fact and not to The maxim respondent superior (‘act done
law”. by the order of a superior’) has no application
Ignorance of fact is excusable (Ignorantia in criminal law. The order of a superior to an
facti). Ignorance includes mistake, but mistake inferior servant to commit an offence is not a
does not necessarily includes ignorance. Igno- valid defence.
rance means lack of knowledge, a general inertia Thus, where under order of their naik, three
of human mind. while, mistake is not the result sepoys of a regiment fired a shot at a mob which,
of foolhardiness nor it is caused by any lack of otherwise, was by no means violent, it was held
mental alertness. Thus, mere forgetfulness is not that they were guilty of culpable homicide not
mistake. Mistake of fact implies mistake as to amounting to murder, and they were not bound
true identities or mistake in sensory perceptions to obey an illegal order.
such as temporary distortion of imagination.
Section 79. Act done by a person justified, or
It is to be remembered that ‘mistake of law’
by mistake of fact believing himself justified,
is no defence, but ‘mistake of fact’ is. Everybody
by law.
is bound to know the law of the land, and ig-
norance of the law is no excuse. This is based Nothing is an offence which is done by any
on the assumption that if a person exercises due person who is justified by law, or who by reason
care and diligence (e.g., by consulting a lawyer), of a mistake of fact or by a reason of a mistake of
he would know the law. law in good faith, believes himself to be justified
However, there may arise circumstances by law, in doing it.
neutralizing the presumption of knowledge of
Illustration
law in every individual, as in the case of newly
enacted statute. A sees Z commit what appears to be murder. He
A mixed question of law and fact is treated in good faith seizes Z, to hand him over to the
as a question of fact if the accused was mislead police. A has committed no offence, though it
into an error of fact on account of an error of may turn out that Z was acting in self-defence.
law. The following rules determined the ques- Unsoundness Of Mind
tion of justification of an offence either due to
mistake of fact or of law: Section 84. Act of person of unsound mind
(i) When an act is in itself plainly criminal, “Nothing is an offence which is done by a
but is more severely punishable if cer- person who, at the time of doing it, by reason of
tain circumstances exist, then ignorance unsoundness of mind, is incapable of knowing
of those circumstances is no answer to a the nature of the act or that he is doing what is
charge for the aggravated offence. either wrong or contrary to law”.
(ii) If, however, an act is prima facie innocent, This section provides that a man who, by
but is an offence if certain circumstances reason of unsoundness of mind, prevented from
exist, then ignorance of those circum- controlling his own conduct and deprived of the
stances is a good defence to the charge. power of passing a rational judgement, on the
(iii) If the act itself is wrong, and becomes material character of the act he meant to do, can-
criminal under certain circumstances, the not be legally responsible for the act.
person who commits such a wrongful act The section is based on the principle that
cannot argue that he was ignorant of the an action does not constitute an offence unless
facts which turned the wrong into crime. done with a guilty intention. Therefore, insan-
(iv) The state of the defendant’s mind must ity is recognized as a general defence, because
amount to absolute ignorance of the ex- insane persons are incapable of entertaining a
istence of the circumstances which alter blame worthy intention; they do not know what
the character of the act, or to a belief in its they are doing. No culpability can be fastened
non-existence. upon insane persons as they have no free will
When mistake of fact is no defence. Mistake of (Furiosi nulla voluntas est).
fact is no defence if the fact itself is illegal. One The words ‘unsoundness of mind’ include
cannot do an illegal act and then plead ignorance these persons:
of a fact. Thus, a person cannot by mistake of idiot (born);
fact, shoot X and then plead in defence that he one made non compos mentis by illness
did not intend to kill X at all, but mistaken him (temporary failure);
for Y, whom he wanted to kill. a lunatic or a mad man (mental disorder);

CLAT.indb 46 31/03/2009 11:41:41


CRIMINAL LAW 47

a person in unconscious state, if proved from the medical point of view, he could
(e.g., sleep walking); not be exonerated under Section 84. Legal
and intoxicated person. insanity recognizes only the impairment
Tests/Principles to determine insanity of a of cognitive faculties and when cognitive
person faculties are not impaired and only will
and emotions are affected, insane impulses
(1) The crucial point of time for deciding are not a defence.
whether the benefit of Section 84 is to be (8) Insane delusions—Akin to lunacy, is in-
given or not, is the time when the offence sane delusion which is a borderline case.
takes place (“who at the time of doing Whether a person who under an insane
it”). It must be shown that the accused delusion as to the existing facts commits
was of unsound mind at the time of the an offence in consequence thereof, is to
commission of the offence. If he was not be executed, depends upon the nature of
insane at that time but became insane later, the delusion. If he labours under a partial
he cannot take the benefit of Section 84. delusion only, and is not in other respects
(2) In order to see whether the accused was insane, he must be considered in the same
insane at the time of the commission of situation as to responsibility as if the facts,
offence, the state of his mind before and with respect to which the delusion exists,
after the commission of offence is relevant. were real. In other words, he will be re-
For instance, evidence of pre-meditation, sponsible for the crime committed by him.
an attempt to evade or resist arrest etc. Similar is the case with fancied delusions.
(3) History of previous insanity including any Example. A was suffering under an insane
medical history of the same, the behaviour delusion that X and Y were persecuting
of the accused on the day of occurrence, him. He bought a knife in order to revenge
and his post-occurrence behaviour are also himself on them, and that very evening he
relevant to be taken into consideration. went to their club and stabbed them dead.
(4) Absence of any motive, absence of The fact that he actually bought a knife and
secrecy, want of pre-arrangement and want went to the club of his victims shows that
of accomplices are also relevant factors. he knew what he was going to do, namely,
However, taken alone these factors would an act contrary to law, he had an intention
not be sufficient. to kill. Thus, A would be guilty of murder.
(5) What Section 84 expects is an inherent or But this would not be so if, at the time of
organic incapacity (i.e., incapability) and committing the act, A is so deranged as
not a wrong or erroneous belief which not to know what he was doing or that he
may be the result of a perverted illusion. was doing an act contrary to law.
(6) Naturally impaired cognitive faculties (9) Delirium—Persons who are occasionally
of mind form a ground of the defence of “possessed” by the ‘spirits’ and those
unsoundness of mind. The agitation of who, being in fits of delirium, very often
mind (or uncountable impulses) does not conjure up visions or images are given the
necessarily lead to an inference that it has benefit of Section 84.
affected one’s mental capacity. Similarly, However, in cases of delirium
‘moral insanity’ or weak/defective intellect tremens—a kind of madness brought about
is no defence. Further, insanity is different by habitual excessive liquor/illness, if the
from eccentricity or strange behaviour. patient knows as to what he was doing, he
(7) Medical/Legal insanity—The courts are would be criminally liable.
concerned with the legal and not with the (10) Recent trends—The right and wrong test
medical view of the question. A man may (i.e., the accused should be incapable of
be suffering from some forms of insanity knowing whether the act done by him
in the sense in which the term is used by is right or wrong) no longer dominates
medical men but may not be suffering this branch of criminal law and mental
from unsoundness of mind as is described abnormality falling short of complete
in Section 84. If the facts of a particular insanity is a limited defence establishing
case show that the accused knew that he a claim of diminished responsibility under
had done something wrong, it did not Section 2 of the Homicide Act (English),
matter how, though he might be insane 1957.

CLAT.indb 47 31/03/2009 11:41:41


48 CRIMINAL LAW

RIGHT OF PRIVATE DEFENCE and such right continues so long as such


apprehension of danger to the body
Section 96: Things done in private defence continues. This right rests on the general
“Nothing is an offence which is done in the principle that where crime is endeavoured
exercise of the right of private defence”. to be committed by force, it is lawful to
The doctrine of ‘right of private defence’ is repel that force in self-defence. To say
founded on the following ten expediencies: that a person could only claim the right to
(i) A private citizen whose life is threatened use force after he had sustained a serious
by a grave danger, need not wait for the injury by an aggressive wrongful assault is
State aid; however, where aid is available, a complete misunderstanding of the law.
it must be obtained. (vii) The protective measures employed must be
(ii) The right of defence is protective or relative to the danger ahead i.e., violence
preventive and not punitive (i.e., not used must be proportionate to the injury or
meant for punishing the aggressor); threat to be averted, and must not exceed
however, punitive measures may result in such limits; however in such situations it
the exercise of the defence. It is not a right cannot be expected of a person to minutely
of private ‘offence’. The right to punish calculate the correct proportion of force to
is that of State. Thus, if after sustaining a be used in defence.
serious injury there is no apprehension of (viii) The right of defence ends with the necessity
for it. Thus, the person exercising such
further danger to the body then obviously
right need not chase the fleeing attacker
the right of private defence would not be
and then beat him. Similarly, a person is
available.
not entitled to go to the house of a person
(iii) The right cannot be availed of for the sake
who beat his son in order to chastise him.
of self-gratification or to satisfy one’s ego
(ix) The law does not require that a person
or to satisfy one’s malicious or sadistic
should not exercise his right to self-
urges. The act of private defence should defence if by running away he can avoid
not be deliberate. It is available only in injury from his assailant.
case of imminent peril to those who act (x) The law does not require that a person
in good faith and in no case the right be placed in such circumstances should
conceded to a person who stage-manages weigh the arguments for and against an
a situation where in the right can be used attack ‘in golden scales’.
as a shield to justify an act of aggression. The right of private defence is not available in
(iv) The right must be exercised when there the following situations:
is: (i) The aggressors cannot claim the right to
(a) real and immediate threat (not imagi- private defence; an aggressor himself
nary and remote), and creates a danger to his own life.
(b) a reasonable apprehension of such (ii) There cannot be private defence against
threat. The term ‘reasonable’ implies private defence. There is no right of
what any common man in that situa- private defence under the Code against
tion would apprehend. any act which is not in itself an offence
(v) The apprehension must be reasonable and under it (an act done in exercise of the
not fanciful. And, present and imminent right of private defence is not an offence).
danger (not remote or distant) should be (iii) In a case when two parties are having
present. thus, X cannot shoot his enemy a free fight without disclosing as to
Y who is at a great distance, even if Y is who is the initial aggressor, it would be
armed with a sword. The reason is that, at dangerous as a rule to clothe either of
yet Y has not attacked X, and therefore, them or his sympathizer with a right of
it cannot be said that X has a present and private defence. In such a case, no right of
reasonable apprehension of being attacked private defence is available to either party
by X’s sword. and each individual is responsible for his
(vi) The right of private defence commences own acts.
as soon as a reasonable apprehension of
danger to the body arises from an attempt BURDEN OF PROOF
or threat to commit the offence, though
the offence may not have been committed, According to Section 105 of the Indian Evidence

CLAT.indb 48 31/03/2009 11:41:41


CRIMINAL LAW 49

Act, 1872, the burden of proving the exception public servant or is acting under the direction of
is on the person who benefits from it (i.e. on a public servant.
the accused). It is well settled that even if an Thus, the right of private defence of the
accused does not plead self-defence, it is open body/property can be exercised against a public
to the Court to consider such a plea if the same servant only in the following three cases:
arises from the material on record. (a) When the act of the public servant reason-
ably cause apprehension of death/grievous
PRIVATE DEFENCE OF THE BODY hurt.
(b) When the public servant does not act in
Section 97. “Every person has a right (subject to good faith under colour of his office.
restriction in Section 99) to defend his own body (c) When the person exercising the right does
or that of any other person against any offence not know or have any reason to believe that
affecting the human body”. the attacker is a public servant or is acting
Section 98. “Every person has the right of private under the direction of a public servant.
defence of the body against an act, which would Section 99 also lays down the extent to which
otherwise be a certain offence, but is not that the right may be observed. The measure of self-
offence by reason of the doer being of unsound defence must always be proportionate to the
mind, a minor, an intoxicated person or a person quantum of force used by the attacker and which
acting under misconception of fact”. it is necessary to repel. The extent of force which
This section lays down that for the pur- may be used depends upon the circumstances of
pose of exercising the right of private defence, each case. The nature of the attack, the danger
the physical or mental capacity of the person apprehended, the imminence of danger and the
against whom the right is exercised is no bar. In real necessity of inflicting harm by retaliation
other words, the right of defence of the body ex- for the purpose of self-defence, are some of the
ists against all attackers—whether with or with- important factors to be considered in deciding
out mens rea. Thus, if Z, under the influence of whether the right of defence has been exceeded.
madness, attempts to kill A, Z is guilty of no Section 100. When the right of private defence
offence. But A has the same right of private de- extends to causing death.
fence which he would have if Z were sane, The right of private defence of the body extends
Section 99 places four restrictions on the right (subject to the restrictions mentioned in Section
of private defence: 99) to the voluntary causing of death or of any
(i) There is no right of private defence against other harm to the assailant in cases of:
an act which does not reasonably cause the (i) an assault as may reasonably cause the
apprehension of death or grievous hurt, if apprehension that death will otherwise be
done or attempted to be done by a public the consequence of such assault.
servant acting in good faith under colour (ii) Similarly, in case of grievous hurt.
of his office, though that act may not be (iii) An assault with the intention of committing
strictly justifiable by law. rape.
(ii) Similarly, there is no right of private (iv) An assault with the intention of gratifying
defence, when the act is done by the unnatural lust.
direction of a public servant, though that (v) An assault with the intention of kidnapping
direction may not be strictly justifiable by or abduction.
law. (vi) An assault with the intention of wrongfully
(iii) There is no right of defence in cases in confining a person under circumstances
which there is time to have recourse to the which may reasonably cause him to
protection of public authorities. apprehend that he will be unable to have
(iv) The right of private defence in no case recourse to the public authorities for
extends to the inflicting of more harm than release.
it is necessary to inflict for the purpose of In the (i) situation if the defender be so
defence. situated that he cannot exercise the right without
Explanation 1 and 2. Section 99 provides that risk to harm to an innocent person he may even
person is not deprived of the right of defence run that risk (Section 106). Thus, if A is attacked
against an act of public servant, unless he knows by a mob which attempts to murder him and he
or has reason to believe that the attacker is a cannot effectually exercise his right of private

CLAT.indb 49 31/03/2009 11:41:41


50 CRIMINAL LAW

defence without firing on the mob, and he can- (1) Acts against which there is no right of pri-
not fire without risk of harming young children vate defence of property
who are mingled with the mob, A commits no Section 99 lays down the limitations to the right
offence if by so firing he harms the children (Il- of private defence of property:
lustration to Section 106). (i) There is no right of private defence of
Four undermentioned cardinal conditions property against an act which does not
must have existed before the taking of the life of reasonably cause the apprehension
a person is justified on the plea of self-defence: of death/grievous hurt, if done by (or
Firstly, the accused must be free from fault by the direction of) a public servant
in bringing about the encounter; acting in good faith under colour of
Secondly, there must be present an impend- his office.
ing peril to life or of great bodily harm, either (ii) There is also no right of private in
real or so apparent as to create honest belief of cases in which there is time to have
an existing necessity. resource to the protection of the
Thirdly, there must be no safe or reason- public authorities. Further, the right
able mode of escape by retreat; of private defence in no case extends
Fourthly, there must have been a necessity to the inflicting of more than it is
for taking life. necessary to inflict for the purpose of
According to Section 101, in other cases defence.
than those mentioned in Section 100, the person (2) Right when commences and how long it
exercising the right may cause any other harm continues (Section 105)
except death. The right commences when a reasonable
Section 102. Commencement and continu- apprehension of danger to the property
ance of the right of private defence. commences. The right continues
The right of private defence of the body (i) against theft, till the offender has
commences as soon as a reasonable apprehen- effected his retreat with the property,
sion of danger to the body arises from an at- or the assistance of the public
tempt or threat to commit the offence though the authorities is obtained; or the property
offence may not have been committed; and it has been recovered.
continues as long as such apprehension of dan- (ii) against robbery, as long as the offender
ger to the body continues. causes (or attempts to cause) to any
person death or hurt or wrongful
restraint, or the fear of instant death/
PRIVATE DEFENCE OF PROPERTY
hurt/personal restraint continues.
Acts against which right of defence of property (iii) against criminal trespass or mischief,
can be exercised. Every person has the right so long as the offender continues in
to defend the property (whether movable or the commission of such offence
immovable) of himself or of any other person (iv) against house-breaking by night, as
(i) against theft, robbery, mischief or criminal long as the house-trespass continues
trespass, or any act which is an attempt to (thus, where a person followed a
thief and killed him in the open, after
commit theft, robbery, etc. (Section 97);
house-trespass has ceased, held that
and
he could not plead the right of private
(ii) against the act of a lunatic, a minor, or an
defence)
intoxicated person or a person acting un-
Thus, the right of private defence of the
der a misconception of fact (Section 98).
property commences and continues as long as
Illustration the danger lasts. The extent to which the exercise
A enters by night a house, which he is legally of the right will be justified will depend, not on
entitled to enter. Z, in good faith, taking A for a the actual danger, but on whether there was
reasonable apprehension of such danger.
house-breaker, attacks A. Here Z, by attacking A
(3) When the right extends to the causing of
under this misconception, commits no offence.
death, etc.
But A has the same right of private defence
Section 103 enumerates the cases in which
against Z, which he would have if Z were not
the right extends to justifiably causing the
acting under this misconception.
death of the wrong-doer, viz.,

CLAT.indb 50 31/03/2009 11:41:41


CRIMINAL LAW 51

(i) Robbery, nature of the act, or that he is doing what is ei-


(ii) House-breaking by night, ther wrong, or contrary to law: provided that the
(iii) Mischief by fire to any building, tent, thing which intoxicated him was administered
or vessel used as human dwelling or without his knowledge or against his will”.
as a place for the custody of property, Section 86: “In cases where an act done is not
(iv) Theft, mischief, or house-trespass un- an offence unless done with a particular knowl-
der such circumstances as may reason- edge or intent, a person, who does the act in a
ably cause an apprehension that death/ state of intoxication shall be liable to be dealth
grievous hurt will be the consequence, with the act as if he had the same knowledge as
if such right of private defence is not he would have had if he had not been intoxicat-
exercised. ed, unless the thing which intoxicated him was
(4) Under Section 97, every person has the administered to him without his knowledge or
right to defend the property of himself or against his will”.
of any other person against theft. Thus, the It will, therefore, be seen that for an of-
tenant, L, was within his rights to exercise fence committed in consequence of voluntary
the right of private defence. But as J was drunkenness, there is no excuse in law. If it were
committing a theft without reasonably otherwise any murderer would first get himself
causing an apprehension that death/ thoroughly intoxicated, have enough sense to
grievous hurt will be the consequence commit the murder, and then plead intoxication
(Secs. 103, 104), L had no right to cause as a complete defence. However, if a person, by
the death of J. mistake, takes some wrong medicine and get in-
According to Section 105, the right of pri- toxicated, and then commits an offence, surely
vate defence of property against theft continues he cannot be made liable. Similarly, if a fraud is
till the offender has effected his retreat with the practiced on him whereby he is made to take an
property Further, under Section 99, there is no intoxicant, and then he commits an offence, he
right to the protection of the public authorities. cannot be held liable. Such cases are not cases of
When J entered his house, there is retreat, thus L voluntary drunkenness.
cannot exercise right of defence here. Further, as Test of drunkenness—The test to apply in cases
J’s house was known, there was sufficient time of drunkenness is not the test applied in cases
to inform the police. Thus, L’s right of defence of insanity viz., whether the accused person
also ceases here. knew what he was doing was wrong or was able
to appreciate the nature and quality of his act.
INTOXICATION However, insanity produced by drunkenness is a
defence (under Section 84).
Whereas lunacy is a disease and is therefore to The correct test is whether by reason of
be pitted, drunkenness a vice and is, therefore, drunkenness, the accused was incapable of
to be condemned. Drunkenness is a species of forming an intention of committing the offence.
madness for which the man is to blame. A man is taken to intend the natural consequenc-
The law pronounces that the obscuration es of his acts. This presumption may be rebutted
and divestment of that judgement and human in the case of drunken man, by showing that he
feeling which in a sober state would have pre- did not know what he was doing was dangerous
vented the accused from offending, shall not, or incapable of forming the specific intent es-
when produced by his voluntary act, screen him sential to constitute the crime.
from punishment, although he may no longer Section 86 says that a person voluntarily
capable of self-restraint. intoxicated will be deemed to have the same
Qui pecat ebrius luat sobrius: Let him knowledge as he would have had if he had not
who sins when drunk be punished when sober. If been intoxicated. The section does not say that
a man chooses to get drunk, it is his own volun- the accused shall be liable to be dealt with as if
tary act; it is very different from madness which he had the same intention as might have been
not caused by any act of the person. presumed if he had not been intoxicated. There-
Now, so far as intoxicated persons are con- fore, there is no presumption, under Section 86,
cerned, Sections 85 and 86 are relevant. with regard to intention. In such cases, his inten-
Section 85: “Nothing is an offence which is tion would have to be gathered from the facts
done by a person who, at the time of doing it, by and circumstances of every individual case, hav-
reason of intoxication, incapable of knowing the ing due regard to the degree of intoxication.

CLAT.indb 51 31/03/2009 11:41:41


52 CRIMINAL LAW

MULTIPLE-CHOICE QUESTIONS
CULPABLE HOMICIDE AND Issue: The offence committed is murder
MURDER or culpable homicide.
(Sections 299 and 300) Decision:
(a) The offence is murder as the fact of
the case shows that the infliction of
1. Facts: One Bherun and his father were 19 injuries are sufficient the cause of
on inimical terms with the appellants. On death in the ordinary cause of nature.
29 June 1961, Behrun was going towards (b) It is culpable homicide. because the
his fields when he was caught hold of by intension of the accused is missing.
Anda and Ropa (appellants). They dragged (c) It is neither murder nor culpable ho-
him inside a house and beat him severally.
micide.
His arms and legs were smashed and many
(d) It is a case of simple fight.
bruises and lacerated wounds were caused
3. Facts: In a case there was a long standing
on his person. The doctor who conducted
dispute between the deceased and the ap-
the autopsy opined that injuries and that
pellants over a certain piece of land. On
all the injuries collectively were sufficient
one night, the appellants (N and R) in pur-
in the ordinary course of nature to cause
suance of a conspiracy to commit murder
death.
of M, deceased, set fire to the single room
Issue: Are the appellants guilty of murder
hut in which he was sleeping after locking
or culpable homicide.
Decision: the door from outside. When the servants
(a) They are not guilty because the doctor of the deceased and other villagers tried
who conducted autopsy said that the to rescue the deceased they were kept at
deceased diet of shock. bay by the superior force of the accused
(b) They are guilty of culpable homicide. and their associates. Consequently, death
Because there is no indication of use of the deceased occurred.
of lethal weapon. Issue: Murder or culpable homicide.
(c) They are not guilty of murder because Decision:
intension is missing. (a) It is not a murder. Because the facts
(d) It’s a clear cut case of murder. Be- and circumstances are not sufficient.
cause severely beating the deceased (b) It is not culpable homicide too be-
and smashing his legs is clear cut in- cause it is nowhere evident that the
dication of the presence of intension. very action of the accused to set the
2. Facts: In a case, there was bad blood be- house of the deceased on fire, resulted
tween the two factions and they were in- in to his death.
volved in number of incidents and crimi- (c) It is murder because in the pursuance
nal proceedings. On one day, deceased of the conspiracy to commit murder
(belonging to one faction) was followed of M, the accused lock the deceased
by members of other faction. When the and set the room on fire and stop the
deceased, a 55 year old man, tried to ran servants and villagers to come to his
away, he was not allowed by the accused rescue.
who indiscriminately pounded the legs and (d) It will depend on the discretion of the
arms of his by heavy sticks and continued court.
the beating till he became unconscious. 4. Facts: The accused with intention of kill-
The accused than left the spot. The doctor ing A, gave him some poisoned halva. A
found as many as 19 injuries out of which ate a portion of it and threw the rest away
no less than 9 were (internally) found to and this was picked up by accused’s broth-
be grievous. The doctor gave the opinion er in law’s daughter (a girl of 8 years) who
that the injuries were cumulatively suffi- ate it and also gave some to another child.
cient to cause death in the ordinary course The two child died, but A eventually re-
of nature. covered. It was found as a fact that two

CLAT.indb 52 31/03/2009 11:41:41


CRIMINAL LAW 53

child ate halva without the knowledge of inflicted is sufficient to cause death in
the accused, who did not intend to cause the ordinary cause of nature.
their death. The accused wanted to kill A (b) It is culpable homicide.
on whose life he had effected large insur- (c) It is grievous hurt.
ance. 7. Facts: In this case, the question was when
Issue: Whether the accused committed of- death is caused by a single blow, whether
fence of murder or culpable homicide. clause three of Section 300 is attracted.
Decision: In the case in question, the accused per-
(a) The accused has committed neither sons came armed with deadly weapons
the murder nor culpable homicide. and there was an altercation and exchange
Because the facts of the case show of hot words whereafter the accused as-
that he has no relation with the chil- saulted victim with a bhala causing injury
dren who died of consuming poison- on the chest rupturing major blood vessels
ous halva. resulting in her instantaneous death.
(b) It is murder because he had intended Issue: The accused are guilty of murder or
to kill A. Though A, couldn’t be killed culpable homicide.
but the indirect death of children Decision:
amounts to killing with intention. (a) It is not murder because a single blow
(c) No clear cut offence because the facts can’t be sufficient to cause death.
and circumstances do not give clear (b) It is culpable homicide.
picture. (c) It is grievous hurt.
5. Facts: About a year before the date of oc- (d) It is murder because a single blow can
currence, Bachhan Singh, son of the de- be the cause of death if it is inflicted
ceased, had caused a severe injury on the with as much force and severity which
leg of one Pritam Singh resulting in the is sufficient to cause death in the ordi-
amputation of leg of one appellant. Fa- nary course of nature.
ther of Pritam Singh harboured a grudge 8. Facts: Dhirajja was a young woman mar-
against Bachhan Singh and his father. ried to Jhagga. They had a six months old
On one day, the appellant and his associ- baby. The husband did not treat his wife
ate caught hold of the deceased (father of well. The wife desired to go to visit her
parents but the husband had objected to
Bachhan Singh) and inflicted as many as
it and on the day of occurrence they had
18 injuries on the arms and legs of the de-
quarrelled with each other and the hus-
ceased with a gandasa.
band had threatened to beat her. Late that
Issue: The accused are guilty of murder or
night Jagga woke up and found his wife
culpable homicide.
and baby missing. He went out in pursuit
Decision:
of them and close to the railway line he
(a) It is culpable homicide not amounting
saw her making her way along the path.
to murder.
When she heard him coming after her,
(b) It is murder.
Dhirajja turned around in a panic, ran a
(c) It is neither murder nor culpable ho-
little distance, and then jumped into a well
micide.
with the baby girl in her arms. The result
(d) It is a case of grievous hurt. was that the baby died while the woman
6. Facts: The appellant Virsa Singh thrust a was eventually rescued. She was charged
spear in the abdomen of the deceased with with the murder of her baby and with at-
such force that it penetrated the bowels tempt to commit suicide.
and three coils of the intestines came out Issue: The wife is guilty of murder or cul-
of the wound. The doctor who conducted pable homicide.
the post mortem said that the injury was Decision:
sufficient to cause death in the ordinary (a) The wife is guilty neither of murder or
course of nature. culpable homicide.
Issue: The accused are guilty of murder or (b) It is case of self-defense so she can’t
culpable homicide. be charged with any offence.
Decision: (c) She is guilty of culpable homicide not
(a) It is a clear cut murder as the injury amounting to murder.

CLAT.indb 53 31/03/2009 11:41:41


54 CRIMINAL LAW

9. Facts: The appellant, her husband and her 12. Facts: A pulled B out of his classroom,
sister in law used to reside together. There gave blows on his ribs fracturing two of
were constant quarrels between the appel- them and kicked him causing rupture of
lant and her sister-in-law and very often B’s enlarged spleen. A was not aware of
the appellant was slapped by her husband this physical infirmity of B. These injuries
for such quarrelling. One day, a quarrel cumulatively resulted in B’s death.
took place and her sister-in-law asked her Issue: What offence, if any, A has com-
to leave the house. Thereupon, the appel- mitted?
lant left the house and jumped into a well Decision:
along with her three children, in order (a) A has committed offence of his mur-
to escape harassment at the hands of her der as the severe blows on the ribs
sister-in-law. She survived but the three causing fracture and rupture of spleen
children died. are sufficient to cause the death in or-
Issue: Whether the appellant is guilty of dinary course of nature.
murder or culpable homicide? (b) It is culpable homicide.
Decision: (c) It is neither murder nor culpable ho-
(a) She should be charged with offence of micide.
murder. (d) The facts of the case show that it is a
(b) She is guilty of culpable homicide. matter of grievous hurt.
(c) It is neither murder nor culpable ho- 13. Facts: X gave a lathi blow on Y’s head
micide. with such a force that his head was badly
10. Facts: A, a pickpocket, puts his hand in fractured. Y became unconscious and was
B’s pocket. Incidentally B had a pistol in rushed to the hospital where he died after
his pocket and as A put his hand in the 12 days. The postmortem report showed
pocket, it touched the trigger and the pistol that Y had died because of septic infec-
went off killing B. tion. When X is charged with murder, he
Issue: The accused is guilty of murder or pleads that the death was caused because
culpable homicide? of septic infection and not because of head
Decision: injury. Will he succeed in his plea? Dis-
(a) A is guilty of murder who triggered cuss.
the pistol and killed B. Issue: Whether X is guilty of culpable ho-
(b) A is guilty of culpable homicide not micide or murder?
amounting to murder as he didn’t in- Decision:
tend to kill B. (a) He is guilty of murder as a blow of
(c) It is neither murder nor culpable ho- lathi on the head is sufficient to prove
micide. A is guilty of pick pocketing. the intension of the accused and the
11. Facts: A, a school teacher, had given a cause of death.
few slaps to B, a student, with a view to (b) He is not guilty of murder. As post
disciplining him. B, who was a patient of mortem report shows he died of septi
epilepsy fell down and became uncon- infection.
scious. A believing B to be dead, in a panic (c) He is guilty of culpable homicide.
hung him to give an impression of suicidal (d) He is neither guilty of culpable homi-
death. Does A possess the required guilty cide nor murder. He is guilty of griev-
mind for culpable homicide? ous hurt.
Issue: Does a posses the required guilty 14. Facts: A intentionally attacks B. While
mind of culpable homicide? doing so, A does not know that B is suffer-
Decision: ing from an enlarged appendix. The blow
(a) A does not possess the required guilty falls on the appendix as a result of which
mind for culpable homicide unless it the appendix bursts and B dies. Discuss
is shown that he was aware that B was A’s liability.
an epileptic patient. Issue: What is the liability of A?
(b) A must be charged with murder. Decision:
(c) A must be charged with conspiracy of (a) A intentionally attacks B to kill him.
murder. Though, the blow falls on the appen-

CLAT.indb 54 31/03/2009 11:41:41


CRIMINAL LAW 55

dix which burst. But it is not a case of 18. Facts: A, B, C and D caught E red handed
accident. A is liable for murder. while destroying their crops at night. The
(b) As the blow accidentally falls on ap- four of them tied E to a tree and began
pendix which burst resulting in to pounding him with kicks and blows. In
death. So A is not liable for murder. order to compel him to speak out (E de-
(c) A is liable for culpable homicide. nied his hand in previous destruction of
(d) A is liable for simple hurt. crops), A thrust a burning bamboo into his
15. Facts: A intentionally gives B a sword cut mouth causing extensive injuries. When E
sufficient to cause the death of a man in was later taken to the hospital he died the
the ordinary course of nature. B dies in next day. The doctor testified that the in-
consequence. Refer to relevant legal pro- juries were cumulatively sufficient in the
visions and the decided case. ordinary course of nature to cause death.
Issue: What offence, if any has been com- A is prosecuted for the offence of culpable
mitted by A? homicide/murder. Decide.
Decision: Issue: What is liability of A, B, C and D?
(a) A has committed the offence of mur- Decision:
der. (a) A, B, C and D liable for murder.
(b) A has committed the offence of cul- (b) Only A is liable for murder.
pable homicide. (c) A is liable for murder and B, C and D
(c) A has committed neither culpable ho- are liable for attempt to murder.
micide nor murder. (d) It is not easy to fix the liability.
16. Facts: A without any excuse fires a loaded 19. Facts: A had some political rivalry with
pistol into a crowd of persons and kills one B. Finding B alone one day, A with his
of them. Refer to relevant legal provisions friends beat him with lathis. B suffered
and the decided case. multiple fractures and died due to internal
Issue: What offence, if any, has been com- bleeding. On being prosecuted for murder,
mitted? A and his friends argued that they had in-
Decision: tended not to kill but only to cause him
(a) A has not committed any offence. hurt. Decide.
(b) A has committed the offence of mur- Issue: What offence, if any has been com-
der. mitted by A and his friends?
(c) It is a case of accident. Decision:
17. Facts: A refused to marry B due to some (a) A and his friends are liable for mur-
social reasons. B became angry and threat- der.
ened A with lingering death. On the fateful (b) They are liable for hurt.
day when A was a sleeping along with her (c) They are liable for culpable homicide.
infant son and mother at a rest house on 20. Facts: X is cultivating possession of ag-
their way to a pilgrimage, B threw a bottle ricultural land belonging to Z. A suit is
of acid on them. A got 60 per cent burn in- pending before the court for recovery of
juries and died 12 days after the incident. the said land. X along with his close ally
The medical report established death due Y intending to kill Z to keep the disputed
to acid burns. B has been charged of mur- land in possession, plan and set fire to his
der under Section 300 (3). Discuss about thatched house by locking it from outside
the success of prosecution in this case. just before midnight and by preventing
Issue: Is B liable for murder? others from coming just before midnight
Decision: and by preventing others from coming to
(a) B is not liable for murder. Because his rescue in response to his “SOS” calls
reasonable man of ordinary prudence amidst the rising fire. Z is roasted alive.
can not think of that throwing acid What offence X and Y are guilty of? Will
one’s face can result in to death. they be awarded the same penalty as in a
(b) It is a clear cut case of murder. As B similar case taught to you?
had already threatened A of lingering Issue: What offence, if any has been com-
death. mitted by X and Y?
(c) It is neither murder nor culpable ho- Decision:
micide. (a) X and Y are liable for murder because

CLAT.indb 55 31/03/2009 11:41:42


56 CRIMINAL LAW

setting some one on fire and not let- having estranged relations with each other
ting any one to come to his rescue is and W was living with her parents. H tried
sufficient to cause death. to bring back W many times but she al-
(b) The facts are not sufficient to hold X ways refused. One day when he went to
and Y liable for murder. the father-in-law house, he found that W
(c) They can be held liable for culpable was alone in the house. H tried to make
homicide. advances towards W. But W did not yield
21. Facts: A, intending to kill his wife W, to H’s advances and told him that she
gave her a poisoned chocolate which she will shout and collect the village people.
left on the table and herself went to attend There-upon H found an iron rod nearby
a telephone call. Meanwhile, their child C and gave a blow on her head. W swooned
came in, picked up, and ate the chocolate. and became unconscious. H thought that
A, who was standing nearby, got quite up- W is dead. In order to fabricate the evi-
set by the sudden turn of events but kept dence H hanged the body with a rope on
quiet due to fear of being exposed of his the ceiling fan. The post mortem report
evil designs to kill W. C died of poisoning established that the cause of the death was
and A is being tried for C’s murder. How strangulation and not the head’s injury.
will you argue the case for prosecution? What offence, if any, has H committed?
Issue: What offence, if any has been com- Issue: What offence, if any has been com-
mitted by A? mitted by H?
Decision: Decision:
(a) A can not be charged with the offence (a) He is guilty of murder.
of murder because he didn’t intend to (b) He is guilty of culpable homicide.
kill his child C. (c) He is neither guilty of culpable homi-
(b) A can be charged with offence of mur- cide nor murder.
der because he had intended to kill his (d) He is guilty of offence of grievous
wife and he administered her a poison- hurt and attempt to create false evi-
ous chocolate, which was consumed dence by hanging his wife.
by the child. A stood near by when 24. Facts: A saw X lying motionless on the
the child consumed that chocolate. bedroom floor of his wife B’s 10th floor
The mere silence of A, is sufficient to apartment. Believing X to be dead and
determine the culpability of A. fearing his wife’s involvement in the mur-
(c) It’s a case of accident. der. A pushed X out of the window of the
22. Facts: Mallika was abducted along with room to give an impression of suicide. X
her 6 months old son and kept locked in died due to multiple fractures and shock
a room. She somehow managed to escape on account of the fall from the 10th floor.
with her son but soon found her abductors Issue: What offence, if any, has A com-
pursuing her. In a state of panic she ran mitted?
some distance and then jumped into a well Decision:
some distance away from the path. She (a) He has committed murder.
was later rescued but the child died. Mal- (b) He has committed culpable homicide
lika is being prosecuted for murder. Give not amounting to murder.
arguments on behalf of either the prosecu- (c) He has neither committed murder nor
tion or defence. culpable homicide.
Issue: What offence, if any has been com- (d) A has not committed culpable homi-
mitted. cide in terms of Section 299, as his
Decision: intention was directed to what he be-
(a) Mallika can be charged with offence lieves to be a lifeless body. A is also
of culpable homicide not amounting not guilty of grievous hurt as he had
to murder. not attacked X, when X was alive. He
(b) Mallika can be charged with murder. attacked X only when he believed X
(c) Mallika can neither be charged with to be dead. However, A is guilty of an
murder nor culpable homicide. attempt to create false evidence.
23. Facts: H(husband) and W(wife) were 25. Facts: A was friendly with many male

CLAT.indb 56 31/03/2009 11:41:42


CRIMINAL LAW 57

and female children in the neighbourhood. Decision:


One day A took a 5 years old girl on the (a) It is a case of culpable homicide not
riverbank on the pretext of giving her a amounting to murder.
boat ride. On reaching a lonely spot, A (b) It is a case of murder.
raped the girl violently and to silence her (c) It is a case of molestation.
A stuffed her mouth with sand. The girl (d) It is a case of neither of murder nor
was found dead next morning. The post culpable homicide nor molestation
mortem report revealed that death was due because the woman has taken the lift
to asphyxia caused by the sand blocking on her own and she is responsible for
the respiratory organs. The doctor testified all that happened.
that the injury of filling sand in the mouth 28. Facts: The accused A initially abused the
was sufficient in the ordinary course of na- deceased D by holding his collar and bran-
ture to cause death. dishing the knife. However, D brought a
Issue: What offence, if any has been com- stout and stronger man who overpowered
mitted A? A and sat on his chest. The coaccused B
Decision: at a the call of A inflicted a knife blow on
(a) He is guilty of offence of rape. D’s thigh. When D had to release the grip,
(b) He is guilty of offence of rape but nor A without any provocation dealt a very se-
murder. vere knife blow on the stomach of D who
(c) He is guilty of offence of rape and was unarmed. The knife blow caused sev-
culpable homicide not amounting to eral injuries on the vital parts of the body
murder. of D resulting in his death in a very short
(d) He is guilty of rape and murder. time.
26. Facts: A ravished his private secretary in Issue: Whether it is a case of culpable ho-
a state of unconsciousness by administer- micide only or a case of murder?
ing a strong dose of chloroform. She failed Decision:
to regain her consciousness and died. The (a) It’s a case of murder as A had an in-
doctor’s opinion was that mere adminis- tention to cause murderours assault on
tration of chloroform was not sufficient D.
to cause death in the ordinary case, but (b) It is not a case of murder as D was
stout and stronger and had overpow-
the shock of forcible ravishment on an
ered A. So, A killed him in right of
unconscious person may at times lead to
private defence.
irreversible coma. Can A be successfully
(c) It is merely a case of accident.
prosecuted for the murder?
29. Facts: One day B made indecent remarks
Issue: What offence, if any has been com-
about A’s sister. Next day, when B was go-
mitted by A?
ing to his office, A who was hiding on the
Decision:
route, gave two blows on B’s head with a
(a) He is guilty of murder.
hockey stick. Consequently, B fell down
(b) It is neither murder nor culpable ho-
unconscious and was taken to hospital for
micide.
treatment but he died the same evening.
(c) It is culpable homicide as the bodily According to medical evidence the inju-
injury cause in death was neither in- ries caused to B were sufficient to cause
tended nor known. death in the ordinary course of nature. A is
27. Facts: The accused tried to molest an in- being tried for B’s murder. How will you
nocent woman who took lift in his vehicle. decide?
She tried to come out of the moving ve- Issue: What offence, if any has been com-
hicle. The accused speeded up the vehicle. mitted?
She eventually came out and was run over Decision:
by a truck. It was contended on behalf of (a) A has committed murder because he
the accused that there was no intention of was standing there with full prepara-
the accused to kill the woman. It happened tion, guilty mind and the action taken
accidentally. How will you decide? by him, are sufficient to cause the
Issue: What offence, if any has been com- death of person in ordinary course of
mitted? nature.

CLAT.indb 57 31/03/2009 11:41:42


58 CRIMINAL LAW

(b) He is guilty of culpable homicide not and severe damage to the brain. B was
amounting to murder. taken to the hospital where two operations
(c) He is guilty of causing grievous hurt. were performed to remove blood clots. B
(d) He is guilty of causing simple hurt. died in the hospital a week after the inci-
30. Facts: Accused A-1, A-2 and A-3 came dent. The doctor certified that the injury
armed with knife, lathi and pistol to attack was sufficient in ordinary course of nature
the deceased D. D was lonesome. The ac- to cause death. For what offence can A be
cused severely assaulted D by kicks and prosecuted? Give reasons.
fist blows. The death of D occurred due Issue: What offence, if any has been com-
to shock, hemorrhage and strangulation mitted.
caused by such blows. What offence the Decision:
accused are guilty of? Decide. (a) The accused is not guilty of murder.
Issue: What offence, if any has been com- (b) He is guilty of causing grievous hurt.
mitted? (c) A is guilty of the offence of murder.
Decision: As per clause thirdly of Section 300,
(a) The accused are guilty of murder. if the injury that the offender intends
(b) It is case of culpable homicide not causing and does cause is sufficient to
amounting to murder. The accused if cause death in the ordinary course of
intended to murder D could have eas- nature, the offence is murder, whether
ily done so by their weapons, but they the offender intended causing death
must be attributed with knowledge or not and whether the offender had
that by their acts they were likely to a subjective knowledge of the conse-
cause death. quences or not.
(c) They are guilty of causing grievous 33. Facts: A quarrel took place between the
hurt. accused persons and the deceased regard-
31. Facts: Accused A-1 and A-2 actuated by ing purchase of cinema tickets. The ac-
group malice, beat an old man D (the de- cused who was already armed with a knife
ceased) to death by causing nine grievous inflicted fatal injury (in the chest region,
hurts in the legs and arms which included cutting the aorta of heart) on the helpless
fractures in the right thumb, right femur, deceased caught by the other accused. The
right tibia, right fibula, left tibia, right pa- doctor certified that the injury by itself was
tella and dislocation of bones and joints sufficient in the ordinary course of nature
in the left middle finger and right and left to cause death. For what offence can the
little fingers. Death occurred due to shock accused be prosecuted?
and hemorrhage resulting from multiple Issue: What offence, if any has been com-
injuries. What offence the two accused mitted?
can be convicted of? Discuss the statutory Decision:
provision and case law on the point. (a) It is clear-cut offence of murder.
Issue: What offence, if any has been com- (b) It is a case of culpable homicide not
mitted? amounting to murder.
Decision: (c) It is a case of causing grievous hurt.
(a) They have committed offence of mur- 34. Facts: A and B belonged to the rival fac-
der. tions. In order to establish the superiority
(b) They have committed culpable homi- of his faction A decided to give B sound
cide not amounting to murder. thrashing in the presence of other students.
(c) They have committed grievous hurt. On a crowded day in the college A cor-
32. Facts: A who was speeding past a police nered B alone, and struck blows with bare
check post in his car after committing a fists and kicked him several times. On B’s
robbery, was required to stop by B, a po- falling down on the ground A mounted on
lice officer. While B was interrogating A, his chest and struck three violent blows on
A suddenly gave a spanner blow on B’s the face and head region. The blows on
head mainly with a view to making good the head region caused extravasations of
his escape. The spanner blow, given with blood in the brain, and as a consequence,
considerable force, led to skull fracture B died the next day in the hospital. A is

CLAT.indb 58 31/03/2009 11:41:42


CRIMINAL LAW 59

prosecuted for murder. Decide giving ar- committed? Would it make any difference
guments and case law. if A died due to failure to give prompt
Issue: What offence, if any has been com- medical aid or due to head injury after 10
mitted? days in the hospital?
Decision: Issue: What offence, if any has been com-
(a) It is a clear cut case of murder. mitted?
(b) It is culpable homicide not amounting Decision:
to murder, because the accused didn’t (a) Accused is not guilty of murder.
intended to kill him. (b) Accused is guilty of culpable homi-
(c) It is a case of grievous hurt. cide.
35. Facts: P, a dreaded terrorist, driving his (c) The accused is guilty of murder with
Mercedes, was apprehended by a police in the meaning of clause (3) of Sec-
constable on a traffic red signal. The con- tion 300, so far as the injury was in-
stable asked P to take the car to his left tentional and it was sufficient in the
towards a petrol pump and to stop there ordinary course of nature to cause
in the parking space. P began to do so but death. In the light of Explanation (2)
suddenly accelerated. The constable man- to Section 299 it would make no dif-
aged to cling to the driver’s window, P ference in the answer if A died due to
tried to shake him off by giving hard blows failure to give prompt medical aid.
with the help of his fist but could not suc- 37. Facts: C, intending to kill his wife D,
ceed in throwing him off on the road. P injects poison in a banana and gives it to
then took a spanner lying in his car and her to eat. D keeps the poisoned banana
struck a violent blow on the constable fin- on a table to relish it at her leisure. Im-
mediately after that, L, a neighbour comes
gers, and as constable loosened his grip, P
to their house: D offers the banana to him.
pushed and threw him on the road. There
L consumes it and dies consequently. The
was very heavy traffic on the road and the
incident occurs in C’s presence and he
constable got killed by a speeding truck.
does not interfere. What are the respective
Can P be successfully prosecuted for the
penal liabilities of C and D? Will your an-
murder of the constable?
swers be different, if D offers the banana
Issue: What offence, if any has been com-
to L in C’s absence?
mitted?
Issue: What offence, if any has been com-
Decision:
mitted?
(a) P is guilty of murder. Decision:
(b) P is guilty of culpable homicide. (a) C is guilty of culpable homicide.
(c) P is not guilty of homicide by rash or (b) D is guilty of murder.
negligent act because Section 304-A (c) C is guilty of murder because C had
does not apply where the death has the intention to kill someone and his
arisen from any other supervening act action was sufficient to cause the
or interventions which could not have death. D is not guilty of any offence.
been anticipated. P is, however, guilty C’s presence or absence is immate-
of causing ‘grievous hurt’ within the rial.
meaning of Section 320, as he had the 38. Facts: Examine as to whether a person
intention of causing such hurt. who causes death of a pregnant woman
36. Facts: A, a physical training instructor is guilty of committing one homicide or
was acting as a referee in a friendly hockey two.
match being played between class XI and Issue: What offence, if any has been com-
class XII. X, an ex-student of the school, mitted?
arrived on the ground and was very anx- Decision:
ious to play the game but was not allowed (a) A person who causes the death of a
by A. X snatched a hockey stick from a pregnant woman is guilty of commit-
player and struck a blow on A’s head with ting two homicides.
considerable force. A fell down and died (b) A person who causes death of a preg-
instantaneously. What offence has been nant woman is guilty of committing

CLAT.indb 59 31/03/2009 11:41:42


60 CRIMINAL LAW

only one homicide, that is, of the of the axe. As a result of the head injury, B
woman. As per Explanation 3 to Sec- died. What is the liability of A?
tion 299 the causing of the death of a Issue: What offence, if any has been com-
child in the mother’s womb is not ho- mitted?
micide. But it may amount to culpable Decision:
homicide to cause the death of a living (a) A is guilty of culpable homicide not
child, if any part of that child has been amounting to murder because injury
brought forth, though the child may inflicted was not sufficient in the ordi-
not have breathed or been completely nary course of nature to cause death.
born. (b) A is guilty of murder.
39. Facts: A kicked his daughter aged about (c) A is guilty of causing grievous hurt.
9 years on her back with his barefoot. She 42. Facts: S, a snake charmer, was exhibiting
fell down and died immediately? What is a poisonous snake in public whose fangs
the liability of A. he knew had not been extracted. He put
Issue: What offence, if any has been com- the snake on the head of one of the spec-
mitted? tators, who while trying to push off the
Decision: snake away from his head was bitten by
(a) He is guilty of murder. it.
(b) He is guilty of causing grievous hurt. Issue: What offence, if any has been com-
(c) A is guilty of culpable homicide not mitted?
amounting to murder because to kick Decision:
a girl of tender age with such force as (a) S is guilty of culpable not amounting
to produce rupture of abdomen in a to murder.
(b) He is guilty of murder.
healthy subject, is an act of such char-
(c) It’s a case of accident.
acter that no reasonable man could be
43. Facts: A placed a bomb in a medical store
ignorant of the likelihood of its caus-
and gave the people inside three minutes
ing death.
to get out before the bomb exploded. B, an
40. Facts: A had gone to his father in laws
arthritic patient, failed to escape and was
house to fetch his wife. On account of
killed. Discuss the liability of A.
some quarrel between A and his brother
Issue: What offence, if any has been com-
in law, B, A lost his temper and gave one
mitted?
blow with a knife on B’s chest which re-
Decision:
sulted in his death. What is the liability of (a) The act of A was imminently danger-
A? ous act. Therefore, A will be liable for
Issue: What offence, if any has been com- committing murder of B under clause
mitted? 4 of Section 300.
Decision: (b) He is guilty of culpable homicide.
(a) A is guilty of murder. (c) He is guilty of causing grievous hurt.
(b) A is guilty of grievous hurt. 44. Facts: D, a burglar, breaks into a house
(c) A is guilty of culpable homicide not carrying an unloaded pistol which he in-
amounting to murder because death tends to use to frighten the inmates of the
was caused by doing an act with the house should he be detected. The owner of
knowledge that it was likely to cause the house confronts the burglar who there-
death. upon points the empty gun at him. The
41. Facts: A along with others while cutting owner dies of fright. Discuss the liability
illegally a forest tree were apprehended of D.
by a forest guard, B. On being rebuked Issue: What offence, if any has been com-
and threatened by A and others, B kept mitted?
quiet and tried to retreat. But A and others Decision:
chased B with axe in their hands. A gave (a) D would be liable for culpable homi-
a blow with the handle of his axe on B’s cide not amounting to murder.
back who fell down. A then assaulted B on (b) He will be liable for murder.
his legs and knee joints with the blunt side (c) It is a case of accident.

CLAT.indb 60 31/03/2009 11:41:42


CRIMINAL LAW 61

EXCEPTIONS TO THE OFFENCE to benefit of Exception 1 to Section 300?


OF MURDER Decision:
(a) He is guilty of murder as he can’t be
45. Facts: The accused K. M. Nanavati was benefitted with the exception of grave
an officer. In the year 1956, the Nanava- and sudden provocation.
tis were introduced to Ahuja. Gradually, (b) He is not guilty of murder as his situa-
a friendship developed between Ahuja tion is covered under the exception of
and Mrs Nanavati which culminated in grave and sudden provocation.
criminal intimacy between them. One day (c) He is guilty of culpable homicide.
Mrs Nanavati confessed to Mr Nanavati of (d) He is guilty of causing grievous hurt.
her illicit intimacy with Ahuja. Enraged at 47. Facts: The accused armed themselves
the conduct of Ahuja, the appellant wanted with deadly weapons and secured them-
to settle the matter with Ahuja. Thereafter, selves on the terrace of a house, collected
he drove his wife and two children to a the opponents by beat of drum and threw
cinema and left them there, and then went out a challenge of fighting. Thereupon
to his ship and took a revolver. He then the deceased used provocative language
went to the flat of Ahuja and entered his challenging the accused to come out of
bedroom. He called Ahuja a filthy swine the house and fight. The accused did and
and asked him whether he would marry killed the deceased.
his wife and look after the children. The Issue: Whether the accused are guilty of
deceased retorted, “Am I to marry every murder?
woman I sleep with”? The accused be- Decision:
came enraged and shot Ahuja dead. (a) The accused are guilty of murder.
Issue: Whether the accused is guilty of (b) The accused are guilty of culpable ho-
murder? micide.
Decision: (c) The accused are guilty of causing
(a) He is guilty of murder as all facts and grievous hurt.
circumstances are against him. 48. Facts: The accused and the deceased were
(b) No, he is not guilty of murder. Be- relatives. On the day of the occurrence of
cause his action is covered under the crime, the deceased attended the marriage
exception to murder that is grave and of his late brothers daughter. The accused
sudden provocation. had a grievance that the deceased did not
(c) He is guilty of culpable homicide. invite him to the marriage. After the mar-
46. Facts: The deceased Sabir and the ac- riage, armed with a gandhala, the accused
cused Akhtar were two brothers who lived and his brothers emerged suddenly. There
in two parts of the same house while their was a sudden quarrel and the accused as-
parents and other brothers lived at some saulted the deceased with the blunt side of
distance in the same street. Sabir and his the gandhala on the head in the heat of the
wife Chhoti were people of bad temper moment. The doctor opined that the head
and quarrelsome nature. One day, Chhoti injury was sufficient in the ordinary course
was quarreling with other members of the of nature to cause death.
family who had come to visit Akhtar. In the Issue: Whether the accused are guilty of
meanwhile Sabir came home and abused murder?
every one there. He used highly objection- Decision:
able language against his own father as (a) The accused are guilty of murder.
well as against Akhtar, in the presence of (b) The accused are guilty of culpable ho-
his two other brothers, mother and other micide as the present case is covered
neighrbours. Akhtar was provoked by by exception 4 to Section 300. Be-
such vulgar language but still kept quiet. cause the accused acted in the heat of
Sabir, to teach Akhtar a lesson, grappled moment and didn’t strike the diseased
him and in the fight Akhtar struck Sabir with sharp edged side.
twice with his knife. (c) They are guilty of causing grievous
Issue: Whether Akhtar acted under grave hurt because they assaulted with blunt
and sudden provocation and thus entitled side.

CLAT.indb 61 31/03/2009 11:41:42


62 CRIMINAL LAW

49. Facts: Sheela informed her friend Roma friend) and her plans of initiating divorce
that her husband Ramu was staying in a proceedings and marriage with him, the
local 5 star hotel. Roma took her car and husband was completely shattered. He
left for the hotel with a revolver. Roma bolted himself inside a room and went
made enquiries from the reception and without food or water for over 12 hours.
came to know the room number where her However, he could take it no more when
husband was staying. Roma entered the his friend was heard chatting with his wife
room and saw her husband in a compro- the same evening. He dashed out of the
mising position with a lady. Roma became room holding a khukari and chased him,
angry and fired a shot at her husband from and finally stabbing him several times in
her revolver, but it missed and killed the chest region. On his prosecution for mur-
lady sitting with Ramu. Roma pleaded the der, can he take the plea of acting under
defence of grave and sudden provocation. grave and sudden provocation?
Issue: Whether the accused is guilty of Issue: Whether the accused is guilty of
murder? murder?
Decision: Decision:
(a) The accused is not guilty of murder (a) He is not guilty of murder. He can be
as she acted under sudden and grave benefited with the exception of grave
provocation. and sudden provocation. Because
(b) The accused is guilty of murder be- whatever he did, he did in a shocked
cause the provocation was grave but state of mind.
not sudden, because the action of the (b) He is guilty of murder. Because he
accused was deliberate and premedi- killed the deceased out of retaliation.
tated. (c) He can be acquitted giving him a ben-
(c) She is guilty of culpable homicide be- efit of doubt.
cause no wife can tolerate the adulter- 52. Facts: A was very fond of his wife, B.
ous character of her husband. However, B had a paramour, C. One day
50. Facts: On A’s reaching home after a long A saw B in a compromising position with
absence his wife confessed that she had C. A controlled his emotions and did not
developed intimacy with his friend and react. He decided to settle down with B
was carrying his child in her womb. A was in a far flung area. One day when he was
shocked but did not display any external coming back home at the new place he saw
signs of his shock. However, he did nei- C coming out of his new house. He saw C
ther eat nor drink anything nor talked to was wearing his watch which he had re-
any one thereafter. Three hours after the ceived as a gift from his wife B. A imme-
incident, A drove his wife to her mothers diately attacked C with his knife which he
house and left her there. From there he was carrying in his pocket and caused C’s
went to his friends house and on his open- death by attacking vital parts of C’s body.
ing the door shot him without any argu- On his prosecution for murder, can he take
ment. Can A successfully plead grave and the plea of acting under grave and sudden
sudden provocation mitigation. provocation?
Issue: Whether the accused is guilty of Issue: Whether the accused is the guilty
murder? of murder?
Decision: Decision:
(a) The accused is guilty of murder be- (a) No, he is not guilty of murder. Be-
cause he can’t take benefit of grave cause the conduct of wife was un-
and sudden provocation because the pardonable and she created the same
provocation was grave but not sud- circumstances time and again which
den. gave rise to grave and sudden provo-
(b) The accused is guilty of culpable ho- cation. The facts of the case are self
micide. speaking truth.
(c) The accused is innocent. (b) He is guilty of murder.
51. Facts: On hearing from his wife about (c) He is guilty of culpable homicide.
her intimacy with another man (his best 53. Facts: C was engaged to be married with

CLAT.indb 62 31/03/2009 11:41:42


CRIMINAL LAW 63

D. Once C saw D flirting with G, another Issue: Whether the accused is guilty of
girl. C inquired, but D ignored. C became murder?
terribly upset. Suspecting illicit liaison be- Decision:
tween D and G, a week later, C went to (a) B is guilty of murder. Because stran-
D’s flat. Peeping through the door’s chink, gulating any one on such flimsy
she was shocked to see D and G in fla- grounds is sufficient evidence.
grante delicto. On D’s opening the door, (b) B is not guilty of murder.
C immediately took out a penknife which (c) B is guilty of culpable homicide.
she habitually used to carry along and at- (d) B can be acquitted giving him benefit
tacked him. of doubt.
Issue: Whether the accused is the guilty 56. Facts: On learning about his wife’s extra-
of murder? marital affair with his best friend. A went
into a state of shock. He did not have food
Decision:
that afternoon and did neither talk to any-
(a) C is guilty of murder as the reports
one nor watched his favourite day time TV
of the doctor say that the injuries in-
serial. Shortly after forenoon, A was seen
flicted were sufficient in the ordinary
on his friends flat, where he is alleged to
course of nature to cause the death. have shot him dead without uttering a sin-
Moreover she was not legally wedded gle word. A is prosecuted for the murder
wife. of B. Give arguments in A’s defence.
(b) C is not guilty of murder. Issue: Whether the accused is guilty of
(c) C is guilty of culpable homicide. murder?
(d) C is guilty of causing grievous hurt. Decision:
54. Facts: X and Y, two brothers, live sepa- (a) The case is clear cut category of grave
rately. The parents live with X, Y often and sudden provocation. The accused
comes to X’s place and abuses his parents cannot be guilty of murder.
in his presence. One day when X’s father (b) The accused can be held guilty of cul-
in law, K and his friend P, were present. Y pable homicide. Because the provoca-
came there and started abusing his parents. tion was grave but not sudden and the
X who had not said anything to Y earlier, action was deliberate and premeditat-
got enraged because of the presence of K ed.
and P and picked up a knife lying nearby (c) The case is not covered under any
and killed Y. X is charged for murder. He provision of murder or culpable ho-
pleads grave and sudden provocation in micide.
his defence. 57. Facts: A, an army personnel, was posted
Issue: Whether the accused is the guilty in Kargil, a non-family station having dif-
of murder? ficult conditions, for the last one year. Now
Decision: since normalization process had started A
(a) X is guilty of murder because the facts wanted to go on ten days leave to attend
his brother marriage. He pleaded fervently
of the case do not mitigate the culpa-
with his superior officer, B, to grant him
bility of X.
ten days leave. But B was adamant and
(b) X is not guilty of murder. Because the
was not prepared to grant more than seven
mitigating circumstances of the case
days leave. Frustrated by the refusal A
are enough to entitle the accused to took out his service revolver and shot B at
get the benefit of grave and sudden point blank range. A wants you to defend
provocation. him. Give arguments in A’s defence.
(c) X is guilty of culpable homicide. Issue: Whether the accused is guilty of
(d) X can be acquitted giving him benefit murder?
of doubt. Decision:
55. Facts: A’s car slightly scratched B’s car (a) The case is not covered under grave
on a red light. B shouted and grabbed A by and sudden provocation. The accused
the neck and a fight ensued between them. is guilty of murder. Mere refusal of
In the heat of passion, B strangulated A to leave is no ground to give some one
death. B is being prosecuted for murder. power to kill someone.

CLAT.indb 63 31/03/2009 11:41:43


64 CRIMINAL LAW

(b) It is a culpable homicide. Because the cused pleads ‘grave and sudden provoca-
accused did in the heat of passion. tion’ in his defence. Prosecution pleads on
(c) It is neither murder nor culpable ho- the other side the lapse of interval between
micide. act and consequences.
58. Facts: X and Y were happily married, Issue: Whether the accused is guilty of
living in Bombay. M, an old classmate of murder?
Y, was transferred to Bombay. He started Decision:
visiting X and Y at their flat and became (a) The case is covered by Exception 1 to
quite friendly with X also. One afternoon Section 300 notwithstanding the fact
X quite unexpectedly came back from his of time gap between the seeing of the
office and to his uttar shock and disbelief act of adultery and the killing of B. A
he found his wife and M in a compro- having acted under grave and sudden
mising position. He, however, controlled provocation was guilty under Section
himself and kept his cool and asked M 304.
never to saw his face again lest he would (b) The accused is guilty of murder as the
kill him. After about a week X left on a case is not covered under the Excep-
tour telling his wife that he would be back tion 1 to Section 300.
after four days. He, however, returned the (c) The facts of the case are confusing so
very next day in the evening and found M no conclusion can be drawn.
and Y sitting together on the sofa in the 60. Facts: In the past, there used be quarrel
drawing room with M’s hand around Y’s between the accused A and his deceased
shoulder. X straightway went to his bed- uncle D. On the date of occurrence D ap-
room, took out his pistol from the almirah, proached A drunk and abused him in filthy
came back to the drawing room and shot language thus, subjecting A to grave and
M dead. sudden provocation. A reacted immedi-
Issue: Whether the accused is guilty of ately with a stick that was in his hand and
murder? killed D. Whether A is guilty of murder?
Decision: Issue: Whether the accused is guilty of
(a) The accused is not guilty of murder. murder?
The case is covered by Exception 1 to Decision:
Section 300. The mental background (a) The case is covered by Exception 1 or
created by the previous act of the vic- 4 of Section 300. There was no prepa-
tim may be taken into consideration ration and prior deliberation. The re-
for ascertaining whether the subse- action was as sudden as the provoca-
quent act caused grave and sudden tion.
provocation. (b) Case is not covered by Exception 1
(b) The accused is guilty of murder. or 4 of Section 300. So the accused is
(c) The accused is guilty of culpable ho- guilty of murder.
micide. (c) It is neither murder nor culpable ho-
59. Facts: A and his friend B were sleeping micide.
in one room and A’s wife in the adjoining 61. Facts: B had teased A’s sister. Coming to
room. Sometime in the night B got up and know of it, A took a sword and went to B’s
went into the adjoining room and bolted residence. B, his mother and sister were
the door behind him. A also got up and present in the house when A reached there
peeping through a chink in the door saw and started abusing them. He said that he
B and A’s wife having sexual intercourse. would forcibly take away B’s sister. B’s
A returned to the room and lied down. Af- mother took her daughter inside a room
ter sometime B came out of that room and and bolted the door. She then went for-
slept by the side of A. After a short time, ward with folded hands and requested A to
when B began dozing. A stabbed him pardon them. A immediately gave a blow
several times with a knife and killed him. on her neck with the sword and she fell
There was no evidence that the accused down and died. At this B started running
had to go anywhere in search for the knife, towards the fields. A chased him abusing.
which apparently, was with him. The ac- When A was still about 20 feet from him.

CLAT.indb 64 31/03/2009 11:41:43


CRIMINAL LAW 65

B jumped into a well to save himself. B’s 63. Facts: A, who was sexually impotent tried
head hit a hard substance in the well with in vain to have sexual intercourse with a
the result that he lost consciousness and prostitute P, who jeered at him. A stabbed
died of asphyxia due to drowning. Discuss P with a knife and killed her. In a charge
the liability of A. of murder, A pleads the defence of grave
Issue: Whether the accused is guilty of and sudden provocation. Can A succeed?
murder? If not, state the reasons.
Decision: Issue: Whether the accused is guilty of
(a) The act of teasing by B of A’s sister murder?
was not of such a nature whereby it Decision:
can be said that A chopped off B’s (a) The accused is guilty of murder as he
mother head under grave and sud- cannot be benefited with exception of
den provocation. Provocation even grave and sudden provocation.
though sudden was not so grave as to (b) The accused is guilty of rape.
deprive A of his power of self control. (c) The accused is guilty of molestation.
So, A will be liable for murder of B’s
mother. However, B’s death was not HOMICIDE BY RASH OR
the direct result of any act done by the NEGLIGENT ACT
accused i.e., there was no causal con-
nection. A’s chasing cannot be said to 64. Facts: A left his car engine running on a
be the cause of B’s death, so, he will busy street when he stepped out for a min-
neither be liable for murder nor for ute to purchase a bottle of water from a
culpable homicide. But A will be li- shop on the side. While he was in the shop,
able for abetment to commit suicide. some miscreant released the hand break of
(b) The accused is guilty of murder and his car. The car being on a slope, crashed
abetment to suicide. into a person standing nearby. He died as
(c) The facts of the case do not derive to a consequence. Can A be convicted for
any decision. causing death by rash and negligent act?
62. Facts: H suspected his wife of infidelity. Issue: What offence, if any, has been com-
Therefore H started keeping watch on her mitted?
movements. One day H found that his Decision:
wife did not come back to the house dur- (a) A can be charged with the death by
ing the last night. At about 11.00 a.m. next rash and negligent act as he left the car
day when the wife came back to the house running on a busy street.
H asked her whereabouts. The wife got (b) A cannot be guilty of death, because
furious and said, “How long are you going the death was caused by miscreants.
to watch my movements? Only two days (c) It is a case of accident.
back I was sleeping with Z. Do what you 65. Facts: X started from Delhi on his car to
like”. At this H got enraged and picked up go to Gurgaon, at about 11.00 p.m. He
a lathi lying nearby and gave a number of was driving at a reasonable speed and
blows on the head and chest. Within fif- quite carefully. But unfortunately, on the
teen minutes the wife died. way he knocked down two coolies who
He is prosecuted for murder under Section were lying and sleeping on the road itself.
302, IPC. X is prosecuted for causing death of the
Issue: Whether the accused is guilty of two coolies by rash and negligent driving
murder? under Section 304 A, IPC. Can X be con-
Decision: victed? Discuss.
(a) The accused is guilty of murder as per Issue: What offence, if any, has been com-
the facts and circumstances of case. mitted?
(b) He is not guilty of murder it is a clear Decision:
cut of case grave and sudden provoca- (a) X can be convicted for rash and negli-
tion. gent driving.
(c) The facts and circumstances are in- (b) He can not be convicted for rash and
conclusive. negligent driving because the facts of

CLAT.indb 65 31/03/2009 11:41:43


66 CRIMINAL LAW

the case clearly state that he was driv- committed by A? Support your answer
ing the car at a reasonable speed and with case law.
quite carefully. Issue: What offence, if any, has been com-
(c) It’s a clear cut case of murder. mitted?
66. Facts: X who was driving a car at night, Decision:
entered a road which was under repairs and (a) A can be convicted under Section
closed for traffic. He ran over and killed 304-A for rash and negligent driving.
two coolies who were sleeping on the A should not have driven the bus at
road with their bodies completely covered such a speed that he cannot stop it in
under dark blankets. X is prosecuted for time. Driving at a high speed, and that
causing death of the two coolies by rash too at a road crossing, shows that he
and negligent driving under Section 304 acted rashly and negligently.
A, IPC. Can X be convicted? Discuss. (b) A cannot be committed for rash and
Issue: What offence, if any, has been com- negligent driving.
mitted? (c) It’s a case of normal accident.
Decision: 69. Facts: A, an owner of a circus company
(a) X can be convicted for rash and negli- was perpetually troubled by the trespass-
gent driving causing death. ers who would harass the lady perform-
(b) He cannot be convicted for rash and ers by peeping into their dressing rooms.
negligent driving causing death. In order to keep the trespassers away, A
(c) It’s a case of accident. decided to leave two circus lions into the
67. Facts: A, a scientist, working on a rare dressing room region. B a trespasser who
fuel project had fixed an electronic device had sneaked into the dressing room area
was badly mauled and killed by one of the
that not only set in motion an alarm, but
circus lions. Discuss A’s liability under
also released high voltage electric shock
Section 304 A, IPC.
to guard his lab against thieves and intrud-
Issue: What offence, if any, has been com-
ers. B who had sneaked into the lab with
mitted?
a view to steal the fuel formula died on
Decision:
account of an electric shock. Can A be
(a) A’s act is rash as it is reckless disre-
prosecuted for an offence under Section
gard of the presence of the trespassers.
304A?
The criminality lies in running the risk
Issue: What offence, if any, has been
of doing such an act with recklessness
committed. or indifference as to its consequences
Decision: i.e., without belief in the result of such
(a) A can not be convicted for rash and doing.
negligent act causing death. Because (b) It is not rash and negligent act. It is a
B who had sneaked into lab with a precautionary measure.
view steal the formula died because of (c) No conclusive evidence so no conclu-
his own commission of offence. sive decision.
(b) He can be convicted for rash and neg- 70. Facts: X, a notorious smuggler, built
ligent act causing death. a strong house for himself away from
(c) Case is inconclusive. the city. The house was fortified by high
68. Facts: A, bus driver, seeing the green compound wall around the house and live
signal at a road crossing, raised the speed electric wire ran on the upper edges of the
of his bus to clear the crossing before compound wall which used to be activated
the lights turned red. However, the lights during the night. Well lit warning boards
turned red before A could reach the cross- with danger signs of high voltage were
ing and due to his failure to stop the bus placed all around. Y, a thief, in order to
in time he collided with an autorickshaw commit theft attempted to scale the wall
coming on green lights from right direc- in the night, came in contact with the wire
tion. As a result the autorickshaw driver and was electrocuted. X is charged under
died on the spot and 2 passengers in it got Section 304 A for causing the death of Y.
seriously injured. What offence, if any, is How will you decide?

CLAT.indb 66 31/03/2009 11:41:43


CRIMINAL LAW 67

Issue: What offence, if any, has been com- al at a speed of 80 km per hour near a pri-
mitted? mary school in the metropolis where the
Decision: speed limit was notified to be only 30 km
(a) X is not liable because he had already per hour. Suddenly a school child tried to
taken the due caution placing well lit cross the street and the driver L couldn’t
warning boards and danger signs. stop the car, as the brakes did not work at
(b) X is liable for rash and negligent act the crucial time. The child was run over
causing death. and killed.
(c) X is guilty of murder. Issue: What offence, if any, has been com-
71. Facts: Accused A received from her par- mitted?
amour a packet to administer to her hus- Decision:
band as a charm. A administered it, and (a) L is guilty of murder.
this resulted in the death of her husband. (b) L is guilty of criminal rashness and
A did not know that the substance given negligence—running a defective car,
to her was poisonous until she saw its ef- driving at a high speed near a primary
fects. Discuss A’s liability under Sector school in disregard of the specified
304 A, IPC. speed limit.
Issue: What offence, if any, has been com- (c) L is innocent as the car was defec-
mitted? tive.
Decision: 74. Facts: X was driving a tourist bus at a fast
(a) A is guilty of murder. speed. When he was 100 yards away from
(b) A is guilty of culpable homicide. a traffic crossing he saw a green signal.
(c) A is guilty of criminal rashness and In a bid to cross the road he enhanced the
negligence under 304A. speed but before he could cross the road
72. Facts: X has an illicit intimacy with Y, the green signal turned red, but X contin-
a married woman. X suggests to Y that ued to drive on and hit C, a school child,
they should do away with her husband by on the zebra crossing. For what offence
murdering him, to which Y does not give
can X be convicted?
her consent. After two months, X gives a
Issue: What offence, if any, has been com-
white coloured poison to Y saying that it
mitted?
is a “magic powder”. He asks Y to mix the
Decision:
poison in drink so that her husband would
(a) It’s a matter of accident.
not object to Y’s illicit relations with X. Y
(b) X can be convicted under Section
carries out the instructions given by X and
304-A for rash and negligent driving.
her husband dies consequently.
(c) The accused is innocent as he had no
Issue: What offence, if any, has been com-
knowledge of green signal.
mitted?
Decision: 75. Facts: The residents of the adjoining
(a) The provisions of Section 304-A ap- slums were persistently using the park for
ply to case where there is no intention defecation. On the order of the D.D.A.
to cause death and no knowledge that Chairman, the park was fenced. The elec-
the act done in all probability would tric current was run on the wires on the
cause death. In the present case, look- top. A trespassing slum-dweller touched
ing at the past behaviour, X it was im- a live wire and was electrocuted. Can the
perative upon Y to ascertain whether Chairman be held guilty for an offence un-
the powder was noxious or not. Fail- der Section 304-A?
ing which it can be said that she failed Issue: What offence, if any, has been com-
to exercise reasonable and proper care mitted?
and hence she is liable under Section Decision:
304-A. (a) The chairman is not liable as he was
(b) The accused is not guilty of any discharging his public duties.
offence. (b) The D.D.A. Chairman will be liable.
(c) The accused is guilty of murder. There should be warning in some
73. Facts: L was driving his pretty old car the form about the electrical fencing of
brakes of which had become dysfunction- the park. This is more so because the

CLAT.indb 67 31/03/2009 11:41:43


68 CRIMINAL LAW

slum-dwellers were in the habit of us- Decision:


ing the park for defecation. (a) A cannot be held guilty because he
(c) It’s a case of judicial prerogative of was just performing his duty of a doc-
course. tor.
76. Facts: In order to prevent the plucking of (b) He is liable for murder.
the fruits by neighbours children, Mush- (c) A can be convicted under Section
eer guarded his orchard with a live electric 304-A for causing B’s death by rash
wire and put up a written warning about and negligent act.
it at the front gate. Bala, 6 years old, un- 79. Facts: A carrying a loaded gun stood qui-
aware of the electric wire or the warning, etly in a corner of a public place and was
was killed by electrocution while trying to watching a dramatic performance. One
scale the wall of orchard from the back- of the actors, who was playing the part
side. Can Musheer be convicted for caus- of a drunken dacoit, to enhance the effect
ing death by rash and negligent act? of his acting approached A and grappled
Issue: What offence, if any, has been com- with him and in the course of the struggles
mitted? the gun went off and the actor was killed.
Decision: Is A guilty of a negligent act?
(a) Musheer will not be liable for any of- Issue: What offence, if any, has been com-
fence because it is his right to defend mitted?
his property. Decision:
(b) Musheer will be liable inspite of a (a) A is guilty of negligent act.
written warning about the live electric (b) A is guilty of murder.
wires. Because such a warning would (c) A is not guilty of a negligent act or
be ineffective against the children. causing death by a negligent act.
Moreover, he adopted a rather drastic
measure. He should have adopted a HURT AND GRIEVOUS HURT
reasonable precaution to save his or-
chard. 80. Facts: M owed Rs 1000 to P but did not
pay back though the latter made demands
(c) He must be charged with murder.
several times. P lastly made another de-
77. Facts: ‘A’ takes up a gun not knowing it
mand but M as usual promised to pay lat-
is loaded. Points in sport at ‘B’ and pulls
er whereupon P kicked him twice on the
the trigger. ‘B’ is shot dead. Can ‘A’ be
abdomen and M collapsed and died. What
convicted for causing death by rash and
offence is P guilty of?
negligent act?
Issue: What offence, if any, has been com-
Issue: What offence, if any, has been com-
mitted?
mitted?
Decision:
Decision:
(a) It could not be said that P intended
(a) A is liable for murder.
or knew that kicking on the abdomen
(b) ‘A’ will be liable under Section 304A was likely to endanger life and conse-
for causing death by negligent act. quence he is guilty of causing death.
(c) A is liable for culpable homicide. (b) P is guilty of murder.
78. Facts: B was brought unconscious to a (c) P is guilty of culpable homicide.
private nursing home. In the absence of (d) P is guilty of causing grievous hurt.
the surgeon, his attendant, A, entered it 81. Facts: A causes an injury to Z, which re-
as an emergency case and operated upon sults in his death. It was never intended by
the B’s stomach to extract undigested poi- A to cause his death nor in normal condi-
soned food. During the operation, B died. tions it could have caused Z’s death. What
The postmortem report attributed the B’s offence A has committed?
death to improper handling of such cases Issue: What offence, if any, has been com-
independently yet he carried on the experi- mitted?
ment as B needed an immediate treatment. Decision:
Discuss the criminal liability of A. (a) The offence committed is neither of
Issue: What offence, if any, has been com- causing grievous hurt nor of culpable
mitted? homicide, but of causing simple hurt.

CLAT.indb 68 31/03/2009 11:41:43


CRIMINAL LAW 69

(b) The offence committed is murder. 85. Facts: A gave severe beating to B on a
(c) It is case of culpable homicide. highway at night and left him with frac-
(d) It is a case of causing grievous hurt. tured legs and bleeding. Soon thereafter
82. Facts: With a view to chastising her B became unconscious due to substantial
daughter, the accused gave a kick on the blood loss. The next morning B was found
back and two slaps on the face, the result dead, crushed by a speeding truck/ falling
of which was death. tree on account of cyclone. Has A com-
Issue: What offence, is any, is caused by mitted culpable homicide in terms of Sec-
the accused in the above case? tion 299, IPC?
Decision: Issue: What offence, if any, has been com-
(a) It could not be said that accused in- mitted?
tended or knew that kicking on the Decision:
back and slapping were likely to en- (a) He is guilty of murder.
danger life and consequently he is (b) A is guilty of grievous hurt only. The
guilty of causing hurt only. line between culpable homicide not
(b) He is guilty of murder. amounting to murder and grievous
(c) He is guilty of culpable homicide. hurt is very thin. In one case the inju-
(d) He is guilty of causing grievous hurt. ries must be such as are likely to cause
83. Facts: The accused threw his wife from death and in the other they endanger
the window about 6 feet high, but the fall life. A’s act was not the direct or sole
was broken by a weather board fixed just cause of B’s death.
below it and resulted in the fracture of (c) He is guilty of culpable homicide.
knee and in several small wounds. 86. Facts: In the course of an altercation be-
Issue: What offence, if any, has been com- tween A and B on a dark night, the former
mitted? aimed a blow with his stick at the head of
Decision: the later. To ward off the blow, B’s wife X,
(a) If the offender intended or knew him- who had a child on her arm intervened be-
self to be likely to cause only simple tween them. The blow missed its aim and
hurt, he cannot be convicted for griev- fell on the child head causing severe inju-
ous hurt even if the resultant hurt was ries, due to which the child died. Discuss
grievous. The accused in the present the liability of A.
case could not have intended or knew Issue: What offence, if any, has been com-
that he was likely to cause grievous
mitted?
hurt (which resulted due to a weather
Decision:
board). Thus, he is guilty of simple
(a) He is guilty of murder.
hurt only.
(b) A is guilty of causing simple hurt, as
(b) He is guilty of grievous hurt.
the blow, if it had fell upon the com-
(c) No offence can be made out.
plainant would have caused simple
84. Facts: A, a chain snatcher, forcibly pulled
hurt.
the ear rings from the ears of an old lady.
(c) He is guilty of culpable homicide.
Both the ear lobes were torn and the old
87. Facts: A struck violent blows on the head
lady suffered pain and suffering for over
and shoulders of a woman who was carry-
three weeks. For what offence can A be
prosecuted? ing an infant child in her arms. One of the
Issue: What offence, if any, has been com- blows fell on the child and killed it while
mitted? the woman received simple injuries. Dis-
Decision: cuss the liability of A.
(a) He is guilty of theft. Issue: What offence, if any, has been com-
(b) A is guilty of voluntarily causing mitted?
‘grievous hurt’. The case comes under Decision:
clause eighthly of Section 320 (‘any (a) The accused is guilty of murder.
hurt which caused the sufferer to be (b) A is liable for causing simple hurt
during the space of 20 days in severe to the woman under Section 323 but
bodily pain’) would be liable under Section 304-A
(c) He is guilty of rash and negligent. for causing death of the child by his

CLAT.indb 69 31/03/2009 11:41:43


70 CRIMINAL LAW

rash and negligent act. A knew that Decision:


the woman was carrying the child in (a) A and C are guilty of abetment to sui-
her arms and his blows might hit the cide.
child. Thus, he ought to have taken (b) A and C are not guilty under section
care. 306 because instigation necessarily
(c) Guilty of culpable homicide. indicates some active suggestions or
88. Facts: The wife of the accused died in support all stimulations to the com-
mysterious circumstances. There was mission of the act itself, and advice
history of harassment and cruelty caused can become instigation only if it is
to her by the in-laws and the husband. found that it was advice which was
The post mortem report indicated by the meant actively to suggest or stimulate
C.F.S.L. report found symptoms of cru- the commission of an offence.
elty. The accused was charged under Sec- (c) It is an offence of conspiracy.
tion 304-B and Section 498-A.
Issue: What offence, if any, has been com- KIDNAPPING AND ABDUCTION
mitted?
Decision: 91. Facts: P, aged 17 years, the daughter of
(a) The facts and circumstances prove M, develops friendship with R, a young
that the accused and his family mem- man living in a neighbouring house. She
bers were liable to be committed un- wants to marry him. On learning this, M
takes her and entrusts her to S, a relative
der section 304 B, and section 498 A.
living in a different part of the same city.
(b) The mysterious circumstances show
Shortly thereafter, P goes out of S’s house
that the offence amounting to murder.
and calls R on the telephone. R joins her
(c) The offence committed is culpable
and both go from place to place for some
homicide.
days as husband and wife until the police
89. Facts: A, a married woman, died within
arrests them on a complaint of kidnapping
seven years of her marriage in unnatural
given by M. R is prosecuted for the of-
circumstances. The dead body was recov-
fence of kidnapping with intent to con-
ered from a well adjacent to the family of tempt P to marry. Can R be held guilty of
the accused. The cause of death, as per the offence?
post mortem report was not drowning but Issue: What offence, if any, has been com-
strangulation. There was witness account mitted?
to the effect that since the marriage the Decision:
girl was being tortured and even assaulted (a) R is not guilty of the offence of kid-
for dowry. She was even deserted but was napping.
taken back under the pressure of the me- (b) R is guilty of the offence in kidnap-
diator. ping.
Issue: Is the accused and his family liable (c) R is guilty of offence of conspiracy
for dowry death or any other offence? and blackmailing.
Decision: 92. Facts: A developed friendship with a
(a) The accused are guilty of murder. girl aged 17 years. In course of time the
(b) The accused are guilty under Section friendship developed into intimacy and
304 B and section 498 A. three months before her 18th birthday A
(c) The accused are guilty of culpable ho- took the girl several times to movies and
micide. public parks during the college hours, but
(d) The accused are guilty of causing the girl returned home every night. Two
grievous hurt. weeks before her 18th birthday the girl
90. Facts: A and B were in love with C, a girl, insisted that they should stay overnight.
and quarrelled over her. A, the accused, A arranged for the girl’s stay in his own
along with C, taunted B to commit sui- house. Next morning on the complaint of
cide. The frustration thus caused led B to the girl’s father A was arrested. A is pros-
commit suicide. ecuted for the kidnapping of the girl.
Issue: What offence, if any, has been com- Issue: What offence, if any, has been com-
mitted? mitted?

CLAT.indb 70 31/03/2009 11:41:44


CRIMINAL LAW 71

Decision: in going to meet her uncle in Agra she


(a) A is not guilty as the girl voluntarily could accompany him. G, then, asked for
left her guardian’s protection so it not her parent’s permission and they allowed.
a case of kidnapping. On reaching Agra A dropped G at her un-
(b) A is guilty of kidnapping as he delib- cle’s house and in the evening proceeded
erately developed intimacy with her to take her back to New Delhi. While still
to induce her to leave her guardian’s in Agra, A suggested to G that he wanted
house. to marry her and keeping in view the or-
(c) A is guilty of conspiracy, kidnapping thodox views of their parents it would be
and offence of taking undue liberty better to avail the opportunity. G, though
with the girl. initially reluctant, ultimately agreed to A’s
93. Facts: A, a boy aged 19 years and B, a suggestion. Accordingly, they purchased
girl aged 17½ years, are convent students some ornaments and wedding dress and
in Delhi. Being classmates they developed then went to a temple and got married.
friendship and on several occasions A and They stayed in Agra for a night and de-
B had visited restaurants and movies in cided to go to Bombay. However, the next
each other’s company. On the fateful day day they were arrested. A is tried for kid-
A invited B to his house. B readily accept- napping G?
ed the invitation and reached A’s house Issue: What offence, if any, has been com-
in her car. B also insisted to stay in A’s mitted?
house for the night. A reluctantly permit- Decision:
ted B to stay with him. The next morning (a) A is not guilty.
on the complaint of the father of the girl (b) A is guilty. If the accused has laid a
the police recovered the minor girl from foundation by inducement, allure-
the house of A. ment, etc., and if this can be consid-
Issue: What offence, if any, has been com- ered to have influenced the minor or
mitted? weighed with her in leaving her guard-
Decision: ian’s custody or keeping and going to
(a) A is guilty because the girl is below the accused, then it would be difficult
18 years of age. for the accused to plead that the minor
(b) A is not guilty, as the girl voluntarily had voluntarily come to him.
left her guardian’s protection. She is (c) The facts of the case are not clear and
an educated girl on the verge of at- no conclusive decision can be given.
taining majority, and was capable of 95. Facts: X, a boy of 15 years asks a girl of
knowing what was good and what was 16 years to run away with him for getting
bad for her. There is no promise or in- married. On this suggestions the girl runs
ducement from the side of A. If fact, away with X, from Delhi to Madras. Can
he was reluctant to let her stay with X be convicted for kidnapping? Discuss.
him. Merely allowing her to stay at Issue: What offence, if any, has been com-
house since it was late hours of night mitted?
that too after her insistence does not Decision:
amount to taking or enticing as envis- (a) X is not guilty as he is minor.
aged in Section 361. (b) The minorship of the X does not lead
(c) A is guilty of inducing and enticing to any conclusion.
the girl and kidnapping. (c) X will be guilty (though the court may
94. Facts: G, a 17 years and 9 month old girl take into consideration his juvenile
and student of B.A. was residing next door age and award a lesser punishment).
to the house of A, a young businessman in The word “Whoever” in Section 361
a posh colony of New Delhi and both had suggests that a “minor” can kidnap
become friendly to each other. G’s uncle another “minor”.
was doing business in Agra. One evening 96. Facts: A girl of about 16 years was in love
A told G that early next morning he was with the accused. The evidence showed
going to Agra by car and would come back that she left her home on her own accord
the same evening, and if G was interested and joined the accused for getting their

CLAT.indb 71 31/03/2009 11:41:44


72 CRIMINAL LAW

marriage registered and live as husband for sometime and accepted it. They went
and wife. to temple and got married. X is tried for
Issue: What offence, if any, has been com- kidnapping the girl. He pleads that the girl
mitted? had abandoned her guardianship because
Decision: of ill-treatment of her mother.
(a) Accused is guilty of kidnapping. Issue: What offence, if any, has been com-
(b) Accused is guilty of seduction and mitted?
elopement. Decision:
(c) The accused is not guilty of kidnap- (a) X will not succeed, as G had not
ping the girl. abandoned her guardianship. In fact,
97. Facts: Two girls of around 17 years of X tried to take advantage of her tender
age were alleged to have been kidnapped age and predicament and thus, this act
from lawful guardianship. Both the girls amount to ‘enticing’ the girl out of the
categorically stated that they had gone keeping of her lawful guardian within
voluntarily. Rather they had persuaded the the meaning of Section 361. G’s con-
accused to take them for an outing. sent is immaterial.
Issue: What offence, if any, has been com- (b) X is liable for seduction and elope-
mitted? ment.
Decision: (c) X is not liable.
(a) The accused is guilty of kidnapping (d) X married the girl with her consent.
because the girls are below 18 years 100. Facts: Asha aged 16 years and a 10th
of age. class student in a public school in Delhi,
(b) The accused is guilty of seduction and once told her friend Rajesh aged 16 years
elopement. and her class fellow that her father was ill–
(c) The accused is not guilty of kidnap- treating her and suggested to him to elope
ping the girls. together and marry. Thereafter Rajesh
98. Facts: A enticed a minor girl M without made the plan and explained it to Asha and
the consent of her lawful guardian G and she accordingly left her parental house to
kept her with him for a month after which join Rajesh. They left for Chandigarh to
he deserted her. While M was loitering on get married in an Arya Samaj temple, but
the street immediately thereafter. B took were caught by the police. Is Rajesh guilty
her with him from whose custody M, was of kidnapping Asha?
later recovered by the police. Issue: What offence, if any, has been com-
Issue: What offence, if any, has been com- mitted?
mitted? Decision:
Decision: (a) The word “Whoever” in Section 361
(a) A is not guilty of kidnapping. suggests that a “minor” can kidnap
(b) B is guilty of kidnapping. another “minor”. As noted above,
(c) G is guilty of negligence. Rajesh will be guilty of kidnapping
(d) A is guilty of kidnapping from lawful Asha.
guardianship under Section 361. B is (b) Rajesh is not guilty of kidnapping
not guilty as he merely gave shelter to Asha because both are minor.
a girl who was loitering on the street. (c) The court itself put the action of mi-
In other words, there was no taking nor under the exceptions.
from the keeping of lawful guardian- 101. Facts: A Hindu girl of 17 years studying
ship. in XII standard under the care and protec-
99. Facts: G, a girl of 16 years, 11 months tion of her parents was living with them.
and 24 days left her parental home for her She was having criminal intimacy with a
aunt’s house because of the ill-treatment of shopkeeper running a shop near her house.
her stepmother with an intention to return One day the girl left her house and went
back after one month. On the way, she met to the shopkeeper and asked him to take
X, a 20 year old man and narrated her woes her away permanently. The shopkeeper
to him. X, who was a kind man offered to took her away to several places. Later
marry her. G thought about this proposal on, he was arrested under Section 363 of

CLAT.indb 72 31/03/2009 11:41:44


CRIMINAL LAW 73

the Indian Penal Code. State with reasons Issue: What offence, if any, has been com-
whether the shopkeeper be convicted. mitted?
Issue: What offence, if any, has been com- Decision:
mitted? (a) A is not guilty of kidnapping.
Decision: (b) A is accused of mock/invalid mar-
(a) The girl left her father’s house of her riage.
own without any kind of inducement (c) A is accused of taking marriage
from the shopkeeper. Since she was ceremony in unlawful manner.
having criminal conspiracy with the 104. Facts: G, a girl of 17 years, was in love
shopkeeper, she might have thought with the accused, a widower who wanted
that the shopkeeper may agree to her to marry her. G’s mother was in favour of
suggestion. No active part was played such marriage but the father was not. One
by the shopkeeper in causing the girl day when the father was out of town the
to leave her guardianship. Further, accused took G away with her mother’s
when he took her away she was no consent so that they could get married at
more in the keeping of lawful guard- another place. Is the accused guilty of kid-
ianship. Thus, the shopkeeper is not napping?
liable for kidnapping of the girl. Issue: What offence, if any, has been com-
(b) The shopkeeper is guilty of kidnap- mitted?
ping. Decision:
(c) The shopkeeper is guilty of criminal (a) The accused is guilty of kidnapping.
conspiracy, seduction and elopement. Although the girl’s father was away
102. Facts: A girl, aged 17 years and 10 on a tour, she would be said to be still
under lawful guardianship of her fa-
months, was participant in a social service
ther.
camp of two months duration. M, a busi-
(b) The accused is guilty of bigamy
nessman, who had became acquainted to
(c) The accused is guilty of adultery.
the girl during the train journey (from the
(d) The accused is guilty of seduction and
girl’s place to the camp), met her several
elopement.
times in the course of the camp and also
105. Facts: A is a minor wife of B. C takes her
represented gifts to her. On the completion
forcibly and without the consent of B and
of the camp, M took the girl to his home-
keeps her in his house for two months.
town and got married with her. Is M guilty
There she develops intimacy with Z a
of kidnapping? neighbour of C and runs away with Z to
Issue: What offence, if any, has been com- a distant place. However, both of them got
mitted? arrested on the report of B. Discuss the
Decision: liability of C and Z in relation to the of-
(a) M would not be liable for kidnapping fences of kidnapping and abduction.
because by the time of completion Issue: What offence, if any, has been com-
of the camp, the girl had become 18 mitted?
years old and has thus, ceased to be a Decision:
minor. (a) C is liable for the offence of abduction
(b) M will be liable for kidnapping of mi- under Section 362. Z is not liable for
nor. kidnapping because C was not lawful
(c) M is liable for seduction and rape of guardian of A at that time. Z may be
the minor. liable only when A is taken for some
103. Facts: A girl on her own accord went to illegal purpose.
A’s house, persuaded him to execute a (b) C is liable for illegal detention.
marriage agreement, returned to her fa- (c) A is liable for not taking care of his
ther’s house and told him about the agree- wife.
ment and her father locked her in a room. (d) Z is liable for kidnapping and crimi-
After three days the girl forced open the nal intimacy.
door and went to the accused A. Is A 106. Facts: X takes a girl of 13 years of age
guilty of kidnapping? without her knowledge to Mumbai to send

CLAT.indb 73 31/03/2009 11:41:44


74 CRIMINAL LAW

her to Dubai to handover for marriage to her father F to be taken away by relatives
a Sheikh over there and dispatches her to of her elder sister to her home to look after
Dubai for the above purpose. her elder sister for some time. After some
Issue: What offence, if any, has been com- time F received the information that these
mitted? relative have got G married to A. F lodged
Decision: a report with the police that his daughter
(a) X is not guilty because it may be his has been kidnapped by the relatives of
business. G’s elder sister and she has been forced
(b) X is guilty of ‘kidnapping from India’ by them to have illicit intercourse with A.
(Section 360) and ‘kidnapping, ab- Police recovered G from A’s house after a
ducting or inducing woman to compel month. G deposed that A had sexual inter-
her marriage’. (Section 366). course with her many a times. However,
(c) X is guilty of immoral trafficking. medical report states that no injuries exist
on G’s body. In a charge of rape against A,
RAPE he pleads that G has consented to such act
as in indicative of her staying at his house
107. Facts: A, a 15 year old girl, left her moth- for one long month. Can A succeed?
er’s house and joined the accused because Issue: What offence, if any, has been com-
her mother has turned down the proposal mitted?
of her marriage with the accused on the Decision:
ground that she was too young. While she (a) A is not guilty.
was with the accused he had sexual inter- (b) A cannot succeed because sexual in-
course with her against her will. tercourse with a girl/woman with or
Issue: What offence, if any, has been com- without her consent when she is under
mitted? 16 years amounts to rape. [Sc. 375, cl.
Decision: (vi)]. In such a case medical evidence
(a) The accused is not guilty of kidnap- showing that the girl was habituated
ping. to sexual intercourse and that there
(b) The accused is guilty of rape because were no signs of rape would also be of
sexual intercourse with a woman with no help if intercourse by the accused
or without her consent when she is un- with the girl is proved.
der 16 years amounts to rape. [Section (c) F is guilty who allowed a minor girl
375, cl. (vi)]. of 13 years of age to be taken by the
(c) The mother is accused of maltreat- relatives.
ment. 110. Facts: Kitty, a girl of 15 years and 10
108. Facts: A, a 15 year old girl, having been months was married to Guddu. Believing
rebuked by her mother leaves her house. Guddu’s identical twin Chunnu to be Gud-
At railway station she met the accused du, Kitty had willful sexual intercourse
who takes her to his house. He provides with Chunnu who knew of her mistaken
her clothes, money and ornaments at his belief. Kitty realized her mistake later.
house and has sexual intercourse with the Issue: What offence, if any, has been com-
girl with her consent. mitted?
Issue: What offence, if any, has been com- Decision:
mitted? (a) Chunnu is innocent.
Decision: (b) Chunnu can be successfully pros-
(a) The mother is accused of maltreat- ecuted for rape of Kitty. A man is said
ment. to commit ‘rape’ who has sexual in-
(b) The accused is guilty of rape because tercourse with a women with her con-
sexual intercourse with a woman with sent, when the man knows that he is
or without her consent when she is un- not her husband and that her consent
der 16 years amounts to rape. [Section is given because she believes that he
375, cl. (vi)]. is another man to whom she is lawful-
(c) The accused is not guilty. ly married [Section 375, cl. (vi)]. Fur-
109. Facts: G, a 13 year old girl, was allowed by ther, a sexual intercourse with a girl/

CLAT.indb 74 31/03/2009 11:41:44


CRIMINAL LAW 75

woman with or without her consent A, by a change of mind, decides not to


when she is under 16 years amounts take the ring. Has he still committed any
to rape [Section 375, cl. (vi)]. crime?
(c) Kitty is liable for inducement for sex- Issue: What offence, if any, has been com-
ual intercourse. mitted?
111. Facts: A, along with other young people Decision:
was partying at a farmhouse. During the (a) A is not guilty of theft as he had
course of the evening, A and a girl aged changed his mind.
17, both drunk, were alone in the upstairs (b) It is a case of criminal misappropria-
bedroom and he asked the girl whether tion.
she wished to lose her virginity. A knew (c) A at the time of first moving the ring
that if the girl had been sober she would commits theft, it does not matter that
not have agreed. Sexual intercourse then he later changes his mind [See illus-
took place briefly. The girl subsequently tration (h) to Section 378].
told her parents what had happened. Is A 114. Facts: A, a washer man, washed a carpet
guilty of rape? on the bank of a river and hung it up there
Issue: What offence, if any, has been com- to dry. A went to eat his lunch when B, a
mitted? boatman, tempted by the look of the car-
Decision: pet, moved it to furnish his boat for a day
(a) A is not guilty of rape. and thereafter return it back to A. Before
(b) The girl is guilty of drinking, loitering B could run away with the carpet he was
and inducing A. apprehended by A.
(c) A was guilty of rape. He took advan- Issue: What offence, if any, has been com-
tage of the state of the victim as she
mitted?
was under the influence of alcohol.
Decision:
Section 375 (v), negatives the consent
(a) B is not guilty of theft.
of the women for the purpose of the
(b) No offence can be made against B.
offence of rape, if the woman is of un-
(c) B is guilty of theft.
sound mind, or is under the influence
115. Facts: X took away Y’s purse from his
of intoxication at the relevant time.
pocket while he was asleep. On getting up
112. Facts: A befriended a girl aged 17 and of-
Y found the purse in X’s hand. He asked
fered her money and drinks. She and sev-
X to return it to him, where upon X cursed
eral of her friends went to A’s house. A
pornographic video-tape was played on a him and threatened him with dire conse-
T.V. set. A then had intercourse with the quences if he attempted to take the purse
girl in a separate room, the girl being un- or to inform the police. Y struck by X’s
der the influence of alcohol and video. Is fear kept quiet. However, some bystanders
A guilty of rape? reported the matter to the police.
Issue: What offence, if any, has been com- Issue: What offence, if any, has been com-
mitted? mitted?
Decision: Decision:
(a) A is not guilty of rape because the girl (a) X is guilty of criminal intimidation.
had gone willingly. (b) X is not guilty because mere being the
(b) The girl is guilty of inducing the ac- purse to be in the hands of X does not
cused through pornographic movies. amount to theft.
(c) A was guilty of rape. (c) X can be prosecuted for the offence of
committing theft.
THEFT AND EXTORTION 116. Facts: A gives an expensive suit length
to the tailor for stitching. The tailor failed
113. Facts: A finds a ring belonging to Z on to give the suit even two weeks after the
a table in the latter’s house and puts the appointed date. On his fourth visit to the
same under the carpet there with the inten- tailor’s shop A forcibly removed the uncut
tion of taking it afterwards. The ring still suit length from the almirah, despite lot of
lies in the house of Z undetected. Later, resistance from the shop assistants.

CLAT.indb 75 31/03/2009 11:41:44


76 CRIMINAL LAW

Issue: What offence, if any, has been com- was not committed, but A was guilty
mitted? of abetment of theft.
Decision: (b) A is guilty of theft.
(a) may have committed criminal tres- (c) A is innocent.
pass and assault, but not theft as what (d) B is guilty of theft.
he did was not done dishonestly. See 120. Facts: A, a government employee in the
illustration (i) to Section 378. Post Office Department, while assisting in
(b) is guilty of theft. the sorting of letters, secreted two letters
(c) is not committed any offence as suit with the intention of handling them to the
length belonged to him. delivery peon and sharing with him cer-
117. Facts: X had given his watch for repairs to tain money payable upon them.
Y, who had promised to do the job within Issue: What offence, if any, has been com-
a week. After 15 days, when X visited Y’s mitted?
shop, he found his watch still kept un- Decision:
repaired. He took the watch from the shelf (a) A is guilty of theft and of an attempt
and came back without telling anything to to commit dishonest misappropriation
Y. of property.
Issue: What offence, if any, has been com- (b) A is guilty of dereliction of duty.
mitted? (c) A is guilty of conspiracy.
Decision: (d) A is guilty of invading the privacy of
(a) Yes, X commits theft though the the person to whom the letters relat-
watch is his own property but he takes ed.
it dishonestly. See illustration (k) to 121. Facts: A, at a railway station, inserts
Section 378. counterfeit coins into an automatic ma-
(b) X is not guilty of theft. chine and causes it to eject railway tickets
(c) Y is guilty of negligence and can be which A and his friends B, C and D make
prosecuted in the consumer protection use of.
act for the deficiency of services. Issue: What offence, if any, has been com-
118. Facts: A gave his tape recorder to B for mitted?
repairs who promises to carry out the re- Decision:
(a) A is guilty of Criminal Misappropria-
pairs within two days. The repair charges
tion. B, C and D are not liable.
were settled at Rs 300. B completed the
(b) A will be guilty of theft; B, C and D
repairs within time, satisfactorily. A with
would be liable under Section 411,
no intention to pay took away the tape re-
IPC for receiving stolen property as
corder from B’s shop in his absence.
they had used the tickets.
Issue: What offence, if any, has been com-
(c) A, B, C and D are innocent. Railway
mitted?
authorities are liable for negligence.
Decision:
122. Facts: A intending to cause a theft to be
(a) A is guilty of theft.
committed, instigates B to take property
(b) A is not guilty of theft.
belonging to J out of J’s possession. A
(c) A is guilty of criminal misappropria-
induces B to believe that the property be-
tion. longs to A. B takes the property out of J’s
(d) A is guilty of cheating. possession in good faith believing it to be
119. Facts: A sought the aid of B with the in- A’s property.
tention of committing a theft of the prop- Issue: What offence, if any, has been com-
erty of B’s master. B, with the knowledge mitted?
and consent of his master, and for the pur- Decision:
pose of procuring A’s punishment, aided (a) B is liable of theft.
A in carrying out his object. (b) A is liable for abetting theft. B will not
Issue: What offence, if any, has been com- be guilty because he lacks dishonest
mitted? intention to take another’s property.
Decision: (c) A is not liable for any offence. But B
(a) As the property removed was so taken is liable for criminal misappropria-
with the owner’s knowledge, theft tion.

CLAT.indb 76 31/03/2009 11:41:44


CRIMINAL LAW 77

123. Facts: A intending to commit theft enters a criminal charge, and dishonestly in-
B’s house at night and removes from one duces Y to deliver money.
of the rooms a heavy box to the courtyard 127. Facts: A refused to register the marriage
where he opens it. He does not find in the of B and C. C refused to enter their mar-
box anything worth taking and leaving it riage in the register unless he was paid
there, goes away. Rs 50. B paid the money.
Issue: What offence, if any, has been com- Issue: What offence, if any, has been com-
mitted? mitted?
Decision: Decision:
(a) A is not liable for any offence. (a) A has committed criminal misappro-
(b) A will be liable for attempting theft. priation.
He will also be liable for house tres- (b) A is liable for redtapism.
pass under Section 442, IPC. (c) A cannot be convicted for extortion,
(c) B is liable for negligence. as there is no ‘fear of injury’, an es-
124. Facts: A with a dishonest intention to sential requirement of extortion under
steal enters his hand in B’s pocket at a fair. Section 383. The ‘fear’ must be of
But B’s pocket was empty and A could not such a nature and extent as to unsettle
steal anything. the mind of the person on whom it
Issue: What offence, if any, has been com- operates and takes away from his acts
mitted? that element of free voluntary action
Decision: which alone constitutes consent. A is,
(a) A is liable for theft. however, guilty of ‘illegal gratifica-
(b) A will be liable for attempting theft. tion’.
(c) A is not liable for any offence.
128. Facts: P, a police officer, arrested B and
125. Facts: A gave his watch for repair to B.
refused to accept bail until Rs 500 was
A dispute occurred between the two over
paid and released him only when the
the extra parts replaced by B, which were
amount demanded was paid.
not included in the earlier estimate. B was
Issue: What offence, if any, has been com-
unwilling to deliver the watch unless ex-
mitted?
tra payment was made. In the course of
Decision:
heated argument A took out a knife, see-
(a) P is not guilty of extortion.
ing which B did not object to the watch
(b) P is liable for misuse of his position.
being delivered without excess charges.
Issue: What offence, if any, has been com- (c) P is guilty of extortion.
mitted? 129. Facts: A severed the strings of W’s
Decision: necklace while she was trying to board
(a) A is not liable for any offence. Be- a crowded bus. The necklace instead of
cause the watch belong to him and he falling down got lodged in W’s clothes.
took it away. Whether A has committed the theft?
(b) A has committed the offence of extor- Issue: What offence, if any, has been com-
tion. mitted?
(c) B is liable for cheating. Decision:
126. Facts: X saw Y removing a ‘Stereo’ from (a) A is liable for attempt to theft.
a car parked in Kamla Nagar. X demands (b) A is not liable for any offence.
Rs 200 from Y, giving him a threat that if (c) W is liable for negligence.
he didn’t give the money, he would inform 130. Facts: A, a rag-picker, collects plastic
the police. Y gave the money. bags, metal pieces and scraps from the
Issue: What offence, if any, has been com- garbage heaps and areas adjoining fac-
mitted? tories. The City Development Authority
Decision: lodges a complaint of theft of moveable
(a) X is not committed any offence. properties from different parts of the city
(b) Y is guilty of theft. against A.
(c) X has committed the offence of extor- Issue: What offence, if any, has been com-
tion. As he puts Y in fear of injury i.e., mitted?

CLAT.indb 77 31/03/2009 11:41:44


78 CRIMINAL LAW

Decision: movable property from one place to


(a) A is not liable for any offence. another place amounts to theft.
(b) A is liable for theft. (c) X has committed the offence of extor-
(c) City development authority will not tion. She might have concealed the
succeed in their complaint. necklace to blackmail Y.
131. Facts: A group of spirited young persons, 133. Facts: X threatens Z that he will keep Z’s
opposed to animal slaughter, issue a stern child in wrongful confinement unless Z
warning letter to all the butchers in the agrees to leave the village permanently
town to either free all the animals detained and donate his entire property to a village
for slaughter within 24 hours or face dire temple. Z in order to avoid evil conse-
consequences. Scared by the warning the quences of the threat agrees to do so.
butchers free the animals. Issue: What offence, if any, has been com-
Issue: What offence, if any, has been com- mitted?
mitted? Decision:
Decision: (a) There is no extortion, as there is no
(a) The young persons can be prosecuted delivery of property by Z to X. How-
for ‘Criminal Intimidation’ (Section ever, X is guilty under Section 385
503, IPC). According to this section. which punishes the very act of putting
“Whoever threatens another with any or the attempting to put any person in
injury to his person, reputation or fear of any injury for the above pur-
property, with intent to cause alarm pose.
to that person, or to cause that per- (b) X is guilty of extortion.
son to do any act which he is not le- (c) X is guilty of criminal intimidation.
gally bound to do so, as the means of 134. Facts: A obtains property from B by say-
avoiding the execution of such threat, ing, “Your child is in the hands of my
commits criminal intimidation”. The gang and will be killed unless you send us
young persons are not guilty of ‘ex- Rs 5,000.”
tortion’, as there can be no extortion Issue: What offence, if any, has been com-
unless a person is by threat of injury mitted?
induced to deliver any property to the Decision:
culprit. (a) A is guilty of extortion.
(b) The young person are guilty of extor- (b) A is not guilty of extortion.
tion. (c) A is guilty of robbery.
(c) They are innocent people. They are 135. Facts: The accused husband took his wife
not liable for any offence because to a forest and obtained her ornaments un-
they are doing social service. der threats to kill her. The ornaments were
132. Facts: X, an elderly woman, was employed subsequently recovered from him.
as maid servant by Y, a young woman. Y Issue: What offence, if any, has been com-
was very negligent and never bothered mitted?
about her valuables. X being honest and Decision:
sincere advised Y to keep valuables under (a) The accused is guilty of the offence of
lock and key. One day X finding Y’s gold extortion, not robbery.
necklace on the table in an open verandah (b) The accused is guilty of offence of
removed the necklace stealthily and con- robbery.
cealed the same in the cowshed to teach Y (c) The accused is not guilty, because he
a lesson or to cause her a little anxiety. is the husband and he has equal right
Issue: What offence, if any, has been com- over the ornaments.
mitted? 136. Facts: X, a villager rushed to the field
Decision: where victims of an air-crash were lying
(a) X has not committed the offence of dead. X removed gold biscuits and dia-
theft as there was no dishonest inten- mond ring from the pocket of a dead per-
tion on X’s part. son.
(b) X has committed the offence of theft. Issue: What offence, if any, has been com-
Because the mere moving of any mitted?

CLAT.indb 78 31/03/2009 11:41:45


CRIMINAL LAW 79

Decision: (c) A has committed the offence of cheat-


(a) X is guilty of theft. ing.
(b) Removing ornaments from a dead 140. Facts: A borrowed a bicycle from his
body cannot be taking property out friend, B, promising to return the same
of possession of a person and thus not within a period of two or three days. He
a theft, but it is a criminal misappro- failed to fulfill the promise, disposed of
priation under Section 404. the bicycle and appropriated the proceeds
(c) X is not guilty of any offence. to his own use.
137. Facts: A finds B’s briefcase and writes Issue: What offence, if any, has been com-
to him that he will give it on payment of mitted?
Rs 500. B pays Rs 500. Is A guilty of ex- Decision:
tortion? (a) A is guilty of criminal misappropria-
Issue: What offence, if any, has been com- tion.
mitted? (b) A is guilty of criminal breach of trust.
Decision: (c) A is guilty of cheating.
(a) A is not liable for any offence. 141. Facts: X finds a brief case at the railway
(b) A will be liable for extortion. If B station. After trying hard to find the own-
does not pay then A will be liable for er, he brought the brief case to his house.
attempt to commit extortion and also When opened, he found Rs 25000 cash
for criminal misappropriation if he in it with no indication as to whom it be-
does not return the briefcase. longed. He notified in ‘lost and found’ col-
(c) A is liable for criminal misappropria- umn of all the leading English dailies of
tion. the city asking the owner of the ‘lost’ brief
case to collect it from X. After waiting for
CRIMINAL MISAPPROPRIATION about one month, M utilized the money in
AND BREACH OF TRUST purchasing a scooter.
Issue: What offence, if any, has been com-
138. Facts: A, in good faith, believing to be his mitted?
own bag takes the bag which indeed be- Decision:
longs to Z. Later, A discovers the truth but (a) X has committed the offence of crimi-
nevertheless keeps it and does not return it nal misappropriation.
to the owner. (b) No conclusive decision can be drawn.
Issue: What offence, if any, has been com- (c) X hasn’t committed any offence, as he
mitted? didn’t take or misappropriated prop-
Decision: erty dishonestly and used reasonable
(a) A is not guilty of any offence. means to discover the owner.
(b) A is guilty of the offence of criminal 142. Facts: A, a railway reservation clerk,
misappropriation, as he acted dishon- openly declared that he will entertain the
estly in retaining the bag. See illustra- reservation forms of only those passengers
tion (a) of Section 403. who are willing to give an additional sum
(c) A is guilty of attempt to theft. of Rs 25 per ticket as a premium. A sells
139. A is a warehouse keeper. Z, going on a 100 tickets and deposits the total ticket
journey, entrusts his rare furniture to A amount in railway treasury, but pockets
under contract that it shall be returned on the sum of Rs 2500 that he has collected
payment of stipulated sum of money. A from the passengers as premium.
dishonestly sells the furniture. Issue: What offence, if any, has been com-
Issue: What offence, if any, has been com- mitted?
mitted? Decision:
Decision: (a) A is guilty of criminal breach of trust.
(a) A has committed the offence of crimi- (b) A is guilty of criminal misappropria-
nal misappropriation. tion.
(b) A has committed criminal breach of (c) He is not guilty of criminal breach of
trust. See illustration (b) of Section trust. As the premium was intended to
45. be paid to A as his own personal profit

CLAT.indb 79 31/03/2009 11:41:45


80 CRIMINAL LAW

there could be no question of entrust- with the fees an additional amount of Rs 50


ment in such payment. A is guilty of for the New Building Fund. Since there is
receiving illegal gratification. no mention of such fund in the prospectus
143. Facts: A railway booking clerk had in ad- the students pay Rs 50 reluctantly and un-
dition to selling tickets, provided for sale der confusion. However, after collection
of “travel packs” containing soap, oil, of the fund the cashier diverts part of it for
comb, etc. for the benefit of passengers, the construction of his own house, which
purely as a personal business. The railway also has a temple, visited by the students
audit team while checking the accounts during the examination days.
prepared by the clerk found the collections Issue: What offence, if any, has been com-
under three heads: (i) Rs 3,45,765, the sale mitted?
proceeds of 200 tickets; (ii) Rs 557, the Decision:
excess amount paid by the customers by (a) The cashier is guilty of criminal mis-
mistake; and (iii) Rs 1,500, the sale pro- appropriation.
ceed of 30 travel packs. The clerk had only (b) He is guilty of breach of trust.
deposited the amount under head (i) and (c) He is not guilty of any offence.
has pocketed the rest. 146. Facts: Certain sum of money is deposited
Issue: What offence, if any, has been com- in the library cash box by the students as
mitted? late fee for the books kept beyond the pre-
Decision: scribed time. Since no receipt is issued to
(a) The clerk has not committed any of- the students for unascertained amount in
fence. the cash box, Librarian A decides to uti-
(b) No conclusive decision can be drawn. lize the collection for unauthorized beau-
(c) The clerk has committed a criminal tification of the garden and his personal
breach of trust in respect of amount expenses.
under head (ii), as he received the Issue: What offence, if any, has been com-
money on behalf of or as an agent of mitted?
the Government. If he had received Decision:
the money on his own behalf (e.g., (a) As A dishonestly misappropriated the
by way of illegal gratification), then property that was entrusted to him, so
he wouldn’t have been so guilty. That he is liable for dishonest misappro-
is why he is not guilty in respect of priation.
amount under head (iii). (b) He is liable for criminal breach of
144. Facts: X, a parcel clerk at a railway sta- trust.
tion, was asked to destroy a wagonload (c) He didn’t commit any offence as he
of apples which had gone rotten and de- made the best possible use of money.
clared unfit for human consumption. He 147. Facts: A weighing clerk at a sugar mill
took those apples to the ‘mandi’ and auc- was found to be involved in the following
tioned them for Rs 1000 and pocketed the malpractices: (i) Receiving a premium for
amount. out of turn sugarcane delivery facility; (ii)
Issue: What offence, if any, has been com- Overweighing sugarcane and thus causing
mitted? a loss to the company; and (iii) Collection
Decision: from the villages road cess to be deposited
(a) X has not committed any offence. with the government.
Because he was asked to destroy the Issue: What offence, if any, has been com-
apples and he auctioned them using mitted?
his wisdom. Decision:
(b) X has committed criminal breach of (a) The clerk is not liable for criminal
trust. breach of trust in respect of (i) in re-
(c) X misappropriated the property en- spect of (ii) and (iii). Because in (i),
trusted to him and acted in violation he received money on his own behalf,
of legal direction prescribing the mode while in (ii) and (iii) he has put the
in which trust is to be discharged. company in loss and received money
145. Facts: A, the school cashier, collects along on behalf of the company.

CLAT.indb 80 31/03/2009 11:41:45


CRIMINAL LAW 81

(b) The clerk is liable for criminal misap- possession has been handed over.
propriation. The expression ‘entrustment’ carried
(c) The sugar mill is liable for negli- with it the implication that the person
gence. handing over any property to another,
148. Facts: A lost his briefcase containing continues to be its owner. Thus, the
valuables. X, a friend of A informed him jeweller, by appropriating the gold to
that his briefcase was seen lying in Y’s himself, is guilty of criminal breach of
shop. Thereupon A reached Y’s shop and trust. If the jeweller has returned the
finding his briefcase lying there pulled out gold but had failed to make the ring,
his revolver and asked Y to hand over the then it would have been a case of a
briefcase to him. Under the fear of inju- simple breach of contract.
ry, Y gave the briefcase to A. However, (b) It is a breach of contract.
on reaching home he discovered that the (c) It is no offence because there was no
briefcase did not belong to him but decid- contract and there was no trust so it is
ed to retain it. not breach of trust or breach of con-
Issue: What offence, if any, has been com- tract.
mitted? 150. Facts: Z found a wallet at a place con-
Decision: taining a visiting card showing the name
(a) A is guilty of ‘criminal misappro- and address of the owner of the wallet,
priation’ of property because after Rs 2,000 and postal stamps worth Rs 10.
discovering the mistake he instead of Z threw away the card, used the stamps
returning the briefcase to Y, decided and delivered the wallet to his friend R,
to retain it. The retaining by A, thus, for safe custody. R later took out Rs 700
has become wrongful and fraudulent. from the wallet to deposit his examination
A is not guilty of ‘extortion’ because fee. On receipt of a money order from his
when he threatened Y, there was no father R put back the amount of Rs 700 in
dishonest inducement on A’s part as the wallet and returned it to Z.
he believed briefcase to be his own. Issue: What offence, if any, has been com-
(b) A is guilty of extortion. mitted?
(c) A is guilty of robbery. Decision:
149. Facts: A customer went to the shop of a (a) Z is guilty of ‘criminal misappropria-
jeweller and handed over gold for manu- tion’; from the visiting card he knew
facturing a gold ring for his wife and a that the property belonged to whom
week’s time was fixed for delivery. After and despite its legal owner being
the expiry of one week the jeweller re- known he misused the property.
fused to deliver the ring as well as gold. R is guilty of ‘criminal breach of
The jeweller was prosecuted for criminal trust’, as a breach of trust involves
breach of trust. The jeweller argued that it misappropriation and misappropria-
is a breach of contract. Decide. tion or conversion need not be per-
Issue: What offence, if any, has been com- manently, it may even be for a time
mitted? only. (Explanation 1 to Section 403,
Decision: and illustration to it). Further, where
(a) It is a case of ‘criminal breach of trust’ embezzlement by the accused is es-
and not a simple case of a breach of tablished, the fact that he refunded the
contract. The expression ‘entrustment’ amount when his dishonest act came
includes all cases in which property is to be discovered, does not absolve
voluntarily handed over for a specific him of the offence committed by him.
purpose and is dishonestly disposed (b) Z is not guilty of any offence.
of contrary to the terms on which (c) Z is guilty of negligence.

CLAT.indb 81 31/03/2009 11:41:45


82 CRIMINAL LAW

ANSWERS

1. (d) 2. (a) 3. (c) 4. (b) 5. (a) 6. (a) 7. (d) 8. (c) 9. (a) 10. (c)
11. (a) 12. (a) 13. (a) 14. (a) 15. (a) 16. (b) 17. (b) 18. (c) 19. (a) 20. (a)
21. (b) 22. (a) 23. (d) 24. (d) 25. (d) 26. (c) 27. (a) 28. (a) 29. (a) 30. (b)
31. (a) 32. (c) 33. (a) 34. (b) 35. (c) 36. (c) 37. (c) 38. (b) 39. (c) 40. (c)
41. (a) 42. (a) 43. (a) 44. (a) 45. (b) 46. (b) 47. (a) 48. (b) 49. (b) 50. (a)
51. (a) 52. (a) 53. (a) 54. (b) 55. (a) 56. (b) 57. (a) 58. (a) 59. (a) 60. (a)
61. (a) 62. (b) 63. (a) 64. (a) 65. (b) 66. (a) 67. (a) 68. (a) 69. (a) 70. (a)
71. (c) 72. (a) 73. (b) 74. (b) 75. (b) 76. (b) 77. (b) 78. (c) 79. (c) 80. (a)
81. (a) 82. (a) 83. (a) 84. (b) 85. (b) 86. (b) 87. (b) 88. (a) 89. (b) 90. (b)
91. (a) 92. (a) 93. (b) 94. (b) 95. (c) 96. (c) 97. (c) 98. (d) 99. (a) 100. (a)
101. (a) 102. (a) 103. (a) 104. (a) 105. (a) 106. (b) 107. (b) 108. (b) 109. (b) 110. (b)
111. (c) 112. (c) 113. (c) 114. (c) 115. (c) 116. (a) 117. (a) 118. (a) 119. (a) 120. (a)
121. (b) 122. (b) 123. (b) 124. (b) 125. (b) 126. (c) 127. (c) 128. (c) 129. (a) 130. (a)
131. (a) 132. (a) 133. (a) 134. (a) 135. (a) 136. (b) 137. (b) 138. (b) 139. (b) 140. (b)
141. (c) 142. (c) 143. (c) 144. (c) 145. (a) 146. (a) 147. (a) 148. (a) 149. (a) 150. (a)

CLAT.indb 82 31/03/2009 11:41:45


Chapter 3
CIVIL LAW
CONTRACT AND AGREEMENT It has been noted above that an agreement
enforceable by law is a contract. All such agree-
Contract ments which satisfy the conditions mentioned in
Section 10 of the Act are contracts.
According to Section 2(h) of the Indian Contract
Act, “An agreement enforceable by law is a Section 10 is as under:
contract.” “All agreements are contracts if they are made
All agreements are not enforceable by law by the free consent of parties competent to con-
and therefore, all agreements are not contracts. tract, for a lawful consideration and with a law-
Some agreements may be enforceable by ful object and are not hereby expressly declared
law and others might not be. For example, an to be void.”
agreement to sell a radio set may be a contract, The essentials needed for a valid contract,
but an agreement to go to see a movie may be a therefore, are as under:
mere agreement not enforceable by law. 1. An agreement between the two parties.
Thus all agreements are not contracts. An agreement is the result of a proposal
Only those agreements which satisfy the essen- or an offer by one party followed by its
tials mentioned in section 10 become contracts. acceptance by the other.
However, all contracts are agreements. 2. Agreement should be between the parties
who are competent to contract.
Agreement 3. There should be a lawful consideration and
According to Section 2(e): “Every promise and lawful object in respect of that agreement.
every set of promises forming the consideration 4. There should be free consent of the parties,
for each other is an agreement.” when they enter into the agreement.
In an agreement there is a promise from 5. The agreement must not be one, which has
both sides. For example, A promises to deliver been expressly declared to be void.
his watch to B and in return B promises to pay Proposal or Offer
a sum of Rs 2,000 to A, there is said to be an
agreement between A and B. The term ‘proposal has been defined in section
A promise is a result of an offer (proposal) 2(a) as follows:
by one person and its acceptance by the other. “When one person signifies to another
his willingness to do or to abstain from doing
For example, when A makes a proposal to sell
anything, with a view to obtaining the assent of
his watch to B for Rs 2,000 and B accepts his
that other to such act or abstinence, he is said to
proposal, there results a promise between the
make a proposal.”
two persons.
Acceptance
Section 2(b) of the Act defines promise as
under: According to Section 2(b): When the person to
whom the proposal is made signifies his assent
“When the person to whom the proposal is made thereto, the proposal is said to be accepted. A
signifies his assent thereto, the proposal is said proposal, when accepted, becomes a promise.”
to be accepted. A proposal, when accepted, be-
comes a promise.” Essentials of a valid acceptance
Thus, when there is a proposal from one In order that acceptance of an offer results
side and the acceptance of that proposal by the in a contract, the acceptance must satisfy the
other side, it results in a promise. This promise following requirements:
from the two parties to one another is known as 1. Acceptance should be communicated by
an agreement. the offeree to the offeror.

CLAT-3-Civil Law.indd 83 01/09/2010 17:29:01


84 CIVIL LAW

2. Acceptance should be absolute and Although the above stated categories of


unqualified. persons are not competent to contract, yet they
3. Acceptance should be made in some usual may sometimes be making some bargains, tak-
and reasonable manner, unless the proposal ing some loans, or be supplied with some goods
prescribes the manner of acceptance. by third parties, or be conferred with some ben-
4. Acceptance should be made while the efits, etc.
offer is still subsisting.
Free Consent
Consideration What is free consent?
Section 2 (d), Indian contract Act defines One of the essentials of a valid contract
Consideration as under: mentioned in Section 10 is that the parties should
“When, at the desire of the promisor, the prom- enter into the contract with their free consent.
isee or any other person has done or abstained According to Section 14, consent is said to
from doing, or does or abstains from doing, or be free when it is not caused by
promises to do or to abstain from doing some- 1. coercion, as defined in Section 15, or
thing, such act or abstinence or promise is called 2. undue influence, as defined in section
a consideration for the promise.” 16, or
The definition requires the following es- 3. fraud, as defined in Section 17, or
sentials to be satisfied in order that there is valid 4. misrepresentation, as defined in Sec-
consideration: tion 18, or
1. Consideration to be given ‘at the de- 5. mistake, subject to the provisions of
sire of the promisor’. Sections 20, 21 and 22.
2. Consideration to be given ‘by the Consent is said to be so caused when it
promisee or any other person’. would not have been given but by the existence
3. Consideration may be past, present or of such coercion, undue influence, fraud, mis-
future, in so far as definition says that representation or mistake.
the promise: If the consent of one of the parties is not
(i) has done or abstained from doing, free consent, i.e., it has been caused by one or
or the other of the above stated factors, the contract
(ii) does or abstained from doing, or is not a valid one. When consent to an agreement
(iii) promises to do or to abstain from is caused by coercion, undue influence fraud or
doing, something. misrepresentation, the agreement is a contract
4. There should be some act, abstinence voidable at the option of the party whose con-
or promise by the promise, which con- sent was so caused. If, however, the consent is
stitutes consideration for the promise. caused by mistake, the agreement is void.

Capacity to Contract Legality of Object and Consideration


One of the essentials of a valid contract, One of the essentials of a valid contract is that
mentioned in Section 10, is that the parties to the consideration and the object should be
the contract should be competent to make the lawful. Every agreement of which the object or
contract. consideration is unlawful is void.
Section 23 mentions the circumstances
According to Section 11: when the consideration or object of an agree-
“Every person is competent to contract who is ment is not lawful. The section reads as under:
of the age of majority according to the law to What consideration and objects are lawful,
which he is subject, and who is of sound mind, and what not.
and is not disqualified from contracting by any The consideration or object of an agreement is
law to which he is subject.” lawful, unless:
It means that the following three categories it is forbidden by law, or
of persons are not competent to contract: is of such a nature that, if permitted,
1. A person who has not attained the age it would defeat the provisions of law;
of majority, i.e., one who is a minor. or
2. A person who is of unsound mind. is fraudulent; or
3. A person who has been disqualified involves or implies injury to the per-
from contracting by some law. son or property of another,

CLAT.indb 84 31/03/2009 11:41:45


CIVIL LAW 85

or the Court regards it as immoral, or agrees for money, without the knowledge
opposed to public policy. of his principal, to obtain for B a lease
In each of these cases, the consideration or of land belonging to his principal. The
object of an agreement is said to be unlawful. agreement between A and B is void, as it
Every agreement of which the object or consid- implies a fraud by concealment by A, on
eration is unlawful is void. his principal.
(h) A promises B to drop a prosecution which
Illustrations
he has instituted against B for robbery,
(a) A agrees to sell his house to B for 10,000 and B promises to restore the value of the
rupees. Here B’s promise to pay the sum of things taken. The agreement is void, as its
10,000 rupees is the consideration for A’s object is unlawful.
promise to sell the house, and A’s promise (i) A’s estate is sold for arrears of revenue
to sell the house is the consideration for under the provisions of an Act of the
B’s promise to pay 10,000 rupees. These Legislature, by which the defaulter is
are lawful considerations. prohibited from purchasing the estate, B,
(b) A promises to pay B 1,000 rupees at the upon an understanding with A, becomes
end of six months if C, who owes the sum the purchaser, and agrees to convey the
to B fails to pay it. B promises, to grant estate to A, upon receiving from him the
time to C, accordingly. Here the promise price which B has paid. The agreement is
of each party is the consideration for the void, as it renders the transaction in effect,
promise of the other party, and they are a purchase by the defaulter, and would so
lawful considerations. defeat the object of the law.
(c) A promises for a certain sum paid to him (j) A, who is B’s mukhtar, promises to
by B to make good to B the value of his exercise his influence, as such with B in
ship if it is wrecked on a certain voyage. favour of C and C promises to pay 1,000
Here, A’s promise is the consideration rupees to A. The agreement is void because
for B’s payment, and B’s payment is the it is immoral.
consideration for A’s promise and these (k) A agrees to let her daughter to hire to B
are lawful considerations. for concubinage. The agreement is void,
(d) A promises to maintain B’s child, and because it is immoral, though the letting
B promises to pay 1,000 rupees yearly may not be punishable under the Indian
for the purpose. Here, the promise of Penal Code.
each party is the consideration for the
In the following cases, the consideration
promise of the other party. They are lawful
or object has been considered to be unlawful by
considerations.
Section 23:
(e) A, B and C enter into an agreement for the
1. It is forbidden by law
division among them of gains acquired, or
to be acquired, by fraud, the agreement is 2. It would defeat the provisions of any law
void, as its object is unlawful. 3. It is fraudulent
(f) A promises to obtain for B an employment 4. It involves or implies injury to the person
in the public service, and B promises to pay or property of another
1,000 rupees to A. The agreement is void, 5. The Court regards it as immoral or
as the consideration for it is unlawful. 6. The Court regards it as opposed to public
(g) A being an agent for a landed proprietor, policy

CLAT.indb 85 31/03/2009 11:41:45


86 CIVIL LAW

MULTIPLE-CHOICE QUESTIONS
1. Fact: Mr Mukesh invites Mr Vijay to a (b) The seller are not liable as there was
dinner at a hotel. Mr Vijay accepts the no contract as the parties never in-
invitation. It is purely a social agreement. tended to create legal relationship be-
Mr Vijay fails to arrive at the dinner or cause they had clearly mentioned that
Mr Mukesh has to go out and is not avail- their agreement is not a legal docu-
able at his place at the dinner time due to ment nor a memorandum nor subject
some important work. to the jurisdiction of law courts.
Issue: Is Mr Vijay liable for the non-per- (c) The firms may make mutual adjudica-
formance of contract with Mukesh. tion.
Decision: 4. Fact: A who owns two cars, one Maruti
(a) Vijay is liable because he has not per- and the other Santro, offers to sell B one
formed the contract. car. A intending it to be the Maruti car. B
(b) Vijay is not liable because there was accepts the offer thinking that it is the San-
no contract, it was just a social agree- tro.
ment. Issue: Is A liable?
(c) It will be the discretion of the court. Decision:
(a) A is not liable because there is no con-
2. Fact: Mr Balfour was employed in Cey- sensus and hence no contract.
lon. Mrs Balfour owing to ill health, had (b) A is liable because once he had of-
to stay in England and could not accom- fered the contract is complete.
pany him to Ceylon. Mr Balfour prom- (c) It is upto the court to decide the liabil-
ised to send her £ 30 per month while he ity.
was abroad. But Mr Balfour failed to pay 5. Fact: A, a person of weak intelligence
that amount. So Mrs Balfour filed a suit made a gift of his entire property to B,
against her husband for recovering the who was in a position to dominate him.
said amount. The gift having been obtained by undue
Issue: Is Balfour liable for breach of con- influence is voidable at the option of A.
tract? Issue: Is this contract valid?
Decision: Decision:
(a) Mr Balfour is not liable because the (a) The contract is valid as A voluntarily
intention not to create a legal obliga- made a gift.
tion was clear from the conduct of the (b) The contract is not valid because A
parties. made a gift under undue influence so
(b) Mr Balfour is liable for breach of con- it is voidable at the option of A.
tract. (c) It is void abinitio.
(c) No conclusive decision can be drawn 6. Fact: S, intending to deceive Y, falsely
as it is a private affair of husband and represents that 500 maunds of indigo are
wife. made annually at X’s factory and thereby
3. Fact: Two firms entered into a written induces Y to buy the factory.
contract for the sale and purchase of tissue Issue: Is it a valid contract?
paper. The agreement contained a clause Decision:
to the effect that “this agreement is not en- (a) It is a valid contract.
tered into, nor is this memorandum writ- (b) It is a contract voidable at the option
ten, as a formal or a legal document, and of Y because he has been falsely mis-
shall not be subject to legal jurisdiction in representated.
the law courts”. Since the goods were not (c) It is void abinitio.
delivered, the buyers brought an action for 7. Fact: X borrows Rs 50,000 from Y for the
non-delivery. purpose of smuggling goods. Y knows of
Issue: Are the sellers liable? the purpose of the loan.
Decision: Issue: Is this agreement valid?
(a) The sellers are liable as they had en- Decision:
tered into a written agreement. (a) The agreement is valid.

CLAT.indb 86 31/03/2009 11:41:45


CIVIL LAW 87

(b) The agreement between X and Y Decision:


is collateral to the main agreement (a) Yes, there is a valid contract and B
which is illegal so the agreement is will be liable if the horse didn’t prove
also illegal because anything done lucky.
which is unlawful, immoral against (b) There is no valid contract as the terms
public policy can never result into a of contract are so vague and loose.
valid agreement. (c) It is voidable contract at the option of
(c) Nothing conclusive can be said. A.
8. Fact: The Carbolic Smoke Ball Co. of- 11. Fact: D sent his servant P to trace his
fered by advertisement, a reward of £ 100 missing nephew. D in the meantime an-
to any person who should contract influen- nounced a reward for providing informa-
za after having used the smoke-ball three tion about the missing boy. P, in ignorance
times daily for two weeks according to the of the announcement traced the boy and
printed directions. It also added that £ 100 informed D. P later on came to know and
have been deposited in the bank showing he claimed it.
its sincerity in the promise. The plaintiff Issue: Is the servant entitled to reward?
Mrs Carlill used the smoke-ball according Decision:
to the directions to the company but con- (a) Servant is entitled to claim.
tracted influenza. (b) Servant is not entitled to claim be-
Issue: Is company liable to give the re- cause he was ignorant of the offer of
ward and Mrs Carlill is entitled to award? reward so there was no agreement be-
Decision: cause there was no acceptance.
(a) Yes, company is liable. Mrs Carlill is (c) The contract is voidable at the option
of the servant.
entitled to the reward because the per-
12. Fact: N advertised in the newspaper to ef-
formance of the conditions is a suffi-
fect sale of his goods on a particular day at
cient acceptance without notification.
a particular place. H travelled a long dis-
(b) Company is not liable.
tance to bid for the things. On arrival, he
(c) Mrs Carlill is not entitled because
found that the sale was cancelled. He sued
there is no valid contract.
N for the breach of contract.
9. Fact: A agreed to take B’s house on rent
Issue: Is N liable?
for three years at the rent of £ 85 per an- Decision:
num provided the house was put to thor- (a) N is not liable because advertisement
ough repair and the drawing rooms were was merely expression of an inten-
decorated “according to present style”. tion and not an offer which could be
Issue: Is there a valid contract between A accepted by travelling to the place of
and B? intended sale.
Decision: (b) N is liable for all the expenses in-
(a) There is not valid contract because curred by H for traveling such a long
it is a vague term, because the term distance.
“present style” may mean one thing to (c) It is voidable at the option of H.
A and another to B. Hence, the agree- 13. Fact: T, who could not read, took an ex-
ment was void on the ground that the cursion ticket on the railway. On the front
terms of offer were vague and uncer- of railway ticket was written that the com-
tain. pany would not be liable for personal in-
(b) There is a valid contract because there juries to passengers. T was injured by a
is an offer from the side of A and ac- railway accident.
ceptance from the side of B. Issue: Is the railway company liable?
(c) It is voidable contract at the option of Decision:
A. (a) Yes railway company is liable because
10. Fact: A purchased a horse from B and the traveler could not read.
promised that if the horse was lucky to (b) Railway company is not liable be-
him, he would give Rs 50 more or he cause T was bound by the conditions
would purchase another horse. and could not recover any damages.
Issue: Is there a valid contract between A (c) The contract is voidable at the option
and B? of T.

CLAT.indb 87 31/03/2009 11:41:45


88 CIVIL LAW

14. Fact: M delivered one new saree to a asking whether his offer was accepted.
laundry for washing. On the back of the He also added that he was ready to accept
printed receipt it was stated that the cus- any higher price if found reasonable. The
tomer would be entitled to recover only agent replied, “would not accept less than
15% of the market price of the article in Rs 10,000”. D accepted this and brought a
case of loss. The saree was lost owing to suit for specific performance.
the negligence of the laundry. Issue: Is P liable for specific perfor-
Issue: Is M entitled to claim the loss? mance?
Decision: Decision:
(a) M is entitled to claim the loss and the (a) P is liable for the specific performance
laundry shop is liable. of the contract entered into D and P.
(b) M is not entitled because the condi- (b) P is not liable because there is only
tion was printed on the back of the offer or counter offer but no formal/
receipt. legal offer and acceptance. Hence no
(c) Neither (a) nor (b) valid contract, hence P not liable.
15. Fact: A writes to B, “I offer to sell my (c) The contract is voidable at the option
house for Rs 40,000. If I do not receive a of D.
reply by Monday next, I shall assume that 18. Fact: A offered to sell a farm for £ 1,000,
you have accepted the offer”. X said he would give £ 950. A refused and
Issue: Is there a valid contract between A X then said he would give £ 1,000, and
and B? when A declined to adhere to his original
Decision: offer tried to obtain specific performance.
(a) Yes, there is a valid contract. Issue: Is there a valid contract between A
(b) No, there is no valid contract because and X?
A made a offer but didn’t accept it. Decision:
(c) Because A had made an offer so the (a) There is no valid contract as X’s offer
contract is voidable at the option of to pay £ 950 is a refusal of the offer
A. and a counter offer.
16. Fact: Harvey sent a telegram to Facey (b) There is a valid contract and A is li-
stating “will you sell us the Estate of Bum- able for specific performance.
(c) It is voidable contract at the option of
ber Hall Pen for £ 900”?
X.
Harvey sent another telegram to Facey
19. Fact: P bought a steamer ticket on the face
stating ‘We agree to buy Bumper Hall
of which were these words only, “Dublin
Pen for sum of £ 900 asked by you. Please
to Whitehaven.” On the back were printed
send us your title deeds in order that we
certain conditions one of which excluded
may get early possession.’
the liability of the company for loss, injury
But Facey did not send any reply to the
or delay to the passenger or his luggage. P
last telegram sent by Harvey. Hence, Har-
had not seen the back of the ticket, nor was
vey filed a case against Facey claiming the
there any indication on he face about the
Bumber Hall Pen Estate.
conditions on the back. P’s luggage was
Issue: Is there valid contract between Har-
lost on the way due to the negligence of
vey and Facey? the company’s servants.
Decision: Issue: Is the company liable?
(a) Yes, there is a valid contract because Decision:
Harvey made an offer for three times (a) Company is not liable because com-
and Facey didn’t respond which pany had printed the conditions on the
amounts to his acceptance. back of the ticket that company is not
(b) There is no valid contract because liable for any loss, injury or delay.
there is no acceptance of the offer. (b) Company is liable because it was
(c) It is voidable contract at the option printed on the front side that the con-
of Harvey because Harvey had made ditions are printed on the back side.
repeated offers. (c) Company is not liable because it is the
17. Fact: D offered to purchase a lodge owned duty of the traveller to take care of its
by P for Rs 6,000. He wrote to P’s agent luggage.

CLAT.indb 88 31/03/2009 11:41:46


CIVIL LAW 89

20. Fact: P deposited his bag at the clock the condition limiting the company’s
room at a railway station and received a liability.
ticket containing on its face, the words “see (c) It will be at the discretion of the
back.” On the back of the ticket there was court.
a condition that, “the company will not be 23. Fact: P and her husband hired a room at
responsible for any package exceeding the a hotel and paid a week’s rent in advance.
value of £ 10.” A notice to the same effect When they went up to occupy the room
was hung up in the cloak room. P’s bag there was a notice on one of the walls dis-
was lost and he claimed the actual value claiming the owners liability for damage,
of the lost bag, £ 24, 10 s. The claim was loss or theft of articles in the room. A thief
negatived and only £ 10 was awarded. entered the room due to the negligence of
Issue: Is the railway company liable? the hotel servants.
Decision: Issue: Is the owner of the hotel liable?
(a) The railway company is not liable. Decision:
(b) The railway company is liable. (a) No, he is not liable because it is clearly
(c) Neither (a) nor (b) written on the wall that hotel authori-
21. Fact: P purchased from D company, a ties will not be liable damage, loss or
steamer ticket containing conditions print- theft.
ed in the French Language. At the foot of (b) Owner of the hotel is liable since the
the ticket, there was an intimation in red notice was not a part of the agreement
letters that the ticket was issued subject to as it came to the knowledge of the cli-
the conditions printed on the back. One of ent after the contract had been entered
these conditions was that the company in- into.
curred no liability for any damage which (c) The decision will be in the hands of
the luggage might sustain. The vessel was client because it is a voidable contract
wrecked by the fault of the company’s ser- at the option of the suffering party.
vants and P’s baggage was lost. P sued D 24. Fact: D wrote to P on 28 November
for damages and claimed that he was not 1971, offering to sell 800 tonnes of iron at
bound the conditions being unable to read Rs 6900 per ton. On the same day P wrote
French. to D offering to buy 800 tonnes of iron
Issue: Is the company liable? Rs 6900 per ton. The two letters crossed
Decision: in post and neither of them knew anything
(a) Company is not liable because P had about the offer to the other. P contended
reasonable notice of the conditions that there was a good contract.
and it was his own fault if he had not Issue: What is the nature of contract and
made himself acquainted with them. liabilities of the parties?
(b) Company is liable because company Decision:
had used the language which was not (a) There was no valid contract because
easily understandable to everybody. an agreement results into a contract
(c) Neither (a) nor (b) when there is an offer from one side
22. Fact: T, an illiterate lady, took a ticket for and its acceptance from the other side
a journey from a railway company. On the so no question of any liability rises.
face of the ticket were the words, “for con- (b) It is a voidable contract at the option
ditions see back.” One of the conditions of D because D had offered to sell the
absolved the railway company from liabil- product.
ity for personal injuries to passenger. (c) It is a voidable contract at the option
Issue: Is company liable? of P because P had offered to buy the
Decision: product.
(a) Yes, company is liable because the 25. Fact: D sent an offer to a firm with whom
lady was illiterate so she is entitled to he had accounts. P who had just taken over
damages. the said firm got the letter addressed to the
(b) Company is not liable because T old firm, accepted the offer and sent the
could not recover damages for the goods. P sued for the price of the goods.
injury received as she was bound by The court held that there was no contract

CLAT.indb 89 31/03/2009 11:41:46


90 CIVIL LAW

since the order was to the held firm and the resolved to appoint P as headmaster. One
acceptance was by the new firm. of the managers, in his individual capacity,
An offer may also be made to the world informed P of the same. But there was no
at large, as the instance by an advertisement formal communication of the resolution
in the newspaper. In such a case only person by the board. The board later rescinded
or persons with notice of the offer can come the resolution.
forward and accept the offer. Issue: What is the nature of contract and
Issue: Are the claims of P genuine? liability of the board of managers?
Decision: Decision:
(a) D is liable for all the claims made by (a) There is no contract as there is no
P because D was an old customer of communication from the side of board
the firm. of managers.
(b) As there is no contract between P and (b) There is a valid contract as one of the
D so no liability. managers had informed in his indi-
(c) The contract is voidable at the option vidual capacity.
of D. (c) Neither (a) nor (b)
26. Fact: M offered to sell land to N at $ 280. 29. Fact: The manager of a railway company
N replied accepting the offer and encloing received an offer by a letter relating to the
$ 30 and promising to pay balance amount supply of coal; he wrote on the letter ‘ac-
by monthly instalments of $ 50 each. cepted’ and kept it in his drawer and forget
Since N accepted the offer subject to mak- all about it.
ing payments in instalments. Issue: What is the nature of contract?
Issue: Is there a valid contract? Decision:
Decision: (a) Once it has been written “ACCEPT-
(a) It is not a valid contract because the ED”, it means there is a valid contract
acceptance is conditional and quali- and the coal company can sue the rail-
fied. It is a voidable contract at the op- way company.
tion of M who had made an offer and (b) There is no valid contract because
N sent the conditional acceptance. there is no communication of the ac-
(b) It is a valid contract as M had made ceptance.
(c) It is a voidable contract at the option
the offer and N accepted it.
of the railway company.
(c) It is a contract void abinitio.
30. Fact: An offer was made in the follow-
27. Fact: F offered to buy his nephew’s horse
ing terms. “I intend to sell my house for
for £ 30 15 s, adding, “If I hear no more,
Rs 1,000. If you are willing to have it,
I shall consider the horse as mine at £ 30
write to F at his address”. Instead of writ-
15 s.” The nephew did not reply, but told
ing to F, the purchaser sent an agent to F
his auctioneer not to sell the horse, as it
and agreed to purchase.
was sold to his uncle. But the auctioneer
Issue: What is the nature of contract and
sold it by mistake to a third party. F sued
liability of the seller?
for conversion of his property.
Decision:
Issue: What is the liability of nephew?
(a) It is a valid contract because the pur-
Decision: chaser has communicated his accep-
(a) Nephew is liable as his action of tell- tance in person.
ing his auctioneer not to sell the horse (b) It is not a valid contract because seller
as it was sold to his uncle had resulted had clearly written that if the offer is
into a valid contract. acceptable then the acceptance should
(b) Nephew is not liable because the mere be communicated through writing.
mental acceptance and uncommuni- (c) It is a contract void ab initio.
cated acceptance does not result into 31. Fact: A sold his business to his manager B
contract. without disclosing the fact to his customer
(c) It is a voidable contract at the option C, who had a running account with A, sent
of F and to determine the liability of an order for the supply of goods to A by
nephew is the discretion of the court. name. B received the order and executed
28. Fact: The board of managers of a school the same. C refused to pay the price.

CLAT.indb 90 31/03/2009 11:41:46


CIVIL LAW 91

Issue: What is the nature of contract and cause revocation of offer is complete
liability of C? as against A when the telegram is dis-
Decision: patched.
(a) As there was no contract between B (b) Revocation of offer is not valid be-
and C because C never made any of- cause it is not complete.
fer to B and as such C was not liable (c) Neither (a) nor (b)
to pay the price to B. 35. Fact: Jones had applied for shares of
(b) There was a valid contract though in- Bank, on 26 October he withdrew his of-
direct and C is liable. fer. The letter revoking the offer was re-
(c) It is the discretion of the court to de- ceived by the company at 8.30 A.M. on
termine the validity of contract. 27th October but before that, the bank and
32. Fact: A landlord served a notice on the already made the allotment at 7.30 A.M.
tenant demanding enchancement of rent. When Jones refused to take the shares of
The tenant did not protest against it and the company, bank filed a suit for specific
continued to occupy the premises. The performance of the contract.
conduct of the tenant amounts to accep- Issue: What is the liability of Jones?
tance of the offer to pay the rent at a higher Decision:
rate. (a) Mr Jones is not bound to accept the
Issue: What is the liability of tenant? shares because offer was revoked be-
Decision: fore the letter of allotment was actu-
(a) The tenant is liable to pay the higher ally posted.
rent. (b) Mr Jones is liable
(b) Tenant is not liable because tenant did (c) It is a case of fraud by Mr Jones with
not send his acceptance of enhanced the company.
rent. 36. Fact: At an auction sale, A made the high-
(c) The tenant is liable to be evacuated. est bid for B’s goods. He withdrew the bid
33. Fact: D, in an answer to enquiry as to the before the fall off the hammer. B knocked
price of pig iron wrote to H, “We shall be down the goods in favour of A. B sued A
glad to supply you with 2000 tonnes of pig for the price of goods.
iron at 65s per ton”, and after further cor- Issue: Is A liable for the price of goods?
Decision:
respondence wrote on the 28th January,
(a) A is liable for the price of goods as
that the price was 65s net. He received this
well as damages cost to B.
letter on the 30th January and on the same
(b) A is not liable as A’s bid was no more
day wrote “We will take the 2000 tonnes
than offer and he was entitled to with-
pig iron you offer us”. The post was then
draw the same before it was accepted.
delayed and the acceptance was received
(c) Neither (a) nor (b)
six hours later than the scheduled hour.
37. Fact: D offered to sell wool to H on
Issue: What is the liability of D?
Thursday and agreed to give him three
Decision:
days time to accept. H accepted the offer
(a) D is not liable because he did not re-
on Monday, but by that time D had sold
ceive the acceptance.
the wool.
(b) D is liable because once the letter of Issue: What is the nature of contract and
acceptance is posted, the process of liability?
acceptance is complete and unquali- Decision:
fied so D is liable. (a) There was a valid contract and D is
(c) It is a contract voidable at the option liable
of D. (b) As there was no valid contract so D is
34. Fact: A proposes by letter, to sell a house not liable.
to B at a certain price. A revokes his pro- (c) It is a contract voidable at the option
posal by telegram. of the H.
Issue: Whether the revocation of an offer 38. Fact: A offers to sell his house to B for
is valid? Rs 1,000. B replies offering to pay Rs 950.
Decision: A refuses. Subsequently B writes accept-
(a) Yes, revocation of offer is valid be- ing the original offer.

CLAT.indb 91 31/03/2009 11:41:46


92 CIVIL LAW

Issue: Is the offer sustaining or lapsed? cause D who made the promise gained
Decision: nothing in return for the promise
(a) The offer is still sustaining and A is made. The secretary of the commit-
liable to sell his house. tee to whom the promise was made
(b) Once B has made a counter offer, he suffered nothing as nothing had been
has no right to make a subsequent ac- done to carry out the repairs.
ceptance, so there is no offer, no ac- (b) D is liable because he had promised
ceptance, no contract and no liability (c) D is liable because the promise made
because the offer has already lapsed. by D was duly accepted.
(c) B can compel A for specific perfor- 42. Fact: X, Y and Z enter into an agreement
mance of the contract. under which X pays Rs 4,00,000 to Y and
39. Fact: A young boy ran away from his fa- Y agreed to build a house for Z.
ther’s home. The father issued a pamphlet Issue: Is the contract enforceable?
offering a reward in these terms. “Any- Decision:
body who finds a trace of the boy and (a) Contract is not enforceable because Z
brings home will get Rs 500”. P was at the is not a party to the contract.
Dharmsala of a railway station. There he (b) Here Z is a party to the contract but
saw a boy, overheard a part of the conver- stranger to consideration and can en-
sation, realized that this boy was missing force the contract.
boy and promptly took him to the railway (c) Neither (a) nor (b)
police station, where he made a report and 43. Fact: A lawyer gave up his practice and
sent a telegram to the boy’s father saying served as manager of a land-owner at the
that he had found his son. latter’s request in lieu of which the land
Issue: Is P entitled to the reward? owner subsequently promised a pension.
Decision: Issue: What is the nature of the contract?
(a) Yes, P is entitled to reward because Decision:
the mere performance of any condi- (a) There was a valid contract as there
tion of the contract results into the was a good past consideration.
entering into valid contract. (b) Such vague promises do not result
(b) P is not entitled because he was not into a contract.
aware of the offer.
(c) Contract is voidable at the option of
(c) P is not entitled because he did not
land-owner.
send his acceptance.
44. Fact: P rendered services to D during his
40. Fact: D promised to pay P a commis-
minority at D’s request, which were con-
sion on articles sold by him in a bazaar in
tinued after D ceased to be a minor. After
which he occupied a shop in consideration
attaining majority D promised to pay an
of P having expended money in the con-
annuity to P for the services rendered in
struction of such bazaar. The money had
the past.
not been spent by P at the request of D but
Issue: What is the nature of contract and
was spent by him at the desire of the Col-
whether P can recover the money?
lector of the District.
Decision:
Issue: What is the nature of contract?
Decision: (a) It was a good contract and P can re-
(a) It was a valid contract and P is liable. cover the money.
(b) There was no consideration and hence (b) It was not a valid contract because the
no contract, no liability. contract with the minor is void ab ini-
(c) Neither (a) nor (b) tio.
41. Fact: D promised Rs 500 to a fund P for (c) It is voidable contract at the option of
re-building a Mosque. Nothing had been minor.
done to carry out repairs and reconstruc- 45. Fact: Due to B’s negligent driving, a lady
tion. The secretary of the mosque filed a was injured. She signed a release agree-
suit for the recovery of Rs 500. ment with insurance company of all her
Issue: What is the liability of D? claims for just $ 21,000. She was not held
Decision: to be bound by the agreement due to in-
(a) The promise was not enforceable be- adequate amount of earlier claim and in-

CLAT.indb 92 31/03/2009 11:41:46


CIVIL LAW 93

equality of her bargaining power with pro- surable interest in the life of M. Thus
fessional insurance company. the contract of insurance was in the
Issue: Will she be entitled to her claim for nature of a wagering contract and thus
subsequent damages? void.
Decision: (b) It is a valid contract and the company
(a) Yes, law does not debar her. is liable.
(b) She cannot, because law of estoppel (c) Neither (a) nor (b)
applies against her. 48. Fact: A promises to obtain for B, an
(c) Neither (a) nor (b) employment in the public service and B
46. Fact: A borrowed Rs 50,000 at 12 per promises to pay Rs 1,000 to A.
cent per annum interest from B, repayable Issue: What is the nature of contract?
on demand. B made the demand and on Decision:
failure of A to pay back, he was about to (a) It is a valid contract.
sue A to get back his money. A requested (b) The agreement is void as the consider-
B to postpone the filing of suit and agreed ation for it is unlawful.
to pay interest at the rate of eighteen per (c) It is a voidable contract.
cent per annum. On refusal to pay interest 49. An old lady, Laxmi Rani gifted her prop-
at 18 per cent B filed a suit. erty to her own daughter Ramayya, with
Issue: Whether B will succeed in his the direction to pay a certain sum of mon-
suit? ey annually to Chinayya, her maternal un-
Decision: cle. On the same day Ramayya executed
(a) No, he will not succeed because he an agreement with Chinayya agreeing to
himself offered to pay 18% interest pay the amount annually. Later, Ramayya
per annum.
refused to honour the agreement on the
(b) He will succeed in his suit because he
ground that there is no consideration. Chi-
was forced to pay 18% under undue
nayya sued for the recovery of the annu-
influence which vitiates the validity
ity.
of contract because one of the basic
Issue: What is the nature of contract?
essential features of a valid contract is
Decision:
that it should be entered into, with free
(a) There was sufficient consideration
consent.
i.e., the property given to her by the
(c) It is a voidable contract so B will not
sister of Chinayya so Ramayya was li-
succeed.
47. Fact: On 15 January 1951 M, the de- able to pay.
ceased uncle of A made an application for (b) There was no agreement between
insurance for Rs 35,000. In that applica- Chinayya and Ramayya so Ramayya
tion, M stated his occupation as landlord is not liable.
and mill-owner. On the basis of the appli- (c) There is no contract.
cation, the company issued a policy to M 50. Fact: An arrangement was made between
on 19 January 1951. In the policy, A was the fathers of a groom and a bride before
mentioned as nominee. The first two quar- their marriage that each would pay a cer-
terly premiums on the policy were paid by tain sum of money to the husband. Father
A. M died on the night of 19th and 20th of the boy paid the promised amount but
August 1951, and the notice of his death, the bride’s father failed to pay the amount.
and that of the claim was sent to the com- The boy sued his father-in-law for the re-
pany on 15 September 1951. After making covery of the promised amount of money.
inquiries, the insurance company refused Issue: Whether the boy can recover the
claim. A brought a suit for the recovery of money.
Rs 35,000. Decision:
Issue: What is the nature of contract? (a) He cannot recover the amount.
Decision: (b) He can recover because they had en-
(a) The policy in question was effected tered into a contract before the mar-
by A in the name of M for his own use riage.
and benefit and further A had no in- (c) Neither (a) nor (b)

CLAT.indb 93 31/03/2009 11:41:46


94 CIVIL LAW

51. Fact: A sold a large quantity of tyres to B ecuted a mortgage in his favour. He be-
at a certain price on entering into a cov- came a major a few months later and filed
enant not to sell the tyres below the price a suit for the declaration that the mortgage
mentioned in price list supplied by A. B executed by him during his minority was
sold the tyres to C a retail dealer under a void and should be cancelled.
contract stipulating the same covenant as Issue: Is the agreement void or valid?
between A and B. C sold the tyres at a Decision:
price less than the listed price. A sued C (a) The agreement is valid and the minor
for the breach. is not entitled to the repayment of
Issue: Can A sue C? money.
Decision: (b) A mortgage by a minor was void and
(a) Yes, A can because A had contract B was not entitled to repayment of
with B and C automatically becomes money.
a third party. (c) It is a discretion of the court.
(b) A could not sue C as A was not a party 55. Fact: In order to pay-off the promissory
to the contract between B and C. note and the mortgage debt of his father,
(c) Neither (a) nor (b) the minor son and his mother sold a piece
52. Fact: U was appointed by his father as his of land to the holders of the promissory
successor and was put in possession of his note in satisfaction of the note and mort-
entire property. In consideration thereof U gage debt. Later the minor brought an ac-
agreed with his father to pay a certain sum tion to recover back the land.
of money and to give a village to J, the Issue: Is the action for recovery of land
illegitimate son of his father, on his attain- valid?
ing majority. Decision:
Issue: What is the nature of contract? (a) The action will be rejected on the
Decision: ground that the sale of the land in
(a) It is a valid contract. As there was a question was valid as it was done by
concrete consideration between U and the mother for her minor son and on
his father and if U does not fulfill any his behalf.
condition then J is entitled to sue U. (b) The action for recovery of land is val-
(b) As there is no contract between U and id and the minor will be entitled to the
J so J has no claims against U. recovery of land.
(c) It is a contract void ab initio because (c) It will be the discretion of the court
consideration agreed upon is for ille- because the present proposition is not
gitimate son which goes against the covered by the bare provisions of In-
morality. dian Contract Act.
53. Fact: D agreed with P’s father that he 56. Fact: A minor borrowed a sum of money
would pay P, Rs 500 per month as kharch- executing a simple bond for it, and after
i-pandan if she marries D’s son. The al- attaining majority executed a second bond
lowance was stopped sometime after the in respect of the original loan and inter-
celebration of marriage. P sued her father- est.
in-law for Rs 1,500 as arrears of allow- Issue: What is the nature of contract?
ance. Decision:
Issue: Can P sue the D and was there any (a) The contract is valid because the mi-
contract between P and D? nor had executed the second after at-
Decision: taining majority.
(a) Yes P can sue the D as there is a valid (b) The contract is not valid because the
contract because D had already per- contract was the renewal of first bond
formed the conditions of contract. which was done by minor.
(b) There is no valid contract so P cannot (c) Neither (a) nor (b)
sue D. 57. Fact: X, a minor, insured his goods with
(c) Neither (a) nor (b) an insurance company. The goods were
54. Fact: “As a minor borrowed Rs 20,000 damaged. X filed a suit for claim. The
from B and as a security for the same ex- insurance company took the plea that the

CLAT.indb 94 31/03/2009 11:41:46


CIVIL LAW 95

person on whose behalf the goods were in- (b) The contract is not valid because the
sured was a minor. The court rejected the application of coercion vitiates the va-
plea and allowed the minor to recover the lidity of contract.
insurance money. (c) It is a quite confusing situation so it
Issue: Will the minor be entitled to recov- will be the discretion of the court.
er the insurance money? 60. Fact: A girl of 13, lost her husband and
Decision: her husband’s relatives refused to have
(a) No, he will not be entitled as he is mi- the husband’s corpse removed unless she
nor. adopted one child of their choice. It was
(b) He will be entitled though he was mi- held that the adoption wad not binding on
nor at the time he insured his goods her as her consent was obtained under co-
with the company. The contract with ercion within the meaning of Section 15
the minor has been held to be void to since any person who obstructed a dead
protect the interest of minor but this body from being removed would be guilty
provision cannot be misused against of an offence under Section 297 of the In-
the interest of minor. dian Penal Code.
(c) It will be the discretion of the court as Issue: Is the adoption valid?
such contracts are not covered by the Decision:
Indian Contract Act. (a) No, the adoption is not valid as it was
58. Fact: An illiterate woman executed a deed imposed through coercion.
under the impression, that she was execut- (b) The adoption is valid.
ing a deed authorizing her nephew to man- (c) The adoption is voidable at the option
age her lands, while in fact it was a deed of of the girl.
gift in favour of her nephew. The evidence 61. Fact: By threat of suicide, a Hindu in-
showed that the woman never intended to duced his wife and son to execute a release
execute such a deed of gift nor was the in favour of his brother in respect of cer-
deed read or explained to her. tain properties which they claimed as their
Issue: Is the deed valid and is operative? own.
Decision: Issue: Is the deed valid?
(a) The deed is valid and operative. Decision:
(b) The deed is not valid and not operative (a) The threat of suicide amounted to
because any contract done under the
coercion within section 15 and the re-
wrong impression, misrepresentation,
lease dead was voidable.
mistake, undue influence, threat, force
(b) The deed is valid and can be execut-
or coercion is not a valid contract as
ed.
per the Section 14 of the Indian Con-
(c) Neither (a) nor (b)
tract Act.
62. Fact: A railway company refuses to de-
(c) As Indian Contract Act does not cover
liver certain goods to the consignee, ex-
such situations so it will be the discre-
cept upon the payment of illegal charges
tion of the court.
for carriage. The consignee pays the sum
59. Fact: A, on board an English ship on the
charged in order to obtain the goods.
high seas, causes B to enter into an agree-
Issue: Are the claims of consignee main-
ment by an act amounting to criminal in-
timidation under the Indian Penal Code. A tainable?
afterwards sues B for breach of contract Decision:
at Calcutta. A has employed coercion al- (a) The claims of consignee are maintain-
though his act is not an offence by the law able and he is entitled to recover so
of England, and although Section 506 of much of the charges as was illegally
the Indian Penal Code was not in force at excessive.
the time when or where that act was done. (b) It is a matter of mutual understand-
Issue: Is the contract valid and is bound to ing between the company and the
be performed? consignee so the claims of consignee
Decision: will not be maintainable in the court
(a) The contract is valid and bound to be of law.
performed. (c) Neither (a) nor (b)

CLAT.indb 95 31/03/2009 11:41:46


96 CIVIL LAW

63. Fact: A, a police officer purchased a 66. Fact: A, a money lender, advances Rs 100
property worth Rs 2 lakhs for Rs 20,000 to B, an agriculturist and by undue influ-
from B, an accused under his custody. But ence induces to B to execute a bond for
later on B wants to cancel the sale on the Rs 200 with interest at 6 per cent per
ground of undue influence. month.
Issue: Is the contract of sale is valid? Issue: Is it a case of undue influence?
Decision: Decision:
(a) Contract of sale is valid and B will not (a) It is not a case of undue influence
be entitled to claim the recovery of because it is purely and technically a
land. business deal and the motive of busi-
(b) A, the police officer is in a position to ness is to earn profit.
dominate the will of B and the exis- (b) It is a case of undue influence because
tence of undue influence can be pre- it is a mere exploitation of one’s con-
sumed so the contract is not valid and ditions and a clear cut case of induce-
B will be entitled to recovery of land. ment of undue influence.
(c) Neither (a) nor (b) (c) Neither (a) nor (b)
64. Fact: A solicitor sold certain property to 67. Fact: A youth of 18 years of age, spend
one of his clients. The client subsequently thrift and a drunkard, borrowed Rs 900 on
alleged that the property was considerably a bond bearing compound interest at 2%
overvalued and his consent was caused by per mensem.
undue influence. Issue: Is it a case of undue influence?
Issue: Is it a case of imposing undue influ- Decision:
ence and what is the nature of contract? (a) No, it is not a case of undue influ-
Decision: ence.
(a) The contract is not valid since the re- (b) It is a case of undue influence.
lationship of solicitor and client is of (c) It is a contract voidable at the option
fiduciary nature, the existence of un- of the youth.
due influence can be presumed to ex- 68. Fact: A, a horse dealer sold a mare to B.
ist and the client will be entitled to the A knew that the mare had a cracked hoof,
recovery of property. which he filled up in such a way as to defy
(b) It is a valid contract because it cannot
detection. The defect was subsequently
be presumed that such a learned so-
discovered by B.
licitor can resort to any illegal means.
Issue: Is it a valid contract?
(c) It will be the discretion of the court to
Decision:
decide on the basis of available evi-
(a) Yes, it is a valid contract.
dence and circumstances.
(b) It is not a valid contract because the
65. Fact: A poor Hindu widow who was in
consent of the B has been obtained by
dire need of money, was forced by a mon-
fraud and misrepresentation. It can be
ey lender to agree to pay 100% rate of in-
avoided by B.
terest.
(c) It cannot be avoided by B though it is
Issue: Is it a case of using undue influ-
not a valid contract.
ence?
Decision: 69. Fact: The director of a company issued
(a) It is a case of exerting undue influence a prospectus containing false representa-
upon a person of mental distress. So tion on the faith of which Z agreed to buy
the contract is not valid and the mon- some shares from the company.
ey lender cannot plead for the specific Issue: Can the Z avoid the contract?
performance of the contract. Decision:
(b) The money lender can ask for the spe- (a) Z may avoid the contract because the
cific performance of contract as there directors are deemed to be the agents
is no indication of any undue influence of the company.
because lady herself had approach the (b) Z cannot avoid the contract.
money lender and voluntarily agreed (c) There is no contract as such so com-
to pay 100% interest. pany is not liable.
(c) Neither (a) nor (b) 70. Fact: A broker was asked to buy shares

CLAT.indb 96 31/03/2009 11:41:46


CIVIL LAW 97

for client. He sold his own shares without tract of sale was signed. During this period
disclosing this fact. the sales came down to Rs 5000 a month.
Issue: Can client avoid the contract? A unintentionally keeps quite.
Decision: Issue: Is there misrepresentation and B is
(a) No, client cannot avoid the contract. entitled to rescind the contract?
(b) The client was entitled to avoid the Decision:
contract or affirm it with a right to (a) There was misrepresentation and B
claim secret profit made by broker on was entitled to rescind the contract.
the transaction since the relationship (b) There was no misrepresentation and
between the broker and the client was B was not entitled to rescind the con-
relationship of utmost faith. tract.
(c) Neither (a) nor (b) (c) The fluctuations in the profits is the
71. Fact: A intends to sell his horse to B and matter of fact, no one can be held li-
says, “My horse is perfectly sound”. A able for that.
genuinely believes the horse to be sound, 74. Fact: X, a widow is entitled to certain oc-
although he does not know that the horse cupancy rights. X remarries and believing
has fallen ill yesterday. B there upon buys that she has lost her occupancy rights by
the horse. reason of her second marriage, agrees to
Issue: Is there misrepresentation on the take the land from Y, her zamindar, on an
part of A? increased rate of rent. Both X and Y hon-
Decision: estly believe that X has lost her occupancy
(a) There is no misrepresentation because rights.
it’s a business and everything is fair. Issue: What is the nature of contract?
(b) There is misrepresentation on the part Decision:
of A because no business can be con- (a) The contract is voidable.
ducted which involves fraud, mispre- (b) The contract is not voidable.
sentation, undue influence, force or (c) Neither (a) nor (b)
coercion. 75. Fact: A by misrepresentation leads B er-
(c) Neither (a) nor (b) roneously to believe that 500 maunds of
72. Fact: A company’s prospectus contained a indigo are made annually at A’s factory.
representation that the company had been B examines the accounts of the factory,
authorized by a Special Act of Parlia- which show that only 400 maunds of in-
ment to run trans by steam or mechanical digo have been made. After this, B buys
power. The authority to use steam was, in the factory.
fact, subject to the approval of the board Issue: Is the contract void?
of Trade, but no mention was made of this. Decision:
The Board refused consent and conse- (a) The contract is not voidable on ac-
quently the company was wound up. The count of A’s misrepresentation, be-
plaintiff having bought some shares, sued cause A had the means of discovering
the directors for fraud. the truth with ordinary diligence.
Issue: Are the directors liable? (b) The contract is void as it is a clear cut
Decision: case of misrepresentation.
(a) They are not liable because whatever (c) Neither (a) nor (b)
they stated in their prospectus was not 76. Fact: A man and a woman made a separa-
at all intended to misrepresent any- tion deed under which the man agreed to
body. pay a weekly allowance to the woman un-
(b) They are liable because misrepresen- der a mistaken assumption that they were
tation is there whether it is intentional lawfully married.
or unintentional. Issue: Is it a valid contract and enforce-
(c) The plaintiff is equally liable. able in the court of law?
73. Fact: A, before signing a contract with B Decision:
for sale of business, correctly states that (a) Yes, it is a valid contract and enforce-
the monthly sales are Rs 50,000 Negotia- able in the court of law.
tions lasted for five months, when the con- (b) It is a void agreement as there was a

CLAT.indb 97 31/03/2009 11:41:46


98 CIVIL LAW

common mistake on the point of fact from B, though contrary to the belief of
which was material to the existence of both parties at that time, A was already a
agreement. tenant for life by inheritance of the fishery
(c) It is a voidable contract. and B had no title at all.
77. Fact: There is an agreement between A Issue: Is the agreement valid?
and B for the purchase of a certain horse. Decision:
But the horse is dead at the time of the (a) The agreement is valid.
contract. (b) The said agreement is void.
Issue: What is the nature of contract? (c) The agreement is voidable.
Decision: 82. Fact: H contracted with the N Corpora-
(a) The contract is valid and enforceable tion to build a number of houses. In cal-
in the court of law. culating the cost of the houses, H, by mis-
(b) The contract is void. take deducted a particular sum twice and
(c) The contract is voidable. submitted his estimates accordingly. The
78. Fact: A agrees to sell to B a specific cargo Corporation agreed to the terms which
of goods supposed to be on its way from were naturally lower than the actual cost.
England to Bombay. It turned out that be- Issue: Is the agreement binding?
fore the day of the bargain, the ship carry- Decision:
ing the cargo had been cast away and the (a) The agreement was binding even
goods lost. Neither party was aware of the though it was based upon erroneous
facts. estimates.
Issue: What is the nature of contract? (b) The agreement is void.
Decision: (c) The agreement is voidable.
(a) The contract is valid and A is liable. 83. Fact: A, an illiterate old man, was made to
(b) The contract is voidable at the option put his signature on a document which was
of B. a pronote. A thought the document to be a
(c) The contract is void. will where his signature was required as a
79. Fact: A agreed to buy from B, 125 bales witness. Under this presumption he signed
of cotton “to arrive ex pearless from Bom- the document which was in fact a pronote.
bay”. There were two ships of that name Subsequently B endorsed the pronote to C
sailing from Bombay. One of which was who paid value for it in good faith. C sued
in the mind of A and the other in the mind A on the pronote.
of B. Issue: Is A liable?
Issue: What is the nature of agreement? Decision:
Decision: (a) A is liable because the pronote was
(a) It is a valid agreement. signed by him.
(b) That agreement is void. (b) A is not liable because A had not
(c) The agreement is voidable. signed the pronote with the intention
80. Fact: D wrote to P asking for a quotation of endorsement to C.
of 50 rifles. Later he telegraphed, “Send (c) Neither (a) nor (b)
three rifles”. The telegraph clerk by mis- 84. Fact: A fraudulent person named Blen-
take sent the message wrongly as “Send karn by imitating the signature of a re-
the rifles”. P sent 50 rifles and upon D’s spectable firm Blenk & Co., sent an order
refusal to accept, filed a suit for damages. for goods, which were duly delivered to
Issue: What is the nature of contract? him by Lindsay. Blenkarn sold the goods
Decision: to Cundy who acted as a bonafide.
(a) Contract is void even if the mistake Issue: Should Cundy returns the goods to
was caused by the third party. Lindsay?
(b) The contract is valid because the mis- Decision:
take was made by the clerk who was (a) Cundy must return the goods to Lind-
not a party to the contract. say, as although Cundy was an in-
(c) The contract is voidable at the option nocent purchaser, he had no title to
of D. them, the contract between Lindsay
81. Fact: A agreed to take a lease of fishery and Blenkarn being void.

CLAT.indb 98 31/03/2009 11:41:46


CIVIL LAW 99

(b) Cundy is not liable, Blenkarn is li- George Bullough (a name known to P as a
able. man of good credit). N pledged the jewels
(c) Neither Blenkarn nor Cundy are li- with D for £ 350. P sued D for the return
able, it is the negligence of the com- of the jewels pleading that no title passed
pany so they should suffer. to N as he was not Sir George Bullough.
85. Fact: Boulton had taken over the business Issue: Is the contract valid/void?
of Brocklehurst, with whom Jones had Decision:
been in the habit of dealing and against (a) There was no mistake as to the identi-
whom he had a set off. Jones set an order ty of the person and hence the contract
for goods to Brocklehurst. Boulton ex- was not void.
ecuted the order without notifying Jones (b) The contract was void.
of the change in the ownership of the busi- (c) The contract was voidable.
ness. 89. Fact: A’s estate is sold for arrears of
Issue: Is the contract valid? revenue under the provision of an Act of
Decision: the legislature by which the defaulter is
(a) Contract is valid and Jones is liable. prohibited from purchasing the estate. B,
(b) Jones was not liable to pay for the upon an understanding with A becomes
goods as the contract was void. the purchaser, and agrees to convey the
(c) Contract is voidable. estate to A upon receiving from him the
86. Fact: X falsely representing herself as the price which B has paid.
wife of a well known millionaire takes a Issue: Is the agreement valid?
ring from a jewellers shop for the approval Decision:
of her husband. She pledges it with a pawn (a) The agreement is valid and the trans-
broker, who is in good faith and without action is workable and seller and pur-
notice of the first transaction pays her chase are liable.
Rs 10,000. (b) The agreement is void, as it renders
Issue: Can the jeweller recover his ring? the transaction, in effect a purchase by
Decision: the defaulter and would so defeat the
(a) The jeweller could recover the ring object of the law.
from the pawn broker as there was no (c) The agreement is voidable.
contract between the jeweller and the 90. Fact: A asks an editor of a newspaper to
lady. publish a defamatory article against B and
(b) The jeweller could not recover the promises to pay Rs 4000 for the work.
ring from the pawn broker Issue: Can the editor sue A?
(c) The jeweller can sue the lady. Decision:
87. Fact: S was a cinema critic, who always (a) The agreement is void as it involves
wrote adverse criticisms about all the injury to the person of B and there-
picture he saw. A, the owner of a theatre fore, a suit cannot be brought to re-
advertised new picture in the theatre and cover Rs 4000 by the editor.
gave instructions that no ticket should be (b) The agreement is valid and the edi-
issued to S. S obtained ticket for a first tor can sue A for the recovery of
night performance through a friend. But at Rs 4000.
the gate S was refused admission. S sued (c) B can sue A and editor both.
for breach of contract. 91. Fact: P advanced money to D, a married
Issue: Can S sue the theatre owner? woman to enable her to obtain a divorce
Decision: from her husband and D agreed to marry
(a) S can sue the theatre owner. her as soon as she obtained a divorce.
(b) S cannot sue the theatre owner as Issue: Can P sue D for the recovery of
there was no contract between the the- money?
atre owner and S. Decision:
(c) Neither (a) nor (b) (a) P can sue D for the recovery of mon-
88. Fact: One N entered P’s shop, selected ey.
jewellery and wrote out a cheque for (b) P was not entitled to recover back the
£ 3000 representing himself to be Sir amount as the agreement had for its

CLAT.indb 99 31/03/2009 11:41:47


100 CIVIL LAW

object the divorce of D from her hus- sum of Rs 2000. Miss B sued A for the
band which is unlawful and against recovery of that sum on the ground that A
morality and public policy. married someone else.
(c) Neither (a) nor (b) Issue: What is the nature of the agree-
92. Fact: A bank found that the goods in a ment?
godown which were pledged to it against Decision:
a loan, were either fraudulently overval- (a) Agreement is valid and B can sue A.
ued or withdrawn in collusion with bank (b) The agreement is voidable at the op-
officials. The borrower agreed to make up tion of B.
for the deficiency by hypothecating more (c) The agreement was in restraint of
property. Some delay having taken place marriage and as such void.
in the hypothecation, the bank filed a com- 97. Fact: D agreed with P, not to carry on the
plaint which was withdrawn after the hy- same business of dress makers on the ex-
pothecation was completed. piry of the period of his service anywhere
Issue: Is the agreement valid? within 800 miles of Madras.
Decision: Issue: What is the nature of agreement?
(a) The agreement was valid. Decision:
(b) Agreement is voidable. (a) The agreement is valid.
(c) Agreement is void. (b) The agreement is void.
93. Fact: A paid B, a public servant a certain (c) The agreement is voidable.
amount inducing him to retire from ser- 98. Fact: A contracts to sell to B the good-
vice, thus paving the way for A to be ap- will of a business. A then sets up a similar
pointed in his place. business close-by B’s shop and solicits
Issue: Is the agreement valid? his customers. This is contrary to the con-
Decision: tract.
(a) Agreement is valid. Issue: Can B obtain an injunction to re-
(b) The agreement is voidable. strain A from soliciting customers?
(c) The agreement is void. Decision:
94. Fact: A proposed the marriage of his wid- (a) B can obtain an injunction.
ow niece to B and offered to give her gold (b) B cannot obtain an injunction.
and jewels and land. The marriage took (c) Neither (a) nor (b)
place, but A refused to fulfil the rest of his 99. Fact: A agreed to become an assistant for
promise. 3 years to B who was a doctor practising
Issue: Is the agreement enforceable? at Zanzibar. It was agreed that during the
Decision: term of the agreement A was not to prac-
(a) It is enforceable as there is a valid tice on his own account in Zanzibar. At the
agreement. end of one year, A ceased to act as B’s as-
(b) It is not enforceable. sistant and began to practice on his own
(c) Neither (a) nor (b) account.
95. Fact: A promises B, the owner of a news- Issue: Can A be restrained?
paper Rs 500 in consideration of the pub- Decision:
lication by B, in his newspaper of false (a) The agreement was valid and A could
statements in regard to a candidate for be restrained by an injunction from
election. B published them. doing so.
Issue: What is the nature of agreement? (b) A cannot be restrained.
Decision: (c) Neither (a) nor (b)
(a) The agreement is voidable at the op- 100. Fact: A company manufacturing special
tion of B. yarn was offered collaboration by a for-
(b) The agreement is valid and B can sue eign producer on the condition that the
A. company shall maintain secrecy of all the
(c) The agreement is void as it is against technical information and that it should
the public policy. obtain corresponding secrecy arrange-
96. Fact: A promised to marry none else ex- ment from its employees. One employee
cept Miss B, and in default to pay her a was appointed for five years with the con-

CLAT.indb 100 31/03/2009 11:41:47


CIVIL LAW 101

dition that during this period he would not and B has accepted, then B is legally
take service anywhere even if he left this bound, if he doesn’t turn up it amounts
service. to breach of contract.
Issue: Is the agreement valid? (b) It is a contract voidable at the option
Decision: of A. it will be option available to A to
(a) Agreement is valid. sue B or not to sue B.
(b) Agreement is void. (c) No, A cannot sue B because the agree-
(c) Agreement is voidable. ment was purely social in nature and
101. Fact: Mohan polished Mr Ram Prasad’s there was no intention to create legal
shoes without using aspect to do so. relationship. Agreements purely so-
Mr Ram did not make any attempt to stop cial and domestic in nature are not
Mohan from polishing his shoes. enforceable by law.
Issue: Can Mohan make Mr Ram liable 104. Fact: X invited Y and this family to din-
for payment for his services? ner on a certain night. Y accepted X’s in-
Decision: vitation. On the date fixed Y drove with
(a) Mr Ram is liable to pay because he his family from Sector 14 to Industrial
accepted Mohan’s Implied offer by Area and found his house locked. They
not stopping him from polishing his waited upto 9.30 p.m. but the hosts did not
shoes. turn up. They left the place and had their
(b) Ram is not liable because the present meals in Piccadilly in Sector 17. The cost
situation does not indicate any offer or of meal came to Rs 100.
its acceptance. Hence no contract, no Issue: Can Y recover the amount?
validity, no liability. Decision:
(c) It is a vague contract, it will be discre- (a) No, Y cannot recover the amount
tion of the court to interpret it at its from X because agreement was purely
will. social in nature and there was no in-
102. Fact: State whether there is any contract tention to create a legal relationship.
in the following cases: (a) A and B agree (b) Y can recover the amount because X
to go for hunting. (b) A and B agree to is responsible for all the botheration
marry each other. (c) A agrees to sell some caused to Y, so X is liable.
goods to B at a price to be fixed by C. (d) (c) Neither (a) nor (b)
A takes food in a restaurant. (e) A takes a 105. Fact: A agreed orally to sell his house
seat in a local bus. to B for Rs 5,000, “Subject to contract”.
Issue: What is the nature of contract? The next day C offered A Rs 6,000 for the
Decision: house and A accepted this offer and sold
(a) All the said contracts are void con- the house to C. Can B sue A for breach of
tracts. contract? Would it make any difference to
(b) There is valid contract in all cases ex- your answer if the agreement between A
cept (a) because agreement to go for and B was in writing?
hunting is a purely social agreement Issue: What is the liability of A and C?
not intending to create legal relations. Decision:
In (b) and (c), there is intention to cre- (a) A and C both are liable because it
ate legal relationship whereas (d) and does not matter whether the agree-
(e) amount to implied contracts. ment is oral or written.
(c) In all the above statements there is no (b) An oral contract is perfectly valid ex-
contract because the offers and accep- cept where writing, registration etc.,
tances are very vague. is required by some statute. B cannot
103. Fact: A invites B to dinner. B accepts the sue A for breach of contract because
invitation but does not turn up at the din- the formalities required under the
ner. Transfer of Property Act 1882 had
Issue: Can A sue B for the loss he has suf- not been completed. Had this agree-
fered? ment been in writing and registered, B
Decision: could have sued A.
(a) A can sue because once A has offered (c) Neither (a) nor (b)

CLAT.indb 101 31/03/2009 11:41:47


102 CIVIL LAW

106. Fact: A, engages B for doing some work 109. Fact: A agrees to marry B. But B dies be-
and promises to pay such remuneration as fore the marriage took place.
shall be fixed by a third person C. B does Issue: Is this a void agreement?
the work. Decision:
Issue: What will B get? (a) It is not a void agreement but a void-
Decision: able agreement.
(a) B will not get anything because A did (b) It is a agreement void ab initio.
not promise to pay remuneration. (c) It is not a void agreement, but a void
(b) C will be liable if B proves the con- contract because it was absolutely val-
tract with C. id when entered into but subsequently
(c) There is a contract between A and B became void.
and A is bound to pay the remunera- 110. Fact: Romeo promised a gold necklace to
tion as fixed by C. If C does not fix or Juliet at the time of their marriage. Romeo
refuses to fix the remuneration, then A fails to bring the necklace.
is bound to pay the reasonable remu- Issue: Can Juliet claim the necklace?
neration. Decision:
107. Fact: X promises to pay Y, his wife a (a) Juliet cannot claim the necklace as
pocket money Rs 500 per month. After 2 there is no consideration from Juliet.
months he stops the pocket money. (b) Juliet can claim the necklace.
Issue: Can Y sue X? (c) Neither (a) nor (b)
Decision: 111. Fact: A, an auctioneer gives advertisement
(a) No, because agreement between X in the newspaper wherein he mentions that
and Y is domestic agreement which is certain furniture would be sold at Jaipur
not intended to be enforceable at law. on a certain day. Mr B, a prospective buy-
(b) Y can sue X because the matrimonial er of Delhi, on reading the advertisement
is purely and technically a legal rela- reaches Jaipur on the fixed date. But the
tionship. The breach of any condition/ auctioneer cancels the auction sale.
promise results into breach of contract Issue: Can B sue A, for not conducting the
and hence amounts to liability. auction sale and can he claim damages for
(c) It will be the duty of the court to de- loss of his time and expenses?
duce the relevant facts and give judge- Decision:
ment. (a) Yes, he can because once the auction-
108. Fact: A agrees to pay Rs 50,000 to B if B eer gave an advertisement and B read
kills C. To pay to B, A borrowes Rs 50,000 it, the contract is complete.
from D who is also aware of the purpose (b) No, because advertisement was only
of loan. B kills C but A refuses to pay. A an invitation to offer and not offer.
also refuses to repay the loan to D. (c) Neither (a) nor (b)
Issue: What is the nature of contract? 112. Fact: A offers, by a letter, to sell certain
Decision: article to B who receives the letter the
(a) A is liable to pay Rs 50,000 to B next day B immediately posts his letter
though B is guilty of offence of mur- of acceptance. The same evening A posts
der. But the agreement between A and another letter revoking his offer. A’s let-
B is valid and hence enforceable. ter revocation and B’s letter of acceptance
(b) The agreement between A and B is an cross in the post.
illegal agreement because its object Issue: Is there any contract between A and
is unlawful. Hence B cannot recover B?
anything from A. Since main agree- Decision:
ment between A and B is illegal the (a) No, B cannot because there is no con-
agreement between A and D which tract as there is no receipt of offer and
is collateral to the main agreement is acceptance by A and B.
also void and hence D cannot recover (b) It is an agreement voidable at the op-
anything from A. tion of B.
(c) A is liable to pay amount to D because (c) Yes, a contract comes into existence
D has no involvement in the offence between A and B as per Section 4 and
of murder. Section 5 of the Indian Contract Act,

CLAT.indb 102 31/03/2009 11:41:47


CIVIL LAW 103

because once the letter for offer and 116. Fact: A offers to sell his house to B for
letter for acceptance are posted the Rs 5 lakhs. B says, “I accept your offer.
contract is complete and the parties Here is Rs 2 lakhs in cash and a 3 month
are bound and liable. promissory note for the balance.”
113. Fact: A makes an offer to B by letter Issue: Is there any contract between A and
wherein he mentions that acceptance can B?
be made by B posting a letter to A. A gives Decision:
the letter to his peon for posting the same (a) No, there is no contract between A and
but peon who gets to post it immediately, B because acceptance is conditional.
posts it after 7 days. In the meantime, on (b) Yes, there is contract, payment of Rs 2
not receiving any reply from B, A sells the lakh amounts to the performance of
goods to C. But B as soon as the receives contract and hence contract is valid.
A letter, gives his acceptance by posting a (c) The contract is voidable at the option
letter. of A.
Issue: What are the legal remedies avail- 117. Fact: An auctioneer advertised in a news-
able to B, if any? paper that a sale of office furniture will
Decision: be held at Bangalore. Ajay, a broker of
(a) There is contract between A and Mumbai reached Bangalore on the ap-
B. But since A has already sold the pointed date and time. But the auctioneer
withdrew all the office furniture from the
goods, B can recover damages from
auction sale. The broker sued for his loss
A.
of time and expenses.
(b) There is no contract between A and B
Issue: Will be succeed?
because the letter was posted after the
Decision:
goods were sold so B cannot sue for
(a) The broker will succeed and for the
damages.
loss of time and expenses he will get
(c) Neither (a) nor (b) the damages.
114. Fact: Mr A stops a taxi and steps in to (b) In sale by auction, goods are sold by
it and asks the driver to take him to cer- inviting bids from the prospective
tain destination. The driver refuses on the buyers. The bids are offers and invi-
ground that the trip is very short and not tation to people to participate in auc-
profitable for him. tion sale through advertisement is not
Issue: What can Mr A do? a proposal but merely an invitation to
Decision: offer. Therefore, the brokers will not
(a) A cannot do anything. succeed in claiming compensation for
(b) Mr A can force the driver to take him loss of time and expense in reaching
to the destination, as a contract comes Bangalore for advertised auction sale
into existence between Mr A and taxi cancelled later on.
driver as soon as A steps into taxi. (c) Neither (a) nor (b)
(c) It is a contract voidable at the option 118. Fact: Mr A is a merchant of Delhi. He
of driver. sends his son B to Mumbai with a letter
115. Fact: An article is on display in a show- addressed to a merchant of Mumbai. B
room with price tag of, “Rs 100”. Mr A hands over the letter to C and keeps wait-
offers Rs 100 to the shopkeeper for the ing there. C reads the letter which contains
article, but shopkeeper refuses to sell it. an offer from A for sale of certain quantity
Issue: Can Mr A sue B for not selling the of rice. The letter also contains that if C
article? accepts the offer, then he must give his
Decision: reply to B. C sends B without any reply,
(a) A can sue B because display of article but later decides to accept A’s offer by
is an offer and B has accepted it for telegram. The telegram reaches A before
the price tagged on it. B returns.
(b) Mr A can not force the shopkeeper to Issue: Has a valid contract come into exis-
sell the article, because display of the tence?
article is only invitation to offer and Decision:
not offer. (a) There is a valid contract once the C
(c) Neither (a) nor (b) sends the telegram.

CLAT.indb 103 31/03/2009 11:41:47


104 CIVIL LAW

(b) It is a contract voidable at the option Issue: Is A bound by his promise?


of A. Decision:
(c) No, But A must inform C within rea- (a) Yes, Section 25(2) is applicable, be-
sonable time that acceptance is not cause there is a valid consideration
made according to the prescribed and in return for the consideration B
mode. has performed the conditions of con-
119. Fact: Mr A tells B during conversation, tract.
that he will give Rs 10,000 to anyone who (b) A is not bound by the promise because
marries his daughter with his consent. B it is an oral contract.
marries A’s daughter with A’s consent. (c) A is not bound by the promise because
Issue: Is B entitled to recover the money the action of B is just of philanthropic
from A? nature.
Decision: 123. Fact: A, promises to contribute Rs 5,000
(a) B can’t recover the money from A be- for repairs of a temple. The trustee of
cause what A says is only a statement temple, relying on A’s promise incure cer-
of intention and not offer. tain liabilities. A latter on does not pay the
(b) Yes, B can recover the money because amount.
once the offer was made and accepted Issue: Can trustee take action against A?
it had resulted into valid contract and Decision:
liability arises. (a) Trustee cannot take action against A
(c) Neither (a) nor (b) because the mere promise by A does
120. Fact: A is in dire need of money, so sells not bind him.
his newly purchased car worth Rs 3 lakhs (b) Yes, the trustee can take action be-
for Rs 50,000. Later on, he wants to set cause the promise by A leading to
aside the contract on the ground of inad- the trustee incurring certain liabilities
equacy of consideration? make A liable.
Issue: Can he do so? (c) Neither (a) nor (b)
Decision: 124. Fact: A promises to contribute Rs 20,000
(a) Yes, he can do so on the ground of in- towards National Defence Fund. He does
adequacy of consideration. not pay the promised money.
Issue: Can any legal action be taken
(b) No, he cannot do so because there was
against him?
no undue influence and consideration
Decision:
was valid.
(a) Yes, he can be sued.
(c) It is a contract voidable at the option
(b) Neither (a) nor (c).
of A.
(c) No. The mere promise by A does not
121. Fact: When A is out of station on holiday,
result into any contract.
a storm damages the roof his house. B his
125. Fact: A was badly in need of money, so
neighbour and friend carries out the neces-
he offered to sell his motor cycle worth
sary repairs. On coming back, A promises
Rs 50,000 to B for Rs 25,000. B refused
to pay Rs 10,000 for the expenses incurred
to buy. A further lowered the price until B
and time spent by B.
agreed to pay Rs 20,000. Before the deliv-
Issue: Can B recover the amount if A does ery of motor cycle to B, A received anoth-
not pay it later on? er attractive offer from C, for the purchase
Decision: of motorcycle for Rs 30,000. A refused to
(a) B cannot recover because there is no deliver the motorcycle to B arguing that
valid contract. the purchase price was inadequate and
(b) It will be the discretion of the court. sells it to.
(c) Yes, B can recover the amount from Issue: Can B sue A for damages?
A as per conditions being fulfilled of Decision:
Section 25(2). (a) Yes, because inadequacy of consider-
122. Fact: A car breaks down on a road. A asks ation is no ground for avoiding a con-
B, a passer-by, to help him tow his car to a tract.
nearby garage. B tows the car and in return (b) No, because B was already exploiting
A promises to B at the garage Rs 100. the situation of A.

CLAT.indb 104 31/03/2009 11:41:47


CIVIL LAW 105

(c) It is a contract voidable at the option Issue: Can the Vakil claim Rs 1000?
of B so A is liable. Decision:
126. Fact: A and B are friends. B treats A dur- (a) Yes, Vakil can claim.
ing A’s illness. B does not accept payment (b) Vakil cannot claim.
from A for the treatment and A promises (c) It will be the discretion of the court to
B’s son X to pay him Rs 1,000. A being decide.
in poor circumstances in unable to pay. X 130. Fact: A promises B his nephew, a reward
sues A for money. of Rs 500 if he refrained from smoking for
Issue: Can X recover? two years. B does so.
Decision: Issue: Is he entitled to the reward?
(a) Yes, X can recover as his father B has Decision:
already performed the conditions of (a) Yes, B is entitled to reward because
contract for which the payment is to he has, at the desire of uncle refrained
be made. from smoking for 2 years. This is a
(b) X cannot recover amount from A be- valid consideration as per definition
cause there is no agreement between in Section 2(d) in the form of absti-
X and A because there is no consid- nence.
eration. Hence, no consideration no (b) No, he is not entitled because the
contract. promise by uncle is just an invitation
(c) Neither (a) nor (c). to offer and does not amount into any
127. Fact: X gives to Y Rs 300 to be given valid contract and there was no valid
to Z. Y informs Z that he is holding the consideration.
money for him. But afterwards Y refuses (c) It is a contract voidable at the option
to pay the money. Z sues Y for the money. of uncle.
Y contends that Z cannot sue as he was 131. Fact: A, a Muslim lady sues her father-in-
not a party to the contract. law for the arrears of allowance payable to
Issue: Will the contention of Y be up- her by him, under an agreement between
held? him and her own father, in consideration
Decision: on marriage.
(a) Y’s contention will not be upheld be- Issue: Will she succeed?
cause X has created privity of contract
Decision:
by his conduct.
(a) No
(b) Y’s contention will be upheld because
(b) Yes, because although she is a stranger
he was not party to contract.
to contract, but she can sue as contract
(c) Neither (a) nor (b).
was made for her benefit.
128. Fact: A promises to give Mr B Rs 500 as
(c) Neither (a) nor (b)
birthday present on B’s birthday. A fails
132. Fact: X who is a major, performs some
to fulfils this promise. Mr B wants to
services for B, a major at B’s request. B on
file a suit against A for the realization of
attaining the age of majority, promised to
amount.
compensate X by giving him a promissory
Issue: Can B sue A?
note.
Decision:
(a) B cannot sue A because there is no Issue: Is the promissory note valid?
contract, the mere promise of birth- Decision:
day present out of love and affection (a) No, because a minor can’t ratify an act
do not result into any legal agreement done during minority, even on attain-
and hence no liability. ing the age of majority.
(b) B can sue A because once the promise (b) Yes, the promissory note is valid once
is made the valid contract arises. it has been ratified after attaining ma-
(c) Neither (a) nor (b) jority.
129. Fact: After the Vakil had accepted the (c) It will be the discretion of the court.
case and the signed the Vakalatnama, the 133. Fact: A, a minor, borrowed Rs 50,000
client says. ‘If you do your utmost to win from B by executing a mortgage of his
the case. I will give you rupees one thou- property in favour of B. Subsequently, the
sand more” The case is won. minor sued for setting aside the mortgage.

CLAT.indb 105 31/03/2009 11:41:47


106 CIVIL LAW

Issue: Do you think that the mortgage is avoid the contract on the plea of his mi-
valid? Can B recover the amount advanced nority. Can he do so?
to A? Issue: What are the options available to
Decision: L?
(a) Minor is not competent to contract, Decision:
so minor’s contract is void. Thus the (a) An agreement with minor is void.
mortgage executed by A is not valid But a minor cannot defraud a person
and B can’t recover the money lent to on the pretext of his minority. In this
A. case, court may direct the minor to re-
(b) The B can recover the amount once store the car to L.
the minor will attain majority. (b) An agreement with minor is void so
(c) It is a voidable contract at the option minor does not have any liability.
of minor. (c) An agreement is voidable.
134. Fact: A, aged 16, agreed to purchase 137. Fact: A, a minor who wished to become a
a second hand motorcycle from B for professional billiards player, entered in to
Rs 12,000. He paid Rs 2,000 as advance a contract with B, a noted billiard payer,
and agreed to pay the balance the next day to pay him (B) a certain sum of money to
and collect the motorcycle. When A came learn the game and play matches with him
with the money the next day, B says that during his world tour. B spent time and
he has changed his mind and offered to re- money in making arrangements for bil-
turn the advance. liards matches.
Issue: Can B do so? Issue: Is A liable to pay?
Decision: Decision:
(a) Yes, because the agreement with mi- (a) No, because the agreement with mi-
nor is void. nor is void.
(b) No, B can’t avoid the agreement sim- (b) Yes, as the agreement was for neces-
ply because A is a minor and an agree- saries. Necessaries include education,
ment with minor is void although A is training for trade, medical services
a minor but law does not prevent him etc.
from becoming a promise or benefi- (c) This agreement is voidable at the op-
ciary. tion of B.
(c) Neither (a) nor (b). 138. Fact: X, a minor borrowed from Y, a sum
135. Fact: M, a minor, a under a contract of of Rs 10,000 the payment of the loan was
sale delivered goods to the buyer. The guaranteed by Z who is major. X refuses
buyer later on said, “I will not pay you the to pay.
price. You are a minor and you cannot sue Issue: Can Y hold Z liable for the mon-
me.” ey?
Issue: Is the buyer right in saying so? Decision:
Decision: (a) No, Y cannot hold Z liable because Z
(a) No, because although an agreement is not party to contract.
with or by minor is void but a minor (b) Neither (a) nor (c)
can be a promise or beneficiary. (c) Yes, Y can hold Z liable for the mon-
(b) Yes, because the agreement of the ey. The rule is: If an adult stands sure-
minor is void hence there is no agree- ty for a minor, the adult is liable to the
ment, no liability. creditors though minor is not person-
(c) Neither (a) nor (b). ally liable.
136. Fact: S, a minor by fraudulently repre- 139. Fact: S, a minor agreed to take 500 shares
senting himself to be full age, enters into of rupees ten each in a company and paid
an agreement with L, for selling his (S’s) rupees five on each share. He received no
propertly. Under the agreement S receives dividend on the shares. While still a mi-
the full amount of Rs 5 lakhs in advance nor he repudiated the contract and brought
from L and uses it for purchasing a new an action (i) to recover the money he had
car worth Rs 3 lakhs and spends the bal- paid, and (ii) for declaration that he was
ance on a world tour. Later on, S tries to not liable for future calls.

CLAT.indb 106 31/03/2009 11:41:47


CIVIL LAW 107

Issue: Will he succeed? (c) Yes, but only minor’s property is li-
Decision: able.
(a) S a minor will succeed because mi- 143. Fact: A grocer supplied certain necessar-
nor cannot apply for shares and make ies to M, a minor makes a promissory note
himself liable for future calls. Howev- in favour of grocer.
er, fully paid shares can be transferred Issue: Can the grocer claim payment from
in his name. M under the promissory note?
(b) S will not succeed because the minor Decision:
cannot take disadvantage of his mi- (a) Grocer can claim payment only from
nority. M’s estate and not from minor person-
(c) Neither (a) nor (b). ally.
140. Fact: An infant made the following con- (b) No, the grocer cannot claim any pay-
tracts: (i) an engagement to marry a girl, ment because agreement with the mi-
(ii) a credit purchase of an engagement nor is void.
ring for his fiancee (iii) a hire-purchase (c) Neither (a) nor (b)
contract for a set of gold clubs. He now 144. Fact: A purchased a machine from dealer,
wishes to call off his engagement. relying on the dealer’s representation that
Issue: Is he liable upon any of the above it was a new model. After paying the pur-
contracts? chase price, A came to know that the ma-
Decision: chine was not new but an old one.
(a) He is not liable upon any of these Issue: What are the legal rights of A?
contracts. (i) is invalid because it is Decision:
prohibited under Child Marriage Re- (a) A cannot sue the dealer because the
straint Act. (2) and (3) are not neces- maxim of caveat emptor applies on
saries of life. So, he is not liable upon him and it debars him to sue the deal-
any contract. er.
(b) He can be held liable for engagement (b) A can avoid the contract on the ground
with minor girl. of fraud and get the damages from the
(c) Though he cannot be held liable for dealer.
engagement with girl but he can be (c) Neither (a) nor (b)
145. Fact: A advances a sum of money to
held liable for the credit of ring and
his son, B during his minority. When B
the contract for higher purchase of
becomes major, A by misusing his pa-
gold clubs.
rental influence, gets a bond from B, of
141. Fact: M, a guardian , on behalf of a minor,
an amount more than the amount due. B
L entered into a contract with S for pur-
refuses to give the amount stated in the
chase of certain property for the benefit of
bond.
L.
Issue: Is B bound by the bond?
Issue: Is the contract valid?
Decision:
Decision:
(a) No, B is not bound by the bond. Con-
(a) No, it is not a valid contract because
tract is voidable at the option of B be-
any contract with the minor or on his
ing caused by undue influence.
behalf is void. (b) B is bound by the bond.
(b) Yes, provided it is within the scope of (c) It will be the discretion of the court to
guardian’s authority. decide.
(c) Neither (a) nor (b). 146. Fact: L was the highest bidder at an auc-
142. Fact: A minor broke his elbow while play- tion sale of a public plot. At the time of
ing football game. He went to the doctor making the bid, L believed that certain
for treatment. field was the part of plot offered for sale.
Issue: Does the doctor has any claim for Later he found out that it was not the part
his services? of plot and the seller never said so.
Decision: Issue: L wants to cancel the contract, can
(a) No, because the minor does not owe he do so?
any liability. Decision:
(b) Neither (a) nor (c). (a) L can not cancel the contract because

CLAT.indb 107 31/03/2009 11:41:47


108 CIVIL LAW

in this case only L is mistaken. Thus it pretext of showing them to her husband
is a case of unilateral mistake. before buying. She later on pledges them
(b) L can cancel as it is not a valid con- with a broker, who in good faith gives Rs 5
tract because the contract entered into lakh to the lady. The jeweller on knowing
was mistaken. this, files a suit against the broker.
(c) Neither (a) nor (b) Issue: What will be the legal position in
147. Fact: A, contracts with B to buy a neck- this case?
lace, believing it to be made of very costly Decision:
rare pearls, while in reality, the pearls are (a) The jewellers cannot get back the
only an imitation having very little value. necklace because there is no privity of
B knows that A is mistaken but does not contract between the jeweller and the
disclose the fact. broker.
Issue: Is A bound by the contract? (b) The broker does not get good title
Decision: over the jewellery, so jeweller can get
(a) No, A is not bound by the contract be- back the necklace from the broker.
cause it is not a unilateral mistake. (c) The jeweller can sue the lady.
(b) Yes, because the rule of caveat emptor 151. Fact: A, is an illiterate old man of about
will apply. 90 yrs, physically infirm and mentally in
(c) No, A is not bound by the contract be- distress. He made a gift of his entire prop-
cause the rule of caveat venditor ap- erty in favour of B, his nearest relative
plies on B. who was looking after him (A) and his
148. Fact: A agrees to buy certain horse from business.
B for Rs 10,000 and pays Rs 5,000 as ad- Issue: Can A subsequently avoid the con-
tract of gift? If so, on what ground?
vances. It turns out that the horse was dead
Decision:
at the time of bargain, though neither party
(a) No, A cannot avoid the contract of gift
knew this fact.
because the gift has been made volun-
Issue: Can A recover the advance money
tarily.
given?
(b) Neither (a) nor (c)
Decision:
(c) Yes, A can avoid the contract of gift
(a) No, A cannot get the money back be-
on the plea of undue influence, Sec-
cause money advanced is not return-
tion 16(2).
able. 152. Fact: A railway company refuses to de-
(b) A cannot get the money back because liver certain goods to the consignee un-
the rule of caveat emptor applies on less the consignee pays an illegal charge
him. for the carriage. The consignee pays the
(c) Yes, A can get the money paid in amount demanded to get the goods.
advance because in this case there is Issue: Can consignee get back the
bilateral mistake as to the subject mat- amount?
ter. Decision:
149. Fact: M buys a painting from N, which (a) Yes, the consignee can get his money
both believe to be the original work of a back because coercion was applied for
great artist. M therefore pays a high price getting illegal charges.
for the painting but later on discover that (b) No the consignee cannot get his mon-
it is only a modern copy of an old one. ey back because the money given for
Issue: What can M do now? illegal purposes cannot be returned
Decision: back.
(a) M can do nothing because the doc- (c) Neither (a) nor (b)
trine of caveat emptor will apply. 153. Fact: A threatens to shoot C (B’s son),
(b) M can sue N. if B does not release him (A) from a debt
(c) It is an agreement voidable at the op- which A owes to B. B signs necessary
tion of M. documents and thus releases A under the
150. Fact: A lady by falsely representing her- threat.
self to be wife of a millionaire, obtains Issue: Can B get back his loan after-
costly necklaces from a jeweller, on the wards?

CLAT.indb 108 31/03/2009 11:41:48


CIVIL LAW 109

Decision: case are such that, it is the duty of the


(a) B can get back his loan amount, as his person keeping silence to speak, the
consent for release was obtained by silence is ‘constructive fraud’ e.g., in
coercion. contracts of ‘utmost good faith’. Since
(b) No, B cannot get back his loan after- the relationship between A and B is
wards. fiduciary in nature. A is under duty to
(c) It will be the discretion of the court. disclose all facts to B.
154. Fact: A sells a horse to B knowing that the (c) A cannot be held liable because the
horse is vicious. A does not disclose the rule of caveat emptor applies on B.
nature of the horse to B. 157. Fact: X, a patient of T.B. is induced by
Issue: Is the sale valid? Y his medical attendant to agree to pay Y
Decision: an unreasonable sum for his professional
(a) Yes, the sale is valid because A is un- services.
der no obligation to disclose the na- Issue: Can Y enforce the agreement?
ture of horse to B, the rule of ‘Caveat Decision:
Emptor’ being applicable. (a) Yes Y can enforce the agreement be-
(b) Neither (a) nor (c) cause it is a professional deal and Y
(c) No sale is not valid because there has has demanded for his services to be
been misrepresentation. rendered.
155. Fact: A fraudulently informs B that A’s (b) This contract is voidable at the op-
house if free from encumbrances. B there- tion of X, a T.B. patient. Y will have
upon buys the house. The house is subject to prove that contract was not induced
to a mortgage. by undue influence.
Issue: What are the rights of B? (c) Neither (a) nor (b)
Decision: 158. Fact: A being indebted to B, the money
(a) B cannot recover back the money lender of his village, contracts a fresh loan
nor can rescind the contract nor can on terms which appear to be unconscio-
ask the seller to redeem the mortgage nable.
because the rule of caveat emptor ap- Issue: Is the contract valid?
plies on him. Decision:
(b) B can either rescind the contract or he (a) The contract is valid.
can ask for restitution and insist that (b) Contract is void.
the contract shall be performed. B can (c) The contract is voidable if the creditor
ask the seller to redeem the mortgage rebuts the assumption of undue influ-
and carry out the contract because ac- ence.
cording to Section 17 ‘fraud’ means 159. Fact: A applies to a banker for a loan at
and includes any act or omission as a time when there is a stringency in the
the law specially declares to be fraud- money market. The banker refuses to
ulent. Under section 55 of Transfer of make the loan expect at an unusually high
Property Act, seller of immoveable rate of interest. A accepts the loan on these
property is bound to disclose to the terms.
buyer all material defects in the prop- Issue: Is the contract induced by undue
erty or in seller’s title (e.g., the prop- influence?
erty is mortgaged). Decision:
(c) Neither (a) nor (b) (a) Contract is void.
156. Fact: B, A’s daughter has just come to (b) The contract is not valid because it
age. A sells to B a horse which A knows has been caused by undue influence.
to be unsound. A says nothing to B about (c) No, the contract is not influenced by
the horse’s unsoundness. undue influence. It is a transaction in
Issue: Does A’s silence amount to fraud? the ordinary course of business and
Decision: does not fall in the category of uncon-
(a) No, mere silence is not fraud. scionable transactions.
(b) As a rule, mere silence is not fraud. 160. Fact: An agent refused to hand over the
However, if the circumstances of the account books of the business at the end

CLAT.indb 109 31/03/2009 11:41:48


110 CIVIL LAW

of the term of office to a new agent sent has been a tangible consideration from
in his place unless the principal gave him both the sides.
a release from all liabilities. The principal (c) Neither (a) nor (b)
gave him the release in order to get the 164. Fact: A agrees to pay B Rs 10,000 if B
books. marries C. C was already married to D at
Issue: Is the release valid? the time of agreement.
Decision: Issue: What will be the legal position?
(a) The release is valid because to demand Decision:
one’s own rights is not unlawful. (a) Agreement between A and B is void.
(b) Neither (a) nor (c) (b) Agreement is valid.
(c) The release deed is voidable at the (c) Agreement is voidable at the option of
option of principal because it was B.
obtained by employing coercion. The 165. Fact: A agrees to sell to B “100 tonnes of
agent had threatened to detain account oil”. A deals in a wide variety of oils and
books belonging to the principal. there is nothing to show what kind of oil
161. Fact: A’s son had B’s name on a promis- was intended.
sory note. B under threat of prosecting A’s Issue: Is it a contract?
son obtains a bond from A for the amount Decision:
of the forged note. B sues on this bond. (a) Yes, there is a contract because the
Issue: Will he succeed? mere specification or non-specifica-
Decision: tion regarding the purchase of a par-
(a) No, court will set aside the bond ob- ticular oil does not vitiate the validity
tained by B from A under Section 19 of contract so the contract is valid and
enforceable.
A. When consent of one party has been
(b) No, there is no contract because un-
obtained by coercion, fraud or misrep-
certainity is involved.
resentation, the contract is voidable at
(c) Contract is voidable at the option of
the option of the aggrieved party.
B.
(b) Yes, B will win the case.
166. Fact: A agrees to sell a horse to B for
(c) Neither (a) nor (b)
Rs 10,000 if it wins a race and Rs 100 if it
162. Fact: M an old man of poor sight endorsed
does not. The horse wins the race but B re-
a bill of exchange of Rs 3,000 thinking
fuses to pay Rs 10,000 and buy the horse.
that it was a guarantee.
Issue: Can A compel B to buy the horse?
Issue: Is M liable to pay the amount? Decision:
Decision: (a) Contract is valid and hence enforce-
(a) No, M is not liable to pay the able.
amount. (b) Agreement between A and B is a wa-
(b) Neither (a) nor (c) ger so the contract is not valid and
(c) Yes, M is liable because ignorance of hence not enforceable.
law is no excuse. (c) Contract is voidable.
163. Fact: A promised to pay Rs 150 per month 167. Fact: M promises to marry S and no one
to his wife so long she remained away else and to pay Rs 2 lakhs in case of de-
from him and did not object to his living fault. M marries R, S sues M for the
in adultery. promised money but M refuses to pay.
Issue: Is the agreement valid? Issue: What will be the legal position?
Decision: Decision:
(a) No, the agreement is void, because the (a) S cannot recover anything from A, be-
consideration is such that it defeats cause the agreement between M and S
the provisions of Hindu Law. Under was in restraint of marriage.
Hindu Law, after marriage husband (b) S can sue because M has made a fraud
and wife are to live together and adul- upon S.
tery is punishable under Indian Penal (c) Neither (a) nor (b)
Code. 168. Fact: A a debtor agrees with his money
(b) The agreement is valid because there lender that he would not, without the mon-

CLAT.indb 110 31/03/2009 11:41:48


CIVIL LAW 111

ey lender’s written consent, leave his job, Decision:


or borrow money or dispose of his prop- (a) No, because it is opposed to public
erty or change his residence. policy.
Issue: Is the agreement valid? (b) Yes, because there is a valid consider-
Decision: ation.
(a) Agreement is not valid. (c) Neither (a) nor (b)
(b) Agreement is valid. 173. Fact: A borrows Rs 50,000 from B for
(c) Agreement is voidable. doing gambling. A subsequently does not
169. Fact: A, a tailor, employs B as his assis- return the money.
tant under a contract by which B aggress Issue: Can B sue A?
not to carry on the business as a tailor Decision:
within 6 km of A shop, on the termination (a) B can do nothing, because B knew the
of his employment. purpose for which A took the money
Issue: Is the contract valid? was unlawful and against the public
Decision: policy.
(a) The contract between A and B is not (b) B can sue A because it does not mat-
valid being an agreement in restraint ter for which the money has been bor-
of trade. rowed.
(b) The contract is valid because there is (c) Neither (a) nor (b)
no use of undue influence. 174. Fact: A promises B to drop a prosecution
(c) The contract is voidable. which he has instituted against B for thef-
170. Fact: X lends Rs 20,000 to Y to enable Y tand in return B shall restore the goods to
to obtain divorce from her husband. Y in A.
turn promises that she will marry X after Issue: Can A enforce this promise?
getting the divorce. Y obtains divorce and Decision:
refuses to marry X. (a) Yes, A can enforce this promise
Issue: Can X sue Y? (b) No, A can’t enforce this promise be-
Decision: cause it’s object, that is, droping a
(a) Yes prosecution already instituted for theft
(b) No is forbidden by law.
(c) Neither (a) nor (b) (c) Neither (a) nor (b)
171. Fact: Mr X, the father of a minor son, 175. Fact: A promises to look after on behalf
agrees to transfer the guardianship of his of B, a legal manufacture of liquors and
son in favour of Mr Y and also agrees that an illegal traffic in other narcotic drugs. B
he will never revoke the transfer. But after promises to pay A salary of Rs 2,000 per
a few days, Mr X wants to get back the mensum.
guardianship of his son and files a suit for Issue: Is the agreement valid?
the same. Decision:
Issue: What will be the legal position in (a) The agreement is void and illegal,
this case? because a part of object is legal and
Decision: another part is unlawful and both the
(a) The agreement between Mr X and parts are inseparable because consid-
Mr Y is void because it is in restraint eration of Rs 2,000 p.m. is for both
of parental rights and thus opposed to promises. Where a legal part can-
public policy. Thus Mr X can get back not be separated from illegal part, in
the guardianship of his son. promise for single money, the whole
(b) Neither (a) nor (c) agreement is illegal and void.
(c) He cannot get back the guardianship (b) The agreement is valid because both
of his son because he has voluntarily the functions are not unlawful and A
transferred the guardianship. can sue B for the recovery of payment
172. Fact: X, promises to pay Rs 5 lakhs to for lawful purposes.
Y, if Y secures for X an employment in a (c) It will be the discretion of the court.
public office. 176. Fact: A instructs B to enter on his behalf
Issue: Is the agreement valid? into a wagering transaction. B loses in the

CLAT.indb 111 31/03/2009 11:41:48


112 CIVIL LAW

transaction and pays from his pocket. He Decision:


thereafter sues A for reimbursement. (a) The agreement is vague and hence not
Issue: Can A raise the plea of wager? enforceable.
Decision: (b) Here, the price is capable of being
(a) Yes, A can raise the plea of wager be- made certain. So, there is no uncer-
cause the agreement in itself was to tainty or vagueness in terms. The
enter into a wagering transaction. agreement is valid.
(b) No, A cannot raise the plea of ‘wa- (c) The agreement is voidable.
ger’. Though ‘agreements by way of 180. Fact: A aggress to sell B, “My White
wager’ are void according to Section House” for Rs 500 or 1,000.
30 of the Contract Act but transactions Issue: Is the agreement valid?
collateral to the void agreements are Decision:
valid. So, B is entitled to recover the (a) The agreement is invalid because
amount from A. there is nothing to show as to which
(c) Such propositions of law are not cov- of the two prices is to be paid. There
ered under the bare provisions of law. is no certainty of prices.
Hence it is not tenable and not en- (b) The agreement is valid.
forceable. (c) The agreement is voidable.
177. Fact: A machinegun manufacturer sold 181. Fact: A promises B in consideration of
his business to a company in consideration Rs 1,000 never to marry a particular indi-
of some amount of goodwill and conve- vidual.
nanted that he would not carry on a similar Issue: Is the agreement valid?
business for a certain time anywhere in the Decision:
world. (a) No, an agreement restraining a person
Issue: Is such a covenant binding upon from marrying anybody or from mar-
him? rying anybody except a particular per-
Decision: son is invalid.
(a) The covenant is not binding upon him (b) The agreement is valid.
because the restrictions are unreason- (c) The agreement is voidable.
able. 182. Fact: A promises B in consideration of
Rs 1,000 never to marry throughout his
(b) The covenant is binding because the
life.
restrictions imposed are the outcome
Issue: Is the agreement valid?
of the mutual understanding between
Decision:
the two parties.
(a) The agreement is valid.
(c) The covenant is voidable at the option
(b) The agreement is void.
of buyer company.
(c) The agreement is voidable.
178. Fact: A and B made an agreement to enter
183. Fact: X, a shopkeeper agrees to pay Y,
into wrestling contest in which the winner
who is his rival in the business, a sum of
was to be awarded the whole of the pro-
money as compensation if Y closes his
ceeds of the sale of tickets and the party
business. Y closes his business.
failing to appear on that day was to forfeit
Issue: Is the agreement valid?
Rs 500 to the other. Decision:
Issue: Is the agreement enforceable? (a) The agreement is valid.
Decision: (b) The agreement is voidable.
(a) The agreement is enforceable because (c) The agreement is void.
it is not a wagering agreement. 184. Fact: A sells goodwill of his business to
(b) Neither (a) nor (c) because the nature B, and agrees with him to refrain from
of the agreement is not clear. carrying on a similar business within the
(c) The agreement is not enforceable be- specified local limits.
cause it is a wagering agreement. Issue: Is the agreement valid?
179. Fact: A agrees to sell to B, 1,000 maunds Decision:
of rice at a price fixed by C. (a) Yes, the agreement is valid.
Issue: What is the validity of the agree- (b) The agreement is not valid.
ment? (c) The agreement is voidable.

CLAT.indb 112 31/03/2009 11:41:48


CIVIL LAW 113

185. Fact: Miss Rita agreed to sing at Balram’s years on condition that after the expiry of
theatre for a period of six months begin- three years, Y is not to practice in Chan-
ning from 1 January 1972. She further digarh on his own for a period of 5 years.
agreed not to sing at any other theatre dur- After the first three years had expired, Y in
ing this period. breach of his agreement starts practicing
Issue: Is the contract valid? in Chandigarh.
Decision: Issue: Is the agreement valid?
(a) The agreement is valid. Decision:
(b) The agreement is voidable. (a) The agreement is valid.
(c) The agreement is unreasonable. (b) The agreement is invalid.
186. Fact: X, a doctor in Chandigarh engages (c) The agreement is voidable.
Y as his assistance for a period of three

ANSWERS

1. (b) 2. (a) 3. (b) 4. (a) 5. (b) 6. (b) 7. (b) 8. (a) 9. (a) 10. (b)
11. (b) 12. (a) 13. (b) 14. (a) 15. (b) 16. (b) 17. (b) 18. (a) 19. (b) 20. (a)
21. (a) 22. (b) 23. (b) 24. (a) 25. (b) 26. (a) 27. (b) 28. (a) 29. (b) 30. (a)
31. (a) 32. (a) 33. (b) 34. (a) 35. (a) 36. (b) 37. (b) 38. (b) 39. (a) 40. (b)
41. (a) 42. (b) 43. (a) 44. (b) 45. (a) 46. (b) 47. (a) 48. (b) 49. (a) 50. (a)
51. (b) 52. (a) 53. (a) 54. (b) 55. (a) 56. (a) 57. (b) 58. (b) 59. (b) 60. (a)
61. (a) 62. (a) 63. (b) 64. (a) 65. (a) 66. (b) 67. (b) 68. (b) 69. (a) 70. (b)
71. (b) 72. (a) 73. (a) 74. (b) 75. (a) 76. (b) 77. (b) 78. (c) 79. (b) 80. (a)
81. (b) 82. (a) 83. (b) 84. (a) 85. (b) 86. (a) 87. (b) 88. (a) 89. (b) 90. (a)
91. (b) 92. (a) 93. (c) 94. (b) 95. (b) 96. (c) 97. (b) 98. (b) 99. (a) 100. (a)
101. (a) 102. (b) 103. (c) 104. (a) 105. (b) 106. (c) 107. (a) 108. (b) 109. (c) 110. (a)
111. (b) 112. (c) 113. (a) 114. (c) 115. (b) 116. (a) 117. (b) 118. (c) 119. (a) 120. (b)
121. (c) 122. (a) 123. (b) 124. (c) 125. (a) 126. (b) 127. (a) 128. (a) 129. (b) 130. (a)
131. (b) 132. (a) 133. (a) 134. (b) 135. (a) 136. (a) 137. (b) 138. (c) 139. (a) 140. (a)
141. (b) 142. (a) 143. (a) 144. (b) 145. (a) 146. (a) 147. (b) 148. (c) 149. (a) 150. (b)
151. (c) 152. (a) 153. (a) 154. (a) 155. (b) 156. (b) 157. (b) 158. (c) 159. (c) 160. (c)
161. (a) 162. (a) 163. (a) 164. (a) 165. (b) 166. (b) 167. (a) 168. (a) 169. (a) 170. (b)
171. (a) 172. (a) 173. (a) 174. (b) 175. (a) 176. (b) 177. (a) 178. (a) 179. (b) 180. (a)
181. (a) 182. (b) 183. (c) 184. (a) 185. (a) 186. (b)

CLAT.indb 113 31/03/2009 11:41:48


Chapter 4
CONSTITUTIONAL LAW
WHAT IS CONSTITUTION? Constitution. The judiciary is vested with power
to declare a law unconstitutional, if the law is
A Constitution means a document having found to have contravened any provision of the
a special legal sanctity which sets out the Constitution.
framework and principal functions of the
government. CONSTITUTION—A LIVING
Constitution of a country gives an idea DOCUMENT
about the basic structure of the political system
under which its people are to be governed. A Constitution is the vehicle of a Nation’s
It defines the powers of the main organs of progress. It is a legal and social document. It is
the State, demarcates their responsibilities and intended to serve the needs of the day when it
regulates their relationships with each other and was enacted and also to meet the needs of the
with the people. changing conditions in new circumstances. It has,
It is the “Supreme and Fundamental thus, been said that the words and expressions,
Law” of the land which reflects people’s faith used in the Constitution, have no fixed meaning
and aspirations. and must receive interpretation based on the
experience of the people in the course of
ORGANS OF THE GOVERNMENT working of the Constitution. A Constitution, is
thus, said to be a living and organic document,
Traditionally, there are three main organs of the which, of all instruments, has the greatest claim
government of a country: to be construed broadly and liberally. Being a
1. Legislature, whose function is to make
living organ, it is ongoing and with the passage
laws, amend them or repeal them.
of time, law must be changing. Horizons of
2. Executive, which implements, executes or
Constitutional law, it is said, are expanding. It is
administers laws.
requried to be kept young, energetic and alive.
3. Judiciary, whose function is to interpret
Political, social and economic developments
and enforce laws and to administer
can throw light on its meaning.
justice.
While these three organs are the principal
organs of the government of the country, its THE MAKING OF INDIAN
Constitution may also provide for the creation of CONSTITUTION
other organs or institutions which it may consid-
er as significant and fit for inclusion in the Con- The idea to have a Constitution was given by
stitution. For example, the Constitution of India M. N. Roy (Political Philosopher). The present
provides for the setting up of a Finance Com- Constitution was framed by the Constituent
mission for the settlement of the distribution of Assembly of India set up under Cabinet Mission
revenue between the Centre and the States as a Plan of 1946. The first meeting of the Constituent
whole and between the States inter se. Assembly took place on 9 December 1946 with
The governmental organs owe their ori- Dr Sachidanand Sinha as its interim President.
gin to the Constitution and drive their authority Dr Rajendra Prasad was elected as its President
from, and discharge their responsibilities, within on 11 December 1946.
the framework of the Constitution. The Assembly had 13 committees for fram-
None of the organs is sovereign. The validity of ing the Constitution. The all-important Drafting
a law, whether passed by the Union Parliament Committee, which bore the responsibility of
or a State Legislature, is judged with reference drafting the Constitutional document was formed
to their respective jurisdiction as defined in the on 29 August 1947. Its members were:

CLAT-4-Constitution.indd 114 01/09/2010 17:28:27


CONSTITUTIONAL LAW 115

1. Dr B. R. Ambedkar—Chairman (d) Dominance of legal luminaries in the


2. N. Gopalaswami Ayyangar Constituent Assembly.
3. Alladi Krishnaswami Ayyar (a 2. Derived from Different Sources. The
distinguished jurist) Constitution of India can be divided into 3
4. K. M. Munshi (a distinguished jurist) basic parts: Structural, Political and Philo-
5. Syyed Mohd. Saadulla sophical.
6. N. Madhav Rao (in place of B.L. Mitra) The structural part of the Constitution is,
7. D. P. Khaitan (T. Krishnamachari, after to a large extent, derived from the government
Khaitan’s death in 1948) of India Act 1935. The philosophical part of
It was finally passed and accepted on 26 the Constitution (the fundamental rights and
November 1949. The last session of the As- the directive principles of state policy) derive
sembly was held on 24 January 1950, which their inspiration from American and Irish Con-
unanimously elected Dr Rajendra Prasad as the stitutions. The political part of the Constitution
President of India. In all, 284 members of the (the principle of cabinet government, leadership
Assembly signed the official copies of the In- of Prime Minister, bicammerailism and the re-
dian Constitution which came into effect on 26 lationship between legislature and executive)
January 1950, known and celebrated as the Re- have been derived from the Constitution of UK.
public Day of India. The other provision of Constitution have been
Although the Constitution was ready on 26 derived from the Constitutions of Canada, Aus-
November 1949 but was delayed till 26 Janu- tralia, Germany, Africa and Russia.
ary 1950 because in 1929 on this day, the Indian 3. Blend of Rigidity and Flexibility. The
National Congress demanded ‘Poorna Swaraj’ Indian Constitution is partly flexible and
in Lahore session under J. L. Nehru. [some of partly rigid. Some of its parts can be
the provisions as those related to citizenship, amended by simple majority. But some
elections, provisional Parliament etc. were of the parts can be amended by special
given immediate effect]. Constituent Assembly majority.
took 2 years 11 months 18 days to complete the 4. It is federal in form and unitary in spirit.
Constitution. Originally it had 395 articles (450 The Constitution of India establishes a
at present) and 8 schedules (12 at present) and federal system of government. Federalism
22 parts (24 at present). Constituent Assembly means division of powers between
adopted our National Flag on 22 July 1947. It Centre and the States. The Unitary form
was designed by Pingali Venkaiah of Andhra of government means when there is tilt
Pradesh. towards Union Government inspite of
division of powers between the Centre
SALIENT FEATURES OF THE and States. So, the Unitary form means
INDIAN CONSTITUTION when the Centre is strong. In India, there
is formal division of powers between the
1. Lengthiest Written Constitution. The Centre and the States. All the subjects have
Constitution of India is the lengthiest of been divided into three lists: Union List,
all the Constitutions of the world. Origi- State List and Concurrent List, but the
nally, it contained 395 articles, 22 parts residuary powers lies with the Centre. It
and 8 schedules. Presently, it consists of contains all usual features of a federation,
450 articles, 24 parts and 12 schedules. viz., two government, division of powers,
The reasons responsible for the bulky size written Constitution, supremacy of
of the Indian Constitution are: Constitution, rigidity of Constitution,
(a) Geographical factors, that is, the independent judiciary and bicameralism.
vastness of the country and its However, the Indian Constitution also con-
diversity. tains a large number of unitary or non-federal
(b) Historical factors, e.g., the influence of features, viz., a strong Centre, single Constitu-
the Government of India Act of 1935, tion, single citizenship, flexibility of Constitu-
which was bulky. tion, integrated judiciary, appointment of state
(c) Single Constitution for both the Centre governor by the Centre, all-India services, emer-
and the States except Jammu and gency provisions, and so on.
Kashmir. 5. Parliamentary form of Government. The

CLAT.indb 115 31/03/2009 11:41:49


116 CONSTITUTIONAL LAW

Constitution of India has opted for the 9. Directive Principles of State Policy—Ar-
British Parliamentary System of Govern- ticle 36–51. Part IV Article 36–51 contains
ment rather than American Presidential a charter of Directive Principles of State
System of Government. The parliamen- Policy. They are in the form of directions/
tary system is based on the principle of instructions to the successive governments
cooperation and co-ordination between to be taken into consideration at the time
the legislature and executive organs while of framing of any rule/regulation policy or
the presidential system is based on the programme for the equal distribution of
doctrine of separation of powers among resources and providing economic rights
the three organs. to the weaker sections of the society. But
Even though the Indian Parliamentary Sys- they could not be translated into reality till
tem is largely based on the British pattern, there the date because of lack of political will,
are some fundamental differences between the lack of resources, over population and ab-
two. For example, the Indian Parliament is not sence of any unanimous national policy
a sovereign body like the British Parliament. cutting across the class, caste, religion,
Further, the Indian State has an elected head region and gender differences. The non-
(republic) while the British State has hereditary enactment of uniform civil code under Ar-
head (monarchy). ticle 44 is a blatant example of sheer lack
6. Indian Constitution adopted synthesis of of political will and misuse of political
Parliamentary Sovereignty and Judicial positions for political gains and personal
Supremacy. Unlike the British Parliamen- aggrandisement.
tary sovereignty and American Judicial 10. Fundamental Duties. The fundamental
Supremacy, Indian Constitution preferred duties were incorporated through 42nd
the synthesis of both. The Indian Parlia- Amendment Act, 1976. They are con-
ment is the sovereign law making body, tained in Part IV A article 51 A. Originally
but Indian judiciary has the power of judi- there were 10 fundamental duties. Now
cial review which empowers the judiciary there are 11 fundamental duties. The 11th
to review any law made by the legislature fundamental duty has been incorporated
and declare it ultra vires if it contravenes through 86th Amendment Act, 2002. It
the Constitution. makes the fundamental duty of the parent
7. Indian Constitution has single, unified, or the guardian to impart compulsory edu-
integrated and independent judiciary.
cation to the children between the age of
Indian Constitution has made various pro-
6–14 years old.
vision to ensure the independence of ju-
11. Secular State. The Constitution of India
diciary. The security of the tenure of the
declares India to be a secular state in its
judges, fixed service conditions, ban on
preambular Article. Though the word
practice after retirement, power to punish
Secularism was not there in the preamble
for contempt of court are some of the pro-
at the outset. It was later on incorporated
vision outlined in the Constitution.
through 42nd Amendment Act, 1976. The
8. Fundamental Rights. Part III of the Con-
word secularism means that India as a state
stitution guarantees six fundamental rights
has no religion of its own. The state which
to all the citizens:
has its own religion is known as theocratic
(a) Right to Equality (Article 14 – 18);
(b) Right to Freedom (Article 19 – 22); states. Unlike that, India is a secular state
(c) Right against Exploitation (Articles which means it neither favours nor disfa-
23 – 24); vours any religion. It is neither (re)ligious,
(d) Right to Freedom of Religion (Articles nor (irre)ligious, nor (anti)religious, nor
25 – 28); (pro)religious.
(e) Cultural and Educational Rights 12. Universal Adult Franchise. The Indian
(Articles 29 – 30); Constitution adopts universal adult fran-
(f) Right to Constitutional Remedies chise as a basis of elections to the Lok
(Article 32). Sabha and the state legislative assemblies.
Note: Right to Property (Article 31) Every citizen who is not less than 18 years
repealed—now no more a fundamental right but of age has a right to vote without any dis-
a legal right under Article 300 A; and crimination of caste, race, religion, sex,

CLAT.indb 116 31/03/2009 11:41:49


CONSTITUTIONAL LAW 117

literacy, wealth and so on. The voting age 7. Independence of Judiciary and judicial
was reduced to 18 years from 21 years in review
1989 by the 61st Constitutional Amend- 8. Preamble
ment Act, Act of 1988. 9. Removal of Supreme Court and High
13. Single Citizenship. Though the Indian Court judges
Constitution is federal and envisages a
From USSR
dual polity (Centre and States), it provides
for a single citizenship, that is, the Indian 1. Fundamental Duties
citizenship. 2. Five Year Plan
14. Independent Bodies. Bulwarks of democ- From Australia
racy—The Indian Constitution not only
1. Concurrent list
provides for the legislative, executive and
2. Language of the preamble
judicial organs of the government (Central
3. Provision regarding trade, commerce and
and State) but also establishes certain in-
intercourse
dependent bodies. They are envisagd by
the Constitution as the bulwarks of the From Japan
democratic system of Government in In- 1. Law on which the Supreme Court func-
dia. These are: tions
(a) Election Commission
(b) Comptroller and Auditor-General From Weimar Constitution of Germany
(c) Union Public Service Commission 1. Suspension of Fundamental Rights during
(d) State Public Service Commission the emergency
15. Three-tier Government. Originally, the
Indian Constitution provided for dual poli- From South Africa
ty i.e., Centre and States. But through 73rd 1. Procedure of Constitutional Amendment
and 74th Amendment Act, Acts 1991, 3rd Act(s).
tier of Governance (Panchayats and Mu-
From Canada
nicipalities) rural and urban local self gov-
ernments, has been introduced. 1. Scheme of federation with strong center
2. Distribution of powers between Centre
BORROWED FEATURES OF THE and the States and placing Residuary
Powers with the Centre
CONSTITUTION
From Ireland
From UK
1. Concept of Directive Principles of State
1.
Nominal Head —President (like Queen) Policy (Ireland borrowed it from Spain)
2.
Cabinet System of ministers 2. Method of election of the President
3.
Post of the Prime Minister 3. Nomination of members in the Rajya
4.
Parliamentary Type of Government Sabha by the President
5.
Bicameral Parliament
6.
Lower House more powerful SCHEDULES IN CONSTITUTION
7.
Council of Ministers responsible to Lower
House First Schedule
8. Speaker in the Lok Sabha
List of States and Union Territory.
9. Single citizenship
Second Schedule
From US
Salary of President, Governors, Chief Judges,
1. Written Constitution Judges of High Court and Supreme Court,
2. Executive head of State known as President Comptroller and Auditor General.
and his being the Supreme Commander of
the Armed Forces Third Schedule
3. Vice-President as the ex-officio Chairman Forms of oaths and affirmations.
of Rajya Sabha
4. Fundamental Rights Fourth Schedule
5. Supreme Court Allocate seats for each State of India in Rajya
6. Provision of States Sabha.

CLAT.indb 117 31/03/2009 11:41:49


118 CONSTITUTIONAL LAW

Fifth Schedule ARTICLES OF THE


Administration and control of Scheduled Areas CONSTITUTION
and Tribes.
Sixth Schedule Constitution at a glance through Articles
Provisions for administration of Tribal Area IMPORTANT ARTICLES
in Assam, Meghalaya, Tripura, Mizoram and
Arunachal Pradesh. Union and its Territory
Seventh Schedule 1. Name and territory of the union
Gives allocation of powers and functions be- 3. Formation of new states and alteration
tween Union and States. It contains 3 lists. of areas, boundaries or names of
1. Union list (for Central Govt): 99 subjects existing states
2. States list (Powers of State Govt): 66 Citizenship
subjects
3. Concurrent list (Both Union and States): 5. Citizenship at the commencement of
46 subjects the Constitution
11. Parliament to regulate the right of citi-
Eighth Schedule zenship by law
List of 22 languages of India recognized by
Fundamental Rights
Constitution.
1. Assamese 2. Bengali 12. Definition of state
3. Gujarati 4. Hindi 13. Laws inconsistent with or in deroga-
5. Kannada 6. Kashmir tion of the fundamental rights—judi-
7. Konkanis 8. Malayalam cial reviews
9. Manipuris 10. Marathi 14. Equality before law
11. Nepalis 12. Oriya 15. Prohibition of discrimination on
13. Punjabi 14. Sanskrit grounds of religion, races, caste, sex
15. Sindhi+ 16. Tamil or place of birth.
17. Telegu 18. Urdu 16. Equality of opportunity in matters of
19. Santhali# 20. Bodo# public employment.
21. Maithili# 22. Dogri# 17. Abolition of untouchability
+ [Sindhi was added in 1967 by 21st Amendment 18. Abolition of titles
Act] 19. Protection of certain rights regarding
s
[Konkani, Manipuri and Nepali were added in freedom of speech, etc.
1992 by 71st Amendment Act] 20. Protection in respect of conviction for
# [Santhali, Maithili, Bodo and Dogri were offences
added in 2003 by 92nd Amendment Act]
21. Protection of life and personal liberty
Ninth Schedule 21 A. Right to elementary education
Added by 1st Amendment Act, in 1951. Contains 22. Protection against arrest and detention
acts and orders related to land tenure, land tax, in certain cases
railways, industries. [Right of Property not a 23. Prohibition of traffic in human beings
fundamental right now]. and enforced labour
24. Prohibition of employment of chil-
Tenth Schedule dren in factories, etc.
Added by 52nd Amendment Act, in 1985. 25. Freedom of conscience and free pro-
Contains provisions of disqualification of fession, practice and propagation of
grounds of defection. religion
Eleventh Schedule 26. Freedom to manage religious affairs
27. Freedom as to payment of taxes for
By 73rd Amendment Act, in 1992. Contains
promotion of any particular religion
provisions of Panchayati Raj
28. Freedom as to attendance at religious
Twelfth Schedule instruction or religious worship in cer-
By 74th Amendment Act, in 1992. Contains pro- tain educational institutions
visions of Municipal Corporation. 29. Protection of interests of minorities

CLAT.indb 118 31/03/2009 11:41:49


CONSTITUTIONAL LAW 119

30. Right of minorities to establish and 49. Protection of monuments and places
administer educational institutions and objects of national importance
31. (Repealed) 50. Separation of judiciary from execu-
31 A. Saving of laws providing for acquisi- tive
tion of estates, etc. 51. Promotion of international peace and
31 B. Validation of certain acts and regula- security
tions
Fundamental Duties
31 C. Saving of laws giving effect to certain
directive principles 51 A. Fundamental duties
31 D. (Repealed) President and Vice-President
32. Remedies for enforcement of funda-
52. The president of India
mental rights including writs
53. Executive power of the union
32 A. (Repealed)
54. Election of president
33. Power of Parliament to modify the
55. Manner of election of president
fundamental rights in their application
56. Term of office of president
to forces, etc. 57. Eligibility of re-election
34. Restriction on fundamental rights 58. Qualifications for election as presi-
while martial law is in force in any dent
area 61. Procedure for impeachment of the
35. Legislation to give effect to some of president
the provisions of fundamental rights. 63. The vice-president of India
Directive Principles of State Policy 64. The vice-president to be ex–officio
chairman of the council of states.
36. Definition of State
66. Election of vice-president
37. Application of the directive princi-
67. Term of office of vice-president
ples
70. Discharge of president’s functions in
38. State to secure a social order for the
other contingencies
promotion of welfare of the people
72. Power of president to grant pardons,
39. Certain principles of policy to be fol-
etc., and to suspend, remit or com-
lowed by the State
mute sentences in certain cases
39 A. Equal justice and free legal aid
40. Organization of village panchayats Union Ministers and Attorney-General
41. Right to work, to education, and to 74. Council of ministers to aid and advise
public assistance in certain cases president
42. Provision for just and humane condi- 76. Attorney-General for India
tions of work and maternity relief 78. Duties of prime minister as respects
43. Living wage, etc., for workers the furnishing of information to the
43 A. Participation of workers in manage- president, etc.
ment of industries
44. Uniform civil code for the citizens Parliament
45. Provision for early childhood care and 80. Composition of the council of states
education to children below the age of 81. Composition of the House of the
six years people
46. Promotion or educational and eco- 86. Right of president to address and send
nomic interests of scheduled castes, messages to Houses
scheduled tribes and other weaker 102. Disqualifications for membership
sections 110. Definition of ‘money bills’
47. Duty of the state to raise the level of 120. Language to be used in Parliament
nutrition and the standard of living
and to improve public health Supreme Court
48. Organization of agriculture and ani- 124. Establishment and Constitution of the
mal husbandry Supreme Court
48 A. Protection and improvement of envi- 126. Appointment of acting Chief Justice
ronment and safeguarding of forests 127. Appointment of adhoc judges
and wild life 129. Supreme Court to be a court of record

CLAT.indb 119 31/03/2009 11:41:49


120 CONSTITUTIONAL LAW

131. Original jurisdiction of the Supreme Subordinate Courts


Court 233. Appointment of district judges
132. Appellate jurisdiction of Supreme 235. Control over subordinate courts
Court in appeals from high courts in
certain cases. Union Territories
137. Review of judgements or orders by 239. Administration of union territories
the Supreme Court 239 AA. Special provisions with respect to
139 A. Transfer of certain cases Delhi
143. Power of president to consult Supreme
Court Panchayats
243 B. Constitution of panchayats
Comptroller and Auditor-General of 243 D. Reservation of seats
India 243 G. Powers, authority and responsibilities
148. Comptroller and Auditor General of of panchayats
India 244 K. Elections to the panchayats
149. Duties and powers of the Comptroller
Municipalities
and Auditor General
243 Q. Constitution of municipalities
Governor 243 X. Power to impose taxes by, and funds
153. Governors of states of, the municipalities
154. Executive power of state
Centre–State Legislative Relations
155. Appointment of governor
156. Term of office of governor 248. Residuary powers of the legislation
157. Qualification for appointment as Right to Property
governor
160. Discharge of the functions of the 300 A. Persons not to be deprived of property
governor in certain contingencies save by the authority of law
161. Power of governor to grant pardons, Public Services
etc., and to suspend, remit or com- 312. All-India services
mute sentences in certain cases
Public Service Commissions
State Ministers and Advocate-General
315. Public Service Commissions for the
165. Advocate-General for the state Union and for the States
State Legislature 316. Appointment and term of office of
members
169. Abolition or creation of legislative
317. Removal and suspension of a member
councils in states
of a Public Service Commission
175. Right of governor to address and send
messages to the House of Houses Tribunals
191. Disqualifications for membership 323 A. Administrative tribunals
199. Definition of ‘money bills’.
210. Language to be used in the legislature Elections
213. Power of governor to promulgate or- 324. Superintendence, direction and con-
dinances during recess of legislature trol of elections to be vested in an
Election Commission.
High Courts
215. High courts to be courts of record Special Provisions for SCs, STs and
216. Constitution of high courts Others
217. Appointment and conditions of the of- 331. Representation of the Anglo-Indian
fice of a judge of a high court community in the House of the
221. Salaries etc., of judges people
223. Appointment of acting Chief Justice 333. Representation of the Anglo-Indian
227. Power of superintendence over all community in the legislative assem-
courts by the high court blies of the states

CLAT.indb 120 31/03/2009 11:41:49


CONSTITUTIONAL LAW 121

Official Language Major ports


343. Official language of the Union Airways; aircraft and air navigation; provi-
348. Language to be used in the Supreme sion of aerodromes
Court and in the high courts and for Posts and telegraphs; telephones, wireless,
acts, bills, etc. broadcasting and other like forms of com-
munication
Emergency Provisions Currency, coinage and legal tender; foreign
352. Proclamation of emergency (national exchange
emergency) Reserve Bank of India
356. Provisions in case of failure of Con- Post office savings bank
stitutional machinery in states (presi- Lotteries organized by the Union or State
dent’s rule) Banking
360. Provisions as to financial emergency Insurance
Stock exchange and futures markets
Amendment Act, of the Constitution Oil fields and mineral oil resources; petro-
368. Power of Parliament to amend the leum and petroleum products; other liquids
Constitution and procedure thereof and substances which are inflammable.
Regulation of mines and mineral develop-
Temporary, Transitional and Special ment in the public interest
Provisions Fishing and fisheries beyond territorial
370. Temporary provisions with respect to waters
the State of Jammu and Kashmir National Library, the Indian Museum,
the Imperial War Museum, the Victoria
SUBJECTS OF UNION, STATE Memorial and the Indian War Memorial,
and any other like institutions of national
AND CONCURRENT LISTS
importance
Union Public Services; All-India Services;
Union List (List-I) Union Public Service Commission
Defence of India Elections to Parliament, the legislatures of
Naval, military and air forces; any other states and the offices of president and vice-
armed forces of the Union president; the Election Commission
Defence industries Organization, jurisdiction and powers of
Central Bureau of Intelligence and inves- the Supreme Court
tigation Organization of the high courts
Preventive detention for reasons connected Taxes on income other than agricultural
with defence, foreign affairs, or the secu- income
rity of India Duties of customs including export duties
Foreign affairs Duties of excise on tobacco and other
Diplomatic, consular and trade representa- goods except alcoholic liquors for human
tion consumption and opium, Indian hamp and
Treaties, agreements and conventions with other narcotic drugs and narcotics, but in-
foreign countries cluding medicinal and toilet preparations
containing alcohol
War and peace
Corporation tax
Foreign jurisdiction
Taxes on the capital value of the assets (ex-
Citizenship, naturalization and aliens
clusive of agricultural land) of individu-
Extradition
als and companies; taxes on the capital of
Passports and visas
companies
Pilgrimages to places outside India
Estate duty in respect of property other
Piracies and crimes committed on the high
than agricultural land
seas or in the air and offences against the
law of nations. State List (List-II)
Railways Pubic order
Shipping and navigation on national Police
waterways Local government

CLAT.indb 121 31/03/2009 11:41:49


122 CONSTITUTIONAL LAW

Public health and sanitation Governors of states outside their respective


Intoxicating liquors states
Agriculture, including agricultural educa- 9. Judges of the Supreme Court
tion and research 9 A. Chief Election Commissioner
Water that is water supplies, irrigation and Comptroller and Auditor General
canals, drainage and embankments, water 10. Deputy Chairman, Rajya Sabha
storage and water power Deputy Chief Ministers of states
Land, that is, right in or over land, land ten- Deputy Speaker, Lok Sabha
ures and the collection of rents Members of Planning Commission
Fisheries Ministers of state of the Union and any
Betting and gambling other minister in the ministry of defence
States public services; State Public Service for defence matters
Commission 11. Attorney General of India,
Land revenue, including maintenance on Cabinet Secretary,
land records Lt Governors within their respective
Taxes on agricultural income Union Territories
12. Chiefs of staff holding the rank of full
Concurrent List (List-III)
general or equivalent rank
Criminal Law, including all matters includ- 13. Envoys extraordinary and ministers pleni-
ed in the Indian Penal Code potentiary accredited to India
Marriage and divorce; infants and minors; 14. Chairman and Speaker of state legislatures
adoption; wills intestacy and succession; within their respective states,
joint family and partition Chief justice of high courts within their
Civil procedure, including all matters respective jurisdiction
included in the code of Civil Procedure
Forests CONSTITUTIONAL AMENDMENT
Protection of wild animals and birds ACTS AT A GLANCE
Adulteration of foodstuffs and other goods
Population control and family planning First Amendment Act, 1951
Trade unions; industrial and labour dis- 1. Empowered the state to make special pro-
putes visions for the advancement of socially
Education, including technical education, and economically backward classes.
medical education, and universities 2. Provided for the saving of laws providing
Electricity for acquisition of estates, etc.
3. Added Ninth Schedule to protect the land
TABLE OF PRECEDENCE reform and other laws included in it from
the judicial review.
1. President 4. Added three more grounds of restrictions
2. Vice-president on freedom of speech and expression, viz.,
3. Prime minister public order, friendly relations with for-
4. Governors of states within their respective eign states and incitement to an offence.
states Also, made the restrictions ‘reasonable’
5. Former presidents and thus, justiciable in nature.
5 A. Deputy prime minister 5. Provided the state trading and nationaliza-
6. Chief Justice of India tion of any trade or business by the state is
Speaker of Lok Sabha not to be invalid on the ground of violation
7. Cabinet ministers of the Union of the right to trade or business.
Chief ministers of states within their Twenty-first Amendment Act, 1967
respective states Included Sindhi as the 15th language in
7 A. Holders of Bharat Ratna decoration the Eight Schedule.
8. Ambassadors extraordinary and plenipo- Twenty-fourth Amendment Act, 1971—came
tentiary and high commissioners of Com- as reaction to Golak Nath
monwealth countries accredited to India, 1. Affirmed the power of Parliament to
Chief Ministers of states outside their amend any part of the Constitution in-
respective states, cluding fundamental rights.

CLAT.indb 122 31/03/2009 11:41:49


CONSTITUTIONAL LAW 123

2. Made it compulsory for the president to Sabha and the state legislative assemblies
give his assent to a Constitutional Amend- (i.e., 5 years)
ment Act, Bill. 2. Restored the provisions with regard to
Thirty-first Amendment Act, 1972 quorum in the Parliament and state legis-
Increased the number of Lok Sabha seat latures.
from 525 to 545. 3. Empowered the president to send back
Forty-first Amendment Act, 1976 once the advice of cabinet for reconsidera-
Raised the retirement age of members tion. But, the reconsidered advice is to be
of State Public Service Commission and binding on the president.
Joint Public Service Commission from 60 4. Restored some of the powers of the Su-
to 62. preme Court and high courts.
Forty-second Amendment Act, 1976 (The 5. Replaced the term ‘internal disturbance’
most comprehensive Amendment Act, made by ‘armed rebellion’ in respect of national
so far to the Constitution; it is known as emergency.
‘Mini-Constitution’; it gave effect to the rec- 6. Made the President to declare a national
ommendations of Swaran Singh Committee) emergency only on the written recommen-
1. Added three new words (i.e., socialist, dation of the cabinet.
secular and integrity in the Preamble). 7. Deleted the right to property from the list
2. Added Fundamental Duties by the citizens of Fundamental Rights and made it only a
(new Part IV A). legal right.
3. Made the president bound by the advise of 8. Provided that the fundamental rights
the cabinet. guaranteed by Articles 20 and 21 cannot be
4. Froze the seats in the Lok Sabha and state suspended during a national emergency.
legislative assemblies on the basis of 1971 9. Omitted the provisions which took away
census till 2001. the power of the court to decide the elec-
5. Made the Constitutional Amendment Acts tion disputes of the president, the Vice-
beyond judicial scrutiny. President, the prime minister and the
6. Curtailed the power of judicial review and Speaker of the Lok Sabha.
writ jurisdiction of the Supreme Court and Fifty-second Amendment Act, 1978
high courts. (popularly known as Anti-Defection Law)
7. Added three new Directive Principles viz., Provided for disqualification of members
equal justice and free legal aid, participa-
of Parliament and state legislatures on
tion of workers in the management of in-
the ground of defection and added a new
dustries and protection of environment,
Tenth Schedule containing the details in
forests and wild life.
this regard.
8. Extended the one-time duration of the
Sixty-first Amendment Act, 1989
President’s rule in a state from 6 months
Reduced the voting age from 21 years to
to one year.
18 years for the Lok Sabha and state legis-
9. Shifted five subjects from the state list to
lative assembly elections.
the concurrent list, viz., education, for-
Seventy-first Amendment Act, 1992
ests, protection of wild animals and birds,
Included Konkani, Manipuri and Nepali
weights and measures and administration
languages in the Eight Schedule. With
of justice, Constitution and organisation
of all courts except the Supreme Court and this, the total number of scheduled lan-
the high courts. guages increased to 18.
10. Did away with the requirement of quorum Seventy-seventh Amendment Act, 1995
in the Parliament and the State legisla- Provided for reservation in promotions in
tures. government jobs for SCs and STs. This
11. Provided for the creation of the All-India Amendment Act, nullified the Supreme
Judicial Service. Court ruling with regard to reservation in
Forty-fourth Amendment Act, 1978 (Enacted promotions.
by the Janata Government mainly to nullify Eighty-fourth Amendment Act, 2001
some of the other distortions introduced by Extended the ban on readjustment of seats
the 42nd Amendment Act, 1976) in the Lok Sabha and the state legislative
1. Restored the original term of the Lok assemblies for another 25 years (i.e., up

CLAT.indb 123 31/03/2009 11:41:49


124 CONSTITUTIONAL LAW

to 2026) with the same objective of en- tution of India. It is a key to the minds of
couraging population limiting measures. the makers of the Constitution. The sig-
In other words, the number of seats in the nificance of the Preamble lies in the fact
Lok Sabha and the assemblies are to re- that it tells us about the three things i.e.,
main same till 2026. It also provided for ultimate source of authority, form of gov-
the readjustment and rationalization of ernment and form of state, and the objec-
territorial constituencies in the states on tives to be attained.
the basis of the population figures of 1991 2. Preamble declares that sovereignty lies
census. ultimately with the people of India. The
Eighty-sixth Amendment Act, 2002 Preamble declares that it is the people of
1. Made elementary education a fundamen- India who had enacted, adopted and given
tal right. The newly added, Article 21-A, the Constitution to themselves. It is the
declares that “the State shall provide free people of India who are ruled and ruling,
and compulsory education to all children who are elected and electing.
of the age of six to fourteen years in such 3. The Preamble declares India to be a Sov-
manner as the State may determine”. ereign, Socialist, Secular, Democratic,
2. Changed the subject matter of Article 45 Republic.
in Directive Principles. It now reads— Sovereignty means we are independent
“The State shall endeavour to provide in our internal and external affairs. We
early childhood care and education for all are not subjected to any outside power.
children until they complete the age of six The word socialism in the Indian con-
years”. text means removal of inequalities and
3. Added a new fundamental duty under Ar- discriminations, equal distribution of re-
ticle 51-A which reads—“It shall be the sources, equal access to public places and
duty of every citizen of India who is a par- public offices. There shall be no discrimi-
ent or guradan to provide opportunities for nation on the basis of caste, class, religion,
education to his child or ward between the race, colour, creed, sex or place of birth.
age of six and fourteen years”. The aim of socialism in the Indian context
is to remove poverty, illiteracy, ignorance
and unemployment. The Constitution of
PREAMBLE India declares India to be a secular state in
its preambular pArticle Though the word
The Preamble reads:
Secularism was not there in the preamble
“WE THE PEOPLE OF INDIA having solemnly
at the outset. It was later on incorporated
resolved to constitute India into a SOVEREIGN
through 42nd Amendment Act, 1976. The
SOCIALIST SECULAR DEMOCRATIC RE-
word secularism means that India has no
PUBLIC and to secure to all its citizens;
religion of its own. The state which has
JUSTICE social, economic and political; LIB-
its own religion is known as theocracy.
ERTY of thought, expression, belief, faith and Unlike that, India is a secular state which
worship; means it neither favours nor disfavours
EQUALITY of status and opportunity; and to any religion. It is neither (re)ligious, nor
promote among them all (irre)ligious, nor (anti)religious, nor (pro)
FRATERNITY assuring the dignity of the indi- religious. Democracy means the govern-
vidual and the unity and integrity of the Nation; ment of the people, by the people, for
IN OUR CONSTITUENT ASSEMBLY this the people. But nowadays the democracy
twenty–sixth day of November, 1949, do has been hijacked by the muscle power,
HEREBY ADOPT, ENACT AND GIVE TO money power, mafia power for the muscle
OURSELVES THIS CONSTITUTION”. power, money power, mafia power. Re-
The Preamble and Its Importance public means head of the Indian state is
not hereditary as compared to the head of
1. The ‘Objective Resolution’, proposed the British monarchy. Rather it is elected
by Pandit Nehru and passed by the Con- though indirectly.
stituent Assembly, ultimately became the 4. The Preamble declares certain objectives
Preamble to the Constitution of India. The to be achieved i.e., justice, liberty, equal-
Preamble is an introduction to the Consti- ity and fraternity.

CLAT.indb 124 31/03/2009 11:41:49


CONSTITUTIONAL LAW 125

5. As the Supreme Court has observed, the The Interim Government headed by Jawa-
Preamble is a key to unravel the minds of harlal Nehru felt it vital to bring all the princely
the makers of the Constitution. states into the fold of Indian Union in the inter-
6. The Constitution (42nd Amendment Act,) ests of India’s unity and security. Therefore it set
1976 amended the Preamble and added up Home Ministry (5 July 1947) with Sardhar
the words Socialist, Secular and Integrity Vallabhbhai patel as minister in charge and V. P.
to the Preamble. Menon as the Secretary to deal with the princely
7. The Preamble is non-justiciable in nature. states.
But it has been declared as a part of the The princely states were appealed to join
Indian Constitution by the Supreme Court the Indian Union by signing the Instrument of
in Keshavanandan Bharti case. Accession. Lord Mountbatten also advised the
8. It can neither provide substantive power princely states to accede to either Indian Union
(definite and real power) to the three or- or Pakistan before 15 August 1947. As a result
gans of the State, nor limit their powers of the appeal, all the princely states expect Juna-
under the provisions of the Constitution. gadh, Hyderabad, and Jammu and Kashmir ac-
9. The Preamble cannot override the specific ceded to the Indian Union by 15 August 1947.
provisional of the Constitution. 1. Accession of Junagadh, 29th January, 1949
10. In case of any conflict between the two,
Junagadh was a small princely state of kathi-
the later shall prevail.
awad district of Gujarat. Its ruler was Muslim
and its population were predominantly Hindus.
TERRITORY OF THE UNION Its neighboring states viz., Baroda and Bhavna-
(PART–I) gar were also Hindu states. It had not direct ter-
ritorial link with Pakistan. Inspired by religious
Part-I of the Constitution comprises four articles sympathy, the Muslim ruler of Junagadh ac-
concerned with the territory of India. Article I corded to Pakistan. But the people rose in revolt
stipulates that, India, that is Bharat, shall be a and protested against the action of the ruler. The
‘Union of States’. It is to be noted that expres- situation became grave and the ruler of Juna-
sion. ‘Union of India’, is not synonymous with gadh fled to Pakistan.
‘the territory of India’; The union includes only The Government of India was forced to
the states which are members of the federal sys- take over the administration of Junagadh by
tem and share a distribution of powers with the sending its troops. In 1948, a referendum was
union while the ‘territory of India’ includes the held and the people of Junagadh favoured ac-
entire area over which the sovereignty of India cession to India. The merger finally took place
extends.
on 29 January 1949.
Integration of Princely States 2. Hyderabad, November 23, 1949
On the eve of Independence and Partition of Hyderabad was a princely state. Like Junagadh
India in 1947, there existed two India viz., (1) its ruler was a Muslim (Nizam) and its popula-
British India, (2) Native India. The British India tion were predominantly Hindus. The Hindus
was ruled and controlled by the British. The Na- constituted 85 per cent of the population. The
tive India consisted of 565 Native states ruled Government of India appealed Nizam, the ruler
over by native princes. These native states were of Hyderabad to accede to Indian Union. Being
under the paramountcy of the British. influenced by the Muslim organization “Itteh-
The cabinet Mission Plan 1946 and Mount- adul Mussalman” and its leader Kasim Razvi,
batten plan of 3 June 1947 recognised the legal the Nizam did not respond. In the meanwhile,
right of the princely states on the British para- Lord Mountbatten promised some special con-
mountcy. Hence, the Indian Independence Act cessions to the Nizam, if he acceded to the In-
of 1947 not only abolished the British para- dian Union. This was also rejected by Nizam.
mountcy over the native states but also divided However, he signed a temporary agreement with
India into two independent nations viz., Indian the Government of India known as standstill
Union and Pakistan. Thus the Princely states Agreement on 20 November 1947. The Raza-
were given full freedom either to join Indian kars, under the leadership of Razvi committed
Union or Pakistan or to remain independent. atrocities on Hindus. The lawlessness in the
This status of princely states posed a threat to state became so grave that the government of
the very existence of Indian Union. India had no other alternative except to take Po-

CLAT.indb 125 31/03/2009 11:41:49


126 CONSTITUTIONAL LAW

lice action. The Indian troops headed by General Orissa, Sind, Assam etc., were constituted more
Chaudhury entered Hyderabad on 13 September or less on the linguistic basis during the Brit-
1948. Nizam surrendered to he Government of ish period. However, Andhras were agitating for
India. At last, the acceded to the Indian Union a separate state for the Telugu speaking people
on 23 November 1949. since a long time.
3. Kashmir Dhar Commission After independence the
Kashmir was also a big princely state of India. It Indian National Congress feared that the accep-
was ruled over by Hari Singh, a Hindu. But most tance of linguistic principle would endanger the
of its population was Muslim. It was a neigh- unity of India. But on the recommendation of
bouring state to Pakistan. In spite of appeals the Drafting Committee of the Constituent As-
from both the countries, Hari Singh did not join sembly, a commission known as Dhar Commis-
either India or Pakistan but remained indepen- sion was appointed to study and report the desir-
dent. In connivance with Pakistan, the Muslim ability of the reorganization of states on linguis-
tribesmen of the frontier invaded Kashmir on 22 tic basis. In 1948 the Dhar Commission submit-
October 1947. Soon, they occupied some towns ted its report to the Constituent Assembly. The
and threatened Srinagar, the capital of Kashmir. Dhar Commission rejected the reorganization
Then the ruler of Kashmir sought the help of the of states purely on linguistic basis. However, it
Government of India. The Government of India proposed that geographical entity, financial self-
informed him that the Indian troops could not sufficiency, administrative convenience etc.,
enter the state, unless he signed the ‘Instrument should also be taken into consideration before
of Accession’. Hari Singh signed the Instru- accepting the linguistic principle. Therefore, the
ment of Accession’ on 26 October 1947. On 27 Congress was not enthusiastic about the reorga-
October 1947, the Indian troops entered Kash- nization of states on linguistic principle.
mir, checked the Pakistan aggression and drove
away the infiltrators. Thus, Kashmir became an J. V. P. Committee, 1948 Owing to public
integral part of the Indian Union. But Pakistan pressure, the Congress appointed a Committee
disputed the accession of Kashmir to Indian in 1948 consisting of Jawaharlal Nehru, Vallab-
Union. Instead of expelling the aggressors from hbhai Patel and Pattabhi Sitaramayya to consid-
the occupied for a ceasefire on 1 January 1949. er the question. This committee was called the
As a result, one third of Kashmir was under the J. V. P. Committee also felt that while reorga-
occupation of Pakistan and the rest constituted nizing the states, all the relevant factors such as
an integral part of the Indian Union. Now it be- the unity of India, national security and defence,
came an issue, called Kashmir issue between the cultural and linguistic affinities, economic and
two countries. administrative should be borne in mind. As a
result the Government of India headed by Jawa-
Reorganization of States harlal Nehru was not in favour of the creation of
One of the problems that confronted Indepen- linguistic states.
dent India was the reorganization of states. Formation of Andhra State, 1 October 1953
The idea that the provinces should be The J. V. P. report disappointed many people
formed on the basis of the language of the peo-
especially the Andhras who were agitating for
ple has its genesis during the British rule. Not
a separate linguistic state since a long time. Sri
only the British Government but also the Indian
Potti Sree Ramulu undertook a fast unto death
national Congress favoured the redistribution of
for the formation of Andhra State. His martyr-
provinces on linguistic and ethnic consideration.
dom on 15 December 1952 created a great com-
The Indian National Congress at its Nagpur ses-
motion in Andhra. Then the government of India
sion in 1920 adopted a resolution to that effect.
was left with no choice but to accept the demand
The Nehru Committee appointed by the Indian
of Andhra. Thus by the Act of Parliament, the
National Congress under the chairmanship as
state of Andhra came into existence on 1 Octo-
Motilal Nehru in 1928 also expressed the view
that the British provinces should be regrouped ber 1953. The Andhra Pradesh was the first lin-
on the linguistic basis. Similar resolutions were guistic state of Independent India.
passed by the Indian National Congress in its Fazal Ali Commission (December 1953) The
session Calcutta in 1937. Wardha in 1938 and in formation of Andhra State gave an impetus to
its Election Manifesto of 1945-46. As a result, the people of other regions also to demand states

CLAT.indb 126 31/03/2009 11:41:50


CONSTITUTIONAL LAW 127

based on their languages. Then realizing the ur- the national integration. However, the functions
gency of the problem, the government of India of the zonal councils were purely advisory.
appointed a commission under the Chairman- Aftermath of the Reorganization Act The
ship of Syed Fazal Ali (with Pandit Kunzru and states reorganization did not end by the states
K. Panikkar as its members) in 1953 to examine Reorganization Act of 1956. The arrangements
the whole question of recorganization of states made by the act did not satisfy some linguistic
objectively and dispassionately. This is called groups and hence the agitation for the creation of
the Fazal Commission. The Fazal Commission more states was on now. As a result, the govern-
reported the distinctions of old A, B, C, D states ment of India almost yielded to such demands.
should be abolished and the states should be re- Fifteenth State The agitations of Samyukta
organized rationally on linguistic basis. It also Maharashtra Samithi and Maha Gujarat Samithi
recommended that Indian union should consist led to the division of Bombay state into two
of sixteen states and three centrally adminis- states viz., Maharashtra and Gujarat. Bombay
tered areas. was given to Maharashtra and Gujarat set up a
States Reorganization Act of 1956 The act new capital called Gandhi Nagar. Thus Gujarat
made some modifications to the report of Fazal became the Fifteenth state of the Indian Union.
Ali Commission, The Government of India Sixteenth State In 1962, the state of Nagaland
passed the States Reorganisation Act in 1956. was granted for the Naga people. The capital of
According to this Act the A, B, C, D states were Nagaland is Kohima. It is the sixteen state.
abolished and Indian Union was divided into Seventeenth State The agitation in Punjab led
fourteen states and six centrally administered to its division into three units viz., the creation
areas. They were: of Haryana state (seventeenth state) and Chan-
Fourteen States digarh as Union Territory and Punjab in 1966.
State Capital Chandigarh was the capital for both the states.
1. Andhra Pradesh Hyderabad Eighteenth State On 25 January 1971 the
2. Assam Dispur Union Territory of Himachal Pradesh was con-
3. Bihar Patna verted into a state. Thus Himachal Pradesh be-
4. Bombay Bombay came the eighteenth state of India. Its capital is
5. Punjab Chandigarh Shimla.
6. West Bengal Calcutta Nineteenth, Twentieth and Twenty-first States
7. Jammu and Kashmir In winter In view of the strategic importance of North
Jammu and in East Frontier agency, the state of Meghalaya
summer Srinagar was carved out of Assam and the Union terri-
8. Orissa Bhuvaneswar tories of Manipur and Tripura were elevated to
9. Tamil Nadu Madras statehood in 1972. Thus three more states, viz.
10. Madhya Pradesh Bhopal 19th , 20th , 21st , viz., Meghalaya, Manipur and
11. Uttar Pradesh Lucknow Tripura came into existence on January 1972.
12. Kerala Trivandrum Twenty-second State In 1975, according to
13. Karnataka Bangalore the people of Sikkim, by the 36th Constitutional
14. Rajasthan Jaipur Amendment Act, Sikkim became the Twenty-
second of the Indian Union.
Six Centrally Administered Areas on Union
Territories Twenty-third and Twenty-fourth States On
1. Delhi Delhi 20 February 1987, Arunachal Pradesh and Mi-
2. Himachal Pradesh Shimla zoram (Union Territories) were given statehood.
3. Manipur Imphal (23rd and 24th states).
4. Tripura Agartala Twenty-fifth State On 30 May 1987, the Union
5. Andaman & Nicobar Port Blair Territory of Goa was elevated to statehood.
Islands Twenty-sixth, Twenty-seventh and Twenty-
6. Lakshadweep and Kavaratty eighth States By the bifurcation of UP, MP
Minicoy Islands and Bihar, three new states of Uttranchal,
The Act also regrouped the states in four Chhatisgarh and Jharkhand came into exis-
zones, each with a zonal council in the interest of tence.

CLAT.indb 127 31/03/2009 11:41:50


128 CONSTITUTIONAL LAW

CITIZENSHIP to these 12 months, he has resided or has


been in the Government service for not
Part II of the Constitution simply describes less than four years.
classes of persons living in India at the com- (e) He must take an oath of allegiance.
mencement of the Constitution, i.e., 26 January (f) He is of a good character.
1950, and leaves the entire law of the citizenship (g) He has an adequate knowledge of a lan-
to be regulated by legislations made by the Par- guage recognized by the Constitution.
liament (Article 11). By Incorporation of Territories: If any
In the exercise of its power the Parliament new territory becomes a part of India, after a
enacted the Indian Citizenship Act, 1955, which popular verdict, the Government of India shall
was subsequently amended in 1986. specify the person of that territory to be the citi-
zen of India.
Five Modes to Acquire Citizenship of In India, there is single citizenship i.e., citi-
India are: zenship of India.
By Birth: born in India, either parent must Article 9 says that ‘citizenship means full
be an Indian citizen i.e., Jus Soli (Law of Soil) citizenship’. The Constitution does not recogn-
By Descent: born outside India, father/ ise divided allegiance.
mother must be a citizen of India, i.e., Jus San- A citizen is an individual who enjoys all the
guine (Law of Blood) rights given by the law, available in the country.
By Registration: The prescribed authority Article 11 says that the Parliament will reg-
may, on application, register as a citizen of In- ulate the right of citizenship by the Law.
dia, any person who is not a citizen by virtue of
Three Models of Losing Citizenship
Constitution or the provisions of the Citizenship
Act. This mode of acquiring citizenship is avail- Renunciations: voluntarily given up the
able to any of the following categories: citizenship
(a) persons of Indian origin who are ordinar- Termination: by operation of law
ily resident in India for 5 years immedi- Deprivation: compulsory termination of
ately before making an application for the citizenship obtained by Registration or Natu-
registration. ralization on charges of using fraudulent means
(b) persons of Indian origin who are ordinar- to acquire the citizenship.
ily resident in any country or place outside Can a person not born in India, but having ac-
India. quired citizenship by registration or otherwise,
(c) women who are, or have been married to become the Prime Minister of India?
citizens of India. The likelihood of the President of Indian
(d) minor children of persons who are citizens National Congress (Indira), Mrs Sonia Gandhi,
of India. becoming the Prime Minister of India after be-
(e) persons of full age and capacity who are ing elected to the 13th Lok Sabha generated a
citizens of a country mentioned in the controversy whether a person who becomes a
First Schedule. citizen of India either by registration or natu-
By Naturalization: Citizenship by natural- ralisation should occupy, what effectively is, the
ization can be acquired by making an applica- highest position in the country.
tion in the prescribed manner. The qualifications
for naturalization are the following: The Constitutional Position is as Follows:
(a) He must be a person of full age and capac- Article 14 guarantees equality before law. Arti-
ity. cles 15 and 16 prohibit discrimination and com-
(b) He must not be a citizen of a country mit the government to equality of opportunity.
where Indian citizens are prevented from Article 16 guarantees equality of opportu-
becoming citizen by naturalization. nity in matters of Government employment and
(c) He has renounced the citizenship of the appointment.
other country. The Constitution does not differentiate be-
(d) He has either resided in India or has been tween an ordinary and a naturalized citizen as
in Government service for 12 months be- far as eligibility to contest for the Lok Sabha
fore the date of making the application seat is concerned and thereafter, any member is
for naturalisation, or during 7 years prior Constitutionally entitled to become the Prime

CLAT.indb 128 31/03/2009 11:41:50


CONSTITUTIONAL LAW 129

Minister provided the requisite support in Lok for e.g., during the National Emergency under
Sabha is available. Article 352 (i.e., war or external aggression).
Therefore, the India Constitution does The Constitution empowers the President,
not bar a naturalised citizen from becoming a under Article 359, to suspend any or all the Fun-
Prime Minister. However, in the US, the Con- damental Rights by issuing a separate Procla-
stitution permits naturalized citizens to become mation during a National Emergency. The 44th
members of the Senate only (Federal House) Amendment Act, 1978 prohibits the suspension
but not the President. of Article 20 and 21 (protection in respect of
conviction for offences and protection of Life
FUNDAMENTAL RIGHTS and Personal Liberty, respectively) even during
a National Emergency.
The fundamental rights are the rights given (to) The Definition of State
the individuals (by) the Constitution (for) their
According to Article 12, ‘the State’ includes the
development (against) the State action. The
Government and Parliament of India and the
state is defined in Article 12. The state action
Government and the Legislature of each of the
means any action—legislative and executive, by
States and all local or other authorities within
any organ instrumentality or agency of the state
the territory of India or under the control of the
as outlined in the Ajay Hassia case by Honour-
Government of India.
able Supreme Court.
The judiciary, it is said, though not express-
The rights are called fundamental right be-
ly mentioned in Article 12 should be included
cause they are fundamental, essential and intrin- within the expression ‘other authorities’ since
sic for the lending of the individual. They are courts are set up by statutes, and exercise pow-
also called fundamental because they are given ers conferred by the law.
by the fundamental law of the land having Con- In Ajay Hassia case, the Supreme Court
stitutional sanction as compared to the other or- summarized the criterion for judging whether a
dinary laws. body is a state. The following have been held to
Part III of the Constitution contains a long be State:
list of the Fundamental Rights. Part III of the 1. Regional Engineering College established
Constitution is called the ‘Cornerstone’ of the by a Society registered under a State Act
Constitution, and together with Part IV (Direc- 2. Indian Statistical Institute
tive Principles), constitutes the ‘conscience’ of 3. Indian Council of Agricultural Research
the Constitution. 4. Food Corporation of India
This chapter of the Constitution has been 5. Steel Authority of India Limited
described as the ‘Magna Carta’ of India. They 6. Nationalized Banks
are the limitations upon all the powers of the 7. Rajasthan Electricity Board
Government, legislative as well as executive. 8. International Airport Authority
The object behind the inclusion of the 9. Oil and Natural Gas Commission
chapter on the Fundamental Rights in the Indian 10. Hyderabad Stock Exchange
Constitution is to establish ‘a Government of 11. Council for Indian School Certificate Ex-
law and not of man’. aminations
Fundamental Rights are the restricted 12. United India Insurance Company
rights, they do not give absolute power to the 13. Council of Scientific and Industrial Re-
individual. Article 17 and Article 24 are the ab- search
solute rights. It is settled law that a body which is an agency
Article 13 provides for the Judicial Review or instrumentality of the government is includ-
of all the legislations in India. Judicial Review is ed in State.
the power conferred on the High Courts and the
Supreme Court of India to declare a law uncon- Amendibility of Fundamental Rights
stitutional, if it is inconsistent with any of the The Supreme Court in the number of cases, from
provisions of Part III of the Constitution, to the Shankari Prasad vs Union of India (1952) to
extent of the contravention. Sajjan Singh vs State of Rajasthan (1965), held
The Constitution of India contains provi- that by exercising its amending power under Ar-
sions for automatic suspension of the Funda- ticle 368, the Parliament can amend even Part
mental Rights under certain circumstances, as III of the Constitution.

CLAT.indb 129 31/03/2009 11:41:50


130 CONSTITUTIONAL LAW

In Golaknath vs State of Punjab (1967) Fundamental Rights Available to any


case, the Supreme Court overruled its earlier Person on the Soil of India (except to
decision and held that the Fundamental Rights the Enemy Aliens)
embodied in Part III had been given a ‘Tran- 1. Article 14: Equality before law and equal
scendental position’ by the Constitution and no protection of law.
authority, including the Parliament through its 2. Article 20: Protection in respect of convic-
amending power under Article 368, was compe- tion for offences.
tent to amend the Fundamental Rights. 3. Article 21: Protection of Life and Personal
By the 24th Amendment Act, 1971, the Liberty.
Parliament suitably amended Article 13 and Ar- 4. Article 23: Prohibition of traffic in human
ticle 368 to empower itself to amend Part III of beings and forced labour.
the Constitution. 5. Article 25: Freedom of Religion.
This Amendment Act, was challenged be- 6. Article 27: Freedom as to payment of taxes
fore the Supreme Court in a landmark case, Ke- for promotion of any particular religion.
shavananda Bharati vs State of Kerala (1973), Fundamental Rights Exclusive to the
the Court held that the Parliament can amend Citizens
any of the provisions of the Constitution includ- 1. Article 15: Prohibition of discrimination
ing Fundamental rights by its amending power only on grounds of religion, race, caste,
under Article 368, provided such Amendment sex or place of birth.
Act(s) do not touch the “basic structure” of the 2. Article 16: Equality of opportunity in mat-
Constitution. ters of Public employment.
The basic structure of the Constitution in- 3. Article 19: Protection of certain rights, re-
cludes: garding freedom of speech etc.
1. Supremacy of the Constitution 4. Article 30: Right to the minorities to es-
2. Separation of powers tablish and administer the educational in-
stitutions.
3. Rule of law
4. Judicial review Right to Equality (Article 14–18)
5. Democratic Republic Article 14
6. Democratic Socialism The State shall not deny to any person, equality
7. Secular State before the law or equal protection of the laws
8. Federalism within the territory of India. This concept is
9. Synthesis of Parliamentary Sovereignty negative and has been borrowed from the Brit-
and Judicial Supremacy ish Constitution. It means ‘no man is above law’
10. Free and fair elections and every person, whatever be his/her social sta-
11. Independence of Judiciary tus, is subject to the jurisdiction of the Courts.
12. Limited power of the Parliament to amend Equal protection of law is a positive concept,
the Constitution borrowed from the US Constitution. It only
13. Harmony and balance between Funda- means that all persons in similar conditions/
mental Rights and Directive Principles circumstances shall be treated alike. There can
be a discrimination between the groups but not
Classification of Fundamental Rights within the groups.
There are six groups of Fundamental Rights: Rule of Law
1. Right to Equality (Article 14–18) The guarantee of Equality before law is an as-
2. Right to Freedom (Article 19–22) pect of what Lord Dicey calls, the ‘Rule of Law’
3. Right against Exploitation (Article 23 and that originated in England. It means—no man is
24) above the law and that every person, whatever
4. Right to Freedom of Religion (Article be his rank or status is subject to the jurisdiction
25–28) of ordinary Courts. Also, it says that no person
5. Cultural and Educational Rights (Article shall be subject to harsh, uncivilized or discrimi-
natory treatment even for the sake of maintain-
29 and 30)
ing law and order.
6. Right to Constitutional Remedies (Article
There are three basic meanings of ‘Rule of
32)
Law’:

CLAT.indb 130 31/03/2009 11:41:50


CONSTITUTIONAL LAW 131

1. Absence of Arbitrary power or supremacy (e) to reside and settle in any part of the terri-
of law—“a man can be punished for a tory of India
breach of law but he cannot be punished (f) to property (removed by the 44th Consti-
for anything else”. tutional Amendment Act, 1978 and trans-
2. Equality before law—no one is above the ferred to Article 300 A)
law. (g) to practice any profession, or to carry on
3. The Constitution is the Supreme law of the any occupation, trade or business.
land and all laws passed by the legislature The State can impose reasonable restric-
must be consistent with the provisions of tions. The right to form association, unions etc.
the Constitution. does not give right to strike. The Indian Consti-
Article 15 tution does not provide for the freedom of Press
Article 15 directs the State not to discriminate separately. It is implicit in Article 19, which
against a citizen on the grounds only of race, grants freedom of speech and expression.
caste, religion, sex or place of birth etc. Article 20
The word ‘only’ indicates that the discrimi- Protection in respect of conviction for offenc-
nation cannot be made merely on the ground es: This protection is available against the fol-
that one belongs to a particular caste, religion, lowing three types of convictions:
race etc. (a) Ex-post facto legislation: This means en-
Article 16 acting a law and giving it a retrospective
Equality of opportunity in matters of public em- (i.e., from a previous date/year) effect/
ployment. this is applicable only for civil legisla-
No citizen shall, on grounds only of re- tions while criminal legislations cannot be
ligion, race, caste, sex, descent, place of birth given retrospective effect.
or residence be ineligible for, or discriminated (b) Double jeopardy: This means that an in-
against in respect of any employment or office dividual can be punished for a crime only
under the State. once and also not beyond the period pre-
scribed by the authority.
Article 17
(c) Prohibition against self incrimination:
Abolition of Untouchability: ‘Untouchability’ is
No person, accused of an offence, shall
abolished and its practice in any form is forbid-
be compelled to be a witness against him-
den. The Constitution itself does not prescribe
self.
any punishment under this Article. The Parlia-
ment enacted the ‘Untouchability (offences) Article 21
Act, 1955’ which prescribes the punishment for Protection of Life and Personal Liberty: No
the practice of untouchability. person shall be deprived of his/her life and per-
Article 18 sonal liberty except according to the procedure
Abolition of Titles: ‘No title, no being a Military established by law.
or Academic distinction, shall be conferred by The Supreme Court, through a liberal inter-
the State’. No citizen of India shall accept any pretation of the Article, has derived a number of
title from any foreign State. Article 18 does not inferred rights.
prescribe any punishment for the offence. The Article stands not merely for the right
The conferment of titles of ‘Bharat Ratna’, to life and personal liberty, but also for the right
‘Padma Vibhushan’, ‘Padma Bhushan’, ‘Padma to dignity and all other attributes of human per-
Shri’ etc., are not violative of Article 18. sonality that are essential for the full develop-
ment of a person.
Right to Freedom (Article 19–22) Article 21 has become the ‘Foundation
Article 19 stone of Part III of the Constitution’.
Article 19 of the Constitution guarantees to the Meaning of Life and Personal Liberty: Life
citizens of India the following six fundamental is not merely an animal existence or survival.
rights/freedoms: It would include the right to live with human
(a) freedom of speech and expression dignity and all those aspects of life which go
(b) to assemble peacefully and without arms to make a man’s life meaningful, complete and
(c) to form associations or unions worth living.
(d) to move freely throughout the territory of The expression, personal liberty, covers
India a wide variety of rights which go to constitute

CLAT.indb 131 31/03/2009 11:41:50


132 CONSTITUTIONAL LAW

the personal liberties of a man other than those 6. Right to die or commit suicide is not in-
which are already included in Article 19. The cluded in Article 21.
courts are inclined to give the widest amplitude 7. It is the duty of the State to permit estab-
to the expression. On account of the liberal in- lishment of private law colleges and give
terpretation, Article 21 has now come to be in- them grants-in-aid.
voked almost as a residuary right, even to the 8. Handcuffing should be resorted to only
extent which the founding fathers never dreamt when there is a clear and present danger of
of. escape.
From the judgements of the Supreme Court, 9. Long delay in execution of death sentence
the following are some of the rights that are to may evoke the protection of Article 21.
be read in Article 21: Death sentence would be commuted to life
1. Right not to be subjected to bonded la- imprisonment. No fixed period has been
bour declared. The court will decide from case
2. Right to livelihood by means which are to case.
not illegal, immoral or opposed to public 10. Article 21 will provide protection of life
policy and liberty to persons who are not citi-
3. Right to decent environment zens.
4. Right to shelter 11. In suitable cases, the court may grant com-
pensation to the victim who has been de-
5. Right to travel abroad
prived of his right. For example, victims
6. Right to speedy trial
of torture, custodial death and fake en-
7. Right to legal aid
counters.
8. Right to privacy
12. Capital punishment is not violative of Ar-
9. Right against solitary confinement
ticles 14, 19 and 21.
10. Right against bar fetters
13. Hanging as a mode of execution is fair and
11. Right against handcuffing
just and does not offend against Article
12. Right against delayed execution 21.
13. Right against custodial violence 14. Sexual harassment of working women in
14. Right to education (not professional or the workplace is denial of Article 21.
special)
15. Right to pure drinking water Article 21-A
Right to Education: By the Constitution Act,
16. Right to good roads
2002 (86th Amendment Act,) a new fundamental
Instances of protection under Article 21 right has been provided by inserting Article
Spreading the protection of Article 21 the Su- 21-A. It casts a duty on the State to provide
preme Court has issued directions in numerous free and compulsory education to all children
cases favouring the individual. between the age of 6 to 14 years. To implement
Some examples are: this right the State will enact appropriate laws.
1. Telephone tapping is an invasion of an Education being a concurrent subject laws may
individuals right. It can be resorted in the be enacted either by the Union or the States.
interest of public safety and that too with By the same Amendment Act, (86th) Ar-
procedural safeguards. ticle 45, a Directive Principle, relating free and
2. All medical practitioners, whether govern- compulsory education for children has been re-
ment or private are under an obligation to placed. Now this new Article 45 directs the State
extend medical aid to the injured immedi- to provide childhood care and education to chil-
ately. dren below the age of 6 years.
3. Denial of medical aid by a government The 86th Amendment Act, has added one
hospital to an injured person violates Ar- more Fundamental Duty to Article 51 A. Every
ticle 21. parent or guardian has the duty to provide edu-
4. The state as well as the industry (public cational opportunities to his child or ward be-
and private sector) must take such action tween the age of 6 to 14 years.
as will promote the health of the work- Article 22
men. Protection against arrest and detention in cer-
5. An honest judgement debtor who is not tain cases: The authority cannot arrest or detain
able to pay his debts cannot be sent to a person without properly informing him/her of
jail. the grounds for such arrests/decision.

CLAT.indb 132 31/03/2009 11:41:50


CONSTITUTIONAL LAW 133

The detained/arrested person must be pro- Our Constitution considers that preventive
duced before the nearest Magistrate within 24 detention is a necessity, but it must be subject to
hours of arrest (excluding the holidays and time limitations to prevent misuse.
taken during the journey). Examples of law providing from preven-
The period of the detention cannot be ex- tive detention are:
tended beyond what is authorized by the Mag- 1. Preventive Detention Act, 1950 (Re-
istrate. pealed)
Protection against arrest: Article 21 guarantees 2. Maintenance of Internal Security Act,
right to life and personal liberty. A law may be 1971 (MISA) (Repealed)
enacted by a legislature depriving a person of 3. Conservation of Foreign Exchange and
his right. Article 22 sets out certain limitations Prevention of Smuggling Activities Act,
upon the powers of the legislature. If a law con- 1974 (COFEPOSA)
travenes the conditions or limitations prescribed 4. National Security Act, 1980.
by Article 22 the law would be a nullity. Article 5. Prevention of Terrorism Act, 2002.
22 is not a complete code. It has to be construed All such laws conform to and contain the
with Article 21. safeguards provided in Article 22.
Article 22 has two parts. The first part con- The safeguards are:
sists of Clauses (1) and (2). These clauses apply 1. If the detention is for more than 3 months
the matter must be referred to an advisory
to all arrests made under any law except under a
board. The detention may be continued
law of preventive detention.
only where the advisory board considers
The rights flowing from this Article in case
that there are sufficient grounds for the de-
of ordinary arrests are:
tention.
1. Right to be informed of the ground of ar-
2. Grounds of detention must be communi-
rest
cated to the detenue.
2. Right to consult and be defended by a law-
3. The detenue must be given an opportunity
yer to make a representation against the order
3. Right to be produced before a magistrate of detention.
within 24 hours of his arrest (excluding The 44th Amendment Act purports to re-
the time of journey) duce the period of detention without reference to
4. Right to be detained for more than 24 the Advisory Board from 3 months to 2 months.
hours without the authority of a magis- But this provision of the 44th Amendment Act,
trate. has not yet been brought into force.
Protection of Article 22 when not available:
Right Against Exploitation (Article 23
The above rights are not available to
and 24)
(a) an enemy alien and
(b) a person detained under a law of preven- Article 23
tive detention Prohibition of traffic in human beings and
forced labour: Traffic in human beings and beg-
Preventive detention: The second part of Ar- gar and other similar forms of forced labour are
ticle 22 comprising of Clauses (4) and (5) pro- prohibited. Any contravention of this provision
vide safeguards to detenues under a law for shall be an offence punishable in accordance
preventive detention. It is notworthy that these with law.
safeguards are available even to enemy aliens. One shall not be forced to provide labour
Preventive detention is the detention of a person or services against his will even if remuneration
where the evidence against him is not sufficient is paid.
for his conviction by a court but is sufficient to If remuneration is less than minimum wag-
justify his detention for reasons for defence, es, it amounts to forced labour under Article 23.
foreign affairs, security of state, maintenance of
Article 24
public order etc. Punitive detention punishes a
Prohibition of employment of children in fac-
person for something done by him. Preventive
tories, etc.: The essence of Article 24 is the pro-
detention prevents a person from doing some- hibition of employment of children below 14
thing. Preventive detention is a precaution taken years of age in hazardous jobs. This provision is
by the executive. Its justification is reasonable in the interest of public health and safety of the
suspicion or probability. lives of children.

CLAT.indb 133 31/03/2009 11:41:50


134 CONSTITUTIONAL LAW

Right to Freedom of Religion (Article 2. The Muslims have no right to stop the pro-
25–28) cession of a community on the ground that
Article 25 the music being played operates as a nui-
Freedom of conscience, profession, practice sance or offends against their sentiments.
and propagation of religion: Subject to public 3. Abolition of hereditary rights of archakas
order, morality and health and to the other provi- (priests) is not interference with essential
sions of this part, all persons are equally entitled religious practice.
to the freedom of conscience and the right to 4. Sacrifice of cows on the occassion of
profess, practice and propagate does not mean Bakrid was not an essential part of Mus-
alluring a person to join any religion. lim religion and the State may prohibit
cow slaughter.
Article 26
5. The Act taking over the management of
Freedom to manage religious affairs: Subject
to public order, morality and health, every reli- Aurobindo Society was held valid because
gious denomination or any section thereof shall the Society and Auroville were not reli-
have the right: gious institutions. The teachings of Shri
1. To establish and maintain institutions for Aurobindo do not constitute a religion.
religious and charitable purposes. 6. Polygamy is not an essential part of the
2. To manage its own affairs in the matters of Hindu religion so it can be prohibited by
religion. law. This is for the reform of the Hindus.
3. To own and acquire movable and immov- 7. The Aligarh Muslim University was estab-
able property and to administer such prop- lished by the Parliament and the Muslims
erty in accordance with law. cannot claim a right to run that university.
8. The Jagannath Temple Act by which the
Article 27
government vested the secular activities
Freedom as to payment of taxes for promotion
in a committee is valid because it did not
of any particular religion: No person shall be
interfere with religious activities.
compelled to pay any tax for religious purpos-
9. There is no right that a brahmin alone can
es.
perform the rites and rituals in a temple.
Article 28
Articles 28 divides educational institutions into Cultural and Educational rights (Article
four categories: 29–30)
1. Wholly maintained by the State Article 29
2. Recognized by the State Protection of interests of minorities: Any sec-
3. Receive aids out of the State funds tion of the citizens, residing in the territory of
4. Administered by the State, but established
India or any part there of, having a distinct lan-
under a religious endowment.
guage, script or culture of its own, shall have the
In the first case, there can be no religious
right to conserve the same.
instructions whatsoever. In the second and third
case, religious instructions can be imparted, but Article 30
the pupils cannot be compelled to attend such Right of minorities to establish and administer
instructions. Whereas in the fourth case, there educational institutions: All minorities, wheth-
is no restriction whatsoever, as far as religious er based on religion or language, shall have the
instructions are concerned. right to establish and administer educational in-
Persmissible Restrictions stitutions of their choice. It provides to religious,
Articles 25 and 26 are not absolute. The com- educational and cultural institutions, the right to
mencing words of both the articles are subject to own possess and dispose immovable property.
public order, morality and health. On that basis, Administer means the management of affairs of
the following are some of the restrictions which the institution. The State can regulate the work-
have been held to be permissible: ing of such institutions.
1. The State can impose restrictions on the
Right to Constitutional Remedies
use of loudspeakers for Ramzan or for
singing kirtans etc. The prohibition is for Article 32
safeguarding the health of the people. Use Dr B. R. Ambedkar called this Article as “The
of loudspeakers is not an essential and in- fundamental of Fundamental Right” and “the
tegral part of a religion. heart and soul of the Constitution”. To enforce

CLAT.indb 134 31/03/2009 11:41:50


CONSTITUTIONAL LAW 135

the Fundamental Rights, the Supreme Court is Mandamus


empowered, under Article 32, to issue writs of Mandamus literally means a command or an
various forms. order. It is a command directing a person, cor-
Five Forms of Writs Under Article 32 poration, inferior court or government or any
public authority to do the thing or perform the
The Writs—Clause (2) of Article 32 empowers
act specified in the writ. The thing or act must
the Supreme Court to issue directions, orders or
be in the nature of a public duty. This writ is a
writs which may be appropriate. The writs enu-
wakening call. It tells the sleeping authority to
merated are Habeas Corpus, mandamus, prohi-
bition, quo warranto and certiorari. wake up and perform its public duty. It is also
Parliament has the power to make law used to order the authority which refuses to per-
empowering any court to issue these writs. No form a public duty. It is a writ which demands
such law has been made. So the Supreme Court activity. It sets the authority in motion.
(Under Article 32) and the High Courts (under Before Mandamus is issued the following
Article 226) are alone authorised to issue such conditions must be satisfied.
writs. 1. The petitioner must show that he has a le-
Habeaus Corpus—Habeas Corpus liter- gal right and that the person against whom
ally means to have body. By this writ, the court the mandamus is sought has a legal duty.
can make any person, who has been detained or 2. The duty must be imposed by law (Consti-
imprisoned, to be physically brought before the tution, Act, subordinate legislation etc.)
court. The court then examines the reasons of 3. The petitioner moved the authority for
this detention. If there is no legal justification, performing the legal duty but the authority
he is set free. The Supreme Court has laid down has either not acted or refused to act. This
that production of the body of the prisoner be- is in short called demand and refusal.
fore the court is not an essential feature of the 4. The public authority is acting under a law
writ. which is unconstitutional.
The writ shall be issued where: Mandamus will not lie
(a) the detention is in violation of the pre- (i) where there is a discretion to act or not to
scribed rules of procedure. Example, the act as distinguished from a duty the au-
detinue is not produced before a magis- thority cannot be ordered to exercise the
trate within the prescribed period. discretion.
(b) the order of arrest is in violation of any (ii) against a private individual or organisa-
law. For example, an arbitrary order. tion unless they are entrusted with a public
(c) a person has been detained by a private duty.
individual. (iii) to enforce a contract against the govern-
(d) a person is detained under a law which is ment, but may in appropriate cases issue
unconstitutional. as promissory estoppel to enforce a prom-
(e) the order of detention is malafide. ise made by the government relying on
The general rule is that the person who pe- which the petitioner has altered his posi-
titions for a writ must be the person whose right tion to his prejudice.
has been infringed. This does not apply to Ha- (iv) against a company even though it may be
beas Corpus. The person detained in a prison is a government company.
severely handicapped. The law understands this Mandamus will not issue against
and allows a petition to be moved on behalf of (a) The President or the Governor of a State
the prisoner by his friend or a social worker or for the exercise or performance of the du-
even a stranger. ties of his office.
Habeas Corpus would not lie where the (b) The High Court or its chief justice or a
return to the writ discloses that a person is in judge acting in judicial capacity but may
prison in execution of a sentence by a competent lie in its administrative capacity to ques-
court. The writ is not issued when a person is un- tion appointments, or disciplinary orders
der physical restraint under a valid law. Where etc.
it is shown that the petitioner was arrested and
Prohibition
imprisoned without the authority of law or with
mailicous intent the court may award suitable The writ of prohibition is issued by a superior
monetary compensation or exemplary costs. court to an inferior court or tribunal to prevent

CLAT.indb 135 31/03/2009 11:41:50


136 CONSTITUTIONAL LAW

it from usurping jurisdiction which is not vested out of a statute. A commission of inquiry
in it. It forbids and compels the inferior court or does not determine the rights of a person
tribunal to travel outside its legal limits or juris- so no certiorari will lie against it.
diction and also prevents it from violating the (c) having the duty to act judicially It means
rules of natural justice. that principles of natural justice should be
The court readily grants prohibition where observed. The deciding authority must act
(a) the defect of jurisdiction is apparent on the in a fair and reasonable manner and must
face of the record that is, it is patent and not have any bias or prejudice.
does not need much argument to make it (d) acts in excess of its authority The judicial
clear. or quasi-judicial authority acts without or
(b) a fundamental right is infringed. in excess of jurisdiction. Violation of the
(c) an inferior court or tribunal has entertained rules of natural justice is acting without
an appeal where no appeal lies under an jurisdiction. Committing an error apparent
Act. on the face of the record leaves an author-
(d) the inferior tribunal is biased. ity open for being corrected by certiorari.
(e) an authority is acting under a law which is
Difference between Prohibition and
unconstitutional or ultra vires. Certiorari
It must be noted that prohibition lies only
against judicial or quasi-judicial functions There are many similarities between the two
(courts and tribunals). It does not lie against the writs—Prohibition and Certiorari. They are is-
legislative or executive functions. It cannot be sued against judicial or quasi-judicial authori-
issued against private persons or associations. ties. They are meant to confine the authorities
Prohibition issues so long as proceedings within the limits of their jurisdiction. The
are pending before a court or tribunal. If the grounds on which they are issued are similar.
court or tribunal has passed final orders in the But there are material differences in the scope
matter, no prohibition would lie. Where an elec- of these two writs.
tion tribunal had given its finding and finally They are issued at different stages of pro-
decided the election petition it was held that ceedings. If an inferior court having no juris-
prohibition would not lie. diction in a matter takes it up for hearing the
aggrieved person can petition for Prohibition.
Certiorari If such a court hears the matter and gives a deci-
Whenever any body of persons sion the proper remedy is Certiorari and not Pro-
(a) having legal authority hibition. Certiorari quashes the decision on the
(b) to determine questions affecting the rights ground of excess or want of jurisdiction. Pro-
of subjects and hibition issues where the court has not reached
(c) having the duty to act judicially and passed the stage of giving a decision. Prohi-
(d) acts in excess of their legal authority, cer- bition lies where the matter is pending and there
tiorari lies to quash the decision which is something to be prevented. After decision the
goes beyond its jurisdiction. matter is disposed of so the proper remedy is
All the four conditions enumerated above must Certiorari.
co-exist before a writ of certiorari may be is- Quo Warranto
sued. Four conditions can be dealt in detail in
Quo Warranto literally is a question—Where
the following manner:
is your warrant of appointment? The Court, by
(a) having legal authority It issues against
this writ examines the legality of the claim of a
a body vested with legal authority. Hence
person to a public office or franchise. If he is not
it does not lie against a private individual
able to show his legal right he may be ousted by
or a domestic tribunal having no statutory
the Court.
power. The writ of Quo Warranto will issue if the
(b) to determine questions affecting the following conditions are satisfied.
rights of the subject The determination 1. The office must be a public office, an of-
of the authority must affect the rights of fice of public nature. An office in which
the people. Rights may be personal, pecu- the public are interested. The treasurer of a
niary, proprietary or of any other type. It society or a charitable trust is not a public
may be a common law right or right born office. The office of advocate-general is a

CLAT.indb 136 31/03/2009 11:41:50


CONSTITUTIONAL LAW 137

public office and so is a judge of the High tion where it had none or exceeding its ju-
Court. risdiction where it had some, handed over
2. The office must be a substantive office. It a final decision.
means an office independently entitled. A 6. The object of Prohibition is prevention.
ministerial post where a person has to dis- The object of Certiorari is prevention and
charge the functions of a deputy or servant cure both.
at the pleasure of another is not a substan- 7. Mandamus commands a person to perform
tive office. a function which it is under a legal duty
3. The petitioner in case of Quo Warranto to perform. Quo Warranto is an enquiry
does not seek to enforce any right of to ascertain whether a person holding an
his. The petitioner is not required to be a office has legal authority to do so. When
claimed to that office or to have any per- he is not able to support his claim he be
sonal interest in it. Quo Warranto proceed- ordered to vacate.
ing is an exception to the rule that only a Article 32 and 226
person who is aggrieved may apply for a The Supreme Court has been empowered by Ar-
writ. Any public minded person can move ticle 32 to issue directions, orders or writs for
the court to obtain this writ. the enforcement of fundamental rights. A High
It is not necessary to implead the appoint- Court has under Article 226 such power not only
ing authority as a respondent in the proceedings. in relation to fundamental rights but also for any
If the Governor has appointed a person as Chief other purpose. Thus the power of a High Court
Minster, in a writ seeking quo warranto against is wider. It can be invoked for any other purpose
the Chief Minister only the person occupying while access to Supreme Court under Article 32
the office of Chief Minister will be a party. The is only for the purpose of enforcement of a fun-
Governor need not be made answerable to the damental right.
Court. Article 32 is a basic feature of the Constitu-
Various Writs Distinguished from Each tion. If cannot be omitted by amending the Con-
Other stitution. Article 32 being a fundamental right
an aggrieved person may access the Supreme
1. Mandamus commands activity. Prohibi-
Court directly, that is, without first resorting to
tion orders inactivity. Prohibition stops an
the High Court.
authority in the middle of a proceedings. It
prevents unsurpation of jurisdiction. Article 33
2. Mandamus is generally addressed to ad- Article 33 provides an exception to the fun-
ministrative authorities. Certiorari and damental rights. There are certain arms of the
Prohibition generally control the courts executive where freedom has to be controlled.
and tribunals. Armed Forces, Police, intelligence agencies
3. Certiorari and Prohibition are instruments are such organisations. This article confers on
to control an inferior tribunal which has the Parliament power to make law setting out
exceeded its jurisdiction or wrongly exer- the limits within which the fundamental rights
cised its jurisdiction. Mandamus is issued would be available to the members of the or-
against an inferior tribunal which has de- ganisations specified in Article 33.
clined to exercise its jurisdiction. The Army Act, The Navy Act, The Air
4. Mandamus may direct a tribunal to pro- Force Act, The Border Security Force Act and
ceed according to law. Certiorary quashes many such other Acts curtail the right of asso-
the proceedings or removes the proceed- ciation under Article 19 (1) (c). The Police Forc-
ings to itself on the ground of lack of ju- es (Restriction of Rights) Act, 1966 declared a
risdiction or error apparent on the face of member of police force cannot be a member of
the record etc. a trade union or labour union or political asso-
5. The grounds for issue of Certiorari and ciation.
Prohibition are the same. Prohibition is The power to make law under Article 33
issued at an earlier stage when the mat- is given to Parliament and not to State legisla-
ter has not come to a close, to prevent the tures.
tribunal from going ahead. Certiorari lies Article 34
where a tribunal after exercising jurisdic- Article 34 contemplates a situation where mar-

CLAT.indb 137 31/03/2009 11:41:50


138 CONSTITUTIONAL LAW

tial law has been declared in a particular part of The Gandhian Principles
the country. If during martial law any illegali- Article 40—Organization of Village Pancha-
ties are committed in the restoration of order the yats.
Parliament is given the power to pass an Act of Article 46—Promotion of educational and
Indemnity. Martial law is different from a proc- economic interests of the SCs, the STs and the
lamation under Article 352 (1). other weaker sections of the society.
Article 48—Organization of agriculture
DIRECTIVE PRINCIPLES OF STATE and animal husbandry on modern and scientific
POLICY lines to prohibit the slaughter of cows, calves
and other milch and draught animals.
Articles 36 to 51 deal with the provisions of the Article 43—To promote cottage industry.
Directive Principles which are contained in Part Article 47—To bring about the prohibition
IV of the Constitution. This novel feature of the of intoxicating drinks and drugs that are injuri-
Constitution has been adopted from the Consti- ous to health.
tution of the Ireland. The Western Liberal Principles
This concept is the latest development in
the Constitutional Governments throughout the Article 44—Uniform Civil Code for the
citizens.
world with the growing acceptance of a ‘Welfare
Article 45—Provide free and compulsory
State’. The Directive Principles of the Constitu-
education for children below 14 years.
tion of India are a unique blend of Socialism,
Article 50—Separation of the Judiciary
Gandhism. Western liberalism and the ideals of from the Executive.
the Indian freedom movement. Article 51—To promote international peace
They are in the nature of directions or in- and amity.
structions to the State. Article 36 clearly directs
the State to secure and protect a social order Difference Between FR and DPSP
which stands for the welfare of the people. The Fundamental Rights provide the foundation
Article 37 says that Directive Principles are of political democracy in India whereas the
not justiciable but are fundamental to the Gover- Directives spell out the character of social and
nance of the country, and the State has the duty economic democracy in India.
in applying the Directive Principles of State Fundamental rights are in the form of
Policies (DPSPs) negative obligations of the State i.e. injunction
These principles can be classified under the against the actions of the State. The Directive
following categories: Principles are, on the contrary, positive obliga-
tions of the State towards the citizen. Whereas
Socialist Principles the Fundamental Rights are justiciable, the Di-
Article 38—to secure a social order for the pro- rective Principles are non-justiciable.
motion of welfare of the people. Relationship between Fundamental
Article 39—to strive to minimise inequali- Rights and Directive Principles—
ties of income. Doctrine of Harmonization
Article 39 (b)—Ownership and control of The Supreme Court in various cases has evolved
material resources of the community shall be so a ‘Doctrine or Theory of Harmonization’.
distributed so as to subserve the common good. It has further stated that both the Funda-
Article 39 (d)—Equal pay for equal work. mental Rights and the Directive Principles are
Article 39 (e)—Health and strength of in fact supplementary to each other and together
workers, and the tender age of children must not constitute an integrated scheme.
be abused. It has also held that where this is not pos-
Article 39 A—Equal justice and fee legal sible, the Fundamental Rights shall prevail over
aid. the Directive Principles.
Article 42—Provision of just and humane The present position is that only Article 39
conditions for work and maternity relief. (a) and Article 39 (b) can be given precedence
Article 43 A—Participation of workers in over Article 14, 19 and not all the Directive
the management of the industries. Principles.

CLAT.indb 138 31/03/2009 11:41:50


CONSTITUTIONAL LAW 139

Importance of the Directive Principles tus, facilities and opportunities among individu-
Article 37 declares Directive Principles as fun- als and groups [Article 38(2)]
damental in the Governance of the Country. 86th Amendment Act, 2002
Since the Government is answerable to the Provision for early childhood care and educa-
people, the Directive Principles act as a sign tion to children below the age of six years [Ar-
post to all succeeding Governments. ticle 45],
The Directive Principles provide the yard-
Directives in other parts (not in Part IV
stick for assessing the successes or failures of
of the Constitution
these Governments.
However, following are the areas where Article 350 A: It enjoins every State and every
substantial success has been achieved: local authority within the State to provide ade-
1. Article 37 embodies provisions specifical- quate facilities for the instructions in the mother
ly to promote the concept of Welfare State. tongue at the primary stage to children of lin-
In pursuance of these goals the State has guistic minority areas.
enacted legislations on land reforms, land Article 351: It enjoins the Union to pro-
ceiling, law providing for minimum wage mote the spread of Hindi Language so that it
and equal pay for equal work for both men may serve as a medium of expression of all the
and women, etc. elements of the composite culture of India.
2. The 73 Constitution (Amendment Act,) Article 335: It says that the claims of SC/
Act, 1992, is in pursuit of implement- ST shall be taken into consideration, consis-
ing Article 40. Even before this, Village tently with the maintenance of efficiency of ad-
Panchayats were organised on the rec- ministration, in the making of appointments to
ommendations of the Balwant Rai Mehta serves and posts in connection with affairs of the
Committee in 1959. Union or of a State.
3. There have been many factory legislations
to make the conditions at the work places
humane for workers (Article 42). FUNDAMENTAL DUTIES
4. Promotion of cottage industries has been
These are included in the Indian Constitution
one of the main aspects of the economic
by the 42nd Amendment Act, 1976. It is based
policy of the government and there exists
on the Japanese model. 10 duties of the citizens
the Khadi and Village Industries Commis-
towards the State have been enumerated by in-
sion for the purpose. Apart from this, the
serting Article 51-A in Part IVA of our Consti-
Silk Board, the Handloom Board, in the
tution. Rights and duties are correlative. These
SSI Board, NABARD, etc., have been cre-
serve as constant reminder to every citizen that,
ated.
while the Constitution specifically confers on
5. The policy of preferential treatment in ed-
them certain FRs, it also requires them to ob-
ucation, administration and economy for
serve certain basic norms of democratic conduct
the weaker sections including the women,
and behaviour.
the SCs, the STs and the OBCs has been
a consistent plank of the Government Comparison with Directive Principles
Welfare Policy, the most recent being the Fundamental Duties are comparable to the Di-
implementation of the Mandal Commis- rective Principles. The duties are addressed to the
sion Report, for which Judicial clearance citizens. The country expects them to perform
was given by the Supreme Court in 1992
certain duties as citizens. They are not specta-
(Article 46)
tors but active participants in attainment of na-
6. Many schemes like the Integrated Child
tional goals. The Directive Principles are goals
Development Services, the Mid-day Meal
set up by the Constitution for all governments. It
Scheme and the policy of prohibition fol-
is the duty of the State to apply these principles
lowed by some states are a number of leg-
in making laws (Article 37). But in case a State
islations to implement the DPSPs. In fact,
does not implement a Directive Principle it can-
the very first Amendment Act, Act was for
not be penalized. There is no legal sanction. The
implementing the DPSPs.
only sanction is public opinion. Similarly the
44th Amendment Act, 1978 Duties have no sanction attached. The citizen
State shall minimize inequality in income, sta- must introspect and endeavour to perform these

CLAT.indb 139 31/03/2009 11:41:51


140 CONSTITUTIONAL LAW

duties. The sanction should be self imposed. the Vice-President, the Prime Minister and
They cannot be enforced by a court. Council of Ministers and the Attorney General.
For determining the Constitutionality of a
The President of India
law the court looks in appropriate cases at the
(Article 52)
Directive Principles. Similarly in suitable cases
the court will pay due regard to the Fundamental The President—the Head of the Union Execu-
Duties. The courts will uphold as valid any law tive
which prohibits an act which is violative of the Article 52 provides that there shall be a Presi-
duties. dent of India, article 53 (1) declares the Presi-
dent to be the head of the Union Executive. It
Enforcement of Duties
provides that the executive power of the Union
The duties incorporated in the Constitution by “shall be vested in the President and shall be ex-
the 42nd Amendment Act, are statutory duties ercised by him either directly or through officers
and shall be enforceable by law. Parliament, by subordinate to him”. However, he shall exercise
law, will provide penalties to be imposed for this power in accordance with the provisions of
failure to fulfill those duties and obligations. the Constitution.
The success of this provision would, however, Yet he is the nominal, titular, unreal and
depend much upon the manner in which and the dejure head of the state because the real head is
person against whom these duties would be en- the Prime Minister.
forced.
Qualifications for Election as President
Eleven Fundamental Duties for Citizens (Article 58)
1. To abide by and respect the Constitution, Article 58 lays down the following qualifica-
the National Flag and the National An- tions which a person must possess for being eli-
them. gible for election as President:
2. To cherish and follow the noble ideals of (a) He must be a citizen of India;
the freedom struggle. (b) He must have completed the age of thirty-
3. To uphold and protect the sovereignty, five years;
unity and integrity of India. (c) He must be qualified for election as a
4. To defend the country and render national member of the House of the People. He
service when required. must, therefore, be registered as a voter in
5. To promote common brotherhood and es- any Parliamentary Constituency.
tablish dignity of women. (d) He must not hold any office of profit un-
6. To preserve the rich heritage of the na- der the Government of India or the Gov-
tion’s composite culture. ernment of any State or under any local
7. To protect and improve natural environ- or other authority subject to the control of
ment. any of the said governments.
8. To develop scientific temper, humanism Explanation attached to Article 58 declares
and spirit of inquiry. that for the purpose of Article 58, a person shall
9. To safeguard public property and abjure not be deemed to hold any office of profit by
violence. reason only that he is the President or Vice-
10. To strive for excellence in all sphere of in- President of the Union or the Governor of any
dividual and collective activity. State or is a Minister either for the Union or for
11. It shall be duty of every citizen of India
any State.
who is a parent or guardian to provide op-
The expression office of profit is not de-
portunities for education to his child or as
fined in the Constitution.
the case may be, ward between the age of
6 and 14 years (Added by 86th Amend- Election
ment Act, 2002). (Article 54)
Article 54 provides: “The President shall be
UNION EXECUTIVE elected by the members of an electoral college
consisting of:
(Articles 52 to 78 and 123) (a) the elected members of both Houses of
The Union Executive consists of the President, Parliament; and

CLAT.indb 140 31/03/2009 11:41:51


CONSTITUTIONAL LAW 141

(b) the elected members of the Legislative As- be counted as one and other fractions would be
semblies of the States”. disregarded.
Article 54 thus provides for the constitu-
Doubts and Disputes in Respect of the
tion of an Electoral College for the election of
Election of the President
the President.
(Article 71)
The term “State” in Article 54 does not in-
clude a Union Territory. The Constitution (Sev- Article 71 (1) declared that all doubts and dis-
entieth Amendment) Act, 1992 has inserted an putes arising out of or in connection with the
Explanation at the end of Article 54 to the ef- election of the President shall be inquired into
fect that “in this Article and in Article 55, ‘State’ and decided by the Supreme Court whose deci-
includes the National Capital Territory of Delhi sion shall be final. If the election of the Presi-
and the Union Territory of Pondicherry”. dent is declared void by the Supreme Court,
It may be noticed that only elected mem- Clause (2) declares that the acts done by him in
bers of the Houses of Parliament and that of the exercise of his powers and performance of
the Legislative assemblies of the States are the his duties of the office of President, shall not be
members of the Electoral College the Nominat- invalidated by reason of the declaration his elec-
ed members do not take part in the election of tion as void.
the President. Article 71 empowers the Parliament to reg-
ulate by law any matter relating to or connected
Manner of Election with the election of the President. In the exercise
[Article 55 (1) and (2)] of the powers so conferred, Parliament enacted
Article 55 requires that, as far as practicable, the Presidential and Vice-Presidential Elections
there shall be maintained uniformity in the scale Act, 1952. The Act regulates various matters in
of representation among the States inter se as respect of the election of the President and that
well as parity between the States as a whole and of the Vice-President. The Act has been amend-
the Union. To achieve such uniformity and pari- ed by the Amendment Acts of 1974 and 1997.
ty, Article 55 contains the formula to be adopted Clause (4) of Article 71 declares that the
at the election of the President for determining election of a person as President shall not be
the number of votes which each member of the called in question on the ground of the existence
Electoral College shall be entitled to cast. of any vacancy for whatever reason among the
The formula provided is that every elected members of the Electoral College electing him.
member of the Legislative Assembly of a State
Procedures for Impeachment of the
shall have as many votes as there are multiples
President—Removal of the President
of one thousand in the quotient obtained by di-
(Article 61)
viding the population of the State by the total
number of the elected members of the Assem- Article 56 (1) (b) provides that the President
bly. may be removed from his office for the violation
If, after taking the said multiples of the of the Constitution by impeachment in the man-
thousand, the remainder is not less than five ner provided in Article 61. The only ground on
hundred, then the votes of each member referred which the President can be removed from his
to above, shall be further increased by one. If the office is the violation of the Constitution. The
remainder is less than 500, it would be ignored. procedure contained in Article 61, to be fol-
When the votes, which the elected mem- lowed for the impeachment of the President, is
bers of all the State Legislative Assemblies are as follows:
entitled to cast, are calculated, then the votes to (a) the proposal to prefer a charge against the
be cast by the elected members of the Parlia- President for violating the Constitution
ment shall be determined. Article 55 provides may be preferred or initiated in either
that each elected member of either House of House of Parliament,
Parliament shall have such number of votes as (b) the proposal to prefer such charge must be
may be obtained by dividing the total number contained in the form of a resolution,
of votes assigned to the members of the Legisla- (c) the resolution can be moved in either house
tive Assemblies of the States by the total number of the Parliament only after the expiry of
of the elected members of both the Houses of a notice of 14 days. The notice must be in
Parliament. Fraction exceeding one-half shall writing and signed by not less than one-

CLAT.indb 141 31/03/2009 11:41:51


142 CONSTITUTIONAL LAW

fourth of the total number of members of (b) No criminal proceedings, whatsoever,


the House in which the resolution is to be shall be instituted or continued against the
moved. The notice must express the inten- President in any court during the term of
tion to move such a resolution, his office.
(d) such resolution must be passed by the (c) No process for the arrest or imprisonment
House in which it is initiated by a major- of the President shall be issued from any
ity of not less than two-thirds of the total court during the term of his office.
membership of the House, (d) No civil proceedings in which the relief
(e) when the resolution is passed by the House is claimed against the President shall be
in which the charge has been preferred, the instituted during the term of his office in
other House shall investigate the charge or any court in respect of any act done or pur-
cause the charge to be investigated. Dur- ported to be done by him in his personal
ing this investigation, the President shall capacity, whether before or after he has
have the right to appear and to be repre- entered upon his office as President, until
sented, a notice in writing has been given to the
(f) If after such investigation, the House, in President and two months have elapsed
which the charge against the President since the service of that notice. The notice
is investigated, passes a resolution by a must state the nature of the proceedings,
majority of not less than two-thirds of the the cause of action therefore, the name,
total membership of the House, declaring residence and description of the party
that the charge preferred against the Presi- conducting the proceedings and the relief
dent, has been sustained, such resolution claimed.
shall have the effect of removing the Pres-
ident from his office, as from the date on Powers of the President
which the resolution is so passed. Legislative Powers
Privileges of the President 1. The President is in integral part of the Par-
(Article 361) liament of India.
2. He can summon or prorogue the Parlia-
Article 361 provides the following privileges for ment and dissolve the Lok Sabha. He can
the President: also summon a joint sitting of both the
(a) The President is not answerable to any Houses of Parliament, which is presided
court for the exercise and performance
over by the Speaker of the Lok Sabha.
of the powers and duties of his office or
3. He can address the Parliament at the com-
for any act done or purported to be done
mencement of the first session after each
by him in the exercise and performance
general election and the first session of
of those powers and duties. However, the
each year. He can send messages to the
conduct of the President may be brought
Houses of Parliament, whether with re-
under review by any Court or tribunal or
spect to a bill pending in the Parliament or
body which may be appointed or desig-
otherwise.
nated by the House of Parliament for the
4. He can appoint any member of the Lok
investigation of the charge against the
Sabha to preside over its proceedings
President in the impeachment proceedings
when the offices of both the Speaker and
taken under Article 61.
It may, however, be noticed that the im- the Deputy Speaker fall vacant. Similarly,
munity afforded to the President does not he can also appoint any member of the Ra-
restrict the right of any person to bring jya Sabha to preside over its proceedings
appropriate proceedings against the Gov- when the offices of both the Chairman and
ernment of lndia, for the exercise and the Deputy Chairman fall vacant.
performance of the powers and duties of 5. He nominates 12 members of the Rajya
his office, by the President. Also, the im- Sabha from amongst persons having spe-
munity available to the President under cial knowledge or practical experience in
Article 361, cannot be extended to the or- literature, science, art and social service.
ders passed by Ministers, in the name of He can nominated two members to the
the President under Article 77(l) or Article Lok Sabha from the Anglo-Indian com-
77(2) of the Constitution. munity.

CLAT.indb 142 31/03/2009 11:41:51


CONSTITUTIONAL LAW 143

6. He decides on questions as to disqualifi- (a) to prevent hasty and ill-considered legisla-


cations of members of the Parliament, in tion by the Parliament; and
consultation with the Election Commis- (b) to prevent a legislation which may be un-
sion. constitutional.
7. His prior recommendation or permission The veto power enjoyed by the executive
is needed to introduce certain types of in modern states can be classified into the fol-
bills in the Parliament. For example, a bill lowing four types:
involving expenditure from the Consoli- 1. Absolute veto, that is, withholding of as-
dated Fund of India, or a bill for the altera- sent to the bill passed by the legislature.
tion of boundaries of states or creation of 2. Qualified veto, which can be overridden
a new state. by the legislature with a higher majority.
8. When a bill is sent to the President after 3. Suspensive veto, which can be over ridden
it has been passed by the Parliament, he by the legislature with an ordinary major-
can: ity.
(i) give his assent to the bill, or 4. Pocket veto, that is, taking no action on
(ii) withhold his assent to the bill, or the bill passed by the legislature.
(iii) return the bill (if it is not a money bill) Of the above four, the President of India
for reconsideration of the Parliament. is vested with three—absolute veto, suspensive
However, if the bill is passed again by the veto and pocket veto. There is no qualified veto
Parliament, with or without amendments, in the case of Indian president; it is possessed by
the President has to give his assent to the the American president.
bill.
9. When a bill passed by a state legislature is Executive Powers
reserved by the governor for consid- 1. All executive actions of the Government
eration of the president, the president can: of India are formally taken in his name.
(i) give his assent to the bill, or 2. He appoints the prime minister and the
(ii) withhold his assent to the bill, or other ministers. They hold office during
(iii) direct the governor to return the bill his pleasure.
(if it is not money bill) for reconsider- 3. He can seek any information relating to
ation of the state legislature. It should the administration of affairs of the Union,
be noted here that it is not obligatory and proposals for legislation from the
for the president to give his assent prime minister. He can require the prime
even if the bill is again passed by the minister to submit, for consideration of
state legislature and sent again to him the council of ministers, any matter on
for his consideration. which a decision has been taken by a min-
ister but, which has not been considered
Veto Power of the President by the council.
A bill passed by the Parliament can become an 4. He appoints the attorney general of India
act only if it receives the assent of the president. and determines his remuneration. The at-
When such a bill is presented to the president for torney general holds office during the
his assent, he has three alternatives under Article pleasure of the president.
111 of the Constitution): 5. He appoints the comptroller and auditor
1. He may give his assent to the bill, or general of India, the chief election com-
2. He may withhold his assent to the bill, or missioner and other election commis-
3. He may return the bill (if it is not money sioners, the chairman and members of
bill)for reconsideration of the Parliament. the Union Public Service Commission,
However, if the bill is passed again by the the governors of states, the chairman and
Parliament with or without amendments members of finance commission, and so
and again presented to the president, the on.
president must give his assent to the bill. 6. He can appoint a commission to investi-
Thus, the president has the veto power over gate into the conditions of SCs, STs and
the bills passed by the Parliament, that is, he can other backward classes.
withhold his assent to the bills. The object of 7. He can appoint an inter-state council to
conferring this power on the president is two- promote Centre-state and inter-state coop-
fold: eration.

CLAT.indb 143 31/03/2009 11:41:51


144 CONSTITUTIONAL LAW

8. He directly administers the union territo- the president. However, they are subject to
ries through administrators appointed by the approval of the Parliament. He repre-
him. sents India in international forums and af-
9. He can declare any area as scheduled area fairs and sends and receives diplomats like
and has powers with respect to the admin- ambassadors, high commissioners, and so
istration of scheduled areas and tribal ar- on.
eas. (b) He is the supreme commander of the de-
10. He can make regulations for the peace, fence forces of India. In that capacity, he
progress and good government of the An- appoints the chiefs of the Army, the Navy
daman and Nicobar Islands, Lakshadweep, and the Air Force. He can declare was or
Dadra and Nagar Haveli and Daman and conclude peace, subject to the approval of
Diu. In case of Pondicherry also, the presi- the Parliament.
dent can legislate by making regulations
Ordinance Making Power (Article 123)
but only when the assembly is suspended
Article 123(1) provides: “If, at any time, ex-
or dissolved.
cept when both Houses of Parliament are in
Financial Powers Session, the President is satisfied that circum-
(a) Money bills can be introduced in the Par- stances exist, which render it necessary for him
liament only with his prior recommenda- to take immediate action, he may promulgate
tion. such Ordinances as the circumstances appear
(b) He causes to be laid before the Parliament to him to require.” An Ordinance promulgated
the annual financial statement (i.e., the by the President has the same force and effect
Union Budget). as an Act of the Parliament. The President may
(c) No demand for a grant can be made except withdraw the Ordinance at any time.
on his recommendation. The Ordinance promulgated by the Presi-
(d) He can make advances out of the contin- dent is required to be laid before both Houses of
gency fund of India to meet any unfore- Parliament. It ceases to operate at the expira-
seen expenditure. tion of six weeks from the reassembly of Parlia-
(e) He constitutes a finance commission after ment. However, if resolutions disapproving the
every five years to recommend the distri- Ordinance are passed by both Houses before the
bution of revenues between the Centre and expiry of these six weeks, the Ordinance ceases
the states. to operate on the day the second of these resolu-
tions is passed. The period of six weeks shall be
Judicial Powers
reckoned from the date, the latter House meets.
(a) He appoints the Chief Justice and the
The President may promulgate Ordinances
judges of the Supreme Court and High
under Article 123 (1), with respect to all those
Courts.
matters with respect to which Parliament is
(b) He can seek advice from the Supreme
competent to make laws. The power of the Pres-
Court on any question of law or fact. How-
ident to promulgate Ordinances is thus declared
ever, the advice tendered by the Supreme
to be co-extensive with the power of the Parlia-
Court is not binding on the president.
ment to make laws. However, the President can
(c) He can grant pardon, reprieve, respite and
promulgate an ordinance only when the follow-
remission of punishment, or suspend, re-
ing two conditions are existing:
mit or commute the sentence of any per-
(a) when both Houses of Parliament are not in
son convicted of any offence: (i) In all
Session. Thus, he can promulgate the or-
cases where the punishment or sentence is
dinance when one of the House is in Ses-
by a court martial; (ii) In all cases where
sion.
the punishment or sentence is for an of-
(b) when circumstances exist which render it
fence against a Union law; and (iii) In all
necessary for the President to take imme-
cases where the sentence is a sentence of
diate action.
death.
Ordinances per se are against the spirit of
Diplomatic and Military Powers democracy and not conducive to the develop-
(a) The international treaties and agreements ment of the best Parliamentary traditions. How-
are negotiated and concluded on behalf of ever, the issuance of Ordinances has been held

CLAT.indb 144 31/03/2009 11:41:51


CONSTITUTIONAL LAW 145

desirable to deal with an unforeseen and urgent the President. The court further explained that
situation. Justifying the provision in the Constit- the object of conferring the Ordinance making
uent Assembly, Dr B. R. Ambedkar said— power on the President was to enable the Execu-
“The emergency must be dealt with, and tive to deal with any unforeseen or urgent mat-
it seems to me that the only solution is ters which might well include a situation created
to confer upon the President the power by a law being declared void by a court of law.
to promulgate a law which will enable In A. K. Roy vs Union of India, the National
the executive to deal with that particular Security Ordinance, 1980 provided for, deten-
situation because it cannot resort to the tion of persons, acting in a manner prejudicial to
ordinary process of law because ... the the defence of India, security of India, security
legislature is not in session”. of the State. and friendly relations with foreign
Case Laws powers. The Ordinance was challenged on the
ground that it suffered from vagueness and arbi-
In R. C. Cooper vs Union of India, the Banking
trariness. The Supreme Court upheld the consti-
Companies (Acquisition and Transfer of Under-
tutional validity of the ordinance and held that it
takings) Ordinance, 1969 was challenged on the
was not violative of Article 14.
ground that the President had not satisfied him-
However, the Supreme Court held that Or-
self as regards the urgency of the circumstances.
dinance making power of the President would
The Supreme Court, however, held that “under
be subject to the tests of vagueness, arbitrari-
the Constitution, the President being the con-
ness, reasonableness, public interest, and that it
stitutional head, normally acts, in all matters,
was passed during the recess of the Union Par-
including the promulgation of an Ordinance,
liament.
on the advice of his Council of Ministers”. The
Ordinance is promulgated in the name of the In D. C. Wadhwa vs State of Bihar, it was
President, but it is, in truth, a promulgation on brought to the notice of the court that between
the advice of his Council of Ministers and on 1967 and 1980, in the State of Bihar; 256 Or-
their satisfaction. dinances were promulgated and all these were
It has been held that an Ordinance passed kept alive for periods ranging from one year to
under Article 123 stands on the same footing as 14 years by repromulgation from time to time.
an Act passed by the Legislature. It cannot be Out of these, 69 Ordinances were repromul-
treated as an executive action or an administra- gated several times with the prior permission of
tive decision. The courts cannot infer a legisla- the President. The Supreme Court struck down
tive malice in passing a statute. It is clothed with this exercise of the Ordinance making power as
all the attributes of an Act of the Legislature, a “subversion of the democratic process” and
carrying with it all its incidents, immunities and “colourable exercise of power”. The Court held
limitations under the Constitution. An Ordinance that such exercise of power amounted to a fraud
has been held to be a law under Article 21 of upon the Constitution and hence unconstitution-
the Constitution. As the Legislature can repeal al. The Court observed that the Executive could
an existing enactment or amend it, so also, the not usurp the functions assigned to the Legisla-
President, by an Ordinance, can repeal or amend ture under the Constitution.
an existing legislation. The Karnataka High Court in B. A. Hasan-
In R. K. Garg vs Union of India, 1 the Spe- hba vs State of Karnataka, has rightly pointed
cial Bearer Bonds (Immunities and Exemptions) out that since an Ordinance was not subjected
Ordinance, 1981 was challenged on the ground to consideration, reconsideration, modification
that it was beyond the Ordinance making power and redrafting (as is done while enacting a law
of the President as it related to the tax laws. Up- by the Legislature), the scrutiny of an Ordinance
holding the constitutionality of the Ordinance, must be undertaken by the Court with a high de-
the Supreme Court observed that the President gree of meticulousness, care and caution.
was competent to issue an Ordinance, amend- The power to issue an Ordinance has to be
ing or altering the tax laws also. The court fur- exercised on special and urgent occasions. But,
ther said that the Ordinance making power of nowadays it has become a routine exercise. The
the President was co-extensive with the power Ordinances have been promulgated and repro-
of the Parliament to make laws and no limita- mulgated without these being even considered
tion could be read into the Legislative Power of by the Legislature. It all makes a mockery of the

CLAT.indb 145 31/03/2009 11:41:51


146 CONSTITUTIONAL LAW

parliamentary democracy and a negation of the undergo the full term of imprisonment inflicted
rule of law. by the Court, though the order of conviction and
sentence passed by the Court still stands as it
Power to Grant Pardons (Article 72)
was. A remission of sentence, therefore, does
Article 72 confers power on the President “to
not mean acquittal.
grant pardons, reprieves, respites or remissions
of punishment, or to suspend, remit, or commute In Kuljeet Singh vs Lt Governor of Delhi, the
the sentence of any person convicted of an Supreme Court had the occasion of analysing
offence in the following cases: the exercise of power conferred by Article 72. In
(a) in all cases where the punishment or this case, the petitioners, Ranga and Billa, were
sentence is by a Court Martial. convicted for committing murder of two inno-
(b) in all cases where the punishment or cent children and were awarded death sentence
sentence is for an offence against any law by the Sessions Court, which was confirmed by
relating to a matter to which the executive the High Court. Their petition for Special Leave
power of the Union extends. under Article 136, against the judgment of the
(c) in all cases where the sentence is a sentence High Court, was dismissed by the Supreme
of death. Court. Thereafter, they presented a mercy pe-
The power conferred on the President, tition to the President for the grant of pardon,
however, does not affect the power conferred which was also rejected by him, without assign-
by any law on any officer of the Armed Forces ing any reason.
of the Union to suspend, remit, or commute a On this, the petitioners invoked the jurisdic-
sentence passed by Court Martial, and also the tion of the Supreme Court by a writ petition and
power exercisable by the Governor of the State contended that power conferred on the President
under any law, for the time being in force, to by Article 72 to grant pardons, etc. was coupled
suspend, remit or commute a sentence of death. with a duty to act fairly and reasonably.
However, the power conferred under Arti- The Supreme Court admitted the petition
cle 72 is absolute and cannot be fettered by any and by a general order stayed the execution
statutory provision such as Sections 432, 433 of all those convicts whose mercy petitions,
and 433A of the Code of Criminal Procedure, against the death sentence, were rejected by the
1974. President or Governor of a State. The Supreme
Court accepted the contention raised by the pe-
A pardon completely absolves the guilt of titioners that the power conferred under Article
the offender. When a convict is granted pardon, 72 should be exercised fairly and reasonably.
he is completely absolved from the punishment The Court further said, as was argued by the
imposed on him as also from all penal conse- petitioners, that they did not know whether the
quences and such disqualifications as disentitle Government of India had formulated any uni-
him from following his occupation and as are form standards or guidelines by which the exer-
concomitant of the conviction. cise of the constitutional power under Article 72
Reprieve means temporary suspension was intended to be and was, in fact, guided. The
of death sentence, for example, pending a pro- Supreme Court further ruled that the exercise of
ceeding for pardon or commutation. President’s power under Article 72 would have
Respite means awarding a lesser punish- to be examined from case to case. 12
ment on some special ground, for example, The Court, however, by an order, vacated
pregnancy of a woman offender. the stay granted to the petitioners and ordered
their execution on the ground that this was not
Commutation means the exchange of one
appropriate case in which the question of laying
form of punishment for another. For example,
down the guidelines would arise.
rigorous imprisonment is commuted to simple
The Court further said that even the most
imprisonment.
liberal use of the power under Article 72 could
Remission means reduction of the amount not have persuaded the President to impose any-
of sentence without changing its character. An thing less than a sentence of death in the present
order of remission does not, in any way, inter- case and more so, in view of the considerations
fere with the order of the Court. It merely af- taken by the Court in its judgment while con-
fects the execution of the sentence passed by the firming their death sentence.
Court and frees the convict from his liability to The Supreme Court thus retained the pow-

CLAT.indb 146 31/03/2009 11:41:51


CONSTITUTIONAL LAW 147

er of judicial review even in a matter which has arbitrarily or mala fide and ordinarily guidelines
been vested by the Constitution solely in the for fair and equal execution are guarantors of
Executive. The question of standards and guide- valid play of power”. The Bench stressed the
lines for the exercise of the power, by the Presi- point that the power to pardon, grant remission
dent, under Article 72, however, was left open and commutation, being of the greatest moment,
by the Court. could not be a law unto itself but it must be in-
In Sher Singh vs State of Punjab, the Supreme formed by the finer canons of constitutionalism.
Court took serious note of the delay in decision In Swarn Singh vs State of Uttar Pradesh, one
of the mercy petitions filed under Articles 72 Shri Doodh Nath was co-convicted for murder-
and 161. The Court observed that “a self-im- ing Shri Joginder Singh, the father of the ap-
posed rule should be followed by the Executive pellant and was sentenced to imprisonment for
authorities vigorously, that every such petition life, which sentence was finally approved by the
shall be disposed of within a period of three Supreme Court. His petition filed under Article
months from the date when it is received. Long 161 for grant of reprieve and remission was dis-
delays in the disposal of these petitions are a se- missed by the Governor. However, within less
rious hurdle in the dispensation of justice and than two years, he presented a second petition
indeed such delays tend to shake the confidence and succeeded in escaping from prison as the
of the people in the system of justice”. Governor granted him remission of the remain-
In Kehar Singh vs Union of India, a Constitu- ing period of his life sentence. A writ petition
tion Bench, of the Supreme Court held that while of the appellant, challenging this aforesaid ac-
exercising his pardoning powers it was open to tion of the Governor, was dismissed by a Divi-
the President to scrutinize the findings/witnesses sion Bench of the Allahabad High Court, which
on the record and come to a different conclusion opined that the decision of the Governor under
both on the guilt of the accused and the sentence Article 161, was not justiciable.
imposed on him. In doing so, the President did In the appeal before the Supreme Court,
not amend or modify or supersede the judicial it was alleged by the appellant that Shri Doodh
record which remained intact. The petitioner, Nath was already accused in five other criminal
the Court held, had no right to be heard in per- cases involving serious offences. That, many
son by the President. The manner of consider- other relevant materials were not posted before
ation of the mercy petition, the Court said, was the Governor when he considered the question of
entirely within the discretion of the President. granting reprieve to the convict. Further, the fact
The Court need not spell out specific guidelines that the same Governor had earlier dismissed a
for the exercise of power under Article 72. This petition for grant of reprieve and remission of
was so because the power under Article 72 was the sentence passed on him by the Court in the
of widest amplitude and could contemplate myr- very same case, was not brought to the notice of
iad kinds and categories of cases with facts and the Governor.
situations varying from case to case. The Court Since, the Governor was not posted with
further said that the President could not be asked material facts, the Supreme Court held that the
to give reasons or his order. The power to grant Governor was apparently deprived of the oppor-
pardon was a part of the constitutional scheme tunity to exercise the power in a fair and just
and the order of the President could not be sub- manner.
jected to judicial review on its merits. Holding that the order of the Governor
The Constitution Bench, however, laid granting remission of the sentence suffered from
down that the Presidential order under Article arbitrariness, the Court quashed the order to en-
72 could be subjected to judicial review within able the Governor to pass a fresh order in the
the strict limitation defined in Maru Ram vs light of the material facts not disclosed earlier.
Union of India. The Court thus ruled that the orders of the
In Maru Ram case, a Constitutional Bench of Governor under Article 161 were subjected to
the Supreme Court had observed that the power judicial review if they were mala fide and ar-
of pardon, commutation and release, under Arti- bitrary.
cles 72 and 161, though very wide, could not run Later, in Satpal vs State of Haryana, the Su-
riot. The Court said, “all public power, including preme Court set aside the order of the Governor
constitutional power, shall never be exercisable granting pardon. It was found that the Governor

CLAT.indb 147 31/03/2009 11:41:51


148 CONSTITUTIONAL LAW

had not applied his mind to the material on re- While this Proclamation was continuing in
cord and had mechanically passed the order just force, a fresh Proclamation of emergency was
to allow the prisoner to overcome the conviction made by the President on 25 June 1975, on the
and sentence passed by the Court. It was noticed ground of internal disturbance.
that the entire file for the grant of pardon, had Both these proclamations were continued
been processed with uncanny haste by the au- in force until revoked by the President on 27
thorities. The unusual interest and zeal shown March 1977 and 21 March 1977, respectively.
by the authorities in preparing the case for grant Proclamation of an emergency results into
of pardon, was also apparent. The Court reiter- serious consequences.
ated the law laid down in Maru Ram case, and It has virtually the effect of amending the
followed in the Swam Singh case. Constitution by converting the federal fabric, for
It has thus been said to be undeniable that the duration, into a unitary State.
judicial review of the Order of the President or It also adversely affects the enforcement of
the Governor under Article 72 or Article 161, as the fundamental rights of the people.
the case may be, is available and their Orders The question as to whether a grave emer-
can be impugned on the following grounds: gency exists or whether there is imminent dan-
(a) that the Order has been passed without ap- ger thereof, is left to the satisfaction of the ex-
plication of mind; ecutive, for it is obviously in the best position to
(b) that the Order is mala fide; judge the situation.
(c) that the Order has been passed on extrane- At the same time, there is the co-relative
ous or wholly irrelevant considerations; danger of the abuse of such extraordinary power
(d) that relevant materials have been kept out leading to totalitarianism.
of consideration; Such a power should therefore, be invoked
(e) that the Order suffers from arbitrariness. only in exceptional and emergency

Emergency Powers (Articles 352 to 360) National Emergency


The Constitution of India, envisages three types (Articles 352 to 354, 250, 83, 358, 359)
of emergencies. These are: Proclamation of Emergency (Article 352)
(1) Emergency arising from a threat to the se-
curity of India. It is commonly known as Who can proclaim Emergency?
“National Emergency” (Article 352). Article 352 (1), as originally enacted provided
(2) Failure of constitutional machinery in a that the President when satisfied that a grave
State. In a common man’s language, it is emergency existed whereby the security of India
termed as “State Emergency” or “Presi- in or any part of the territory of India was threat-
dent’s rule” or “Governor’s Rule” in the ened, could by making a proclamation, declare
State (Article 356). emergency.
(3) Financial Emergency (Article 360). However, ours being a parliamentary sys-
Constitutionally speaking, the expression tem, the President could proclaim emergency
“Proclamation of Emergency” refers only to a only when advised by the Council of Ministers
proclamation made under Clause (1) of Article through the Prime Minister. This provision was
352. Therefore, the expression “Proclamation said to have been misused by the Prime Min-
of Emergency” or the term “Emergency” when- ister, in 1975, when the President was advised
ever, used, should not include the situations pro- to make a proclamation of emergency on the
vided for under Articles 356 and 360. ground of internal disturbance.
For the first time, a Proclamation of emer- In order to minimize the chances of abuse
gency was issued by the President on 26 Octo- of power to declare emergency and to ensure that
ber 1962, on the ground of external aggression a proclamation of emergency is issued only after
from Chinese side. due consideration, Article 352 was amended by
This Proclamation continued to be in force the Constitution (44th Amendment) Act, 1978.
until 10 January 1968, when it was withdrawn After 44th Amendment, it is provided that
by the President by making a fresh proclama- the President can make a Proclamation declaring
tion. Emergency, only when he receives in writing the
Then, on 3 December 1971, another Procla- decision of the Union Cabinet to this effect.
mation of emergency was issued, on the ground Grounds for Proclamation of Emergency
of external aggression from Pakistan side. Prior to the 44th Amendment, Article 352 (1)

CLAT.indb 148 31/03/2009 11:41:51


CONSTITUTIONAL LAW 149

provided that the President could make a Proc- House of People takes place during the period
lamation declaring Emergency on the ground of one month (within which the Proclamation is
of threat to the security of India. The security required to be approved by both Houses of Par-
might be threatened by: liament) without the House of People passing a
(1) War resolution approving the Proclamation before
(2) External Aggression the expiry of the said period of one month, the
(3) Internal Disturbance Proclamation ceases to operate at the expiration
The expression “internal disturbance” is of 30 days from the date on which the House of
too vague and wide. It may cover a minor dis- People first sits after its reconstitution after elec-
turbance of law and order or eyen a political tions. But, the Council of States (Rajya Sabha,
agitation. the permanent House) must have passed a reso-
With a view to exclude the possibility of lution, approving the Proclamation within the
an emergency being proclaimed on the ground initial period of one month.
of internal disturbance of any nature, minor or If during the initial period of one month,
grave, the 44th Amendment has substituted the none of the Houses of Parliament takes any ac-
expression “internal disturbance” by the expres- tion, neither approves nor disapproves the Proc-
sion “armed rebellion”. Thus after 44th Amend- lamation, the Proclamation ceases to operate at
ment, internal disturbance not amounting to the expiration of this one month.
armed rebellion would not be a ground for the If only one House passes a resolution with-
issue of a Proclamation of Emergency. This in the initial period of one month, approving the
change has, thus restricted the scope of what Proclamation and no such resolution is passed
may be known as internal emergency. by the other House, the Proclamation ceases to
After 44th Amendment, a Proclamation of operate at the expiration of this one month.
Emergency may be issued on any of the follow- If Rajya Sabha passes a resolution, within
ing grounds: the initial period of one month rejecting or dis-
(a) War approving the Proclamation, the Proclamation
(b) External Aggression ceases to operate at the expiration of this one
(c) Armed Rebellion month.
However, actual occurrence of war, exter- If Lok Sabha passes a resolution, within
nal aggression or armed rebellion, is not a con- the initial period of one month, disapproving or
dition precedent. rejecting the Proclamation, the President must
Such a Proclamation may be made by the revoke the Proclamation immediately.
President, even before the actual occurrence of
Duration of Proclamation of Emergency—
the above events, when the President is satisfied
With Approval [Article 352(5)]
that there is imminent danger of war, external
Prior to the 44th Amendment, 1978, the Procla-
aggression or armed rebellion.
mation of Emergency issued under Article 352,
Duration of Proclamation of Emergency— was required to be approved by both Houses of
Without approval [Article 352 (4)] the Parliament only once, i.e., within 2 months
Prior to the 44th amendment, a Proclamation of from the date of Proclamation. Thus, once ap-
emergency made under Article 352(1) could be proved by both the Houses, the Proclamation
continued in force for a period of two months could continue in force indefinitely, until the
from the date of Proclamation without the Proc- President revoked it by making another Procla-
lamation being approved by both Houses of Par- mation for this purpose. For example, the Proc-
liament. lamation made on 26 October 1962 continued
The 44th amendment Act, 1978, has re- in force till 10 January 1968, without its con-
duced this period to one month. Clause (4) of tinuance being reconsidered by the Houses of
Article 352, thus, provides that a Proclamation Parliament. How long the Proclamation could
made under Article 352(1) shall cease to operate be continued was a matter for the Executive to
at the expiration of “one month” unless before decide, as the Executive could know the require-
the expiration of that period it has been approved ments of the situation and the effect of compul-
by resolutions of both Houses of Parliament. sive factors which operated during periods of
However, if such a Proclamation is issued grave crisis.
at a time when the House of People (Lok Sabha) After the 44th Amendment, 1978, a Procla-
has been dissolved, or the dissolution of the mation is required to be approved by both Hous-

CLAT.indb 149 31/03/2009 11:41:51


150 CONSTITUTIONAL LAW

es of the Parliament within one month of the If a Proclamation of Emergency is in op-


date of such Proclamation. A Proclamation so eration only in a part of the territory of India
approved, shall remain in force for six months and not in the whole of India, the Union may
from the date of such approval. issue executive directions not only to the State
If the resolutions for approval are passed in which the Proclamation is in operation, but
by the two Houses on different dates, the period also to any other State if and in so far as the
of six months would run from the date of the security of India or any part of the territory of
passing of the later of the resolutions. India thereof is threatened by activities in or in
For continuance of the Proclamation of relation to the part of the territory of India in
Emergency beyond this period of six months. which the Proclamation of Emergency is in op-
Clause (5) of Article 352 requires the approval eration. This provision is added in order to make
by both Houses of Parliament Thus, each time, effective the Emergency in the area where it had
it is approved by both Houses of Parliament, the been imposed by curbing undesirable activities
duration of such Proclamation is for six months. in adjoining areas.
Therefore, after the 44th Amendment, 1978, the Legislative Power of Parliament Extends to
question, as to whether the Emergency should State Matters [Articles 250 and 353 (b)]
continue in force or not, be periodically deter- Article 250 provides that during the operation
mined by both the Houses of Parliament, every of a Proclamation of Emergency, Parliament be-
six months. comes empowered to make laws with respect to
If during such period of six months for any matter contained in the State List.
which the Proclamation is duly approved, the The normal peacetime distribution of leg-
dissolution of the House of People takes place islative powers between the Union and States is
and resolution approving its continuance in therefore, suspended.
force has been passed by the Rajya Sabha, but Clause (b) of Article 353, further empow-
no resolution with respect to the continuance ers the Parliament to make laws with respect to
in force of such Proclamation, has been passed any matter notwithstanding that it is one which
by the Lok Sabha during the said period of six is not enumerated in the Union List.
months, the Proclamation shall cease to oper- In the exercise of this law-making power,
ate at the expiration of 30 days from the date Parliament may confer powers and impose du-
on which the Lok Sabha first sits after its recon- ties upon the Union or its officers and authori-
stitution. ties even though the law pertain, to a matter not
contained in the Union List.
Consequences of Proclamation of
If the Proclamation of Emergency is in
Emergency
operation only in a part of the territory of In-
The following consequences ensue on the Proc- dia, such law-making power of Parliament also
lamation of Emergency made under Article extends to other States if the security of India
352: or other States is threatened by activities in that
Extension of Executive Power of the Union part of India in which the Proclamation of Emer-
[Article 353 (a)] gency is in operation.
Article 353 (a) provides that while a Proclama- Since the power of the President to Promul-
tion of Emergency is in operation, the executive gate Ordinances is co-extensive with legislative
power of the Union extends to giving directions power of the Parliament, during the operation of
a Proclamation of Emergency, President’s pow-
to any State as to the manner in which the execu-
er also extends to promulgate Ordinance with
tive power of the State is to be exercised.
respect to matters not contained in the Union
During normal times, the Union cannot
List.
issue executive directions to the States laying
down, the manner in which, the States should Alteration in Distribution of Revenue [Ar-
exercise their executive power, except for the ticle 354(1)]
purposes of Articles 256 and 257. Clause (1) of Article 354 provides that while a
During the operation of a Proclamation of Proclamation of Emergency is in operation, the
Emergency, the Union may issue directions to President may, by order, alter or modify the fi-
States, for any purpose. The Union may control nancial arrangements contained in Articles 268
or direct the exercise of executive power by the and 279 relating to distribution of revenue be-
States. tween the Union and States.

CLAT.indb 150 31/03/2009 11:41:51


CONSTITUTIONAL LAW 151

Every order made under Clause (1) of Arti- had originated earlier and continued from year
cle 354 is required to be laid before each House to year for a decade till the issuance of the Proc-
of Parliament. Such an order or the modification lamation of Emergency in 1971.
so effect in the financial arrangements, would The Newsprint Policy of 1972-73 which
not remain in force beyond the end of the finan- was violative of Article 19 (1) (a), was not pro-
cial year in which the Proclamation of Emer- tected during the operation of the Proclamation
gency ceases to operate. of Emergency from attack under Article 19 (1)
(a).
Extension in Duration of House of People and
Since, the restrictions under newsprint
Legislative Assemblies of States (Article 83
policy were imposed before the Proclamation of
and Article 172)
Emergency, it could be challenged as violating
Article 83 provides that while a Proclamation
Article 191 (1) (a).
of emergency is in operation, the life of the
The Court further laid down that the Proc-
House of People (Lok Sabha) may be extended
lamation of Emergency would not authorize the
beyond its normal duration of five years. It can
taking of detrimental executive action during
be done by Parliament by making a law to this
Emergency affecting the fundamental rights of
effect. However, this extension would be for one
Article 19, without any legislative authority.
year at a time. It may further be extended by law
This observation has been expressly incorpo-
by Parliament each time for another one year,
rated into Article 358 by the Constitution (44th
but not extending beyond six months after the
Amendment) Act, 1978.
Proclamation of Emergency ceases to operate.
The Constitution (44th Amendment) Act,
Proviso to Clause (1) of Article 172 pro-
1978 has made following two important chang-
vides that Parliament may by law extend the
es, restricting the scope of Article 358:
life of the Legislative Assemblies of the States
Firstly, Article 19 will be suspended un-
by one year each time during the operation of a
der Article 358 only in case a Proclamation of
Proclamation of Emergency, but not beyond six
Emergency is made on the ground of ‘war’ or
months from the date on which the Proclama-
‘external aggression’, and not in case of a Proc-
tion ceases to operate.
lamation issued on the ground of ‘armed rebel-
Suspension of Fundamental Freedoms of Ar- lion’.
ticle 19 (Article 58) Secondly, Article 358 will not protect any
Prior to 44th Amendment, 1978, Article 358 law (violative of Article 19) which does not
provided that as soon as a Proclamation of contain a recital to the effect that such law is
Emergency was made, under Article 352, the in relation to the Proclamation of Emergency in
operation of Article 19 got suspended. operation when it is made. Article 358, will also
Thus as soon as, a Proclamation of Emer- not protect any executive action taken otherwise
gency was issued under Article 152, Article 358 than under a law containing the above said re-
removed the fetters created on the legislative cital.
and executive powers of the State by Article 19. It means that after 44th Amendment, 1978,
The suspension of Article 19 was complete it is only that law which is enacted during and
during such period and legislative and executive in relation to the Proclamation of Emergency,
action which contravened Article 19 could not which is immune under Article 358.
be questioned even after the Proclamation of Article 358 (1), however, further provides
Emergency ceased to operate. that such a law shall cease to have effect, as soon
Article 358 was said to be prospective and as, the Proclamation ceases to operate, “except
therefore, it did not operate to validate a legis- as respects things done or omitted to be done
lative provision which was invalid before the before the law so ceases to have effect”.
issuance of the Proclamation of Emergency. It Explaining the scope of Article 158, the
did not protect an executive action in purported Supreme Court in M. M. Pathak vs Union of
exercise of power conferred by any pre-Emer- India, observed that, during the operation of a
gency law, which was invalid being violative of Proclamation of Emergency, the fundamental
the freedoms contained in Article 19. rights guaranteed by Articles 14 and 19 were
In Bennett Coleman and Co. Ltd. vs Union not suspended, but their operation was only sus-
of India, the Newsprint Policy of 1972–73 was pended. It meant that only the validity of an at-
a continuance of the old newsprint policy which tack based on Articles 14 and 19 was suspended

CLAT.indb 151 31/03/2009 11:41:52


152 CONSTITUTIONAL LAW

during the Emergency. But once, this embargo of India, any such order is not to extend to any
was lifted, Articles 14 and 19, whose use was other part of Indian territory unless the Presi-
suspended, would strike down any legislation dent, being satisfied that the security of India or
which would have been bad. any part thereof is threatened by activities in or
However, the expression “the things done in relation to the part of the territory of India
or omitted to be done”, occurring in Article 358, in which the Proclamation of Emergency is in
the Court held, did not mean that the right con- operation, considers such extension to be neces-
ferred under the settlement was washed off com- sary.
pletely. The expression is to be interpreted very Clause (3) of Article 359 requires that ev-
narrowly. Therefore, as soon as the emergency ery Order made by the President under Clause
was over, the Court laid down, the rights would (1) of Article 359, shall as soon as, be laid be-
revive and what could not be demanded during fore each House of Parliament.
the emergency, would become payable even for The Constitution (44th Amendment) Act,
the period of emergency of which payment was 1978 has amended Article 359 to curtail the
suspended. Otherwise, the enactment will have vast power vested in the Executive under this
effect even after the Emergency had ceased. Article. This Amendment has made the follow-
ing significant changes:
Suspension of Enforcement of Other Firstly, to check the misuse of the Emer-
Fundamental Rights (Article 359) gency provisions and to put the right to life and
Prior to the 44th Amendment, 1978, Article 359 liberty on a secure footing, amended Article 359
(1) empowered the President to suspend, by provides that the Presidential power to suspend
issuing order, the operation of all fundamental the right to move the Court for the enforcement
rights other than those contained in Article of a fundamental right cannot be exercised in
19, during the operation of a Proclamation of respect to the fundamental rights secured by Ar-
Emergency. All proceedings pending in any ticles 20 and 21.
court for enforcement of the rights so mentioned Thus, the operation of Articles 20 and 21
in the Presidential Order, were to remain cannot be suspended during the operation of a
suspended. The rights so mentioned and pending Proclamation of Emergency.
proceedings relating to such rights might be Secondly, Article 359 will not protect any
suspended for the duration of the Proclamation law, violative of any fundamental right, the op-
or for shorter period as might be specified in the eration of which is suspended under the Presi-
Presidential Order. dential order made under Article 359(1), which
No fundamental rights, mentioned in the does not contain a recital to the effect that such
said Order, could restrict the power of the State law is in relation to the Proclamation of Emer-
to make any law or to take any executive action, gency in operation when it is made.
which the State would be competent to make or Thirdly, Article 359 will also not protect
take, but for the fundamental rights in question. any executive action taken otherwise than under
Provison to Article 359 (1-A) further pro- a law containing such a recital.
vides that if the Proclamation of Emergency is in
operation only in a part of the territory of India, Distinction between Article 358 and Article
the State may make such a law or take any such 359
executive action in relation to or in any part of A comparative examination of the respective
India in which the Proclamation of Emergency provisions contained in Articles 358 and 359,
is not in operation, if and in so far as the security brings out the following points of distinction-
of India or any part of its territory, is threatened Firstly, while Article 19 gets suspended
by activities in the part of the territory of India automatically under Article 358 as soon as the
in which the Proclamation of Emergency is in Proclamation of Emergency is issued under Ar-
operation. ticle 352, Article 359 does not purport expressly
Clause (2) of Article 359 provides that to suspend any of the fundamental rights. Article
the Presidential Order made under Article 359 359, on the other hand, authorises the President
(1) (suspending the operation of fundamental to issue all order for the suspension of the en-
rights) may be extended to the whole or any part forcement of the fundamental rights specified in
of the territory of India. such order.
However, if the Proclamation of Emergen- Secondly, while Article 358 suspends Ar-
cy is in operation only in a part of the territory ticle 19 itself, what can be suspended under

CLAT.indb 152 31/03/2009 11:41:52


CONSTITUTIONAL LAW 153

Article 359 is the enforcement of unspecified Proclamation issued under Article 356 shall be
rights and not the rights themselves. laid before each House of Parliament. It is with
Thirdly, while under Article 358, Article 19 a view to afford an opportunity to the Parlia-
remains suspended for the whole of the duration ment to approve the action taken by the Execu-
of Emergency, under Article 359 the President tive in regard to the situation prevailing in the
may declare the suspension of the enforcement concerned State.
of other fundamental rights either for the whole Clause (3) further provides that the Procla-
of the duration of Emergency or for a shorter mation so laid before the Houses of the Parlia-
period. ment, shall cease to have effect at the expiration
Fourthly, the suspension of Article 19 under of 2 months unless before the expiration of these
Article 358, applies to the whole of the country 2 months, it has been approved by resolutions of
and so, covers all Legislatures and all States, the both Houses of Parliament.
Presidential Order issued under Article 359, on Thus, without the approval by the Houses
the other hand, may extend to the whole of India of Parliament, a Proclamation relating to State
or may be confined to a part of India. Emergency can continue in operation only for
Fifthly, while Article 359 operates during 2 months from the date it is made by the Presi-
any Proclamation of Emergency made under dent.
Article 352 on any ground, Article 358 operates Proviso to Clause (3) of Article 356, how-
only in case the Proclamation of Emergency is ever, lays down that if such Proclamation (not
issued under Article 352 on the ground either of
being a Proclamation revoking a previous Proc-
‘war’ or ‘external aggression’.
lamation) is issued at a time when the House of
Sixthly, while Article 358 is confined to Ar-
People (Lok Sabha) is dissolved or the dissolu-
ticle 19, Article 359 extends to all fundamental
tion of the House of People takes place during
rights except those contained in Articles 20 and
the said period of 2 months, and if a resolution
21.
approving the Proclamation has been passed by
State Emergency (Articles 355 to 357) the Council of States but no resolution with re-
Failure of Constitutional Machinery in States— spect to such Proclamation has been passed by
President’s Rule in States (Article 356) the House of People before the expiration of that
Article 356 provided: “If the President, period of 2 months, the Proclamation shall cease
either on receipt of a report from the Governor to operate at the expiration of 30 days from the
of a State or otherwise, is satisfied that a situa- date on which the House of People first sits after
tion has arisen in which the government of the its reconstitution.
State cannot be carried on in accordance with However, if before the expiration of the
the provisions of this Constitution, he may make said period of 30 days, a resolution approving
a Proclamation to that effect.” the Proclamation has been passed by the House
of People, the Proclamation so approved shall,
Grounds for Proclamation under Article 356. unless revoked earlier, cease to operate on the
The only ground on which a Proclamation can expiration of 6 months, from the date of the is-
be made under Article 356 is “the failure of the
sue of the Proclamation.
Constitutional Government in the State”.
Prior to the 44th Amendment, 1978, the
Non-observance of the provisions of Ar-
Proclamation so approved could continue in op-
ticle 174(1), in case of pre-mature dissolution of
eration for one year from the date of the passing
Legislative Assembly of the State cannot be a
of the second of the resolutions approving the
ground for the invocation of Article 356(1) in
Proclamation.
that State.
Originally, the Proclamation could contin-
The Apex Court in special reference, ruled
that framing of election schedule for the new ue in operation for 6 months .from the passing
Legislative Assembly was the function of the of the second resolution approving the Proc-
Election Commission but the infraction of Ar- lamation. The words “6 months” were substi-
ticle 174 (1), could not be remedied by resort to tuted by “one year” by the Constitution (42nd
article 356 (1) by the President. Amendment) Act, 1976, which is no more appli-
cable after the passing of the Constitution 44th
Duration of Proclamation Issued Under Amendment) Act, 1978.
Article 356 If the Proclamation is required to be con-
Clause (3) of Article 356 requires that every tinued in operation beyond the six months for

CLAT.indb 153 31/03/2009 11:41:52


154 CONSTITUTIONAL LAW

which it is approved by the Parliament, then Article 352 (1)] is in operation, in the whole of
the continuance o[ the Proclamation is to be ap- India or, as the case may be, in the whole or any
proved by each House of Parliament. part of the State, at the time of the passing of
Proviso to Clause (4) of Article 356 says such resolution for the continuance of the Proc-
that if and so often as a resolution approving the lamation issued under Article 356.
continuance in force of such a Proclamation is (1) beyond one year, and
passed by both Houses of Parliament, the Proc- (2) the Election Commission certifies that the
lamation shall, unless revoked earlier, continue continuance in force of the Proclamation
in force for a further period of 6 months from the approved under Clause (3) of Article 356,
date on which it would otherwise have ceased during the period specified in such resolu-
to operate. tion (of the Houses of Parliament} is nec-
Thus, the duration of the Proclamation can essary on account of difficulties in holding
be extended by six months each time by both general elections to the Legislative As-
Houses of Parliament passing resolutions ap- sembly of the State concerned.
proving its continuance. In case, the House of It has, however, been held that law and or-
People is dissolved within any such period of der situation prevailing in the State, is no ground
6 months, without approving the continuancc to defer fresh election.
of the Proclamation for further period of six The 44th Amendment, 1978 as inserted
months, the Proclamation shall cease to oper- these two conditions with a view to ensure that
ate on the expiration of 30 days from the date democratic rule is restored in the State con-
on which the House of People first sits after its cerned after the minimum period which will be
reconstitution. necessary for holding elections.
If during these 30 days, the House of People
Procedure for Approval of Proclamation [Ar-
passes a resolution approving the continuance of
ticle 356 (3)]
the Proclamation, it shall continue in operation
Article 356(3) provides that a Proclamation is-
until the expiration of six months from the date
sued by the President under Article 356 (1), as
on which it would otherwise have ceased to op-
well as the continuance of the Proclamation so
erate. Provided, before the expiration of such
issued for every six months, is required to be ap-
period of six months the Rajya Sabha must have
proved in each House of Parliament by a resolu-
passed a resolution approving the continuance
tion passed by the simple majority of members
of the Proclamation for that further period of six
present and voting.
months.
The maximum period for which a Procla- Consequences of Issuance of Proclama-
mation can remain in operation three years from tion Under Article 356(1)
the date it is issued under Clause (1) of Article The following consequences ensue on the issu-
356. ance of a Proclamation by the President under
Continuance of a Proclamation Issued Article 356 (l):
Under Article 356 (l)—Beyond One Year 1. The President may assume to himself, all
or any of the functions of the Government
The Constitution (44th Amendment) Act, 1978 of the State and all or any of the powers
has incorporated two conditions precedent for vested in or exercisable by the Governor
the continuance of a Proclamation under Article or any body or authority in the State other
356 (1) (State Emergency) if it is required to be than the Legislature of the State.
continued in operation beyond one year from 2. The President may declare that the pow-
the date of its issue. ers of the Legislature of the State shall be
Clause (5), inserted by the 44th Amend- exercisable by or under the authority of
ment, 1978 (w.e.f. 20 June 1979) provides that Parliament.
a resolution with respect to the continuance in 3. The President may make such incidental
force of a Proclamation, approved by the Hous- and consequential provisions as appear to
es of Parliament under Clause (3), for any pe- him to be necessary or desirable for giving
riod beyond the expiration of one year from the effect to the objects of the Proclamation.
date of issue of such Proclamation shall not be These may include provisions for sus-
passed by either House of Parliament unless: pending in whole or in part the operation
(a) a Proclamation of Emergency [issued under of any provisions of the Constitution relat-

CLAT.indb 154 31/03/2009 11:41:52


CONSTITUTIONAL LAW 155

ing to any body or authority in the States. bodies whose powers can be taken over by the
The President, however cannot assume President under Article 356 (1) (a).
to himself any of the powers vested in or Where the President after making a Proc-
exercisable by the High Court, nor can he lamation under Article 356 (J) declares that the
suspend in whole or in part the operation of powers of the State Legislature are to be exer-
any provision of the Constitution relating cised by the Parliament and the Parliament un-
to the High Courts. der Article 357 (1) (a) not only can confer on the
4. Article 357 (1) provides that where the President, the powers of the State Legislature to
President has declared under Article 356 make laws, but, it can even authorise the Presi-
(1) (b) that the powers of the Legislature dent to delegate the powers so conferred to any
of the State shall be exercisable by or authority to be specified by him in that behalf.
under the authority of Parliament then: Under such circumstances, the Ordinances
Parliament may: issued by the President or his delegate under
(a) confer on the President the power of Article 357 (1) (a) partake fully of legislative
the Legislature of the State to make character and are made in the exercise of leg-
laws, and islative power within the contemplation of the
(b) authorize the President to delegate Constitution.
the power conferred on him by the By virtue of Article 357 (1) (a) the power to
Parliament, on any other authority enact State laws can be conferred on the Presi-
to be specified by the President in dent by the Parliament. There is nothing in this
that behalf. This delegation shall Article which could be read to curtail the author-
be subject to such conditions as the ity of Parliament to confer powers to enact laws
President may think fit to impose. of the State Legislature only when Parliament is
(c) or the President or other authority, in not in Session.
whom power to make: laws is vested
Guidelines for the Invocation of Article
(by virtue of Articles 356 (1) and 357
356
may make laws (i) for conferring
powers and imposing duties, or (ii) for In S. R. Bommai vs Union of India, the Su-
authorising the conferring of powers preme Court has laid down a number of impor-
and imposing duties, upon the Union tant guidelines in regard to the exercise of power
or Officers and authorities thereof. under Article 356. These guidelines reflect the
The President may, when the House of Peo- majority view and with which the majority con-
ple is not in Session, authorize expenditure from curred. These may be discussed as follows:
the Consolidated Fund of the State pending the 1. Article 356 confers extraordinary power
sanction of such expenditure by Parliament. on the President. This power should be
Article 357 (2) further provides that any exercised sparingly and with great cir-
law made by the Parliament or by the Presi- cumspection. That resort to Article 356
dent or by other authority, in the exercise of the (1) should be only as a last measure, which
power of the Legislature of the State, conferred may be taken to restore constitutional ma-
under Article 357 (1) (a), shall continue in force, chinery into the State.
even after the Proclamation made under Article In its support, the Court referred to the ob-
356 (1) has ceased to operate, until altered or servations made by Dr B. R. Ambedkar on
repealed or amended by a competent Legislature Article 356 that “the proper thing we ought
or other authority. to expect is that such Article will never be
Article 356 (1) (a) imposes a bar against called into operation and they will remain
the assumption by the President of the legisla- a dead letter”. Dr Ambedkar had hoped
tive powers of the State Legislature, which can that the emergency provisions would be
only be transferred to Parliament. Yet, the pro- invoked in rarest of rare cases. The Court
visions of Article 356 (J) (a) read with Article also endorsed the recommendations of the
357 do not operate as an absolute bar on any Sarkaria Commission in regard to the use
expenditure which can be legally incurred by of Article 356. The Commission inter alia
the President or under the Presidential author- recommended that before invoking Ar-
ity in accordance with pre-existing State Laws ticle 356 (1) a warning in specific terms
authorising expenditure by other authorities or should be given to the erring State.

CLAT.indb 155 31/03/2009 11:41:52


156 CONSTITUTIONAL LAW

All alternatives should be exhausted to ate case and if the situation demands, the
contain the situation and all attempts to re- Court may also stay the dissolution of the
solve the crisis at the State level should be Legislative Assembly but not in such a
made. Such alternatives may be dispensed manner as to allow the Assembly to con-
with only in case of extreme urgency. tinue beyond its original term.
The report of the Governor under Article 5. Clause (3) of Article 356 is conceived as a
356 (1) should be a speaking document control on the power of the President and
and the material facts and grounds on also as a safeguard against abuse. Thus, if
which Article 356 was to be invoked must the two Houses of Parliament do not ap-
be made, an integral part of the Proclama- prove the issuance of Proclamation, both
tion issued under Article 356 (1) for the the State Government and the State As-
purpose of judicial review. sembly can be revived.
2. Though Article 356 does not expressly 6. If the Proclamation issued is held invalid
speak of the dissolution of the Legisla- by the Court then, notwithstanding the
tive Assembly of the State, however, such fact that it is approved by both Houses of
power’ is implicit in sub-clause (a) of Parliament, it will be open to the Court to
Article 356 (1). Since Article 174 (2) (b) restore the status quo ante to the issuance
empowers the Governor to dissolve the of the Proclamation and hence to restore
Legislative Assembly and that the Presi- the Legislative Assembly and the minis-
dent under Article 356 (1) (a) assumes to try.
himself the powers and functions of both The Court, it was held, in appropriate cas-
of the government and the Governor, he es will not only be justified in preventing
may dissolve the Legislative Assembly as holding of fresh elections but would be
part of the Proclamation issued under Ar- duty bound to do so by granting suitable
ticle 356 (1) or by a subsequent order. interim relief to make effective the consti-
3. As regards the stage at which, the power to tutional remedy of judicial review and to
dissolve the Legislative Assembly should prevent the emasculation of the Constitu-
be exercised. The Court said that Clause tion.
(3) of Article 356 requires the Proclama- 7. Secularism is a part of the basic structure
tion to be laid before both Houses of Par- of the Constitution. Hence, the acts of a
liament. It would therefore be consistent State Government which were calculated
with the scheme and spirit of the Consti- to subvert or sabotage secularism as en-
tution-particularly in the absence of a spe- shrined in the Constitution, the Court held,
cific provision in the Constitution express- could lawfully be deemed to give rise to
ly empowering the President to do so-to a situation in which the Government of
hold that “the power of dissolution can the State could not be carried on in accor-
be exercised by the President only after dance with the provisions of the Constitu-
both Houses of Parliament have approved tion. Reference was made to Section 123
the Proclamation and not before such ap- of the Representation of People Act, 1951,
proval. The President may, however have which prohibits the commission of such
the power to suspend the Legislative As- Acts.
sembly under Article 356 (1) (c), before 8. In all cases where the Ministry loses ma-
the approval of the Proclamation by the jority support or where the support to the
Parliament. Ministry is claimed to have been with-
The Sarkaria Commission has also recom- drawn by some legislators, it was held that
mended for the amendment of the Consti- the proper course for testing the strength
tution in order to incorporate a provision of the Ministry was holding the test on the
to this effect. floor of the House, except in cases where
4. A High Court or the Supreme Court shall the holding of the floor test was considered
have the jurisdiction to entertain a writ not possible for reasons given in writing.
petition questioning the Proclamation if 9. Article 74(2) bars an enquiry into the
it is satisfied that the writ petition raises question as to whether any, and if yes,
arguable question with respect to the va- what advice is tendered by the Council of
lidity of the Proclamation. In an appropri- Ministers to the President. It was held that

CLAT.indb 156 31/03/2009 11:41:52


CONSTITUTIONAL LAW 157

Article 74 (2) did not bar the Court from sits after its reconstitution. However, the Coun-
calling upon the Union Council of Min- cil of States (Rajya Sabha) must have passed a
isters to disclose to the court the material resolution approving the Proclamation within
upon which the President had formed the the said period of two months.
requisite satisfaction If the House of People passes a resolution
10. As to the question of judicial review, it approving the Proclamation before the expi-
was held that there was no dispute that ration of 30 days from the date on which the
the Proclamation issued under Article 356 House of People first sits after its reconstitu-
(1) was subject to review by the Supreme tion and a resolution approving the Proclama-
Court or the High Court. The examination tion has already been passed by the Council of
by the Court would necessarily involve States within the initial period of two months,
the scrutiny as to whether there existed the Proclamation is said to be approved by both
material for the satisfaction of the Presi- Houses of Parliament.
dent that a situation had arisen in which Thus, with the approval by both Houses of
the Government of the State could not be Parliament, a Proclamation of Financial Emer-
carried on in accordance with the provi- gency shall continue in operation indefinitely
for all times, until it is revoked by the President
sions of the Constitution. The material in
by making another Proclamation.
question has to be such as would induce a
reasonable man to come to the conclusion Effects of Financial Emergency
in question. The object of issuing a Proclamation of Finan-
cial Emergency is to achieve financial stability.
Financial Emergency (Article 360)
A Proclamation so issued has the following ef-
Article 360 contains provisions concerning fi- fects:
nancial emergency. Article 360 (1) provides: (a) During the period, a Proclamation of Fi-
“If the President is satisfied that a situation has nancial Emergency, is in operation the ex-
arisen whereby the financial stability or credit ecutive authority of the Union extends:
of India or any part of its territory is threatened, (i) to the giving of directions to the States
he may by a Proclamation make a declaration to to observe such canons of financial
that effect.” propriety as may be specified in the
The ground for the declaration of financial directions, and
emergency is “a threat to the financial stability (ii) to the giving of such other directions
or credit of India or any part of the territory of as the President may deem necessary
India.” and adequate for the purpose,
(iii) to the giving of directions requiring
Duration of Financial Emergency the reduction of salaries and allow-
Clause (2) of Article 360 provides that the ances of all or any class or persons
Proclamation issued by the President, may be serving in connection with the affairs
revoked or varied by’ a subsequent Proclama- of the States,
tion made by him. The Proclamation concerning (iv) to the giving of directions requir-
financial emergency is required to be laid before ing all, Money Bills or other Bills to
each House of Parliament. The Proclamation which the Provisions of Article 207
so made ceases to operate at the expiration of apply (i.e., Financial Bill or Bills in-
two months unless before the expiration of these volving expenditure from the Con-
two months, it has been approved by resolutions solidated fund of the States) to be
passed by both Houses of Parliament. On such reserved for the consideration of the
approval, it shall continue to operate until re- President after they are passed by the
voked by the President by making a fresh Proc- Legislatures of the States.
lamation under Article 360 (2) (a). (b) During the period a Proclamation of Fi-
If a Proclamation of Financial Emergency nancial Emergency is in operation, it shall
is issued at a time when the House of People be competent for the President to issue di-
(Lok Sabha) has been dissolved or the dissolu- rections for the reduction of salaries and
tion of the House of People takes place during allowances of all or any class of persons
the said period of two months, the Proclamation serving in connection with the affairs of
shall cease to operate at the expiration of 30 days the Union including the Judges of the Su-
from the date on which the House of People first preme Court and the High Court.

CLAT.indb 157 31/03/2009 11:41:52


158 CONSTITUTIONAL LAW

The Constitution (38th Amendment) Act, Article 77 lays down that all executive action
1975, inserted Clause (5) in Article 360 to the of the Union of India, shall be expressed to be
effect of declaring the satisfaction of the Presi- taken in the name of the President and the orders
dent, in respect to the situation mentioned in and other instruments made or expressed in his
Clause (1) of Article 360, as “final and conclu- name, are to be authenticated in such a manner,
sive” and not questionable in any Court on any as may be specified in the rules framed by the
ground. The Constitution (44th Amendment) President.
Act, 1978 has omitted Clause (5). As yet, no oc- Thus, Article 77 gives effect to the pro-
casion has arisen for the issuance of a Proclama- visions of Article 53 which provides that the
tion under Article 360 (1). President may act through his officers. “Rules
Position of the President of Business’ so framed under Article 77, practi-
cally transfer the whole decision-making power
The Constitution confers very vast powers on the from the President to other authorities.
President in relation to the Executive, Legisla- Article 74 (1), as it stood before the Con-
ture, the Armed forces, Ordinance making, etc., stitution (Forty-second amendment) Act, 1976,
discussed above. Though, formally vested in the provided that the Council of Ministers was to
President, the intention was not that he should aid and advise the President in the exercise of
exercise all his powers personally. It would have his functions. It could mean to make the Council
been a task physically impossible and constitu-
of Ministers merely an advisory body. However,
tionally undesirable. All the powers expressly
the phrase “aid and advise” used Article 74 (1),
vested, have to be interpreted in the light of the
has been said to be a masterly understatement
Parliamentary System of Government, adopted
and in actual practice, the decisions are made
under the Constitution. The essential character-
and implemented by the Ministers themselves.
istic of such a system is to make the Head of the
In day to day administration, the President
State, a mere formal head, and all his powers
has not much a role to play.
are to be exercised by Officers subordinate to
The matter was discussed in the Constitu-
the President. For this purpose, the Ministers are
ent Assembly and it was made clear beyond
deemed to be Officers subordinate to him. The
doubts that the Assembly ‘had adopted the Brit-
Constitution thus contains provisions by which
ish Cabinet System of the Government. There-
the responsibility for decision making may be
fore, the relevant conventions which were oper-
passed from the President to others.
ating in England were to be relevant under the
Constitutional Provisions Indian Constitution and that the formal provi-
Article 53 (1), which vests the executive power sions contained therein, should be read in the
of the Union in the ‘President, provides that the light of those conventions. In England, it has
power may be exercised by the President either been described that the term ‘Crown’ represents,
directly or through Officers subordinate to him: the sum total of governmental powers and is
It thus provides for delegation of the executive synonymous with the executive. Accordingly,
power by the President to others. the term “President” used in various provisions
Article 77(3) provides that ‘the Presi- of the Constitution of India, denotes the Central
dent shall make· rules for the more convement Executive.
transaction of the business of the Government Article 74 (1) incorporates the idea of mak-
of India and for allocation of work among the ing the President a formal agency and though,
Ministers. Minister would superficially seem to be an advi-
Article 53 (3) (b) empowers the Parlia- sory body, by convention, its advice is binding
ment, by law, to confer any function on authori- on the President.
ties other than the President and the officer, on Krishna Ayyar, a member of the Draft-
which the power is so conferred, can act in his ing Committee of the Constituent Assembly,
own name. observed that the word “President” used in the
Article 53 (1) does not require that he Constitution “merely stands for the fabric re-
should personally sign all the executive and ad- sponsible to the Legislature”. What he meant by
ministrative orders. In actual practice, he signs the term ‘President’ was the Union Council of
only few crucial orders, and other orders are Ministers which was declared to be collectively
issued in his name by the subordinate officers responsible to the House of the People. Dr B. R.
even without reference to him. For this purpose, Ambedkar, the Chairman of the Drafting Com-

CLAT.indb 158 31/03/2009 11:41:52


CONSTITUTIONAL LAW 159

mittee of the Constituent Assembly, explained These constitutional provisions require that
the position of the President in the following there should be in office a Council of Ministers
words: which is in a position to secure Parliamentary
The President is merely a nominal figure approval, sanction and finance for its policies
head. He has no discretion and no powers of and programmes. It is, therefore, absolutely, es-
administration at all. He occupies the same po- sential for the President, to maintain in office, a
sition as the King under the English Constitu- Council of Ministers enjoying the confidence of
tion. He is the Head of the State, but not of the the Houses of Parliament.
Executive. He represents the nation but does not The power to impeach the President is also
rule the nation. His place in the administration vested in the Houses of Parliament. In case, the
is that of a ceremonial device on a seal by which President acts’ in violation of the Constitution,
the nation’s decisions are made known. The he may be impeached by the majority party in
President of India will be generally bound by power. In this respect, violation of a Convention
the advice of the Ministers. He can do nothing would be deemed to be a violation of the Consti-
contrary to their advice nor can he do anything tution. The role of the President in regard to giv-
without their advice. ing assent to the Bills has also been explained
Mr Nehru, the first Prime Minister of the under the Constitution. All these provisions ex-
country, observed: plain that an activist President is going to force,
“We want to emphasise the ministerial sooner or later, a showdown with the Cabinet.
character of the Government and that power Indian President in Working Mode—
really resided in the ministry and in the Leg- Ideals and Realities
islature and not in the President. At the same
time, we did not want to make the President just Many a times, controversies had erupted in re-
a mere figurehead ... We did not give him any gard to the position of the President and each
real power but we have made his position one time it had ended in confirming the position that
of great authority and dignity he is also to be the the President is merely a constitutional Head,
Commander-in-Chief of the Defence Forces.” having no effective powers.
In 1951, the President, Dr Rajendra Pras-
Safeguards Against an Activist ad, expressed his desire to act in his individual
President judgment, in regard to giving assent to two Bills
The role of the President as a figurehead is re- sent to him for his assent. It made Mr Nehru, the
flected in his indirect election. It may also be Prime Minister, worried. He consulted the then
noted that the Constitution, nowhere uses the Attorney-General Mr M. C. Setalvad and Sir Al-
terms like “discretion” and individual judgment ladi K. Ayyer. Both these authorities expressed
for the President which were used for the Gov- the view that the President had no discretion
ernor-General under the Government of India under Article 111 and that it would be constitu-
Act, 1935. tionally improper on his part not to seek and be
The question arose in the Constituent as- guided by the advice of the Council of Minis-
sembly to make it clear that the President would ters. Dr Prasad did not precipitate the matter and
be bound” by ministerial advice. But, it was acted in accordance with the Council’s advice.
thought better to leave the matter for Conven- Again, in 1960, the controversy started
tions. with the remarks made by Dr Prasad in regard
However, the matter has not been left to the President’s position. While laying down
entirely on Conventions. There are effective the foundation stone of the Indian Law Institute
safeguards contained in the Constitution itself, Building, New Delhi, on November 28, 1960,
against an activist President. These are: Dr Prasad in his speech said that,
(i) The responsibility of the Council of Min- “It is generally believed that like the sov-
isters to the House of People. ereign of Great Britain, the President of India is
(ii) Parliament’s supreme power of legisla- also a constitutional head and has to act accord-
tion, taxation and appropriation of funds. ing to the advice of his Council of Ministers.
(iii) Subjection of the exercise of Ordinance There are, however, in the Constitution many
making power and also proclamation of provisions which lay down specific duties and
Emergency to the approval of the Parlia- functions of the President. The question which
ment. needs to be studied and investigated is the extent

CLAT.indb 159 31/03/2009 11:41:52


160 CONSTITUTIONAL LAW

to which, and the matters in respect of which, if being the Chief Justice of the Supreme Court
any, the powers and functions of the President and the second time being the Vice-President of
differ from those of the British Sovereign”. India in October 1982.
The speech made by the President led to In the History of Presidential elections,
speculation regarding the position of the Presi- V. V. Giri is the only person who won the elec-
dent. However, the controversy was laid to rest tion of the President as an independent candi-
by Mr Nehru in a Press Conference addressed date in 1969.
on 15 December 1960, where he declared that In July 1977, Neelam Sanjeeva Reddy was
the President’s remarks were merely casual and elected unopposed as no one else filed nomina-
that politically and constitutionally his position tion for the post of the President.
was that of a constitutional head and that he had Any Bill passed by the Parliament can be-
always acted as such. come a law only after it has been assented by
In 1967, as a result of the fourth general the President.
elections held in May 1967, the Congress mo- The example of the use of Pocket Veto is
nopoly in the States was broken down. In a num- the Postal Bill of 1986, in which the then Presi-
ber of States, non-Congress Governments came dent Gyani Zail Singh withheld the bill and sub-
into power and in the Centre also, the Congress sequently the Bill was withdrawn in 1989.
was faced with effective opposition. Soon after (ix) According to Article 143, the President
can seek legal advice from the Supreme Court
the general elections, the election of the Presi-
on matters of Public importance, but the Presi-
dent was to be held. For the first time, opposi-
dent is not bound by such advice.
tion parties got united to set up their common
(x) So far no President has been impeached
candidate against the Congress candidate in that
for the violation of the Constitution.
election. They, while soliciting support for their
candidate, expressed that the President was not
merely a figurehead, but he had a constructive
THE VICE-PRESIDENT OF INDIA
and meaningful role to play in the affairs of the
(Article 63)
country, specially that he should act as sort of Article 63 provides that there shall be a Vice-
mediator between the Centre and the States. On President of India. The Vice-President is declared
the other hand, the Congress manifesto was that to be the ex officio Chairman of the Rajya Sabha.
the President was merely a constitutional figure- In the event of the occurrence of any vacancy
head. Controversy could be said to have been in the office of the President, by reason of his
laid at rest with the election of the Congress can- death, resignation or removal, or otherwise,
didate as the President. the Vice-President shall act as the President
Important Things to Remember until the date on which the new President,
elected in accordance with the provisions of the
There is no limit as to how many times a person Constitution to fill such vacancy, enters upon
can become the President. The President gets a his office. Again, when the President is unable
monthly salary of Rs 1,50,000. to discharge his functions owing to absence,
Dr Rajendra Prasad was the first and lon- illness or any other cause, the Vice-President
gest serving President of India. shall discharge his functions until the date on
In any case, if both the President and the which the President resumes his duties.
Vice-President are not a available to perform the
duties of the President, the Chief Justice of Su- Qualification for the Office of Vice-
preme court discharges the duties of the Presi- President [Article 66(3) and (4)]
dent and in his absence the next senior-most Clause (3) of Article 66 lays down that a person
Judge of the Supreme Court performs the func- to be eligible for the office of the Vice-President,
tions of the President. must possess the following qualifications:
Only once in the History of India, Justice (a) He must be a citizen of India.
M. Hidayatullah, Chief Justice of Supreme (b) He must have completed the age of thirty-
Court discharged the duties of the President five years.
from 20 July 1969 to 20 August 1969. (c) He must be qualified for election as a
Justice M. Hidyatullah is the only person to member of the Rajya Sabha.
perform the functions of the President two times It requires that he must be registered as a
in two different capacities, the first time in 1969 voter in any Parliamentary Constituency.

CLAT.indb 160 31/03/2009 11:41:52


CONSTITUTIONAL LAW 161

(d) He must not hold any office of profit un- Oath of Office (Article 69)
der the Government of India or the Gov- Article 69 says that the Vice-President, before
ernment of any State or under any local entering upon his office, shall make and sub-
or other authority subject to the control of scribe before the President or some person ap-
any of the said Governments. pointed in that behalf by him, an oath or affirma-
Election of the Vice-President [Article tion in the following form, that is to say:
66(1)] “I, A. B., do swear in the name of God sol-
emnly affirm that I will bear true faith and al-
The Vice-President is elected by the members of
legiance to the Constitution of India as by Jaw
an Electoral College consisting of the members
established and that I will faithfully discharge
of both Houses of Parliament in accordance with
the duty upon which I am about to enter”.
the system of proportional representation by
means of single transferable vote and the voting Conditions of Office
at such election is done by secret ballot. The [Articles 66(2), 64 and 65 (3)]
Presidential and Vice-Presidential Elections Act, 1. The Vice-President shall not be a mem-
1997, provides that there should be 20 proposers ber of either House of Parliament or of
and 20 seconders to support the candidature of a a House of the Legislature of any State,
person in the election of the Vice-President. and if a member of either of such House is
Time for Holding Election (Article 68) elected as the Vice-President then, he shall
be deemed to have vacated his seat in that
Clause (1) Article 68 provides that an election
House on the date on which he enters upon
to fill a vacancy caused by the expiration of the
his office as Vice-President.
term of Vice-President shall be completed before
2. The Vice-President shall be ex officio
the expiration of the term of the earlier Vice-
Chairman of the Rajya Sabha and shall
President. While an election to fill a vacancy
not hold any other office of profit. Dur-
occurring by reason of his death, resignation or
ing any period when the Vice-President
removal or otherwise, shall be held as soon as
acts as President or discharges the func-
possible after the-occurrence of the vacancy.
tions of the President under Article 65, he
Term of Office of the Vice-President, shall not perform the duties of the office
Resignation (Article 67) of Chairman of the Rajya Sabha. During
Article 67 lays down that the Vice-President such period, he shall have all the powers
holds office for a term of five years from the date and immunities of the President and be
on which he enters upon his office. He shall con- entitled to such emoluments, allowances
tinue in his office even after the expiration of his and privileges as may be determined by
term of five years until his successor enters upon Parliament by law and, until provision in
his office. The Vice-President may, by writing that behalf is so made, such emoluments,
under his hand, addressed to the President, re- allowances and privileges as are specified
sign his office. in the Second Schedule to the Constitu-
tion.
Removal of Vice-President It may be noticed that the Constitution does
[Article 67 (b)] not provide the salary, allowances or the privi-
The Vice-President may be removed from his leges to· which the Vice-President is entitled to
office by a resolution of the Rajya Sabha passed as Vice-President. He, therefore, does not receive
by a majority of all the members of the Rajya any salary or allowance as Vice-President.
Sabha and agreed to by the Lok Sabha. How- When he is acting as President or discharg-
ever, no resolution for the removal of theVice- ing his functions, the Vice-President is entitled
President shall be moved in the Rajya Sabha un- to such salary and allowances as are payable to
less at least fourteen days’ notice has been given the President.
of the intention to move the resolution. And, when he is not so acting, he shall be
It may be noticed that the Constitution ex-officio Chairman of the Rajya Sabha and
does not prescribe any ground on which a reso- shall be entitled to such salary and allowances
lution for the removal of Vice-President can be as are determined under Article 97 for the Chair-
moved. man of the Rajya Sabha.

CLAT.indb 161 31/03/2009 11:41:52


162 CONSTITUTIONAL LAW

COMPARISON OF THE OFFICES OF THE PRESIDENT AND


THE VICE-PRESIDENT
President Vice-President
Election
The President is elected by an Electoral The Electoral College is limited to members
College consisting of the elected Members of both Houses of Parliament. Members of
of both Houses of Parliament and Legislative State assemblies do not participate.
Assemblies of the States.
Election in both the cases is by secret ballot and in accordance with the system of proportional
representation by single transferable vote.
Qualifications
a. Citizen of India a. Citizen of India
b. Completed the age of 35 years b. Completed the age of 35 years
c. Qualified for election to Lok Sabha c. Qualified for election to Rajya Sabha
Both must not hold any office of Profit.
Term of Office
5 years from the date of entering office 5 years from the date of entering office.
Resignation
May resign office by writing May resign office by writing
addressed to the Vice-President. Addressed to the President.
Removal
May be removed by impeachment. No impeachment but may be removed by
resolution passed by a majority of members
of the Rajya Sabha and agreed to by the
Lok Sabha.
Re-election
Eligible for re-election any number Eligible for re-election any number of times.
of times.
Functions
Numerous functions under the The only function is acting as a Chairman of
Constitution. of the Council of States. When the office of the
President is vacant he acts as the President or
discharges the functions of the President.

After scanning the provisions relating to to aid and advise the President who shall, in the
the office of the Vice-President, it nay be noticed exercise of his functions, act in accordance with
that not much importance is given to this office. such advice”.
The office is created for the purpose of provid-
ing for an event, when there is a vacancy in the Appointment of the Prime Minister
office of the President or when the President is [Article 75 (1)]
unable to discharge his functions owing to ab- The Prime Minister is the head of the Union
sence, illness or any other cause. The normal Council of Ministers. He is appointed by the
function the Vice-President on the other hand, is President. This power of the President must be
to preside over the Rajya Sabha. read in the light of the requirement contained in
Clause (3) of Article 75 which provides: “The
THE PRIME MINISTER OF INDIA Council of Ministers shall be collectively re-
(Articles 74 and 75) sponsible to the House of the People”.
The Constitution, thus, incorporates the
Article 74(1) provides: “There shall be a Council principle of “Responsible Government” accord-
of Ministers with the Prime Minister at the head ing to which, the executive is made answerable,

CLAT.indb 162 31/03/2009 11:41:52


CONSTITUTIONAL LAW 163

for every act it does in relation to the administra- In such situations also the President’s
tion of the affairs of the country, to the popular discretion is guided by certain Conventions. A
House of the Legislature. Convention operating in England is followed
In this regard, the framers of the Constitu- when the ruling party is defeated in the lower
tion of India have adopted the British Cabinet House or when the Prime Minister has submit-
System. Therefore, the conventions operating ted resignation of his Government. According to
under the English Constitution, in regard to the this Convention, the President should invite the
relationship between the Crown and the Minis- leader of the opposition to explore the possibility
ters, are relevant in the interpretation of the In- of forming a stable ministry. It was done by the
dian Constitution as well. President, Shri N. Sanjeeva Reddy, by inviting
One of the well established conventions op- Shri Y. B. Chavan, the leader of the opposition,
erating in England is that the Crown shall invite to form the Government after Shri Morarji De-
the leader of the majority party in the popular sai tendered his resignation in 1979. However,
House to be the Prime Minister and to form the
‘after four days of hectic activities, Shri Chavan
Government. So, interpreted, the President of
informed the President his inability to form the
India must invite the leader of the majority party
Government.
in the Lok Sabha to form the Government. How-
ever, if a person is not a member of the House, if Where none of the parties has attained ab-
he has the support and confidence of. the House, solute majority in the Lok Sabha, the President
he can be chosen to head the Council of Minis- may invite the leader of the single largest party
ters without violating the norms of democracy to form the Government. Sarkaria Commission
and the requirement of Article 75. But, such a also recommended the same. This was the case,
person must possess the necessary qualifications when, after the Lok Sabha elections in May,
and not be disqualified for being a member of 1991, the President invited Shri Narasimha Rao,
the Legislature.” the leader of the Congress Party, which was the
The Prime Minister has been described as single largest party, to form the Government.
“the keystone of the Cabinet arch, who is central Again, the President appointed Shri A. B. Vaj-
to its formation, central to its life, and central to payee the leader of the B.J.P., the single largest
its death.” Therefore, he must be a person who party in the twelfth Lok Sabha, after the 1996
can secure colleagues, and with his colleagues general elections. This Convention has been fol-
he must be sure of the support of the popular lowed by the Governors of the States in consti-
House of Parliament. The system of Parliamen- tuting State Ministries.
tary Government requires that the Prime Min- If two or more parties form a coalition be-
ister, along with his colleagues; not only be re- fore the election and secure absolute majority in
sponsible to the lower House, but that he shall the election, the acknowledged leader of such
be able to justify his policy in Parliament. a coalition should be invited to form the Gov-
Under normal circumstances, when a po- ernment. This practice was followed in India in
litical party has attained absolute majority in 1977 and again in 1989. In 1977, Shri Morarji
the Lok Sabha, the President has no choice or Desai, the leader of the Janta Party, a coalition
discretion but to invite the recognised leader of of several parties, who fought election on the
the party and appoint him the Prime Minister. In
common platform, formed the Government. In
case of death or resignation of the Prime Minis-
1989, Shri V.P. Singh, the leader of the Janata
ter, the ruling party elects a new leader. In such
Dal (a National Front. consisting of several lo-
a case also the President shall appoint the new
leader as the Prime Minister. cal and national parties), was invited to form the
But, in the case of multi-party system as is Government.
prevailing in India, and when none of these par- Instances are there when the leader of the
ties has secured absolute or workable majority coalition or alliance formed after the election,
in the lower House, the President can exercise was invited to form the Government. For in-
his personal discretion in selecting the Prime stance, Shri H. D. Deve Gowda, who was elect-
Minister. However, his discretion is conditioned ed, the leader of the United Front, constituting
by the requirement that he must choose the per- 13 parties, formed after the 1996 general elec-
son who can command the support of the major- tions to Lok Sabha, was invited and appointed
ity in the Lok Sabha. The President must explore the Prime Minister by the President.
the possibility of finding a person who can form It may be stated that the appointment of the
a stable government. Prime Minister by the President is not amend-

CLAT.indb 163 31/03/2009 11:41:52


164 CONSTITUTIONAL LAW

able to scrutiny in a court of Law. The Madras sioners, chairman and members of the fi-
High court rejecting a writ petition by Mr. M.R. nance commission and so on.
Parthasarthy, held that the President had to use 9. He advises the president with regard to
his own criteria when appointing the Prime summoning and proroguing of the ses-
Minister. sions of the Parliament.
10. He can recommend dissolution of the Lok
Deputy Prime Minister Sabha to president at any time.
The Constitution does not provide for the office 11. He announces government policies on the
of Deputy Prime Minister. Describing a person floor of the House.
as Deputy Prime Minister, therefore, is descrip- 12. He is the chairman of the Planning Com-
tive only and such description does not confer mission, National Development Council,
on him any powers of Prime Minister. He is for National Integration Council and Inter-
all purposes only a Minister. His taking the oath State council.
as Deputy Prime Minister, however, does not in- 13. He plays a significant role in shaping the
validate his appointment, nor the oath taken as foreign policy of the country.
such, in invalid. 14. He is the chief spokesman of the Union
government.
Powers and Functions of the Prime 15. He is the crisis manager-in-chief at the po-
Minister litical level during emergencies.
1. He recommends persons who can be ap- 16. As a leader of the nation, he meets various
pointed as ministers by the president. The sections of people in different states and
president can appoint only those persons receives memoranda from them regarding
as ministers who are recommended by the their problems, and so on.
prime ministers. 17. He is leader of the party, and so on.
2. He allocates and reshuffles various port- 18. He is political head of the services.
folios among the ministers.
3. He can ask a minister to resign or advise COUNCIL OF MINISTERS
the president to dismiss him in case of dif-
ference of opinion. Article 74 (1) provides that “There shall be a
4. He presides over the meeting of council of Council of Ministers with the Prime Minister
ministers and influences its decisions. as its head to aid and advice the President who
5. He guides, directs, controls, and coordi- shall, in exercise of his functions act in accor-
nates the activities of all the ministers. dance with such advice.”
6. He can bring about the collapse of the Article 75 (1), “the Prime Minister shall be
council of ministers by resigning from of- appointed by the President and other Ministers
fice. Since the prime minister stands at the shall be appointed by the President on the advice
head of the council of ministers, the other of the Prime Minister.”
ministers cannot function when the prime The Council of Ministers is formed as soon
minister resigns or dies. In other words, as the Prime Minister is sworn in. The number of
the resignation or death of an incumbent the Ministers in the Council has not been fixed
prime minister automatically dissolves the in the Constitution, except in the Delhi Legisla-
council of ministers and thereby generates tive Assembly where the number has been pro-
a vacuum. The resignation or death of any visioned not to exceeds 1 / 10th of the number
other minister, on the other hand, merely of the MLAs.
creates a vacancy which the prime minis- As per Article 75 (3), the Council of Minis-
ter may or may not like to fill. ters is collectively responsible to the Lok Sabha
7. He is the principal channel of communica- i.e., if a resolution is defeated in the Parliament,
tion between the president and the council the entire ministry collapses.
of ministers. The Prime Minister has the right to re-
8. He advise the president with regard to the fer to the President, the removal of dissident
appointment of important officials like at- minister(s) because technically the ministers are
torney general of India, comptroller and responsible individually to the President.
The Council of Ministers consists of three
auditor general of India, chairman and
categories of ministers—Ministers of Cabinet
members of the UPSC, election commis-
rank, State Ministers and Deputy Ministers.

CLAT.indb 164 31/03/2009 11:41:53


CONSTITUTIONAL LAW 165

Cabinet Ministers are the senior most Min- In the performance of his duties, the Attor-
isters to head a department with portfolio. They ney-General shall have right of audience in all
constitute the Cabinet and have the right to at- Courts in the territory of India. He shall neither
tend all the Cabinet meetings convened by the advise nor hold a brief against the Government
Prime Minister. of India in cases in which he is called upon to
The word ‘Cabinet Ministers’ has been advise the Government of India. Not should he
incorporated into the Constitution through the defend accused persons for criminal prosecu-
42nd Amendment Act in Article 352. The Cabi- tions without the permission of the Government
net is the smaller body of the Council of Min- of India. He is prohibited to take appointment as
isters. a Director in any company.
Ministers of State are lower in rank to Cab- The Attorney-General represents the Union
inet Ministers and normally assist the latter. and the States before the Courts but is also al-
Ministers of State are paid the same salary lowed to take up private practice provided, the
as the Cabinet Ministers, usually they are not other party is not the State. He is not paid a sal-
given independent charge of a ministry but the ary but a retainer that is determined by the Presi-
Prime Minister has the prerogative to allot an dent, although he is not a member of the either
independent charge if he desires so. House of the Parliament, he enjoys the right to
They cannot attend the Cabinet meetings attend and speak in the parliamentary delibera-
normally but can be invited to attend them. tions and meetings (of both the Lok Sabha and
The Deputy Minister cannot hold indepen- the Rajya Sabha), without a right to vote. He is
dent charge and always assist the Cabinet or entitled to all the privileges and immunities as a
State Minister or both. Member of the Parliament.
They never attend the Cabinet meetings. The retainer of the Attorney-General is
They are paid lesser salary than the Cabinet equal to the salary, of a Judge of the Supreme
rank ministers. Court. He is assisted by two Solicitors- General
The Cabinet is the supreme policy making and four assistant Solicitors-General.
body. The Attorney-General holds office during
The Cabinet is an extra Constitutional the pleasure of the President, and receives remu-
growth based upon convention. neration as the President may determine.
A Minister can be a member of either
House of the Parliament, but he is liable to vote
only in the House to which he belongs. COMPTROLLER AND AUDITOR
A person not belonging to any House can GENERAL
be appointed as a Minister but he has to get
elected wither House within a period of six Provisions regarding the Comptroller and Au-
moths [Article 75 (5)]. Non-member cannot be ditor-General (CAG) are given under Articles
re-appointed without being elected. 148–151. He is appointed by the President for a
According to article 75 (2), Ministers hold full term of 6 years or 65 years of age whichever
office during the pleasure of the President. is earlier.
Duties of CAG
ATTORNEY-GENERAL To audit the accounts of the Union and the
Article 76 states that the President shall appoint States and submit the report to the President or
a person who is qualified to be appointed as a the Governor, as the case may be. To ensure that
Judge of the Supreme Court to be the Attorney all the expenditures from the Consolidated Fund
General of India. of India or States are in accordance with the
He is the first legal officer of the Govern- Law. To oversee that the money sanctioned by
ment of India. It is convention that, after the the Parliament or the State Legislature is being
change of the Government, the Attorney-Gener- spent for the particular purpose for which it has
al resigns and the new Government appoints one been issued.
of its own choice. He advises the Government Also, to audit and report on the receipts and
of India on any legal matter. He performs any expenditure of the
legal duties assigned by the President of India. (i) Government companies
He discharges any functions conferred on him (ii) All bodies and authorities ‘substantially
by the Constitution or the President. financed’ from the Union or the State rev-

CLAT.indb 165 31/03/2009 11:41:53


166 CONSTITUTIONAL LAW

enues; and but he continues to audit the a citizen of India and has completed the age of
accounts of the Union, the States and the thirty-five years.
Public Sector undertakings under these He is not to be a member of either House
Governments. of the Parliament or a House of the state legis-
(iii) Other corporations or bodies when so re- lature and if he is so his seat would be regarded
quired by the Laws relating to such corpo- as vacant. When he enters upon the office of
rations or bodies. the Governor, he has also not to hold any office
Since the enactment of the Comptroller and of profit. His oath or affirmation (under Article
Auditor-General (Duties and Power) Act, 1976, 159) prescribes that he would preserve, protect
he ceases to prepare the accounts of the Union and defend the Constitution and law.
and the States, and Public Sector Undertakings The Constitution vests in the Governor all
under these Governments. the executive powers of the State Government.
The report of the CAG relating to the ac- He or she appoints the Chief Minister who
counts of a State shall be submitted to the Gov- enjoys the support of the majority in the Vidhan
ernor of the State, who shall cause it to be laid Sabha. He or she also appoints the other mem-
before the Legislature of the State. bers of the Council of Ministers and distributes
This report is immediately referred to the portfolios to them on the advice of the Chief
Public Accounts Committee of the Parliament Minister.
which, after a detailed study prepares another He also makes other major appointments of
report which is placed before the Parliament. the state like appointment of members of public
The discussions in the Parliament takes service commission, advocate general and sub
place on the secondary report of the Public Ac- ordinate judicial officers (ADJs).
counts Committee. The Council of Ministers remains in office
The CAG is an officer of the Parliament during the ‘pleasure’ of the Governor, but in the
and he is called Ears and Eyes of the Public Ac- real it means the pleasure of the Vidhan Sabha.
counts Committee. As long as the majority in the Vidhan Sabha
The CAG is concerned only at the state supports government, the Council of Ministers
of audit after the expenditure has already taken cannot be dismissed.
place. He or she is consulted by the President
in the appointment of the judges of the High
Courts and he or she appoint the judges of the
STATE EXECUTIVE District Courts.
The Governor has certain legislative pow-
The Governor
ers as well. The Governor summons the sessions
Article 153 says that there shall be a Governor of both Houses of the state legislature and pro-
for each state whereas Article 154 says that all rogues them. He or she can even dissolve the
executive powers of the state shall be vested in Vidhan Sabha. These powers are formal and the
the Governor who would exercise them either Governor while using these powers must act ac-
himself or through officers subordinate to him. cording to the advice of the Council of Ministers
However, the Governor’s functions can be headed by the Chief Minister. He or she inaugu-
increased and the Parliament or the State leg- rates the state legislature by addressing it after
islature can confer by law, any function to any the assembly elections and also at the beginning
authority subordinate to Governor. of the first session every year. His or her address
The Governor is appointed by the Presi- on these occasions is generally meant to outline
dent (Article 155). There can be one Governor the new policies of the state government (Ar-
for two or more states (Article 153). Article 156 ticles 174, 175, 176).
says that the term of office for the Governor is 5 A bill that the state legislature has passed,
years and he may continue in office till his suc- can become a law only after the Governor gives
cessor enters upon his office, not withstanding his assent to it. If he or she can return a bill to
the five-year term. the state legislature. if it is not money bill. for
The Governor holds office during the plea- reconsideration (Article 200). However, if the
sure of the President who can also transfer him state legislature sends it to him or her for the
to any other state. second time, he or she has to give his or her as-
Article 157 says that the person can be eli- sent to it. The Governor has the power to certain
gible for appointment as Governor unless he is bills for the President (Article 200).

CLAT.indb 166 31/03/2009 11:41:53


CONSTITUTIONAL LAW 167

When the state legislature is not in session preserves, protects and defends the Constitution
and the Governor considers it necessary to have and the law.
a law then he or she can promulgate ordinances In normal circumstances, the Governor has
(Article 213). These ordinances are submitted to to act as a constitutional and nominal head. But
the state legislature at its next session. They re- when emergency is declared, especially under
main valid for no more than six weeks from the Article 356. The Governor rules the state on the
date the state legislature is reconvened unless dictates of the Union Government and in the
approved by it earlier. name of the President.
To grant pardon, reprieve, remit the punish-
ment relating to a matter to which the executive CHIEF MINISTER AND COUNCIL
power extends, is Governor’s judicial power OF MINISTERS
(Article 161).
A money bill (Governor’s financial power) Article 163 of the Constitution says that there
is introduced in the Vidhan Sabha only with the shall be a Council of Ministers with the Chief
prior recommendation of the Governor. Minister as its head to aid and advise the Gover-
Normally, the Governor has to act on the nor in the exercise of his functions except those
aid and advice of the Council of Ministers head- where he exercises his discretion.
ed by the Chief Minister. It is the Governor who has to decide what
However, there are situations when he or is his discretion. This article also says that the
she has to act as per his or her own judgement advice given to the Governor by the Council of
and take decisions on his or her own. These are Ministers would not be questioned in any count
called the discretionary powers of the Gover- of law.
nor. He or she exercises them in the following This is clear from Article 163 that the
cases: Governor in normal circumstances, acts on
(a) In the appointment of the Chief Minister the advice of the Council of Ministers. normal
of a State (Article 164). When no party circumstances mean circumstances where the
gets a majority in the Vidhan Sabha. the Governor has no discretion. But where he has
Governor exercises his or her own volition the discretionary powers. The Governor acts on
and can either ask the. leader of the single his own. Before the Council of Ministers is con-
largest party or the consensus leader of stituted. the Governor has to appoint the Chief
two or more parties (that is, a coalition Minister who is usually the person who com-
party) to from the government. He or she mands the majority support in the State legisla-
then appoints him as Chief Minister. tive assembly.
(b) In informing the President of the failure of After the appointment of the Chief Minis-
constitutional machinery in a state (Article ter, the Governor appoints other Ministers on the
356). recommendations of the Chief Minister.
The Governor can send a report to the The State Council of Ministers exercises
President informing him or her that the all the executive power in the name of the Gov-
governance of the state cannot be carried ernor, formulates policy with regard to the state
out as per the provisions given in the administration, makes recommendation, with
Constitution of India and advises the regard to all the major appointments in the state
President to impose President’s rule in that carries on the state administration, and help,
state. Such a situation has arisen in every make legislation in the state. The Council of
state. The Governor, then, acts as an agent Ministers is collectively responsible to the state
of the President and rules on behalf of the legislative assembly.
President. The Chief Minister is like the Prime Min-
(c) In reserving certain bills for the consider- ister at the centre, the real executive head in the
ation of the President (Article 200). The state. He presides over the meetings of the state
Governor can rescue bills introduced in ei- cabinet, allocates departments among the state
ther House of the state legislature (Vidhan ministers. supervises their functioning. He is a
Sabha or Vidhan Parishad for the Presi- link between the Governor and the State Coun-
dent’s consideration if he or she feels the cil of Ministers. Article 167 of the Constitution
need to do so. prescribes the duty of the Chief Minister:
Like the President, it is the Governor who (a) To communicate to the Governor all de-

CLAT.indb 167 31/03/2009 11:41:53


168 CONSTITUTIONAL LAW

cisions of the State Council of Ministers ticipation in the legislative process is ensured by
relating to administration. making him a part of Parliament. The fact that
(b) To furnish such information relating to the he is the chief executive authority and that the
administration and proposal for legislation executive power is co-extensive with the leg-
as the Governor would call for. islative power also makes it necessary that he
(c) To submit, if the Governor so requires, for should become an integral part of the Legisla-
the consideration of the Council of Min- ture.
isters any matter on which a decision has
been taken by a Minister but which has not LOK SABHA
been considered by the Council of Minis-
ters. Maximum strength—550 + 2 nominated mem-
bers. [530—States/20—Union Territories]
Present strength of Lok Sabha—545.
UNION LEGISLATURE
The Eighty Fourth Amendment, 2001, ex-
Under the Constitution, the Legislature of the tended freeze on Lok Sabha and State Assembly
Union is called Parliament. The Indian Parlia- seats till 2006.
ment is constituted on the basis of the principle The normal tenure of the Lok Sabha is
of bi-cameralism, that is, the Legislature having five years, but it may be dissolved earlier by the
two Houses or Chambers. As the Constitution President. The life of the Lok Sabha can be ex-
established a federal system of government, tended by the Parliament beyond the five year
there was almost unanimity among the fram- term, when a proclamation of emergency under
ers for achieving a balance between the direct Article 352 is in force.
representation of people and the representation But the Parliament cannot extend the nor-
of units as such by setting up two Houses, one mal life of the Lok Sabha for more than one year
at a time (no limit on the number of times in the
representing the people as a whole and the other
Constitution).
the federated units.
To be a member of the Parliament, the can-
The two Houses of Parliament are the
didate must be:
House of the People (the Lok Sabha) and the
(a) Citizen of India.
council of States (the Rajya Sabha). The name
(b) Atleast 25 years of age.
of the Houses fairly reflect the character of the
(c) Must not hold any office of profit.
composition. The House of the People is com-
(d) No unsound mind/insolvent.
posed of directly elected representatives on the
(e) Has registered as voter in any Parliamen-
basis of adult franchise and territorial constitu-
tary constituency.
encies. The council of States is composed main-
Oath of MPs is conducted by the Speaker.
ly of representatives of the States elected by the
MPs can resign by writing to the Speaker.
State Assemblies. Presiding officer is Speaker (In his absence
The President is an integral part of Parlia- Deputy Speaker). The members among them-
ment. Under Article 79, Parliament shall consist selves elect him.
of the President and the two Houses. Making the The Speaker continues in office even after
President a part of Parliament is in conformity the dissolution of the Lok Sabha till a newly
with the principles and traditions of parliamen- elected Lok Sabha meets.
tary government in England. Usually, the Speaker, after his election
Parliament is constituted of the King, the cuts-off all connection with his party and acts
House of Lords and the House of Commons. in an impartial manner. He does not vote in the
In contrast, the President of the United States first instance, but exercises his casting vote only
is not a part of the US Congress. Whereas the to remove a deadlock.
presidential system of government emphasizes He charges his salary from Consolidated
the separation of the executive and legislative Fund of India. Speaker sends his resignation to
powers. the Deputy Speaker.
The parliamentary system lays stress on the The majority of the total membership can
intimate relationship and the interdependence of remove the Speaker after giving a 14 days no-
the Executive and the Legislature. tice. (During this time, he doesn’t preside over
Members of the Government are at the the meetings). After his removal, he continues in
same time members of the Legislature. His par- office till his successor takes charge.

CLAT.indb 168 31/03/2009 11:41:53


CONSTITUTIONAL LAW 169

RAJYA SABHA amendment of the Parliament by the State


Legislature(s) etc., are passed by the sim-
Maximum Strength—250 [Out of these, Presi- ple majority.
dent nominates 12 amongst persons having 2. Absolute Majority. It is the majority of
special knowledge or practical experience in more than the fifty per cent of the total
the fields of literature, science, art and social strength of the House, which includes
service]. even those members who are abstaining.
Presently, the Parliament, by law, has pro- For example, in case of the Rajya Sabha
vided for 233 seat for the States and the Union which has the total strength of 345 mem-
Territories. The total membership of Rajya Sab- bers, 123 and above shall be an absolute
ha is thus 245. majority.
All the States and the Union Territories of 3. Effective Majority. This is more than the
Delhi and Pondicherry are represented in the fifty per cent of the effective strength of
Rajya Sabha. the House (vacancies are not taken into
Representatives of the State are elected by account). In other words, the effective
members of State Legislative assemblies on the strength of the House is the total strength
basis of proportional representation through a of the House minus the number of vacan-
single transferable vote. [States are represented cies. In case of Rajya Sabha (total strength
on the basis of their population]. 245). If there are 15 vacancies, 230 shall
There are no seats reserved for Scheduled be the effective strength and more than
Castes and Scheduled Tribes in Rajya Sabha. 50% of this (i.e., 230–116 or more) is
The candidate must be: called the effective majority. Removal of
(a) Citizen of India the Vice-President of India (resolution for
(b) 30 years of age this can be introduced in the Rajya Sabha
(c) Be a parliamentary elector in the State in only) requires effective majority for pas-
which he is seeking election. sage of such a resolution to this effect (Ar-
(d) Others are prescribed by parliament from ticle 67 (b)).
time-to-time. 4. Special Majority. All types of majorities
The Rajya Sabha MPs are elected for a other than the above three are called the
term of 6 years, as 1/3rd members retire every special majorities. These are of the follow-
2 years. ing types:
Vice-President is the ex-officio chairman Special Majority under Article 249.
of Rajya Sabha. He presides over the proceed- This is basically a majority of 2/3rd
ings of the Rajya Sabha as long as he does not of the members of the House present
act the President of the India during a vacancy in and voting excluding the number of
the office of the President of India. members abstaining. For example, in
Also a deputy chairman is elected from its the Rajya Sabha (total strength 245) if
members. only 200 members are present and vot-
In Rajya Sabha, any bill can originate, apart ing, only 2/3rd of this (200) shall be
from money bill (including budget). special majority under Article 249 (i.e.,
for creation of one or more All-India
TYPES OF MAJORITY Services). To make it more clear, if 100
members of the House are present and
1. Simple Majority. Also called the ‘work- 10 of them abstain from voting then in
ing majority’, this is the majority of more this only 2/3rd of this (100 – 10 = 90)
than the fifty per of the members of the i.e., special majority under Article249.
legislature present and voting, excluding Special Majority under Article 61 (Im-
the members abstaining. For example, if peachment of the President of India).
the total number of the Members of the A resolution under Article 61 must be
Parliament present and voting is 500, a passed by not less than the two thirds
strength of 251 or more will be a simple of the total strength of the House, in-
majority. A confidence, No-confidence cluding the number of vacancies. For
or Censure Motion, Money, Financial or example, a resolution for the impeach-
Ordinary Bill, Budget, ratification of an ment of the President of India requires

CLAT.indb 169 31/03/2009 11:41:53


170 CONSTITUTIONAL LAW

the support of 1/3rd of total strength not less than one-tenth of the Members
of the Upper House (245, two third of can, on prior notice of 14 days, write
which shall be 164 or more). to the President for convening a session
Special Majority under Article 368. for the rejection of the National Emer-
(Constitutional Amendment). A bill gency (Article352).
seeking the Constitutional Amendment The Council of Ministers does not play
requires its passage by the 2/3rd of the any role in this.
members of the House present and vot-
End of the Session
ing. There is no provision of joint sit-
ting for this. Both the Houses must pass 1. Prorogation. This is done by the President
it separately. Also, this majority should on the advice of the Council of Ministers.
be the absolute majority of the House/ This can be done even when the House is
Constitutional Amendment Bills, Reso- adjourned. It brings a session of the House
lutions for removal of the Judges of to an end.
the Supreme Court or the High Courts, 2. Adjournment. This is a short recess within
Chief Election Commissioner, Comp- a session of the Parliament, called by the
troller and Auditor General, etc., are Presiding Officer of the House. Its dura-
passed by the special majority under tion may be from a few minutes to days
Article 368. together.
However, whenever the Constitution does 3. Another type of adjournment is when the
not specially mention the type of majority re- House is adjourned by the Presiding Of-
quired, it means the simple majority. ficer without fixing any date or time of the
next meeting. This is called Adjournment
Quorum in Parliament sine die, that is without fixing any time/
The Quorum to constitute a meeting of day. The adjournment does not bring to an
either House of the Parliament shall be end a session, but merely postpones the
one-tenth of the total number of Mem- proceedings of the House to a future time
bers of the House. and date.
If, at any time during a meeting of a
House, there is no quorum, it shall be TYPES OF MOTIONS
the duty of the Chairman or the Speak-
er, or person acting as such, either to Censure Motion This motion, seeking disap-
proval of the policy of the ruling Government,
adjourn the House or to suspend the
can be introduced in the Lok Sabha only by the
meeting until there is a quorum.
Opposition parties under the Rule 184 of the
Sessions of the Parliament Rules and Procedures of the Lok Sabha. If a cen-
The Constitution only states that there should not sure motion is passed in the House, the Council
be a gap of more than six months between two of Ministers is bound to seek the confidence of
consecutive sittings of the Parliament. There are the Lok Sabha as early as possible.
three types of sessions: Further, if a Money Bill or the Vote of
1. Budget Session Between February to thanks to the President is defeated, this also
May. This is the most important and the amounts to the censure of the Government poli-
longest session. cy and the Government needs to seek the confi-
2. Monsoon Session July–August. dence of the Lok Sabha.
3. Winter Session November–December. No-confidence Motion This is introduced
This is the shortest session. only in the Lok Sabha by the Opposition party.
There is a provision for special ses- When such a motion is admitted in the House,
sions in the Constitution. In this case the Members of Parliament have the right to
it can convened by the President on discus any acts of commission or omission on
the recommendation of the Council of the part of the Government on any policy mat-
Ministers on the basis of a 14 days of ter for which substantial time is allotted. When
advance notice to the Chairman or the admitted in the House, it takes precedence over
Speaker, as the case may be. In another all other pending business of the House. After
case, if the Lok Sabha is not in session, the adoption of a no-confidence motion in the

CLAT.indb 170 31/03/2009 11:41:53


CONSTITUTIONAL LAW 171

Lok Sabha, the Council of Ministers is obliged ing to the given agenda, but a matter of urgent
to resign. public importance can be brought before the
Legislature and discussed through Adjournment
Confidence Motion The provision of Confi-
Motion by interrupting its regular business. An
dence Motion is not found under the Rules and
Adjournment Motion should be supported by
Procedures of the Parliament but has come in
not less than 50 members of the House for being
vogue, under the Indian Parliamentary practice,
accepted. If accepted, it results in the holding
with the emergence of the coalition Govern-
the ongoing business of the House and discuss-
ments. The first incident of this was in Febru-
ing the matter raised in the Adjournment Mo-
ary 1979, when the then Charan Singh Govern-
tion. The basic idea behind this motion is to give
ment was asked by the President to seek the
an opportunity to the House to discuss a matter
confidence of the Lok Sabha. It is similar to the
‘No-Confidence Motion’ in all respects, except of urgent public importance. The matter should
that it is introduced by the Government itself be of definite nature and should have factual
to prove that it commands the approval of the basis. The discussion on the matter takes place
House. Thus, if a Confidence motion is defeat- at 1600 hours and continues for two and half
ed, the Council of Ministers is obliged to resign. hours. At the end of the discussion, voting takes
Examples of this are the fall of the V.P. Singh place. Since voting exposes the Government’s
Government in 1990 and of the Deve Gowda strength in the House, the ruling party tries not
Government in 1997. to allow the acceptance of the Adjournment Mo-
tion in the House.
Cut Motions These are a part of the budget-
ary process which seek to reduce the amount of Categories and Passage of the Bills
grants. These are moved in the Lok Sabha only. Ordinary Bills
They are classified into 3 categories: All the bills, other than Financial Bills, Money
(i) Policy Cut. A policy cut motion implies Bills, and the Constitutional Amendment Bills
that, the mover disapproves of the policy are Ordinary Bills. Such Bills can be introduced
underlying the demand. Its form of expres- in either House of the Parliament without the
sion is “that the amount of the demand be recommendation of the President, except those
reduced by Re 1”. Bills under Article 3.
(ii) Economy Cut. This means reduction in These Bills are passed by a simple major-
the amount of the expenditure. It clearly ity by both the Houses. Both the Houses enjoy
states the amount to be reduced and its equal jurisdiction over such Bills and in case of
form of expression is “that the amount of deadlock due to any reason, the tie is resolved
the demand be reduced by Rs… (a speci- by a joint sitting. The President has the right to
fied amount).” return such Bills for reconsideration to the Par-
(iii) Token Cut. It is introduced where the ob- liament once.
ject of the motion is to ventilate a specific Each House has laid down a procedure for
grievance within the sphere of the respon- the passage of a Bill. According to the procedure
sibility of the government of India. Its of the House, a Bill has to pass through three
form of expression is “that the amount of stages commonly known as Readings.
the demand be reduced by rupees 100.” First Reading: The Bill is introduced in the
Motions and Resolutions House. At his stage, no discussion takes place.
Second Reading: This is the consider-
Motion: It is a procedural device by which func-
ation stage when the Bill is discussed clause by
tions of the House are sought to be achieved. It
clause.
proposes a question or suggests a course of ac-
Third Reading: During this stage, a brief
tion before the House.
general discussion of the Bill takes place and the
Resolution: It is a self-contained motion. If a Bill is finally passed.
Resolution is passed in the form of a Statute, it
has a legally binding effect. But if it is passed as Money Bills
an expression of opinion, it has only a persua- Money Bill is defined in Article 110 of the Con-
sive effect. stitution. As per the Article, and Bill dealing
with all or any of the matters enumerated from
Adjournment Motion (a) to (g) of the same Article shall be a Money
The Legislature carries on its business accord- Bill. If there arises any question over the validity

CLAT.indb 171 31/03/2009 11:41:53


172 CONSTITUTIONAL LAW

of the Money Bill, the decision of the Speaker of Such a Bill must be passed by each House
the Lok Sabha is final. The speaker duly certifies separately with a special majority required un-
the Bill as Money Bill because this Bill passes der Article 368 i.e., not less than two-third of the
through special procedures (Article 109). Members of the House present and voting.
A money Bill can only originate in the Lok This majority should be more than the ab-
Sabha after the recommendation of the Presi- solute majority of the House. The joint-sitting
dent. of Parliament is not possible for passing such a
After being passed by the Lok Sabha, the Bill (Article 108). If the Bill is passed by both
Money Bill passes on to the Rajya Sabha which the Houses, it goes for the President’s assent.
has four options: By the 24th Constitutional Amendment Act, it is
(a) Pass the Bill in the original form obligatory for the President to give his assent to
(b) Reject the Bill the Bill amending the Constitution.
(c) Take no action for 14 days But the Amending power of the Parliament
(d) Send the Bill with suggestive amendments is subject to the ‘Basic structure of the Consti-
to the Lok Sabha. tution’. Thus the amending power is limited.
If the case is either (b) or (c), the Bill shall The Supreme Court can strike down any such
be automatically deemed to have been passed by amendment, if it is not in concurrence with the
the Rajya Sabha. In case of (d), the Lok Sabha basic structure of the Constitution.
has sole authority to accept or rject one or all of
the recommendation(s) and in this case also the
STATE LEGISLATURE
Bill shall be deemed as passed with or without
recommendations. The Legislative Council
There is no provision for a joint sitting of (Vidhan Parishad)
the Parliament to pass a Money Bill.
After the Money Bill is passed by the Lok As per the Constitution, the number of members
Sabha and the Rajya Sabha, it is presented to the of the Legislative Council is not to exceed one-
President who unlike in the case of other Bills, third of the total strength of the State Assembly.
has not right to withhold it (Article111). However, its strength should not be less than 40
either. The members of the Legislative Council
Financial Bills are derived from various sections and streams
Any Bill dealing with revenue or expenditure, of the society:
but not certified as a Money Bill by the speaker, (a) Not less than one-third to be elected by
is a Financial Bill. These Financial Bills are of the Panchayats, Municipalities, District
two classes: (a) A Bill containing any of the mat- Boards, etc
ters specified in Article 110, but not exclusively (b) Not less than one-third to be elected by the
dealing with those matters. This is called the Fi- Legislative Assembly.
nancial Bill of first Class. (b) An Ordinary Bill (c) Not less than one-twelfth to be elected by
contains provisions involving expenditure from the graduates of three years standing re-
the Consolidated Fund of India. This is called siding in the State.
the financial Bill of Second Class. (d) Not less than one-twelfth to be elected by
As regards the procedure for its passage, a the person having teaching experience of
Financial Bill is as good as Ordinary ill except three years in educational institutions.
that a Financial Bill cannot be introduced with- (e) The remainder one-sixth to be nominated
out the President’s recommendation, and it can by the Governor from among the distin-
only be introduced in the Lok Sabha. Thus a Fi- guished persons of the society in the field
nancial Bill is passed according to the ordinary of literature, science, arts, co-operative
procedure provided for passing of an Ordinary movement and social service.
Bill.
Just like the Upper House at the Centre,
Constitutional Amendment Bills the Legislative Council of a State is never dis-
Article 368 deals with the power of the Parlia- solved. The members are elected for a term of 6
ment to amend the Constitution, and the proce- years and 1/3rd of its members retire every two
dure thereof. A Bill for this can be introduced years.
in either House (the Lok Sabha or the Rajya The Legislatures of J & K, Bihar, Maha-
Sabha) of the Parliament and there is no need of rashtra, Karnataka and U.P., Andhra Pradesh are
the President’s recommendation for this. bicameral i.e., having Legislative Council also.

CLAT.indb 172 31/03/2009 11:41:53


CONSTITUTIONAL LAW 173

The Legislative Assembly • It has the exclusive authority to decide


(Vidhan Sabha) any dispute involving a question of law
The Legislative Assembly is the popular House or fact between the Government of In-
of the State Legislature where members are di- dia and one or more States or between
rectly elected by the people for a term of five two or more States inter se.
years, unless the House is dissolved by the Gov- • According to the Constitution (Seventh
ernor earlier. The strength of this popular House Amendment) Act, 1956, the Origi-
should not be less than 60 or more than 500. nal Jurisdiction of the Supreme Court
However, the President has the power to does not extend to a dispute that arises
alter this number and, in fact, the strength of out of any provision of a treaty, agree-
Goa and Sikkim Legislatures are less than 60. ment, covenant, management, sanad
The Governor may nominate one member from etc. which has been entered into or ex-
the Anglo–Indian community to this House, if ecuted before 26 January 1950, and has
he thinks that the community is not adequately been continued in operation after that,
represented. The sessions of the State Legisla- or which provides that the said jurisdic-
ture, and its officers as well as their functions are tion shall not extend to such a dispute.
almost similar to those at the Union level. • There are certain provisions in the Con-
stitution which exclude from the Origi-
JUDICIARY IN INDIA nal Jurisdiction of the Supreme Court,
certain disputes, the determination of
The Supreme Court which is vested in other tribunals:
i. Disputes specified in the provision
The Constitution consists provisions related to
to Article 363 (1)
the Union Judiciary in Article 124-147. Initially,
there was a Chief Justice and seven other Judges ii. Complaints as to interference with
in the Supreme Court. The number of the Judges inter-State water supplies, referred
excluding the Chief Justice was increased to 25 to the statutory tribunal mentioned
in 1985. At present, there is one Chief Justice in Article 262 (since the Parliament
and 25 other Judges in the Supreme Court. has enacted the Inter State Water
The power to increase or decrease the num- Disputes Act 1956).
ber of Judges in the Supreme Court rests with iii. Matters referred to the Finance
the Parliament. Commission (Article 280).
The President, if deems fit, may appoint ad iv. Adjustment of certain expanses be-
hoc Judges in the Supreme Court. tween the Union and the State (Ar-
The senior most Judges of the Supreme ticle 290).
Court is appointed as the Chief Justice of India. 2. Writ Jurisdiction
The Judges of the Supreme Court are appointed • Article 32 imposes duty on the Su-
by the President after consultation with such preme Court to enforce the Fundamen-
Judges of the Supreme Court and of the High tal Rights.
Courts as the President may deem necessary. • Under this Article, every individual
In the appointment of a Judge, other than has a right to move the Supreme Court
Chief Justice, the Chief Justice of India shall al- directly if there has been any infringe-
ways be consulted. ment on his fundamental Rights.
Article 129 of the Constitution declares that • The Writ Jurisdiction some times is
the Supreme Court shall be a court of record and referred to as the Original Jurisdiction
shall have all the powers of such a Court includ- of the Supreme Court but in the strict
ing the power to punish for contempt of itself. sense. Original Jurisdiction relates to
the federal character of the constitu-
Jurisdiction of the Supreme Court tion.
The Jurisdiction of the Supreme Court are five- 3. Appellate Jurisdiction
fold viz., Original, Writ, appellate, Advisory and The Appellate Jurisdiction of the Supreme
Revisory jurisdictions. Court is three fold:
1. Original Jurisdiction (a) Constitutional: In the Constitutional
• The Original Jurisdiction of the Su- matters, an appeal lies to the Supreme
preme Court is purely federal in char- Court if the High Court certifies that
acter. the case involves a substantial ques-

CLAT.indb 173 31/03/2009 11:41:53


174 CONSTITUTIONAL LAW

tion of law as to the interpretation of cle 138), disputes arising out of any treaty
the Constitution. agreement etc., which had been entered
If the High Court refuses to give the into or executed before the commence-
certificate, the Supreme Court may ment of the Constitution.
grant special leave for appeal if it is • In such cases, it is obligatory for the
satisfied that the case does involve courts, under the Indian Constitution,
such a question. to give its opinion to the President.
(b) Civil: In civil cases, an appeal lies to 5. Revisory Jurisdiction
the Supreme Court if a High Court • The Supreme Court under Article 137
certifies that the value of the subject is empowered to review any judgement
matter of the dispute is not less than or order made by it with a view to re-
Rs 20,000 or that the case is fit for ap- move any mistake or error that might
peal to the Supreme court. have crept in the judgement or order.
The appellate jurisdiction of the Court This means that even though all the judge-
in civil cases can be enlarged if the ments and orders passed by the Supreme
Parliament passes a law to that effect. Court are binding on all the Courts of In-
(c) Criminal: In the criminal cases, an dia, they are not binding on the Supreme
appeal lies to the Supreme Court if the Court.
High Court
i. has on appeal, reversed the order THE HIGH COURTS
of acquittal of an accused and
sentenced him to death; or • The High Courts stands at the head of
ii. has withdrawn for trial before the judiciary in a State.
itself any case from any subor- • There shall be a High Court for each
dinate and has in such trial con- State (Article 214).
victed the accused and sentenced • The Judiciary in the States consists
him to death; or of a High Court and the Subordinate
iii. certifies that the case is fit for ap- Courts.
peal to the Supreme court. • The Parliament can, however, establish
• The Appellate Jurisdiction of by law, a common High Court for one
the Supreme Court in criminal or more State(s) and one or more Union
matters can be extended by Territory (Article 231).
the Parliament, subject to such • Every High Court shall be a Court of
conditions and limitations as record (Article 215).
may be specified therein.
• The Supreme Court under Ar- Jurisdiction of the High Court
ticle 136 enjoys the power of (a) Original Jurisdiction: In their judicial
granting special leave to ap- capacity, the High Courts of Presidency
peal from any judgement, de- Towns (Calcutta, Madras and Bombay)
cree, order or sentence in any have both original and appellate jurisdic-
case or matter passed by any tion, while other High Courts have mostly
Court or tribunal except court appellate jurisdiction.
martials. • Only in matters of admirally, probate
4. Advisory Jurisdiction matrimonial and contempt of court,
One of the salient features of the Supreme they have original jurisdiction.
Court of India is its consultative role (Art, • The Presidency High Courts have origi-
143). The President can refer to the Court nal jurisdiction in civil cases in which
either a question of law or a question of the amount involved is more than Rs
2,000 and in criminal cases which are
factor, provided that it is of public impor-
committed to them by the Presidency
tance.
Magistrates.
However, it is not compulsory for the
(b) Appellate Jurisdiction: As courts of ap-
Court to give its advice.
peal, all High Courts entertain appeals in
The President is empowered to refer to the civil and criminal cases from their subor-
Supreme Court for its opinion (under Arti- dinates courts as well as on their own.

CLAT.indb 174 31/03/2009 11:41:53


CONSTITUTIONAL LAW 175

COMPARISON BETWEEN THE SUPREME COURT AND


THE HIGH COURTS
Supreme Court High Court

1. The Supreme Court is a federal Court. Its 1. There is a provision for a High Court in
only seat is located at Delhi. Its bench can each State and each Union Territory but
be established at other places also but so two or more States or two or more Union
far it has not been established. Territories or States Union Territories,
together, may establish a common High
Court.
2. The Judges of the Supreme Court are 2. The Judges of the High Court are also
appointed by the President. appointed by the President.
3. A person shall have the following 3. A person shall not be eligible to become
qualification to become eligible for the a Judge of a High Court unless such a
appointment as a Judge of the Supreme person.
Court, if: (i) has been a judicial officer for not less
(i) He has been a Judge of a High than 10 years within the territory of
Court for not less than five years in India; or
succession; or (ii) has been an advocate for not less than
(ii) Has been an advocate of a High 10 years in a High Court in India.
Court for not less than 10 years in
succession; or
(iii) He is a distinguished Jurist in the
opinion of the President.
4. The Judges of the Supreme Court retires 4. The Judges of the High Courts retire from
from their office after attaining the age of their office after attaining the age of 62
65 years. years.
5. The President can remove the Chief 5. The judges and the Chief Justice of the
Justice and the other judges on the basis High Courts are removed from the office
of impeachment motion passed in the by the President in the same manner as
Parliament. adopted in the case of the Supreme Court.
6. The salary of the Chief Justice is Rs 33,000 6. The salary of the Chief Justice is Rs 30,000
and that of the other judges is Rs 30,000 and that of other judges is Rs 26,000 per
per month. month.
7. The judges of the Supreme Court after 7. The judges of the High Courts cannot
their retirement and during their term plead before any Court during the term of
of office are not eligible to plead before their office. After retirement they cannot
any court/authority within the territory of plead before any Court below the High
India. Court. That means they can plead only
before other High Courts and the Supreme
Court.
8. The Judges of the Supreme Court cannot 8. The judges of the High Court and may
be transferred not can they be demoted in be promoted as the Judges of the Supreme
office. Court.
9. The Supreme Court is not bound to abide 9. The High Courts are bound to abide by the
by the decisions of the High Courts. decisions of the Supreme Court.
10. The salary and the allowances of the 10. The salary and other allowances of the
Judges of the Supreme Court are charged Judges of the High Courts are charged
upon the Consolidated Fund of India. upon the Consolidated Fund of the States.
11. The cases involving the interpretation of 11. The cases involving the interpretation of
the Constitution are decided only by the the Constitution are not decided by the
Supreme Court. High Courts.

CLAT.indb 175 31/03/2009 11:41:53


176 CONSTITUTIONAL LAW

• They have, however, no jurisdiction ELECTION COMMISSION


over tribunals established under the
laws relating to the armed forces of the The Election Commission is a permanent and an
country. independent body established by the Constitution
(c) Writ Jurisdiction: Under Article 226 of of India. Article 324 of the Constitution provides
the Constitution, the High Courts are that the power of superintendence, direction and
given powers of issuing writs not only for control of the elections, to the Parliament, State
the enforcement of Fundamental Rights, Legislatures, the office of the President of India
but also for other purposes. and the office of the Vice-President of India,
• In exercise of this power, a Court may shall be vested in the Election Commission.
issue the same type of writs, orders or Focus
directions which the Supreme Court is
The Electoral System in India is borrowed from
empowered to issue under Article 32. the one operating in Great Britain.
The jurisdiction to issue writs under The Election commission is not concerned
this Article is larger in the case of High with the elections to Panchayats and Municipali-
Courts, for while the Supreme Court can ties in the States. The elections to the Panchayats
issue them only where a fundamental right and the Municipalities in the States are conduct-
has been infringed, a High Court can issue ed by State Election Commissions.
them not only in such cases, but also where The Election Commission consists of one
an ordinary legal right has been infringed. chief Election Commissioner (CEC) and two

THE HIGH COURTS: SEATS AND JURISDICTION

Name Year Territorial Jurisdiction Seat


of Estb.
1. Allahabad 1866 Uttar Pradesh Allahabad (Bench at Lucknow)
2. Andhra Pradesh 1954 Andhra Pradesh Hyderabad
3. Bombay 1862 Maharashtra, Dadar and Nagar Bombay (Bench at Nagpur,
Haveli, Goa, Daman and Diu Panji, Aurangabad)
4. Kolkata 1862 West Bengal, Andaman and Kolkata (Bench at Port Blair)
Nicobar Islands
5. Delhi 1966 Delhi Delhi
6. Guwahati 1948 Assam, Manpur, Meghalaya, Guwahati (Benches at Kohima,
Nagaland, Tripura, Mizoram Imphal, Agartala and Shillong)
and Arunachal Pradesh
7. Gujarat 1960 Gujarat Ahmedabad
8. Himachal Pradesh 1966 Himachal Pradesh Shimla
9. Jammu and Kashmir 1928 Jammu and Kashmir Srinagar and Jammu
10. Karnataka 1884 Karnataka Bangalore
11. Kerala 1958 Kerala and Lakshdweep Ernakulam
12. Madhya Pradesh 1956 Madhya Pradesh Jabalpur
(Bench at Indore, Gwalior)
13. Madras 1862 Tamil Nadu and Puducherry Madras
14. Orissa 1948 Orissa Cuttack
15. Patna 1916 Bihar Patna
16. Punjab and Haryana 1975 Punjab, Haryana, Chandigarh Chandigarh
17. Rajasthan 1949 Rajasthan Jodhpur (Bench – Jaipur)
18. Sikkim 1975 Sikkim Gangtok
19. Chhattisgarh 2000 Chhatisgarh Bilaspur
20. Uttaranchal 2000 Uttaranchal Nainital
21. Jharkhand 2000 Jharkhand Ranchi

CLAT.indb 176 31/03/2009 11:41:54


CONSTITUTIONAL LAW 177

CHIEF ELECTION COMMISSIONERS OF INDIA


Name From To

1. Sukumar Sen 21 March 1950 19 December 1958


2. K. V. K. Sundaram 19 December 1958 30 September 1967
3. S. P. Sen Verma 30 September 1967 30 September 1972
4. Dr Nagendra Singh 30 September 1972 6 February 1973
5. T. Swaminathan 6 February 1973 18 June 1977
6. S. L. Shakdhar 18 June 1977 18 June 1982
7. R. K. Trivedi 18 June 1982 31 December 1985
8. R. V. S. Perishastri 1 January 1986 15 November 1990
9. Smt V. S. Rma Devi (Acting) 15 November 1990 12 December 1990
10. T. N. Shesan 12 December 1990 11 December 1997
11. M. S. Gill 12 December 1997 12 June 2001
12. J. M. Lyngdosh 14 June 2001 7 February 2004
13. T. S. Krishnamurthy 8 February 2004 2007
14. N. Gopalaswami 2007 till date

Election Commissioners. By an ordinance of Every candidate has to make a deposit


1993, the powers of lection Commissioners of Rs 10,000 for the Lok Sabha election and
have been made equal to those of the Chief Rs 5,000 for the Rajya Sabha or the Vidhan
Election Commissioner. The commission works Sabha elections, except for candidates from the
under the overall supervision of the Chief Elec- Scheduled Castes and Scheduled Tribes who
tion Commissioner. It conducts and supervises pay half of these amounts. The deposit is re-
elections and by-elections. turned if the candidate receives more than one
If fixes the limit of election expenses and sixth of the total number of valid votes polled in
examines the accounts of the electoral expendi- the constituency.
tures.
The term of the Chief Election Commis-
sioner is for 6 years or till he/she attains the age
PUBLIC SERVICE COMMISSIONS
of 65 years, whichever is earlier. He/she can be
The Constitution visualizes the Union Public
removed from the office in the same manner as
Service Commission to be the ‘Watch dog of
the Judges of the Supreme Court. He/she is not
merit system’ in India. Provisions regarding the
eligible for re-appointment. He/she cannot hold
any office of profit after his/her retirement. His/ Public Service Commissions are contained from
her salaries and allowances are met from the Article 315 to Article 323. There shall be a Pub-
Consolidated Fund of India. The electoral sys- lic Service Commission for the Union and Pub-
tem is based on adult-suffrage, the citizen not lic Service Commissions for the States.
less than 18 years of age and not otherwise dis- The State Public Service Commission is
qualified has the right to vote. It is based on the created, like UPSC, directly by the Constitu-
geographical representation. tion, a Joint State Public Service Commission
(JSPSC) can be created by an act of the Parlia-
Focus
ment on the request of the Legislature of the
In India, election results are determined on the State concerned. The UPSC can also service the
basis of “First past the post” system. needs of a State on the request of the State Gov-
ernor and with the approval of the President of
QUALIFICATIONS FOR India. The UPSC is the central recruiting agency
CONTESTING THE ELECTION in India. It is an independent constitutional body
in the sense that it is directly created by the Con-
Any Indian citizen who is registered as a voter
stitution of India.
and is over 25 years of age is allowed to contest
elections to the Lok Sabha or the State Legis- The members of the UPSC are to be ap-
lative Assemblies. For the Rajya Sabha the age pointed by the President and the SPSCs are to
limit is 30 years. be appointed by the Government.

CLAT.indb 177 31/03/2009 11:41:54


178 CONSTITUTIONAL LAW

Composition for any other employment either under the Gov-


The composition of the Commission is deter- ernment of India or any State.
mined by the President or the Governors in case A member other than the Chairman of a
of the UPSC and the SPSCs respectively. The State Public Service Commission shall be eli-
Constitution provides that one half of the mem- gible for appointment as the Chairman or the
bers of the Commission should be such persons member of the Union Public Service Commis-
who had held any office for at least ten years sion or as the Chairman of that or any other State
under the government. Public Service Commission, but not for any oth-
er employment either under the Government of
Term of Office India or under the Government of any State.
The Chairman and the members of the UPSC The expenses of the Union or a State Pub-
shall hold office for a term of six years or un- lic Service Commission, including salaries, al-
til they attain the age of 65 years, whichever is lowances and pensions payable to or in respect
earlier. to the members or the staff of the Commission,
The Chairman and the members of the shall be charged on the Consolidated Fund of
SPSC and the JSPSC shall hold office for a term India or, as the case may be, the Consolidated
of six years or until they attain the age of 62 Fund of the State.
years whichever is earlier. By any reason, if the The function of the UPSC is to advise, not
office of the Chairman of the Commission be- to decide.
comes vacant, and until the new Chairman has
resumed, his duties are to be performed by such PLANNING COMMISSION
one of the other members of the Commission as
the President, in the case of the UPSC and the It is an extra-constitutional and a non-statutory
JSPSC and the governor of the State in the case body. It is the supreme organ of Planning for so-
of the SPSC, may appoint for the purpose. cial and economic development in India. It was
The Chairman and the members of the Pub- established on 15 March 1950 by an executive
lic Service Commissions shall, on the expiration resolution of the Government of India. It was
of their terms of office, be ineligible for re-ap- established on the recommendation of the Ad-
pointment to that office. The Chairman or any visory Planning Board constituted in 1946 un-
other member of a Public Service Commission der the chairmanship of K. C. Neogi. The Prime
shall only be removed from his office by an or- Minister of India is the ex-officio chairman of
der of the President on the ground of misbehav- the Planning Commission. The Deputy Chair-
iour, after the Supreme Court, on reference being man is the de facto executive head (i.e., full time
made to it by the President, has on inquiry held functional head) of the Commission.
in accordance with the procedure prescribed in The Commission has four to seven full-
that behalf, reported that the Chairman or such time expert members. The full-time members
other member, as the case may be, ought on any enjoy the rank of a Minister of State. The Plan-
such ground to be removed. ning Commission has a member-secretary. He is
The Chairman of the Union Public Service usually a senior member of IAS. Planning Com-
Commission, shall be ineligible for further em- mission works as a staff agency with an advi-
ployment either under the government of India sory role.
or under the Government of any State. A mem-
ber other than the Chairman of the Union Public Focus
Service Commission shall be eligible for ap- Planning Commission is also known as ‘Super
pointment as the Chairman of the Union Public Cabinet’, an ‘Economic Cabinet’ a ‘Parallel
Service Commission or as the Chairman of the Cabinet’, the Fifth ‘wheel of the coach’ etc.
State Public Service Commission, but not for At present, K. C. Pant is the Deputy Chair-
any other employment either under the Govern- man of the Tenth Planning Commission
ment of India or under the Government of any
State. NATIONAL COMMISSION FOR
The Chairman of a State Public Service WOMEN
Commission shall be eligible for appointment as
the Chairman or member of the Union Public The National Commission for Women was set
Service Commission or as the Chairman of any up as a statutory body in January 1992 under the
other State Public Service Commission, but not National commission for Women Act, 1990 to

CLAT.indb 178 31/03/2009 11:41:54


CONSTITUTIONAL LAW 179

review the Constitutional and Legal Safeguards (2) an officer who is member of a Civil
for the women. Services of the Union or of an All In-
dia Service with appropriate experi-
Focus
ence.
The First Commission was constituted on 31
January 1992 with Mrs Jayanti Patnaik as the
DIFFERENCE BETWEEN AN
Chairperson.
The Second Commission was constituted ATTORNEY GENERAL AND A
on July 1995 with Dr Mohini Giri as the Chair- SOLICITOR GENERAL
person.
An Attorney-General is an officer appointed by
The Third Commission has been consti-
the President of India under Article 88 of the
tuted and the Government has nominated Mrs
constitution. He represents the Union govern-
Vibha Parthasarthy as the Chairperson.
ment in the Supreme court. A Solicitor General
The Fourth Commission Chairperson: Dr
is with the law ministry and drafts the bills which
Purnima Advani.
become laws after being passed by Parliament.
The Fifth Commission Chairperson: Girija
A Solicitor General cannot appear in any court
Vyas.
while and Attorney-General is the first person,
Composition by right, to address the Supreme Court.
The Commission shall consist of:
(a) A Chairperson (who is committed to the FINANCE COMMISSION
cause of the women) to be nominated by
the Central government. Article 280 of the Constitution provides for a
(b) Five members—to be nominated by the finance Commission. It is a quasi-judicial body.
Central government from amongst the per- It is constituted by the President of India every
sons of ability integrity and standing—and fifth year or at such a earlier time as he considers
have the experience in various fields, like necessary. The Finance Commission consists of
law or legislation, trade unionism, man- a Chairman and four other members, to be ap-
agement of industry, potential of women, pointed by the President. They are eligible for
women’s voluntary organization, admin- reappointment.
istration, economic development, health The Finance Commission is required to
education or social welfare. make recommendations to the President of In-
(c) A Member Secretary—to be nominated dia on the following matters (Article 280). The
by the Central government. The Member- distribution of the net proceeds of taxes to be
Secretary should be either: shared between the Centre and the States. The
(1) an expert in the field of management, recommendations made by the Finance Com-
organization structure or social move- mission are only of advisory nature and hence,
ment, or not binding on the Government.

Finance Commission Chairman Constitution Report

First Finance Commission K. C. Neogy 1951 1953


Second Finance Commission K. Santhanam 1956 1957
Third Finance Commission A. K. Chanda 1960 1962
Fourth Finance Commission Dr Rajamannar 1964 1965
Fifth Finance Commission Mahavir Tyagi 1968 1969
Sixth Finance Commission Brahmananda reddy 1972 1973
Seventh Finance Commission Shelat 1977 1978
Eighth Finance Commission Y. B. Chavan 1982 1984
Ninth Finance Commission N. K. P. Salve 1987 1989
Tenth Finance Commission K. C. Pant 1992 1994
Eleventh Finance Commission A. M. Khusro 1998 2000
Twelfth Finance Commission C. Rangarajan 2002 2007
Thirteenth Finance Commission Vijay Kelkar 2007 till date

CLAT.indb 179 31/03/2009 11:41:54


180 CONSTITUTIONAL LAW

The Constitution of India envisages the Fi- There is no provision of joint sitting in case
nance Commission as the ‘Balancing wheel of of a dead-lock in the matter of an Amendment
Fiscal Federalism’ in India. Bill.
The President cannot withhold his assent
Focus
to amendments of returning such Bills to Parlia-
Till now the Presidents have constituted Twelve ment for reconsideration.
Finance Commissions.
STATE OF J & K, GOVERNMENT
AMENDMENT OF THE AND JUDICIARY
CONSTITUTION
Procedure: Constitution making is a continuous The State of Jammu and Kashmir
process. The founding father of the Indian Con- The State of Jammu and Kashmir holds a spe-
stitution kept it in mind while introducing the cial position under the Government of India.
amendment procedure of the Constitution. Though it is one of the States specified in the
Part XX, Article 368 of the Constitution First schedule and forms a part of the territory of
prescribes three methods for amendment of dif- India as defined in Article 1, all the provisions of
ferent provisions of the Constitution: the Constitution of the India relating to the State
Some part of the Constitution can be do no apply to Jammu and Kashmir. The State
amended by a simple majority in both Houses alone of all the States of the Indian union has its
of Parliament. For example, matters relating to own Constitution.
creation/reorganization of State, creation/aboli- Article 370: Article 370 under Part XXI
tion of Upper House (Legislative Council in the of the Indian Constitution(Temporary, Transis-
States), qualifications for Indian citizenships tional and Special Provisions) accords special
etc. status to the State of Jammu and Kashmir. It was
Certain other provisions of the Constitution incorporated in the Constitution because, under
can be amended only if they are the circumstances to which the State acceded to
(a) passed by a majority of the total member- India, the Government of Indian gave a com-
ship of each House of Parliament and by mitment to the effect that people of Jammu and
a majority of not less than two thirds of Kashmir acting through their Constituent assem-
the members present and voting in each bly, were to finally determine the Constitution of
house; and the State. Article 370 stipulates that the power
(b) ratified by the legislatures of one-half of of the Parliament to make laws for Jammu and
the States. This provision is applicable Kashmir is limited to those matters in the union
while amending those parts of the Consti- list and the concurrent list which correspond to
tution in which the interest of the State are matters specified in the instrument of accession
involved. For example, method of elec- and such other matters which the President may
tion of the President, extent of executive specify with the concurrence of the government
and legislative powers of the Union or the of the State.
State, the lists in the Seventh Schedule, Clause 3 of Article 370 provides that the
provisions regarding the Supreme Court President can, by public notification, declare this
and the High Courts, the representation of Article to cease to be operative if the Constitu-
the State in Parliament, or the process of ent Assembly of the State recommends so to the
amendment. President. Recently, political groups have de-
The amendment of the remaining provi- manded the repeal of Article 370 so that the spe-
sions of the Constitution requires to be passed cial status of the State is abolished. It is pointed
by a majority of the total membership of each out that the Constituent assembly of the State no
House and a majority of not less than two-thirds longer exists and the President is free to act.
of the members present and voting in each The Constituent Assembly of Jammu and
House. Kashmir ratified the accession to India early in
An amendment bill may be introduced in 1954. The Constitutional (Application to Jammu
either House of Parliament. The Constitution and Kashmir) order issued by the Parliament in
does not empower the State Legislatures to initi- consultation with the State Government in 1954
ate Constitutional amendment. extended the Union’s jurisdiction to all Union

CLAT.indb 180 31/03/2009 11:41:54


CONSTITUTIONAL LAW 181

subjects under the Constitution of India (subject The provisions of Part IV relating to the
to slight alterations). Directive Principles of State Policy do not ap-
Special Relationship: Some of the features ply to Jammu and Kashmir. Special rights in
of special relationship of Jammu and Kashmir relation to employment, acquisition of property
with the Indian union are as follows: Jammu and settlement have been given to ‘permanent
and Kashmir has its own Constitution framed residents’ of the State. No amendment to the
by a special Constituent Assembly set up by the Constitution of India is applicable to Jammu and
State. Only some subjects on the concurrent list Kashmir unless it is extended to it by an Order
can be legislated on by the union. In the case of of the President under Article 370 (i).
other States, the residuary power of legislation By amendment of the Constitution Order,
belongs to Parliament, but in the case of Jammu the Jurisdiction of the Comptroller and Auditor-
and Kashmir, the residuary power belongs to General, the Election Commissioner, and the
the State Legislature (except in certain matters special leave jurisdiction of the Supreme Court
for which Parliament has exclusive power. For have been extended to the State of Jammu and
example, provision of activities relating to ces- Kashmir.
sion and secession). Now law of preventive de-
tention made by Parliament will extend to that
LOCAL GOVERNMENT
State. However, by the Constitution (application
to Jammu and Kashmir) Order 1986, Article 249 The idea of local government has undergone a
has been extended to the State so that it is al- transformation through the 73rd and 74th Con-
lowed to extend the jurisdiction of Parliament to stitutional Amendment Acts. Local government
the State in national interest by passing a resolu- in both urban and rural areas has been listed as a
tion in the Rajya Sabha. State subject in the seventh schedule. The union,
Parliament cannot make any law without therefore, cannot legislate regarding the rights
the consent of the State Legislature relating to and liabilities relating to these subjects. Hence
(i) alteration of name or territories of the State, the union has drafted the outlines of the schemes
(ii) international treaty of the State. No procla-
in Articles 243–243 ZG to be implemented by
mation of an emergency made by the President
the States after making or amending their own
rule Article 352 on the ground of armed rebel-
existing laws to conform to the 73rd and 74th
lion shall have effect on the State of Jammu and
Amendment Acts.
Kashmir without the State Government’s con-
Article 40 under the Directive Principles
currence. The union has no power to suspend
of State Policy of the Constitution directs the
the Constitution of the State on the ground of
State “to organize village Panchayats as units
failure to comply with the directions given by
of self-Government”. It was in pursuance of this
the union. In the event of a breakdown of the
objective and as a part of the planning process
constitutional machinery provided by the State
that eh Government sponsored the Programme
Constitution, Governor’s rule is to be imposed
of Community Development. The programme
(a feature not provided for any other State in
India). However, in 1964, Article 356 and 357 was launched on 2 October 1952 with a view
were extended to that State and the President to create in the rural people a sense of partici-
was authorized to take over the administration pation in the nation-building programmes. On 2
of the State in the event of a breakdown of Con- October 1953 a network of National Extension
stitutional machinery (as per provisions of the Services (NES) for rural development was cre-
State’s Constitution, and not those in Part VI of ated. In 1956, a study team under Balwant Raj
the Constitution of India). Parliament was also Mehta was appointed to review the working of
given the power to legislate for the State dur- the two schemes. Balwant Raj Mehta committee
ing Emergency under Article 356. (The First recommended the establishment of Panchayati
occasion when President’s Rule under Article Raj on the principle of ‘democratic decentraliza-
356 was imposed in Jammu and Kashmir was in tion’. The scheme envisaged a three-tier system
1986 to follow governor’ Rule. From 1990, fol- of local government.
lowing a short duration popular rule, President’s In view of the various shortcomings of the
rule continued in the State till 1996 when the Panchayati Raj institution, a Committee under
National conference was voted into power). The the chairmanship of Ashoka Mehta was set up
union has no power to make a proclamation of in December 1977 to review the working of
Financial Emergency in the State. Panchayati Raj. The Committee submitted its

CLAT.indb 181 31/03/2009 11:41:54


182 CONSTITUTIONAL LAW

report in 1978, in its report the committee fa- Powers and Responsibilities: State Leg-
voured the replacement of the existing three- islatures may by law confer on Panchayats the
tier system by a two-tier system with Mandal power and authority necessary for them to func-
Panchayats at the base and Zila Parishads at the tion as institutions of self-Government, accord-
top. ing to Articles 243 G–243 H. Their responsibili-
But these recommendations were not ac- ties could cover preparing plans for economic
cepted and the Panchayati Raj Institutions con- development and social justice, implementing
tinued to work on the basis of the recommenda- these plans, and control over certain matters
tions of the Balwant Raj Mehta Committee. listed in the Eleventh Schedule which contains
The New Dispensation: The fact that the 29 items, such as land improvement, minor ir-
Panchayati Raj system was not taken too seri- rigation, animal husbandry, fishery, women and
ously by the State and there was no effort to hold child development and education. Thus the elev-
elections regularly prompted the Rajiv Gandhi enth schedule envisages a distribution of powers
Government to try and reorganize the system between the State legislature and the Panchayat.
by inserting certain provisions in the Constitu- The powers of local bodies are to be delegated
tion. The 64th Amendment Bill that was put for- by the State legislature.
wards, however, did not meet with success. The The State Legislatures can authorize
ideas were, therefore, modified and presented in Panchayats to levy, collect and appropriate suit-
the 72nd Amendment Bill, which was passed by able local taxes and also provide for making
Parliament in 1992. The Amendment inserted grants-in-aid to the Panchayats from the Con-
Part IX containing Articles 243 to 243–O in the solidated Fund of the State.
Constitution. It got Constitutional status as the Finance Commission: Article 243 I says
73rd Amendment Act in April 1993 after ratifi- that within a year from the date of Constitution’s
cation by more than half the number of State. 73rd Amendment comes into force, that is, from
By April 24, 1994, all State had passed the nec- 25 April 1993, and afterwards every five years,
essary legislation to implement the Act, as re- the State Government is to appoint a Finance
quired. The Act is not applicable to Jammu and Commission to review the financial position of
Kashmir, Meghalaya, Mizoram, Nagaland and the Panchayats and to make recommendations
certain scheduled areas of some States. on the following matters—(i) distribution be-
tween the State and the Panchayats of the net
The Act presents the following features:
proceeds of taxes, duties, tolls and fees leviable
Three-Tier System: A three-tier system of by the State which may be divided between them
Panchayats is envisaged in Part IX: (i) the village and how the allocation is to be made among the
level; (ii) the district level; (iii) the intermediate various tiers; (ii) what taxes, tolls and fees may
level standing between the village and district be assigned to the Panchayats; (iii) grants-in-aid
panchayats in State with population above 20 to Panchayats. The Commission’s report with a
lakh. Thus, States within the limit of population memorandum of action taken on it is to be laid
have the opinion of not having a Panchayat at before the State legislature.
the intermediate level. State Election Commission: In order to
Composition: Members of the Panchayat ensure free and fair elections to the Panchayats,
at all levels will be chosen by direct election Article 246 K provides for the Constitution of
from territorial constituencies in the Panchayat a State Election Commission in respect of the
Area. The electorate is the Gram Sabha, that is, Panchayats. This Election Commission will
persons registered in the electoral rolls relating have the powers to superintend, direct and con-
to a village within the area of a Panchayat. Thus trol the elections to the Panchayats and also pre-
representative democracy at the grassroots is pare the electoral rolls. The independence of the
sought to be established. Commission is ensured by laying down that he
Reservation of Seats: Article 243 D pro- can be removed in the same mannered and on
vides for reservation of seats for scheduled the same grounds as a judge of a High Court.
castes and scheduled tribes in proportion to their
population at each level. Out of the seats so re- LOCAL URBAN GOVERNMENT
served, not less than one-third of the seats shall
be reserved for women belonging to the sched- In most of the States, municipal corporations
uled castes and tribes respectively. have been established for major cities under

CLAT.indb 182 31/03/2009 11:41:54


CONSTITUTIONAL LAW 183

specific Acts of municipal legislature. But now mittee, and then passed as the Constitution (74th
a Constitutional basis has been given to local Amendment) Act, 1992. It was brought into
self-Government units in urban areas though the force on 1 June 1993 after ratification by more
74th Amendment Act. Originally introduced as than half the number of States. The one-year
the Constitution (65th Amendment) Bill in 1989 provision for the State Governments to come
by the Rajiv Gandhi Government, it did not get out with matching legislations came to an end in
the assent of the Rajya Sabha. In 1991, it was June 1994 when the Act, the 74th Constitutional
introduced in an amended form as 73rd Amend- Amendment Act introduced Part IX A, Articles
ment Bill, referred to a joint Parliamentary com- 243 P to 243 ZG in the Constitution.

CLAT.indb 183 31/03/2009 11:41:54


184 CONSTITUTIONAL LAW

MULTIPLE-CHOICE QUESTIONS
1. The Constitution of India was enacted by (c) about 2 years since 15 August 1947
a Constituent assembly set up in (d) about 3 years since 9 December 1946
(a) July 1948 (b) July 1950 10. Who was the Chairman of the Constituent
(c) July 1946 (d) August 1947 Assembly?
2. Who among the following was the Chair- (a) Jawaharlal Nehru
man of the Drafting Committee of the In- (b) Rajendra Prasad
dian Constitution? (c) B. R. Ambedkar
(a) Rajendra Prasad (d) C. Rajagopalachari
(b) Tej Bahadur Sapru 11. India became a sovereign, democratic re-
(c) C. Rajagopalachari public on
(d) B. R. Ambedkar (a) 15 August 1947
3. The Constituent assembly which framed (b) 30 January 1948
the Constitution for Independent India (c) 26 January 1950
was set up in (d) 26 November 1929
(a) 1945 (b) 1946 12. The Constitution names our country as
(c) 1947 (d) 1949 (a) Bharat
4. Who among the following was a member (b) India, that is Bharat
of the Constituent Assembly established (c) Hindustan
in July 1946? (d) Aryavarta
(a) Vallabhbhai Patel 13. A constitution is
(b) Mahatma Gandhi (a) a set of ordinary laws
(c) K. M. Munshi (b) a set of official laws
(d) Abdul Kalam Azad (c) a set of financial laws
5. What is the special Constitutional position
(d) the basic structure defining the powers
of Jammu and Kashmir?
of the state and the rights and duties of
(a) Indian laws are not applicable
the citizens
(b) It has its own Constitution
14. Who among the following is known as the
(c) It is not one of the integral parts of In-
Father of the Indian Constitution?
dian Union
(a) B. R. Ambedkar
(d) None of these
(b) Mahatma Gandhi
6. The idea of the Constitution of India was
(c) Jawaharlal Nehru
first of all given by
(d) Vallabhbhai Patel
(a) Mahatma Gandhi
(b) B. R. Ambedkar 15. Who proposed the Preamble before the
(c) Jawaharlal Nehru Drafting Committee of the Constitution?
(d) M. N. Roy (a) Jawaharlal Nehru
7. The Indian Constitution was enforced on (b) B. R. Ambedkar
(a) 30 January 1950 (c) B. N. Rau
(b) 26 January 1950 (d) Mahatma Gandhi
(c) 15 August 1947 16. When did the first linguistic state of
(d) 26 November 1949 Andhra come into existence?
8. The Constitution of India was adopted by (a) 2 October 1953
the (b) 1 October 1953
(a) Governor General (c) 1 April 1953
(b) British Parliament (d) 5 January 1953
(c) Constituent Assembly 17. Who was the first speaker of the Lok
(d) Parliament of India Sabha?
9. How long did the Constituent Assembly (a) Malgaonkar
take to finally pass the constitution? (b) P. Upendra
(a) about 6 months in 1949 (c) Anantha Sayanam Ayyanagar
(b) exactly a year since 26 November 1948 (d) Hukam Singh

CLAT.indb 184 31/03/2009 11:41:54


CONSTITUTIONAL LAW 185

18. The state of Bombay was bifurcated into (c) Vesting of residuary powers with the
Maharashtra and Gujarat on 1 May in the Centre
year (d) Distribution of powers between the
(a) 1958 (b) 1959 Centre and States
(c) 1962 (d) 1960 28. The concept of welfare state is included in
19. Which of the following Union Territories which part of the Indian Constitution?
attained statehood in February 1987? (a) The Preamble of the Constitution
(a) Goa (b) Fundamental Rights
(b) Arunachal Pradesh (c) Directive Principles of State Policy
(c) Pondicherry (d) 4th Schedule of the Constitution
(d) Daman and Diu 29. Secularism means
20. The 25th Indian state to achieve statehood (a) Suppression of all religions
was (b) Freedom of worship to minorities
(a) Goa (c) Separation of religion from State
(b) Arunachal Pradesh (d) A system of political and social phi-
(c) Mizoram losophy that does not favour any par-
(d) Sikkim ticular religious faith
21. The Constituent Assembly for undivided 30. Which of the following words were add-
India first met on ed to the ‘Preamble to the Constitution’
(a) 6 December 1946 through the 42nd amendment?
(b) 9 December 1946 (a) Sovereign (b) Socialist
(c) 20 February 1947 (c) Secular (d) Democratic
(d) 3 June 1947 31. The Constitution of India borrowed the
22. The first state to become bifurcated after Parliamentary system of Government
independence was from
(a) Bengal (b) Bombay (a) United Kingdom
(c) Punjab (d) Assam (b) France
23. When the Constituent Assembly for the (c) Switzerland
Dominion of India reassembled on 31 Oc- (d) USA
tober 1947 its reduced membership was 32. Which of the following is a feature of the
(a) 299 (b) 311 Parliamentary form of government?
(c) 319 (d) 331 (a) Executive is responsible to Judiciary
24. For the philosophy underlying our Consti- (b) Executive is responsible to Legisla-
tution, the historic ‘Objectives Resolution’ ture
was moved in the Constituent Assembly (c) Judiciary is responsible to Executive
on 22 January 1947 by (d) Legislature is responsible to Execu-
(a) Dr B. R. Ambedkar tive is responsible is responsible to
(b) Jawaharlal Nehru Executive
(c) Dr Rajendra Prasad 33. The Indian Constitution is regarded as
(d) Dr S. Radhakrishnan (a) Federal
25. The Indian Constitution closely follows (b) Unitary
the Constitutional system of (c) Parliamentary
(a) USA (b) UK (d) Federal in form and Unitary in spirit
(c) Switzerland (d) USSR 34. The Constitution of India borrowed the
26. India opted for a federal form of govern- scheme of Indian Federation from the
ment because of Constitution of
(a) vast territory (a) USA (b) Canada
(b) cultural integration (c) Ireland (d) UK
(c) linguistic and regional diversity 35. Separation of the Judiciary from the Ex-
(d) administrative convenience ecutive is enjoined by
27. Which of the following determines that (a) Preamble
the Indian Constitution is federal? (b) Directive Principle
(a) A written and rigid Constitution (c) Seventh Schedule
(b) An independent Judiciary (d) Judicial decision

CLAT.indb 185 31/03/2009 11:41:54


186 CONSTITUTIONAL LAW

36. The word ‘Economic Justice’ is found in (a) Presidential Government


(a) Preamble (b) Parliamentary Government
(b) Preamble, Fundamental Rights (c) Federal Government
(c) Preamble, Fundamental Rights, Di- (d) Independence of the Judiciary
rective Principles 44. Which of the following is not a salient fea-
(d) Preamble, Directive Principles ture of our Constitution?
37. The Preamble to the Indian Constitution (a) A Parliamentary form of government
reads (b) A Sovereign Democratic Republic
(a) We, the people of India….adopt, enact (c) Contains Directive Principles of State
and give to India this Constitution Policy
(b) We, the people of Constituent Assem- (d) A federal policy akin to the American
bly…. adopt, enact and give to India model
this Constitution 45. In a parliament form of government, the
(c) We, the citizens of India….adopt, en- real powers of the state are vested in the
act and give to ourselves this Consti- (a) Council of Ministers headed by the
tution Prime Minister
(d) We, the people of India….in our Con- (b) President
stituent Assembly…adopt, enact and (c) Government
give to ourselves this constitution (d) Parliament
38. The part of the constitution that reflects 46. India is a democratic republic, because
the mind and ideals of the framers is (a) there is independence of the judiciary
(a) Directive Principles (b) the Head of the state is elected by the
(b) Fundamental Rights people
(c) Preamble (c) there is a distribution of powers be-
(d) Citizenship tween the Centre and the States
39. The insertion of ‘Liberty, Equality and (d) there is a Parliamentary supremacy
Fraternity’ was derived from 47. The Constitution of India
(a) American revolution (a) does not provide the Judicial Review
(b) French revolution (b) provides for Judicial Review on US
(c) Russian revolution pattern
(d) None of these (c) provides for Judicial Review with a
40. The correct nomenclature of India accord- limited scope
ing to the Preamble is (d) provides for Judicial Review as its
(a) Sovereign, Secular, Democratic Re- integral part
public 48. The Preamble of our Constitution
(b) Sovereign, Democratic republic (a) is a part of the Constitution
(c) Sovereign, Socialist, Secular, Demo- (b) contains the spirit of the Constitution
cratic Republic (c) is a limitation upon the granted pow-
(d) Sovereign, Secular, Socialist Democ- er
racy (d) none of these
41. The source of India’s Sovereignty lies in 49. Which of the following is not a part of the
the Preamble to the Indian Constitution?
(a) President (a) Secularism
(b) Prime Minister (b) Socialism
(c) People of India (c) Democratic Republic
(d) Preamble to the Constitution (d) Federalism
42. Which of the following describes India as 50. Which of the following Fundamental
a Secular State? rights is also available to a foreigner on
(a) Fundamental Rights the soil of India?
(b) Ninth Schedule (a) Equality of opportunity in the matter
(c) Directive Principles of public employment
(d) Preamble to the Constitution (b) Freedom of movement, residence and
43. Which of the following is not a basic fea- profession
ture of the Indian Constitution? (c) Protection from discrimination on

CLAT.indb 186 31/03/2009 11:41:54


CONSTITUTIONAL LAW 187

grounds of only religion, race, caste (c) Prime Minister


or sex (d) Chief Justice of India
(d) Protection of life and personal liberty 60. Who appoints the Chairman of UPSC of
against action without authority of India?
law (a) President
51. Which article says that there should be a (b) Vice-President
President? (c) Prime Minister
(a) Article 54 (b) Article 53 (d) Chief Justice of India
(c) Article 52 (d) Article 51 61. Under which Article can the President
52. Who administers oath to the President? declare National Emergency?
(a) Speaker of Lok Sabha (a) Article 352 (b) Article 360
(b) Vice-President (c) Article 350 (d) Article 356
(c) Prime Minister 62. Under which Article can the President
(d) Chief Justice of India declare State Emergency?
53. What is the salary of President? (a) Article 352 (b) Article 360
(a) 45,000 (b) 50,000 (c) Article 350 (d) Article 356
(c) 30,000 (d) 1,50,000 63. Under which Article can the President
54. Who holds the office in the absence of declare Financial Emergency?
President? (a) Article 352 (b) Article 360
(a) Chief Justice of India (c) Article 350 (d) Article 356
(b) Prime Minister 64. How many times has the National Emer-
(c) Speaker of Lok Sabha gency been declared in our country?
(d) Vice President (a) Once
55. In which house of the Parliament can the (b) Thrice
proceedings of impeachment of President (c) More than 100 times
be initiated? (d) Not even once
(a) Lok Sabha 65. How many times has the Financial Emer-
(b) Rajya Sabha gency been declared in our country?
(c) Both Lok Sabha and Rajya Sabha (a) Once
(d) Either in Lok Sabha or in Rajya Sab- (b) Thrice
ha (c) More than 100 times
56. Who appoints the Attorney General of In- (d) Not even once
dia? 66. Which Article describes about the im-
(a) Chief Justice of India peachment of President?
(b) Prime Minister (a) Article 53 (b) Article 52
(c) President (c) Article 60 (d) Article 61
(d) Vice-President 67. If both the President and Vice-President
57. Who appoints the Comptroller and Audi- are not available, who performs the duties
tor General of India? of the President?
(a) President (a) Chief Justice of India
(b) Vice-President (b) Supreme Court Judge
(c) Prime Minister (c) Prime Minister
(d) Chief Justice of India (d) Governor
58. Who appoints the Chief Election Commis- 68. How many times has the Chief Justice of
sioner of India? India performed the duties of President?
(a) President (a) Once (b) Twice
(b) Vice-President (c) Thrice (d) Not even once
(c) Prime Minister 69. Which Chief Justice of India has per-
(d) Chief Justice of India formed the duties of President?
59. Who appoints the other Election Commis- (a) Justice V. N. Khare
sioners of India? (b) Justice M. M. Punchi
(a) President (c) Justice S. P. Barucha
(b) Vice-President (d) Justice M. Hidaytullah

CLAT.indb 187 31/03/2009 11:41:54


188 CONSTITUTIONAL LAW

70. Which President of India was elected un- (d) Chief Justice of the concerned High
opposed? Court
(a) N. S. Reddy 80. Which Union Territory has a representa-
(b) K. R. Narayanan tion in the Rajya Sabha?
(c) R. Venkataraman (a) Lakshdweep
(d) S. D. Sharma (b) Delhi
71. What is the salary of Vice-President of In- (c) Chandigarh
dia? (d) Andaman and Nicobar
(a) Rs 45,000 (b) Rs 30,000 81. Does the Governor have the powers to
(c) Rs 50,000 (d) Rs 1,25,000 give pardon for the death sentence?
72. Which Article says that there should be a (a) Yes
Vice-President? (b) No
(a) Article 63 (b) Article 62 (c) With the advice of the President
(c) Article 60 (d) Article 61 (d) With the advice of the Chief Minister
73. Which Article says that there should be a of the state concerned
Governor? 82. How many members in the Rajya Sabha
(a) Article 163 (b) Article 153 are nominated?
(c) Article 160 (d) Article 161 (a) 12 members
74. What is the salary of Governor of a State? (b) 10 members
(a) Rs 45,000 (b) Rs 30,000 (c) 2 members
(c) Rs 1,10,000 (d) Rs 80,000 (d) no member is nominated
75. What is the quorum to constitute a meet- 83. Who is the ex-officio chairman of Plan-
ing of either House of the Parliament? ning Commission?
(a) One tenth of the total members of the (a) Planning Minister
House (b) President
(b) One eighth of the total members of the (c) Prime Minister
House (d) Vice-President
(c) One fourth of the total members of the 84. Who is the chairman of National Integra-
House tion Council?
(d) One sixth of the total members of the (a) Finance Minister
House (b) President
76. Chandigarh comes under which High (c) Prime Minister
Court? (d) Vice-President
(a) Punjab and Haryana 85. Who is the chairman of Inter-State Coun-
(b) Haryana cil?
(c) Chandigarh (a) Planning Minister
(d) Punjab (b) President
77. Who administers oath to the Governor? (c) Prime Minister
(a) President (d) Vice-President
(b) Chief Minister 86. Who is known as the keystone of the Cabi-
(c) Chief Justice of India net arch?
(d) Chief Justice of the concerned High (a) Planning Minister
Court (b) President
78. Who appoints the Advocate General? (c) Prime Minister
(a) President (d) Vice-President
(b) Governor 87. Who is known as Primus-Inter Pares?
(c) Chief Minister (a) Planning Minister
(d) Chief Justice of the concerned High (b) President
Court (c) Prime Minister
79. Who appoints the Chairman of State Pub- (d) Vice-President
lic Service Commission? 88. How many finance commissions have
(a) President been constituted till now?
(b) Governor (a) 13 (b) 12
(c) Chief Minister (c) 10 (d) 11

CLAT.indb 188 31/03/2009 11:41:54


CONSTITUTIONAL LAW 189

89. Which organ of the Constitution is known (c) Fraternity


as the ‘Fifth wheel of the Coach’? (d) Justice
(a) Planning Commission 102. Protection of the interests of the minority
(b) Finance Commission is envisaged in which of the following ar-
(c) Public Service Commission ticles?
(d) None of the above (a) 14 (b) 19
90. By which constitutional amendment (c) 29 (d) 32
Panchayati Raj was introduced? 103. The Fundamental Right that aims at the
(a) 74th (b) 73rd abolition of social distinctions is the right
(c) 42nd (d) 44th (a) to property
91. From which country’s Constitution the (b) against exploitation
Right to Equality have been borrowed? (c) to equality
(a) Irish (b) Canada (d) to freedom
(c) British (d) US 104. On whom does the Constitution confer
92. From which country’s Constitution the Ju- special responsibility for the enforcement
dicial Review have been borrowed? of Fundamental Rights?
(a) Irish (b) Canada (a) Parliament (b) Supreme Court
(c) British (d) US (c) President (d) State Legislature
93. Which Article defines Equality before law 105. In the Constitution, opportunities for the
and equal protection of law? development of scientific temper, human-
(a) Article 16 (b) Article 14 isms and spirit of inquiry and reform are
(c) Article 20 (d) Article 21 found in
94. Which Articles defines Protection of life (a) Fundamental Rights
and Personal liberty? (b) Preamble
(a) Article 16 (b) Article 14 (c) Fundamental Duties
(c) Article 20 (d) Article 21 (d) Directive Principles
95. Which Article defines Judicial Review? 106. Under which Article of the Constitution
(a) Article 16 (b) Article 13 are the Cultural and Educational Rights
(c) Article 20 (d) Article 21 granted?
96. Which Article defines Protection in re- (a) Article 29 and 31
spect of conviction for offences? (b) Article 29 and 32
(a) Article 16 (b) Article 14 (c) Article 29 and 30
(c) Article 20 (d) Article 21 (d) Article 30 and 31
97. Which Article defines Prohibition of dis- 107. Which of the following is not a Directive
crimination only on grounds of religion, Principle of the State Policy?
race, caste, sex or place of birth? (a) To raise the level of nutrition
(a) Article 16 (b) Article 14 (b) To develop the scientific temper
(c) Article 15 (d) Article 21 (c) To promote economic interests of
98. What are the Articles which defines Direc- weaker sections
tive Principles of State Policy? (d) To separate the Judiciary from the Ex-
(a) 36 to 51 (b) 36 to 51-A ecutive
(c) 39 to 51 (d) 39 to 51-A 108. Which of the following is not a Funda-
99. Which Article says for Uniform Civil mental Right?
Code? (a) Right to Equality
(a) Article 43 (b) Article 40 (b) Right against Exploitation
(c) Article 44 (d) Article 45 (c) Right to Property
100. Which part of Constitution defines Funda- (d) Right to Freedom of Religion
mental Duties? 109. Which part of the constitution deals with
(a) Part I (b) Part II the Directive Principles of State Policy?
(c) Part IV (d) Part IV-A (a) Part III (b) Part IV
101. The Preamble of our Constitution includes (c) Part V (d) Part II
all except 110. Protection and interests of the minorities
(a) Adult franchise is envisaged in which Article of the Con-
(b) Equality of status stitution?

CLAT.indb 189 31/03/2009 11:41:54


190 CONSTITUTIONAL LAW

(a) Article 14 (b) Article 19 (a) Article 22 (b) Article 23


(c) Article 29 (d) Article 32 (c) Article 24 (d) Article 25
111. Which of the following is not specifically 119. The Right to Property is a
mentioned in Article 19 of the Constitu- (a) Legal Right
tion? (b) Fundamental Right
(a) Freedom of speech and expression (c) Free Right
(b) Freedom to assemble peacefully with- (d) Universal Right
out arms 120. Every person who is arrested or detained
(c) Freedom to move freely is required to be produced before the near-
(d) Freedom to the press est magistrate within a period of
112. Right to Constitutional Remedies comes (a) 24 hours including the time necessary
under for journey
(a) Fundamental Rights (b) 48 hours including the time necessary
(b) Legal rights for journey
(c) Constitutional Rights (c) 72 hours including the time necessary
(d) Natural Rights for journey
113. Right to Property was removed from the (d) None of these
last of Fundamental Rights by which 121. If the person is refused freedom of move-
amendment? ment, it means denial of
(a) 14th (b) 42nd (a) Civil Liberty
(c) 39th (d) 44th (b) Economic Liberty
114. Which of the following Directive Prin- (c) National Liberty
ciples is based on Gandhian ideology? (d) Political Liberty
(a) Equal pay for equal work for both
122. Which one of the following fundamental
men and women
rights has been described by Dr Ambed-
(b) Uniform civil code for all citizens
kar as the ‘heart and soul of the Constitu-
(c) Separation of Judiciary from Execu-
tion’?
tive
(a) Right to Equality
(d) Promotion of cottage industries
(b) Right to Constitution Remedies
115. Which of the following is Fundamental
(c) Right to Freedom
duty of an Indian citizen?
(d) Right against Exploitation
(a) To cast his vote during election
123. In Indian Constitution, there is no provi-
(b) To develop scientific temper and spirit
of inquiry sion for
(c) To work for removal of illiteracy (a) Religious Rights
(d) To honour the elected leaders. (b) Political and Social Rights
116. Untouchability is associated with (c) Education Rights
(a) Social inequality (d) Economic Rights
(b) Political inequality 124. Chapter III on the Fundamental Rights
(c) Economic inequality contains twenty-four articles from
(d) Religious inequality (a) Article 12 to 35
117. Which of the following is a right against (b) Article 14 to 37
exploitation? (c) Article 10 to 33
(a) Prohibition of discrimination on (d) Article 16 to 19
grounds of religion. 125. What is the minimum permissible age for
(b) Protection in respect of conviction for employment in any factory or mine?
offences. (a) 12 years (b) 14 years
(c) Prohibition of employment of chil- (c) 16 years (d) 18 years
dren in factories and hazardous em- 126. Which of the following is not a Funda-
ployment. mental Right?
(d) Protection of language, script or cul- (a) Right to strike
ture of minorities. (b) Right against exploitation
118. The prohibition of traffic in human beings (c) Right to equality
and ‘beggar’ comes under (d) Right to freedom of religion

CLAT.indb 190 31/03/2009 11:41:55


CONSTITUTIONAL LAW 191

127. Prohibition of discrimination on grounds (c) Right to Equality


of religion etc., (Article 15 of the Con- (d) Right to constitutional Remedies
stitution of India) is a Fundamental right 135. The writ of Prohibition is issued by the
classifiable under Supreme court or a High Court against
(a) the Right to Freedom of Religion (a) judicial or quasi judicial authorities
(b) the Right against exploitation (b) administrative and judicial authorities
(c) the Cultural and Education Rights (c) administrative authorities and private
(d) the Right to Equality individuals
128. The total number of Fundamental duties (d) administrative authorities and govern-
mentioned in the Constitution is ment
(a) 9 (b) 10 136. Total number of members in a Legislative
(c) 11 (d) 12 Council should not exceed that of a Legis-
129. Fundamental Duties were incorporated in lative Assembly by
the Indian Constitution in (a) 1/3 (b) 2/3
(a) 1971 (b) 1972 (c) 1/2 (d) 1/4
(c) 1975 (d) 1976 137. For those Union Territories, which have
130. Which state government has prohibited no Legislative Councils of their own, laws
use of the word ‘Harijan’ and replaced it are passed by
with ‘Anusuchit Jati’ in official work? (a) Union Ministry
(a) Karnataka (b) President
(b) Jammu and Kashmir (c) Parliament
(c) Rajasthan (d) Appointed Administrator
(d) Uttar Pradesh 138. The Rajya Sabha can be dissolved by
131. Any dispute regarding the violation of (a) Lok Sabha
Fundamental Rights can be presented (b) Constitutional amendment
(a) in the Supreme Court only (c) President
(b) in the High Court only (d) None of these
(c) either in the Supreme Court or in the 139. The first woman film star nominated/elect-
High Court ed to the Rajya Sabha was
(d) first in the Supreme Court and then in (a) Nargis Dutt
(b) Vyjayanthimala
the High Court
(c) Hema Malini
132. Which of the following signifies an im-
(d) Jayalalitha
portant distinction between fundamental
140. Which of the following bills cannot be in-
Rights and Directive Principles?
troduced first in the Rajya Sabha?
(a) While the former are rights of citizens,
(a) Money Bills
the latter are duties of the state.
(b) Bills pertaining to the removal of the
(b) While the former relate largely to
President by impeachment
civic aspects, the latter relate to socio-
(c) Bills bringing a state under the Presi-
economic and international aspects.
dent’s Rule
(c) While the former relate to the present,
(d) Bills pertaining to the powers of the
the latter are not so timebound.
Supreme Court or State Judiciary
(d) While the former are judicially en- 141. Which of the following best defines the
forceable the latter are not so enforce- Parliamentary term ‘Crossing the Floor’?
able. (a) Leaving the opposition to join the
133. Article 17 of the Constitution party in power or vice versa by a Par-
(a) protects individual freedom liamentarian
(b) grants right to equality (b) An attempt to occupy the seat of some
(c) abolishes untouchability other Parliamentarian
(d) grants right to Constitutional rem- (c) Leaving a House by minister in be-
edies tween in order to attend the other
134. Contempt of Court places restriction on House
which fundamental Right? (d) Walk out by some Parliamentarians
(a) Right to Freedom in order to boycott the proceedings of
(b) Right against exploitation the House

CLAT.indb 191 31/03/2009 11:41:55


192 CONSTITUTIONAL LAW

142. An ordinance promulgated by the Presi- (b) when matters of utmost importance
dent are raised
(a) will lapse automatically after 2 (c) interval between the morning and af-
months ternoon sessions
(b) will lapse on the expiration of 6 weeks (d) when a Money Bill is introduced in
from the meeting of the Parliament the Lok Sabha
(c) will automatically become a law after 149. The Deputy Chairman of the Rajya Sabha
6 months can be removed by a resolution
(d) will continue to be in force till it is su- (a) passed by 2/3rd majority of its total
perseded by an Act of the Parliament members present
143. The President can promulgate an ordi- (b) passed by a simple majority of its total
nance members present
(a) when the Lok Sabha has been dis- (c) moved by Rajya Sabha but passed by
solved Lok Sabha
(b) when there is conflict between the two (d) none of these
Houses regarding a particular bill 150. The maximum strength of Lok Sabha and
(c) when a bill passed by the Lok Sabha is Rajya Sabha respectively is
not passed by the Rajya Sabha (a) 552 and 250 (b) 525 and 238
(d) when both the Houses of the Parlia- (c) 537 and 275 (d) 545 and 250
ment are not in session 151. Which part of Constitution defines Funda-
144. If the Minister of a State wants to resign, mental Rights?
to whom should he address the letter of (a) Part I (b) Part II
resignation? (c) Part IV (d) Part III
(a) Chief Minister 152. Which part of the Constitution defines Di-
(b) Speaker of Vidhan Sabha rective Principles of State Policy?
(c) Governor of the State (a) Part III (b) Part IV
(d) Leader of his political party (c) Part I (d) Part II
145. To which of the following Bills, the Presi- 153. From which country’s Constitution the
dent must accord his sanction without Independence of Judiciary have been bor-
sending it back for fresh consideration? rowed?
(a) Ordinary Bills (a) Germany (b) British
(b) Finance Bills (c) US (d) Canada
(c) Bills passed by both the Houses of the 154. From which country’s Constitution the
Parliament Rule of Law have been borrowed?
(d) Bill seeking Amendment to the Con- (a) Germany (b) British
stitution. (c) US (d) Canada
146. In the case of a conflict between the Centre 155. From which country’s Constitution the
and State in respect of a subject included Distribution of Powers between Union
in the Concurrent List and the State have been borrowed?
(a) the State Law prevails (a) Germany (b) British
(b) the Union Law prevails (c) US (d) Canada
(c) the matter is resolved by the Supreme 156. India adopted a federal system with a
Court strong center from
(d) the law which had been passed first (a) New Zealand (b) Australia
would prevail (c) USA (d) Canada
147. Who is the highest Law Officer of a 157. The emergency provisions of the Consti-
state? tution of India were greatly influenced by
(a) Attorney General (a) the Constitution of Canada
(b) Advocate General (b) the Constitution of the United States
(c) Solicitor General (c) the Government of India Act, 1939
(d) Secretary General Law Department (d) the Weimar Constitution of Germany
148. What is ‘zero hour’? 158. The Council of Ministers has to tender its
(a) When the proposals of the opposition resignation if a vote of no-confidence is
are considered passed against it

CLAT.indb 192 31/03/2009 11:41:55


CONSTITUTIONAL LAW 193

(a) by the Lok Sabha (b) the Constitution is supreme


(b) by the Rajya Sabha (c) the Executive is supreme
(c) by the two houses at a joint sitting (d) the Legislature is supreme
(d) by the two houses of Parliament by 167. Which of the following amendments cur-
two-thirds majority tailed the Supreme Court’s power of Judi-
159. Who decides whether a bill is a Money cial Review?
Bill or not? (a) 29th Amendment
(a) President (b) 44th Amendment
(b) Prime Minister (c) 42nd Amendment
(c) Speaker of Lok Sabha (d) None of the above
(d) Select Parliamentary Committee 168. How many judges of the Supreme Court
160. The maximum strength of the nominated have been removed from their office
members in both the houses of the Parlia- before the expiry of their normal term
through impeachment?
ment can be
(a) only one (b) two
(a) 20 (b) 14
(c) three (d) none
(c) 10 (d) 12
169. Which article of the Constitution permits
161. Who among the following was the first
the Supreme Court to review its own
Speaker of the Lok Sabha? judgement or order?
(a) G. N. Mavalankar (a) Article 139 (b) Article 138
(b) G. S. Dhillon (c) Article 130 (d) Article 137
(c) Hukum Singh 170. The Fundamental Duties of Indian citizens
(d) Ananthaswayanam Ayenger were incorporated in the Constitution in
162. The Speaker of the Lok Sabha has to ad- (a) 1981 (b) 1799
dress his letter of resignation to (c) 1952 (d) 1976
(a) the Vice-President 171. Who acted as the Chairman of the Draft-
(b) the Prime Minister ing Committee of the Constituent Assem-
(c) the President bly?
(d) the Deputy Speaker of the Lok Sabha (a) Vallabhbhai Patel
163. The Constitution of India is (b) B.R. Ambedkar
(a) very rigid (c) Dr. Rajendra Prasad
(b) flexible (d) Jawaharlal Nehru
(c) rigid 172. The Rajya Sabha is a permanent house
(d) partly rigid and partly flexible but
164. The Supreme Court of India was set up (a) half of its members retire every two
(a) under the Government of India Act, years
1935 (b) one-fifth of its members retire every
(b) through an Act of Parliament in 1950 year
(c) by the Constitution (c) one-third of its members retire every
(d) under the Indian Independence Act, two years
1947 (d) half of its members retire every three
165. The judges of the Supreme Court years
(a) cannot be removed from office during 173. What can be the maximum gap between
their tenure the two sessions of Parliament?
(a) nine months (b) six months
(b) can be removed from office by the
(c) three months (d) four months
President at his discretion
174. The Chairman of the Rajya Sabha
(c) can be removed by the President on
(a) is nominated by the President
the recommendation of the Parlia-
(b) is elected by the members of Rajya
ment Sabha
(d) can be removed by the President on (c) is elected by the members of Lok Sab-
the recommendation of the Council of ha and Rajya Sabha jointly
Ministers (d) none of the above
166. In India, power of Judicial Review is re- 175. The Chairman of the Rajya Sabha has
stricted because (a) two votes—an ordinary vote and cast-
(a) the judges are transferable
ing vote

CLAT.indb 193 31/03/2009 11:41:55


194 CONSTITUTIONAL LAW

(b) no vote because he is an ex-officio (b) The President


Chairman of the House (c) The Speaker of Lok Sabha
(c) a vote only in case of a tie (d) The Parliament
(d) a vote like any other member of the 184. India is a Secular State because
Rajya Sabha (a) state has no religion of its own
176. The power to control the expenditure of (b) all religions are equally protected
the Government of India rests exclusively (c) there will be no discrimination on
with grounds of religion or caste
(a) the Union Finance Minister (d) all the above
(b) the Comptroller and auditor General 185. At present, India consists of
(c) the Parliament (a) 21 states and 11 union territories
(d) the President (b) 28 states and 7 union territories
177. Who among the following is considered as (c) 25 states and 9 union territories
the custodian of Lok Sabha? (d) 24 states and 7 union territories
(a) the Prime Minister 186. In which year were the states reorganized
(b) the Speaker on linguistic basis?
(c) the Chief Whip of the ruling party (a) 1966 (b) 1956
(d) the leader of the Opposition (c) 1951 (d) 1947
178. Which of the following committees of 187. The first commission appointed by the
Parliament is concerned with the regular- Government in 1948 to examine the case
ity and economy of expenditure? for the reorganization of states on linguis-
(a) Estimates Committee tic basis was headed by
(b) Committee of Public Undertakings (a) Justice S. K. Dhar
(c) Public Accounts Committee
(b) Justice Wanchoo
(d) All the above
(c) Justice M. C. Mahajan
179. Which of the following parliamentary
(d) none of the above
committee scrutinies the report of the
188. Which state enjoys the distinction of being
Comptroller and Auditor-General of In-
the first linguistic state of India?
dia?
(a) Punjab
(a) Estimates Committee
(b) Tamil Nadu
(b) Committee of Public Undertakings
(c) West Bengal
(c) Public Accounts Committee
(d) Andhra Pradesh
(d) All the above
180. Lok Sabha is superior to the Rajya Sabha 189. The administrators of Union Territories
because are designated as
(a) it alone controls the purse (a) Administrators
(b) it is directly elected (b) Lieutenant Governor
(c) it can cast the Council of Ministers (c) Chief Commissioners
through a vote of no-confidence (d) all these
(d) of all the above reasons 190. In which of the following points is the In-
181. The structure of the Indian Constitution is dian Constitution similar to that of USA?
(a) Purely Federal (a) Rigid Constitution
(b) Rigid (b) Rule of Law
(c) Unitary (c) Fundamental Rights
(d) Federal in form and Unitary in spirit (d) Directive Principles of State Policy
182. The number of Articles in the Indian Con- 191. The greatest impact on the Constitution of
stitution is India was left by
(a) 395 (a) The Constitution of Canada
(b) over 440 (b) The Constitution of UK
(c) less than 350 (c) The Government of India Act, 1935
(d) more than 400 (d) The Constitution of USA
183. Who is the final authority to expound the 192. Disputes regarding the election of the
meaning of Constitution? Vice-President are decided by
(a) The Supreme Court (a) the Supreme Court

CLAT.indb 194 31/03/2009 11:41:55


CONSTITUTIONAL LAW 195

(b) the Parliament 201. Anglo Indian representatives in the Lok


(c) the Speaker Sabha are nominated in terms of the Ar-
(d) the President ticle
193. The members of the Council of Ministers (a) 331 (b) 80
are (c) 370 (d) 333
(a) appointed by the President at his dis- 202. Who among the following fixes the sala-
cretion ries and the allowances of the Speaker of
(b) appointed by the President on the ad- Lok Sabha?
vice of the Prime Minister (a) President
(c) appointed by the President on the rec- (b) Council of Minister
ommendation of the Parliament (c) Cabinet
(d) appointed by the Prime Minister (d) Parliament
194. The Council of Ministers is collectively 203. Which of the following is also called the
‘House of Elders’?
responsible to
(a) Rajya Sabha (b) Lok Sabha
(a) the Rajya Sabha
(c) Ram Sabha (d) Vidhan Sabha
(b) the Prime Minister
204. The Rajya Sabha was first constituted on
(c) the President of India
(a) 13 May 1952 (b) 2 April 1954
(d) the Parliament
(c) 3 April 1952 (d) 1 January 1951
195. Who presides over the meetings of the 205. The Legislative Council in a state can be
Council of Ministers? created or disbanded by the
(a) Cabinet Secretary (a) State Legislative Assembly alone
(b) the President (b) Parliament alone
(c) the Prime Minister (c) Parliament on recommendation on the
(d) Different ministers by rotation State Legislature
196. The Prime Minster is (d) President on recommendation of the
(a) the head of the state as well as govern- Governor
ment 206. The maximum number of Anglo-Indians
(b) the head of the state that can be nominated to the Lok Sabha
(c) the head of the government are
(d) none of the above (a) 3 (b) 2
197. The Prime Minister is (c) 5 (d) 4
(a) appointed by the President in consul- 207. In case of disagreement on a Bill in the
tation with the Speaker two Houses of Parliament,
(b) appointed by the President (a) a special parliamentary Committee is
(c) elected by the Lok Sabha formed to resolve the situation
(d) elected by the two Houses of Parlia- (b) the Prime Minister intervenes
ment at a joint sitting (c) the President casts a deciding vote
198. Which of the following is the chief source (d) a joint sitting of the two Houses of
of political power in India? Parliament is convened
(a) The people 208. What is the maximum number of elected
(b) The Parliament members in a State Assembly?
(c) The Constitution (a) 250 (b) 300
(d) The Parliament and State Legislatures (c) 600 (d) 500
209. Which of the following is not correct re-
jointly
garding a Money Bill?
199. Which part of the Constitution reflects the
(a) It can be introduced in either House of
mind and ideals of the framers?
Parliament
(a) Emergency Provisions
(b) It can be passed by the Lok Sabha
(b) Preamble
even if the Rajya Sabha rejects it.
(c) Fundamental Rights (c) It is deemed to have been passed by
(d) Directive Principles both Houses if it is not returned by
200. The name of the Union is India or Rajya Sabha within 14 days
(a) Bharatvarsha (b) Hindustan (d) It requires the prior approval of the
(c) Bharat (d) None of these President

CLAT.indb 195 31/03/2009 11:41:55


196 CONSTITUTIONAL LAW

210. Who presides over the Lok Sabha if nei- 218. Joint sittings of the two Houses of Parlia-
ther the Speaker nor the Deputy Speaker ment are held to
is available? (a) elect the President of India
(a) A member of the House of People ap- (b) elect the Vice President of India
pointed by the President (c) adopt a Constitution amending Bill
(b) A member chosen by Council of Min- (d) consider and pass a bill on which two
isters Houses disagree
(c) The senior most member of the Rajya 219. One third of the members of the Rajya
Sabha Sabha retire after every
(d) The senior most member of the Lok (a) one year (b) second year
Sabha (c) third year (d) fifth year
211. Who is the ex-officio Chairman of Rajya 220. A money bill passed by the Lok Sabha has
Sabha? to be passed by Rajya Sabha within
(a) President (a) 14 days (b) 21 days
(b) Vice-President (c) 1 month (d) 3 months
(c) Minister of Parliamentary Affairs 221. Parliament of India is composed of
(d) Leader of Opposition (a) Lok Sabha only
212. When a bill passed by Parliament is sent to (b) Rajya Sabha only
the President for his assent, which option (c) Lok Sabha and Rajya Sabha
is not true to him? (d) Lok Sabha, Rajya Sabha and Presi-
(a) He may sign it dent
(b) He may decline to sign it 222. Maximum number of M.P.s from back-
(c) He may return it to Parliament for re- ward communities are elected from which
consideration state?
(d) He may amend it (a) Bihar
213. The Union Territories get representation (b) Madhya Pradesh
in (c) Nagaland
(a) Lok Sabha (d) Rajasthan
(b) Rajya Sabha 223. In Parliamentary government, Ministers
(c) both Houses of Parliament remain in office as long as they enjoy
(d) None of these (a) confidence of the upper house of the
214. How many members of the Rajya Sabha legislature
are nominated by the President of India? (b) support of the armed forces
(a) 10 (b) 11 (c) confidence of the popular chamber of
(c) 12 (d) 13 legislature
215. In the case of a deadlock between the two (d) popular support
Houses of the Parliament, the joint sitting 224. A bill for alteration of boundaries of state
is presided over by the should not be introduced in the Parliament
(a) President without the recommendation of
(b) Vice-President (a) the presiding officers of both Houses
(c) Speaker of Lok Sabha of Parliament
(d) Member of the Lok Sabha specifically (b) the legislatures of the states con-
elected for the purpose cerned
216. The Chairman of the Rajya Sabha is (c) Supreme Court
(a) elected by the members of Rajya Sab- (d) President
ha 225. Which of the following states has the larg-
(b) nominated by the President est representation in the Lok Sabha?
(c) elected by members of both Houses of (a) Bihar
Parliament (b) Maharashtra
(d) elected by Parliament and State Legis- (c) Madhya Pradesh
latures jointly (d) Uttar Pradesh
217. The time gas between two sessions of the 226. Which of the following non-members of
Parliament should not exceed Parliament has the right to address it?
(a) 3 months (b) 6 months (a) Solicitor-General of India
(c) 9 months (d) 1 year (b) Chief Justice of India

CLAT.indb 196 31/03/2009 11:41:55


CONSTITUTIONAL LAW 197

(c) Attorney General of India (a) 2 and 4 (b) 1, 2, 3 and 4


(d) Chief Election Commissioner (c) 1, 2 and 3 (d) 1 and 4
227. How many members of the Lok Sabha 235. Parliament or a State Legislature can de-
must support a motion of ‘no confidence’ clare a seat vacant if a member absents
in the government, before it can be admit- himself without permission form the ses-
ted by the Speaker? sions for
(a) 20 (b) 25 (a) 30 days (b) 60 days
(c) 35 (d) 55 (c) 90 days (d) 120 days
228. The Constitution of India does not men- 236. Who presides over the meetings of the Ra-
tion the post of jya Sabha?
(a) the Deputy Chairman of the Rajya (a) President
Sabha (b) Vice President
(b) the Deputy Prime Minister (c) Prime Minister
(c) the Deputy Speaker of the Lok Sabha (d) Speaker
(d) the Deputy Speaker of the State Leg- 237. The Rajya Sabha has a life of
islative Assemblies (a) 2 years (b) 6 years
229. Who administers the oath of office to the (c) 7 years (d) Permanency
President? 238. The term of Lok Sabha is normally
(a) Chief Justice of India (a) 4 years (b) 5 years
(b) Speaker of Lok Sabha (c) 6 years (d) 2 years
(c) Prime Minister 239. The Constitution provides for the nomina-
(d) Vice President tion to the Lok Sabha two members be-
230. The members of the State Legislative As- longing to the community of
sembly are elected for what period? (a) Syrian Christians
(a) 2 years (b) 6 years (b) Anglo-Indians
(c) 4 years (d) 5 years (c) Parsis
231. Money Bill can be introduced in (d) Scheduled Castes
(a) Lok Sabha only 240. The legislative powers are vested in the
(b) Rajya Sabha only (a) President
(c) Either Lok Sabha or Rajya Sabha (b) Parliament
(d) None of these (c) Prime Minister
232. Who among the following decides wheth- (d) Governor
er particular bill is a Money Bill or not? 241. Which are the two States (other than UP)
(a) President having maximum representation in Lok
(b) Speaker of Lok Sabha Sabha?
(c) Chairman of Rajya Sabha (a) Bihar and Madhya Pradesh
(d) Finance Minister (b) Bihar and Maharashtra
233. Which of the following states does not (c) Bihar and Andhar Pradesh
have a bicameral legislature? (d) Bihar and Tamil Nadu
(a) Bihar (b) Karnataka 242. The term of the Lok Sabha
(c) Rajasthan (d) Maharashtra (a) cannot be extended
234. Which of the following is true regarding (b) can be extended for another full term
the ‘No Confidence Motion’ in the Parlia- of 5 years
ment? (c) can be extended for an unlimited pe-
1. There is no mention of it in the Con- riod
stitution (d) can be extended by 1 year at a time
2. A period of 6 months must lapse be- 243. How many times can the President of In-
tween the introduction of one No dia return a non-money Bill, passed by
Confidence Motion and another parliament?
3. At least 100 persons must support (a) Twice (b) Once
such motion before it is introduced in (c) Thrice (d) Never
the House 244. Which of the following can be abolished,
4. It can be introduced in the Lok Sabha but not dissolved?
only (a) Rajya Sabha

CLAT.indb 197 31/03/2009 11:41:55


198 CONSTITUTIONAL LAW

(b) Municipal Bodies (d) Parliament can amend them by special


(c) State Legislative Council majority
(d) None of the above 253. Who is empowered to suspend the opera-
245. The distribution of seats of the Parliament tion of Fundamental Rights?
are based on which of the following cen- (a) Prime Minister
sus? (b) Parliament
(a) 1951 (b) 1961 (c) Supreme Court
(c) 1971 (d) 1981 (d) President
246. If the Speaker of the Lower House of a 254. Which of the following does not constitute
state wants to resign, his letter of resigna- an exception to the provision of Article 14
tion is to be addressed to the (‘equally before the law’)?
(a) Chief Minister (a) The foreign diplomats
(b) Deputy Speaker (b) The President
(c) Governor (c) The Prime Minister
(d) President (d) The Governor
247. The Governor of a State is appointed by 255. Which Fundamental Rights cannot be sus-
the President on the advice of the pended even during an emergency under
(a) Chief Minister article 352 of the Constitution?
(b) Chief Justice (a) Right to constitutional remedies
(c) Prime Minister (b) Right to life and liberty
(d) Vice-President (c) Right to freedoms of speech and ex-
248. The President gives his resignation to the pression
(a) Prime Minister (d) Right to equality
(b) Vice-President 256. The right against exploitation prohibits
(c) Chief Justice (a) beggar
(d) Parliament (b) traffic in human beings
249. The second President of the Indian Repub- (c) employment of children below 14
lic was years of age in factories, mines etc.
(a) V. V. Giri (d) all the above
(b) S. Radhakrishnan 257. Which of the following Fundamental
(c) Zakir Hussain Rights is restrained by the Preventive De-
(d) Rajendra Prasad tention Act?
250. For what period does the Vice President of (a) right to equality
India hold office? (b) right to freedom
(a) 5 years (c) right to religion
(b) 6 years (d) right to constitutional remedies
(c) 2 years 258. The Government of India introduced
(d) till the age of 65 years Bharat Ratna and Padam Shri awards un-
251. In which case did the Supreme Court give der
a ruling that the Preamble was a part of the (a) Article 25 of the Constitution
Constitution? (b) Article 18 of the Constitution
(a) Berubari case (c) Article 14 of the Constitution
(b) Golak Nath case (d) None of the above
(c) Keshavananda Bharati Case 259. Under the provisions of which Article of
(d) All the above cases the Constitution the government abolished
252. Can Parliament amend or modify any of the practice of untouchability?
the Fundamental Rights given in the Con- (a) Article 20 (b) Article 18
stitution? (c) Article 16 (d) Article 17
(a) Only the President can issue direc- 260. The Sikhs in India are permitted to carry
tions to amend them Kirpan. Under which one the following
(b) Parliament cannot amend any funda- Fundamental Rights are they permitted to
mental rights do so?
(c) Parliament can do so only after a free- (a) right to freedom
dom (b) right to freedom of religion

CLAT.indb 198 31/03/2009 11:41:55


CONSTITUTIONAL LAW 199

(c) right to life and liberty (c) the Union Defence Minister
(d) none of the above (d) none of the above
261. At present, right to property is a 270. For the first time, the President made a
(a) Natural Right Proclamation of Emergency under Article
(b) Legal Right 352 in
(c) Fundamental Right (a) 1971 (b) 1965
(d) Human Right (c) 1952 (d) 1962
262. Under the Indian Constitution, the respon- 271. The President made a Proclamation of
sibility for the enforcement of Fundamen- Emergency on grounds of internal distur-
tal Rights rests with bances for the first time in
(a) the Supreme Court (a) 1975 (b) 162
(b) all the Courts (c) 1965 (d) none of the above
(c) the High Courts 272. When a bill passed by the Parliament it is
(d) both (a) and (c) sent to the President, he can
263. A person can move the Supreme Court (a) consult the Speaker of Lok Sabha
directly in the event of violation of Funda- (b) amend the bill
mental Rights under (c) return it for reconsideration
(a) Article 34 (b) Article 19 (d) refuse to sign it
(c) Article 32 (d) none of the above 273. The value of a vote of a member of Parlia-
264. For the enforcement of Fundamental ment for election of the President of India
Rights, the courts can issue is determined by dividing the
(a) a notification (b) a writ (a) total population of the country as per
(c) an ordinance (d) a decree the latest census by the number of Lok
Sabha members
265. Which Fundamental Right of the Indian
(b) population of country as per the latest
Constitution prohibits traffic in human be-
census by the total strength of the two
ings?
houses of Parliament
(a) right to freedom
(c) the total value of votes of members of
(b) right to equality
all the state legislative assemblies by
(c) right against exploitation
the elected members of the two hous-
(d) none of the above
es of Parliament
266. Which one the following rights was de-
(d) none of the above
scribed by B. R. Ambedkar as ‘the heart
274. The tenure of the Vice-President is
and soul of the Constitution’? (a) six years
(a) Right to equality (b) five years
(b) Right to property (c) co-terminus with that of the President
(c) Right to freedom of religion (d) dependent on the will of the President
(d) Right to constitutional remedies 275. The Vice-President discharges the duties
267. Who of the following is regarded as the of the President in the event of
architect of the Indian Constitution? (a) his death
(a) Rajendra Prasad (b) his resignation
(b) B. N. Rao (c) his absence due to illness
(c) Jawahar Lal Nehru (d) all the above situations
(d) B. R. Ambedkar 276. The parliamentary system of government
268. The Constituent Assembly of India takes in India is based on the patter of parlia-
all decisions by mentary government existing in
(a) consensus (a) Britain
(b) simple majority (b) Canada
(c) two-thirds majority (c) France
(d) all the above methods (d) all the above countries
269. Who is the Supreme Commander of the 277. The Constitution of India describes India
armed forces of India? as
(a) the President (a) a union of states
(b) the Prime Minister (b) quasi-federal

CLAT.indb 199 31/03/2009 11:41:56


200 CONSTITUTIONAL LAW

(c) a federation of states and union terri- (c) the Chief Minister
tories (d) none of the above
(d) partly unitary and partly federal 287. The Governor of a state is
278. The Preamble to the Constitution declares (a) appointed by a Special Committee
India constituted for this purpose
(a) a Sovereign Socialist, Secular, Demo- (b) appointed by the President
cratic Republic (c) elected by the State legislature
(b) a Socialist, Democratic Republic (d) nominated by the President
(c) a Sovereign, Democratic Republic 288. Who administers the oath of office to the
(d) none of these members of council of ministers in the
279. The Preamble was proposed before the state?
Drafting Committee by (a) the Attorney-General of the state
(a) Sardar Patel (b) the Governor
(b) B. N. Rao (c) the Chief Justice of the State High
(c) Jawaharlal Nehru Court
(d) B. R. Ambedkar (d) none of the above
280. The words ‘socialist secular’ and ‘the unity 289. The basic condition imposed on the citi-
and integrity of the nation’ were added to zen’s right to assemble is that the assem-
the Constitution by the ….. Amendment bly should be
(a) 52nd (b) 44th (a) peaceful
(c) 42nd (d) none of the above (b) non-violent
281. How many times has the Preamble of the (c) peaceful and unarmed
Indian Constitution been amended so far? (d) for constructive aims
(a) thrice (b) twice 290. The Indian Constitution declares that pro-
(c) once (d) never
tection of life and liberty
282. The Preamble was for the first time
(a) can be taken away during the Emer-
amended by the
gency through President order
(a) 24th Amendment
(b) can be taken away only according to
(b) 42nd Amendment
procedure established by law
(c) 44th Amendment
(c) can never be taken away in any condi-
(d) none of the above
tion
283. The granting of fundamental rights to citi-
zens aims to ensuring (d) none of above
(a) individual government 291. Which one of the following is the 28th
(b) socialist government State of the Union of India
(c) an independence judiciary (a) Chhattisgarh
(d) none of these (b) Uttarakhand (Uttaranchal)
284. Which fundamental right is concerned (c) Jharkhand
with abolition of social distinctions? (d) none of the above
(a) Cultural and educational rights 292. Which of the following is not a condition
(b) Right to life and liberty for becoming a citizen of India?
(c) Right to equality (a) citizenship by naturalization
(d) Right against exploitation (b) citizenship through acquisition of
285. The Constitution prescribes the property
(a) punishment for the practice of un- (c) citizenship by birth
touchability (d) citizenship by descent
(b) abolition of untouchability as a Fun- 293. Who is competent to prescribe the condi-
damental Right tions for acquisition of citizenship?
(c) way of detecting the practice of un- (a) Parliament and state legislatures joint-
touchability ly
(d) abolition of untouchability as a Direc- (b) Election Commission
tive Principle of State Policy (c) Parliament
286. Who among the following is the executive (d) President
head of a state? 294. The original Constitution classified the
(a) the President Fundamental Rights into seven categories
(b) the Governor but now there are only

CLAT.indb 200 31/03/2009 11:41:56


CONSTITUTIONAL LAW 201

(a) six categories (a) Parliament (b) Citizens of India


(b) five categories (c) Lok Sabha (d) President
(c) three categories 304. The first woman Governor of a state in
(d) four categories free India was
295. Which one of the following has been (a) Mrs Sarojini Naidu
added to the list of original Fundamental (b) Mrs Sucheta Kripalani
Rights? (c) Mrs Indira Gandhi
(a) right to Freedom of Religion (d) Mrs Vijaya Laxmi Pandit
(b) right to Constitutional Remedies 305. Minimum age required to contest for Pres-
(c) right to Property identship is
(d) none of the above (a) 30 years (b) 35 years
296. Which one of the following is not a Fun- (c) 23 years (d) 21 years
damental Right? 306. The charge of impeachment against the
(a) right to freedom or religion President of India for his removal can be
(b) right to equality preferred by
(c) right against exploitation (a) Rajya Sabha
(d) right to strike (b) Lok Sabha
297. Which one of the following has since long (c) Speaker of Lok Sabha Sabha and
ceased to be Fundamental Right? Chairman of Rajya Sabha
(a) right to freedom of speech (d) Both Houses of Parliament
(b) right to property 307. The Chief Minister of Union Territory
(c) right to constitutional remedies where such a set up exists, is appointed by
(d) all the above the
298. Right to private property was dropped (a) President
from the list of Fundamental Rights by (b) Prime Minister
the (c) Lt Governor
(a) 52nd Amendment (d) Majority party in Legislature
(b) 44th Amendment 308. Who was the first Prime Minister of In-
(c) 42nd Amendment dia?
(d) none of the above (a) Rajendra Prasad
299. How many types of emergencies have (b) Mahatma Gandhi
been envisaged by the Constitution? (c) Jawaharlal Nehru
(a) four (b) three (d) Indira Gandhi
(c) only one (d) two 309. The union council of Ministers consists
300. The President of India made use of his of
veto power once in (a) Prime Minister
(a) the Dowry Prohibition Bill (b) Cabinet Minister
(b) the Indian Post Office (Amendment) (c) Cabinet Ministers and Chief Ministers
Bill of the State
(c) the Hindu Code Bill (d) Cabinet Ministers, Ministers of State
(d) the PEPSU Appropriation Bill and Deputy Ministers.
301. Who among the following holds office 310. Which of the following is the correct
during the pleasure of the President? chronological order of the Prime Ministers
(a) Speaker of the Lok Sabha in India?
(b) Prime Minister I. Indira Gandhi II. Jawaharlal Nehru
(c) Election Commissioner III. Morarji Desai IV. Charan Singh
(d) Governor (a) I, II, III, IV (b) II, III, I, IV
302. Which of the following appointments is (c) II, I, III, IV (d) III, II, IV, I
not made by the President of India? 311. Which of the following is true in context
(a) Speaker of the Lok Sabha of the President?
(b) Chief Justice of India (a) He addresses the first session of the
(c) Chief of the Air Force Parliament after each general elec-
(d) Chief of the Army tion of the Lok Sabha and at the com-
303. Who appoints the Prime Minister of In- mencement of the first session of each
dia? year.

CLAT.indb 201 31/03/2009 11:41:56


202 CONSTITUTIONAL LAW

(b) He addresses the first session of Par- 319. Who was the member of the Rajya Sabha
liament at the beginning of each year when first appointed as the Prime Minister
as well as the last session held at the of India?
end of each year. (a) Lal Bahadur Shastri
(c) He addresses the Parliament daily. (b) Indira Gandhi
(d) None of these (c) Morarji Desai
312. Who administers the oath of office to the (d) Charan Singh
President of India before he enters upon 320. Who is the longest serving Chief Minister
the office? in India?
(a) Vice-President (a) Bhajan Lal
(b) Prime Minister (b) Hiteshwar Saikia
(c) Chief Justice (c) Chimanbhai Patel
(d) Speaker (d) Jyoti Basu
313. Who among the following enjoys the 321. What is the minimum age for appointment
rank of a Cabinet Minister of the Indian as a Governor?
Union? (a) 25 years (b) 30 years
(a) Deputy Chairman, Rajya Sabha (c) 35 years (d) 40 years
(b) Deputy Chairman of the Planning 322. Chief Minister of a State is responsible to
Commission (a) Governor
(c) Secretary to the Government of India (b) Legislative Assembly
(d) None of these (c) Prime Minister
314. A person who is not a member of Parlia- (d) Rajya Sabha
ment can be appointed as a Minister by the 323. The administrative and operational control
President for a maximum period of of the Armed Forces is exercised by the
(a) 9 months (b) 6 months (a) Ministry of Defence
(c) 12 months (d) 3 months (b) Three Chiefs of the Army, Navy and
315. When can a President use his discretion in Air Force Staff
appointing the Prime Minister? (c) President
(a) In all circumstances (d) Cabinet Committee on Political Af-
(b) In no circumstances fairs
(c) When no political party enjoys a clear 324. How many times the President of India
majority in the Lok Sabha can seek re-election to his post?
316. Chief Ministers of all the states are ex-
(a) Once
officio members of the
(b) 2 times
(a) Finance Commission
(c) 3 times
(b) Planning Commission
(d) Any number of times
(c) National Development Council
325. Under whose advice the President of India
(d) Inter State Council
declares Emergency under Article 352?
317. The President nominates 12 members of
(a) Council of Ministers
the Rajya Sabha according to
(b) Cabinet
(a) the performance as office bearers of
(c) Chief Minister of all States
cultural societies
(d) Prime Minister
(b) their role played in political set up of
the country 326. Who acts as the President of India when
(c) the recommendations made by the neither the President nor the Vice-Presi-
Vice-President dent is available?
(d) their distinction in the field of science, (a) Speaker of Lok Sabha
art, literature and social service (b) Chief Justice of India
318. Who among the following held office of (c) Auditor General of India
the Prime Minister for the shortest pe- (d) Seniormost Governor of a State
riod? 327. Which is true regarding the President of
(a) Atal Bihari Vajpayee India?
(b) Lal Bahadur Shastri I. He is the Chief Executive
(c) V. P. Singh II. He is the Supreme Commander of the
(d) Chandra Shekhar armed forces

CLAT.indb 202 31/03/2009 11:41:56


CONSTITUTIONAL LAW 203

III. He is the titular head of the State 336. When a financial emergency is pro-
IV. He is part of the Union Legislature claimed
(a) I and III (b) I and II (a) repayment of government debts will
(c) II, III and IV (d) I, II, III and IV stop
328. Who among the following are appointed (b) payment of salaries to public servants
by the President of India? will be postponed
(a) Chairman, finance Commission (c) Salaries and allowances of any class
(b) Deputy Chairman, Planning commis- of employees may be reduced
sion (d) Union budget will not be presented
(c) Chief Minister of a Union Territory 337. Who appoints the Chairman of the
(d) none of these UPSC?
329. The President can proclaim emergency on (a) President
(b) Chief Justice of India
the written advice of the
(c) Prime Minister
(a) Prime Minister
(d) Speaker of Lok Sabha
(b) Speaker of Lok Sabha
338. If the President wants to resign from of-
(c) Union Cabinet
fice, he may do so by writing to the
(d) Speaker of Rajya Sabha (a) Vice- President
330. The Union Council of Ministers is collec- (b) Chief Justice of India
tively responsible to (c) Prime Minister
(a) Prime Minister (d) Speaker of Lok Sabha
(b) Rajya Sabha 339. Who is the ex-officio Chairman of the
(c) President Council of States?
(d) House of the People (a) Vice-President (b) President
331. Article 78 of the Constitution deals with (c) Speaker (d) Prime Minister
(a) Emergency powers of the President 340. Under which Article of the Constitution
(b) Prime Minister’s duty regarding keep- is the President’s rule promulgated on any
ing the President informed about the state in India?
government’s Decisions and policies (a) 356 (b) 326
(c) President’s power to send advisory (c) 380 (d) 370
messages to the Parliament 341. At a time, President’s rule can be imposed
(d) President’s power to get information on a state for a maximum period of
from the Council of Ministers (a) 1 year (b) 2 years
332. The maximum duration for which the (c) 4 years (d) 5 years
Vice-President may officiate as the Presi- 342. The Chairman of the Planning Commis-
dent, is sion in India is the
(a) 4 months (b) 6 months (a) Minister of Planning
(c) 9 months (d) 1 year (b) Finance Minister
333. The President’s Rule in a State can be con- (c) President
(d) Prime Minister
tinued for a maximum period of
343. In case of which of the following, the
(a) 2 ½ years (b) 2 years
method of removal from office is the
(c) 1 year (d) 6 months
same?
334. The Chairman and members of State Pub-
I. A judge of the Supreme Court
lic Service Commission are appointed by II. Chief Election Commissioner
the III. President of India
(a) President (b) Prime Minister (a) I and II (b) II and III
(c) Governor (d) Chairman, UPSC (c) I and III (d) I, II and III
335. When a State Governor dies or resigns, 344. If the office of the President of India falls
who normally exercises his functions till vacant, within what time should the next
a new Governor is appointed? President be elected?
(a) Secretary General of the Governor (a) Immediately
(b) A person designated by State Cabinet (b) Within 2 months
(c) Chief Justice of the High Court (c) Within 6 months
(d) Advocate General of the State (d) Within 1 year

CLAT.indb 203 31/03/2009 11:41:56


204 CONSTITUTIONAL LAW

345. The President can proclaim a state of 353. The President of India is elected on the ba-
emergency in case of sis of
(a) External aggression or internal distur- (a) proportional representation by a single
bance threatening the security of the transferable vote
country (b) proportional representation
(b) Failure of Constitutional machinery in (c) single member territorial representa-
a particular state tion
(c) Threat to financial stability of the (d) none of the above method
country 354. The President can proclaim national emer-
(d) All of the above gency only on the written advice of
346. The Indian President is (a) the Chief Justice of India
(a) Real executive (b) the Prime Minister
(b) Titular executive (c) the Speaker of Lok Sabha
(c) Real/Titular executive (d) the Union Cabinet
(d) None of these 355. The President can impose his rule in a state
347. The minimum age required for becoming on account of failure of constitutional ma-
the Prime Minister of India is chinery under
(a) 25 years (b) 30 years (a) Article 356 (b) Article 361
(c) 40 years (d) 35 years (c) Article 371 (d) Article 379
348. The Chief Minister of a Union Territory 356. The proclamation of National Emergency
whenever such a set-up exists, is appoint- ceases to operate unless approved by the
ed by Parliament within
(a) the President (a) three months (b) two months
(c) one month (d) six months
(b) the Prime Minister
357. Which of the following three types of
(c) the Lt Governor
emergencies has been declared by the
(d) the majority party in the legislature
President for maximum number of times?
349. Under what article of the Constitution of
(a) constitutional emergency
India can the President take over the ad-
(b) national emergency
ministration of a state in case its constitu-
(c) financial emergency
tional machinery breaks down?
(d) all the three emergencies have been
(a) Article 83 (b) Article 352
declared equal number of times
(c) Article 356 (d) Article 343
358. The President can declare financial emer-
350. How many types of Emergency have been gency
visualized in the Constitution of India? (a) on the recommendation of the Comp-
(a) One (b) Two troller and Auditor-General
(c) Three (d) Four (b) if there is a threat to the financial sta-
351. The President can promulgate an ordi- bility or credit of India
nance only when (c) to meet the extraordinary expenses of
(a) the bill was sponsored by the President conducting a war
but the Parliament refused to pass the (d) if the majority of the state legislatures
same so recommend
(b) the Parliament is not in session 359. The President, who is the head of the State
(c) there is disagreement between the two under the Parliamentary system prevailing
houses of the Parliament in India
(d) the bill has been pending in the Parlia- (a) enjoys limited but real powers
ment for over a year (b) enjoys only nominal powers
352. The President can dismiss a member of (c) enjoys absolute powers
the Council of Ministers (d) enjoys no powers
(a) on the recommendation of the Prime 360. The right which is available even to a non-
Minister citizen is
(b) at his discretion (a) Freedom to acquire property or to
(c) with the consent of the Speaker carry on any occupation, trade or busi-
(d) none of the above ness

CLAT.indb 204 31/03/2009 11:41:56


CONSTITUTIONAL LAW 205

(b) Freedom to move, reside and settle in 367. What is the maximum age prescribed for
any part of the territory of India election as President of India?
(c) Freedom of speech, assembly and as- (a) 62 years (b) 60 years
sociation (c) 58 years (d) No such limit
(d) Right to Constitutional Remedies 368. Regarding his power of pardon, the Presi-
361. Writ of Mandamus is dent can employ it in
(a) a writ from a superior court com- (a) pardoning a sentence of death
manding some officer of particular (b) all the cases of punishment by a court
authority to do a specific act martial
(b) an order from a superior court to any (c) offence against laws in the Union and
official to show his right to the office Concurrent lists
(c) an order to produce the body of a per- (d) all of the above instances
son 369. In the election of the President
(d) an order from a higher court to stop (a) parity between the Centre and the
proceedings in a certain case States has been maintained
362. The writ of prohibition issued by the Su- (b) the States are given greater weight-
preme Court or a High Court is issued age
against (c) the Centre enjoys greater weightage
(a) administrative and judicial authorities (d) none of the above
(b) administrative authorities and govern- 370. Legally speaking, who can declare war or
ment peace?
(c) judicial or quasi-judicial authorities (a) Defence Minister
(d) administrative authorities only (b) President of India
363. The Fundamental Duties were added to
(c) Prime Minister of India
the Constitution
(d) Chiefs of Army, Navy and Air Force
(a) to accord priority to the directive prin-
together
ciples over fundamental agitation
371. The President of India can declare
(b) to make the fundamental rights more
(a) Constitutional Emergency
effective
(b) National Emergency
(c) to check anti-national, subversive and
(c) Financial Emergency
un-constitutional agitations
(d) All of these
(d) to achieve all the above objectives
372. In the election to the post of the President
364. A socialistic ideology is reflected in the
Directive Principles which calls for (a) MPs and MLAs of a State have the
(a) securing equitable distribution of ma- same number of votes
terial resources of the country to pre- (b) MLAs of different States have differ-
vent concentration of wealth ent number of votes
(b) free and compulsory education for (c) Each elected MP or MLA has an equal
children upto 14 years of age number of votes
(c) promotion of cottage industries (d) All MPs and MLAs have one vote
(d) all the above each
365. The enforcement of the Directive Prin- 373. The Law Officer who holds office at the
ciples depends on pleasure of the President are
(a) an effective opposition (a) The Attorney-General and the Law
(b) independent judiciary Minister
(c) the whips of the government (b) The Law Minister and any judge of
(d) resources at the disposal of the gov- the Supreme Court or of a High court
ernment (c) The Chief Justice of the Supreme
366. The Constitution of India vests the execu- Court and the Attorney-General
tive power of the Indian Union in (d) The Attorney-General and the Solici-
(a) the President tor-General
(b) the Parliament 374. The President of India is elected for
(c) the Council of Ministers (a) 6 years (b) 5 years
(d) the Prime Minister (c) 4 years (d) life

CLAT.indb 205 31/03/2009 11:41:56


206 CONSTITUTIONAL LAW

375. Which one of the following is not a judi- 383. Which one of the following emergencies
cial power of the President of India? can be declared by the President on his
(a) He can remove the judges of the Su- own?
preme Court on the ground of miscon- (a) Emergency due to threat to the finan-
duct. cial stability or credit of India
(b) He can consult the Supreme Court on (b) Emergency due to external aggression
any question of law or fact which is of or internal disturbances
public importance. (c) Emergency due to failure of the con-
(c) He appoints Chief Justice and other stitutional machinery in a state
judges of the Supreme Court. (d) None of the above
(d) He can grant pardon, reprieves and re- 384. The main characteristic of the cabinet sys-
spites to the people. tem is
376. The Vice-President of India holds office (a) leadership of the Prime Minister
(a) for a term of four years (b) inclusion of the President
(b) for a term of five years (c) cabinet decisions are not binding on
(c) during the pleasure of the President all Ministers
(d) for a term which is decided at the time (d) all the above
of his appointment by the Parliament 385. How long is the tenure of the Prime Min-
377. On the death of the President, the Vice- ister in India?
President succeeds him as President for (a) It depends on the President’s direc-
(a) a maximum period of three years tion
(b) a maximum period of one year (b) As long as he enjoys the confidence of
(c) a maximum period of six months the Lok Sabha
(d) the remaining period of the term (c) As long as he is leader of the majority
378. The constitution of Jammu and Kashmir party in the Lok Sabha
came into force on (d) 5 years
(a) 26 January 1951 386. The formation of the Council of Ministers
(b) 26 January 1957 starts with the appointment of
(c) 26 January 1958 (a) the Prime Minister
(d) None of these (b) the President
379. The Council of Ministers is responsible to (c) the Speaker
(a) the President (d) None of these
(b) the People 387. The Council of Ministers is
(c) the Prime Minister (a) unrelated to the Cabinet
(d) the Parliament (b) a smaller body than the Cabinet
380. The tenure of the Union Council of Minis- (c) identical with the Cabinet
ters is (d) a larger body than the Cabinet
(a) five years 388. The legislative functions of the Council of
(b) uncertain Ministers include the right to
(c) fixed and co-terminus with the Presi- (a) nominate some members to Rajya
dent Sabha
(d) co-terminus with the Lok Sabha (b) summon and prorogue the two Houses
381. If the office of the President falls vacant, of Parliament
the same must be filled within (c) nominate two Anglo-Indian members
(a) one year (b) 18 months of Lok Sabha
(c) three months (d) six months (d) introduce important bills and resolu-
382. How many members of the Rajya Sabha tions in the Parliament
can be nominated by the President from 389. The remuneration for the Attorney-Gener-
amongst the persons who have distin- al of India is determined by
guished themselves in art, literature, social (a) the Council of Ministers
service, etc. (b) the Constitution
(a) 10 (b) 12 (c) the Parliament
(c) 2 (d) none (d) the President

CLAT.indb 206 31/03/2009 11:41:56


CONSTITUTIONAL LAW 207

390. Which one of the following wanted the (d) to meet expenditure for a period be-
Directive Principles of State Policy to be fore passing of the annual budget
the basis of all future legislation? 397. What is the number of seats allotted to
(a) Jawaharlal Nehru Scheduled Tribes in Lok Sabha?
(b) K. M. Munshi (a) 84 (b) 78
(c) B. R. Ambedkar (c) 50 (d) 30
(d) B. N. Rao 398. Who can dissolve the Lok Sabha before
391. In the event of non-enforcement of the the expiry of its term?
Directive Principles of State Policy by the (a) The President
government, the citizen can approach (b) Prime Minister
(a) any court of his choice (c) The President on the advice of the
(b) the High court Prime Minister
(c) the Supreme Court (d) None of these
399. In addition to the Speaker, the Lok Sabha
(d) none of these
has a Deputy Speaker who is
392. Which amendment of the constitution ac-
(a) appointed by the President
corded precedence to the Directive Prin-
(b) appointed by the Speaker
ciples of State Policy over Fundamental
(c) appointed by the Prime Minister
rights?
(d) elected by the members of the Lok
(a) 44th amendment Sabha
(b) 42nd Amendment 400. The members of Rajya Sabha are
(c) 39th Amendment (a) nominated by the President and State
(d) none of these Governors
393. In which case did the Supreme Court (b) elected by voters of India directly
strike down the provisions of the Consti- (c) elected by elected members of the
tution that accorded primacy to Directive State Vidhan Sabha
Principles over Fundamental Rights? (d) elected by the members of the Lok
(a) Minerva Mills case Sabha
(b) Golak Nath case 401. The impeachment proceedings against the
(c) Sajjan Singh case Vice-President can be initiated
(d) Keshavananda Bharati case (a) Only in Lok Sabha
394. Which one of the following factors has (b) Only in Rajya Sabha
been responsible for the slow implemen- (c) In either House of Parliament
tation of the Directive Principles? (d) In neither House of Parliament
(a) Vastness of the country 402. Who appoints the Governor of Jammu and
(b) Lack of resources with the govern- Kashmir?
ment (a) Chief Minister of the State
(c) Lack of political will (b) Chief Justice of the High Court
(d) All of these (c) President
395. If the Auditor-General wants to resign, he (d) Prime Minister
addresses his letter of resignation to 403. The President of India can be removed
(a) Speaker of Lok Sabha from his office by the
(b) the Prime Minister (a) Prime Minister
(c) the President (b) Lok Sabha
(c) Chief Justice of India
(d) None of the above
(d) Parliament
396. Vote on account is made by the Lok Sab-
404. In addition to the powers given in the
ha
Union and Concurrent List, the Union
(a) to meet expenditure on unexpected
Government has the
demand
(a) Residuary power
(b) to meet expenditure on secret service (b) Federal power
(c) to meet the additional expenditure in- (c) Power to legislate in State list
curred by the government on any ap- (d) None of these
proved item of expenditure during the 405. Sarkaria Commission has been appointed
supplementary budget by the Government of India to report on

CLAT.indb 207 31/03/2009 11:41:56


208 CONSTITUTIONAL LAW

(a) Centre-State relations (c) Must have been an advocate of a High


(b) Electoral reforms Court for not less than 10 years.
(c) Inter-State conflicts (d) Must be, in the opinion of the Presi-
(d) Tribal development dent, a distinguished jurist.
406. Planning in India derives its objectives 414. Judicial Review function of the Supreme
from Court
(a) Fundamental Rights (a) reviews its own judgment
(b) Directive Principles (b) reviews the functioning of judiciary in
(c) Fundamental Duties the country
(d) None of these (c) examines the constitutional validity of
407. The residuary powers mean the the laws
(a) powers shared between the Union and (d) undertakes periodic review of the
State Governments Constitution
(b) powers related to country’s internal 415. The High Courts in India were first started
affairs at
(c) powers related to country’s external (a) Bombay, Madras, Calcutta
affairs (b) Delhi and Calcutta
(d) powers not included in any of the (c) Bombay, Delhi, Madras
three lists (d) Madras and Bombay
408. Under the Constitution, the residuary 416. For the enforcement of Fundamental
powers vest with the Rights, the Supreme Court may issue a/an
(a) President (a) decree (b) ordinance
(b) Supreme Court (c) notification (d) writ
(c) Union Government 417. What is meant by a ‘Court of Record’?
(d) State Government (a) The court that preserves all its
409. Which of the following is not in the State records
List under the Constitution of India? (b) The court that maintains records of all
(a) Fisheries (b) Agriculture lower courts
(c) Insurance (d) Gambling (c) The court that can punish for its
410. Railways is a subject on the concept
(a) Concurrent list (d) The court that is competent to give
(b) Union list
directions
(c) State list
418. Judges of the High Court are appointed by
(d) Residual list
the
411. Which of the following writs is issued by
(a) Governor
the court in case of illegal detention of a
(b) Chief Justice of India
person?
(c) President
(a) Habeas Corpus
(d) Chief Justice of the High Court
(b) Mandamus
419. After retirement, a judge of a High Court
(c) Certiorari
can undertake practice in
(d) Quo Warranto
(a) the same court
412. Which of the following cases cannot be
(b) lower courts only
filed directly in the Supreme Court?
(a) Disputes between two or more States (c) any other court except the same court
(b) Cases against encroachment on Fun- (d) wherever he intends to practice
damental Rights 420. The age of retirement of the Judges of the
(c) If one’s property is forcefully occu- High Court is
pied by the other (a) 62 years (b) 65 years
(d) Both (a) and (b) (c) 58 years (d) 60 years
413. Which is not an eligibility criterion for ap- 421. Separation of the Judiciary from the
pointment as a Judge of the High Court? Executive is enjoined by
(a) Must have been a High Court Judge (a) Preamble
for at last 5 years. (b) Directive Principle
(b) Must have attained the age of 55 (c) VII Schedule to the Constitution
years. (d) Judicial decision

CLAT.indb 208 31/03/2009 11:41:57


CONSTITUTIONAL LAW 209

422. Which of the following is not true about (b) let a person free for a temporary
the Supreme Court? period
(a) It has only the Appellate Jurisdiction (c) order to transfer the case from one
(b) It is the highest federal court of India court
(c) It does not have the Original Jurisdic- (d) direct the Government to do or not to
tion do a thing
(d) It can amend the Constitution 431. Which of the following writs is a bulwark
423. The Chief Justice and other Judges of the of personal freedom?
High court are appointed by the (a) Mandamus (b) Habeas Corpus
(a) President (c) Quo Warranto (d) Certiorari
(b) Chief Justice of the Supreme Court 432. The High Courts as Calcutta, Madras
(c) Governor of the concerned state and Bombay, were established under the
(d) Chief Minister of the concerned state Indian High Courts Act of
424. Which High Court has jurisdiction over
(a) 1861 (b) 1865
the state of Arunachal Pradesh?
(c) 1909 (d) 1911
(a) Guwahati
433. Appointments of officers and servants of a
(b) Bombay (Mumbai)
High Court are made by the
(c) Calcutta (Kolkata)
(a) President
(d) Chandigarh
425. A judge of the Supreme Court of India is (b) Governor
to hold office until he attains the age of (c) Chief Justice of the High Court
(a) 58 years (b) 60 years (d) None of these
(c) 62 years (d) 65 years 434. Salaries of the Judges of the Supreme
426. The President of India referred the Ayod- Court are drawn from the
hya issue to the Supreme Court of India (a) Grants-in-aid
under which Article? (b) Contingency Fund
(a) 143 (b) 138 (c) Consolidated Fund
(c) 136 (d) 132 (d) Public Accounts
427. Judicial Review signifies that the Supreme 435. The Constitution gives the powers of su-
Court perintendence over all subordinate courts
(a) has the final authority over all the to the High Courts under Article
cases (a) 226 (b) 227
(b) can impeach the President (c) 228 (d) 229
(c) can review cases decided by the High 436. Which of the following High Courts cov-
Courts ers more than one State/Union Territo-
(d) can declare a state law as unconstitu- ries?
tional (a) Delhi (b) Allahabad
428. Cognizable offence refers to an offence (c) Guwahati (d) None of these
where 437. Which of the following writs may be is-
(a) arrests can be made without warrant sued to enforce a Fundamental Right?
(b) arrests can be made with warrant (a) Habaes Corpus
(c) it is under the jurisdiction of a court (b) Mandamus
(d) police can register a case without for- (c) Prohibition
mal complaints (d) All the above
429. The Supreme Court passed the special 438. The total number of High Court in India at
judgement that the basic structure of our
present is
Constitution cannot be changed by the
(a) 15 (b) 16
parliament by any amendment in the
(c) 18 (d) 21
(a) Keshavanand Bharathi Case
439. The Judges of the Supreme Court can be
(b) Gokalnath Case
(c) Balananda Saraswati Case removed from office by the
(d) Minerva Mills Ltd and Others (a) President
430. Under the writ of ‘Madamus’, the Court (b) Prime Minister
can (c) President on request of Parliament
(a) ask the person to be produced (d) Chief Justice of India

CLAT.indb 209 31/03/2009 11:41:57


210 CONSTITUTIONAL LAW

440. The authority competent to suspend the 448. The Supreme Court originally consisted of
operation of Fundamental Rights guaran- how many judges besides the Chief Jus-
teed under the Constitution of India is tice?
(a) Supreme Court (a) 6 (b) 7
(b) Parliament (c) 14 (d) 12
(c) Prime Minister 449. When can the salaries of the judges of the
(d) President Supreme Court be reduced during their
441. Which of the following is an extensive term of office?
original jurisdiction given by the Consti- (a) If the Parliament passes a bill to this
tution of India to the Supreme Court? effect
(a) Enforcement of Fundamental Rights (b) During a Financial Emergency
(b) Hearing revenue cases of appeal (c) As and when the President desires
(c) Hearing criminal cases of appeal (d) Never
(d) Advising the Chief Executive in legal 450. The Guwahati High Court has territorial
matters jurisdiction over how many states?
442. The High Court of West Bengal (Calcutta) (a) 7 (b) 5
has got the additional jurisdiction to hear (c) 2 (d) 4
cases from 451. Madras High Court has the jurisdiction
(a) Andaman and Nicobar Islands over
(b) Arunachal Pradesh (a) Tamil Nadu and Kerala
(c) Tripura (b) Tamil Nadu and Puducherry
(d) Mizoram (c) Kerala and Lakshadweep
443. Which of the following is enforceable in a (d) Lakshadweep and Tamil Nadu
court of law? 452. Who among the following has the consti-
(a) Directive Principles tutional authority to make rules regarding
(b) Fundamental Rights the manner of enforcing the orders of the
(c) Preamble Supreme Court?
(d) Fundamental Duties (a) Chief Justice of India
444. With whom are the powers of Judicial Re- (b) Attorney General of India
view vested in India? (c) President
(a) President (d) Union Law Minister
(b) Parliament 453. The Supreme Court of India is created by
(c) All the courts (a) the Constitution
(d) Supreme Court and all the High (b) a Parliament Act, 1950
Courts (c) a Presidential Order, 1951
445. Which of the following is covered under (d) Indian Independence At, 1947
the original jurisdiction of the Supreme 454. At present, the Supreme Court consists of
Court? how many judges besides the Chief Jus-
(a) Disputes relating to civil matters tice?
(b) Disputes relating to criminal cases in- (a) 20 (b) 18
volving murder (c) 24 (d) 25
(c) Disputes between two states of the In- 455. The power of Judicial Review has been
dian Union conferred on the Supreme Court to check
(d) Disputes between two citizens from the arbitrary power of the
two different states (a) Executive
446. The power of the Supreme Court to review (b) Parliament
any judgement pronounced or order made (c) Council of Ministers
by it previously is provided in Article (d) President
(a) 126 (b) 138 456. The Judges and magistrates of the Civil
(c) 137 (d) 139 Courts are appointed by the
447. The only Unions Territory which has a (a) Subordinate Judge
High Court of its own (b) Chief Justice of the High Court
(a) Delhi (b) Lakshadweep (c) Governor
(c) Chandigarh (d) Daman and Diu (d) District Magistrate

CLAT.indb 210 31/03/2009 11:41:57


CONSTITUTIONAL LAW 211

457. The Chief Justice of a High Court is ap- (b) 5 years


pointed by the President in consultation (c) 2 years
with (d) of a permanent nature
I. Chief Justice of the Supreme Court 465. In which of the following does the Rajya
II. Former Chief Justice of the High Sabha enjoy exclusive right?
Court (a) Initiating impeachment proceedings
III. Other Judges of the High Court against the President
IV. Governor of the State concerned (b) Taking initiative for the removal of
(a) I and II (b) II and III the Vice-President
(c) I and IV (d) I, III and IV (c) Recommending creation of new All
458. The words ‘social, secular’ and ‘the uni- India Services
ty and integrity of the nation’ have been (d) In all the above matters
added to the Indian Constitution by which 466. The President of India is
amendment?
(a) the head of the state as well as govern-
(a) 38th (b) 40th
ment
(c) 42nd (d) 48th
(b) the head of the government
459. Who is entitled to initiate a Bill for Consti-
(c) the head of the state
tutional amendments?
(d) none of the above
(a) State Legislative Assemblies
(b) Lok Sabha only 467. Which one of the following does not take
(c) Either House of Parliament part in the election of the President?
(d) Rajya Sabha only (a) elected members of Lok Sabha
460. For the creation of a new state in India, (b) members of the legislative councils
the amendment of the constitution will re- (c) elected members of the Rajya Sabha
quire (d) none of the above
(a) simple majority in Parliament 468. The name of the candidate for the office of
(b) simple majority in Parliament and ap- the President of India has to be proposed
proval of majority of states by
(c) two-third majority in Parliament (a) any 50 members of the Electoral Col-
(d) two-third majority in each House of lege
Parliament and approval of majority (b) any five members of the Electoral
of states College
461. The 42nd amendment to the Indian Con- (c) any five members of the Parliament
stitution is notable because it gives (d) any 50 citizens
(a) primacy to the fundamental Rights 469. The election to the office of the President
over Directive Principles is conducted by
(b) primacy to the Directive Principles (a) the Minister for Parliamentary Affairs
over Fundamental Rights (b) the Election Commission of India
(c) Special treatment to Jammu and Kash- (c) the Speaker of Lok Sabha
mir (d) the Prime Minister’s Office
(d) Special treatment to Sikkim 470. Which of the following Presidents held of-
462. 44th amendment deals with fice for two consecutive terms?
(a) abolition of right to Property as a Fun- (a) Dr Rajendra Prasad
damental Right (b) Dr Zakir Hussain
(b) curbing the Fundamental Rights (c) Dr S. Radhakrishnan
(c) curbing the jurisdiction of High Court
(d) Both (a) and (b)
(d) extension of reservation of seats in
471. Who was the second President of the In-
Parliament
dian Republic?
463. By which amendment was the power of
(a) Dr Rajendra Prasad
the Lok Sabha to amend the Constitution
affirmed? (b) Dr V. V. Giri
(a) 24th (b) 42nd (c) Dr S. Radhakrishnan
(c) 43rd (d) 39th (d) Dr Zakir Hussain
464. The duration of the Rajya Sabha is 472. Who decides the disputes regarding elec-
tion of the President?
(a) 4 years

CLAT.indb 211 31/03/2009 11:41:57


212 CONSTITUTIONAL LAW

(a) the Parliament ment to work for the attainment of set


(b) the Supreme Court objectives
(c) the Election Commission (d) negative instructions to the govern-
(d) both Supreme Court and High Courts ment to refrain from encroaching on
473. Who among the following got the Bharat the freedom of the people
Ratna Award before becoming the Presi- 481. In which part of the Constitution does the
dent of India? concept of welfare state find elaboration?
(a) Dr V. V. Giri (a) Fundamental Rights
(b) Dr Zakir Hussain (b) Preamble
(c) Dr Rajendra Prasad (c) Directive Principles of State Policy
(d) None of these (d) all the above
474. The Comptroller and Auditor-General is 482. The Directive Principles of State Policy
associated with seek
(a) Business Advisory Committee (a) to make the Constitution an instru-
(b) Public Accounts Committee ment of social change
(c) Committee on Public Undertakings (b) to strengthen judiciary
(d) All the above Committees (c) to establish the supremacy of the Con-
475. The ordinances promulgated by the Gov- stitution
ernor are subject to the approval of the (d) to curb the authoritarian rule
(a) Prime Minister 483. The Legislative Council is expected to
(b) Vice-President have a strength equal to one-third of the
(c) President membership of the Legislative Assembly.
(d) State Legislature The minimum strength of the Council has
476. The normal term of a Governor’s office is to be
(a) 6 years (a) 50 (b) 40
(b) 5 years (c) 100 (d) 30
(c) 3 years 484. What is the maximum period up to which
(d) dependent on the tenure of the State the Legislative Council of a State can
Legislature delay the consideration of a Bill that has
477. If the Governor of a State dies, who be- been already passed once by the Legisla-
comes the acting Governor? tive Assembly of the State?
(a) The Chief Justice of the State (a) Six months (b) Three months
(b) Anyone appointed by the President of (c) One month (d) Two months
India 485. The phrase ‘bicameral legislature’ means
(c) The Chief Minister (a) Parliamentary system of Government
(d) The Chief Secretary (b) a legislature consisting of lower and
478. The ordinance of the Governor an upper chamber
(a) has little value (c) a single assembly
(b) is much narrower than a law made by (d) an elected legislature
the State Legislature 486. The Union Territories are administered by
(c) has the same force as the law made by (a) the administrator(s) appointed by the
the State Legislature Government of India
(d) is more extensive than the law made (b) the Union Executive
by the State Legislature (c) the Prime Minister of India
479. What is the main sanction behind the Fun- (d) the President through the administra-
damental Duties? tors appointed by him
(a) moral (b) social 487. The Lieut Governor of Daman and Diu is
(c) legal (d) all the above concurrently the administrator of
480. The Directive Principles are (a) Lakshdweep
(a) directives to the government to pursue (b) Maldives
a policy of non-alignment (c) Dadra and Nagar Haveli
(b) directives to the state to enhance the (d) Puducherry
international prestige of the country 488. When can a President refer a matter to the
(c) positive instructions to the govern- Supreme Court for its opinion?

CLAT.indb 212 31/03/2009 11:41:57


CONSTITUTIONAL LAW 213

(a) When the President feels like 495. When was the Panchayati Raj introduced
(b) When a matter is of public impor- in India?
tance (a) 1962 (b) 1952
(c) When the Cabinet is not functioning (c) 1950 (d) 1959
properly 496. The Committee on whose recommenda-
(d) When emergency has to be promul- tion Panchayati Raj was introduced in the
gated country was headed by
489. Who can be appointed as an adhoc judge (a) Balwant Rai Mehta
of the Supreme Court? (b) Jivraj Mehta
(a) A person fully qualified for appoint- (c) Ashok Mehta
ment as a judge to the Supreme Court (d) None of these
(b) All acting judges of the Supreme 497. The lowest unit in the Panchayati Raj in-
Court stitutions is
(c) A retired judge of the Supreme Court (a) The Gram Sabha
(d) A sitting judge of a High Court duly (b) Zila Parishad
qualified for appointment as a Su- (c) A Village Panchayat
preme Court Judge (d) The Panchayat Samiti
490. The privileges of the judges of Supreme 498. The primary aim of the Panchayati Raj ad-
Court ministration is
(a) can be modified only with the approv- (a) to increase agricultural production
al of the Chief Justice of India through the involvement of the people
(b) can be modified by the Parliament in extension programmes
only by two thirds majority (b) to arouse in the people continuous in-
(c) cannot be modified to their disadvan- terest in the community development
tage during their tenure of office programmes
(d) can be modified to their advantage or (c) to work for rural development
disadvantage during their term of of- (d) to ensure the upliftment of Harijans
fice 499. The original scheme of Panchayati Raj, in-
491. The Advisory opinion tendered by the Su- troduced in 1959, operates at (in descend-
preme Court ing order of tiers)
(a) is not made public at all (a) Zila Parishad, Panchayat Samiti, Vil-
(b) is binding on the President only if it is lage Panchayat
unanimously made (b) Gram Sabha, Zila Parishad, Village
(c) is binding on the President Panchayat
(d) is not binding on the President (c) Zila Parishad, Panchayat Samiti,
492. Who decides about the number of Judges Gram Sabha
in a High Court? (d) Panchayat Samiti, Gram Panchayat,
(a) The Parliament Zila Parishad
(b) The State Government 500. Which of the following States has no
(c) The President of India Panchayati Raj institution at all?
(d) The Governor of the State concerned (a) Nagaland (b) Kerala
493. The Chief Justice of the High Court is ap- (c) Assam (d) Tripura
pointed by 501. In which case did the Supreme Court re-
(a) The Chief Justice of India store primacy of fundamental Rights over
(b) The Chief Minister of the State the Directive Principles?
(c) The Governor of the State (a) A. K. Gopalan Case
(d) The President of India (b) Golaknath Case
494. Which list contains the subjects over (c) Minerva Mills Case
which both the Centre and States have ju- (d) Keshvananda Bharati Case
risdiction? 502. Which Article of the Constitution permits
(a) Concurrent List the Supreme Court to have a seat outside
(b) Central List Delhi?
(c) Joint List (a) Article 141 (b) Article 136
(d) None of these (c) Article 130 (d) Article 124

CLAT.indb 213 31/03/2009 11:41:57


214 CONSTITUTIONAL LAW

503. The law declared by the Supreme Court I. Chief Justice of the Supreme Court.
shall be binding on all courts within the II. Former Chief Justice of the High
territory of India in accordance with Ar- court
ticle III. Other Judges of the High court
(a) 124 (b) 130 IV. Governor of the State concerned
(c) 136 (d) 141 (a) I and III (b) II and III
504. Preventive Detention for more than two (c) I and IV (d) I, III and IV
months can be ordered by the recommen- 512. The words ‘social, secular’ and ‘the uni-
dation of an Advisory Board whose Chair- ty and integrity of the nation’ have been
man is added to the Indian Constitution by which
(a) the President amendment?
(b) the Chief Justice of India (a) 38th (b) 40th
(c) a judge of the Supreme Court (c) 42nd (d) 48th
(d) a judge of a High Court 513. Who is entitled to initiate a Bill for Consti-
505. Madras High Court has the jurisdiction tutional amendments?
over (a) State Legislative Assemblies
(a) Tamil Nadu and Kerala (b) Lok Sabha only
(b) Tamil Nadu and Puducherry (c) Either House of the Parliament
(c) Kerala and Lakshadweep (d) Rajya Sabha only
(d) Lakshadweep and Tamil Nadu 514. For the creation of a new state in India,
506. Who among the following has the Consti- the amendment of the Constitution will
tutional authority to make rules regarding require
the manner of enforcing the orders of the (a) simple majority in Parliament
Supreme Court? (b) simple majority in Parliament and
(a) Chief Justice of India approval of majority of states
(b) Attorney General of India (c) two-third majority in Parliament
(c) President (d) two-third majority in each House of
(d) Union Law Minister Parliament and approval of majority
507. The Supreme Court of India is created by of states.
(a) the Constitution 515. The 42nd amendment to the Indian Con-
(b) a Parliament Act, 1950 stitution is notable because it gives
(c) a Presidential Order, 1951 (a) primacy to Fundamental Rights over
(d) Indian Independence Act, 1947 Directive Principles
508. At present, the Supreme Court consists of (b) primacy to Directive Principles over
how many judges besides the Chief Jus- Fundamental Rights
tice? (c) Special treatment to Jammu & Kash-
(a) 20 (b) 18 mir
(c) 24 (d) 25 (d) Special treatment to Sikkim.
509. The power of Judicial Review has been 516. 44th amendment deals with
conferred on the Supreme Court to check (a) abolition of Right to Property as a
the arbitrary power of the Fundamental Right
(a) Executive (b) curbing the Fundamental Rights
(b) Parliament (c) curbing the jurisdiction of High
(c) Council of Ministers Courts
(d) President (d) extension of reservation of seats in
510. The judges and magistrates of the Civil Parliament
Courts are appointed by the 517. By which amendment was the power of
(a) Subordinate Judge the Lok Sabha to amend the Constitution
(b) Chief Justice of High Court affirmed?
(c) Governor (a) 24th (b) 42nd
(d) District Magistrate (c) 45th (d) 42nd
511. The Chief Justice of a High Court is ap- 518. The Constitution 44th amendment
pointed by the President in consultation (renumbered 43rd)
with (a) ensures the right to property

CLAT.indb 214 31/03/2009 11:41:57


CONSTITUTIONAL LAW 215

(b) ensures the Press Freedom amendment of the Constitution is neces-


(c) limits the power of the government to sary to join this?
proclaim internal emergency (a) C. Rajagopalachari
(d) restored to the High Courts and the (b) J. B. Kripalani
Supreme Court their jurisdiction to (c) Jawaharlal Nehru
consider the validity of any Central or (d) Mahatma Gandhi
State Law. 529. The 73rd amendment to the Indian Consti-
519. As a result of the 42nd amendment, which tution pertains to which of the following?
of the following has emerged as supreme (a) Statehood of Delhi
in the Indian constitutional framework? (b) Panchayati Raj
(a) Parliament (b) Executive (c) Land Reforms Law
(c) Judiciary (d) Constitution (d) Inclusion of Konkani Manipuri and
520. The 42nd Constitution Amendment Bill Nepali in the Eighth Schedule
was passed in the year 530. The Right to Property ceased to be a fun-
(a) 1976 (b) 1977 damental Right from
(c) 1979 (d) 1980 (a) 1975 (b) 1976
521. Indian Constitution was amended for the (c) 1978 (d) 1979
first time in 531. For amendment of the Constitution, a Bill
(a) 1976 (b) 1977 may be introduced in
(c) 1979 (d) 1951 (a) Rajya Sabha
522. The 44th Amendment modifies or nulli- (b) Lok Sabha
fies or nullifies a number of provisions of (c) Either House of Parliament
which amendment? (d) Either chamber of State Legislature of
(a) 43rd (b) 42nd Union Parliament.
(c) 41st (d) 40th 532. Which of the following Constitutional
523. Of all the amendments in the Indian Con- Amendments restores the Legislative As-
stitution, the most comprehensive and sembly and a Cabinet headed by the Chief
controversial amendment was Minister in Delhi
(a) 42nd (b) 43rd (a) 62nd (b) 68th
(c) 44th (d) 45th (c) 72nd (d) 74th
524. If India decides to have a Presidential 533. The Constitutional Bill regarding de-link-
form of government, the first and foremost
ing of religion from politics was
amendment has to be made affecting the
(a) 77th (b) 78th
(a) system of judiciary
(c) 79th (d) 80th
(b) composition of Parliament
534. The Indian Constitution provides how
(c) executive-legislature relationship
many methods of amendment?
(d) provisions pertaining to fundamental
(a) One (b) Two
rights
(c) Three (d) More than 3
525. Which amendment restored the power of
535. For all purposes, an amendment Bill has to
judicial review to the Supreme Court and
be passed by
High Courts after it was curtailed by the
(a) only Lok Sabha
42nd amendment?
(b) only Rajya Sabha
(a) 42nd (b) 43rd
(c) 44th (d) 56th (c) both the Houses of Parliament
526. Which amendment Bill proposed to grant (d) both the Houses of Parliament as well
statehood to Delhi? as the State Legislature
(a) 44th (b) 70th 536. For which of the following, the amend-
(c) 72nd (d) 77th ment bill has to be ratified by at least one-
527. By which amendment Bill did the Parlia- half of the State Legislatures?
ment lower the voting age from 21 to 18 I. The manner of election of the Presi-
years? dent
(a) 42nd (b) 44th II. Extent of the executive power of the
(c) 62nd (d) 73rd Union and the states
528. Who gave that no person shall be a Presi- III. Representation of States in the Parlia-
dent for more than two terms, and that no ment

CLAT.indb 215 31/03/2009 11:41:57


216 CONSTITUTIONAL LAW

(a) I and II (b) II and III (b) Supreme Court


(c) I and III (d) I, II and III (c) High Court
537. Which of the following is not true about (d) Election Commission
an amendment Bill? 545. In which year were the first general elec-
(a) There is no limitation to the power of tions held in India?
the Parliament to amend the constitu- (a) 1947–48 (b) 1948–49
tion. (c) 1950–51 (d) 1951–52
(b) No amendment made can be called in 546. Who appoints the Chief Election Commis-
question in any court. sioner of India?
(c) A deadlock between two Houses on (a) President
an amendment Bill can be resolved by (b) Prime Minister
a joint session of the Parliament. (c) Parliament
(d) The State Legislatures cannot initiate (d) Chief Justice of India
any Bill for amendment. 547. Parliamentary elections are organized by
538. The Parliament can amend the Constitu- the
tion by way of (a) President
(a) addition (b) variation (b) Prime Minister
(c) repeal (d) All of these (c) Lok Sabha
539. Which of the following cannot be altered (d) Election Commission
by the Parliament by ordinary legislative 548. How many days time is given for the can-
procedure? didates to file their nominations from the
(a) Acquisition and termination of citi- date of notification of the election?
zenship (a) 2 days (b) 4 days
(b) Privileges of the Parliament
(c) 6 days (d) 8 days
(c) Composition of State Legislative
549. The delimitation of the territorial constitu-
Councils
encies in India is the responsibility of
(d) Representation of states in the Parlia-
(a) President
ment
(b) Governor
540. In India, political parties are given recog-
(c) Election Commission
nition by
(d) Parliament
(a) President
550. The Electorate means
(b) Law Commission
(a) Those people who do not have the
(c) Speaker of Lok Sabha
(d) Election Commission right to elect their representatives.
541. The Vice-President is elected by an Elec- (b) All the citizens who possess the right
toral College consisting of members of to vote and elect their representatives
(a) both Houses of the Parliament in an election.
(b) Lok Sabha (c) Those people who have the right to be
(c) Rajya Sabha elected.
(d) both Houses of the Parliament and (d) Those people who have the right to
State Legislative Assemblies live in a country.
542. The Election Commission does not con- 551. The Panchayati Raj system is aimed at
duct the elections to the (a) establishing economic prosperity of
(a) Lok Sabha the villagers
(b) Rajya Sabha (b) making villagers self-sufficient
(c) A Local bodies (c) having democratic decentralization
(d) President’s elections (d) providing political base for parties in
543. The maximum age prescribed for election villages
as President is 552. The Panchayati Raj institutions in India
(a) 58 years (b) 60 years get their funds mainly from
(c) 62 years (d) No such limit (a) Governments grants
544. The power to decide an election petition is (b) Local taxes
vested in the (c) Voluntary contributions
(a) Parliament (d) Property tax

CLAT.indb 216 31/03/2009 11:41:57


CONSTITUTIONAL LAW 217

553. Which of the following States was the first 561. The members of the State PSC retire at
to establish the new Panchayati Raj insti- the
tutions? (a) age of 60 years
(a) Bihar (b) age of 62 years
(b) Rajasthan (c) age of 65 years
(c) Andhra Pradesh (d) no specific age
(d) Maharashtra 562. The status of a recognized Opposition par-
554. In which part of the Constitution is the ty in the Lok Sabha is secured only when
State enjoined to establish Panchayati Raj the percentage of the total seats in the Lok
institutions? Sabha won by the political party is
(a) Directive Principles (a) 5 (b) 8
(b) Preamble (c) 10 (d) 15
(c) Fundamental Rights 563. Indira Swahney vs Union of India is a case
(d) Fundamental Rights popularly known as
555. The Union Public Service Commission (a) Mandal judgment
was created by (b) Ayodhya judgment
(a) the Civil Services Act,1951 (c) Suicidal judgment
(b) the Constitution (d) Election Commission judgment
(c) Parliament’s Act in 1952 564. What is the meaning of the Latin phrase
(d) Presidential Order of 1952 “audi alteram partem”?
556. The members of the UPSC are (a) No one shall be condemned unheard
(a) appointed by the President (b) No one shall be judge in his own
(b) appointed by the Council of Minis- cause
ters (c) Rights are associated with duties
(c) elected by the people (d) None of the above
(d) elected by the Parliament 565. Who was the first Attorney General of
557. The UPSC consists of a Chairman and India?
(a) six other members (a) Nani A. Palkhiwala
(b) eight other member (b) Niren Dey
(c) eleven other members (c) C. R. Daphtary
(d) unspecified number of members (d) M. C. Setalvad
558. The strength of the UPSC 566. The name of one person has been mostly
(a) is determined by the Parliament associated with cases relating to environ-
(b) is determined by the President from ment in the Supreme Court. The person is
time to time (a) Medha Patkar
(c) has been permanently fixed by the (b) Arun Shourie
Constitution (c) Dr Subramanyam Swamy
(d) was determined by the Presidential (d) M. C. Mehta
Ordinance in 1952 567. Supreme Court’s advisory jurisdiction is
559. The members of UPSC hold office provided under Article …. of the Consti-
(a) for a term of six years or till they at- tution
tain the age of 62 years (a) 226 (b) 143
(b) for term of six years or till they attain (c) 124 (d) 123
the age of 65 years 568. An appeal against the order of Central Ad-
(c) for a term of five years irrespective of ministrative Tribunal (dealing in Service
the upper age limit matters) lies to
(d) during the pleasure of the President. (a) President
560. The members of the UPSC can be removed (b) Supreme Court
from their office during their tenure by (c) High Court
(a) the Parliament (d) No provision of appeal
(b) the President 569. In which case, Supreme court held that
(c) the President on the report of the Su- Parliament has no power to amend funda-
preme Court of India mental Rights?
(d) the President of the report of the Par- (a) Shankri Prasad case
liament. (b) Golaknath case

CLAT.indb 217 31/03/2009 11:41:57


218 CONSTITUTIONAL LAW

(c) Sujjan Singh case 580. Rights and duties are


(d) None of these (a) Coordinated
570. In …. Supreme Court held that Judicial (b) Corelated
Review is a basic feature of Constitution (c) Separated
and cannot be amended (d) None of the above
(a) Minerva Mills case 581. When a person aids the commission of an
(b) Golaknath case offence, he is called
(c) Keshvananda Bharti case (a) Abeyer (b) Accomplice
(d) None of the above (c) Accessory (d) Abettor
571. Which is the only Union Territory having
582. A written statement in the name of a per-
a High Court of its own?
son by whom it is voluntarily signed and
(a) Delhi
(b) Chandigarh sworn to is called
(c) Andaman and Nicobar Islands (a) Affirmation (b) Advowson
(d) None of the above (c) Affidavit (d) Affray
572. A sentence of death by a lower Sessions 583. A person who makes an affidavit or depo-
court sition is called
(a) is operational if no appeal is made to (a) Denizen (b) Demesme
higher courts (c) Deponent (d) Deodand
(b) must be confirmed by the High Court 584. A formal instrument by which one per-
(c) must be confirmed by the Supreme son empowers another to represent him is
Court called
(d) must be confirmed by the President (a) Power
573. A minimum of …. Judges of Supreme (b) Will
Court hear a constitutional matter. (c) Instrument of representation
(a) 3 (b) 5 (d) Power of Attorney
(c) 6 (d) 7 585. A lawyer appointed by court to represent a
574. Which of the following is not a source of poor litigant is
law in India? (a) Lok Nyaya Pradayak
(a) Precedent
(b) Amieus Curiae
(b) Legislation
(c) Legal Aid Man
(c) Public Opinion
(d) Custom (d) None of the above
575. Who is associated with drafting of the In- 586. LLB stands for
dian Penal Code? (a) Legalite Lawyers Baccili
(a) Sir Fredric Pollock (b) Bachelor of Law
(b) Mulla (c) Legum Baccalaureus
(c) Lord Macaulay (d) None of the above
(d) Sir H.S. Maine 587. There are … Schedules in Constitution.
576. Can a woman be charged with the offence (a) Five (b) Eight
of adultery? (c) Nine (d) Twelve
(a) Circumstantial 588. Who among the following is fourth Pillar
(b) Yes of Indian Constitution?
(c) No (a) Attorney General
(d) None of the above (b) Lieutenant General
577. How many persons are required to consti- (c) Advocate General
tute a conspiracy? (d) Comptroller and Auditor General
(a) five (b) three 589. Voting right comes under
(c) four (d) two (a) Fundamental Right
578. Which Section defines murder under
(b) Legal Right
I.P.C?
(c) Constitutional Right
(a) 302 (b) 301
(c) 300 (d) 299 (d) None of the above
579. Possession is said to be…of ownership. 590. The four pillars of democracy are
(a) 7 points (b) 9 points (a) The President, the Prime Minister, the
(c) 5 points (d) synonymous people and the Press

CLAT.indb 218 31/03/2009 11:41:58


CONSTITUTIONAL LAW 219

(b) The Parliament, the President, the Ju- (c) Directive Principles
diciary and the Press (d) Fundamental Duties
(c) The Parliament, the Executive, the Ju- 600. Which Article of the Constitution declared
diciary and the Press Hindi in Devanagari Script as official lan-
(d) None of the above guage?
591. Who among the following had the longest (a) Article 143 (b) Article 243
tenure as the Chief Justice of India? (c) Article 334 (d) Article 343
(a) P. N. Bhagwati 601. Which of the following does not come un-
(b) M. H. Beg der the purview of the Election Commis-
(c) Y. V. Chandrachud sion?
(d) H. J. Kania (a) To conduct the election of PM of In-
592. Theocracy is dia
(a) an authoritarian regime (b) To conduct the election of President
(b) the art of stealing of India
(c) a religious state (c) To give recognition to the political
(d) a state created as a result of coup parties
d’etat (d) To prepare electoral rolls
593. Attorney General of India means 602. An Indian citizen has the right to cast his/
(a) Advisor to the Supreme Court her vote after attaining the age of
(b) President of legal system in India (a) 16 years (b) 18 years
(c) Supreme Judicial Power (c) 21 years (d) 25 years
(d) Top most legal advisor of the Central 603. Lok Sabha elections are held after every
government (a) 3 years (b) 4 years
594. Although he is not a member of both (c) 5 years (d) 6 years
Houses of Parliament yet he can partici- 604. The powers of the Election Commission
pate in their deliberations. Who is he? are given in which of the following Article
(a) Election Commissioner of the Constitution?
(b) Auditor-General (a) Article 286 (b) Article 356
(c) Attorney General (c) Article 324 (d) Article 382
(d) Solicitor General 605. The maximum strength of Lok Sabha en-
visaged by the Constitution is now
595. Once in how many years, the President re-
(a) 552 (b) 548
constructs the Finance Commission?
(c) 545 (d) 542
(a) 2 (b) 3
606. The Vice-President is eligible for re-elec-
(c) 4 (d) 5
tion by
596. When there is dispute on the concurrent
(a) Constitutional Provision
list which enactment stands—central or
(b) Convention
state?
(c) Law of Parliament
(a) Central Act
(d) Supreme Court judgement
(b) State Act
607. Graduates constitute a separate constitu-
(c) The enactment which obtains the
ency for the election of specified number
President’s Approval
of members to the
(d) Both the Acts would be cancelled (a) Nyaya Panchayat
597. Which is the official language of Jammu (b) State Legislative Council
and Kashmir? (c) Rajya Sabha
(a) Hindi (b) Dogri (d) State Legislative Assembly
(c) Kashmiri (d) Urdu 608. The historic Panchayati Raj Bill was ad-
598. Which language was added in the 8th opted by the Parliament in 1992 as which
Schedule of the Constitution? amendment?
(a) Marathi (b) Konkani (a) 73rd (b) 72nd
(c) Kashmiri (d) Sindhi (c) 74th (d) 68th
599. Which part of the Constitution can be agi- 609. In which of the following states was the
tated in a Court of Law? Panchayati Raj first introduced?
(a) Fundamental Rights (a) Rajasthan (b) Gujarat
(b) Preamble (c) UP (d) Bihar

CLAT.indb 219 31/03/2009 11:41:58


220 CONSTITUTIONAL LAW

610. The primary part of the three tier system 620. The three languages included in the Eighth
of Panchayati Raj is Schedule of the Constitution by the 71st
(a) Village Panchayat amendment are
(b) Gram Panchayat (a) Konkani, English, Manipuri
(c) Zila Parishad (b) Konkani, Nepali, Maithili
(d) Panchayat Samiti (c) Konkani, Manipuri, Nepali
611. Panchayati Raj system is (d) Manipuri, Nepali, Dogri
(a) an organization which works accord- 621. Planning Commission is a
ing to the Central Government statute (a) Statutory body
(b) a Constitutional establishment (b) Executive body
(c) established to assist the working of (c) Autonomous body
the government (d) None of the above
(d) None of the above 622. The total number of schedules now in In-
612. Special representation in Panchayati Raj dian Constitution is
Institutions is given to (a) 6 (b) 8
(a) Women (c) 10 (d) 12
(b) Cooperative Societies 623. 26th January is India’s
(c) Backward Classes (a) Independence Day
(d) All of the above (b) Republic Day
613. Which of the following states has no (c) Revolution Day
Panchayati Raj institution at all? (d) Parliament Day
(a) Assam (b) Kerala 624. Which of the following was an associ-
(c) Tripura (d) Nagaland ate State of India before becoming a full
614. Who is the executive head of a Municipal fledged state?
Corporation? (a) Meghalaya (b) Mizoram
(a) Mayor (b) Commissioner (c) Sikkim (d) Manipur
(c) Secretary (d) Deputy Mayor 625. When did Delhi become a Union Terri-
615. Which of the following is not a condition tory?
for becoming a citizen of India? (a) 1947 (b) 1950
(a) Birth (c) 1956 (d) 1960
(b) Acquiring Property 626. A Union Territory does not have
(a) a capital
(c) Descent
(b) a recognized official language
(d) Naturalization
(c) a Governor/Administrator
616. Why is the year 1952 important in Indian
(d) an annual plan outlay
History?
627. The first President of independent India
(a) State Reorganization Act was passed
was
(b) Haryana State was split
(a) Dr Rajendra Prasad
(c) First official census in India was held
(b) B. R. Ambedkar
(d) First general elections to the Lok Sab-
(c) Dr S. Radhakrishnan
ha were held
(d) J. L. Nehru
617. The reference to Hindus in Article 25 of
628. Term of Comptroller and Auditor General
the Constitution does not include
of India is
(a) Buddhists (b) Jains (a) 6 years
(c) Parsees (d) Sikhs (b) 65 years of age
618. English is the official language of which (c) 6 years or 65 years of age whichever
of the following states? is earlier
(a) Manipur (b) Nagaland (d) None of these
(c) Karnataka (d) Andhra Pradesh 629. Indian Constitution recognizes minorities
619. The first Law Officer of the Government on the basis of
of India is (a) Religion (b) Caste
(a) Chief Justice of India (c) Population (d) Colour
(b) Law Minister 630. Which article of Indian constitution pre-
(c) Attorney General scribes Hindi in Devanagri script as the
(d) Auditor-General official language of the union?

CLAT.indb 220 31/03/2009 11:41:58


CONSTITUTIONAL LAW 221

(a) 341 (b) 342 (d) any Court of Law within the territory
(c) 343 (d) 346 of India.
631. Article 370 of the Constitution deals with 641. Who among the following have the right
(a) protection of interest of minorities to vote in the elections to both the Lok
(b) powers of President to grant pardon Sabha and the Rajya Sabha?
(c) formation of new states and alterna- (a) Elected members to the Lower House
tion of boundaries of Parliament
(d) temporary provision with respect to (b) Elected members of the Upper House
Jammu and Kashmir of Parliament
632. Interpretation of Indian Constitution is (c) Elected members of the Upper House
based on the spirit of the of State Legislature
(a) Preamble (d) Elected members of the Lower House
(b) Fundamental Right of State Legislature
(c) Directive Principles 642. Which of the following is found in the
(d) Fundamental Duties seventh schedule of the Constitution?
633. Welfare State is the aim of (a) List of languages
(a) Anarchism (b) Subjects in State and Central List
(b) Individualism (c) Lists of oaths and affirmations
(c) Scientific Socialism (d) Duties of President and Vice-Presi-
(d) Democratic Socialism dent
634. Which of the following is not a Union Ter- 643. In which year were the Indian states rec-
ritory? ognized on a linguistic basis?
(a) Lakshadweep (b) Puducherry (a) 1947 (b) 1951
(c) Nagaland (d) Daman and Diu
(c) 1956 (d) 1966
635. The Tenth Schedule to the Constitution of
644. In the interim government formed in 1946,
India deals with
the Vice-President of the Executive Coun-
(a) Anti-defection Law
cil was
(b) Languages of the Union
(a) Dr Rajendra Prasad
(c) Lists of Legislative Business
(b) C. Rajagopalachari
(d) Special status of Jammu and Kashmir
(c) Jawaharlal Nehru
636. The only President of India who was
(d) Dr S. Radhakrishnan
elected unopposed is
645. Urdu was adopted recently as a state lan-
(a) Dr S. Radhakrishnan
(b) Dr Zakir Hussain guage by
(c) Neelam Sanjeev Reddy (a) UP (b) J & K
(d) Fakhruddin Ali Ahmed (c) Bihar (d) Punjab
637. The Finance Commission is appointed af- 646. Of the total population of India, Christians
ter every comprise about
(a) 2 years (b) 3 years (a) 4% (b) 2.5%
(c) 4 years (d) 5 years (c) 3% (d) 7%
638. Which language was added as the 15th of- 647. All India Services can be created by the
ficial language to the 8th Schedule of the (a) President
Constitution? (b) UPSC
(a) Hindi (b) Urdu (c) Prime Minister
(c) Sindhi (d) Marathi (d) Parliament
639. Golden Jubilee of Parliament of India was 648. The first leader of opposition to be given
celebrated on the status and rank of a Cabinet Minister
(a) 13 May 2002 (b) 8 May 2002 was
(c) 26 April 2002 (d) 18 May 2002 (a) A. K. Gopalan (b) Y. B. Chavan
640. The Attorney General of India has the (c) C. M. Stephen (d) P. Upendra
right to audience in 649. Functions of the UPSC are mentioned un-
(a) any High Court der Article
(b) Supreme court (a) 325 (b) 320
(c) any Sessions Court (c) 322 (d) 324

CLAT.indb 221 31/03/2009 11:41:58


222 CONSTITUTIONAL LAW

650. Next to Hindi, language spoken by the (c) Prime Minister


largest number of people in the Indian (d) All of these
subcontinent is 660. High Courts take up the writs like Habeas
(a) Bengali (b) Tamil Corpus under … Constitutional authority?
(c) Telugu (d) Marathi (a) Article 368 (b) Article 326
651. Articles 25, 26, 27, 28 refer to the rights of (c) Article 268 (d) Article 226
…. . 661. The Comptroller and Auditor General of
(a) Religious Minorities India has been described as the friend, phi-
(b) Children losopher and guide of
(c) Employees (a) The Public Accounts committee
(d) Women (b) The Committee on Public Undertak-
652. “Equal pay for equal work for both males ings
and females”. Which article has proposed (c) All the Parliamentary Committees
this? (d) None of these
(a) Article 48 (b) Article 39 662. Which Article relates to preventive deten-
(c) Article 14 (d) Article 25 tion?
653. Which article gave special recognition to (a) 21 (b) 22
Jammu and Kashmir state? (c) 23 (d) 24
(a) Article 391 (b) Article 381 663. How many administrative items are in-
(c) Article 370 (d) Article 361 cluded in the Central List?
654. The Constitution amendment bill can be (a) 107 (b) 97
introduced (c) 87 (d) 67
(a) Only in Rajya Sabha 664. How many administrative items are in-
(b) Only in Lok Sabha cluded in the State List?
(c) In both Houses of Parliament only (a) 36 (b) 46
(d) In either house of Parliament (c) 56 (d) 66
655. The seventh schedule of Constitution con- 665. When was the Planning Commission
tains… formed?
(a) List of Languages (a) 15 March 1950
(b) States (b) 15 March 1952
(c) Central List, State List and Concur- (c) 15 March 1954
rent List (d) 15 March 1956
(d) None of these 666. Election Commission is….
656. The President be removed before his term (a) A Govt. Institution
expires by (b) A Legal Body
(a) Vice-President (c) An independent Institution
(b) Minister (d) A private body
(c) Supreme court 667. “A noon among stars, sun controlling the
(d) Impeachment movements of several planets” to whom
657. On what basis the states have their repre- does this description refer to?
sentation in Rajya Sabha? (a) President
(a) Population (b) Vice-President
(b) Area (c) Prime Minister
(c) Lok Sabha seats (d) Speaker
(d) None of these 668. De facto recognition means
658. Which list consists of maximum items? (a) Recognition in religion
(a) Concurrent List (b) Recognition in law
(b) Central List (c) Recognition in fact
(c) State List (d) None of these
(d) None of these 669. De Jure recognition means
659. President appoints …. among the follow- (a) Recognition in religion
ing. (b) Recognition in law
(a) Election Commissioner (c) Recognition in fact
(b) Attorney General (d) None of these

CLAT.indb 222 31/03/2009 11:41:58


CONSTITUTIONAL LAW 223

670. Who said that the Supreme Court in In- 677. Who is the head of the Three wings of the
dia has the highest powers which no other Defence Forces?
court in the world possess? (a) Defence Minister
(a) Alladi Krishna Swamy Iyyer (b) Army General
(b) Sardar Vallabh Bhai Patel (c) Vice Admiral
(c) Jawaharlal Nehru (d) President
(d) Mahatma Gandhi 678. Universal Adult Franchise was first time
671. Liberty-Equality-Fraternity, who is the introduced in India under
founder of these slogans? (a) The New Constitution of India
(a) Montesk (b) Russo (b) The Government of India Act, 1935
(c) Einstein (d) John Laun (c) The Indian Independence Act, 1947
672. In democracy, the government belongs to (d) None of the above
the … 679. In a Parliamentary system, the executive is
(a) Ruling Experts responsible to
(b) Able Leaders (a) The legislature
(c) Political Parties (b) The judiciary
(d) People (c) The people directly
673. To whom are these words attributed about (d) None of the above
the Fundamental Rights “…they are in- 680. What is the qualification for obtaining vot-
tended to make all citizens and persons ing right in India?
appreciate that the paramount law of the (a) Caste (b) Property
land has swept away privileges and has (c) Age (d) Education
laid down that there is to be perfect equal- 681. What is called the soul of the Constitu-
ity between one section of the community tion?
and another in the matter of all these rights (a) Fundamental Rights
which are essential for the material and (b) Preamble
moral perfection of men”. (c) Directive Principles
(a) Jawaharlal Nehru (d) Supreme Court
(b) B. R. Ambedkar 682. According to their constitution, which is
(c) C. Rajagopalachari called the official Book of Authority in
(d) Justice Sapru Britain?
(a) White Paper (b) Red Book
674. The Fundamental Rights are
(c) Yellow Book (d) Blue Book
(a) Not justiciable
683. Who advises the Central Government on
(b) Justiciable
legal issues?
(c) Both (a) and (b)
(a) Advocate General of India
(d) Not enforceable at all
(b) President of India
675. The Preamble to the Indian Constitution
(c) Attorney General of India
seeks to secure to Indian citizens
(d) Chief Justice of India
(a) Justice, Liberty, Equality and Frater-
684. An Obiter dicta by a judge is a
nity
(a) Statement of opinions made, by the
(b) A powerful army
judge, while deciding the point at is-
(c) A strong Government
sue.
(d) None of these (b) Statement within the orbit of mores of
676. The Preamble to the Indian Constitution the society.
originally provided for setting up a Sov- (c) Statement within the orbit of facts of
ereign, Democratic Republic. The present the case.
provision is Sovereign, Socialist, Secular, (d) Statement not covered by any of the
Democratic Republic. This modification above classifications.
was introduced under the Prime Minister- 685. Which part of the Constitution is justicia-
ship of ble?
(a) Jawaharlal Nehru (a) Fundamental Rights
(b) Indira Gandhi (b) Preamble
(c) Lal Bahadur Shastri (c) Fundamental Duties
(d) Morarji Desai (d) Directive Principles

CLAT.indb 223 31/03/2009 11:41:58


224 CONSTITUTIONAL LAW

686. In order to be a voter, citizens must be (a) Jawaharlal Nehru


(a) Twenty five years old (b) Mahatma Gandhi
(b) Twenty years old (c) K. M. Munshi
(c) Twenty one years old (d) Visweswarayya
(d) Eighteen years old 696. Rajya Sabha is also called
687. A bill becomes an Act (a) Council of Nominated Members
(a) When it is passed by both the Houses (b) Council of Representatives
and assented to by the President (c) Council of States
(b) When the Prime Minister assents to it (d) None of the above
(c) It is passed by both the Houses 697. Who can legislate on inter-state wager dis-
(d) None of these putes?
688. What is a First Information Report (FIR)? (a) Parliament
(a) It is a private complaint given to a (b) President
magistrate by a person regarding com- (c) Supreme Court
mission of an offence as seen by him.
(d) They cannot make any Act
(b) It is an information given by an infor-
698. National Development Council is a …
mant to the police relating to the com-
(a) Creation of the executive body
mission of cognizable offence.
(b) Constitutional body
(c) It is a report given to a magistrate
about law and order situation by the (c) Cabinet’s Secretariat
police. (d) Joint Right
(d) None of the above 699. Lok Sabha is also called
689. Plaint means…. (a) House of the Government
(a) The claim statement filed by the plain- (b) House of Law
tiff (c) House of the People
(b) Statement which selects the facts (d) House of the Representatives
(c) The cross complaint by an accused 700. Which Article has abolished the child la-
(d) Statement filed by police bour?
690. If a will is altered or new additional matter (a) Article 24 (b) Article 27
is added it is called (c) Article 29 (d) Article 34
(a) Cagnisance (b) Code 701. Who among the following were the mem-
(c) Codicil (d) None of these bers of the drafting committee of the Con-
691. Within how much time should a person stitution?
arrested be brought to the court? 1. K. M. Munshi
(a) 12 hours (b) 20 hours 2. Jawahar Lal Nehru
(c) 24 hours (d) Immediately 3. Alladi Krishnaswami
692. Who is a plaintiff? 4. Sardar Patel
(a) Witness Direct Witness (a) 1, 3 and 4 (b) 1 and 4
(b) Complainant (c) 1, 3 (d) 2, 3 and 4
(c) One who files a complaint 702 The members of the Constituent Assem-
(d) None of these bly were
693. Where the Supreme Court Judges can (a) indirectly elected by the Provincial
practice after retirement? Assemblies
(a) At District and Session Courts (b) all nominated by the British Govern-
(b) At High Courts ment
(c) At Supreme Court
(c) directly elected by the people
(d) At nowhere
(d) all nominated by the Indian National
694. In India, who is the legal advisor to the
Congress and the Muslim League
State Government?
703. The constitutional Adviser to the Constitu-
(a) Minister of Law
ent Assembly was
(b) Advocate General
(c) Chief Justice of High (a) Gopalaswamy
(d) None of the above Court (b) H. N. Kunzru
695. In India, who was the first to think of the (c) Sachchidanand Sinha
Planning System? (d) B. N. Rao

CLAT.indb 224 31/03/2009 11:41:58


CONSTITUTIONAL LAW 225

704. The Indian Constitution establishes a Sec- (a) Various enactments by Parliament
ular State, meaning (b) Schedule I of the Constitution
I. The state treats all religions equally (c) The Citizenship Act, 1955
II. Freedom of faith and worship is al- (d) Part II of the Constitution
lowed to the people 711. Which of the following is a feature of the
III. Educational institutions, without ex- India constitution?
ception, are free to impart religious (a) It has introduced the principle of uni-
instruction versal male suffrage
IV. The State makes no discrimination on (b) It affirms our belief in peaceful co-
the basis of the religion in matters of existence
employment (c) It vests sovereignty in the Executive
(a) I, II, III and IV (d) It declares India a Secular state
(b) I, II and IV 712. How would you describe the Indian Con-
(c) II, III and IV stitution?
(d) I and II (a) It is not a written Constitution
705. Who adopted the Constitution of India on (b) One of the most elaborate constitu-
the 26 November 1949? tions in the world
(a) Parliament of India (c) Very small and compact
(b) People of India (d) Medium in size
(c) Representative of the people of India 713. The States reorganization in 1956 created
in Constituent Assembly (a) 14 States and 6 Union Territories
(d) Cabinet (b) 15 States and 9 Union Territories
706. The Drafting Committee of the Constitu- (c) 17 States and 6 Union Territories
tion did not have as its member (d) 17 States and 9 Union Territories
(a) N. Gopalaswami Ayyangar 714. India in 2000 was a Union of
(b) Sardar Patel (a) 14 States (b) 20 States
(c) Alladi Krishnaswami (c) 25 States (d) 28 States
(d) K. M. Munshi 715. Which of the following is a feature of the
707. To produce the Constitution the Constitu- Indian Constitution?
ent Assembly took (a) Presidential Government
(a) 5 years 5 months and 5 days (b) Double citizenship
(b) 4 years 11 months and 11 days
(c) Unitary Government
(c) 3 years 10 months and 10 days
(d) Protection of minorities and backward
(d) 2 years 11 months and 18 days
classes
708. Consider the following words:
716. India has been described by the Constitu-
I. Socialist
tion as
II. Democratic
(a) A Union of States
III. Sovereign
(b) Quasi-federal
IV. Secular
(c) A Federation of States
Choose the response that gives the correct
(d) None of these
order in which these words occur in the
717. India is a Secular state because
Preamble.
(a) State has no religion of its own
(a) III IV II I
(b) III I IV II (b) There will be no discrimination on
(c) III IV I II grounds of religion or caste
(d) IV I III II (c) All religions are equally protected
709. The preamble enshrines the ideals of ‘lib- (d) Of all the above
erty, equality and fraternity’—ideas in- 718. In a Parliamentary system, the executive is
spired by the responsible
(a) French Revolution (a) To the legislature
(b) Irish Revolution (b) To the Judiciary
(c) Russian Revolution (c) Directly to the people
(d) US Constitution (d) None of these
710. Details on how the citizenship may be ac- 719. In which of the following points is the In-
quired and terminated are enumerated in dian Constitution similar to that of USA?

CLAT.indb 225 31/03/2009 11:41:58


226 CONSTITUTIONAL LAW

(a) Directive Principles of State Policy 725. In which of the following ways can Indian
(b) Fundamental Rights citizenship be acquired?
(c) Rigid Constitution (a) By naturalization
(d) Rule of Law (b) By descent
720. “We the people of India having solemnly (c) By registration
resolved to constitute India into a Sover- (d) all of these
eign Democratic Republic and to secure to 726. A citizen acquiring citizenship through
all citizens…” From this statement we can naturalization
conclude (a) may acquire it in special cases only
(a) Sovereignty lies with the elected rep- (b) must either reside in India or serve the
resentatives of the people of India Government of India for at least one
(b) Sovereignty lies with the executive of year immediately preceding the date
the country of application
(c) Sovereignty lies with the President (c) may keep the citizenship of any other
(d) Sovereignty lies with the people of country
India (d) may belong to any country without
721. The basic aims and objectives of the Con- qualification
stitution are discussed in 727. Which of the following classes cannot
(a) Part III dealing with the Fundamental avail of some fundamental rights available
Rights to citizens?
(b) Chapter IV dealing with the Directive (a) Friendly aliens
Principles of State Policy (b) Enemy aliens
(c) The Preamble (c) All aliens
(d) Nowhere specific (d) None of these
722. Which part of the Constitution reflects the 728. Which of the following are matched
mind and ideals of the framers? wrongly?
(a) Emergency Provisions I. Habeas Corpus—literally means ‘to
(b) Fundamental Rights have a body’
(c) Directive Principles II. Mandamus—commands a person to
(d) Preamble perform a public duty
723. The Preamble to the Constitution of India III. Quo warranto—issued to a lower
reads court to stop proceedings in a case
(a) We, the members of the Constitution IV. Prohibition—issued to a lower court
Assembly, represent the people of In- quashing a decision or order
dia… enact this Constitution. (a) I and II (b) III only
(b) We, the people of India…through the (c) II, III and IV (d) III and IV
representatives of this Constituent As- 729. A proclamation of national emergency au-
sembly….enact and give to ourselves tomatically suspends
this Constituent. (a) Right to Constitutional remedies
(c) We, the people in India… In this Con- (b) All fundamental rights
stituent assembly….decide to enact (c) No fundamental rights
and give to India this Constitution. (d) Right to freedom
(d) We, the people of India…in our Con- 730. Which of the following is/are listed among
stitution Assembly…enact and give to the Directive Principles in Part IV?
ourselves this Constitution. I. Equal pay for equal work
724. The Constitution of India was ordained II. Uniform Civil Code
by III. Small family norm
(a) the British Government before trans- IV. Education through mother tongue at
ferring power to India primary level
(b) the people of India through their rep- (a) I and II (b) I, II and III
resentatives in the Constituent Assem- (c) I, II, III and IV (d) II and III
bly 731. Which of the following were added to the
(c) the people of India directly Directive Principles by amendments to the
(d) None of these Constitutions?

CLAT.indb 226 31/03/2009 11:41:58


CONSTITUTIONAL LAW 227

I. To protect and improve the environ- II. A member of the State Legislature
ment and safeguard wildlife. III. A citizen of India
II. Right of workers to participate in IV. The Vice-President of India
management of industries (a) I, II and III (b) I, II and IV
III. Right to work (c) II and IV (d) II, III and IV
IV. To protect and maintain places of his- 738. The candidate for Vice-Presidential elec-
toric interest tion must possess the qualifications pre-
(a) I and II (b) I and III scribed for the Presidential candidate ex-
(c) I, III and IV (d) II and IV cept that he must be
732. The fundamental Duties were included in (a) Holding no office of profit under the
the Constitution with the Idea of Government
(a) Giving more power to the Executive (b) Qualified to be a member of the Rajya
(b) Giving more importance to the Funda- Sabha
mental Rights (c) A citizen of India
(c) Preventing abuse of Fundamental (d) Over 35 years of age
Rights 739. Who has the authority to remove the Vice-
(d) Curbing subversive and unconstitu- President from his office before the expiry
tional activities of his term?
733. Part V of the Constitution deals with (a) Supreme Court
I. Union Executive (b) Lok Sabha
II. Parliament (c) Rajya Sabha
III. Supreme Court and High Court (d) Parliament
IV. Comptroller and Auditor-General 740. The Constitution
(a) I, II and III (b) I, II and IV (a) allows re-election of a person to the
(c) I and II (d) I only President’s post
734. The executive authority of the Union is (b) restricts a person to remain President
vested by the Constitution in the for only two terms
(a) Union Legislature (c) has been amended to allow a person
(b) Cabinet only one term as President
(c) President (d) is silent on the President’s re-election
(d) Prime Minister to the office
735. The President takes an oath before as- 741. A resolution for impeaching the President
suming office in the presence of the Chief can be moved after at least fourteen days’
Justice of India. If the Chief Justice is not notice signed by
available, he takes the oath in the presence (a) At least 100 members of Lok Sabha
of and 50 members of Rajya Sabha
(a) Election Commissioner (b) Not less than two-thirds of the total
(b) The Attorney-General number of members of the House
(c) The senior-most judge of the Supreme (c) Not less than 50 members of the
Court House
(d) The Vice-President (d) Not less than one-fourth of the total
736. The President holds office for a term of number of members of the House
five years 742. If a resolution impeaching the President is
(a) From a date notified by the Election passed, the President is considered to have
Commission been removed
(b) From a date specified in the Constitu- (a) as soon as the Gazette of India notifies
tion it
(c) From the date of his entering the of- (b) once the new incumbent is elected
fice (c) once the Chief Justice of India takes
(d) From the date of his election out an order to the effect
737. Who among the following can contest a (d) from the date on which the resolution
Presidential election? is passed
I. Anyone who has attained the age of 743. If there is a vacancy in the offices of both
contest for Lok Sabha election President and Vice-President, who has

CLAT.indb 227 31/03/2009 11:41:59


228 CONSTITUTIONAL LAW

been empowered to function as Presi- (b) Vice-Chancellorship of Universities


dent? (c) No function
(a) Chief of Army Staff (d) None of the above is correct
(b) Chief Justice of India 750. When the Vice-President acts as President
(c) A person appointed by Parliament he gets the emoluments of
(d) No one (a) Vice-President
744. The executive power is vested in the Pres- (b) President
ident but it is actually used by him on the (c) President in addition to what he gets
advice of as Chairman of Rajya Sabha
(a) The Council of Ministers (d) Chairman of Rajya Sabha
(b) Parliament 751. A Judicial Review means
(c) The Prime Minister (a) Powers of the courts to declare null
(d) None of these and void any action by any organ of
745. Which of the following matters requires the Government if it is beyond the
the previous sanction of the President for powers granted by the Constitution
introducing legislation in it? (b) Review of the working of judiciary
I. A money bill (c) Power of the Court to try cases
II. A bill affecting taxation in which (d) Review of the judicial power of the
States are interested judges
III. States bills imposing restrictions upon 752. The conduct of the judges should be dis-
the freedom of traders cussed in the Parliament. Which Article
(a) I and II (b) I, III states this?
(c) I, II, III (d) II, III (a) Article 122 (b) Article 121
746. If the President returns a bill to the legisla- (c) Article 120 (d) Article 119
ture for reconsideration 753. The basis of classifying governments as
(a) A re-passage of the bill simple major- unitary or federal is
ity will compel him to give his assent (a) Relationship between the legislative
(b) The legislature must accept amend- and judiciary
ments proposed by the President (b) Relationship between the centre and
(c) A re-passage of the bill by two-thirds states
majority will compel him to give his (c) Relationship between the Prime Min-
assent ister and President
(d) A joint sitting will have to be called to (d) Relationship between the legislative
pass it again and executive
747. Which of the following Emergencies can 754. The Tenth Schedule of the Constitution of
be declared by the President on his own? India relates to
I. Emergency on account of armed re- (a) Administration of scheduled and trib-
bellion al areas
II. Financial Emergency (b) National languages of India
III. President’s Rule in a State (c) Anti-defection law
(a) I, II, III (d) Judicial review
(b) I 755. In which schedule of the Constitution are
(c) III land reform laws included?
(d) None of the above (a) Tenth (b) Ninth
748. The position of the Vice-President of India (c) Eighth (d) Seven
resembles, to a great extent, to the position 756. Which Article of the Indian Constitution
of the Vice-President of provides the Parliament, the power to
(a) Italy (b) Russia amend the Constitution?
(c) New Zealand (d) USA (a) 390 (b) 368
749. What function is specifically prescribed (c) 260 (d) 348
by the Constitution for the Vice-President 757. No Money Bill can be introduced in the
of India? legislative assembly of the state except on
(a) To assist the President in times of the recommendations of
emergency (a) The Governor of the State

CLAT.indb 228 31/03/2009 11:41:59


CONSTITUTIONAL LAW 229

(b) The President of India 764. How many languages have been recog-
(c) Special Committee of MPs nized in the Indian Constitution?
(d) The Parliament (a) 22 (b) 16
758. Which of the following statements about (c) 14 (d) 15
the Vice-President of India is/are correct? 765. Who conducts the State assembly elec-
I. He must be a member of the Rajya tions?
Sabha for his/her election as Vice- (a) Governor of the state concerned
President (b) Chief Justice of the Supreme Court
II. He is the ex-officio chairman of the (c) Chief Justice of the High court con-
Rajya Sabha cerned
III. He is elected by members of both (d) Chief Election Commission
houses of Parliament and state legis- 766. A minister, who is not a member of the
lative assemblies Legislative Assembly, is required to ob-
IV. If the President resigns or dies the tain the people’s verdict in his favour in a
Vice-President officiates as President
bye-election within
for not more than six months
(a) 15 days (b) one month
(a) II and IV
(c) three months (d) six months
(b) I and II
767. Which of the following qualifications is
(c) II, I and III
(d) III and IV not essential for a person to become Vice-
759. What is the power of the Rajya Sabha with President of India?
regard to Money Bill? (a) He must be an Indian citizen
(a) It can withhold it (b) He should be a graduate
(b) It has no power (c) he must not be less than 35 years of
(c) It can amend it age
(d) It can reject it (d) He must be qualified to become a
760. Which of the following states/union terri- member of the Rajya Sabha
tories has no legislative council? 768. Rajya Sabha is less powerful than Lok
(a) Bihar (b) Karnataka Sabha in terms of its financial powers be-
(c) Uttar Pradesh (d) Tripura cause
761. What is Zero hour? (a) no Money Bill can be introduced in
(a) Interval between morning and eve- Rajya Sabha
ning session (b) it does not participate in the passing of
(b) When Money Bill is introduced in Money Bill
Lok Sabha (c) both (a) and (b)
(c) When proposals of Opposition are (d) none of the above
considered 769. A dispute between the two houses of the
(d) When matters of utmost importance Parliament can be resolved by
are raised (a) joint sitting of both houses summoned
762. Which among the following is not true re- by the President
garding Rajya Sabha? (b) ruling of the Lok Sabha Speaker
(a) The nominated members of Rajya (c) ruling of the Rajya Sabha chairperson
Sabha also elect the President of In- (d) none of these
dia 770. Speaker of the lower house of a state can
(b) Money Bill cannot be introduced in step down by submitting his resignation to
Rajya Sabha
the
(c) Vice-President is the ex-officio Chair-
(a) Deputy Speaker of the House
man of the Rajya Sabha
(b) Chief Minister
(d) It is the upper House of Parliament
(c) Governor
763. The amendment of the Constitution can be
(d) President
initiated in
(a) Legislative Assemblies of states 771. Parliament can legislate matters in the
(b) Either House of the Parliament State list
(c) Lok Sabha only (a) by wish of the President
(d) Rajya Sabha only (b) under any circumstances

CLAT.indb 229 31/03/2009 11:41:59


230 CONSTITUTIONAL LAW

(c) by asking the legislature of the con- (c) does not prevent the Union Govern-
cerned state ment from making any international
(d) if Rajya Sabha passes a resolution agreement affecting the State without
772. Which of the following is not a basic fea- taking the consent of the State Legis-
ture of the Indian constitution? lature.
(a) Federal Government (d) does not prevent the President from
(b) Parliamentary exercising his right to suspend the
(c) Independence of the judiciary Constitutional machinery of the State
(d) Presidential Government on the ground that the State Govern-
773. Who among the following did not serve ment has failed to carry out directions
as the Vice-President before becoming the of the Union Government.
President of India? 779. The members of the UPSC can be removed
(a) Dr S. Radhakrishnan from their office during their tenure by
(b) Dr Zakir Hussain (a) the Parliament
(c) Neelam Sanjeeva Reddy (b) the President
(d) R. Venkataraman (c) the President on the report of the Par-
774. Which of the following is part of the elec- liament
toral college for election of President of (d) the President on the report of the Su-
India but does not form part of the forum preme Court of India
for his impeachment? 780. The Parliament can made the law on a
(a) State Legislative assemblies subject in the State List when
(b) State Legislative Councils I. A Proclamation of Emergency is in
(c) Lok Sabha force
(d) Rajya Sabha II. Two or more States make a request to
775. The power of the Supreme Court of India the Parliament to make a law on a sub-
to decide the dispute between the centre ject
and states falls under its III. Rajya Sabha passes a resolution that
(a) Constitutional jurisdiction such subjects have acquired national
(b) Original jurisdiction importance
(c) Appellate jurisdiction (a) I, II (b) II and III
(c) I and III (d) I, II, III
(d) Advisory jurisdiction
781. The salary and other allowances of the
776. Violation of a right in the chapter on fun-
Judges of the High Courts are charged to
damental rights can be challenged in
(a) the Consolidates Fund of the State
(a) Supreme Court
(b) the Consolidated Fund of India
(b) High Court
(c) the Consolidated Fund of India and
(c) Both Supreme Court and High Court
the State on fifty-fifty basis
(d) Cannot be challenged
(d) the Contingency Fund of India
777. The qualifications for the Chairman and
782. The Judges of the High Court can be
other members of the Finance Commis-
removed from their office during their
sion
tenure
(a) have been laid down in the Constitu-
(a) by the Chief Justice on the advice of
tion the President
(b) are determined by the President (b) by the Chief Justice on the recommen-
(c) are determined by the Parliament dation of the Parliament
(d) are determined by the President in (c) by the Governor, if the State Legisla-
consultation ture passes a resolution to this effect
778. The special status of the State of Jammu by two-thirds majority
and Kashmir (d) by the President on the basis of a reso-
(a) prevents the Union government from lution passed by the Parliament by
making any alteration in the name and two-thirds majority.
boundaries of the state. 783. The High Court enjoys the power
(b) does not prevent the Union Govern- I. To issue writs for the enforcement
ment from making changes in the of rights conferred on the citizens of
name and boundaries of the State. India.

CLAT.indb 230 31/03/2009 11:41:59


CONSTITUTIONAL LAW 231

II. To exercise superintendence over the (a) protection of life and personal liberty
working of Courts and Tribunals un- (b) freedom as to payment of taxes for the
der its jurisdiction. promotion of any particular religion
III. To make general rules and prescribe (c) equality before law
forms regulating the practices and (d) equality of opportunity in matters of
proceeding of courts under its juris- public employment
diction. 789. Relate the following:
(a) I, II (b) II, III List I
(c) I, III (d) I, II, III 1. 42nd Amendment
784. A Money Bill can originate in the State 2. 61st Amendment
Legislature only with the prior consent of 3. 71st Amendment
the 4. 54th Amendment
I. Chief Minister List II
II. President A. Konkani, Manipuri, and Nepali in-
III. Governor cluded in the Eighth Schedule
IV. None of these B. Voting age reduced from 21 to 18
(a) I (b) I and III years
(c) II (d) III C. Salaries of the Supreme Court and
785. In which cases can the Governor exercise High Court Judges raised
his discretionary powers? D. Constitutional watershed
I. In appointment of Chief Minister if (a) 1 – D, 2 – C, 3 – B, 4 – A
not a single party has clear-cut major- (b) 1 – A, 2 – B, 3 – C, 4 – D
ity in the House or there is no recog- (c) 1 – D, 2 – A, 3 – B, 4 – C
nized leader of the majority party (d) 1 – A, 2 – B, 3 – C, 4 – D
II. In reservation of a Bill passed by State 790. Generally, the first session of the Parlia-
Legislature for the assent of the Presi- ment starts with an address of the Presi-
dent dent in which
III. Dismissing a ministry if he is con- (a) he makes suggestions to the Govern-
vinced it has lost majority support in ment regarding the policy it should
the Assembly follow in the interest of the country
(a) I, II and III (b) I andIII (b) he outlines the policy and programme
(c) II and III (d) I and II of the government during the ensuing
786. The Governor can reserve year
(a) only Financial Bills for the approval (c) he makes a survey of the achieve-
of the President ments of the Government during the
(b) certain types of Bills passed by State previous year
legislature for the approval of the (d) he does none of the above things
President 791. The provisions of the Constitution relating
(c) no Bill passed by the State Legislature to the establishment and abolition of the
for the approval of the President Legislative Councils can be amended by
(d) all Bills passed by State Legislature (a) simple majority of the Parliament
for the assent of the President (b) two-thirds majority of the two Houses
787. The writ of prohibition issued by the Su- of Parliament
preme Court or a High Court is issued (c) two-thirds majority of the two Hous-
against es of Parliament and majority of the
1. judicial or quasi-judicial authorities States
2. administrative and judicial authorities (d) none of the above
3. administrative authorities only 792. The salary of the Governor is charged on
4. administrative authorities and govern- (a) the Consolidated fund of the State
ment (b) the Consolidated Fund of India
(a) 4 only (b) 2 only (c) the Consolidated fund of India and
(c) 1 only (d) 1 and 4 only Consolidated Fund of the State on
788. Only the citizens of India enjoy the right fifty-fifty basis.
to (d) the Contingency Fund of the State

CLAT.indb 231 31/03/2009 11:41:59


232 CONSTITUTIONAL LAW

793. Which of the following States possess a 3. Governors of the State


bi-cameral Legislature? 4. Speaker of the Lok Sabha
1. Punjab (a) 1, 2, 3, 4 (b) 1, 2, 3
2. Bihar (c) 1, 3, 4 (d) 1, 3
3. Karnataka 799. The words ‘Satyameva Jayate’, inscribed
4. Maharashtra below the National Emblem, are taken
(a) 2 and 4 (b) 2 and 3 from
(c) 1, 2 and 3 (d) 2, 3 and 4 (a) Mundaka Upanishad
794. Which of the following are the merits of (b) Jataka
the Rajya Sabha? (c) Puranas
1. It relieves the Lok Sabha of a consid- (d) Mahabharata
erable burden of work by taking up 800. Which of the following steps can be tak-
for consideration of certain bills en by the President during the Financial
2. It provides representation to the elder Emergency?
and mature persons 1. He can order the reduction of the sala-
3. It acts as a revising chamber and elim- ries of the Supreme Court and High
inates the defects in laws Court Judges.
4. It makes federalism a reality by pro- 2. He can suspend the Fundamental
viding equal representation to all the Rights of the Indian citizens.
States 3. He can order reduction of salaries and
(a) 2 and 3 (b) 1 and 2 allowances of all Civil Servants.
(c) 1, 3 and 4 (d) 1, 2 and 3 4. He can direct the Union and State
795. In the Indian polity, ultimate sovereignty Governments to observe such canons
rests with of financial propriety as he deems de-
(a) the Parliament (b) the Constitution sirable.
(c) the President (d) the People (a) 1, 2, 3, 4 (b) 1, 2, 3
796. Which one of the following features of (c) 3, 4 (d) 2, 3, 4
the Indian Federalism have been correctly 801. When the Chairman of Rajya Sabha acts
listed? as President, the duties of the Chairman
1. it divides the powers between the are performed by
Centre and the States (a) Himself
2. it provides for an independent judi-
(b) The Deputy Chairman
ciary
(c) A newly elected Chairman
3. it provides equal representation to the
(d) A member of Rajya Sabha deputed by
Stages in the Rajya Sabha
the Chairman
4. it has a written constitution
802. The President may send the advice re-
(a) 1, 2, 3 (b) 1, 2, 4
ceived from the Council of Ministers on
(c) 2, 3, 4 (d) 1, 2, 3, 4
a matter back to them for reconsideration.
797. The Directive Principles are in the nature
This power can be exercised by him… on
of
the matter.
(a) instructions to the Government to do
(a) once
certain things
(b) twice
(b) request to the Government to pay at-
tention to certain subjects (c) thrice
(c) injunctions to the Government to re- (d) any number of times
frain from doing certain things 803. The President may appoint the
(d) judicial injunctions to the government I. Prime Minister
to enact certain laws II. Governor
798. Who among the following are appointed III. High Court Judges
by the President on the advice of the Prime IV. Rajya Sabha Chairman
Minister? (a) I, II (b) I, III
1. Chief Justice and other Judges of the (c) I, II, III (d) I only
Supreme Court of India 804. The President has the power to remove the
2. Chairman and members of the Union following
Public Service Commission I. Attorney-General of India

CLAT.indb 232 31/03/2009 11:41:59


CONSTITUTIONAL LAW 233

II. Governor of a State III. The person who is in a position to win


III. Council of Minister the confidence of the majority in Lok
IV. Chairman of a State Public Service Sabha
Commission IV. The leader of the party having a ma-
(a) I, II, IV (b) I, II jority of seats in either Lok Sabha or
(c) I, III, IV (d) II, III, IV Rajya Sabha
805. The Fundamental Duties of a citizen in- (a) I, III or IV (b) II or IV
clude (c) I or III (d) I only
(a) Respect for the President 812. The Prime Minister is
(b) Respect for the Government (a) Elected by the Parliament
(c) Respect for the Constitution, the Na- (b) Appointed by the President
tional Flag and the National Anthem (c) Elected by the Lok Sabha
(d) All of the above (d) Nominated by the party enjoying ma-
806. Fundamental Duties do not enjoy any jority in Lok Sabha
(a) Legal sanction 813. In the event of the resignation or death of
(b) Moral sanction the Prime Minister
(c) Political sanction (a) The President decides what to do
(d) Social sanction (b) The Cabinet may choose another lead-
807. A socialistic ideology is reflected in the er
Directive Principle, which calls for (c) Fresh general elections must take
(a) Promotion of cottage industries place
(b) Free and compulsory education for (d) The Ministry is dissolved
children upto 14 years of age 814. The Prime Minister is said to hold office
(c) Securing equitable distribution of ma- during the pleasure of the President but in
terial resources of the country to pre- reality he stays in office as long as he en-
vent concentration of wealth joys the confidence of
(d) All the above (a) The Lok Sabha
808. Which of the following is the requisite (b) Parliament
qualification for a candidate to be eligible (c) The party to which he belongs
for election as the President of India? (d) The electorate
I. Not less than 35 years of age 815. The salary and allowances of the Prime
II. Must not hold office of profit Minister of India are decided by the
III. Must be eligible to become a member (a) President (b) Parliament
of Lok Sabha (c) Cabinet (d) Constitution
IV. Must be a member of either House of 816. In a Parliamentary democracy, the
Parliament (a) Executive and Legislature are strictly
(a) I, II (b) II, III separate
(c) I, II, III (d) I, II, IV (b) Judiciary controls both Legislature
809. The office of the Prime Minister of India and the Executive
(a) has been created by Parliament (c) Legislature controls the Executive
(b) is based on conventions (d) Executive controls the Legislature
(c) has been created by the Constitution 817. The portfolios are allocated to the minis-
(d) is less powerful than that of the Presi- ters by
dent (a) the Prime Minister
810. The Council of Ministers has no option (b) the President
but to resign if it loses the confidence of (c) the collective decision of the Council
(a) Lok Sabha of Ministers
(b) Rajya Sabha (d) individual choice
(c) Either House of Parliament 818. In practice, the policy of the Government
(d) Parliament is shaped by
811. The President selects as Prime Minister (a) Article 75 (b) Article 78
I. The leader of the party in majority in (c) Convention (d) Statute
Lok Sabha 819. The Attorney-General of India holds of-
II. Any one he wishes to fice during the pleasure of the

CLAT.indb 233 31/03/2009 11:41:59


234 CONSTITUTIONAL LAW

(a) Parliament (a) I, II (b) I, II, III


(b) President (c) I, III (d) II only
(c) Prime Minister 826. Which of the following qualifications for
(d) Chief Justice the office of the Vice-President are cor-
820. Which of the following is/are correct with rect?
regard to the office of Attorney-General of I. He must be a citizen of India
India? II. He must be a member of Rajya Sabha
I. He is a member of the Cabinet III. He must be over 35 years of age
II. He can speak in either House of Par- (a) I and II (b) I, II and III
liament and vote on any issue (c) I and III (d) II and III
III. He must be qualified to become a 827. How long can a person continue to be the
judge of the Supreme Court Minister of the Union Government/State
IV. His remuneration is fixed by Parlia- government without being a member of
ment either House of Parliament/State Legisla-
(a) I, II and IV (b) III only ture?
(c) III and IV (d) II, III and IV (a) Three months (b) Six months
821. The President of the Union of India has (c) One year (d) No time limit
the same constitutional authority as the 828. Ministers in the Union Cabinet are ap-
(a) President of USSR pointed by
(b) President of USA (a) The President
(c) President of France (b) Prime Minister
(d) British Monarch (c) The Prime Minister on the advice of
822. The President can be impeached for the President of India
(a) not taking the Prime Minister’s ad- (d) The President of India on the advice
vice of the Prime Minister
(b) Disregarding Parliament 829. To get appointed, a Minister must
(c) Violating the Constitution (a) not be a nominated member of Parlia-
(d) All the above ment
823. Under our Constitution, the President is (b) be an eminent personality
(a) obliged to accept the advice of the (c) be a member of Parliament or secure
council of Ministers a seat within six months of appoint-
(b) obliged to seek the advice of the ment
Council of Ministers but is not bound (d) be a member of Lok Sabha or be elect-
to follow it ed within six months of appointment
(c) obliged to seek the advice of the 830. The formation of the Council of Ministers
Council of Ministers under certain starts with the appointment of
circumstances only (a) The Prime Minister
(d) not obliged to seek or accept the ad- (b) The Speaker
vice of the Council of Ministers (c) The President
824. The Vice-President of India is (d) None of these
(a) the Speaker of the Lok Sabha 831. The meetings of the Council of Ministers
(b) the President of the Red Cross are presided over by
(c) the ex-officio Chairman of the Rajya (a) The Prime Minister
Sabha (b) The President
(d) the Chairman (ex-officio) of the Lok (c) The Vice-President
Sabha (d) The members of the Council of Minis-
825. When the Vice-President is acting as the ters by rotation
President, he 832. The first Law Officer of the Government
I. Will have all the powers and functions of India is the
of both the President and Vice-Presi- (a) Law Minister
dent (b) Chief Justice of Supreme Court
II. Gets all the allowances and privileges (c) Attorney-General of India
of the President (d) None of the above
III. Should continue to work as the Chair- 833. Cultural and Educational Rights are men-
man of the Rajya Sabha tioned in

CLAT.indb 234 31/03/2009 11:41:59


CONSTITUTIONAL LAW 235

(a) Articles 30 and 31 The correct statement(s) is/are


(b) Articles 28 and 29 (a) 1, 2, 4 (b) 2, 3, 4
(c) Articles 28, 29 and 30 (c) 1, 2, 3, 4 (d) 2, 4
(d) Articles 29 and 30 840. The Annual Financial Statement is caused
834. The Fundamental Duties to be laid before both Houses of Parlia-
I. were introduced by the 44th Amend- ment
ment (a) Speaker
II. are incorporated in Part III-A (b) Vice-President
III. are not justiciable (c) President
(a) I and III (b) III only (d) Finance Minister
(c) I, II and III (d) None of these 841. Which House is better placed with regard
835. An ‘office of profit’ which disqualifies a to control over the executive?
person from being a member of the Union (a) Rajya Sabha
or State Legislature includes office held (b) Lok Sabha
under (c) Both are equally placed
I. The Government of India (d) It depends from which House the
II. A State Government Prime Minister comes
III. A Local Authority 842. All moneys received by or on behalf of the
(a) III only (b) I, II only Government of India are credited to
(c) I, II, III (d) None of these 1. the Consolidated Fund of India
836. An ordinance can be issued by the Presi- 2. the Public Account of India
dent 3. the Contingency Fund of India
(a) on any subject (a) 1 only (b) 1, 2
(b) at any time (c) 1, 3 (d) 1, 2, 3
(c) subject to the same constitutional lim- 843. The Parliament in India consists of
itations as legislation by Parliament I. President
(d) in his individual judgement as well as II. Rajya Sabha
on the advice of the council of Minis- III. Lok Sabha
ters IV. Vice-President
837. The Deputy Chairman of the Rajya Sabha (a) I, II, III and IV (b) II, III and IV
is (c) I, II and III (d) II and III
(a) nominated by the President 844. The members of the Rajya Sabha are
(b) nominated by the Chairman (a) mostly nominated
(c) elected by the Rajya Sabha from (b) elected indirectly
amongst its members (c) elected by the members of the Leg-
(d) elected by elected members of the Ra- islative Assemblies and Legislative
jya Sabha Councils of States
838. When a resolution for the removal is un- (d) elected directly as well as indirectly
der consideration, the Speaker 845. As a result of prorogation which of the fol-
I. does not take part in the proceedings lowing is not affected?
of the House (a) Notices (b) Bills
II. has no right to vote (c) Resolutions (d) Motions
III. has the right to speak in the House 846. Which Union Territories are represented
even through he shall not preside in Rajya Sabha at present?
(a) III (b) II I. Delhi
(c) I (d) I and II II. Pondicherry
839. Money Bill III. Lakshadweep
1. cannot be introduced in the Council of IV. Chandigarh
States (a) I and II (b) II and III
2. needs to be certified as such by the (c) I, II, III and IV (d) I, II and IV
President 847. An Appropriation Bill
3. can be amended by the Council of I. is necessary to draw money from the
States Consolidated Fund of India
4. President has to assent it without II. cannot be amended to vary the amount
delay of any charged expenditure

CLAT.indb 235 31/03/2009 11:41:59


236 CONSTITUTIONAL LAW

III. includes only the expenditure charged 3. when the ruling party loses its major-
on the consolidated Fund of India ity in the Lok Sabha and recommends
IV. is required to withdraw money from for dissolution of the House
the contingency fund of India 4. when he decides to refer the advice of
(a) I and II (b) I and III the Union Cabinet back for reconsid-
(c) I, II, III and IV (d) I, II and III eration
848. The Union Legislature cannot legislate on Select the correct answer from the codes
a subject in the State List unless given below.
(a) there is a national emergency (a) 1, 2, 3, 4 (b) 2, 3, 4
(b) the President calls upon it to do so (c) 2, 4 (d) 1 only
(c) the Rajya Sabha passes a resolution 854. Whose function is to see that no money is
that it is necessary in national interest spent out of the Consolidated Fund of a
to do so State without the authority of the legisla-
(d) the Speaker certifies that it is neces- ture?
sary (a) Finance Commission
849. At a joint sitting of Parliament a bill has to (b) Public Accounts Committee
be passed (c) Comptroller and Auditor-General of
(a) By a simple majority of the total num- India
ber of members of both Houses pres- (d) None of the above
ent and voting 855. The Comptroller and Auditor-General can
(b) By a simple majority of the total num- be removed from office
ber of members of both Houses (a) by the President on a report by the Su-
(c) By a two-thirds majority of the total preme Court
number of members of both Houses
(b) in the same manner as that prescribed
present and voting
for the removal of a Supreme Court
(d) By a two-thirds majority of the total
Judge
number of members of both Houses
(c) in the same way as the President can
850. The two Houses of Parliament enjoy co-
be removed
equal power in all sphere except
(d) by none of the above ways
I. Financial matters
856. The Governor holds office
II. Responsibilities of the Council of
(a) for five years
Minister
(b) for a period fixed by Parliament
III. Amendment procedure
IV. Election of President (c) during the pleasure of the President
(a) I, II (b) III and IV (d) till he enjoys the confidence of the
(c) I, II and III (d) II, III and IV Parliament
851. The term of Rajya Sabha members was 857. The Legislative council of a State
fixed by the I. Is not subject to dissolution
(a) Parliament (b) Cabinet II. Can be abolished by the State Legisla-
(c) President (d) Constitution tive Assembly
852. A national emergency remains in opera- III. Can be abolished by the President on
tion, with the approval of the Parliament, the governor’s recommendation
for (a) I (b) I and II
(a) an indefinite period (c) I, II, III (d) I and III
(b) a maximum period of six months 858. The Governor does appoint
(c) a maximum period of one year 1. Judges of the High Court
(d) a maximum period of three years 2. Chief Minister
853. The President can make use of his discre- 3. Chairman of the State Public Service
tionary powers Commission
1. when there is a dispute between the 4. Advocate-General of the State
Prime Minister and the Cabinet Select the correct answer from the codes
2. when there is no majority party in given below.
the Lok Sabha and he has to choose a (a) 1, 2, 3, 4 (b) 2, 3, 4 only
Prime Minister (c) 1, 2, 4 only (d) 2, 4 only

CLAT.indb 236 31/03/2009 11:42:00


CONSTITUTIONAL LAW 237

859. Following members can participate in the which he enters upon his office. Which of
Vice-Presidential election but not in the the following can be deduced from this?
Presidential election. 1. No Governor can be removed from
(a) Non-member Minister office till the completion of his term
(b) State Legislative Members 2. No Governor can continue in office
(c) Persons absent for a period of six beyond a period of five years
months without prior permission Select the correct answer from the codes
(d) Nominated members of the upper given below.
house (a) 1 only (b) 2 only
860. The word ‘Economic Justice’ is found in (c) 1 and 2 (d) neither
(a) Preamble, Fundamental Rights, Di- 865. India is a Republic. It means
rective Principles (a) India is a Union of States
(b) Preamble, Directive Principles (b) The Head of the State is not heredi-
(c) Preamble, Fundamental Rights tary
(d) Preamble (c) India has Parliamentary system of
861. The Chief Minister of a State in India government
is not eligible to vote in the Presidential (d) People have the final authority in all
election if matters.
(a) he is the caretaker Chief Minister 866. Cabinet includes
(b) he himself is a candidate 1. Cabinet Ministers
(c) he is yet to prove his majority on the 2. Ministers of State
floor of the Lower House of the State 3. Deputy Ministers
legislature Choose the correct answer from the codes
(d) he is a member of the Upper House of given below.
the State legislature (a) 1, 2 and 3 (b) 1 and 2
862. If in an election to a State Legislative (c) 1 and 3 (d) 1 only
Assembly the candidate who is declared 867. Out of the following, which are the taxes
elected loses his deposit, it means that imposed by the Central Government?
(a) the polling was very poor 1. Gift Tax
(b) the elected candidate’s victory over 2. Entertainment Tax
his nearest rival was very marginal 3. Individual Income Tax
(c) the election was for a multi-member 4. Corporate Tax
constituency (a) 1, 2 and 3 (b) 1, 3 and 4
(d) a very large number of candidates (c) 1, 2 and 4 (d) 3 and 4
contested the election 868. The Constitution of India provides that a
863. Which of the following are matters on bill passed by the Union Parliament can-
which a constitutional amendment is pos- not become a law until the President ac-
sible only with the ratification of the leg- cords his approval to it. Normally, the
islatures of not less than one-half of the President accords his approval but he can
States? withhold his assent and can ask the House
1. Election of the President to reconsider it. This power is applicable
2. Representation of States in Parlia- to the
ment 1. Money Bills
3. Any of the subjects in the 7th Sched- 2. Ordinary Bills
ule 3. Finance Bills
4. Abolition of the Legislative Council (a) 1, 2 and 3 (b) 2 and 3
of a State (c) 1 and 2 (d) 2 only
Choose the correct answer from the codes 869. Identify the features which Indian Consti-
given below tution had borrowed from the US Consti-
(a) 1 and 3 (b) 2 and 3 tution.
(c) 1, 2, 3 and 4 (d) 1, 2 and 3 I. Equal protection of law
864. Article 156 of the Constitution of India II. Equality before Law
provides that a Governor shall hold office III. Fundamental Rights
for a term of five years from the date on IV. Fundamental Duties

CLAT.indb 237 31/03/2009 11:42:00


238 CONSTITUTIONAL LAW

(a) I, II, III (b) I and III (d) 12 hours before the commencement of
(c) II, III, IV (d) II and IV polling
870. Which of the following are constituted by 875. Expenses incurred out of the Contingency
the President? Fund of India are
1. The Finance Commission (a) not recouped till the whole fund is ex-
2. The Planning commission hausted
3. The Commission on Official Lan- (b) recouped by collecting contributions
guages from various states
4. The Union Public Service Commis- (c) recouped through supplementary, ad-
sion dition or excess grants by Parliament
Codes: (d) subsequently recouped by transferring
(a) 2, 3 4 (b) 1, 3 savings from other heads of the bud-
(c) 1, 2 (d) 3, 4 get
871. Which of the following duties/functions 876. Which of the following functions of the
are not assigned to the Attorney General Minorities Commission have been cor-
of India? rectly listed?
1. To control state expenditure and ac- 1. It can make recommendations for ef-
counts fective implementation and enforce-
2. To render legal advice to the Central ment of safeguards and laws for mi-
Government norities.
3. To arbitrate in dispute between the 2. It undertakes review of the implemen-
Centre and the States tation of policies formulated by the
(a) 1 only (b) 2 only Union and State Governments with
(c) 1, 2, 3 (d) 1, 3 regard to minorities.
872. Which one of the following is/are correct- 3. It can look into specific complaints
ly matched? regarding deprivation of rights and
Schedule Deals with safeguards of minorities.
I. 10th Schedule: Provision as to dis- (a) 1, 2 and 3 (b) 1 and 2
qualification on grounds of defection (c) 1 and 3 (d) 1 only
II. 8th Schedule: Union, States and Con- 877. The Constitution seeks to protect and pro-
current lists mote the interests of the Schedules Castes/
III. 7th Schedule: Language Scheduled Tribes by reserving seats for
IV. 6th Schedule: Allocation of seats in them in
the Rajya Sabha 1. Public Services
(a) I only (b) I and IV 2. Defence services
(c) I, II and III (d) I, II, III and IV 3. Lok Sabha
873. Which of the following categories of 4. Rajya Sabha
persons are entitled to exercise his vote (a) 1, 2 and 3 (b) 1 and 3
through postal ballot? (c) 1, 2, 3 and 4 (d) 1, 3 and 4
1. members of foreign services posted 878. The Comptroller and Auditor-General is
abroad intimately connected with the following
2. members of the armed forces committee of Parliament:
3. civil servants on election duty 1. Public Accounts Committee
4. Indian nationals settled abroad 2. Estimates Committee
(a) 1 and 3 (b) 2 and 3 3. Committee on Public Undertakings
(c) 1, 2 and 3 (d) 1, 2, 3 and 4 Codes:
874. In terms of the election laws in India, (a) 1, 2 and 3 (b) 1 and 2
electioneering ceases in a constituency at (c) 1 and 3 (d) 1 only
least 879. The main objective of the Zonal Council
(a) 48 hours before the closing hour of is/are to ensure
polling 1. greater cooperation amongst States in
(b) 36 hours before the commencement of the field of planning and other matters
the polling of national importance
(c) 24 hours before the commencement of 2. that the laws passed by various States
polling do not conflict with each other

CLAT.indb 238 31/03/2009 11:42:00


CONSTITUTIONAL LAW 239

3. better utilization of the limited re- (c) an act authorized by law


sources of the States (d) an illegal act
(a) 1 only (b) 1 and 2 886. To betray a nation is an offence and pun-
(c) 1 and 3 (d) 1, 2 and 3 ishable with death, that is
880. The Constitution has vested the ‘residu- (a) Sedition (b) Treachery
ary power’ with the Centre. But the final (c) Treason (d) Anti-nationality
authority to decide whether a matter falls 887. The term res subjudice means
under residuary power or not rests with (a) a case has been decided by a court
(a) the President (b) a case is pending before a court
(b) the Rajya Sabha (c) a case has been dismissed by a court
(c) the Supreme Court (d) None of the above
(d) the Parliament 888. The supreme Court in….case held that Ar-
881. The Parliament can legislate on any sub- ticle 21 includes right to die
ject in the state list (a) P. Rathinam/NagbhusanPatnaik vs
1. if the Rajya Sabha declares by a two- Union of India
third majority resolution that it is ex- (b) Dr M. Ismail Faruqui vs Union of In-
pedient to do so in national interest dia
2. for the implementation of internation- (c) K. Karunakaran vs State of UP
al treaties or agreements (d) None of the above
3. if two or more states make a request of 889. Which section of Indian Penal Code was
Parliament to do so struck down by Supreme Court, while
4. when Article 356 is imposed in the making attempt to commit suicide non-
State punishable (In P. Rathinam etc., vs Union
(a) 1, 2, 3 and 4 (b) 1, 2 and 4 of India)?
(c) 1, 3 and 4 (d) 1, 2 and 3 (a) Section 309 (b) Section 310
882. Among which of the following case the (c) Section 498 (d) Section 51
Supreme Court held that, there is no mor- 890. In which of the following cases, the pris-
al, fundamental or legal rights or equitable oners are entitled to rights under Articles
jurisdiction to go on strike by the govern- 14, 19 and 21 of the Constitution of India
ment employees? and these articles operate within the pris-
(a) T. K. Rangarajan vs Government of on?
Tamil Nadu
(a) Sunil Batra’s case in 1978
(b) Dwarka Prasad Agarwal vs B. D.
(b) Nandini Satpathy’s case in 1977
Agarwal
(c) Maneka Gandhi’s case in 1978
(c) Union of India vs Rajiv Kumar,
(d) Indira Nehru Gandhi’s case in 1976
A.I.T.U.C. vs Union of India
891. Public holidays are declared under
(d) A. K. Puriwar vs T. N. Muthy
(a) Negotiable Instruments Act, 1881
883. In which case, Supreme Court held the
(b) Contract Act, 1872
Parliament has no power to amend Funda-
(c) Public Employees Act, 1967
mental Rights?
(d) None of the above
(a) Golaknath case
892. Guidelines for protection of witnesses
(b) Sajjan Singh case
were issued in
(c) Shanri Prasad case
(d) None of the above (a) Naina Sahni Case
884. In…..Supreme Court held that Preamble is (b) Raja Bhaiya case
a basic feature of Constitution and cannot (c) Shivani Bhatnagar case
be amended. (d) Neelam Katara case
(a) Keshvanand Bharti case 893. Who is responsible for introduction of
(b) Golaknath case Public Interest Litigation in India?
(c) Minerva Mills case (a) Justice P. N. Bhagwati
(d) None of the above (b) Justice M. N. Venkatachaliah
885. ‘Ultra Vires’ is a term used for (c) Justice A. M. ahmadi
(a) a document corrupted by virus (d) Justice V. R. Krishna Iyer
(b) an act beyond the authority of a per- 894. In this famous case, the Supreme Court
son said “the President means, for all practical

CLAT.indb 239 31/03/2009 11:42:00


240 CONSTITUTIONAL LAW

purposes, the Prime Minister or council of (a) Advowson (b) Affidavit


Ministers and his opinion, satisfaction or (c) Affirmation (d) Affray
decision is constitutionally secured when 901. A crime of unlawfully damaging or de-
Minister arrives at such opinion, satisfac- stroying property by fire is called
tion or decision”. (a) Buggery (b) Affray
(a) Ram Jawaya Kapur vs State of Pun- (c) Arson (d) Rioting
jab 902. In civil action, the standard of proof is
(b) Maneka Gandhi vs Union of India (a) beyond reasonable doubt
(c) Dr M. Ismail Faruqui vs Union of (b) on balance of probabilities
India (c) convincing to the judge
(d) None of the above (d) logical and reasonable
895. After 86th Amendment to the Constitution, 903. Prosecution : Accused :: ? : Defendant
Article 45 of the Constitution includes (a) Prosecution (b) Plaintiff
(a) provision for the right to education of (c) Suer (d) Applicant
children 904. An insolvent person is
(b) provision for early childhood care and (a) not able to walk
protection against any offence (b) infirm
(c) provision for early childhood care and (c) financially unsound
education to children below the age of (d) bankrupt
six years 905. What is the legal meaning of the word
(d) provision for children for case and ‘Battery’?
education upto age of 18 years (a) Cells, as used in torch, tape recorder
896. The Shah Bano case is related to etc.
(a) Muslim wife’s right to maintenance (b) Battering a person to death
after divorce (c) Actual or intended striking of another
(b) Muslim wife’s right to divorce person
(c) Muslim wife’s right to separation (d) Assault resulting in, at least, 6 months’
(d) Muslim wife’s right to husband’s hospitalization.
property 906. Capital offences result in
897. When a person aids the commission of an (a) Sentence to death
offence, he is called (b) Sentence of life imprisonment
(a) Abettor (b) Accessory
(c) Imprisonment over 10 years
(c) Accomplice (d) Abeyer
(d) Amputation
898. If a person goes away secretly, or evades
907. What is a Caveat?
the jurisdiction of the court, he is
(a) a warning (b) an injunction
(a) absconding (b) abrogating
(c) certiorari (d) writ
(c) absenting (d) abomining
908. What is a ‘cognizance”?
899. Match the following:
(a) Arousing judicial notice or knowl-
I. Adulteration
edge
II. Adultery
(b) It is a crime
III. Alibi
(c) It means custody without warrant
IV. Alteration
(d) It means custody with warrant
A. Adding of substance to food which
renders it dangerous to health 909. …..is an established defence when it is
B. Having sexual intercourse with mar- proved that the plaintiff failed to take rea-
ried woman sonable care of himself and thus contrib-
C. A dispute utes to his injuries
D. An excuse (a) Contributory negligence
(a) I – D, II – C, III – B, IV – A (b) Self injury
(b) I – A, II – B, III – D, IV – C (c) Volenti non fit injuria
(c) I – A, II – C, III – D, IV – A (d) None of the above
(d) I – B, II – C, III – A, IV – D 910. A doctor has a ….. relationship with his
900. A written statement in the name of a per- patient
son by whom it is voluntarily signed and (a) Fiduciary (b) Financial
sworn to is called (c) Personal (d) Impersonal

CLAT.indb 240 31/03/2009 11:42:00


CONSTITUTIONAL LAW 241

911. ‘Homicide’ is killing of a human being by 920. The meaning of the term quasi is
another human being. It may be (a) As if it were (b) Null
(a) Lawful (c) Void (d) Quashed
(b) Unlawful 921. Who is a recidivist?
(c) Both (a) and (b) (a) Saint (b) Habitual criminal
(d) Never (a) (c) Rash person (d) Reserved person
912. ‘Ignorantia juris nemineam exusal’ stands 922. Match the following:
for I. Void
(a) Ignorance of law is no excuse II. Voidable
(b) Ignorance of fact is no excuse III. Vis major
(c) Laws are universally known IV. Volenti non fit injuria
(d) None of the above A. An act illegal from beginning
913. Match the following: B. An act until recinded, legal
I. Inadvertence C. Consent is a defence
II. Incest
D. Irresistable force
III. Indemnity
(a) I – A, II – B, III – C, IV – D
IV. Indictment
(b) I – C, II – D, III – B, IV – A
A. Written accusation of crime
(c) I – B, II – D, III – A, IV – C
B. Exemption from penalty
C. Sexual intercourse between members (d) I – A, II – B, III – D, IV – C
of a family e.g., mother and son 923. Public Prosecutor is
D. Carelessness (a) Counsel of the State in the case where
(a) I – A, II – B, III – C, IV – D government is a party
(b) I – D, II – C, III – B, IV – A (b) Counsel of the State in a Criminal
(c) I – B , II – C, III – D, IV – A trial
(d) I – C, II – D, III – A, IV – B (c) Counsel of the State in a Civil trial
914. The killing of a new born child by its par- (d) Counsel of the State in both Criminal
ents and Civil trial
(a) Infanticide (b) Abortion 924. A lawyer appointed by court to represent a
(c) Foeticide (d) Malfeasance poor litigant is
915. Innuendo hints at (a) Amicus Curiae
(a) Insnuation (b) Defamation (b) Legal Aid man
(c) Slander (d) Libel (c) Lok Nyaya Pradayak
916. Match the following (d) None of the above
Col. I Col. II 925. ….is a general pardon.
I. Interlocutory A. By the fact (a) Remission (b) Reprieve
II. Ultra vires B. Not final (c) Amnesty (d) Suspension
III. Intra vires C. Beyond Powers 926. Husband and wife have a right to each
IV. Ipso facto D. Within Powers other’s company. This right is called
(a) I – A, II – B, III – C, IV – D (a) Legal right
(b) I – A, II – C, III – D, IV – B (b) Matrimonial right
(c) I – B, II – C, III – D, IV – A (c) Consortium right
(d) I – C, II – D, III – B, IV – A (d) Conjugal right
917. Mens rea is translated as 927. An act done under threat or fear is done
(a) Guilty mind (b) Intention under
(c) Evil (d) Unlawful act
(a) Duress
918. The famous M’ Naghten Rules determine
(b) Dunress
(a) Insanity as defence
(c) Compulsion
(b) Law of sea
(d) None of the above
(c) Distinguish hurt from grievous hurt
928. PIL means
(d) Established seasoning in criminal
cases (a) Private interest litigation
919. A child born after father’s death (b) Public interest litigation
(a) Postliminits (b) Bastard (c) Publicity interest litigation
(c) Posthumous (d) Postea (d) Proactive interest litigation

CLAT.indb 241 31/03/2009 11:42:00


242 CONSTITUTIONAL LAW

929. LLB stands for (c) Keshavanand Bharti case


(a) Law est jury (d) A. K. Gopalan case
(b) Legalite Lawyers Baccili 938. In which of the cases, the Supreme Court
(c) Legum Baccalaureus held that Legislative power of a State or
(d) None of the above Union is subject to the fundamental rights
930. Voting right comes under and the legislature cannot indirectly take
(a) Legal right away or abridge fundamental rights which
(b) Fundamental right it could not do directly for granting either
(c) Constitutional right recognition or aid to minority institu-
(d) None of the above tions?
931. Whoever has sexual intercourse with a (a) TMA Pai Foundation vs State of Kar-
person whom he knows or has reason to nataka
believe to be the wife of another man, (b) TMA Pai foundation vs Union of In-
without the consent or connivance of that dia
man, such sexual intercourse amounts to (c) TMA Pai Foundation vs State of Raj-
(a) Rape asthan
(b) Adultery (d) None of the above
(c) Unnatural offence 939. It is the Fundamental Right of the consum-
(d) Abatement of rape ers to know whether the food products,
932. Cruelty to a women by husband or relative cosmetics and drugs are of non-vegetarian
of husband is defined under or vegetarian origin, otherwise it will vio-
(a) Section 498A of Indian Penal Code late their fundamental rights under Articles
(b) Section 498 of Indian Penal Code 19 (1) (a), 21 and 25 of the Constitution of
(c) Section 497 of Indian Penal Code India. This observation is related to
(d) Section 496 of Indian Penal Code (a) Ozair Hussain vs Union of India
933. The first Indian Barrister (b) Vikash Despandey vs Bar council of
(a) Madhusudan Das India
(b) Chitranjan Das (c) TMA Pai foundation vs State of Kar-
(c) Subhash Chandra Bose nataka
(d) Jnanendra Mohan Tagore (d) None of the above
940. Supreme Court of India recognized sexual
934. The largest PIL (Public Interest Litigation)
harassment as a human rights violation in
winner
India in case of
(a) M. C. Mehta
(a) Vishaka vs State of Rajasthan
(b) Common Cause, a registered society
(b) Birdhichand vs State of Maharashtra
founded by H. D. Shourie
(c) Ahmed Khan vs Shah Bano Begum
(c) Helpage India
(d) Takaram vs State of Maharashtra
(d) M. K. Chawla
941. Among which of the cases the Supreme
935. The maxim ‘Ubi jus ibi remedium’ means
Court held that right of an Advocate to
(a) justice should not only be done but
appear and conduct cases in Courts in
should be seen to be done
still within the control and jurisdiction of
(b) in law there is a remedy for every
courts?
wrong (a) Ex-capt. Harish Uppal vs UOI
(c) justice according to law (b) Ram Narain Popli vs CBI
(d) none of the above (c) Dr Chanchal Goyal vs State of Rajas-
936. Contempt of court means than
(a) both civil contempt and criminal con- (d) Radhe Mohanlal vs Rajasthan High
tempt Court
(b) only criminal contempt 942. Which one of the following regional po-
(c) only civil contempt litical parties was recently accorded rec-
(d) none of the above ognition as a national party?
937. The Doctrine (Rule) of Basic Structure (a) Haryana Lok Dal
was propounded by the Supreme Court in (b) Bahujan Samaj Party
(a) Sajjan Singh case (c) Rashtriya Janata Dal
(b) Golak Nath case (d) None of the above

CLAT.indb 242 31/03/2009 11:42:00


CONSTITUTIONAL LAW 243

943. In which one of the following states, the 951. Who is the incharge of the Cabinet Secre-
Legislative Council was revived recently? tariat?
(a) Tamil Nadu (b) Andhra Pradesh (a) The Cabinet Secretary
(c) Rajasthan (d) Punjab (b) The Prime Minister
944. Who of the following is the Chairperson (c) The Union Home Minister
of the National Commission for the Pro- (d) The Minister for Parliamentary Af-
tection of Child Rights (NCPCR)? fairs
(a) Shiela Dixit 952. Which one of the following is the high-
(b) Savitri Sinha est body concerned with the approval of
(c) Shanta Sinha country’s Five Year Plans?
(d) None of the above (a) Planning Commission
945. Which of the following has been wrongly (b) Union Cabinet
listed as a recommendation of the Second (c) National Development Council
(d) The Parliament
Administrative Reforms Commission?
953. The salary and allowances of the High
(a) Constitutional status be accorded to
Court judges are paid out of
the Lokpal
(a) the Consolidated Fund of funds
(b) Lokpal should be renamed as the
(b) the Public Accounts Fund
Rashtriya Lokayukta
(c) Contingency Fund of India
(c) A National Judicial Council be set up (d) Public Accounts Fund of the con-
for the judges of Supreme Court and cerned states
High Court 954. Who among the following served as
(d) MPs and MLAs should be declared Speaker of Lok Sabha for two terms?
Public Authorities under the Right to (a) Balram Jakhar
Information (b) N. Sanjiva Reddy
946. National Human Rights Commission con- (c) G. M. C. Balayogi
sists of a Chairman and (d) All the above
(a) three members (b) four members 955. When were the High Courts of Bombay,
(c) five members (d) six members Calcutta and Madras established?
947. The first Mobile Court of India was (a) 1861 (b) 1935
launched in (c) 1948 (d) 1950
(a) Haryana (b) Rajasthan 956. A party to be recognized as a national
(c) Gujarat (d) Madhya Pradesh party must secure at least 4 per cent of the
948. Vijay L. Kelkar, who was appointed as the total votes polled in at least
Chairman of the 13th Finance Commis- (a) two states (b) three states
sion is a former (c) four states (d) five states
(a) Comptroller and Auditor-General 957. The government is empowered to collect
(b) Finance Secretary the revenues by
(c) Chairman of Planning Commission (a) the Finance Bill
(d) None of the above (b) the Economic Bill
949. Which one the following High Court en- (c) the Supplementary Bill
joys jurisdiction over largest number of (d) all the above bills
territories? 958. Who among the following has been ap-
(a) Calcutta pointed as the Chairman of the National
Land Reform Council?
(b) Punjab and Haryana
(a) Manmohan Singh
(c) Allahabad
(b) Pranab Mukherjee
(d) Guwahati
(c) M. Veerappa Moily
950. Who among the following allocates grants-
(d) N. R. Madhav Menon
in-aid to the states out of the Consolidated
959. Who among the following is the Chairman
Fund of India? of the Thirteenth Finance Commission?
(a) Public Accounts Committee (a) K. C. Pant
(b) Comptroller and Auditor General (b) Vijay Kelkar
(c) Finance Commission (c) C. Rangarajan
(d) Planning Commission (d) None of the above

CLAT.indb 243 31/03/2009 11:42:00


244 CONSTITUTIONAL LAW

960. Jasraj Chopra Panel was appointed to look (c) Australia


into the problem of (d) None of the above
(a) ragging in educational institutions 969. Who was the Chairman of the first Back-
(b) inclusion of Gujjars in the ST catego- ward Classes Committee?
ry (a) D. P. Mandal
(c) Godhra carnage (b) K. R. Narayanan
(d) none of the above (c) Kaka Kalelkar
961. Who heads the new Commission on Cen- (d) None of the above
tre–State relations appointed by the gov- 970. The Objective Resolution moved by Jawa-
ernment in April 2007? harlal Nehru was adopted by the Constitu-
(a) M. M. Punchhi ent Assembly on
(b) M. Veerappa Moily (a) 25 January 1947
(c) K. C. Pant (b) 22 January 1947
(d) N. R. Madhav Menon (c) 26 November 1947
962. In the present Lok Sabha the seats have (d) 26 January 1949
been allocated to various states on the ba- 971. Who of the following was elected as the
sis of Census of Vice-President of the Constituent Assem-
(a) 1981 (b) 1991 bly of India?
(c) 2001 (d) 1971 (a) H. C. Mukherjee
963. Which one of the following Commission (b) B. N. Rao
was set up in pursuance of the provisions (c) K. M. Munshi
of the Constitution? (d) D. P. Khaitan
(a) National Human Rights Commission 972. When was Presidential rule imposed in a
(b) Central Vigilance Commission state for the first time?
(c) Planning Commission (a) 1959 (b) 1956
(d) Election Commission (c) 1957 (d) 1952
964. Who among of the following was member 973. In which state was Presidential Rule im-
of Rajya Sabha at the time of appointment posed for the first time?
as Prime Minister? (a) Punjab (b) Kerala
(a) P. V. Narsimha Rao (c) West Bengal (d) Uttar Pradesh
(b) Charan Singh 974. In 2006 President Abdul Kalam sent back
a bill passed by Parliament for reconsid-
(c) Lal Bahadur Shastri
eration. The Bill was related to
(d) None of the above
(a) reservations for the OBCs
965. The Privy Purses of the former rulers of
(b) 56 offices of profit
Indian States were abolished by
(c) state funding of elections
(a) 24th Amendment Act
(d) None of the above
(b) 25th Amendment Act
975. The Right to Information Act passed in
(c) 28th Amendment Act
2005 is not applicable to the state of Jam-
(d) 26th Amendment Act
mu and Kashmir because
966. Part IV of the Indian constitution which
(a) the state legislature passed a special
lists the Fundamental Duties comprises of
resolution seeking exemption from
only
the Act
(a) one Article (b) of its special constitutional status
(b) two Articles (c) the National Security Council recom-
(c) three Articles mended the exemption of Jammu and
(d) None of the above Kashmir from its purview
967. At present, how many Fundamental Duties (d) of all the above reasons
are included in the Indian Constitution? 976. Which of the following states is regarded
(a) Ten (b) Eleven as the pioneer of the Right to Information
(c) Twelve (d) Thirteen in India?
968. In which country, the money bills can be (a) Rajasthan
initiated only in the Upper House? (b) West Bengal
(a) Japan (c) Himachal Pradesh
(b) France (d) Kerala

CLAT.indb 244 31/03/2009 11:42:00


CONSTITUTIONAL LAW 245

977. Under whose Prime Minstership was the (c) It sanctions the amounts spent out of
anti-defection law passed? the Consolidated Fund of India
(a) Rajiv Gandhi (d) None of the above
(b) Indira Gandhi 985. Who among the following is entitled to
(c) V. P. Singh exercise vote through postal ballot?
(d) A. B. Vajpayee (a) Official on election duty
978. The practice of ‘zero hour invention’ in (b) Members of armed forces
Parliament originated in India in (c) Member of Foreign Service posted
(a) 1964 (b) 1975 abroad
(c) 1984 (d) 1962 (d) all of these
979. Which of the following states has reserved 986. Which one of the following motion is re-
seats in the Assembly on the basis of reli- lated to the Budget?
gion? (a) Censure Motion
(a) Goa (b) Adjournment Motion
(b) Jammu and Kashmir (c) Cut Motion
(c) Nagaland (d) All the above
(d) None of the above 987. The ninety-third amendment of the consti-
980. Which one of the following Prime Minis- tution carried out in 2005 deals with
ters headed a minority Government? (a) provision of free and compulsory edu-
(a) V. P. Singh cation to children between 6 and 14
(b) I. K. Gujral years of age
(c) Chandra Shekhar (b) reservation of seats for backward
(d) All the above classes in public services
981. Who enjoys the power to determine the (c) reservation of 30 per cent seats for
strength of High Court judges? women in civil services
(a) The Parliament (d) None of the above
(b) The President 988. When was the first Minority Commission
(c) The Chief Justice of India set up?
(d) The Governor of the state in consulta- (a) 1954 (b) 1964
tion with the Chief Justice of India (c) 1979 (d) 1984
989. Who among the following draws the list
982. Who among the following ensures that no
of SC/ST and OBCs?
money is spent out of the Consolidated
(a) The Commissioner for Scheduled
Fund of India without the authority of the
Castes and Scheduled Tribes
Parliament?
(b) Union Home Minister
(a) The Estimates Committee
(c) The President
(b) The Public Accounts Committee
(d) The Parliament
(c) The Comptroller and Auditor General
990. Who appoints the Regional Election Com-
of India
missioners?
(d) The inter-state council
(a) The Chief Election Commissioner
983. Can the election to the office of President
(b) The Parliament
be held if one or more state Legislative
(c) The President, in consultation with
Assemblies have been dissolved? Chief Election Commissioner
(a) No (d) None of the above
(b) Yes 991. The total strength of the elected members
(c) Yes, only if one Assembly has been of the Jammu and Kashmir Assembly is
dissolved (a) 100 (b) 120
(d) Yes, with prior approval of the Chief (c) 135 (d) 150
Election Commissioner 992. The interpretation of the Constitution of
984. The Parliament exercises control over fi- India by the Supreme Court falls within
nances through several methods. Which of its
the following has been wrongly listed? (a) original jurisdiction
(a) It prepares and passes the central Bud- (b) appellate jurisdiction
get (c) advisory jurisdiction
(b) It levies and collects the taxes (d) none of the above

CLAT.indb 245 31/03/2009 11:42:00


246 CONSTITUTIONAL LAW

993. Which of the following statement regard- (a) Madhya Pradesh


ing the Secretary General of Lok Sabha is (b) Himachal Pradesh
incorrect? (c) Tamil Nadu
(a) He is appointed by the Speaker of the (d) Uttar Pradesh
Lok Sabha 997. How many times has the Supreme Court
(b) He acts as adviser to the Speaker and turned down the request of the Govern-
members of Lok Sabha with regard to ment for setting up regional branches of
Parliamentary matters and procedures the Supreme Court at Chennai, Kolkata
(c) The actions of Secretary General and Mumbai?
cannot be criticized in or outside the (a) only once (b) Twice
House (c) Thrice (d) Never
(d) None of these 998. The Nation Judicial Commission set up in
994. In which of the following cases the Su- October 2004 was expected to deal with
preme Court of India held that a Constitu- (a) appointment of judges
ent Assembly be convened to amend the
(b) transfer of judges
Fundamental Rights?
(c) removal of judges
(a) Keshavanand Bharti case
(d) all the above
(b) Sujjan Singh case
999. In February 2005, the National Integration
(c) Golak Nath case
(d) None of the above Council was re-constituted after a gap of
995. The Governor of a state can issue Ordi- (a) 5 years (b) 7 years
nances but these are subject to the approv- (c) 9 years (d) 12 years
al of 1000. Who headed the Committee on Civil Ser-
(a) the Chief Justice of State High Court vice Reforms which submitted its report in
(b) the President of India July 2004?
(c) the state legislature (a) Prof. Satish Chandra
(d) the Parliament (b) General (Retd) Surinder Singh
996. The Supreme Court upheld the decision of (c) Prof. Yashpal
which of the following states, for the abo- (d) P. C. Hota
lition of Administrative Courts?

CLAT.indb 246 31/03/2009 11:42:01


CONSTITUTIONAL LAW 247

ANSWERS

1. (c) 2. (d) 3. (b) 4. (c) 5. (b) 6. (d) 7. (b) 8. (c) 9. (d) 10. (b)
11. (c) 12. (b) 13. (d) 14. (a) 15. (a) 16. (b) 17. (a) 18. (d) 19. (b) 20. (a)
21. (b) 22. (b) 23. (a) 24. (b) 25. (b) 26. (c) 27. (d) 28. (c) 29. (d) 30. (b)
31. (a) 32. (b) 33. (d) 34. (b) 35. (b) 36. (d) 37. (d) 38. (c) 39. (b) 40. (c)
41. (c) 42. (d) 43. (a) 44. (d) 45. (a) 46. (b) 47. (d) 48. (a) 49. (d) 50. (d)
51. (c) 52. (d) 53. (d) 54. (d) 55. (d) 56. (c) 57. (a) 58. (a) 59. (a) 60. (a)
61. (a) 62. (d) 63. (b) 64. (b) 65. (d) 66. (d) 67. (a) 68. (a) 69. (d) 70. (a)
71. (d) 72. (a) 73. (b) 74. (c) 75. (a) 76. (a) 77. (d) 78. (b) 79. (b) 80. (b)
81. (b) 82. (a) 83. (c) 84. (c) 85. (c) 86. (c) 87. (c) 88. (a) 89. (a) 90. (b)
91. (c) 92. (d) 93. (b) 94. (d) 95. (b) 96. (c) 97. (c) 98. (a) 99. (c) 100. (d)
101. (a) 102. (c) 103. (c) 104. (b) 105. (c) 106. (c) 107. (b) 108. (c) 109. (b) 110. (c)
111. (d) 112. (c) 113. (d) 114. (d) 115. (b) 116. (a) 117. (c) 118. (b) 119. (a) 120. (a)
121. (a) 122. (b) 123. (d) 124. (a) 125. (b) 126. (a) 127. (d) 128. (c) 129. (d) 130. (d)
131. (c) 132. (d) 133. (c) 134. (a) 135. (b) 136. (a) 137. (c) 138. (d) 139. (a) 140. (a)
141. (a) 142. (b) 143. (d) 144. (c) 145. (b) 146. (b) 147. (b) 148. (c) 149. (b) 150. (a)
151. (d) 152. (b) 153. (c) 154. (b) 155. (d) 156. (d) 157. (d) 158. (a) 159. (c) 160. (b)
161. (a) 162. (d) 163. (d) 164. (c) 165. (c) 166. (d) 167. (c) 168. (d) 169. (d) 170. (d)
171. (b) 172. (c) 173. (b) 174. (c) 175. (c) 176. (b) 177. (b) 178. (c) 179. (c) 180. (d)
181. (d) 182. (b) 183. (a) 184. (d) 185. (b) 186. (b) 187. (a) 188. (d) 189. (d) 190. (c)
191. (c) 192. (a) 193. (b) 194. (b) 195. (c) 196. (c) 197. (b) 198. (a) 199. (b) 200. (c)
201. (a) 202. (d) 203. (a) 204. (a) 205. (c) 206. (b) 207. (d) 208. (d) 209. (a) 210. (d)
211. (b) 212. (d) 213. (c) 214. (c) 215. (c) 216. (c) 217. (b) 218. (d) 219. (c) 220. (a)
221. (d) 222. (a) 223. (c) 224. (d) 225. (d) 226. (c) 227. (d) 228. (b) 229. (a) 230. (d)
231. (a) 232. (b) 233. (c) 234. (b) 235. (c) 236. (b) 237. (d) 238. (b) 239. (b) 240. (b)
241. (b) 242. (d) 243. (b) 244. (c) 245. (c) 246. (c) 247. (c) 248. (b) 249. (b) 250. (a)
251. (c) 252. (d) 253. (d) 254. (c) 255. (b) 256. (d) 257. (b) 258. (b) 259. (d) 260. (b)
261. (b) 262. (d) 263. (c) 264. (b) 265. (c) 266. (d) 267. (d) 268. (a) 269. (a) 270. (d)
271. (a) 272. (c) 273. (c) 274. (b) 275. (d) 276. (a) 277. (a) 278. (a) 279. (c) 280. (c)
281. (c) 282. (b) 283. (d) 284. (c) 285. (b) 286. (b) 287. (b) 288. (b) 289. (c) 290. (c)
291. (c) 292. (b) 293. (c) 294. (a) 295. (d) 296. (d) 297. (b) 298. (b) 299. (b) 300. (b)
301. (d) 302. (a) 303. (d) 304. (a) 305. (b) 306. (d) 307. (c) 308. (c) 309. (d) 310. (c)
311. (a) 312. (c) 313. (b) 314. (b) 315. (c) 316. (d) 317. (d) 318. (a) 319. (b) 320. (d)
321. (c) 322. (b) 323. (a) 324. (d) 325. (b) 326. (b) 327. (d) 328. (a) 329. (c) 330. (d)
331. (b) 332. (b) 333. (c) 334. (c) 335. (c) 336. (b) 337. (a) 338. (a) 339. (a) 340. (a)
341. (a) 342. (d) 343. (d) 344. (c) 345. (d) 346. (b) 347. (a) 348. (c) 349. (c) 350. (c)
351. (b) 352. (a) 353. (a) 354. (d) 355. (a) 356. (c) 357. (d) 358. (b) 359. (b) 360. (d)
361. (a) 362. (a) 363. (b) 364. (a) 365. (d) 366. (a) 367. (d) 368. (d) 369. (a) 370. (b)
371. (d) 372. (b) 373. (d) 374. (b) 375. (a) 376. (b) 377. (c) 378. (c) 379. (d) 380. (d)
381. (d) 382. (b) 383. (d) 384. (a) 385. (b) 386. (a) 387. (d) 388. (d) 389. (d) 390. (c)
391. (d) 392. (b) 393. (b) 394. (d) 395. (c) 396. (a) 397. (b) 398. (c) 399. (d) 400. (c)
401. (b) 402. (c) 403. (d) 404. (a) 405. (a) 406. (b) 407. (d) 408. (c) 409. (c) 410. (b)
411. (a) 412. (c) 413. (a) 414. (c) 415. (a) 416. (d) 417. (a) 418. (c) 419. (c) 420. (a)
421. (b) 422. (d) 423. (a) 424. (a) 425. (d) 426. (a) 427. (d) 428. (a) 429. (a) 430. (d)
431. (b) 432. (a) 433. (c) 434. (c) 435. (d) 436. (c) 437. (d) 438. (d) 439. (c) 440. (b)
441. (a) 442. (a) 443. (b) 444. (d) 445. (c) 446. (d) 447. (a) 448. (b) 449. (d) 450. (a)
451. (b) 452. (d) 453. (a) 454. (d) 455. (b) 456. (c) 457. (c) 458. (c) 459. (c) 460. (a)
461. (b) 462. (a) 463. (b) 464. (d) 465. (b) 466. (c) 467. (b) 468. (a) 469. (b) 470. (a)
471. (c) 472. (b) 473. (b) 474. (b) 475. (d) 476. (b) 477. (a) 478. (c) 479. (d) 480. (c)
481. (c) 482. (a) 483. (b) 484. (c) 485. (b) 486. (d) 487. (c) 488. (b) 489. (a) 490. (c)
491. (d) 492. (a) 493. (d) 494. (a) 495. (d) 496. (a) 497. (c) 498. (b) 499. (c) 500. (c)

CLAT.indb 247 31/03/2009 11:42:01


248 CONSTITUTIONAL LAW

501. (d) 502. (a) 503. (a) 504. (a) 505. (b) 506. (d) 507. (a) 508. (d) 509. (b) 510. (c)
511. (d) 512. (c) 513. (c) 514. (a) 515. (a) 516. (a) 517. (a) 518. (d) 519. (a) 520. (a)
521. (d) 522. (b) 523. (a) 524. (c) 525. (c) 526. (b) 527. (c) 528. (b) 529. (b) 530. (c)
531. (c) 532. (b) 533. (a) 534. (c) 535. (c) 536. (d) 537. (a) 538. (d) 539. (b) 540. (d)
541. (a) 542. (c) 543. (d) 544. (b) 545. (d) 546. (a) 547. (d) 548. (c) 549. (c) 550. (b)
551. (c) 552. (b) 553. (b) 554. (a) 555. (b) 556. (a) 557. (d) 558. (a) 559. (b) 560. (c)
561. (b) 562. (d) 563. (a) 564. (a) 565. (d) 566. (d) 567. (b) 568. (c) 569. (b) 570. (c)
571. (a) 572. (b) 573. (d) 574. (c) 575. (c) 576. (c) 577. (d) 578. (c) 579. (b) 580. (b)
581. (d) 582. (c) 583. (c) 584. (d) 585. (b) 586. (c) 587. (d) 588. (d) 589. (c) 590. (c)
591. (c) 592. (c) 593. (d) 594. (c) 595. (d) 596. (a) 597. (d) 598. (d) 599. (a) 600. (d)
601. (a) 602. (b) 603. (c) 604. (c) 605. (a) 606. (a) 607. (b) 608. (a) 609. (a) 610. (a)
611. (b) 612. (d) 613. (a) 614. (a) 615. (b) 616. (d) 617. (c) 618. (b) 619. (c) 620. (c)
621. (d) 622. (d) 623. (b) 624. (a) 625. (c) 626. (a) 627. (a) 628. (c) 629. (b) 630. (c)
631. (d) 632. (a) 633. (d) 634. (c) 635. (a) 636. (c) 637. (d) 638. (c) 639. (a) 640. (d)
641. (d) 642. (b) 643. (c) 644. (b) 645. (b) 646. (b) 647. (d) 648. (b) 649. (c) 650. (b)
651. (a) 652. (b) 653. (c) 654. (d) 655. (c) 656. (d) 657. (a) 658. (b) 659. (d) 660. (d)
661. (a) 662. (b) 663. (b) 664. (d) 665. (a) 666. (a) 667. (c) 668. (c) 669. (b) 670. (a)
671. (b) 672. (d) 673. (d) 674. (b) 675. (a) 676. (b) 677. (d) 678. (b) 679. (a) 680. (c)
681. (b) 682. (c) 683. (c) 684. (a) 685. (a) 686. (d) 687. (a) 688. (b) 689. (a) 690. (c)
691. (c) 692. (c) 693. (d) 694. (b) 695. (a) 696. (c) 697. (c) 698. (b) 699. (c) 700. (a)
701. (c) 702. (a) 703. (d) 704. (b) 705. (c) 706. (b) 707. (d) 708. (b) 709. (a) 710. (d)
711. (d) 712. (b) 713. (a) 714. (c) 715. (d) 716. (a) 717. (d) 718. (a) 719. (b) 720. (d)
721. (c) 722. (d) 723. (d) 724. (b) 725. (d) 726. (c) 727. (b) 728. (b) 729. (d) 730. (a)
731. (d) 732. (b) 733. (d) 734. (c) 735. (c) 736. (c) 737. (d) 738. (b) 739. (c) 740. (a)
741. (d) 742. (d) 743. (b) 744. (a) 745. (c) 746. (a) 747. (d) 748. (d) 749. (b) 750. (b)
751. (a) 752. (b) 753. (b) 754. (c) 755. (b) 756. (b) 757. (a) 758. (a) 759. (b) 760. (d)
761. (a) 762. (c) 763. (b) 764. (a) 765. (d) 766. (d) 767. (b) 768. (a) 769. (a) 770. (a)
771. (d) 772. (d) 773. (c) 774. (a) 775. (b) 776. (c) 777. (a) 778. (a) 779. (d) 780. (d)
781. (b) 782. (d) 783. (d) 784. (a) 785. (d) 786. (b) 787. (b) 788. (d) 789. (a) 790. (b)
791. (a) 792. (b) 793. (d) 794. (b) 795. (d) 796. (b) 797. (a) 798. (b) 799. (a) 800. (c)
801. (b) 802. (b) 803. (c) 804. (b) 805. (c) 806. (c) 807. (c) 808. (c) 809. (c) 810. (a)
811. (c) 812. (b) 813. (b) 814. (a) 815. (b) 816. (c) 817. (a) 818. (d) 819. (b) 820. (d)
821. (b) 822. (c) 823. (a) 824. (c) 825. (d) 826. (c) 827. (b) 828. (d) 829. (c) 830. (a)
831. (a) 832. (c) 833. (d) 834. (c) 835. (c) 836. (c) 837. (c) 838. (d) 839. (a) 840. (c)
841. (b) 842. (a) 843. (c) 844. (c) 845. (b) 846. (a) 847. (b) 848. (a) 849. (a) 850. (a)
851. (d) 852. (b) 853. (c) 854. (c) 855. (b) 856. (c) 857. (a) 858. (b) 859. (b) 860. (b)
861. (d) 862. (a) 863. (c) 864. (d) 865. (b) 866. (d) 867. (b) 868. (d) 869. (b) 870. (b)
871. (a) 872. (a) 873. (c) 874. (a) 875. (b) 876. (a) 877. (b) 878. (d) 879. (d) 880. (c)
881. (c) 882. (a) 883. (d) 884. (a) 885. (b) 886. (c) 887. (b) 888. (a) 889. (a) 890. (a)
891. (a) 892. (d) 893. (a) 894. (a) 895. (c) 896. (a) 897. (a) 898. (a) 899. (b) 900. (b)
901. (c) 902. (a) 903. (b) 904. (d) 905. (c) 906. (a) 907. (a) 908. (a) 909. (a) 910. (a)
911. (b) 912. (a) 913. (b) 914. (a) 915. (b) 916. (c) 917. (a) 918. (a) 919. (c) 920. (a)
921. (b) 922. (d) 923. (a) 924. (a) 925. (c) 926. (d) 927. (a) 928. (b) 929. (c) 930. (c)
931. (b) 932. (a) 933. (b) 934. (a) 935. (b) 936. (a) 937. (c) 938. (a) 939. (a) 940. (a)
941. (a) 942. (d) 943. (b) 944. (c) 945. (d) 946. (c) 947. (a) 948. (b) 949. (d) 950. (a)
951. (a) 952. (c) 953. (a) 954. (a) 955. (a) 956. (b) 957. (d) 958. (d) 959. (b) 960. (b)
961. (a) 962. (d) 963. (c) 964. (a) 965. (b) 966. (a) 967. (b) 968. (c) 969. (a) 970. (a)
971. (c) 972. (a) 973. (d) 974. (b) 975. (b) 976. (b) 977. (a) 978. (a) 979. (b) 980. (d)
981. (a) 982. (c) 983. (b) 984. (d) 985. (d) 986. (c) 987. (d) 988. (a) 989. (a) 990. (c)
991. (b) 992. (c) 993. (c) 994. (c) 995. (c) 996. (a) 997. (c) 998. (d) 999. (a) 1000. (d)

CLAT.indb 248 31/03/2009 11:42:01


Chapter 5
MOCK TESTS
PRACTICE PAPER 1
Maximum Marks: 100 Marks 91–100—Outstanding
Time: 30 Minutes Marks 81–90—Excellent
Marks 71–80—Intelligent
Marks 61–70—Good
Marks 51–60—Average
Marks Less than 50—Below Average

1. Who is competent to dissolve the Parlia- shall be duty of every parent or guardian
ment of India? to provide opportunities for education to
(a) President his child or ward between age of 6 to 14
(b) Joint Session of Parliament years
(c) Chairman of Rajya Sabha and Speaker (a) 85th Amendment, 2002
of Lok Sabha together (b) 86th Amendment, 2002
(d) None of the above (c) 87th Amendment, 2003
2. The words “procedure established by (d) 88th Amendment, 2003
law” in Article 21 in Constitution of India 6. Who headed the Committee on Civil Ser-
means vice Reforms which submitted its report in
(a) that due process of law must be fol- July 2004?
lowed (a) Prof. Satish Chandra
(b) a procedure laid down or enacted by a (b) General (Retd) Surinder Singh
competent authority (c) Prof. Yashpal
(c) the same things as “due process of (d) P. C. Hota
law” 7. Which one of the following Prime Minis-
ters headed a minority Government?
(d) a law which is reasonable, fair and
(a) V. P. Singh
just
(b) I. K. Gujral
3. The Right to Free and compulsory Edu-
(c) Chandra Shekhar
cation to Children Act, 2009 and Right
(d) All the above
to Education in the Indian Constitution
8. Under whose Prime Minstership was the
provided under the Chapter, Fundamental anti-defection law passed?
Rights pertains to (a) Rajiv Gandhi
(a) Article 21 (b) Article 21 A (b) Indira Gandhi
(c) Article 21 B (d) Article 21 C (c) V. P. Singh
4. President of India is elected (d) A. B. Vajpayee
(a) by way of people representation 9. Which one of the following states is re-
(b) by an electoral college consisting garded as the pioneer of the Right to In-
of the elected members of both the formation in India?
Houses of Parliament and by the (a) Rajasthan
representatives of Central and State (b) West Bengal
Legislature (c) Himachal Pradesh
(c) directly by people (d) Kerala
(d) Prime Minister appoints him 10. In 2006 President Abdul Kalam sent back
5. Which among the following Indian Con- a bill passed by Parliament for reconsid-
stitutional Amendments provides that it eration. The Bill related to

CLAT-5-Mock Tests.indd 249 13/09/2010 10:36:21


250 MOCK TESTS

(a) reservations for the OBCs 18. Supreme Court of India recognized sexual
(b) 56 offices of profit harassment as a human rights violation in
(c) state funding of elections India in case of
(d) None of the above (a) Vishaka vs State of Rajasthan
11. Which of the following jurisdiction of the (b) Birdhichand vs State of Maharashtra
Supreme Court of India is wrongly listed? (c) Ahmed Khan vs Shah Bano Begum
(a) Advisory Jurisdiction (d) Takaram vs State of Maharashtra
(b) Appellate Jurisdiction 19. The maxim ‘Ubi jus’ibi remedium’ means
(c) Original Jurisdiction (a) Justice should not only be done but
(d) None of the above should be seen to be done
12. As per new Policy of Union Government, (b) In law there is a remedy for every
who among the following sects can caste wrong
their vote in 2014 General Elections in In- (c) Justice according to law
dia? (d) None of the above
(a) Prisoners
20. The largest PIL(Public Interest Litigation)
(b) Foreign Nationals
winner
(c) Both Prisoners and Foreign Nationals
(a) M. C. Mehta
(d) Non-Resident Indians (NRIs) and
(b) Common Cause a registered society
Overseas Citizens having Indian Pass-
ports founded by H. D. Shourie.
13. TRIPs forming part of the World Trade (c) Helpage India
Organization is intended (d) M. K. Chawla
(a) to promote transnational corporate in- 21. The “Legal Services Day” falls on
terests (a) 9 November (b) 10 December
(b) to provide for strong patent protec- (c) 26 January (d) 13 February
tion 22. The “Citizens Day” falls on
(c) to replace World Intellectual Property (a) 18 November (b) 19 November
Organization (c) 20 November (d) 21 November
(d) to harmonize intellectual property 23. Sovereignty in India means
rights internationally (a) Apex Authority or Alternative Author-
14. When the two Houses of Parliament of In- ity
dia differ regarding a Bill, then the dead- (b) Lower Authority or Immediate Au-
lock is solved by thority
(a) a joint of sitting of the two Houses (c) Authority of Court
(b) the President of India (d) Authority of Parliament
(c) Speaker of Lok Sabha 24. There are two classes of Advocates in In-
(d) Subjects Committee dia
15. In the Federation established by the Gov- (a) Barristers and Solicitors
ernment of India Act of 1935, residuary (b) Advocates and Senior Advocates
powers were given to (c) Lawyers and Attorneys
(a) Federal Legislature (d) Supreme Court Advocates and High
(b) Provincial Legislature Court Advocates
(c) Governor General 25. Who is the personality to Head UNESCO
(d) Provincial Governors for first time in history?
16. The first Mobile Court of India was (a) Ms Irina Bokova
launched in
(b) Ms Fatiam Bibi
(a) Haryana (b) Rajasthan
(c) Ms Anjali Roy
(c) Gujarat (d) Madhya Pradesh
(d) Ms Leela Jethamalani
17. Who of the following is the Chairperson
26. ‘Ultra Vires’ is a term used for
of the National Commission for the Pro-
(a) A document corrupted by virus
tection of Child Rights (NCPCR)?
(a) Shiela Dixit (b) An act beyond the authority of a per-
(b) Savitri Sinha son
(c) Shanta Sinha (c) An act authorized by law
(d) None of the above (d) An illegal act

CLAT-5-Mock Tests.indd 250 13/09/2010 10:36:22


MOCK TESTS 251

27. Cabinet includes 35. The Chairman of Rajya Sabha in Parlia-


1. Cabinet Ministers ment of India
2. Ministers of State (a) enjoys right to vote like other mem-
3. Deputy Ministers bers
(a) 1, 2 and 3 (b) 1 and 2 (b) has right to vote in case of a tie
(c) 1 and 3 (d) 1 only (c) can vote only in case of Constitutional
28. India is a Republic. It means: amendment
(a) India is a Union of States (d) does not enjoy any vote
(b) The Head of the State is not heredi- 36. Fundamental Duties do not enjoy any:
tary (a) Legal sanction
(c) India has Parliamentary system of (b) Moral sanction
government (c) Political sanction
(d) People have final authority in all the (d) Social sanction
matters 37. The words “Satyameva Jayate”, inscribed
29. The Governor does appoint the below the National Emblem, are taken
1. Judges of the High Court from
2. Chief Minister (a) Mundaka Upanishad
3. Chairman of the State Public Service (b) Jataka
Commission (c) Puranas
4. Advocate-General of the State (d) Mahabharata
(a) 1, 2, 3, 4 (b) 2, 3, 4 only 38. In the Indian polity, ultimate sovereignty
(c) 1, 2, 4 only (d) 2, 4 only rests with
30. The Term of Rajya Sabha members was (a) the Parliament
(b) the Constitution
fixed by the
(c) the President
(a) Parliament (b) Cabinet
(d) the People
(c) President (d) Constitution
39. A Money Bill can originate in the State
31. A High Court Judge in India is removed
Legislature only with the prior consent of
by
the
(a) the President’s Order
I. Chief Minister
(b) both Houses of Parliament
II. President
(c) the Chief Justice of India
III. Governor
(d) President’s Order, after both the IV. None of these
Houses of Parliament pass the im- (a) I (b) I and III
peachment resolution (c) II (d) III
32. Who among the following was the Chair- 40. Who conducts the State assembly elec-
man of Commission on Corporate Gover- tions?
nance in India? (a) Governor of the state concerned
(a) J. R. Verma (b) Chief Justice of the Supreme Court
(b) K. C. Pant (c) Chief Justice of the High court con-
(c) P. Chidambaram cerned
(d) Jaswant Singh (d) Chief Election Commission
33. Which of the following is not a Financial 41. The President of India appoints Judges of
Committee of the Parliament of India? the Supreme Court after consultation with
(a) Public Accounts Committee (a) Chief Justice of India
(b) Estimates Committee (b) the Prime Minister
(c) Committee on Public Undertakings (c) Law Minister in the Union Cabinet
(d) Committee on Private Members’ Bills (d) the Chief Justice of the High Courts
and Resolutions 42. Which Part of the Constitution of India
34. Who is considered to be the guardian of seeks to provide economic justice to the
the Public Purse in India? citizen of India?
(a) Parliament (a) Preamble
(b) Comptroller and Auditor General (b) Fundamental Rights
(c) Public Accounts Committee (c) Directive Principles of State Policy
(d) President (d) None of the above

CLAT-5-Mock Tests.indd 251 13/09/2010 10:36:23


252 MOCK TESTS

43. Who among the following can create a 51. Article 340 of the Indian Constitution
new State in the Indian Union? deals with
(a) President (a) Backward Classes Commission
(b) Parliament (b) Election Commission
(c) Union Cabinet (c) Law Commission
(d) any two or more States combined (d) Finance Commission
44. Who appoints the Chiefs of the Army, 52. A Non Money Bill passed by the Parlia-
Navy and Air Force in India? ment is returned by the President of India
(a) President of India to Parliament for reconsideration. It is
(b) Prime Minister passed once again by the Parliament with-
(c) Defence Minister out any change. Now
(d) Parliament (a) President can again withhold his as-
45. For what period does the judge of a High sent
Court in India hold office? (b) Bill will automatically lapse
(a) 6 years till the age of 60 years (c) Bill will be referred to the Supreme
(b) till the age of 60 years Court
(c) till the age of 62 years (d) President will give his assent
(d) till the age of 65 years 53. Decision regarding disqualification of
46. Part V of the Constitution deals with members of Lok Sabha in Parliament of
I. Union Executive India are taken by
II. Parliament (a) Speaker
III. Supreme Court and High Court (b) Prime Minister
IV. Comptroller and Auditor-General (c) Minister of Parliamentary Affairs
(a) I, II and III (b) I, II and IV (d) Parliamentary Secretary
(c) I and II (d) I only 54. The salaries and other emoluments of the
47. Which of the following are matched members of Parliament in India are decid-
wrongly? ed by the
I. Habeas Corpus—literally means ‘to (a) Prime Minister
have a body’ (b) Parliament Staff
II. Mandamus—commands a person to (c) President
perform a public duty (d) Ministry of Parliamentary Affairs
III. Quo warranto—issued to a lower 55. Which among the following makes rec-
court to stop proceedings in a case ommendations to the President of India as
IV. Prohibition—issued to a lower court to the principles which should govern the
quashing a decision or order grant in aid of the revenues of the States
(a) I and II (b) III only out of the Consolidated Fund of India?
(c) II, III and IV (d) III and IV (a) Finance Commission
48. Who adopted the Constitution of India on (b) Planning Commission
the 26 November 1949? (c) National Development Commission
(a) Parliament of India (d) Ministry of Finance of Government of
(b) People of India India
(c) Representative of the people of India 56. Why is the year 1952 important in Indian
in Constituent Assembly History?
(d) Cabinet (a) State Reorganization Act was passed
49. Which article has abolished the child la- (b) Haryana State was split
bour? (c) First official census in India was held
(a) 24 (b) 27 (d) First general elections to the Lok Sab-
(c) 29 (d) 34 ha were held
50. The Supreme Court Judges cannot prac- 57. Theocracy is
tise anywhere after retirement (a) an authoritarian regime
(a) At District and Session Courts (b) the art of stealing
(b) At High Courts (c) a religious state
(c) At Supreme Court (d) a state created as a result of coup
(d) At nowhere d’etat

CLAT-5-Mock Tests.indd 252 13/09/2010 10:36:23


MOCK TESTS 253

58. A lawyer appointed by court to represent a 67. The Vice-President is elected by an Elec-
poor litigant is toral College consisting of members of
(a) Lok Nyaya Pradayak (a) both Houses of Parliament
(b) Anucus Curiae (b) Lok Sabha
(c) Legal Aid Man (c) Rajya Sabha
(d) None of the above (d) both Houses of Parliament and State
59. Supreme Court’s advisory jurisdiction is Legislative Assemblies
provided under Article…. of the Constitu- 68. In India, political parties are given recog-
tion nition by
(a) 226 (b) 143 (a) President
(c) 124 (d) 123 (b) Law Commission
60. Indira Swahney vs Union of India is a case (c) Speaker of Lok Sabha
popularly known as (d) Election Commission
(a) Mandal judgment 69. The Parliament can amend the Constitu-
(b) Ayodhya judgment tion by way of
(c) Suicidal judgment (a) addition (b) variation
(d) Election Commission judgment (c) repeal (d) All of these
61. Jurisdiction of a High Court in India can 70. The Right to Property ceased to be a fun-
be restricted or extended by damental Right from
(a) Chief Justice of India (a) 1975 (b) 1976
(b) Chief Justice of State (c) 1978 (d) 1979
(c) Parliament 71. Application to a higher court to examine,
(d) President against a case decided by a lower court
62. Central Government in India can assign and possibly giving a different decision is
any function to the State with the consent called
of (a) petition (b) writ petition
(a) Chief Justice of India (c) suit (d) appeal
(b) Parliament 72. Usually laws are made by Legislature.
(c) State Government Sometimes legislature gives the power of
(d) President making orders, rules, etc. to the Executive.
These orders, rules, etc. are called ……….
63. The provisions of Criminal Procedure
They are also called subordinate legisla-
Code, 1973 provides ‘for arrest of of-
tion
fender by police. Whether the Magistrate
(a) Executive laws
under this code
(b) Sub-laws
(a) can arrest the offender
(c) Bye-laws
(b) cannot arrest the offender
(d) None of the above
(c) can arrest by police
73. Which among the following Articles of
(d) can arrest, himself or direct the police
Constitution of India States that Governor
to arrest
of a State acts on the advice of Council of
64. Within the meaning of Copyright Act,
Ministers of the concerned State
1957, the performer’s right shall subsist
(a) Article 163 (b) Article 167
until (c) Article 170 (d) Article 171
(a) 60 years (b) 55 years 74. What is the time period for which emer-
(c) 50 years (d) 70 years gency is extended once approved by both
65. The Committee on Prevention of Corrup- Houses of Parliament?
tion in India was headed by (a) 6 months (b) 7 months
(a) K. Santhanam (b) M. S. Phukan (c) 9 months (d) one year
(c) A. S. Anand (d) J. S. Verma 75. Name the place in Bombay now Mumbai
66. The Election Commission does not con- where the historic quit India Movement
duct the elections to the declaration was made in 1942?
(a) Lok Sabha (a) Gowalia Tank Maidan
(b) Rajya Sabha (b) Shiraj Maidan
(c) A Local bodies (c) Annie Besant Road
(d) President’s elections (d) Juhu Beach

CLAT-5-Mock Tests.indd 253 13/09/2010 10:36:23


254 MOCK TESTS

76. The correct nomenclature of India accord- 83. Which country among the following is the
ing to the Preamble is first country in Europe to guarantee ani-
(a) Sovereign, Secular, Democratic Re- mal rights in its Constitution
public (a) Germany (b) Switzerland
(b) Sovereign, Democratic republic (c) Belgium (d) Belgrade
(c) Sovereign, Socialist, Secular, Demo- 84. In the Constitution of India, the name of
cratic Republic country is ______
(d) Sovereign, Secular, Socialist Democ- (a) Bharat (b) India
racy (c) Hindustan (d) Aryavart
77. The Constitution of India 85. The Prime Minister in India is
(a) does not provide the Judicial Review (a) appointed by the President
(b) provides for Judicial Review on USA (b) elected by the People
pattern (c) elected by the Lok Sabha
(c) provides for Judicial Review with (d) elected by both the Houses of the Par-
limited scope liament
(d) provides for Judicial Review as an in- 86. Which of the following non-members of
tegral part of it Parliament has the right to address it?
78. Which of the following is not a Directive (a) Solicitor-General of India
Principle of the State Policy? (b) Chief Justice of India
(a) To raise the level of nutrition (c) Attorney General of India
(b) To develop the scientific temper (d) Chief Election Commissioner
(c) To promote economic interests of (e) Chairman, Finance Commission
weaker sections 87. Which of the following states does not
(d) To separate the Judiciary from the Ex- have a bicameral legislature?
ecutive (a) Bihar (b) Karnataka
79. Chapter III on the Fundamental Rights (c) Rajasthan (d) Maharashtra
contains twenty four articles from 88. Which of the following can be abolished,
(a) Article 12 to 35 but not dissolved?
(b) Article 14 to 37 (a) Rajya Sabha
(c) Article 10 to 33 (b) Municipal Bodies
(d) Article 16 to 19 (c) State Legislative Council
80. The Rajya Sabha can be dissolved by (d) None of the above
(a) Lok Sabha 89. The distribution of seats of the Parliament
(b) Constitutional amendment are based on which of the following cen-
(c) President sus?
(d) None of these (a) 1951 (b) 1961
81. Which of the following statements is not (c) 1971 (d) 1981
correct about the objectives of the Plan- 90. The Government of India introduced
ning in India? Bharat Ratna and Padam Shri awards un-
(a) To raise the standard of living in the der
people (a) Article 25 of the Constitution
(b) To open out to the people new oppor- (b) Article 18 of the Constitution
tunities for a richer and more varied (c) Article 14 of the Constitution
life (d) None of the above
(c) To attain economic equality and social 91. “Plebiscite” is a device for democracy
justice used for
(d) To establish progressive society (a) ascertaining the views of people on a
82. The Planning Commission of India was policy matter of public importance
set up by (b) enactment of law
(a) the Parliament (c) appointment of civil servants
(b) joint decision of the Union Govern- (d) appointment of judges
ment and States 92. Which among the following women suc-
(c) executive order of the Union Govern- ceeds Ms Condo Leezza Rice to become
ment the Secretary for State of USA in Barrack
(d) an Ordinance of the President Obama Government?

CLAT-5-Mock Tests.indd 254 13/09/2010 10:36:23


MOCK TESTS 255

(a) Ms Michelle (c) Chief Minister of all States


(b) Ms Hillary Clinton (d) Prime Minister
(c) Ms Chelsea 97. Article 78 of the Constitution deals with
(d) Ms Palin (a) Emergency powers of the President
93. The name of the Parliament of USA is (b) Prime Minister’s duty regarding keep-
(a) House of Lords ing the President informed about the
(b) House of Commons government’s Decisions and policies
(c) Senate (c) President’s power to send advisory
(d) White House messages to the Parliament
94. The Election Commission of India is a (d) President’s power to get information
(a) political body because it deals with from the Council of Ministers
election matters 98. The Chairman of the Planning Commis-
sion in India is the
(b) statutory body
(a) Minister of Planning
(c) constitutional body
(b) Finance Minister
(d) parliamentary body
(c) President
95. Which Indian candidate contested the
(d) Prime Minister
elections for the prestigious post of Secre- 99. The minimum age required for becoming
tary-General of United Nations Organiza- the Prime Minister of India is
tion (UNO)? (a) 25 years (b) 30 years
(a) Shashi Tharoor (c) 40 years (d) 35 years
(b) A. K. Antony 100. Legally speaking who can declare war or
(c) Ashok Chauhan peace?
(d) Sonia Gandhi (a) Defence Minister
96. Under whose advice the President of India (b) President of India
declares Emergency under Article 352? (c) Prime Minister of India
(a) Council of Ministers (d) Chiefs of Army, Navy and Air Force
(b) Cabinet together

ANSWERS
1. (d) 2. (d) 3. (b) 4. (b) 5. (b) 6. (d) 7. (d) 8. (a) 9. (b) 10. (b)
11. (d) 12. (d) 13. (c) 14. (a) 15. (a) 16. (a) 17. (c) 18. (a) 19. (b) 20. (a)
21. (a) 22. (c) 23. (d) 24. (b) 25. (a) 26. (b) 27. (a) 28. (b) 29. (b) 30. (d)
31. (d) 32. (a) 33. (b) 34. (b) 35. (b) 36. (c) 37. (a) 38. (d) 39. (d) 40. (d)
41. (a) 42. (c) 43. (b) 44. (c) 45. (c) 46. (d) 47. (d) 48. (c) 49. (a) 50. (d)
51. (d) 52. (d) 53. (a) 54. (d) 55. (a) 56. (d) 57. (c) 58. (b) 59. (b) 60. (a)
61. (c) 62. (c) 63. (d) 64. (a) 65. (a) 66. (c) 67. (a) 68. (d) 69. (d) 70. (c)
71. (d) 72. (c) 73. (d) 74. (d) 75. (b) 76. (c) 77. (d) 78. (b) 79. (a) 80. (d)
81. (d) 82. (c) 83. (c) 84. (a) 85. (a) 86. (c) 87. (c) 88. (c) 89. (c) 90. (b)
91. (a) 92. (b) 93. (b) 94. (c) 95. (a) 96. (b) 97. (b) 98. (d) 99. (a) 100. (b)

CLAT-5-Mock Tests.indd 255 13/09/2010 10:36:23


256 MOCK TESTS

PRACTICE PAPER 2
Maximum Marks: 100 Marks 91–100—Outstanding
Time: 30 Minutes Marks 81–90—Excellent
Marks 71–80—Intelligent
Marks 61–70—Good
Marks 51–60—Average
Marks Less than 50—Below Average

1. “Sovereignty” in India means (c) Lal Bahadur Shastri


(a) Apex or alternative Authority (d) None of the above
(b) Lower Authority or Immediate Au- 8. Who heads the new Commission on Cen-
thority tre–State relations appointed by the gov-
(c) Authority of Judiciary ernment in April 2007?
(d) Supreme pre-eminence and indepen- (a) M. M. Punchhi
dent authority (b) M. Veerappa Moily
2. Which classes of Advocates are in India? (c) K. C. Pant
(a) Barristers and Solicitors (d) N. R. Madhav Menon
(b) Advocates and Senior Advocates 9. Who of the following is the Chairman of
(c) Lawyers and Attorneys the Thirteenth Finance Commission?
(d) Supreme Court Advocates and High (a) K. C. Pant
Court Advocates (b) Vijay Kelkar
3. Who is present Chief Justice of India? (c) C. Rangarajan
(a) Justice R. C. Lahoti (d) None of the above
(b) Justice K. G. Balakrishnan 10. Which one of the following is the high-
(c) Justice Y. K. Sabharwal est body concerned with the approval of
(d) Justice Kapadia country’s Five Year Plans?
4. The Constitution of India provides for (a) Planning Commission
(a) Right to life (b) Union Cabinet
(b) Right to work (c) National Development Council
(c) Both right to work and right to life (d) The Parliament
(d) Neither right to life nor right to work 11. The Supreme Court of India has declared
following services to come under the Con-
5. Who among the following officials in In-
sumer Protection Act, 1986 recently
dia appoints Regional Election Commis-
(a) Government Medical Hospitals
sioner
(b) Employee’s Provident Fund
(a) President in consultation with Elec-
(c) Government Colleges
tion Commission of India
(d) Procuring Bank Loan
(b) Prime Minister of India
12. The total number of Ministers including
(c) Election Commission of concerned
the Chief Minister, in the Council of Min-
State
isters in a State in India shall not exceed
(d) Governor of the concerned State in (a) 15 per cent of total members of Legis-
consultation with the Chief Minister lative Assembly
6. Who was the Chairman of the first Back- (b) 12 per cent of total members of Legis-
ward Class Committee? lative Assembly
(a) D. P. Mandal (c) per cent of total members of Legisla-
(b) K. R. Narayanan tive Assembly
(c) Kaka Kalelkar (d) 93.33 per cent of total members of
(d) None of the above Legislative Assembly
7. Which one of the following was member 13. In India an election candidate has to ex-
of Rajya Sabha at the time of appointment press in his nomination paper about his
as Prime Minister? (a) movable and immovable assets and
(a) P. V. Narsimha Rao liabilities to government and public
(b) Charan Singh financial institutions

CLAT-5-Mock Tests.indd 256 13/09/2010 10:36:23


MOCK TESTS 257

(b) about his liability only 24. In which famous case, the Supreme Court
(c) about his assets only of India said, “the President means, for
(d) None of the above all practical purposes, the Prime Minister
14. In Indian context which of the following or Council of Ministers and his opinion,
statements is correct? satisfaction or decision is constitution-
(a) Any citizen can be appointed as Prime ally secured when Minister arrives at such
Minister opinion, satisfaction or decision
(b) Only M.P. can be appointed as Prime (a) Ram Jawaya Kapur vs State of Punjab
Minister (b) Maneka Gandhi vs Union of India
(c) Only a member of the Lok Sabha can (c) M.S.M. Sharma vs Sri Krishna
become Prime Minister (d) None of the above
(d) Only leader, of the majority party in 25. The highest paid head of the Government
Lok Sabha can become Prime Minis- in the world at present is in
ter (a) Singapore (b) USA
15. Who administers oath of office to the (c) Russia (d) Japan
Chief Justice of India? 26. The members of the Rajya Sabha are
(a) President (b) Prime Minister (a) Mostly nominated
(c) Speaker (d) Vice-President (b) Elected indirectly
16. The first Indian Barrister (c) Elected by the members of the Leg-
(a) Madhusudan Das islative Assemblies and Legislative
(b) Chitranjan Das Councils of States
(c) Subhash Chandra Bose (d) Elected directly as well as indirectly
(d) Jnanendra Mohan Tagore 27. Which House is better placed with regard
17. Cruelty to a women by husband or relative
to control over the executive?
of husband is defined under
(a) Rajya Sabha
(a) Section 498A of Indian Penal Code
(b) Lok Sabha
(b) Section 498 of Indian Penal Code
(c) Both are equally placed
(c) Section 497 of Indian Penal Code
(d) It depends from which House the
(d) Section 496 of Indian Penal Code
Prime Minister comes
18. Voting right comes under
28. An ordinance can be issued by the Presi-
(a) Legal right
dent
(b) Fundamental right
(a) On any subject
(c) Constitutional right
(d) None of the above (b) At any time
19. ………. is a general pardon (c) Subject to the same constitutional lim-
(a) Remission (b) Reprieve itations as legislation by Parliament
(c) Amnesty (d) Suspension (d) In his individual judgement as well as
20. To betray a nation is an offence and pun- on the advice of the council of Minis-
ishable with death, that is ters
(a) Sedition (b) Treachery 29. An ‘office of profit’ which disqualifies a
(c) Treason (d) Anti-nationality person from being a member of the Union
21. Which amendment to Constitution of In- or State Legislature includes office held
dia relates to political defection? under
(a) 52nd (b) 53rd I. The Government of India
(c) 73rd (d) 90th II. A State Government
22. ‘A certiorari’ means III. A Local Authority
(a) at will (a) III only (b) I, II only
(b) with stronger reason (c) I, II, III (d) None of these
(c) of the same mind 30. The first Law Officer of the Government
(d) in the meanwhile of India is the
23. How many methods are prescribed for the (a) Law Minister
amendment of Indian Constitution? (b) Chief Justice of Supreme Court
(a) 3 (b) 4 (c) Attorney-General of India
(c) 5 (d) 7 (d) None of the above

CLAT-5-Mock Tests.indd 257 13/09/2010 10:36:23


258 MOCK TESTS

31. Child marriage in India means 37. Which of the following states/union terri-
(a) a marriage to which either of the con- tories has no legislative council?
tracting parties is a child (a) Bihar (b) Karnataka
(b) a marriage to which both of the con- (c) Uttar Pradesh (d) Tripura
tracting parties is a child 38. Which Article of the Indian Constitution
(c) is a marriage only when only bride is provides the Parliament, the power to
a child amend the Constitution?
(d) is a marriage only when groom is a (a) 390 (b) 368
child (c) 260 (d) 348
32. For the trial of a criminal case in India the 39. What is the power of the Rajya Sabha with
evidence is to be taken regard to Money Bill?
(a) In the presence of accused in the (a) It can withhold it
court (b) It has no power
(b) In the presence of his advocate in the (c) It can amend it
court (d) It can reject it
(c) In the presence of accused or when his 40. The position of the Vice-President of India
personal attendance is dispensed with resembles, to a great extent, the position
in the presence of his advocate of the Vice-President of
(d) In the presence of his guardian or near (a) Italy (b) Russia
relatives (c) New Zealand (d) USA
33. Securities and Exchange Board of India 41. In the context of Panchayati Raj in India
has been constituted by which among the following is true about
(a) Constitutional of India Gram Sabha?
(b) Securities and Exchange Board of In- (a) It is topmost tier of the Panchayati
dia Act Raj
(c) Special Commission by Central Gov- (b) It consists of all the voters resid-
ernment ing in the jurisdiction of a Village
(d) Bombay Development Act Panchayat
34. Which among the following is not a func- (c) It is the exclusive body consisting of
tion of Parliament of India? selected representatives from Village
(a) Providing the cabinet and holding Panchayat
them responsible (d) It consist of all the adult males of Vil-
(b) Criticizing government policy lage Panchayat
(c) Formulating policy for national devel- 42. In performance of his duties and in the ex-
opment ercise of his powers, the Government of a
(d) Security, relevant information on gov- State in India
ernment action (a) is answerable in the court of law
35. The member of Rajya Sabha in Parliament (b) is not answerable in the court of law
of India are (c) can be impeached by the Vidhan Sab-
(a) elected indirectly ha
(b) mostly nominated (d) is answerable to Vidhan Sabha
(c) elected directly as well as indirectly 43. The President of India has power to de-
(d) elected by the members of the legisla- clare emergency under Article 352 of
tive assemblies and legislative council Constitution of India on which of the fol-
of states lowing grounds
36. What is Zero hour? 1. War
(a) Interval between morning and eve- 2. External Aggression
ning session 3. Internal Disturbance
(b) When Money Bill is introduced in 4. Armed Rebellion
Lok Sabha Codes:
(c) When proposals of Opposition are (a) 1, 2 and 3 (b) 1, 2 and 4
considered (c) 1, 3 and 4 (d) 2, 3 and 4
(d) When matters of utmost importance 44. In the First War of India’s Independence,
are raised which revolutionary figure from Bareilly

CLAT-5-Mock Tests.indd 258 13/09/2010 10:36:23


MOCK TESTS 259

replaced the Mughal Prince, Mirza Mu- 54. Which among the following pronounces
ghal, as Commander-in-Chief of the rabel that, “all members (countries) shall settle
forces in July 1857? their international disputes by peaceful
(a) Abdul Rahman (b) Rhesus Zai resolutions in such a manner that the inter-
(c) Akbar Ali (d) Bakht Khan national peace and security and justice are
45. What is most common name of Paris Stock not endangered”.
Exchange (a) Article 2 (3) of Charter of United Na-
(a) Dalal (b) Bourse tions, 1945
(c) Par Stake (d) Bull and Bear (b) Article 4 (5) of the Covenant of
46. What is the qualification for obtaining vot- League of Nations, 1919
ing right in India? (c) Article 4 (2) of the Safety of United
(a) Caste (b) Property Nations and Associated Personnel,
(c) Age (d) Education 1946
47. “Equal pay for equal work for both males (d) None of the above
and females”. Which article has proposed 55. Which of the following Articles of the
this? Constitution of India deals with Caretaker
(a) Article 48 (b) Article 39 Government?
(c) Article 14 (d) Article 25 (a) 42 (b) 45
48. The first leader of opposition to be given (c) 350 (d) None of these
the status and rank of a Cabinet Minister 56. For how many times Financial Emergency
was has been imposed in India?
(a) A. K. Gopalan (b) Y. B. Chavan (a) Not at all (b) Once
(c) C. M. Stephen (d) P. Upendra (c) Twice (d) Thrice
49. The three languages included in the Eighth 57. The Speaker in Lok Sabha of Parliament
Schedule of the Constitution by the 71st of India can be removed from his office
amendment are before the expiry of his term
(a) Konkani, English, Manipuri (a) by the President on the recommenda-
(b) Konkani, Nepali, Maithili tion of the Prime Minister
(c) Konkani, Manipuri, Nepali (b) if the House passes a resolution to the
(d) Manipuri, Nepali, Dogri effect
50. Which of the following is not a condition (c) if the House and the Prime Minister so
for becoming a citizen of India?
decide
(a) Birth
(d) if both the Houses of Parliament pass
(b) Acquiring Property
a resolution
(c) Descent
58. The Ordinances issued by the Governor in
(d) Naturalization
India are subject to the approval by
51. Members of the Union Public Service
(a) Parliament (b) State Legislature
Commission of India hold office
(c) President (d) None of these
(a) for 6 years
59. The first Indian State to have its Human
(b) for 8 years
Development Report prepared and re-
(c) 6 years or up to the age of 65 years
leased by Amartya Sen in Delhi is
whichever is earlier
(a) Madhya Pradesh
(d) 5 years or up to the age of 60 years
whichever is earlier (b) Kerala
52. The Union Council of Ministers in India is (c) Andhra Pradesh
responsible to (d) West Bengal
(a) Lok Sabha (b) Parliament 60. The detailed provisions regarding acquisi-
(c) President (d) People tion and determination of Indian citizen-
53. To eliminate economic inequality and ship are contained in
concentration of wealth, Sarvodaya advo- (a) Part II of the Constitution of India
cates (b) Part VII of the Constitution of India
(a) non-possession (c) Citizenship Act, 1955 of India
(b) state run co-operatives (d) None of the above
(c) trusteeship 61. The status of a recognized Opposition par-
(d) social ownership of wealth ty in the Lok Sabha is secured only when

CLAT-5-Mock Tests.indd 259 13/09/2010 10:36:23


260 MOCK TESTS

the percentage of the total seats in the Lok (b) Prime Minister
Sabha won by the political party is (c) Deputy Prime Minister
(a) 5 (b) 8 (d) Election Commissioner of concerned
(c) 10 (d) 15 State
62. The members of the State PSC retire at 71. For the creation of a new state in India,
the the amendment of the Constitution will
(a) age of 60 years require
(b) age of 62 years (a) simple majority in Parliament
(c) age of 65 years (b) simple majority in Parliament and ap-
(d) No specific age proval of majority of states
63. The UPSC consists of a Chairman and (c) two-third majority in Parliament
(a) six other members (d) two-third majority in each House of
(b) eight other member Parliament and approval of majority
(c) eleven other members of states.
(d) unspecified number of members 72. Who among the following has the Consti-
64. In which part of the Constitution is the tutional authority to make rules regarding
State enjoined to establish Panchayati Raj the manner of enforcing the orders of the
institutions? Supreme Court?
(a) Directive Principles (a) Chief Justice of India
(b) Preamble (b) Attorney General of India
(c) Fundamental Rights (c) President
(d) Fundamental Rights (d) Union Law Minister
65. The maximum age prescribed for election 73. Madras High Court has the jurisdiction
as President is over
(a) 58 years (b) 60 years (a) Tamil Nadu and Kerala
(c) 62 years (d) No such limit (b) Tamil Nadu and Pondicherry
66. Principal laid down in a judgment is (c) Kerala and Lakshadweep
(a) Res judicata (d) Lakshadweep and Tamil Nadu
(b) Law 74. Which Article of the Constitution permits
(c) Ratio decidendi the Supreme Court to have a seat outside
(d) Obiter dicta Delhi?
67. Who was the member of Clement Atlee’s (a) 141 (b) 136
Cabinet to led the Cabinet Mission Plan to (c) 130 (d) 124
India in 1946? 75. The first state to become bifurcated after
(a) Lord Curzon independence was
(b) Lord Cornwallice (a) Bengal (b) Bombay
(c) Lord Pethick Lawrence (c) Punjab (d) Assam
(d) Lord William Bentinck 76. The Speaker of Lok Sabha in Parliament
68. Which among the following Articles of of India is elected/appointed by
Constitution of India speaks about Ordi- (a) President of India
nance to be proclaimed by President of (b) Prime Minister of India
India? (c) Members of Lok Sabha
(a) Article 123 (b) Article 223 (d) Both members of Lok Sabha and Ra-
(c) Article 233 (d) Article 243 jya Sabha
69. Who appoints the Officers for Linguistic 77. The President of India appoints the Judges
Minorities in India? of High Courts
(a) President (a) the statement is true
(b) Prime Minister (b) the statement is false
(c) Governor of State (c) President cannot appoint
(d) Both (a) and (c) (d) None of these
70. Who among the following appoints Re- 78. Most of the laws passed by the Parliament
gional Election Commissioners in India? of India apply to all States except
(a) President in consultation with Elec- (a) Andaman and Nicobar Islands
tion Commission of India (b) Pondicherry

CLAT-5-Mock Tests.indd 260 13/09/2010 10:36:24


MOCK TESTS 261

(c) Jammu and Kashmir (c) the Vice-President


(d) Mizoram (d) the Governor
79. As per constitutional requirement that the 88. A judgment which does not apply to all the
Parliament of India shall meet at least citizens but applies only to the Litigants is
(a) twice a year (b) once a year called
(c) thrice a year (d) four times a year (a) judgment in rem
80. Which of the following was the main ob- (b) judgment in personam
jective of Non-Co-operation Movement in (c) judgment per incurium
Indian freedom struggle? (d) None of the above
(a) Achievement of complete indepen- 89. Who has the final power as regard with-
dence of India drawing of a list for SCs, STs, and OBC in
(b) Attainment of dominion status for In- India?
dia (a) Parliament
(c) Introduction of bicameral system in (b) President
India (c) State Legislatures
(d) None of the above (d) None of the above
81. India adopted a federal system with a 90. Who is the present Chairman of the Na-
strong center from tional Human Rights Commission of In-
(a) New Zealand (b) Australia dia
(c) USA (d) Canada (a) Justice A. S. Anand
82. In India, power of judicial review is re- (b) Justice K. G. Balakrishanan
stricted because (c) Justice B. N. Kirpal
(a) the judges are transferable (d) Justice S. Rajendra Babu
(b) the Constitution is supreme 91. The Sikhs in India are permitted to carry
(c) the Executive is supreme Kirpan. Under which one the following
(d) the Legislature is supreme Fundamental Rights are they permitted to
83. Who of the following is considered as the do so?
custodian of Lok Sabha? (a) right to freedom
(a) The Prime Minister (b) right to freedom of religion
(b) The Speaker (c) right to life and liberty
(d) None of the above
(c) The Chief Whip of the ruling party
92. The Constitution prescribes the
(d) The leader of the Opposition
(a) punishment for the practice of un-
84. At present, India consists of
touchability
(a) 21 states and 11 union territories
(b) abolition of untouchability as a Fun-
(b) 28 states and 7 union territories
damental Right
(c) 25 states and 9 union territories
(c) way of detecting the practice of un-
(d) 24 states and 7 union territories
touchability
85. The Prime Minister is
(d) abolition of untouchability as a Direc-
(a) appointed by the President in consul-
tive Principle of State Policy
tation with the Speaker
93. Who among the following holds office
(b) appointed by the President
during the pleasure of the President?
(c) elected by the Lok Sabha (a) Speaker of the Lok Sabha
(d) elected by the two Houses of Parlia- (b) Prime Minister
ment at a joint sitting (c) Election Commissioner
86. The terms ‘Law in Force’ in clause (3) of (d) Governor
Article 13 of Constitution of India means 94. Which of the following is the correct
(a) only statutory law chronological order of the Prime Ministers
(b) customs and usages only in India?
(c) both (a) and (b) I. Indira Gandhi
(d) newly formed public order only II. Jawaharlal Nehru
87. The Constitution of India does not contain III. Morarji Desai
any provision for impeachment of IV. Charan Singh
(a) the President (a) I, II, III, IV (b) II, III, I, IV
(b) the Chief Justice of India (c) II, I, III, IV (d) III, II, IV, I

CLAT-5-Mock Tests.indd 261 13/09/2010 10:36:24


262 MOCK TESTS

95. What is the minimum age for appointment al of India is determined by


as a Governor? (a) the Council of Ministers
(a) 25 years (b) 30 years (b) the Constitution
(c) 35 years (d) 40 years (c) the Parliament
96. On the death of the President the Vice- (d) the President
President succeeds him as President for 99. The President of India can be removed
(a) a maximum period of three years from his office by the
(b) a maximum period of one year (a) Prime Minister
(c) a maximum period of six months (b) Lok Sabha
(d) the remaining period of the term (c) Chief Justice of India
97. The tenure of the Union Council of Minis- (d) Parliament
ters is 100. In addition to the powers given in the
(a) five years Union and Concurrent List, the Union
(b) uncertain Government has the
(c) fixed and co-terminus with the Presi- (a) Residuary power
dent (b) Federal power
(d) co-terminus with the Lok Sabha (c) Power to legislate in State list
98. The remuneration for the Attorney-Gener- (d) None of these

ANSWERS
1. (d) 2. (b) 3. (d) 4. (a) 5. (d) 6. (a) 7. (a) 8. (a) 9. (b) 10. (c)
11. (a) 12. (a) 13. (a) 14. (a) 15. (a) 16. (b) 17. (a) 18. (c) 19. (c) 20. (b)
21. (a) 22. (b) 23. (a) 24. (a) 25. (a) 26. (c) 27. (b) 28. (c) 29. (c) 30. (c)
31. (a) 32. (c) 33. (b) 34. (c) 35. (a) 36. (a) 37. (d) 38. (b) 39. (b) 40. (d)
41. (c) 42. (d) 43. (b) 44. (d) 45. (c) 46. (c) 47. (b) 48. (b) 49. (c) 50. (b)
51. (c) 52. (a) 53. (c) 54. (a) 55. (c) 56. (a) 57. (b) 58. (b) 59. (d) 60. (c)
61. (d) 62. (b) 63. (d) 64. (a) 65. (d) 66. (c) 67. (c) 68. (a) 69. (a) 70. (a)
71. (a) 72. (d) 73. (b) 74. (d) 75. (b) 76. (c) 77. (a) 78. (c) 79. (a) 80. (b)
81. (d) 82. (d) 83. (b) 84. (b) 85. (b) 86. (c) 87. (d) 88. (b) 89. (a) 90. (b)
91. (b) 92. (b) 93. (d) 94. (b) 95. (c) 96. (c) 97. (d) 98. (d) 99. (d) 100. (a)

CLAT-5-Mock Tests.indd 262 13/09/2010 10:36:24


MOCK TESTS 263

PRACTICE PAPER 3
Maximum Marks: 100 Marks 91–100—Outstanding
Time: 30 Minutes Marks 81–90—Excellent
Marks 71–80—Intelligent
Marks 61–70—Good
Marks 51–60—Average
Marks Less than 50—Below Average

1. Which Parliamentary Committee in In- (c) the President on the report of the Par-
dian system is chaired by the member of liament
opposition party? (d) the President on the report of the Su-
(a) Joint Parliamentary Committee preme Court of India
(b) Public Accounts Committee 8. The Parliament can make law on a subject
(c) Estimates Committee in the State List when
(d) Privileges Committee I. A Proclamation of Emergency is in
2. Who administers oath of office of a Gov- force
ernor of an Indian State? II. Two or more States make a request to
(a) Chief Justice of High Court the Parliament to make a law on a sub-
(b) President of India ject
(c) Chief Justice of Supreme Court III. Rajya Sabha passes a resolution that
(d) Advocates General of State such subjects have acquired national
3. WE THE PEOPLE OF INDIA, having importance
solemnly ………. to constitute India into (a) I, II (b) II and III
a SOVEREIGN SOCIALIST SECULAR (c) I and III (d) I, II, III
DEMOCRATIC REPUBLIC in the Pre- 9. A Money Bill can originate in the State
amble of Constitution of India. Legislature only with the prior consent of
(a) resolved the
(b) received I. Chief Minister II. President
(c) agreed III. Governor IV. None of these
(d) planned (a) I (b) I and III
4. A legislation enacted by a State Legisla- (c) II (d) III
ture in India falls in the category of 10. Generally, the first session of the Parlia-
(a) subordinate legislation ment starts with an address of the Presi-
(b) supreme legislation dent in which
(c) autonomous legislation (a) he makes suggestions to the Govern-
(d) delegated legislation ment regarding the policy it should
5. Right to work in India is under follow in the interest of the country
(a) Directive Principles (b) he outlines the policy and programme
(b) Fundamental Right of the government during the ensuing
(c) Constitutional Duty year
(d) Not a fundamental right but can be (c) he makes a survey of the achieve-
claimed after employment ments of the Government during the
6. Which one of the following is not a basic previous year
feature of the Indian constitution? (d) he does none of the above things
(a) Federal Government 11. Sovereignty in India under the Constitu-
(b) Parliamentary tion belongs to
(c) Independence of the judiciary (a) Parliament (b) the People
(d) Presidential Government (c) President (d) Supreme Court
7. The members of the UPSC can be removed 12. In India the International Treaties are rati-
from their office during their tenure by fied by
(a) the Parliament (a) President
(b) the President (b) Parliament

CLAT-5-Mock Tests.indd 263 13/09/2010 10:36:24


264 MOCK TESTS

(c) Prime Minister (b) Parliament


(d) Foreign Secretary (c) The Prime Minister
13. Which of the following rights have been (d) None of these
described in the Constitution of India? 20. What is the power of the Rajya Sabha with
(a) Natural rights of the citizens regard to Money Bill?
(b) Fundamental rights of the citizens (a) It can withhold it.
(c) Moral rights of the citizens (b) It has no power.
(d) Hereditary rights of the citizens (c) It can amend it.
14. Who is responsible for the delimitation of (d) It can reject it.
territorial constituencies for Lok Sabha 21. Citizenship in India can be lost by all of
elections in India? the following except
(a) The Delimitation Commission (a) renunciation
(b) State Legislature (b) treason
(c) Regional Election Commissioner (c) termination
(d) State Law Ministry (d) deprivation
15. Parliament of India consists of 22. Indians have
(a) Council of State, the House of People (a) single citizenship
and the President (b) dual citizenship (One of Union and
(b) Council of State and President Second of States)
(c) House of People and President (c) circumstantial citizenship
(d) Prime Minister and President (d) None of the above
16. The Judges of the High Court can be re- 23. Which among the following Union Terri-
moved from their office during their ten- tories of India have High Courts of their
ure own?
(a) by the Chief Justice on the advice of (a) Delhi
the President (b) Delhi and Daman Diu
(b) by the Chief Justice on the recommen- (c) Pondicherry and Nagar Haveli
dation of the Parliament (d) None of these
(c) by the Governor, if the State Legisla- 24. What is the tenure of Governor of a State
ture passes a resolution to this effect in India?
(a) No fixed term
by two-thirds majority
(b) 5 years
(d) by the President on the basis of a reso-
(c) 4 years
lution passed by the Parliament by
(d) At the discretion of State Law Secre-
two-thirds majority
tary
17. A judicial Review means:
25. Who among the following possesses the
(a) Powers of the courts to declare null
authority to set common High Courts for
and void any action by any organ of
States in India
the Government if it is beyond the
(a) Parliament
powers granted by the Constitution
(b) Chief Justice of Supreme Court
(b) Review of the working of judiciary
(c) Chief Justice of either High Court
(c) Power of the Court to try cases
(d) Constitutional Bench of Supreme
(d) Review of the judicial power of the Court
judges 26. Who conducts the State assembly elec-
18. When the Vice-President acts as President tions?
he gets the emoluments of (a) Governor of the state concerned
(a) Vice-President (b) Chief Justice of the Supreme Court
(b) President (c) Chief Justice of the High court con-
(c) President in addition to what he gets cerned
as Chairman of Rajya Sabha (d) Chief Election Commission
(d) Chairman of Rajya Sabha 27. Rajya Sabha is less powerful than Lok
19. The executive power is vested in the Pres- Sabha in terms of its financial powers be-
ident but it is actually used by him on the cause
advice of (a) no Money Bill can be introduced in
(a) The Council of Ministers Rajya Sabha

CLAT-5-Mock Tests.indd 264 13/09/2010 10:36:24


MOCK TESTS 265

(b) it does not participate in the passing of 35. A Money Bill in Indian Parliament
Money Bill (a) cannot be introduced in the Council of
(c) Both of the above States
(d) None of the above (b) needs to be certified as such by the
28. The Tenth Schedule of the Constitution of Speaker
India relates to (c) can be amended by the Council of
(a) Administration of scheduled and trib- States
al areas (d) Both (a) and (b)
(b) National languages of India 36. Rajya Sabha is also called as….
(c) Anti-defection law (a) Council of Nominated Members
(d) Judicial review (b) Council of Representatives
29. The Constitution (c) Council of States
(a) Allows re-election of a person to the (d) None of the above
President’s post 37. National Development Council is a …
(b) Restricts a person to remain President (a) Creation of the executive body
for only two terms (b) Constitutional body
(c) Has been amended to allow a person (c) Cabinet’s Secretariat
only one term as President (d) Joint Right
(d) Is silent on the President’s re-election 38. Which article has abolished the child la-
to the office bour?
30. The States reorganization in 1956 created (a) 24 (b) 27
(a) 14 States and 6 Union Territories (c) 29 (d) 34
(b) 15 States and 9 Union Territories 39. Welfare State is the aim of
(a) Anarchism
(c) 17 States and 6 Union Territories
(b) Individualism
(d) 17 States and 9 Union Territories
(c) Scientific Socialism
31. The relationship between the President
(d) Democratic Socialism
and Prime Minister in India is determined
40. The first Law Officer of the Government
by
of India is
I. Articles 74-78 of the Constitution
(a) Chief Justice of India
II. Conventions
(b) Law Minister
III. Judicial pronouncements
(c) Attorney General
IV. Personal relations
(d) Auditor General
(a) I, II (b) II, III 41. The official language of the Indian Union
(c) III, IV (d) I, IV is
32. Which of the following Articles of the (a) Hindi
Constitution of India specially guarantees (b) English
freedom of the Press? (c) English and Hindi
(a) 16 (b) 19 (d) Urdu, Hindi and English
(c) 22 (d) None of these 42. The act of adopting a foreigner and mak-
33. In a Parliamentary democracy ing him a citizen of the country for all pur-
(a) executive controls the Legislature poses through a legal process is called
(b) executive and Legislature are strictly (a) dual citizenship
separate (b) PIO (Persons of Indian Origin) Status
(c) the Judiciary controls both Legislature (c) naturalization
and Executive (d) right of re-entry
(d) the Legislature controls the Executive 43. In India context which of these statements
34. The Parliament of India consists of is not correct?
(a) Lok Sabha and Rajya Sabha (a) The President can dissolve the Parlia-
(b) President, Lok Sabha and Rajya Sab- ment
ha (b) The President can summon the Parlia-
(c) President, Ministers and the Two ment
Houses (c) The President can dissolve the Lok
(d) None of these Sabha

CLAT-5-Mock Tests.indd 265 13/09/2010 10:36:24


266 MOCK TESTS

(d) The President can prorogue the Parlia- 52. The Supreme Court of India
ment (a) is a court of record and has a power to
44. In India and Ordinance issued by the Pres- punish for its contempt
ident or a Governor can remain in force (b) is the only highest court of appeal
without the approval of Parliament or (c) is like House of Lords on its judicial
State Assembly for a maximum period of side
(a) 6 months (b) 3 months (d) as in all respects like Supreme Court
(c) 2 months (d) 4 months of USA
45. Which of the following statements relat- 53. In 1740, which agency of the Bombay
ing to the Rajya Sabha in Parliament of Government was setup to enforce control
India is not correct? on trade on the Konkan Coast?
(a) It can be dissolved by the President (a) Bombay-Konkan Empire
after a term of 5 years (b) Oudh-Bombay Ghat
(b) It is a permanent body and 1/3rd of its (c) Bombay-Oudh Rule
members retire every 2 years (d) Bombay Marines
(c) Its members are elected by the State 54. The area of Lok Sabha for the purpose of
Assemblies by the method of propor- General Election in India is determined by
tional representation the
(d) It has no control over Money Bills ex- (a) Delimitation Commission
cept to delay them for 14 days (b) Election Commission
46. Indira Swahney vs Union of India is a case (c) Census Commission
popularly known as (d) President
(a) Mandal judgment 55. Term of Lok Sabha in Parliament of India
(b) Ayodhya judgment is
(c) Suicidal judgment (a) five years unless it is dissolved
(d) Election Commission judgment (b) four years from the date of its first ses-
47. Under the Constitution, the residuary sion
powers vest with the (c) five years from the date of oath taken
(a) President by the members
(b) Supreme Court (d) it is a permanent body
(c) Union government 56. From which country’s Constitution the Ju-
(d) State Government
dicial Review have been borrowed?
48. Which organ of the Constitution is known
(a) Irish (b) Canada
as the ‘Fifth wheel of the Coach’?
(c) British (d) US
(a) Planning Commission
57. Which article defines Equality before law
(b) Finance Commission
and equal protection of law?
(c) Public Service Commission
(a) Article 16 (b) Article 14
(d) None of the above
(c) Article 20 (d) Article 21
49. By which constitutional amendment
58. Which articles defines Protection of life
Panchayati Raj was introduced?
and Personal liberty?
(a) 74th (b) 73rd
(a) Article 16 (b) Article 14
(c) 42nd (d) 44th
(c) Article 20 (d) Article 21
50. From which country’s Constitution the
Right to Equality have been borrowed? 59. Which article defines Judicial Review?
(a) Irish (b) Canada (a) Article 16 (b) Article 13
(c) British (d) US (c) Article 20 (d) Article 21
51. Constitution Amending Bill passed by 60. Which article defines Protection in respect
the Parliament of India also requires to be of conviction for offences?
ratified by the Legislature of not less than (a) Article 16 (b) Article 14
half of the States in regard to the (c) Article 20 (d) Article 21
(a) Fundamental Duties 61. Members of Rajya Sabha of India are not
(b) Fundamental Rights associated with
(c) Representation of States in Parlia- 1. Public Accounts Committee
ment 2. Estimates Committee
(d) Constitutional Remedies 3. Committee on Public Undertaking

CLAT-5-Mock Tests.indd 266 13/09/2010 10:36:24


MOCK TESTS 267

(a) 1 and 2 (b) 3 only 71. Generally a judgment of a court consists


(c) 2 only (d) 1 and 3 of certain observations which lay down
62. Which House is better placed with regard certain principles of law. Such observa-
to control over executive in India? tions are called __________
(a) Lok Sabha (a) obiter dicta (b) stare decisis
(b) Rajya Sabha (c) ratio decidendi (d) None of these
(c) Both are equally placed 72. Which of the following is the correct state-
(d) It depends from which House the ment of law?
Prime Minister comes (a) Judges make the law
63. Who among the following is considered (b) Judges do not make the law
as the Guardian of Public Purse in India? (c) Legislature makes the law and the
(a) Parliament Judges interpret them
(b) Comptroller and Auditor-General (d) Legislature makes the law and the
(c) Public Accounts Committee Judges interpret them but the interpre-
(d) the President tation also becomes law
64. A Money Bill in India 73. If there is a conflict between the Constitu-
(a) cannot be introduced in the Council of tion and a State Law in India
States (a) the State Law will prevail
(b) needs to be certified as such by the (b) the Constitution will prevail
President (c) the Constitution will prevail and the
(c) can be amended by the Council of State Law would be called ultra vires
States (d) None of these
(d) Both (a) and (b) 74. Can the lower court in India decide on
65. The Deputy Chairman of the Rajya Saba election issue?
of Parliament of India is (a) No, only the High Court and Supreme
(a) nominated by the Chairman Court
(b) elected by the Parliament (b) Yes
(c) nominated by the President (c) Yes, only the High Court
(d) elected by the Rajya Sabha from (d) Yes, only the Supreme Court
amongst its members 75. Who has the power to create Inter-State
66. Which article defines Prohibition of dis- Councils in India?
crimination only on grounds of religion, (a) President (b) Parliament
race, caste, sex or place of birth? (c) Governor (d) Home Minister
(a) Article 16 (b) Article 14 76. Protection of the interests of the minority
(c) Article 15 (d) Article 21 is envisaged in which of the following ar-
67. What are the articles which defines Direc- ticles?
tive Principles of State Policy? (a) 14 (b) 19
(a) 36 to 51 (b) 36 to 51-A (c) 29 (d) 32
(c) 39 to 51 (d) 39 to 51-A (e) None of these
68. Which article says for Uniform Civil 77. Who appoints the Chairman of UPSC of
Code? India?
(a) Article 43 (b) Article 40 (a) President
(c) Article 44 (d) Article 45 (b) Vice-President
69. Which part of Constitution defines Funda- (c) Prime Minister
mental Duties? (d) Chief Justice of India
(a) Part I (b) Part II 78. Under which article President can declare
(c) Part IV (d) Part IV-A National Emergency?
70. The Preamble of our Constitution includes (a) Article 352 (b) Article 360
all except (c) Article 350 (d) Article 356
(a) Adult franchise 79. Under which article President can declare
(b) Equality of status State Emergency?
(c) Fraternity (a) Article 352 (b) Article 360
(d) Justice (c) Article 350 (d) Article 356

CLAT-5-Mock Tests.indd 267 13/09/2010 10:36:24


268 MOCK TESTS

80. Under which article President can declare 91. Which one of the following terms is not
Financial Emergency? the main ingredient of the Preamble of the
(a) Article 352 (b) Article 360 Constitution of India?
(c) Article 350 (d) Article 356 (a) Justice (b) Liberty
81. Who has the power to legislate on the mat- (c) Equality (d) Comity
ters in the “Concurrent List” of the Consti- 92. The Constitution of India is the
tution of India? (a) most readable document in the world
(a) Both Parliament and State Legisla- (b) most largest constitutional document
tures in the world
(b) Parliament only (c) most shortest document of the world
(c) State Legislature only (d) most idealistic document of the world
(d) President of India only 93. The Preamble of the Constitution of India
82. What is the number of Judges in the Su- grants three kinds of justice to its citizens.
preme Court? Which one of the following is not included
(a) 14 (b) 31 in these three kinds of Justice?
(c) 13 (d) 11 (a) Social (b) Economic
83. What is the term of the Lok Sabha in Par- (c) Cultural (d) Political
liament of India? 94. Which Part of the Constitution of India
(a) 5 years (b) 6 years contains the State of establish Panchayati
(c) 4 years (d) 7 years Raj Institutions?
84. What is the maximum strength of the Ra- (a) Preamble
jya Sabha? (b) Fundamental Rights
(a) 250 Members (b) 238 Members (c) Directive Principles of State Policy
(c) 225 Members (d) 552 Members (d) Ninth Schedule
85. A legal authorization to a debtor to post- 95. The Constituent Assembly has a Drafting
pone payment for a certain time is called Committee whose Chairman was
(a) moratorium (b) debt facility (a) Dr Rajendra Prasad
(c) barring of debt (d) Probate (b) Jawaharlal Nehru
86. How many times National Emergency has (c) Dr B. R. Ambedkar
been declared in our country? (d) K. M. Munshi
(a) Once 96. Which Chief Justice of India have per-
(b) Thrice formed the duties of President?
(c) More than 100 times
(a) Justice V. N. Khare
(d) Not even once
(b) Justice M. M. Punchi
87. How many times Financial Emergency
(c) Justice S. P. Barucha
has been declared in our country?
(d) Justice M. Hidaytullah
(a) Once
97. Which President of India was elected un-
(b) Thrice
opposed?
(c) More than 100 times
(a) N. S. Reddy
(d) Not even once
(b) K. R. Narayanan
88. Which article describes about the im-
(c) R. Venkataraman
peachment of President?
(d) S. D. Sharma
(a) Article 53 (b) Article 52
98. What is the salary of Vice-President of In-
(c) Article 60 (d) Article 61
89. If both the President and Vice-President dia?
are not available who performs the duties (a) 45,000 (b) 30,000
of the President? (c) 50,000 (d) 1,10,000
(a) Chief Justice of India 99. Which article says that there should be a
(b) Supreme Court Judge Vice-President?
(c) Prime Minister (a) Article 63 (b) Article 62
(d) Governor (c) Article 60 (d) Article 61
90. How many times Chief Justice of India 100. Which article says that there should be a
have performed the duties of President? Governor?
(a) Once (b) Twice (a) Article 163 (b) Article 153
(c) Thrice (d) Not even once (c) Article 160 (d) Article 161

CLAT-5-Mock Tests.indd 268 13/09/2010 10:36:24


MOCK TESTS 269

ANSWERS
1. (b) 2. (a) 3. (a) 4. (a) 5. (d) 6. (d). 7. (d). 8. (d). 9. (d). 10. (b)
11. (b) 12. (a) 13. (b) 14. (a) 15. (a) 16. (d). 17. (a) 18. (b) 19. (a) 20. (b)
21. (b) 22. (a) 23. (a) 24. (b) 25. (a) 26. (d). 27. (a) 28. (c) 29. (a) 30. (a)
31. (a) 32. (b) 33. (d). 34. (b) 35. (d) 36. (c) 37. (b) 38. (a) 39. (d). 40. (c)
41. (b) 42. (c) 43. (a) 44. (a) 45. (a) 46. (a) 47. (c) 48. (a) 49. (b) 50. (c)
51. (c) 52. (a) 53. (d). 54. (a) 55. (c) 56. (d). 57. (b) 58. (d). 59. (b) 60. (c)
61. (d). 62. (a) 63. (b) 64. (a) 65. (d) 66. (c) 67. (a) 68. (c) 69. (d). 70. (a)
71. (a) 72. (d). 73. (c) 74. (a) 75. (b) 76. (c) 77. (a) 78. (a) 79. (b) 80. (b)
81. (a) 82. (b) 83. (a) 84. (a) 85. (d) 86. (b) 87. (d). 88. (d). 89. (a) 90. (a)
91. (d). 92. (b) 93. (c) 94. (c) 95. (c) 96. (d). 97. (a) 98. (d). 99. (a) 100. (b)

CLAT-5-Mock Tests.indd 269 13/09/2010 10:36:25


270 MOCK TESTS

PRACTICE PAPER 4
Maximum Marks: 100 Marks 91–100—Outstanding
Time: 30 Minutes Marks 81–90—Excellent
Marks 71–80—Intelligent
Marks 61–70—Good
Marks 51–60—Average
Marks Less than 50—Below Average

1. Under which circumstances can a court 6. What is the salary of Governor of a State?
issue a writ in India? (a) ` 45,000 (b) ` 30,000
(a) Violation of Fundamental Duties (c) ` 36,000 (d) ` 80,000
(b) Defective procedures in case of Direc- 7. What is the quorum to constitute a meet-
tive Principles of State Policy ing of either House of the Parliament?
(c) Violation of Fundamental Right (a) One tenth of the total members of the
(d) None of the above House
2. Which statement is most correct in repre- (b) One eighth of the total members of the
senting the legal structure of India? House
(a) Supreme Court, High Court, Subordi- (c) One fourth of the total members of the
nate Courts House
(b) Supreme Court, High Court, Court of (d) One sixth of the total members of the
District Judge, Court of Additional House
District Judge 8. The Constitution of India was enacted by
(c) Supreme Court, High Court, Court of a Constituent assembly set up in
Session, Court of Judicial Magistrate (a) July, 1948 (b) July, 1950
Class I
(c) July, 1946 (d) August, 1947
(d) Supreme Court, High Court, Court
9. Who among the following was the Chair-
of District Judge, Court of Session,
man of the Drafting Committee of the In-
Court of Additional District Judge,
dian Constitution
Court of Judicial Magistrate Class I
(a) Rajendra Prasad
3. A law made by Parliament of India having
(b) Tej Bahadur Sapru
extraterritorial operation
(a) shall not be deemed invalid (c) C. Rajagopalachari
(b) shall be deemed invalid (d) B. R. Ambedkar
(c) shall be deemed ultravires 10. The Constituent assembly which framed
(d) shall be deemed constitutional the Constitution for Independent India
4. The rule of passing resolution by 2/3rd was setup in
majority of total number of members of (a) 1945 (b) 1946
the House of Parliament of India is appli- (c) 1947 (d) 1949
cable in case of 11. Which of the Fundamental Right under
(a) amendment of the Constitution Constitution of India was considered to be
(b) approval of proclamation of emergen- the heart and soul of the Constitution of
cy India (Dr Ambedkar view)?
(c) impeachment of President (a) Right to Property
(d) disapproval by proclamation of emer- (b) Right to Constitutional Remedies
gency (c) Right to Life
5. Prorogation of the House in Parliament of (d) Right to Information
India means 12. Chairman of Central Sixth Pay Commis-
(a) a House has been brought in session sion in India was
(b) the session of the House has been ter- (a) Justice B. N. Srikrishna
minated (b) Justice R. N. Mishra
(c) the House itself stands terminated (c) Justice L. K. Majumdar
(d) None of the above (d) Justice V. N. Khare

CLAT-5-Mock Tests.indd 270 13/09/2010 10:36:25


MOCK TESTS 271

13. In which of the following cases the Con- 21. Principal laid down in a judgment is
stitution of India allows any exception to (a) Res judicata
the rule of equality? (b) Law
(a) President and Governor (c) Ratio decidendi
(b) Prime Minister and Chief Minister (d) Obiter dicta
(c) Union Law Minister and Law Minis- 22. Who is associated with Drafting of Indian
ter of a State Penal Code
(d) Deputy Prime Minister and Deputy (a) Ambedkar
Chief Minister (b) Sir Woodroof and Amirali
14. Which among the following can impose (c) Lord Macaulay
reasonable restrictions on the Fundamen- (d) Mulla
tal Rights of Indian Citizens? 23. Culpable homicide is defined in the Indian
(a) only Parliament Penal Code, Section
(a) 299 (b) 300
(b) only Supreme Court
(c) 301 (d) 307
(c) both Supreme Court and Parliament
24. Murder is defined in the Indian Penal
(d) only High Court
Code, Section
15. The writs of Prohibition and Certiorari are
(a) 299 (b) 300
available against
(c) 301 (d) 307
(a) Judicial or quasi-judicial authorities 25. In India an aggrieved public servant of the
(b) Legal and semi-legal authorities Union can challenge the decisions of the
(c) Implies authorities Administrative Tribunal
(d) Non-statutory authorities (a) before the Supreme Court
16. How long did the Constituent Assembly (b) before the High Court
take to finally pass the constitution? (c) before both (a) and (b)
(a) About 6 months in 1949 (d) before no court of law
(b) Exactly a year since 26 November 26. Who among the following is known as the
1948 Father of the Indian Constitution?
(c) About 2 years since 15 August 1947 (a) Dr B. R. Ambedkar
(d) About 3 years since 9 December 1946 (b) Mahatma Gandhi
17. Who was the Chairman of the Constituent (c) Jawaharlal Nehru
Assembly? (d) Vallabhbhai Patel
(a) Pt Jawaharlal Nehru 27. Who proposed the Preamble before the
(b) Dr Rajendra Prasad Drafting Committee of the Constitution?
(c) Dr B. R. Ambedkar (a) Jawaharlal Nehru
(d) C. Rajagopalachari (b) B. R. Ambedkar
18. India became a Sovereign, democratic re- (c) B. N. Rau
public on (d) Mahatma Gandhi
(a) 15 August 1947 28. When did the first linguistic state of
(b) 30 January 1948 Andhra come into existence?
(c) 26 January 1950 (a) 2 October 1953
(d) 26 November 1929 (b) 1 October 1953
19. The Constitution names our country as (c) 1 April 1953
(a) Bharat (d) 5 January 1953
29. Who was the first speaker of the Lok Sab-
(b) India, that is Bharat
ha?
(c) Hindustan
(a) Malgaonkar
(d) Aryavarta
(b) P. Upendra
20. A constitution is
(c) Anantha Sayanam Ayyanagar
(a) a set of ordinary laws
(d) Hukam Singh
(b) a set of official laws 30. The state of Bombay was bifurcated into
(c) a set of financial laws Maharashtra and Gujarat on 1 May in the
(d) the basic structure defining the powers year
of the state and the rights and duties of (a) 1958 (b) 1959
the citizens (c) 1962 (d) 1960

CLAT-5-Mock Tests.indd 271 13/09/2010 10:36:25


272 MOCK TESTS

31. In Indian context which of the following to avoid legal proceed-


statements is correct? ings.
(a) Union Territories are not represented B. Abortion 2. The termination of
in the Rajya Sabha pregnancy, a miscar-
(b) It is within the purview of the Chief riage or the premature
Election Commissioner to adjudicate expulsion of a foetus
in Election disputes from the womb before
(c) The Speaker of a Legislature contin- the normal period of
ues in office even after the House is gestation is complete.
dissolved and until the Speaker of the C. Absconding 3. Wrongfully taking
subsequent Legislature is elected away or detaining an-
(d) According to the Constitution of India other person, usually
Parliament consist of the Lok Sabha by force or fraud.
and the Rajya Sabha only A B C
32. The election in India for Lok Sabha and (a) 3 2 1
State Assemblies is made on the basis of (b) 1 3 2
(a) Secret Ballot System (c) 1 2 3
(b) Open Ballot System 39. A. Accomplice 1. One who is a party to a
(c) In case of Lok Sabha Secret Ballot crime, either as a prin-
System and in case of State Assem- cipal or as an acces-
blies Open Ballot System sory
(d) In case of Lok Sabha Open Ballot B. Acquittal 2. A decision by a court
System and in case of State Assem- that a defendant ac-
blies the Secret Ballot System cused of a crime is not
33. Power to punish for its contempt makes a guilty
High Court in India C. Act of God 3. An event due to natural
(a) a court of record causes (storms, earth-
(b) a court of contempt quakes, floods, etc.) so
(c) a court of appellant jurisdiction exceptionally severe
(d) a court of original jurisdiction that no one could rea-
34. For the purpose of three-tier Panchayat in sonably be expected
an Indian State, it must have a population to anticipate or guard
above against it.
(a) 20 Lakhs (b) 50 Lakhs A B C
(c) 75 Lakhs (d) 1 Crore (a) 2 1 3
35. For the purpose of Panchayat election in (b) 3 1 2
India, a candidate has to attain the age of (c) 1 2 3
(a) 21 years (b) 25 years 40. A. Adjournment 1. The postponement
(c) 30 years (d) 35 years or suspensions of the
36. Which of the following Union Territories hearing of a case until
attained statehood in February, 1987? a future date
(a) Goa B. Adjudication 2. The formal judge-
(b) Arunachal Pradesh ment or decision of a
(c) Pondicherry court or tribunal
(d) Daman and Diu C. Admissibility 3. The principles of
37. The 25th Indian state to achieve statehood evidence determining
is whether or not partic-
(a) Goa ular items of evidence
(b) Arunachal Pradesh may be received by
(c) Mizoram the court. The central
(d) Sikkim principle of admis-
38. A. Abduction 1. The failure of a person sibility is relevance.
to surrender to the cus- All evidence that is
tody of a court in order sufficiently relevant

CLAT-5-Mock Tests.indd 272 13/09/2010 10:36:25


MOCK TESTS 273

is admissible and all 47. A. Alienation 1. The transfer of prop-


that is not sufficiently erty
relevant is inadmis- B. Alimony 2. Written statement of
sible. allegation
A B C C. Allegation 3. Maintenance charges
(a) 2 1 3 to wife from husband
(b) 1 2 3 A B C
(c) 2 3 1 (a) 1 3 2
41. Who appoints the Comptroller and Audi- (b) 2 1 3
tor General of India? (c) 3 2 1
(a) President 48. A. Amnesty 1. Political pardon
(b) Prime Minister B. Animus 2. Intention
(c) Elected by Lok Sabha C. Antecedents 3. Previous character
(d) Elected by Lok Sabha and Rajya Sab- A B C
ha (a) 1 2 3
42. Who appoints the judge of a High Court in (b) 2 1 3
India? (c) 3 2 1
(a) Governor 49. A. A posteriori 1. From effect to cause
(b) Chief Minister B. A priori 2. From cause to effect
(c) President C. Arbitration 3. Mediation
(d) Chief Justice of India A B C
43. Who appoints the Attorney General of In- (a) 1 2 3
dia? (b) 2 1 3
(a) President
(c) 3 2 1
(b) Prime Minister
50. A. Bail 1. Luggage at the railway
(c) Chief Justice of India
station
(d) Elected by the Parliament
B. Bailiff 2. Court’s officer
44. A person who holds or who has held office
C. Bailment 3. Release from police
as President of India shall
custody
(a) not be eligible for re-election
A B C
(b) be eligible for re-election as President
(a) 1 3 2
(c) be eligible for re-election as President
(b) 2 1 3
after the gap of one term
(d) be eligible for re-election as President (c) 3 2 1
after 10 years of vacating the office 51. Time limit for filing different kinds of suit
45. Who appoints members of a State Public is prescribed in
Service Commission in India? (a) CPC
(a) President (b) IPC
(b) Governor (c) Limitation Act
(c) Prime Minister (d) High Court Rules
(d) Chief Minister 52. Members of Lok Sabha of Parliament of
46. A. Admonition 1. Offence of using or India are elected by the way of
threatening unlawful (a) People’s Representation
violence. (b) by the State Legislatures
B. Affidavit 2. A solemn declaration (c) nominations
true to the best of the (d) by electoral colleges
knowledge of the de- 53. Which one of the following has been de-
ponent scribed as the soul of the India Constitu-
C. Affray 3. A reprimand from a tion?
judge to a defendant (a) The Preamble
A B C (b) The Chapter on Fundamental Rights
(a) 2 3 1 (c) The Chapter on Directive Principles
(b) 1 2 3 (d) The provisions regarding Judicial Re-
(c) 3 2 1 view

CLAT-5-Mock Tests.indd 273 13/09/2010 10:36:25


274 MOCK TESTS

54. The Constitution of India is A B C


(a) ledger (b) notebook (a) 3 2 1
(c) book (d) document (b) 2 1 3
55. The main objective of the Fundamental (c) 1 2 3
Rights under Indian Constitution is to 61. A person whose guardian is appointed
(a) ensure individual liberty during his minority, becomes a major at
(b) promote a socialism pattern of soci- the age of ____ years
ety (a) 31 (b) 21
(c) to promote equality (c) 28 (d) 18
(d) All the above 62. When a person to be prosecuted for com-
56. A. Bankruptcy 1. Insolvency mitting a criminal offence the burden of
B. Battery 2. Actual use of force proof is on
C. Beneficiary 3. One who benefits from (a) Court (b) Police
will
(c) Himself (d) Prosecution
A B C
63. “Courts and their Judgments: Promises,
(a) 1 2 3
Requisites and Consequences” is a book
(b) 2 1 3
written by
(c) 3 2 1
(a) Vikram Seth
57. A. Bigamy 1. Second marriage when
first marriage is still (b) Ram Jeth Malani
subsisting (c) Arun Shourie
B. Capacity 2. Competence to enter (d) None
to contract into a legal contract 64. Judges for International Court of Justice is
C. Capital 3. Death punishment elected for……years
punishment (a) 5 (b) 10
A B C (c) 9 (d) 6
(a) 3 2 1 65. What is the tenure of the President of In-
(b) 1 2 3 ternational Court
(c) 3 1 2 (a) 5 years (b) 2 years
58. A. Condominium 1. Joint sovereignty (c) 3 years (d) 6 years
B. Conjugal rights 2. Matrimonial rights 66. A. Extradition 1. Red handed
C. Consent 3. Agreement by B. Fiduciary 2. Based on mutual inter-
choice est and faith
A B C C. Flagrante 3. To extradite a criminal
(a) 3 2 1 delicto
(b) 2 1 3 A B C
(c) 1 2 3 (a) 3 2 1
59. A. Discharge of 1. The termination of (b) 2 1 3
contract contractual obliga- (c) 3 1 2
tion 67. A. Genocide 1. Massive killing
B. Duress 2. Pressure/undue influ- B. Hijacking 2. Seizing an aircraft
ence C. Homicide 3. Killing of human be-
C. Embargo 3. The detention of ing
ships in a port: a type A B C
of reprisal.
(a) 3 2 1
A B C
(b) 1 2 3
(a) 3 2 1
(c) 3 1 2
(b) 1 2 3
68. A. Hostage 1. To indict
(c) 3 1 2
60. A. Embezzle- 1. Criminal B. Hostile 2. Adverse witness
ment misappropriation of witness
public property C. Incriminate 3. One held as a security
B. Ex gratia 2. Done as a matter of fa- A B C
vour (a) 3 2 1
C. Ex officio 3. By virtue of holding an (b) 2 1 3
office (c) 3 1 2

CLAT-5-Mock Tests.indd 274 13/09/2010 10:36:25


MOCK TESTS 275

69. A. Innenudo 1. Interim 76. A. Mitigation 1. Reduction in the sever-


B. Insanity 2. Indirect ity of some penalty
C. Interlocu- 3. Mental disorder B. Moot 2. A mock trial
tory C. Mortgage 3. An interest in the prop-
A B C erty created as a form
(a) 1 2 3 of security for a loan.
(b) 3 2 1 A B C
(c) 2 3 1 (a) 1 2 3
70. A. Mala fide 1. Fraudulent or dishon- (b) 2 1 3
est act (c) 2 3 1
B. Mens rea 2. Hostile attitude 77. A. Motive 1. Cheque
C. Malice 3. Guilty mind B. Negligence 2. Gross carelessness
A B C C. Negotiable 3. Purpose behind instru-
(a) 1 3 2 ment action
(b) 3 2 1 A B C
(c) 2 1 3 (a) 1 2 3
71. Does Indian Constitution allow any ex- (b) 3 2 1
ception to the rule of equality? (c) 2 1 3
(a) Yes, in the case of the President and 78. A. Nullity of 1. False evidence
Governors marriage
(b) Yes, in the case of Prime Minister and B. Parole 2. Conditional release
Chief Ministers C. Perjury 3. Invalid marriage
(c) Yes, in the case of Prime Minister and A B C
(a) 1 2 3
Deputy Prime Minister
(b) 3 2 1
(d) Yes, in case of Home Minister of
(c) 2 1 3
Union and States
79. A. Provocation 1. Conduct or words
72. Which among these is not included in the
causing someone to
Constitution of India?
loose his self control
(a) Social, economic and political justice
B. Putative 2. A man alleged to be
(b) Liberty to thought, expression, belief,
father the father of an illegiti-
faith and worship
mate child.
(c) Equality of status and of opportunity
C. Quasi 3. A function that resem-
(d) Right to work judicial bles the judicial func-
73. Which of these is not provided in the Di- tion
rective Principles of State Policy in Con- A B C
stitution of India? (a) 1 2 3
(a) Prohibition of employment of chil- (b) 3 2 1
dren below the age of 14 (c) 3 1 2
(b) Fair distribution of wealth 80. A. Restitution 1. Damage or destruction
(c) The living wage for workers of property
(d) Uniform civil code of the citizens B. Revocation 2. withdrawal offer
74. Which of the following is not a Funda- of offer
mental Right under Constitution of India? C. Sabotage 3. return/restoration
(a) Right to equality A B C
(b) Right to freedom (a) 1 2 3
(c) Right to property (b) 3 2 1
(d) Right against exploitation (c) 3 1 2
75. Under what circumstances can the Funda- 81. The middle lever set up of the Panchayati
mental Rights be suspended in India? Raj system in India is known as
(a) During war time (a) Zila Parishad
(b) When national emergency is in force (b) District Board
(c) If the Parliament passes a bill (c) Panchayat Samiti
(d) During financial emergency (d) Taluka Board

CLAT-5-Mock Tests.indd 275 13/09/2010 10:36:25


276 MOCK TESTS

82. Which of the following is not the source 89. A. Voidable 1. Capable of being
of revenue of the Village Panchayats in avoided
India? B. Ultra vires 2. Within powers
(a) House tax (b) Teh-bazari C. Intra vires 3. Beyond powers
(c) Property tax (d) Land revenue A B C
83. Which of the following is not a function of (a) 1 3 2
the Village Panchayats in India? (b) 3 2 1
(a) Sanitation (c) 2 1 3
(b) Development of agriculture and vil- 90. A. Arrest 1. Refuge granted to ab-
lage industries sconder
(c) Relief of the poor B. Arson 2. Reckless destruction
(d) Co-operatives of property by fire
84. In Indian context the executive commit- C. Asylum 3. Apprehension of a per-
tee of the Gram Sabha known as Gram son by police officer
Panchayat is elected by A B C
(a) the entire adult population of the vil- (a) 1 2 3
lage (b) 3 2 1
(b) the entire population of the village (c) 2 1 3
(c) the entire male population of the vil- 91. “Roses in December” was written by
lage (a) Arun Shourie
(d) the entire male adult population of the (b) M. C. Chagla
village (c) H. R. Khanna
85. In India general pattern of the present (d) Hidayatullah
Panchayati Raj institution is based on the 92. The Finance Commission makes recom-
Report of the mendation regarding
(a) Balwant Rai Mehta Committee 1. Determination of and principles guid-
(b) Local Enquiry Committee ing grant-in-aid
(c) Taxation Enquiry Committee 2. Economy in expenditure
(d) Jawaharlal Nehru Report 3. Distribution of net proceeds of taxes
86. A. Simpliciter 1. A word or a phrase in between Centre and States
a document is used ab- (a) 1, 3 (b) 1
solutely/uncondition- (c) 2 (d) 2, 3
ally 93. Anybody who holds a civil post under a
B. Sine die 2. Without a date State holds his office during the pleasure
C. Slander 3. A defamatory state- of the
ment (a) Governor (b) President
A B C (c) Parliament (d) State Legislature
(a) 1 2 3 94. Kothari Commission relates to
(b) 3 2 1 (a) sports (b) cultural
(c) 2 1 3 (c) education (d) transport
87. A. Stare decisis 1. Doctrine of precedent 95. Panchayati Raj received constitutional
B. Sub judice 2. Pending matter status with which Constitutional Amend-
C. Sui generis 3. Of its own kind ment Act?
A B C (a) 72nd (b) 73rd
(a) 1 2 3 (c) 74th (d) 77th
(b) 3 2 1 96. The “Legal Services Day” falls on
(c) 1 3 2 (a) 9 November (b) 10 December
88. A. Tortfeasor 1. One who commits tort (c) 26 January (d) 13 February
B. Vandalism 2. Damaging property 97. The “Citizens Day” falls on
C. Void 3. Having no legal effect (a) 18 November (b) 19 November
A B C (c) 20 November (d) 21 November
(a) 1 2 3 98. Sovereignty in India means
(b) 1 3 2 (a) Apex Authority or Alternative Author-
(c) 3 2 1 ity

CLAT-5-Mock Tests.indd 276 13/09/2010 10:36:25


MOCK TESTS 277

(b) Lower Authority or Immediate Au- (d) Supreme Court Advocates and High
thority Court Advocates
(c) Authority of Court 100. Who is the personality to Head UNESCO
(d) Authority of Parliament for first time in history?
99. There are two classes of Advocates in In-
(a) Ms Irina Bokova
dia
(b) Ms Fatima Bibi
(a) Barristers and Solicitors
(b) Advocates and Senior Advocates (c) Ms Anjali Roy
(c) Lawyers and Attorneys (d) Ms Leela Jethamalani

ANSWERS
1. (c) 2. (d) 3. (b) 4. (c) 5. (b) 6. (d) 7. (a) 8. (c) 9. (d) 10. (b)
11. (b) 12. (a) 13. (a) 14. (a) 15. (a) 16. (d) 17. (b) 18. (c) 19. (b) 20. (d)
21. (c) 22. (c) 23. (a) 24. (b) 25. (c) 26. (a) 27. (a) 28. (b) 29. (a) 30. (d)
31. (c) 32. (a) 33. (a) 34. (a) 35. (a) 36. (a) 37. (a) 38. (a) 39. (c) 40. (b)
41. (a) 42. (c) 43. (a) 44. (b) 45. (b) 46. (c) 47. (a) 48. (a) 49. (a) 50. (c)
51. (c) 52. (a) 53. (a) 54. (d) 55. (d) 56. (a) 57. (b) 58. (c) 59. (b) 60. (c)
61. (b) 62. (d) 63. (c) 64. (c) 65. (d) 66. (a) 67. (b) 68. (a) 69. (c) 70. (a)
71. (a) 72. (d) 73. (a) 74. (c) 75. (c) 76. (a) 77. (b) 78. (b) 79. (a) 80. (b)
81. (c) 82. (d) 83. (b) 84. (a) 85. (a) 86. (a) 87. (a) 88. (a) 89. (a) 90. (b)
91. (b) 92. (a) 93. (a) 94. (c) 95. (b) 96. (a) 97. (c) 98. (d) 99. (b) 100. (a)

CLAT-5-Mock Tests.indd 277 13/09/2010 10:36:25


278 MOCK TESTS

PRACTICE PAPER 5
Maximum Marks: 100 Marks 91–100—Outstanding
Time: 30 Minutes Marks 81–90—Excellent
Marks 71–80—Intelligent
Marks 61–70—Good
Marks 51–60—Average
Marks Less than 50—Below Average

1. The power to issue the writs in cases in- 7. A Judge of Supreme Court may resign his
volving Fundamental Rights of Constitu- office by writing under his hand addressed
tion of India belong to to the
(a) Supreme Court and High Courts (a) Chief Justice of India
(b) Supreme Courts only (b) Prime Minister
(c) High Court only (c) President
(d) Law Department of Union Ministry of (d) Seniormost judge of the Supreme
Law Court
2. The Supreme Court upheld Mandal Com- 8. The concept of the Judicial Review has
mission Report for Reservation of OBCs been borrowed from the Constitution of
in government job in case of (a) France (b) UK
(a) Indra Sawhney vs Union of India (c) USA (d) Switzerland
(b) Maneka Gandhi vs Union of India 9. The Appellate Jurisdiction of the Supreme
(c) S. R. Bomani vs Union of India Court does not involve
(d) Vishakha vs State of Rajasthan (a) Criminal cases
3. VAT in India which replaced Sales Tax re- (b) Civil Cases
fers to (c) Cases involving interpretation of the
(a) value added tax Constitution
(b) various added tax (d) Disputes rising out of pre-constitution
(c) valuable additional tax treaties and agreements
(d) value-based additional tax 10. Judicial Review means
4. What is true about a retired judge of Su- (a) the power of the courts to make laws
preme Court of India? (b) the power of the courts to define laws
(a) A retired judge can practice in Su- (c) the power of the courts to define
preme Court only
constitution
(b) A retired judge cannot practice in Su-
(d) the power of the courts to annul any
preme Court
legislative or executive act which is
(c) A retired judge is prohibited from prac-
against the provisions of the constitu-
ticing law before any court in India
tion
(d) A retired judge cannot practice in low-
11. In India an aggrieved public servant of the
er courts in India
Union can challenge the decisions of the
5. Though the Constitution of India provides
Administrative Tribunal
for the federal system of Government, In-
dia has (a) before the Supreme Court
(a) dual judicial system (b) before the High Court
(b) plural judicial system (c) before both (a) and (b)
(c) single judicial system (d) before no court of law
(d) multiple judicial system 12. Can the lower court in India decide an
6. For amendment of the Constitution, a Bill election issue
may be introduced in (a) no
(a) Rajya Sabha (b) yes
(b) Lok Sabha (c) only (a) is right
(c) Either House of Parliament (d) None of the above
(d) Either chamber of State legislature or 13. The Chairman of the Finance Commission
Union Parliament of India must be

CLAT-5-Mock Tests.indd 278 13/09/2010 10:36:26


MOCK TESTS 279

(a) qualified to be a High Court Judge 21. Power to superintend, direct and control
(b) a person having special knowledge of elections to the Panchayat in India is vest-
economics ed in
(c) a person having experience in public (a) State Election Commission
affairs (b) Election Commission at Centre
(d) All the above (c) State Legislature
14. In India the Central Government can as- (d) None of the above
sign any function of the States 22. If there is breakdown of Constitutional
(a) on the directive of the President Machinery in the State in India
(b) on the recommendation of Parliament (a) Governor’s rule is imposed
(c) any time it wishes to do so (b) President’s rule is imposed
(d) with the consent of the concerned (c) emergency is declared
State (d) an administrator is appointed for this
15. The constitution of India makes provisions purpose
to ensure the independence of Judges. The 23. Power to establish the common High
provisions do not include one of the fol- Court for two or more States in India be-
lowing. Identify it. longs to the
(a) Though appointed by the President, (a) Governors of two States
their removal is a difficult process (b) Parliament
(b) Their salaries are charged on the Con- (c) Chief Minister of two States
solidated Fund of India (d) Home Minister of two States
(c) The conduct of a judge cannot be dis- 24. In India recognition to Political Parties is
cussed in Parliament except upon a accorded by the
motion regarding his removal
(a) President
(d) None of the above
(b) Law Commission
16. Railways is a subject on the
(c) Election Commission
(a) Concurrent list (b) Union List
(d) None of the above
(c) State list (d) Residual List
25. The Judges of the Supreme Court of India
17. Under the Constitution, the residuary
are appointed by the
powers vest with the
(a) Prime Minister in consultation with
(a) President
the Cabinet
(b) Supreme Court
(c) Union Government (b) President in consultation with the
(d) State Government Chief Justice of India
18. The State according to the Constitution (c) Prime Minister in consultation with
includes the the Chief Justice of India
(a) Executive (b) Legislature (d) President in consultation with the Law
(c) Local Bodies (d) All of the above Commission of India
19. Which of the following is the correct 26. In case a President dies while in office, the
chronological order of the Prime Ministers Vice-President can act as President for a
of India? maximum period of
I. Indira Gandhi (a) 2 years (b) 1 year
II. Jawahar Lal Nehru (c) 3 months (d) 6 months
III. Morarji Desai 27. Total number of members in a Legislative
IV. Charan Singh Council should not exceed that of a Legis-
(a) I, II, III, IV (b) II, III, I, IV lative Assembly by
(c) II, I, III, IV (d) III, II, IV, I (a) 1/3 rd (b) 2/3 rd
20. The Union Council of Ministers consists (c) 1/2 rd (d) 1/4 rd
of 28. The President of India issues an ordi-
(a) Prime Minister nance
(b) Cabinet Ministers (a) if Parliament is not in session
(c) Cabinet Ministers and Chief Ministers (b) if Parliament is not likely to pass it
of the States (c) if there is any danger to the nation
(d) Cabinet Ministers, Ministers of State (d) if there is a deadlock between the two
and Deputy Ministers Houses of Parliament

CLAT-5-Mock Tests.indd 279 13/09/2010 10:36:26


280 MOCK TESTS

29. Who, among the following, is not appoint- (c) It is a mixture of several Constitu-
ed by the President of India? tions.
(a) Ambassadors (d) It is original.
(b) The Prime Minister 37. Who proposed the Preamble before the
(c) The Attorney-General Drafting Committee of the Constitution?
(d) The Vice-President of India (a) Jawahar Lal Nehru
30. For the philosophy underlying our Consti- (b) B. R. Ambedkar
tution the historic ‘Objectives Resolution’ (c) B. N. Rau
was moved in the Constituent Assembly (d) Mahatma Gandhi
on 22 January 1947 by 38. Who among the following is known as the
(a) Dr B. R. Ambedkar Father of the Indian Constitution?
(b) Jawahar Lal Nehru (a) Dr B. R. Ambedkar
(c) Dr Rajendra Prasad (b) Mahatma Gandhi
(d) Dr S. Radhakrishnan (c) Jawahar Lal Nehru
31. Number of languages recognized by the (d) Vallabhbhai Patel
Indian Constitution is 39. The Constituent Assembly became a sov-
(a) 10 (b) 14 ereign body after
(c) 22 (d) 13 (a) 26 January 1950
32. Which of these animals is National Ani- (b) 26 November 1929
mal of India? (c) 15Augugust 1947
(a) Elephant (b) Horse (d) 26 Augugust 1949
(c) Tiger (d) Lion 40. India became a Sovereign, Democratic
33. In India the Chairman and the Members of Republic on
the Union Public Service Commission are (a) 15 August 1947
appointed (b) 30 January 1948
(a) by the President (c) 26 January 1950
(b) by the Prime Minister (d) 26 November 1948
(c) by the Parliament 41. A police officer cannot investigate
(d) None of these a______without the orders of a Magis-
34. Which of the following statements is not trate
correct regarding Supreme Court of In- (a) Non bailable case
dia?
(b) Bailable case
(a) It can hear disputes between the Union
(c) Non cognizable case
and States under its original jurisdic-
(d) Cognizable case
tion.
42. Which member of the Theosophical Soci-
(b) It can issue writs for the enforcement
ety became the first Woman President of
of Fundamental Rights.
the Indian National Congress?
(c) It can grant permission for special
(a) Usha Erana (b) Uma Banerjee
appeal against the refusal of a High
(c) Annie Besant (d) Nandini Gokhale
Court to grant certificate for appeal.
43. The Directive Principles under Constitu-
(d) It can accept mercy petition submitted
tion of India are
by the convicted persons.
(a) positive instructions to the Govern-
35. Who is vested with the power of superin-
tendence, direction and control of election ment to work for the attainment of set
in India? objectives
(a) Supreme Court (b) negative injuctions to the Govern-
(b) President of India ment to refrain from encroaching on
(c) Election Commission the freedom of the people
(d) Chief Election Commissioner (c) directives to the State to enhance the
36. Which of the following is correct regard- international prestige of the country
ing the Indian Constitution? (d) None of the above
(a) It is completely based on British Con- 44. Which Congress President described the
stitution. Durbar of 1903 to proclaim Edward VII as
(b) It is made only on the basis of Gov- Emperor of India, “a Pompous Pegeant to
ernment of India Act, 1935. a perishing people”?

CLAT-5-Mock Tests.indd 280 13/09/2010 10:36:26


MOCK TESTS 281

(a) Lal Mohan Ghosh 52. Under which Article of the Constitution
(b) W. C. Banerjee of India, Union of India gives direction to
(c) Motilal Nehru States?
(d) Sarojini Naidu (a) Article 257 (b) Article 258
45. Which of the following conditions does (c) Article 259 (d) Article 260
not contribute to the independence of the 53. Which Act in India laid down the terms
Judiciary in India? and conditions for registration and carry-
(a) No change in salary and service con- ing the activities of trade union?
dition of judges can be made which (a) Trade Union Act, 1926
operates against their appointment (b) Trade Union Act, 1929
(b) Once appointed, the judges cannot be (c) Trade Union Act, 1922
removed from their office before the (d) Trade Union Act, 1924
date of retirement 54. Jurisdiction to grant anticipatory bail un-
(c) The salary and allowances of the der section 438 of Cr. P.C. vests with
judges are charged to the Consolidat- (a) The High Court
ed Fund of India and are not subject to (b) The Court of Sessions and the High
Parliament Vote Court
(d) Retired judges are not permitted to (c) The Court of Sessions
carry on practice before any court or (d) The Court of Magistrate
tribunal in India 55. For the purpose of State Reorganization,
46. Who was the Chairman of the Constituent India has been divided into how many
Assembly? zones?
(a) Pt Jawahar Lal Nehru (a) 5 (b) 7
(b) Dr Rajendra Prasad
(c) 9 (d) 12
(c) Dr B. R. Ambedkar
56. What type of Constitution in India is
(d) C. Rajagopalachari
(a) Flexible
47. How long did the Constitution Assembly
(b) Rigid
take to finally pass the Constitution?
(c) Partly rigid and partly flexible
(a) About 6 months in 1949
(d) None of the above
(b) Exactly a year since 26 November
57. Homicide literally means
1948
(a) The punishment of death
(c) About 2 years since 15 August 1947
(b) Killing of animals
(d) About 3 years since 9 December 1946
48. The Constitution of India was adopted by (c) Murder
the (d) Killing of a human being by another
(a) Governor-General human being
(b) British Parliament 58. Which country has largest written in the
(c) Constituent Assembly world?
(d) Parliament of India (a) Britain (b) India
49. The Drafting Committee of the Constitu- (c) Canada (d) South Africa
tion, including the Chairman, comprised 59. The Council of Ministers at the Centre is
of responsible for its conduct to the
(a) seven members (b) five members (a) Lok Sabha (b) Parliament
(c) nine members (d) three members (c) President (d) People
50. In Indian Constitution, India is considered 60. An individual can contest for the election
as of the Lok Sabha it he has attained the age
(a) Confederation of
(b) Federation of Free States (a) 30 years (b) 21 years
(c) Union of States (c) 35 years (d) 25 years
(d) None of the above 61. One whom does the Constitution of India
51. In which of the following years the Plan- confer special responsibility for the en-
ning Commission of India was set up? forcement of Fundamental Rights?
(a) 1950 (b) 1945 (a) President (b) Supreme Court
(c) 1944 (d) 1943 (c) Parliament (d) Governor

CLAT-5-Mock Tests.indd 281 13/09/2010 10:36:26


282 MOCK TESTS

62. With whom are the executive powers of (a) Rajya Sabha (b) Lok Sabha
the Union of India vested? (c) Gram Sabha (d) Vidhan Sabha
(a) Prime Minister 71. The structure of the Panchayati Raj in In-
(b) President dia is based on the level as
(c) Council of Minister headed by the (a) village, district and State
Prime Minister (b) village, tehsil and district
(d) Chief Justice of India (c) village, block and district
63. In the Constitution of India, the Right to (d) village, block and State
Equality is granted by Articles. These are 72. For conspiracy, the minimum number of
__________ persons required is
(a) 14 to 18 (b) 15 to 20 (a) 5
(c) 12 to 17 (d) 17 to 22 (b) 1
64. The Planning Commission of India was (c) 2
set up as (d) No minimum requirement
(a) an advisory body 73. Is intention a mental condition?
(b) an executive body (a) Depends
(c) a government department (b) Yes
(d) an autonomous corporation (c) No
65. Who administers oath of office to a Gov- (d) None of the above
ernor of a State in India? 74. The offence of theft is covered by section
(a) President ….IPC
(b) Chief Justice of India (a) 393 (b) 378
(c) Chief Justice of the concerned High (c) 365 (d) 324
Court
75. A proposal when accepted becomes
(d) Chief Minister
(a) Offer (b) Promise
66. The President of India can be removed
(c) Contract (d) Agreement
from office by
76. A minor’s agreement is void. This propo-
(a) the Prime Minister of India
sition is made in
(b) the Chief Justice of India
(a) Nihal Chand vs Jan Khan
(c) the Parliament
(b) Mohari Beevi vs Dharmodas Ghosh
(d) the Lok Sabha
(c) Sreekrishnan vs Kurukshethra Uni-
67. The President of India has to select the
versity
leader of the majority party in Parliament
as Prime Minister because (d) Nanjappa vs Muthuswamy
(a) he enjoys his confidence 77. If a member of Parliament voluntarily ac-
(b) he commands majority support in the quires the citizenship of a foreign country
Lok Sabha (a) he will be penalized
(c) he is the ablest of all the members of (b) he will continue to be a member of
Parliament Parliament
(d) he is the leader of the party which (c) he will be disqualified from member-
commands the maximum number of ship
Parliament members (d) he will have a choice of renouncing
68. The Chairman of the Public Accounts either
Committee of the Parliament is appointed 78. When the Lok Sabha is dissolved, the
by the Speaker continues in office till a new
(a) Prime Minister (a) Lok Sabha is formed
(b) President (b) Speaker is appointed by the President
(c) Finance Minister (c) Speaker is elected when the new
(d) Speaker of the Lok Sabha House meets
69. The Rajya Sabh was first constituted on (d) Government is formed
(a) 13 May 1952 (b) 2 April 1954 79. Anglo-Indian representatives in the Lok
(c) 3 April 1952 (d) 1 January 1951 Sabha are nominated in terms of the
70. Which of the following is also called the (a) Article 331 (b) Article 80
‘House of Elders’? (c) Article 370 (d) Article 333

CLAT-5-Mock Tests.indd 282 13/09/2010 10:36:26


MOCK TESTS 283

80. The maximum strength of Lok Sabha and 88. One feature distinguishing the Rajya Sab-
Rajya Sabha respectively is ha from the Vidhan Parishad is
(a) 500 and 250 (b) 525 and 238 (a) power of impeachment
(c) 537 and 275 (d) 545 and 250 (b) nomination of members
81. The President can declare National Emer- (c) tenure of members
gency (d) indirect election
(a) only in the event of a foreign attack 89. If in a country there happens to be the of-
(b) only in the event of armed rebellion fice of monarchy as well as a Parliamenta-
(c) due to threat arising on account of for- ry form of government, this monarch will
eign attack or armed rebellion be called
(d) None of the above (a) Head of the Government
82. Which of the following Fundamental (b) Head of the State
Rights under Constitution of India is re- (c) Both Head of the Government and
strained by the Preventive Detention State
Laws? (d) Head of the Cabinet
(a) Right to Freedom 90. When can a President use his discretion in
(b) Right to Religion appointing the Prime Minister?
(c) Right to Equality (a) In all circumstances
(d) Right to Constitutional Remedies (b) In no circumstances
83. According to the Constitution of India, (c) Only when Lok Sabha has been dis-
Amendment of which of the following solved
cannot be proposed in either House of (d) When no political party enjoys a clear
Parliament? majority in the Lok Sabha
(a) Annual Financial Statement 91. The name of the candidate for the office of
(b) Appropriation Bill the President of India has to be proposed
(c) Demand for grant by
(d) Any of them (a) any 50 members of the Electoral Col-
84. Deputy Chairman of the Rajya Sabha in lege
Parliament of India is (b) any 50 members of NCT of Delhi
(a) nominated by the Chairman (c) any 50 members of opposition
(b) elected by Parliament (d) any 50 members of Lok Sabha
(c) nominated by the President
92. Which among the following Rights is sub-
(d) elected by Rajya Sabha from amongst
ject to maximum litigation?
its members
(a) Right to Property
85. The Prime Minister of India acts as a
(b) Right to Speech
channel of communication between the
(c) Right to Constitutional Remedy
President and Council of Minister is pro-
(d) None of the above
vided by the
93. The Sikhs are permitted to carry Kirpan
(a) statute
under the
(b) article 75 of the Constitution
(a) right to freedom of religion
(c) convention
(b) right to life
(d) article 78 of the Constitution of India
(c) right to life and liberty
86. The representation of States in the Rajya
Sabha is determined by (d) right against exploitation
(a) the size of each State 94. The Directive Principles of State Policy in
(b) the size and population of the State Constituent of India aim
(c) the population of the State (a) to make the Constitution an instru-
(d) the proportion of number of members ment of social change
in the State Assemblies (b) to reduce the corruption by public of-
87. Chief Ministers of all the States are ex- ficer
officio members of the (c) to establish supremacy of Constitu-
(a) Finance Commission tion
(b) Planning Commission (d) None of these
(c) National Development Council 95. Which among the following is a factor for
(d) Interstate Council basic structure of Constitution of India?

CLAT-5-Mock Tests.indd 283 13/09/2010 10:36:26


284 MOCK TESTS

(a) Preamble 99. Which of the following is true in context


(b) Fundamental Rights of the President?
(c) Fundamental Duties (a) He addresses the first session of the
(d) Centre-State Relations Parliament after each general elec-
96. A person who is not a member of Parlia- tion of the Lok Sabha and at the com-
ment can be appointed as a Minister by the mencement of the First Session of
President for a maximum period of each year
(a) 9 months (b) 6 months (b) He addresses the first session of Par-
(c) 12 months (d) 3 months liament at the beginning of each year
97. Who, among the following, enjoys the as well as the last session held at the
rank of a Cabinet Minister of the Indian end of each year
Union? (c) He addresses the Parliament daily
(a) Deputy Chairman, Rajya Sabha (d) None of these
(b) Deputy Chairman of the Planning 100. Which of the following is the correct
Commission chronological order of the Prime Ministers
(c) Secretary to the Government of India of India?
(d) None of these I. Indira Gandhi
98. Who administers the oath of office to the II. Jawahar Lal Nehru
President of India before he enters upon III. Morarji Desai
the office? IV. Charan Singh
(a) Vice-President (b) Prime Minister (a) I, II, III, IV (b) II, III, I, IV
(c) Chief Justice (d) Speaker (c) II, I, III, IV (d) III, II, IV, I

ANSWERS
1. (a) 2. (a) 3. (a) 4. (c) 5. (c) 6. (c) 7. (a) 8. (c) 9. (d) 10. (d)
11. (c) 12. (a) 13. (c) 14. (d) 15. (d) 16. (b) 17. (c) 18. (d) 19. (b) 20. (d)
21. (a) 22. (b) 23. (b) 24. (c) 25. (b) 26. (d) 27. (a) 28. (a) 29. (d) 30. (b)
31. (c) 32. (d) 33. (a) 34. (d) 35. (d) 36. (c) 37. (a) 38. (a) 39. (a) 40. (c)
41. (c) 42. (c) 43. (a) 44. (d) 45. (d) 46. (b) 47. (d) 48. (c) 49. (a) 50. (c)
51. (a) 52. (a) 53. (b) 54. (b) 55. (a) 56. (c) 57. (d) 58. (b) 59. (a) 60. (d)
61. (b) 62. (b) 63. (a) 64. (a) 65. (c) 66. (c) 67. (b) 68. (b) 69. (a) 70. (a)
71. (c) 72. (c) 73. (b) 74. (b) 75. (d) 76. (b) 77. (c) 78. (c) 79. (a) 80. (d)
81. (b) 82. (d) 83. (a) 84. (d) 85. (d) 86. (b) 87. (d) 88. (a) 89. (c) 90. (d)
91. (a) 92. (a) 93. (a) 94. (a) 95. (a) 96. (b) 97. (b) 98. (c) 99. (a) 100. (b)

CLAT-5-Mock Tests.indd 284 13/09/2010 10:36:26


MOCK TESTS 285

PRACTICE PAPER 6
Maximum Marks: 100 Marks 91–100—Outstanding
Time: 30 Minutes Marks 81–90—Excellent
Marks 71–80—Intelligent
Marks 61–70—Good
Marks 51–60—Average
Marks Less than 50—Below Average

1. The office of the Prime Minister of India 8. The power to control the expenditure of
(a) has been created by Parliament the Government of India rests exclusively
(b) is based on conventions with
(c) has been created by the Constitution (a) Parliament
(d) is less powerful than that of the Presi- (b) President
dent (c) Comptroller and Auditor-General
2. Fundamental Duties do not enjoy any (d) Union Finance Minister
(a) Legal sanction 9. In India Final decision whether a member
(b) Moral sanction of the Lok Sabha has incurred disqualifi-
(c) Political sanction cation under the Anti-Defection Law rests
(d) social sanction with
3. In the Indian polity, ultimate sovereignty (a) Speaker
rests with (b) Election Commission
(a) the Parliament (c) President
(b) the Constitution (d) Supreme Court or High Court
(c) the President 10. What happens when a Money Bill by Lok
(d) the People Sabha is partially amended by Rajya Sab-
4. A Money Bill can originate in the State ha?
(a) Lok Sabha can proceed with the Bill
Legislature only with the prior consent of
without the amendment proposed by
the
Rajya Sabha
I. Chief Minister II. President
(b) Bill can be lapsed
III. Governor IV. None of these
(c) Bill can be postponed for next ses-
(a) I (b) I and III
sion
(c) II (d) III
(d) None of the above
5. A dispute between two houses of the Par-
11. Rajya Sabha is also called as….
liament can be resolved by
(a) Council of Nominated Members
(a) joint sitting of both houses summoned (b) Council of Representatives
by the President (c) Council of States
(b) ruling of the Lok Sabha Speaker (d) None of the above
(c) ruling of the Rajya Sabha chairperson 12. Which part of the Constitution is justicia-
(d) None of these ble?
6. Right to education is added as Article 21A (a) Fundamental Rights
in the Constitution of India by (b) Preamble
(a) 86th Constitution Amendment, 2002 (c) Fundamental Duties
(b) 88th Constitution Amendment, 2003 (d) Directive Principles
(c) 89th Constitution Amendment, 2003 13. An Obiter dicta by a judge is a
(d) 90th Constitution Amendment, 2004 (a) Statement of opinions made, by the
7. The Constitution of India has vested the judge, while deciding the point at is-
executive power of the Union of India in sue
(a) Prime Minister (b) Statement within the orbit of mores of
(b) Council of Minister the society
(c) President (c) Statement within the orbit of facts of
(d) Deputy Prime Minister the case

CLAT-5-Mock Tests.indd 285 13/09/2010 10:36:26


286 MOCK TESTS

(d) Statement not covered by any of the 22. The four pillars of democracy are
above classifications (a) The President, the Prime Minister, the
14. In a Parliamentary system the executive is people and the Press
responsible to (b) The Parliament, the President, the Ju-
(a) The legislature diciary and the Press
(b) The judiciary (c) The Parliament, the Executive, the Ju-
(c) The people directly diciary and the Press
(d) None of the above (d) None of the above
15. “Equal pay for equal work for both males 23. Who among the following is fourth Pillar
and females”. Which article has proposed of Indian Constitution
this? (a) Attorney General
(a) Article 48 (b) Article 39 (b) Lieutenant General
(c) Article 14 (d) Article 25 (c) Advocate General
16. Who is responsible for the registration of (d) Comptroller and Auditor General
the voters in India? 24. A sentence of death by a lower Sessions
(a) Voters court
(b) Political parties (a) is operational if no appeal is made to
(c) Election commission higher courts
(d) None of the above (b) must be confirmed by the High Court
17. Term of Rajya Sabha members in Indian (c) must be confirmed by the Supreme
Parliament was fixed by the Court
(a) Constitution (b) Cabinet (d) must be confirmed by the President
(c) President (d) Parliament 25. In….supreme Court held that Judicial Re-
18. What financial power is enjoyed by the view is a basic feature of Constitution and
President of India? cannot be amended
(a) Certain Money Bills can originate in (a) Minerva Mills case
Parliament only on the recommenda- (b) Golaknath case
tion of the President (c) Keshvananda Bharti case
(b) He can advance money out of the (d) None of the above
Contingency Fund of India 26. A Panchayat Samiti at the block level in
(c) He can appoint the Finance Commis- India is only
sion to recommend distribution of tax- (a) an advisory body
es between the Centre and the State (b) a consultative committee
(d) All the above (c) a co-ordinate and supervisory author-
19. The members of Rajya Sabha in Parlia- ity
ment of India are (d) an administrative authority
(a) nominated by the Governors of the re- 27. The Legislative power of the President of
spective states India includes all the following but not
(b) elected by elected members of the (a) the power to summon or prorogue the
Lok Sabha Houses of Parliament
(c) elected by elected members of the (b) power to summon a joint sitting of
State Legislative Assemblies both the Houses to resolve a deadlock
(d) All the above (c) power to nominate the members to the
20. The salaries and allowances of Union Lok Sabha
Ministers in India are determined by (d) right to address either House at any
(a) President of India time and to require the attendance of
(b) Finance Commission members for this purpose
(c) Parliament 28. Who among the following jurists was the
(d) Ministry of Parliamentary Affairs Chairman of “Committee for Implement-
21. Who among the following had the longest ing Legal Aid Schemes” (CILAS)?
tenure as the Chief Justice of India? (a) A. S. Anand (b) P. N. Bhagwati
(a) P. N. Bhagwati (c) R. S. Pathak (d) Kuldeep Singh
(b) M. H. Beg 29. Democracy does not stand for
(c) Y. V. Chandrachud (a) rule of law
(d) H. J. Kania (b) equal right for all

CLAT-5-Mock Tests.indd 286 13/09/2010 10:36:27


MOCK TESTS 287

(c) liberty of the individual (b) Chairman of Rajya Sabha


(d) unrestricted freedom of the people (c) Speaker of Lok Sabha
30. On the basis of the Government of India (d) Chairman of Public Accounts Com-
Act, 1935 which introduced the political mittee
autonomy elections took place in the prov- 38. Match the following in Indian context
inces in the year I. Upper Chamber
(a) 1937 (b) 1939 II. Lower Chamber
(c) 1940 (d) 1938 III. Fourth Pillar
31. The UPSC consists of a Chairman and IV. Head of Government
(a) six other members A. Rajya Sabha B. Lok Sabha
(b) eight other members C. CAG D. Prime Minister
(c) eleven other members (a) I – A, II – B, III – C, IV – D
(d) unspecified number of members (b) I – B, II – C, III – D, IV – A
32. The Electorate means (c) I – C, II – D, III – A, IV – B
(a) Those people who do not have the (d) I – C, II – D, III – B, IV – A
right to elect their representatives 39. The present Director-General of the World
(b) All the citizens who possess the right Trade Organization is
to vote and elect their representatives (a) Pascal Lamy
in an election (b) Oliver Long
(c) Those people who have the right to be (c) Arthur Dunkell
elected (d) Renalto Ruggiero
(d) Those people who have the right to 40. Which of the following writs is the bul-
live in a country wark of the personal freedom?
33. The delimitation of the territorial constitu- (a) Mandamus
encies in India is the responsibility of (b) Habeus Corpus
(a) President (c) Quo Warranto
(b) Governor (d) Certiorary
(c) Election Commission 41. In which of the following does the Rajya
(d) Parliament Sabha enjoy exclusive right?
34. In India, political parties are given recog- (a) Initiating impeachment proceedings
nition by against the President
(a) President (b) Taking initiative for the removal of
(b) Law Commission the Vice-President
(c) Speaker of Lok Sabha (c) Recommending creation of new All
(d) Election Commission India Services
35. If India decides to have a Presidential (d) In all the above matters
form of government, the first and foremost 42. Who is entitled to initiate a Bill for Consti-
amendment has to be made affecting the tutional amendments?
(a) system of judiciary (a) State Legislative Assemblies
(b) composition of Parliament (b) Lok Sabha only
(c) executive-legislature relationship (c) Either House of Parliament
(d) provisions pertaining to fundamental (d) Rajya Sabha only
Rights 43. The Chief Justice of a High Court is ap-
36. Power to dissolve Rajya Sabha in Parlia- pointed by the President in consultation
ment of India is vested in ………. with
(a) Prime Minister I. Chief Justice of the Supreme Court
(b) President II. Former Chief Justice of the High
(c) President on advice of Prime Minis- Court
ter III. Other Judges of the High Court
(d) Not possible IV. Governor of the State concerned
37. Whether a Bill is a Money Bill in Lok (a) I and II (b) II and III
Sabha or not is certified by the ………. (c) I and IV (d) I, III and IV
and his decision is conclusive 44. The Judges and magistrates of the Civil
(a) President Courts are appointed by the

CLAT-5-Mock Tests.indd 287 13/09/2010 10:36:27


288 MOCK TESTS

(a) Subordinate Judge (c) powers related to country’s external


(b) Chief Justice of the High Court affairs
(c) Governor (d) powers not included in any of the
(d) District Magistrate three lists
45. The Judges of the Supreme Court can be 52. Planning in India derives its objectives
removed from office by the from
(a) President (a) Fundamental Rights
(b) Prime Minister (b) Directive Principles
(c) President on request of Parliament (c) Fundamental Duties
(d) Chief Justice of India (d) None of these
46. In India the main areas of conflict between 53. Who can dissolve the Lok Sabha before
the legislature and the judiciary is/are the expiry of its term?
(a) Existence, extent and scope of Parlia- (a) The President
mentary privileges and power of Leg- (b) Prime Minister
islatures to punish for contempt (c) The President on the advice of the
(b) Interference in the proceedings of Par- Prime Minister
liament/Legislature (d) None of these
(c) Decision given by the presiding of- 54. The remuneration for the Attorney-Gener-
ficers of legislatures under the Anti- al of India is determined by
Defection Law (a) the Council of Ministers
(d) All the above (b) the Constitution
47. The interpretation of Constitution of India
(c) the Parliament
comes under which jurisdiction of Su-
(d) the President
preme Court
55. How long is the tenure of the Prime Min-
(a) Appellate Jurisdiction
ister in India?
(b) Civil Jurisdiction
(a) It depends on the President’s direc-
(c) Special Leave
(d) Both (a) and (b) tion
48. The present Solicitor-General of India is (b) So long as he enjoys the confidence of
(a) Gopal Subramaniam the Lok Sabha
(b) A. Parameswaran (c) So long as he is leader of the majority
(c) Deepankar Gupta party in the Lok Sabha
(d) Milon Banerjee (d) 5 years
49. To adjudicate any dispute relating to Inter- 56. In the 86th Constitutional Amendment
State rivers or Inter-State river valley in Act, 2002 of India the Article 21A viz.,
India, the Right to education is added to the lists of
(a) Supreme Court only is empowered Fundamental Rights. Which among the
under Article 131 of the Constitution following is a guardian of such Funda-
(b) High Court of concerned State enjoy mental Rights?
concurrent jurisdiction (a) Supreme Court and High Courts
(c) President of India has only power to (b) High Court only
decide (c) Supreme Court only
(d) Parliament is empowered to establish (d) Lower courts and High Courts
Tribunal and to that extent jurisdiction 57. Article I of the Constitution of India de-
of Supreme Court may be extended clares India as
50. Which among the following Constitution- (a) Federal State
al amendments in India is otherwise called (b) Quasi Federal State
the “Mini Constitution”? (c) Unitary State
(a) 42nd Amendment (d) Union of States
(b) 24th Amendment 58. Which functionaries can be invited to give
(c) 14th Amendment his opinion in the Parliament of India?
(d) 20th Amendment (a) Attorney General of India
51. The residuary powers mean the (b) Chief Justice of India
(a) powers shared between the Union and (c) Chief Election Commissioner of In-
State Governments dia
(b) powers related to country’s internal (d) Comptroller and Auditor General of
affairs India

CLAT-5-Mock Tests.indd 288 13/09/2010 10:36:27


MOCK TESTS 289

59. Which of the following is the bulwark of 67. Expression ‘socialist’ in the Preamble of
personal freedom? India has been added by taking the views
(a) Mandamus of
(b) Habeaus Corpus (a) Marx and Loski
(c) Quo Warranto (b) Marx and Gandhi
(d) Certiorari (c) Rousseau and Gandhi
60. In India, the “right to free education” in (d) Gandhi and Hegel
Constitution emanates from 68. The main objective of the Regulating Act,
(a) Article 29 and 30 1773 for India was to
(b) Article 14 (a) bring the management of the company
(c) Articles 21 and 21A under the control of the British Parlia-
(d) Article 19 ment and the British Crown
61. The Constitution of India vests the execu- (b) introduce reforms in the Company’s
Government in India
tive power of the Indian Union in
(c) provide remedies against illegalities
(a) the President
and anarchy committed by the Com-
(b) the Parliament
pany’s servants in India
(c) the Council of Ministers
(d) All of the above
(d) the Prime Minister 69. Under which plan separation between rev-
62. The writ of prohibition issued by the Su- enue and judicial function was maintained
preme Court or a High Court is issued in India?
against (a) Plan of 1772 (b) Plan of 1774
(a) administrative and judicial authorities (c) Plan of 1780 (d) Plan of 1782
(b) administrative authorities and govern- 70. The special status of Jammu and Kashmir
ment in Indian Union implies that the State has
(c) judicial or quasi-judicial authorities (a) a separate defence force
(d) administrative authorities only (b) a separate constitution
63. The right which is available even to a non- (c) a separate judiciary
citizen is (d) All the above
(a) Freedom to acquire property or to 71. Who is competent to prescribe conditions
carry on any occupation, trade or busi- for acquisition of citizenship?
ness (a) Parliament and state legislatures joint-
(b) Freedom to move, reside and settle in ly
any part of the territory of India (b) Election Commission
(c) Freedom of speech, assembly and as- (c) Parliament
sociation (d) President
(d) Right to Constitutional Remedies 72. The Indian Constitution declares that pro-
64. The proclamation of National Emergency tection of life and liberty
ceases to operate unless approved by the (a) can be taken away during the Emer-
Parliament within gency through President order
(a) three months (b) two months (b) can be taken away only according to
(c) one month (d) six months procedure established by law
65. Who is the ex-officio Chairman of the (c) can never be taken away in my condi-
Council of States? tion
(d) None of above
(a) President
73. The basic condition imposed on the citi-
(b) Speaker
zen’s right to assemble is that the assem-
(c) Prime Minister
bly should be
(d) Vice-President
(a) peaceful
66. The Preamble of the Constitution of India (b) non-violent
provides equality of status and opportu- (c) peaceful and unarmed
nity in (d) for constructive aims
(a) Public Services 74. Who administers the oath of office to the
(b) Social and Economic affair only members of council of ministers in the
(c) Social, Political and Economic state?
(d) Both (a) and (b) (a) the Attorney-General of the state

CLAT-5-Mock Tests.indd 289 13/09/2010 10:36:27


290 MOCK TESTS

(b) the Governor (a) the presiding officers of both the


(c) the Chief Justice of the State High Houses of Parliament
Court (b) the legislatures of the states con-
(d) None of the above cerned
75. Who of the following is the executive (c) Supreme Court
head of a state? (d) President
(a) the President 82. In Parliamentary government, Ministers
(b) the Governor remain in office so long as they enjoy
(c) the Chief Minister (a) confidence of the upper house of the
(d) None of the above legislature
76. Can the courts inquire into proceedings of (b) support of the armed forces
the Legislature under Article 212 of the (c) confidence of the popular chamber of
Constitution of India? legislature
(a) Yes (d) popular support
(b) No 83. Parliament of India is composed of
(c) with the permission of the Speaker (a) Lok Sabha only
(d) with the permission of the Governor (b) Rajya Sabha only
77. In case of vacancy in the office of the (c) Lok Sabha and Rajya Sabha
Vice-President during his function as of- (d) Lok Sabha, Rajya Sabha and Presi-
ficiating President of India who among the dent
following will act as President of India? 84. The Chairman of the Rajya Sabha is
(a) Speaker of Lok Sabha (a) elected by the members of Rajya Sab-
ha
(b) Deputy Chairman of Rajya Sabha
(b) nominated by the President
(c) Chief Justice of India
(c) elected by members of both Houses of
(d) Chief Election Commissioner
Parliament
78. Who among the following can prescribe
(d) elected by Parliament and State Legis-
an authority for delimitation of constitu-
latures jointly
encies of India?
85. In the case of a deadlock between the two
(a) Parliament
Houses of the Parliament, the joint sitting
(b) State Legislative Assemblies
is presided over by the
(c) Election Commission (a) President
(d) None of the above (b) Vice-President
79. Right to Education viz., Article 21A was (c) Speaker of Lok Sabha
added in Constitution by the (d) Member of the Lok Sabha specifically
(a) Constitution 86th Amendment Act, elected for the purpose
2002 86. ‘Damages’ are awarded except in one
(b) Constitution 85th Amendment Act, case
2001 (a) Tort
(c) Constitution 84th Amendment Act, (b) Breach of contract
2001 (c) Libel Suit
(d) Constitution 83rd Amendment Act, (d) Criminal conduct
2000 87. India is unitary because it has
80. The power to dissolve the Lok Sabha is (a) single Constitution for Union and
vested in the States
(a) President who exercises it on the ad- (b) single Constitution for Union
vice of the Prime Minister (c) single Constitution for States
(b) President at his absolute discretion (d) Union of States
(c) Prime Minister without whose advice 88. Bhairon Singh Shekhawat the former
the Lok Sabha cannot be dissolved at Vice-President of India to whom Hamid
all Ansari succeeded was
(d) Joint opinion of the President, Vice- (a) former Chief Minister of Rajasthan
President and the Speaker (b) former Chief Minister of Kerala
81. A bill for alteration of boundaries of state (c) former Chief Minister of Karnataka
should not be introduced in the Parliament (d) former Chief Minister of Maharashtra
without the recommendation of 89. In India a Bill to change the boundaries of

CLAT-5-Mock Tests.indd 290 13/09/2010 10:36:27


MOCK TESTS 291

a State has to be passed by __________ (c) Speaker of Lok Sabha


majority in __________ to become a law (d) Select Parliamentary Committee
(a) Simple; Parliament 95. The Council of Ministers has to tender its
(b) Simple; State Legislature resignation if a vote of no-confidence is
(c) Two third; Parliament passed against it
(d) Two third; State Legislature (a) by the Lok Sabha
90. Citizenship by descent in India means (b) by the Rajya Sabha
(a) a person born outside India on or after (c) by the two houses at a joint sitting
26th January, 1950 and 10th Decem- (d) by the two houses of Parliament by
ber, 1992 and if father is Indian citi- two thirds majority
zen 96. The only woman member of the Consti-
(b) on or after 10th December, 1992, if ei- tution Review Committee of India which
ther of the parents is an Indian citizen submitted Report in 2002 was
at the time of his birth (a) Mohini Giri
(c) Either (a) or (b) (b) Sumitra Kulkarni
(d) Only (a) is correct (c) Subhashilli Ali
91. Lok Sabha is superior to the Rajya Sabha (d) Vibha Parthasarathi
because 97. The Constituent Assembly of India was
(a) it alone controls the purse formed under the provisions of
(b) it is directly elected (a) Cabinet Mission Plan
(c) it can cast the Council of Ministers (b) Cripps Commission
through a vote of no-confidence (c) Mountbatten Plan
(d) of all the above reasons (d) None of the above
92. What can be the maximum gap between 98. In India, a person can be a member of
the two sessions of Parliament? State Council of Ministers without being
(a) nine months (b) six months a member of State Legislature for
(c) three months (d) four months (a) a maximum period of 9 months
93. The Rajya Sabha is a permanent house (b) a maximum period of 6 months
but (c) a maximum period of 12 months
(a) one-half of its members retire every (d) a maximum period of 2 months
two years 99. An Act of Parliament is called
(b) one-fifth of its members retire every (a) bill (b) legislation
year (c) statute (d) public policy
(c) one-third of its members retire every 100. The doctrine of separation of power signi-
two years fies
(d) one-half of its members retire every (a) a division of power between Centre
three years and State Government
94. Who decides whether a bill is a Money (b) distribution of power between branch-
Bill or not? es of Government
(a) President (c) providing for judicial supremacy
(b) Prime Minister (d) None of the above

ANSWERS
1. (c) 2. (c) 3. (d) 4. (d) 5. (a) 6. (a) 7. (c) 8. (d) 9. (a) 10. (a)
11. (c) 12. (a) 13. (a) 14. (a) 15. (b) 16. (c) 17. (a) 18. (d) 19. (c) 20. (c)
21. (c) 22. (c) 23. (d) 24. (b) 25. (c) 26. (c) 27. (d) 28. (a) 29. (d) 30. (b)
31. (d) 32. (b) 33. (c) 34. (d) 35. (c) 36. (d) 37. (c) 38. (a) 39. (d) 40. (b)
41. (b) 42. (c) 43. (d) 44. (c) 45. (c) 46. (b) 47. (a) 48. (a) 49. (a) 50. (a)
51. (d) 52. (b) 53. (c) 54. (d) 55. (b) 56. (a) 57. (d) 58. (a) 59. (b) 60. (c)
61. (a) 62. (b) 63. (d) 64. (d) 65. (d) 66. (a) 67. (b) 68. (d) 69. (d) 70. (b)
71. (c) 72. (c) 73. (c) 74. (c) 75. (b) 76. (b) 77. (c) 78. (c) 79. (a) 80. (a)
81. (d) 82. (c) 83. (d) 84. (c) 85. (c) 86. (d) 87. (a) 88. (a) 89. (a) 90. (b)
91. (d) 92. (b) 93. (c) 94. (c) 95. (a) 96. (a) 97. (a) 98. (b) 99. (b) 100. (a)

CLAT-5-Mock Tests.indd 291 13/09/2010 10:36:27


292 MOCK TESTS

PRACTICE PAPER 7
Maximum Marks: 100 Marks 91–100—Outstanding
Time: 30 Minutes Marks 81–90—Excellent
Marks 71–80—Intelligent
Marks 61–70—Good
Marks 51–60—Average
Marks Less than 50—Below Average

1. Overseas citizenship of India is granted to tain the people’s verdict in his favour in a
people of Indian origin bye-election within
(a) in any country (a) 15 days (b) one month
(b) in select countries (c) three months (d) six months
(c) provided their home countries allowed 7. Part V of the Constitution deals with
them to do so I. Union Executive
(d) in all countries except Sri Lanka II. Parliament
2. India is a federal; political system. How- III. Supreme Court and High Court
ever, all powers have not been divided be- IV. Comptroller and Auditor-General
tween the Union and the States. Which of (a) I, II and III (b) I, II and IV
the following is/are not so divided? (c) I and II (d) I only
1. Legislative 2. Executive 8. India has been described by the Constitu-
3. Judicial 4. Financial tion as:
(a) 1 and 2 (b) 3 and 4 (a) A Union of States
(c) 3 only (d) 4 only (b) Quasi-federal
3. The fundamental rights under the Consti- (c) A Federation of States
tution of India have (d) None of these
(a) retrospective effect 9. The Indian Constitution establishes a sec-
(b) no effect under Article 13(1) or 13(3) ular State, meaning
(b) I. The State treats all religious equally
(c) only future effect II. Freedom of faith and worship is al-
(d) None of the above lowed to the people
4. In India the first Constitutional Amend- III. Educational institutions, without ex-
ment was made in 1951. The main object ception, are free to impart religious
was instruction
(a) to secure constitutional validity of rul- IV. The State makes no discrimination on
ing by kings the basis of the religion in matters of
(b) to add Article 31A and 31B and employment
Schedule 9 (a) I, II, III and IV (b) I, II and IV
(c) both (a) and (b) (c) II, III and IV (d) I and II
(d) None of the above 10. National Development Council is a …
5. Keshvananda Bharti case, overruled Go- (a) Creation of the executive body
laknath case, and said (b) Constitutional body
(a) Fundamental Rights can be amended (c) Cabinet’s Secretariat
(b) All provision of the Constitution can (d) Joint Right
be amended 11. The Liberhan Commission relates to
(c) Parliament can amend any part of the (a) Anti-Terrorism Commission
Constitution so long as the amend- (b) Demolition of Babri Masjid
ment does not take away or abridge (c) Pollution in Country
any of the basic features of the Con- (d) None of the above
stitution amendments 12. What is the maximum strength prescribed
(d) None of the above in Constitution of India for Lok Sabha?
6. A minister, who is not a member of the (a) 543 (b) 545
Legislative Assembly, is required to ob- (c) 550 (d) 552

CLAT-5-Mock Tests.indd 292 13/09/2010 10:36:27


MOCK TESTS 293

13. The Constitution of India guarantees right (a) Population


against exploitation to (b) territorial jurisdiction
(a) Children and Women (c) number of Prime Ministers sending to
(b) Men and Women Parliament
(c) Children and Oldmen (d) None of the above
(d) Factory Workers only 23. The minimum age of a person required to
14. Name the Commission which organized be Prime Minister in India is
States on the basis of languages (a) 25 years
(a) J. Fazal Ali Commission (b) 30 years
(b) Dharam Vir Commission (c) 35 years specified
(c) Ashok Mehta Commission (d) no age required
(d) Sanathan Commission 24. The defeat of government in Rajya Sabha
15. Which of the following is not necessary of Parliament of India leads to
attribute of federalism? (a) Prime Minister’s resignation
(a) An independent court to arbitrate on (b) Resignation of President
disputes between the Federation and (c) Resignation of Chairman
Federal States (d) None of the above
(b) A written Constitution 25. In India first BJP ministry under Mr Atal
(c) Division of powers between the Fed- Behari Vajpayee held office for a period
eration and Federal Unit of
(d) Dual citizenship of Federation and (a) 13 days (b) 45 days
Federal Unit (c) 10 days (d) 3 months
16. In the interim government formed in 1946, 26. The President can promulgate an ordi-
the Vice-President of the Executive Coun- nance
cil was (a) when the Lok Sabha has been dis-
(a) Dr Rajendra Prasad solved
(b) C. Rajagopalachari (b) when there is conflict between the two
(c) Jawaharlal Nehru Houses regarding a particular bill
(d) Dr S. Radhakrishnan (c) when a bill passed by the Lok Sabha is
17. Golden Jubilee of Parliament of India was not passed by the Rajya Sabha
celebrated on (d) when both the Houses of the Parlia-
(a) 13 May 2002 (b) 8 May 2002
ment are not in session
(c) 26 April 2002 (d) 18 May 2002
27. Which of the following best defines the
18. Which language was added as the 15th of-
Parliamentary term ‘Crossing the Floor’?
ficial language to the 8th Schedule of the
(a) Leaving the opposition to join the
Constitution?
party in power or vice-versa by a Par-
(a) Hindi (b) Urdu
liamentarian
(c) Sindhi (d) Marathi
(b) An attempt to occupy the seat of some
19. Which article of Indian constitution pre-
other Parliamentarian
scribes Hindi in Devanagri script as the
(c) Leaving a House by minister in be-
official language of the union?
tween in order to attend the other
(a) 341 (b) 342
House
(c) 343 (d) 346
20. The reference to Hindus in Article 25 of (d) Walk out by some Parliamentarians
the Constitution does not include in order to boycott the proceedings of
(a) Buddhists (b) Jains the House
(c) Parsees (d) Sikhs (e) None of these
21. The Speaker of Lok Sabha uses his vote in 28. An ordinance promulgated by the Presi-
case of dent
(a) a tie, when votes are equally divided (a) will lapse automatically after 2
(b) amendment of Constitution months
(c) emergency (b) will lapse on the expiration of 6 weeks
(d) in no case from the meeting of the Parliament
22. Seats are allocated to various States in the (c) will automatically become a law after
Lok Sabha on the basis of 6 months

CLAT-5-Mock Tests.indd 293 13/09/2010 10:36:27


294 MOCK TESTS

(d) will continue to be in force till it is su- I. Chief Justice of the Supreme Court.
perseded by an Act of the Parliament. II. Former Chief Justice of the High
29. Indira Swahney vs Union of India is a case court
popularly known as III. Other Judges of the High court
(a) Mandal judgment IV. Governor of the State concerned
(b) Ayodhya judgment (a) I and III (b) II and III
(c) Suicidal judgment (c) I and IV (d) I, III and IV
(d) Election Commission judgment 37. The Advisory opinion tendered by the Su-
30. The status of a recognized Opposition par- preme Court
ty in the Lok Sabha is secured only when (a) is not made public at all
the percentage of the total seats in the Lok (b) is binding on the President only if it is
Sabha won by the political party is unanimously made
(a) 5 (b) 8 (c) is binding on the President
(c) 10 (d) 15 (d) is not binding on the President
31. The writ of Certiorari is issued by a Supe- 38. The phrase ‘bicameral legislature’ means
rior Court (a) Parliamentary system of Government
(a) to an inferior court to transfer the re- (b) a legislature consisting of lower and
cord of case for review an upper chamber
(b) to a public authority to produce a per- (c) a single assembly
son detained by it before the court (d) an elected legislature
within 24 hours 39. The Legislative Council is expected to
(c) to an inferior court to stop further pro- have a strength equal to one third of the
ceeding of case membership of the Legislative Assembly.
(d) None of the above The minimum strength of the Council has
32. Who decides the disputes regarding elec- to be
tion of President of India? (a) 50 (b) 40
(a) Supreme Court (c) 100 (d) 30
(b) High Court 40. Which one of the following does not take
(c) Parliament part in the election of the President?
(d) Supreme Court or High Courts (a) elected members of Lok Sabha
33. In India there are (b) members of the legislative councils
(a) 3 types of emergencies in Constitu- (c) elected members of the Rajya Sabha
tion (d) None of the above
(b) 4 types of emergencies in Constitu- 41. When the offices of both the President and
tion Vice-President of India are vacant, then
(c) 6 types of emergencies in Constitu- their function will be discharged by
tion (a) Prime Minister
(d) 12 types of emergencies in Constitu- (b) Home Minister
tion (c) Chief Justice of India
34. In case the office of President of India (d) Speaker
falls vacant the same must be filled within 42. The appellate jurisdiction of a High Court
__________ in India is
(a) 1 year (b) 18 months (a) civil
(c) 3 months (d) 6 months (b) criminal
35. A National Emergency in India remains in (c) both civil and criminal
operation, with the approval of Parliament (d) None of the above
for 43. __________ is an order of a Court to a
(a) maximum of 1 year person to appear before it
(b) indefinite period (a) Warrant (b) Subpoena
(c) maximum of 2 years (c) Call (d) Remand
(d) maximum of 4 years 44. Which one of the following writs is a bul-
36. The Chief Justice of a High Court is ap- wark of personal freedoms?
pointed by the President in consultation (a) Mandamus (b) Habeas Corpus
with (c) Quo Warranto (d) Certiorari

CLAT-5-Mock Tests.indd 294 13/09/2010 10:36:28


MOCK TESTS 295

45. Sonali Kapoor is a school teacher. The 52. Who has designed the layout of Indian
management of the school terminates her Constitution?
without giving her a show cause notice. (a) Nandlal Bose
She files a writ petition before the High (b) Jamini Roy
Court. Most likely the High Court may set (c) K. C. S. Panicker
aside the termination by issuing writ in the (d) Rabindra Nath Tagore
nature of 53. Subhash Chandra Bose, established Gov-
(a) Mandamus (b) Certiorari ernment of Free India before Indepen-
(c) Quo Warranto (d) Prohibition dence of India in
46. Which of the following is not true about (a) Hong Kong (b) Japan
the Supreme Court? (c) Singapore (d) Rangoon
(a) It has only the Appellate Jurisdiction 54. President of India has position similar to
(b) It is the highest federal court of India (a) German Chancellor
(c) It does not have the Original Jurisdic- (b) American President
tion (c) British Queen
(d) It can amend the Constitution (d) Japanese Prime Minister
47. What is meant by a ‘Court of Record’? 55. The concept of Public Interest Litigation
(a) The court that preserves all its re- which has become quite popular in India
cords originated in
(b) The court that maintains records of all (a) USA (b) Australia
lower courts (c) Canada (d) UK
(c) The court that can punish for its con- 56. A main characteristic of the cabinet sys-
cept tem is
(d) The court that is competent to give di-
(a) leadership of the Prime Minister
rections
(b) inclusion of the President
48. Judicial Review function of the Supreme
(c) cabinet decisions are not binding on
Court
all Ministers
(a) review its own judgment
(d) All the above
(b) review the functioning of judiciary in
57. Which one of the following emergencies
the country
can be declared by the President on his
(c) examine the constitutional validity of
own?
the laws
(d) undertake periodic review of the Con- (a) Emergency due to threat to the finan-
stitution cial stability or credit of India
49. Which is not an eligibility criterion for ap- (b) Emergency due to external aggression
pointment as a Judge of the High Court? or internal disturbances
(a) Must have been a High Court Judge (c) Emergency due to failure of the con-
for at last 5 years stitutional machinery in a state
(b) Must have attained the age of 55 (d) None of the above
years 58. The tenure of the Union Council of Minis-
(c) Must have been an advocate of a High ters is
Court for not less than 10 years (a) five years
(d) Must be, in the opinion of the Presi- (b) uncertain
dent, a distinguished jurist (c) fixed and co-terminus with the Presi-
50. Under the Constitution, the residuary dent
powers vest with the (d) co-terminus with the Lok Sabha
(a) President 59. In the election to the post of the President
(b) Supreme Court (a) M.Ps and M.L.As of a State have the
(c) Union government same number of votes
(d) State government (b) M.L.As of different States have dif-
51. In India Commission which submitted the ferent number of votes
first report on backward classes was head- (c) Each elected M.P. or M.L.A. has an
ed by equal number of votes
(a) B. P. Mandal (b) L. G. Haranur (d) All M.Ps and M.L.As have one vote
(c) Kaka Kalekar (d) Chinnapa Reddy each

CLAT-5-Mock Tests.indd 295 13/09/2010 10:36:28


296 MOCK TESTS

60. In the election of the President (b) Deputy Chairman, Planning commis-
(a) parity between the Centre and the sion
States has been maintained (c) Chief Minister of a Union Territory
(b) the States are given greater weight- 69. Which is true regarding the President of
age India?
(c) the Centre enjoys greater weightage I. He is the Chief Executive
(d) None of the above II. He is the Supreme Commander of the
61. Under which Article of the Constitution armed forces
of India has the untouchability been abol- III. He is the titular head of the State
ished and its practice prohibited? IV. He is part of the Union Legislature
(a) Article 19 (b) Article 21 (a) I and III (b) I and II
(c) Article 17 (d) Article 20 (c) II, III and IV (d) I, II, III and IV
62. Article 21 of the Constitution of India 70. The President of India made use of his
guarantees the life and liberty of the indi- veto power once in
vidual which cannot be deprived of except (a) the Dowry Prohibition Bill
according to the procedure established by (b) the Indian Post Office (Amendment)
_____ Bill
(a) Parliament (b) Law (c) the Hindu Code Bill
(c) Judiciary (d) President (d) the PEPSU Appropriation Bill
63. The Fundamental Rights in India are en- 71. International Court of Justice is one of the
forceable by the _____ principal organs of the United Nations.
(a) President (b) Prime Minister The statement is
(c) Courts (d) Parliament (a) true (b) false
(c) partly true (d) None of the above
64. The Fundamental Rights guaranteed un-
72. Freedom of movement in India throughout
der the Constitution to the People of India
the territory of India can be restricted in
are not absolute because the Constitution
the interest of
of India imposes
(a) general public or for the protection of
(a) reasonable restrictions on their use
the interest of any Scheduled Castes
(b) arbitrary limitations on their use
and Scheduled Tribes
(c) parliamentary checks on them
(b) protection of cows and birds
(d) statutory restrictions on them
(c) public morality, order and public in-
65. The essential freedoms enjoyed by the
terest
People of India are called the Fundamen- (d) sovereignty and integrity of India and
tal Rights which are enshrined in wild life
(a) Part V of the Constitution of India 73. The Indian Constitution
(b) Part IV of the Constitution of India (a) recognizes ‘natural rights’ but does
(c) Part I of the Constitution of India not embody it
(d) Part III of the Constitution of India (b) leaves no scope for natural rights be-
66. The Chairman and members of State Pub- cause it enumerates the Fundamental
lic Service Commission are appointed by Rights exhaustively
the (c) is flexible about Fundamental Rights
(a) President and Duties
(b) Prime Minister (d) says that the Judiciary will decide the
(c) Governor Fundamental Rights based on natural
(d) Chairman, UPSC justice
67. The Rajya Sabha can be dissolved by 74. The Fundamental Rights as per Constitu-
(a) Lok Sabha tion of India are
(b) Constitutional amendment (a) not justiciable
(c) President (b) justiciable
(d) None of these (c) dependent on the law made by the
68. Who among the following are appointed State
by the President of India? (d) not enforceable at all and it depends
(a) Chairman, finance Commission on the court in each case

CLAT-5-Mock Tests.indd 296 13/09/2010 10:36:28


MOCK TESTS 297

75. When the President of India has to resign, (a) Right to Property
he has to address his resignation letter to (b) Right to Equality
(a) Prime Minister (c) Right to Freedom
(b) Speaker of Lok Sabha (d) None of the above
(c) Vice-President 85. Reservation of seats for Scheduled Castes
(d) Chief Justice of Supreme Court and Scheduled Tribes in the House of Peo-
76. The Constituent Assembly of India took ple has been provided for in the Constitu-
all decisions by tion of India under Article
(a) consensus (a) 330 (b) 331
(b) simple majority (c) 332 (d) 333
(c) two-thirds majority 86. When a bill passed by Parliament is sent to
(d) All the above methods the President for his assent, which option
77. Which of the following can be abolished, is not true to him?
but not dissolved? (a) He may sign it
(a) Rajya Sabha (b) He may decline to sign it
(b) Municipal Bodies (c) He may return it to Parliament for re-
(c) State Legislative Council consideration
(d) None of the above (d) He may amend it
78. How many times can the President of In- 87. Who presides over the Lok Sabha if nei-
dia return a non-money Bill, passed by ther the Speaker nor the Deputy Speaker
parliament? is available?
(a) Twice (b) Once (a) A member of the House of People ap-
(c) Thrice (d) Never pointed by the President
79. Which of the following states does not
(b) A member chosen by Council of Min-
have a bicameral legislature?
isters
(a) Bihar (b) Karnataka
(c) The senior most member of the Rajya
(c) Rajasthan (d) Maharashtra
Sabha
80. The Constitution of India does not men-
(d) The senior most member of the Lok
tion the post of
Sabha
(a) the Deputy Chairman of the Rajya
88. Disputes regarding the election of the
Sabha
Vice-President are decided by
(b) the Deputy Prime Minister
(a) the Supreme Court
(c) the Deputy Speaker of the Lok Sabha
(d) the Deputy Speaker of the State Leg- (b) the Parliament
islative Assemblies (c) the Speaker
81. To be appointed a judge of the Supreme (d) the President
Court of India, a person should have been 89. India is a secular State because
an advocate of a High Court for at least (a) state has no religion of its own
(a) 10 years (b) 12 years (b) all religions are equally protected
(c) 13 years (d) 15 years (c) there will be no discrimination on
82. India is federal, because it has grounds of religion or caste
(a) a bicameral legislature at the centre (d) All the above
(b) an Election Commission 90. The administrators of Union Territories
(c) Supreme Court and High Court are designated as
(d) Prime Minister and Chief Minister (a) Administrators
83. Provision for equal justice and free legal (b) Lieutenant Governor
aid was added to the Constitution of India (c) Chief Commissioners
by (d) All these
(a) 42nd Amendment 91. The ……… decided in May 1949 that In-
(b) 44th Amendment dia should become a Republic
(c) 41st Amendment (a) Union Parliament
(d) None of the above (b) President of India
84. Which among the following is not Funda- (c) Constituent Assembly of India
mental Right under Constitution of India (d) Prime Minister of India

CLAT-5-Mock Tests.indd 297 13/09/2010 10:36:28


298 MOCK TESTS

92. Is there provision in the Indian Constitu- (a) the Vice-President


tion for post of Deputy Prime Minister (b) the Prime Minister
(a) Yes (c) the President
(b) No (d) the Deputy Speaker of the Lok Sabha
(c) Yes, during an emergency 97. India adopted a federal system with a
(d) None of the above strong center from
93. In India a candidate is declared to be using (a) New Zealand (b) Australia
illegal means in elections, if he (c) USA (d) Canada
I. gives presents etc., to the voters, to 98. From which country’s Constitution the
cast their votes in his favour Distribution of Powers between Union
II. threatens the voters to cast their votes and the State have been borrowed?
in his favour (a) Germany (b) British
III. appeals for votes on ground of his (c) US (d) Canada
own religion 99. What is ‘zero hour’?
IV. tries to criticize other candidates (a) When the proposals of the opposition
(a) I, II, III and IV are correct
are considered
(b) I, II and III are correct
(b) When matters of utmost importance
(c) I, III and IV are correct
are raised
(d) II, III and IV are correct
(c) Interval between the morning and af-
94. Election Commission of India is a
ternoon sessions
(a) Constitutional body
(b) Political body (d) When a Money Bill is introduced in
(c) Quasi-Judicial and Quasi-Political the Lok Sabha
body (e) None of these
(d) Both (a) and (b) 100. In the case of a conflict between the Centre
95. There was a Constitutional amendment in and a State in respect of a subject included
India arising out of the Shah Bano case– in the Concurrent List
Muslim Women (Protection of Rights) on (a) the State Law prevails
Divorce Act, 1986 (b) the Union Law prevails
(a) True (b) False (c) the matter is resolved by the Supreme
(c) Partly true (d) None of the above Court
96. The Speaker of the Lok Sabha has to ad- (d) the law which had been passed first
dress his letter of resignation to would prevail

ANSWERS
1. (b) 2. (c) 3. (d) 4. (c) 5. (c) 6. (d) 7. (d) 8. (a) 9. (a) 10. (b)
11. (b) 12. (d) 13. (a) 14. (a) 15. (d) 16. (b) 17. (a) 18. (c) 19. (c) 20. (c)
21. (a) 22. (a) 23. (a) 24. (d) 25. (a) 26. (d) 27. (a) 28. (b) 29. (a) 30. (d)
31. (c) 32. (a) 33. (a) 34. (d) 35. (a) 36. (d) 37. (d) 38. (b) 39. (b) 40. (b)
41. (c) 42. (c) 43. (a) 44. (b) 45. (c) 46. (d) 47. (a) 48. (c) 49. (b) 50. (c)
51. (a) 52. (b) 53. (c) 54. (b) 55. (a) 56. (d) 57. (d) 58. (d) 59. (b) 60. (a)
61. (c) 62. (a) 63. (c) 64. (a) 65. (d) 66. (c) 67. (d) 68. (a) 69. (d) 70. (b)
71. (a) 72. (c) 73. (b) 74. (b) 75. (c) 76. (a) 77. (c) 78. (b) 79. (c) 80. (b)
81. (a) 82. (a) 83. (c) 84. (a) 85. (a) 86. (d) 87. (d) 88. (a) 89. (d) 90. (d)
91. (c) 92. (b) 93. (b) 94. (a) 95. (a) 96. (d) 97. (d) 98. (d) 99. (c) 100. (b)

CLAT-5-Mock Tests.indd 298 13/09/2010 10:36:28


MOCK TESTS 299

PRACTICE PAPER 8
Maximum Marks: 100 Marks 91–100—Outstanding
Time: 30 Minutes Marks 81–90—Excellent
Marks 71–80—Intelligent
Marks 61–70—Good
Marks 51–60—Average
Marks Less than 50—Below Average

1. Principal laid down in a judgment is (a) 5 years (b) 2 years


(a) Res judicata (c) 3 years (d) 6 years
(b) Law 11. Which article of the Constitution disqual-
(c) Ratio decidendi ify the holders of office of profit from be-
(d) Obiter dicta ing the member of Parliament
2. Who is associated with Drafting of Indian (a) 100 (b) 102
Penal Code (c) 110 (d) 120
(a) Ambedkar 12. Decree means
(b) Sir Woodroof and Amirali (a) Informal expression of adjudication
(c) Lord Macaulay (b) Formal expression of an adjudication
(d) Mulla but shall not include any adjudication
3. Culpable homicide is defined in the Indian from which an appeal lies
Penal Code, Section (c) Formal expression of adjudication
(a) 299 (b) 300 (d) All the above
(c) 301 (d) 307 13. Cognizable offence has been defined in
4. Murder is defined in the Indian Penal the Code of Criminal Procedure
Code, Section (a) Under Section 2(a)
(a) 299 (b) 300 (b) Under Section 2(i)
(c) 301 (d) 307 (c) Under Section 2(c)
5. Time limit for filing different kinds of suit (d) Not defined
is prescribed in 14. A Magistrate can direct the investigation
(a) CPC (b) IPC into a cognizable offence by
(c) Limitation Act (d) High Court Rules (a) Any Police officer
6. A person whose guardian is appointed (b) Superior Police officer under Sec.36
during his minority, becomes a major at of Cr. P.C.
the age of ____ years (c) Station House Officer
(a) 31 (b) 21 (d) C.B.I.
(c) 28 (d) 18 15. In a bailable offence, the bail granted as a
7. When a person to be prosecuted for com- matter of right
mitting a criminal offence, the burden of (a) By the court
proof is on (b) By the police officer and the court
(a) Court (b) Police (c) By the police officer
(c) Himself (d) Prosecution (d) Either (a) or (b)
8. “Courts and their Judgments: Promises, 16. Every sentence of death passed by the
Requisites and Consequences” is a book Court of Sessions shall be submitted to the
written by High Court
(a) Vikram Seth (b) Ram Jeth Malani (a) For review
(c) Arun Shourie (d) None (b) For revision
9. Judges for International Court of Justice is (c) For confirmation
elected for……years (d) None of these
(a) 5 (b) 10 17. When any person is sentenced to death,
(c) 9 (d) 6 the sentence shall direct that he be
10. What is the tenure of the President of In- (a) hanged till he is dead
ternational Court (b) hanged by neck till death

CLAT-5-Mock Tests.indd 299 13/09/2010 10:36:28


300 MOCK TESTS

(c) hanged by neck (a) Offence (b) Both


(d) None of these (c) Prosecution (d) None
18. “Roses in December” was written by 29. In which of the following cases the Su-
(a) Arun Shourie (b) M. C. Chagla preme Court has held that right to fair trial
(c) H. R. Khanna (d) Hidayatullah is a part of Article 21 of Constitutions of
19. A police officer cannot investigate a India
______ without the orders of a Magistrate (a) Mohammed Ahmad Khan vs Shah
(a) Non bailable case Bano Begum
(b) Bailable case (b) Joginder Nahak vs State of Orissa
(c) Non cognizable case (c) A. R. Antulay vs R. S. Naik
(d) Cognizable case (d) Samta Singh vs State of Punjab
20. Jurisdiction to grant anticipatory bail un- 30. A person can be summoned as a witness
der section 438 of Cr. P.C. vests with under section 160 of Cr. P.C. by;
(a) The High Court (a) any Police Officer
(b) The Court of Sessions and the High (b) An Investigation Officer
Court (c) The Station House Officer
(c) The Court of Sessions (d) A Magistrate
(d) The Court of Magistrate 31. A sessions Judge can pass
21. A Magistrate can take cognizance of an of- (a) Sentence up to life imprisonment
fence (b) Any sentence
(a) Upon a complaint (c) Any sentence except death sentence
(b) Suo Moto (d) Sentence up to 10 years
(c) Upon a police report 32. In kidnapping, the consent of minor is
(d) Any of the above (a) Partly material
22. Right to speedy trial is guaranteed under (b) Wholly material
(a) Cr. P.C. (c) Wholly immaterial
(b) Article 22 of the Constitution of India (d) Party immaterial
(c) Article 21 of the Constitution of India 33. Use of violence by a member of assembly
(d) In special laws consisting of 12 persons in furtherance of
23. Grievous hurt is defined in section their common object will constitute
(a) 319 (b) 320 (a) Unlawful assembly
(c) 338 (d) 351
(b) Assault
24. Permanent deprivation of the sight of ei-
(c) Rioting
ther eye is an offence of
(d) Affray
(a) Grievous hurt (b) Assault
34. A person who consents, suffer no injury is
(c) Hurt (d) None of these
known as
25. The guidelines have been laid down by the
(a) Sensus verborem est anima legis
Supreme Court regarding the rights of ac-
(b) Actus non facit reum nisi mens sit rea
cused at the stage of arrest in
(c) Res judicata pro veritate accipitur
(a) A. K. Gopalan vs State of Mardar
(d) Volenti non fit injuria
(b) A. R. Antuly vs R. S. Naik
35. Offence of theft is related to
(c) State of Haryana vs Bajanlal
(a) Property only
(d) D. K. Basu vs State of West Bengal
26. The inherent powers of the High Court are (b) Movable property only
contained in (c) Immovable property only
(a) Section 492 of CrPc (d) All of these
(b) Section 472 of CrPc 36. A instigates B to murder D. B refuses to
(c) Section 482 of Cr.Pc do so. Murder did not take place. What of-
(d) Section 462 of Cr.Pc fence is A is guilty of?
27. Which of the following forms part of the (a) No offence
substantive law (b) Murder
(a) Prosecution (b) Offence (c) A abetting B to commit
(c) Both (d) None (d) Conspiracy
28. Which of the following forms part of the 37. The essential ingredients of a crime are
procedural law (a) Knowledge, intention and action

CLAT-5-Mock Tests.indd 300 13/09/2010 10:36:28


MOCK TESTS 301

(b) Motive, intention and knowledge 48. The offence of theft is covered by section
(c) Motive, means rea and actus reus ….IPC
(d) Actus reus and mens rea (a) 393 (b) 378
38. The involvement of five or more persons (c) 365 (d) 324
is essential for 49. A proposal when accepted becomes
(a) Robbery (b) Dacoity (a) Offer (b) Promise
(c) Extortion (d) Theft (c) Contract (d) Agreement
39. Robbery becomes dacoity when commit- 50. A minor’s agreement is void. This propo-
ted conjointly by sition is made in
(a) At least ten persons (a) Nihal Chand vs Jan Khan
(b) More than two persons but less than (b) Mohari Beevi vs Dharmodas Ghosh
five persons (c) Sreekrishnan vs Kurukshethra Uni-
(c) Five persons or more versity
(d) Two persons (d) Nanjappa vs Muthuswamy
40. Homicide literally means 51. ‘Damages’ are awarded except in one
(a) The punishment of death case
(b) Killing of animals (a) Tort
(c) Murder (b) Breach of contract
(d) Killing of a human being by another (c) Libel Suit
human being (d) Criminal conduct
41. What is the punishment in the IPC substi- 52. Deceit is
tuted for ‘transportation for life (a) A tort
(a) Rigorous imprisonment (b) A crime
(b) Death (c) A breach of contract
(c) Imprisonment for life (d) Both (a) and (b)
(d) Fine 53. What is mean by affidavit?
42. Murder is punishable with (a) Evidentiary Document
(a) imprisonment for 10 years (b) Document
(b) imprisonment for life or imprison- (c) Letter of oath
ment for 10 years (d) A request made to the court
(c) death or imprisonment for life 54. The doctrine of stare decisis is a feature of
(d) death
Common Law. This means that
43. Attempt to commit suicide is an offence
(a) the decision stares at the judge
under section of IPC
(b) precedents possess law quality in
(a) 360 (b) 405
themselves and are binding
(c) 309 (d) 300
(c) decision of High Court is ‘Law’ even
44. ‘Dowry death’ has been explained in sec-
through it is not binding on any Court
tion of IPC
except those subordinate to it
(a) 302 (b) 303
(d) None of the above definitions are cor-
(c) 310 (d) 304-B
rect
45. There are____ sections in Indian Penal
55. A ratio decidendi means
Code
(a) The proportion of justice in a case
(a) 155 (b) 716
(c) 511 (d) 617 (b) The principle upon which a case was
46. For conspiracy, the minimum number of decided
persons required is (c) The decisive facts of the case
(a) 5 (d) None of these
(b) 1 56. Central Ordinance is the Law making
(c) 2 power of President of India during
(d) No minimum requirement (a) recess of Parliament
47. Is intention a mental condition? (b) recess of State Legislature
(a) Depends (c) any time
(b) Yes (d) during emergency
(c) None of the above 57. How many times has the President of In-
(d) No dia declared Financial Emergency so far?

CLAT-5-Mock Tests.indd 301 13/09/2010 10:36:28


302 MOCK TESTS

(a) Once (b) Thrice (b) Prime Minister


(c) 4 times (d) Never (c) International Court of Justice
58. Any person in case of violation of funda- (d) None of the above
mental right under the Constitution of In- 67. The Head of a State in India is known as
dia can go to the High Court under (a) Governor General
(a) Article 222 of Constitution (b) Governor
(b) Article 226 of Constitution (c) Chief Minister
(c) Article 220 of Constitution (d) Chief Justice
(d) Article 32 of Constitution 68. The right to vote in India is based on the
59. In the case of violation of fundamental principle of
rights under the Constitution of India any (a) restricted franchise
person can go to Supreme Court (b) hereditary privileges
(a) under Article 332 of the Constitution (c) proprietary qualifications
of India (d) universal adult franchise
(b) under Article 226 of the Constitution 69. ………. is the Commander-in-Chief of the
of India Defence Forces of India
(c) under Article 32 of the Constitution of (a) Prime Minister
India (b) President
(d) under Article 19 of the Constitution of (c) Defence Minister
India (d) Army Chief
60. The Preamble of Constitution of India is 70. The Indian Parliament is based on the
also always subjected for comments by Principle of
Supreme Court for next to be amended. (a) Unicameralism
But how many times the Preamble of Con- (b) Nomination
stitution has been amended so far? (c) Bicameralism
(a) Once (42nd Amendment) (d) Hereditary principle
(b) Twice 71. ….. is an inquiry into cause of death
(c) Thrice (a) Coronation (b) Inquest
(d) 4 times (c) Investigation (d) Subpoena
61. Rape is provided as an offence under I.P.C. 72. Justice is the ………….
Section (a) Means, end (b) Cause, effect
(a) 375 (b) 378 (c) Effect, cause (d) End, means
(c) 403 (d) 405 73. Audi aultrem paltrem means
62. Sexual intercourse by a man with his own (a) No one should be condemned un-
wife is a rape if she is below the age of heard
(a) 18 years (b) 15 years (b) When the thing speaks for itself
(c) 16 years (d) 17 years (c) It is a principle of extradition law
63. Theft is defined in the I.P.C. Section (d) None of these
(a) 375 (b) 377 74. Res ispa loquitur means
(c) 378 (d) 383 (a) No one should be condemned un-
64. Theft is offence against heard
(a) Movable property only (b) When the thing speaks for itself
(b) Immovable property only (c) It is a principle of extradition law
(c) Movable and immovable both (d) None of these
(d) None of these 75. Volenti non fit injuria means
65. What is a ‘congnizance’? (a) No wrong is done to one who con-
(a) Arousing judicial notice or knowl- sents
edge (b) No one should be condemned un-
(b) It is a crime heard
(c) It means custody without warrant (c) When the thing speaks for itself
(d) It means custody with warrant (d) It is a principle of extradition law
66. A judgment passed by the Supreme Court 76. Under which Act in India it was provided
of India can be reviewed only by that women doing similar work to men,
(a) President would qualify for equal pay?

CLAT-5-Mock Tests.indd 302 13/09/2010 10:36:28


MOCK TESTS 303

(a) Protection of Women’s Rights Act, to avoid legal proceed-


1980 ings.
(b) Equal Remuneration Act, 1976 B. Abortion 2. The termination of a
(c) Workmen’s Compensation Act, 1948 pregnancy, a miscar-
(d) Gender Equality Act, 1976 riage or the premature
77. How many members have to support a no- expulsion of a foetus
confidence motion to move in Parliament from the womb before
of India the normal period of
(a) 45 members (b) 50 members gestation is complete.
(c) 100 members (d) 150 members C. Absconding 3. Wrongfully taking
78. Who decides on the holding of elections to away or detaining an-
the Panchayat in India? other person, usually
(a) Central Government by force or fraud.
(b) State Government A B C
(c) Election Commission (a) 3 2 1
(d) Regional Election Commissioner (b) 1 3 2
79. Attorney-General for India shall receive (c) 1 2 3
such remunerations as (d) 2 3 1
(a) prescribed by law 86. Parliament of India cannot amend
(b) determined by the President (a) fundamental rights
(c) determined by the Parliament (b) basic structure of the Constitution
(d) None of the above (c) powers of the Supreme Court
80. What is true with the Speaker of Parlia- (d) None of the above
ment of India in case of equality of votes 87. The Chairman of the Drafting Committee
in joint sitting was
(a) Speaker can cast his vote in case of (a) C. D. Deshmukh
tie (b) B. N. Rau
(b) Speaker cannot cast his vote (c) Dr B. R. Ambedkar
(c) Speaker will permit any member to (d) Jaiprakash Narayan
recast 88. The Constitution of India is
(d) Speaker will request to Prime Minis- (a) wholly written Constitution
ter to cast his vote (b) partly based on judicial decision
81. The Prime Minister of India is (c) unwritten Constitution based on cus-
(a) defecto head toms
(b) de jure head (d) based on ancient Indian traditions ad-
(c) neither (a) nor (b) opted by the Constituent Assembly
(d) real head 89. The country that does not have a written
82. Delegates non potest delegare Constitution is
(a) Delegated legislation cannot be fur- (a) Britain (b) Pakistan
ther delegated (c) Canada (d) India
(b) Delegated legislation is meant to be 90. Which among the following can authorize
further delegated Panchayats to Levy Local Taxes in India?
(c) Neither (a) nor (b) (a) State Legislature
(d) Both (a) and (b) (b) Central Government
83. A person who is capable of wrong is (c) Central Board of Direct Taxes
known as (d) Central Board of Indirect Taxes
(a) doli capax (b) doli incapax 91. Among which of the following case the
(c) neither (a) nor (b) Supreme Court held that, there is no mor-
84. A person who is incapable of wrong is al, fundamental or legal rights or equitable
known as jurisdiction to go on strike by the govern-
(a) doli capax (b) doli incapax ment employees?
(c) neither (a) nor (b) (a) T. K. Rangarajan vs Government of
85. A. Abduction 1. The failure of a person Tamil Nadu
to surrender to the cus- (b) Dwarka Prasad Agarwal vs B.D.
tody of a court in order Agarwal

CLAT-5-Mock Tests.indd 303 13/09/2010 10:36:29


304 MOCK TESTS

(c) Union of India vs Rajiv Kumar, (d) Leader of opposition party


A.I.T.U.C. vs Union of India 97. The Prime Minister of India is the head of
(d) A. K. Puriwar vs T. N. Muthy (a) State
92. In which case, Supreme Court held the (b) Government
Parliament has no power to amend Funda- (c) Both (a) and (b)
mental Rights? (d) None of the above
(a) Golaknath case 98. Who among the following presides over
(b) Sajjan Singh case the meetings of Council of Minister in
(c) Shankri Prasad case Parliament of India?
(d) None of the above (a) President
93. In ….. Supreme Court held that Preamble (b) Prime Minister
is a basic feature of Constitution and can- (c) Cabinet Secretary
not be amended. (d) Vice-President
(a) Keshvanand Bharti case 99. Which among the following Vice-Presi-
(b) Golaknath case dents in India resigned from his office to
(c) Minerva Mills case contest for the post of President?
(d) None of the above (a) N. Sanjeeva Reddy
94. ‘Ultra Vires’ is a term used for (b) V. V. Giri
(a) A document corrupted by virus (c) Fakhruddin Ali Ahmad
(b) An act beyond the authority of a per- (d) R. Venkataraman
son 100. Give response to the true statement
(c) An act authorized by law (a) No court has power to compel the
(d) An illegal act Governor to exercise or not to exer-
95. To betray a nation is an offence and pun- cise any power or to perform or not to
ishable with death, that is perform any duty
(a) Sedition (b) Treachery (b) The Governor cannot be prosecuted in
(c) Treason (d) Anti-nationality a civil and criminal court for any act
96. Prime Minister of India is of omission or commission during the
(a) leader of majority party in the Lok period he holds office
Sabha (c) Both above statements are correct
(b) senior most Member of Lok Sabha (d) Statement (a) is correct while (b) is
(c) President’s nominee not

ANSWERS
1. (c) 2. (c) 3. (a) 4. (b) 5. (c) 6. (b) 7. (d) 8. (c) 9. (d) 10. (c)
11. (b) 12. (b) 13. (a) 14. (b) 15. (b) 16. (c) 17. (a) 18. (b) 19. (c) 20. (b)
21. (d) 22. (c) 23. (b) 24. (a) 25. (d) 26. (c) 27. (b) 28. (c) 29. (c) 30. (d)
31. (b) 32. (c) 33. (d) 34. (d) 35. (b) 36. (c) 37. (c) 38. (a) 39. (a) 40. (d)
41. (a) 42. (c) 43. (c) 44. (d) 45. (c) 46. (c) 47. (b) 48. (b) 49. (d) 50. (b)
51. (d) 52. (d) 53. (c) 54. (b) 55. (b) 56. (a) 57. (d) 58. (b) 59. (c) 60. (a)
61. (a) 62. (c) 63. (c) 64. (a) 65. (a) 66. (d) 67. (b) 68. (d) 69. (b) 70. (c)
71. (c) 72. (d) 73. (a) 74. (b) 75. (a) 76. (d) 77. (c) 78. (d) 79. (b) 80. (a)
81. (a) 82. (a) 83. (a) 84. (b) 85. (a) 86. (b) 87. (c) 88. (a) 89. (a) 90. (a)
91. (a) 92. (b) 93. (a) 94. (b) 95. (c) 96. (a) 97. (b) 98. (b) 99. (b) 100. (c)

CLAT-5-Mock Tests.indd 304 13/09/2010 10:36:29


MOCK TESTS 305

PRACTICE PAPER 9
Maximum Marks: 100 Marks 91–100—Outstanding
Time: 30 Minutes Marks 81–90—Excellent
Marks 71–80—Intelligent
Marks 61–70—Good
Marks 51–60—Average
Marks Less than 50—Below Average

1. What among the following relates to main (b) We, the people of Constituent Assem-
sanction behind the Fundamental Duties bly…. adopt, enact and give to India
in Constitution of India? this Constitution
(a) Legal Sanction (c) We, the citizens of India….adopt, en-
(b) Social Sanction act and give to ourselves this Consti-
(c) Moral Sanction tution
(d) Fundamental Sanction (d) We, the people of India….in our Con-
2. Zonal Councils in India are stituent Assembly…adopt, enact and
(a) law making bodies give to ourselves this constitution
(b) dispute settling agencies 9. The insertion of ‘Liberty, Equality and
(c) administrative bodies Fraternity’ was derived from
(d) advisory bodies (a) American revolution
3. Which is the most active Committee of the (b) French revolution
Cabinet of Government of India (c) Russian revolution
(a) Committee on Economic Affairs (d) None of these
(b) Committee on Political Affairs 10. What is the salary of President?
(c) Committee on Appointments (a) ` 45,000 (b) ` 50,000
(d) None of the above (c) ` 30,000 (d) ` 1,20,000
4. India consists of 11. Which among the following Indian Con-
(a) 21 States (b) 24 States
stitutional Amendments included 4 more
(c) 28 States (d) 25 States
languages in the Eighth Schedule to the
5. Who among the following appoints mem-
Constitution of India?
bers of Union Public Service Commission
(a) 92nd Amendment, 2003
in India?
(b) 91st Amendment, 2003
(a) President of India
(c) 90th Amendment, 2003
(b) Prime Minister of India
(c) Chief Justice of India (d) 89th Amendment, 2003
(d) Law Minister of India 12. The ideals of liberty, equality and frater-
6. The Constituent Assembly for undivided nity enshrined in the Constitution of India.
India first met on This was followed from
(a) 6 December 1946 (a) American Revolution
(b) 9 December 1946 (b) French Revolution
(c) 20 February 1947 (c) 1957 Indian Revolt
(d) 3 June 1947 (d) Russian Revolution
7. Secularism means 13. Constitution of India was inaugurated on
(a) Suppression of all religions 26 January 1950. What was exact status of
(b) Freedom of worship to minorities India on that day?
(c) Separation of religion from State (a) A Democratic Republic
(d) A system of political and social phi- (b) Sovereign Democratic Republic
losophy that does not favour any par- (c) Sovereign, Socialistic, Secular Demo-
ticular religious faith cratic Republic
8. The Preamble to the Indian Constitution (d) Sovereign, Secular, Democratic Re-
reads public
(a) We, the people of India….adopt, enact 14. The languages “Maithili” and “Santhali”
and give to India this Constitution are

CLAT-5-Mock Tests.indd 305 13/09/2010 10:36:29


306 MOCK TESTS

(a) recognized in Constitution of India 22. Residuary power of legislation in India in


(b) not recognized in Constitution of In- the case of Jammu and Kashmir belongs
dia to
(c) recognized in the Citizenship Act, (a) State
1955 (b) Union
(d) recognized in Citizenship Act, 1955 (c) Supreme Court
15. Which among the following is not a con- (d) Election Commission
dition for becoming a citizen of India? 23. In India while imposing reasonable re-
(a) citizenship by descent strictions on the right to assemble peace-
(b) citizenship by birth fully and without arms, one of the follow-
(c) citizenship through acquisition of ing considerations is material
property (a) the security of the President
(d) citizenship by naturalization (b) the morality or integrity of the public
16. Which of the following is Fundamental (c) the interest of sovereignty and integ-
duty of an Indian citizen? rity of India or public order
(a) To cast his vote during election. (d) public interest
(b) To develop scientific temper and spirit 24. Nagar Panchayat in India is called
of inquiry. (a) Municipal institutions for transitional
(c) To work for removal of illiteracy. area
(d) To honour the elected leaders. (b) Panchayat for transitional area
17. The Rajya Sabha is a permanent house (c) Tehsil for the transitional area
but (d) Block headquarter for transitional
(a) one-half of its members retire every area
25. The Lok Adalats in India can deal with the
two years
matter of
(b) one-fifth of its members retire every
(a) pending cases in the courts only
year
(b) new cases in the courts only
(c) one-third of its members retire every
(c) all cases which pending before court,
two years
and new cases come under the juris-
(d) one-half of its members retire every
diction
three years
(d) None of the above
18. The Chairman of the Rajya Sabha
26. How many times has the Preamble of the
(a) is nominated by the President
Indian Constitution been amended so far?
(b) is elected by the members of Rajya (a) thrice (b) twice
Sabha (c) once (d) never
(c) is elected by the members of Lok Sab- 27. The basic condition imposed on the citi-
ha and Rajya Sabha jointly zen’s right to assemble is that the assem-
(d) None of the above bly should be
19. In which of the following points is the In- (a) peaceful
dian Constitution similar to that of USA? (b) non-violent
(a) Rigid Constitution (c) peaceful and unarmed
(b) Rule of Law (d) for constructive aims
(c) Fundamental Rights 28. Who is the longest serving Chief Minister
(d) Directive Principles of State Policy in India?
20. Disputes regarding the election of the (a) Bhajan Lal
Vice-President are decided by (b) Hiteshwar Saikia
(a) the Supreme Court (c) Chimanbhai Patel
(b) the Parliament (d) Jyoti Basu
(c) the Speaker 29. The Constitution of India vests the execu-
(d) the President tive power of the Indian Union in
21. The electorate for a Panchayat election in (a) the President
India is the (b) the Parliament
(a) Tehsil (b) Panchayat Samiti (c) the Council of Ministers
(c) Gram Sabha (d) None of the above (d) the Prime Minister

CLAT-5-Mock Tests.indd 306 13/09/2010 10:36:29


MOCK TESTS 307

30. Vote on account is made by the Lok Sab- 35. On what basis Lok Sabha seats in Par-
ha liament of India are allotted in various
(a) to meet expenditure on unexpected states?
demand (a) population (b) territory
(b) to meet expenditure on secret service (c) poverty line (d) literacy
(c) to meet the additional expenditure in- 36. When can a President refer a matter to the
curred by the government on any ap- Supreme Court for its opinion?
proved item of expenditure during the (a) When the President feels like it
supplementary budget (b) When a matter is of public impor-
(d) to meet expenditure for a period be- tance
fore passing of the annual budget (c) When the Cabinet is not functioning
31. Article 352 of Constitution of India pro- properly
vides for (d) When emergency has to be promul-
(a) National Commission for Scheduled gated
Castes and Scheduled Tribes 37. Who is entitled to initiate a Bill for Consti-
(b) Proclamation of emergency tutional amendments?
(c) Special provision for Anglo-Indian (a) State Legislative Assemblies
community in certain services (b) Lok Sabha only
(d) Appointment of a Commission to in- (c) Either House of Parliament
vestigate the conditions of backward (d) Rajya Sabha only
classes 38. The words ‘social, secular’ and ‘the uni-
32. Article 24 of Constitution of India prohib- ty and integrity of the nation’ have been
its employment of children in factories. added to the Indian Constitution by which
According to it amendment?
(a) children below 14 years of age shall (a) 38th (b) 40th
not be employed in a coal mine or fac- (c) 42nd (d) 48th
tory 39. By which amendment was the power of
(b) a child below 14 years of age shall not the Lok Sabha to amend the Constitution
be employed to work in any hazard- affirmed?
ous employment (a) 24th (b) 42nd
(c) a child below 14 years shall not be (c) 43rd (d) 39th
employed in a factory or a mine or any 40. At present the Supreme Court consists of
hazardous employment how many judges besides the Chief Jus-
(d) Both (a) and (b) tice?
33. Right to Property in India is a (a) 20 (b) 18
(a) Fundamental Right (c) 24 (d) 25
(b) Legal right and Constitutional Right 41. Article 30 (1) of Constitution of India the
(c) Fundamental Duty right to establish and administer educa-
(d) None of the above tional institution belongs to
34. As regards the impeachment of the Presi- (a) religious and linguistic minorities
dent of India (b) minorities as well as majorities
(a) any citizen can prefer the charge (c) all citizens of India
(b) any charge can be preferred only by (d) all people of India
a resolution moved by not less than 42. The word ‘practice’ mentioned in Article
one-half of the total number of the 25 of Constitution of India does not in-
members of the House straightway clude
(c) a resolution moved after, at least, (a) propagation (b) religious worship
fourteen days’ notice in writing signed (c) rituals (d) observations
by not less than one-fourth of the total 43. The Council of Minister remains in office
members of the House and passing of as long as it enjoys the confidence of
such resolution by a majority of not (a) President of India
less than two-thirds of the total mem- (b) Parliament of India
bership of the House (c) Lok Sabha of Parliament of India
(d) None of the above (d) Prime Minister of India

CLAT-5-Mock Tests.indd 307 13/09/2010 10:36:29


308 MOCK TESTS

44. Which of the following are included in the (a) Contract of guarantee
concept of ‘the State’ under Article 12 of (b) Contract of indemnity
Constitution of India (c) Contingent contract
(a) Railway Board and Electricity Board (d) Unlawful Contract
(b) University 53. In Unnikrishnan vs Calicut University,
(c) Judiciary Unnikrishnan, a candidate for the L.L.B.
(d) All the above Part-I exam, who was short of attendance,
45. ‘Court fee’ taken by High Court in India did not mention that fact himself in the ad-
comes under mission form for the examination. Neither
(a) Union List of Constitution of India the Head of the Law Department nor the
(b) State List of Constitution of India University authorities made proper scru-
(c) Concurrent List of Constitution of In- tiny to discover the truth. It was held by
dia the Supreme Court that
(d) Decided by the Supreme Court of In- (a) There was fraud by the candidate
dia (b) There was misrepresentation by the
46. Who heads the new Commission on Cen- candidate
tre–State relations appointed by the gov- (c) There was no fraud by the candidate
ernment in April 2007? (d) None
(a) M. M. Punchhi 54. A proposal can be accepted
(b) M. Veerappa Moily (a) By performance of condition
(c) K. C. Pant (b) By notice
(d) N. R. Madhav Menon (c) By acceptance of consideration for a
47. In 2006 President Abdul Kalam sent back reciprocal promise
a bill passed by Parliament for reconsid-
(d) All the above
eration. The Bill related to
55. A tender is
(a) reservations for the OBCs
(a) An agreement
(b) 56 offices of profit
(b) A proposal
(c) state funding of elections
(c) An invitation to offer
(d) None of the above
(d) An offer
48. Which one of the following states is re-
56. The person accepting the proposal is
garded as the pioneer of the Right to In-
called
formation in India?
(a) Promisor (b) Promise
(a) Rajasthan
(b) West Bengal (c) Acceptor (d) Offered
(c) Himachal Pradesh 57. A contracts to pay to B ` 1,000/- if B’s
(d) Kerala house is burnt. This is a
49. Under whose Prime Ministership was the (a) Voidable contract
anti-defection law passed? (b) Wagering contract
(a) Rajiv Gandhi (b) Indira Gandhi (c) Contingent contract
(c) V. P. Singh (d) A. B. Vajpayee (d) None
50. What is ‘cognizance’? 58. Ultra vires implies
(a) Arousing judicial notice or knowl- (a) a writ
edge (b) pending before court
(b) It is a crime (c) Beyond one’s power
(c) It means custody without warrant (d) Beyond one’s knowledge
(d) It means custody with warrant 59. “Law is an ass” who said this?
51. When the consent of a party to a contract (a) Locke
has been obtained by undue influence, (b) Marx
fraud or misrepresentation the contract is (c) Charles Dickens
(a) Legal (b) Void (d) Abrahim Lincoln
(c) Voidable (d) Enforceable 60. The Right to Information Act, 2005 re-
52. “A contract to do or not to do something, pealed:
if some event collateral to such contract (a) The Freedom of Information Act,
does or does not happen” is called 2005.

CLAT-5-Mock Tests.indd 308 13/09/2010 10:36:29


MOCK TESTS 309

(b) The Freedom of Information Act, (b) Bankers Book of evidence


2000. (c) Negotiable Instruments Act
(c) The Freedom of information Act, (d) None of these
2003. 71. Innuendo hints at
(d) None of these (a) Slander (b) Defamation
61. Transfer of Property Act 1882 covers (c) Insuniation (d) Libel
(a) Movable property 72. Which court nullified Mrs Indira Gandhi’s
(b) Immovable property election to Lok Sabha in 1975?
(c) Both (a) and (b) (a) Supreme Court of India
(d) None of the above (b) Allahabad High Court
62. Sarada Act is called (c) Delhi High Court
(a) Domestic Violence Act (d) Bombay High Court
(b) Special Marriage Act 73. The Shah Bano case related to
(c) Dowry Prohibition Act (a) Muslim wife’s right to divorce
(d) Child Marriage Restraint Act (b) Muslim wife’s right to husband’s
63. Which Parliamentary Committee in In- property
dian system is chaired by the member of (c) Muslim wife’s right to maintenance
opposition party? after divorce
(a) Joint Parliamentary Committee (d) Muslim wife’s right to separation
(b) Public Accounts Committee 74. Which of the following are associated
(c) Estimates Committee with the process of planning in India?
(d) Privileges Committee 1. Indian Parliament
64. A sentence of death by a lower court 2. Planning Commission
(a) Must be confirmed by the President 3. National Development Council
(b) Must be confirmed by Supreme Court 4. Finance Commission
(c) Must be confirmed by High Court (a) 1 and 2 (b) 1 and 4
(d) Is operational if no appeal is made to (c) 2 and 3 (d) 2 and 4
higher courts 75. Which one of the following motions is re-
65. All plans, formulated by Planning Com- lated with the Union Budget?
mission must be approved by (a) Adjournment motion
(a) President (b) Cut motion
(b) Parliament (c) Censure motion
(c) National Development Council (d) None of the above
(d) Union Cabinet 76. The House of Peoples (Lok Sabha) can be
66. What is a caveat? adjourned sine die by
(a) Writ (b) Certiorari (a) The Speaker
(b) A warning (d) An injunction (b) The Prime Minister
67. A person who makes an affidavit or depo- (c) The President
sition in called (d) None of the above
(a) Plaintiff (b) Witness 77. Which of the following is not true about
(c) Deponent (d) Defendant the Supreme Court?
68. Deceit is (a) It has only the Appellate jurisdiction
(a) A crime (b) It is the highest federal court of India
(b) A tort (c) It does not have the Original jurisdic-
(c) Both (a) and (b) tion
(d) A breach of contract (d) It can amend the Constitution
69. ‘Stay Order’ has the effect of 78. Which of the following is correct regard-
(a) Pro rata ing the Governor of a State?
(b) Status quo (a) No Money Bill can be introduced in
(c) stopping an order of lower court from the State Legislature without his prior
working permission
(d) sine die (b) He has the power of issuing ordinanc-
70. Cheque is defined in es when the Legislature is not in ses-
(a) Banking Regulation Act sion

CLAT-5-Mock Tests.indd 309 13/09/2010 10:36:29


310 MOCK TESTS

(c) He can recommend to the President to (a) The two Houses of the Parliament
impose President’s Rule in the State pass a Bill by 2/3rd majority
(d) All of the above (b) The President gets a Bill passed by the
79. Is the Prime Minister bound to advise the simple majority of the State Legisla-
President on matters on which his advice ture
is sought? (c) The President calls for the views of
(a) Yes the State and then the two Houses of
(b) No Parliament pass a Bill to this effect by
(c) It is discretionary simple majority
(d) Yes, if the Council of Ministers so de- (d) None of these
sires 86. The President can make laws through or-
80. Public Interests Litigation (PIL), is one in dinances
(a) only on subjects contained in the Con-
which
current List
(a) trial is held ‘in camera’
(b) on certain subjects even when Parlia-
(b) proceedings of the trial are not report-
ment is in session
ed to the press (c) during the recess of the Parliament
(c) court gives precedence to public inter- (d) under no circumstances
est over the points of law 87. Which one of the following committees
(d) issues of public interest are involved does not consist of any member from the
and can be brought before the court by Rajya Sabha?
any citizen (a) Estimates Committee
81. In case of which of the following the meth- (b) Public Accounts Committee
od of removal from office is the same? (c) Public Grievances Committee
I. A Judge of the Supreme Court (d) Committee on Public Undertakings
II. Chief Election Commissioner 88. What is the minimum age for a person to
III. President of India become eligible for the post of Prime Min-
(a) I and II (b) II and III ister of India?
(c) I and III (d) I, II and III (a) 21 years (b) 25 years
82. The Fundamental duties are (c) 30 years (d) 35 years
(a) enforceable at the pains of punish- 89. Supreme Court can grant ‘special leave’
ment ‘to appeal against decisions of lower
(b) not enforceable courts and tribunals by virtue of
(c) useless (a) Section 10 of the Supreme Court
(d) non-enforceable, but of educative val- Rules
ue (b) Article 136 of the Constitution
83. A person who has held office as a Judge of (c) Article 226 of the Constitution
the Supreme Court (d) None of the above
(a) can start legal practice in any court in 90. Members of the Legislative Council (Vid-
han Parishad) are
India after his retirement
(a) directly elected
(b) can start legal practice in Supreme
(b) chiefly nominated
Court only after his retirement
(c) elected by local bodies, graduates,
(c) can start legal practice in a High Court
teachers and partly nominated by the
only after his retirement Governor
(d) cannot start legal practice in any court (d) elected by the State Legislative As-
in India after his retirement sembly
84. The salaries and allowances of Union 91. One of the following is not a feature of a
Ministers are determined by Money Bill:
(a) President of India (a) It must be certified by the Speaker of
(b) Finance Commission the Lok Sabha
(c) Parliament (b) Prior permission of the President is
(d) Ministry of Parliamentary Affairs necessary for introducing it
85. What procedure is followed if a new State (c) It can be introduced in Lok Sabha
is to be formed by separation of territory only
from any State of the Indian Union? (d) It can be stalled by Rajya Sabha

CLAT-5-Mock Tests.indd 310 13/09/2010 10:36:29


MOCK TESTS 311

92. All cases regarding the interpretation of 4. Booth capturers can be arrested and
the Constitution can be brought to the Su- detained for 2 years
preme Court under its Which of the above statements are true?
(a) Appellate Jurisdiction (a) 1, 2 and 3 (b) All the four
(b) Advisory Jurisdiction (c) 2 and 3 (d) 1, 2 and 4
(c) Original Jurisdiction 97. Under what heading do the salaries of
(d) None of the above Government servants come in the budget?
93. Which Article of the Constitution stipu- (a) Capital outlay
lates the establishment of a Public Service (b) Plan expenditure
Commission for the Union and each of the (c) Current expenditure
States? (d) None of the above
(a) Article 360 (b) Article 315 98. The right to vote in an election is
(c) Article 395 (d) None of the above (a) Constitutional (b) Natural
94. The provisions of Nagar Palika Act are (c) Statutory (d) Moral
contained in
99. Which of the following statements would
(a) Tenth Schedule
aptly describe the true nature of the Direc-
(b) Eleventh Schedule
tive Principles of State Policy?
(c) Twelfth Schedule
(a) Negative Injunctions
(b) None of the above
95. Finance Commission consists of (b) Positive Instructions
(a) A Chairman and four members (c) Utopian Ideals
(b) Five members (d) None of these
(c) Chairman and three members 100. The Speaker of the Lok Sabha will vacate
(d) None of the above his office
96. Consider the following statements about (a) as soon as the Lok Sabha is dissolved
booth capturing: (b) when the President asks him to va-
1. Removing of boxes after electorates cate
have voted (c) immediately before the first meeting
2. Preventing genuine persons from vot- of the newly elected Lok Sabha
ing (d) when a vote of no-confidence is passed
3. Attacking the opposition candidate against the ruling party

ANSWERS
1. (c) 2. (d) 3. (b) 4. (c) 5. (a) 6. (b) 7. (d) 8. (d) 9. (b) 10. (d)
11. (a) 12. (b) 13. (b) 14. (a) 15. (c) 16. (b) 17. (c) 18. (b) 19. (c) 20. (a)
21. (c) 22. (a) 23. (d) 24. (a) 25. (c) 26. (c) 27. (c) 28. (d) 29. (a) 30. (a)
31. (b) 32. (b) 33. (b) 34. (c) 35. (a) 36. (b) 37. (c) 38. (c) 39. (a) 40. (d)
41. (a) 42. (d) 43. (c) 44. (d) 45. (b) 46. (a) 47. (b) 48. (b) 49. (a) 50. (a)
51. (b) 52. (c) 53. (c) 54. (d) 55. (c) 56. (b) 57. (c) 58. (c) 59. (c) 60. (b)
61. (b) 62. (b) 63. (b) 64. (c) 65. (c) 66. (b) 67. (d) 68. (c) 69. (c) 70. (c)
71. (b) 72. (b) 73. (a) 74. (c) 75. (b) 76. (a) 77. (c) 78. (d) 79. (a) 80. (d)
81. (d) 82. (d) 83. (d) 84. (c) 85. (c) 86. (c) 87. (b) 88. (b) 89. (a) 90. (c)
91. (d) 92. (c) 93. (b) 94. (c) 95. (b) 96. (b) 97. (c) 98. (a) 99. (b) 100. (c)

CLAT-5-Mock Tests.indd 311 13/09/2010 10:36:29


312 MOCK TESTS

PRACTICE PAPER 10
Maximum Marks: 100 Marks 91–100—Outstanding
Time: 30 Minutes Marks 81–90—Excellent
Marks 71–80—Intelligent
Marks 61–70—Good
Marks 51–60—Average
Marks Less than 50—Below Average

1. ‘Ultra Vires’ is a term used for 8. “In any event, whatever system of Gov-
(a) A document corrupted by virus ernment we may establish here must fit in
(b) An act beyond the authority of a per- with the temper of the people and be ac-
son ceptable to them”. In was said by
(c) An act authorized by law (a) Jawahar Lal Nehru
(d) An illegal act (b) Dr B. R. Ambedkar
2. To betray a nation is an offence and pun- (c) D. D. Basu
ishable with death, that is (d) M. V. Pylee
(a) Sedition (b) Treachery 9. The expression ‘every person’ in Article 5
(c) Treason (d) Anti-nationality of Constitution of India, includes
3. The term res subjudice means (a) prisoner
(a) A case has been decided by a court (b) member of armed forces
(b) A case is pending before a court (c) persons living within the territory of
(c) A case has been dismissed by a court India
(d) None of the above (d) All the above
10. The term “law in force” in clause (3) of
4. The Supreme Court in….case held that
Article 13 of Constitution of India means
Article 21 includes right to die
(a) only statutory law
(a) P. Rathinam/NagbhusanPatnaik vs
(b) customs and usages only
Union of India
(c) both (a) and (b)
(b) Dr M. Ismail Faruqui vs Union of In-
(d) newly formed public order only
dia
11. The constitution gives the power of super-
(c) K. Karunakaran vs State of U.P. intendence over all subordinate courts to
(d) None of the above the High Courts under Article
5. Public holidays are declared under (a) 229 (b) 226
(a) Negotiable Instruments Act, 1881 (c) 226 (d) 227
(b) Contract Act, 1872 12. The High Courts in India were first started
(c) Public Employees Act, 1967 at
(d) None of the above (a) Madras and Bombay
6. The solemn resolution in the Preamble of (b) Bombay, Madras, Calcutta
Constitution of India is made in the name (c) Bombay, Delhi, Madras
of (d) Delhi and Calcutta
(a) People of India 13. The High Court enjoys the power
(b) Constituent Assembly of free India (1) To issue writs for the enforcement of
(c) Constitution of India Fundamental rights
(d) Indian Independence Act, 1947 (2) To exercise superintendence over the
7. ‘Fraternity’ means a ‘sense of common working of courts and tribunals under
brotherhood to all Indians’ and is the prin- its jurisdiction
ciple which gives unity and solidarity to (3) To make general rules and prescribe
social life. It was said by forms regulating the practices and
(a) Dr Rajendra Prasad proceeding of courts under its juris-
(b) Dr B. R. Ambedkar diction
(c) Jawahar Lal Nehru (a) (1) and (3) (b) (1), (2) and (3)
(d) C. Rajagopalchari (c) (1) and (2) (d) (2) and (3)

CLAT-5-Mock Tests.indd 312 13/09/2010 10:36:30


MOCK TESTS 313

14. The High Court which has the distinction 21. The residuary powers mean the
of having the first woman Chief Justice is (a) powers shared between the Union and
(a) Guwahati High Court State Government
(b) Delhi High Court (b) powers related to country’s internal
(c) Himachal Pradesh High Court affairs
(d) Allahabad High Court (c) powers related to country’s external
15. A retired judge of High Court cannot affairs
(a) practice in any Court of India (d) powers not included in any of the
(b) practice in any High Court of India three lists
(c) practice in the High Court from where 22. By which constitutional amendment
he has retired Panchayati Raj was introduced?
(d) practice in the Supreme Court (a) 74th (b) 73rd
16. The Judges of the High Court can be re- (c) 42nd (d) 44th
moved from their office during their ten- 23. From which country’s Constitution the
ure Right to Equality have been borrowed?
(a) by the Chief Justice on the advice of (a) Irish (b) Canada
the President (c) British (d) US
(b) by the Chief Justice on the recommen- 24. From which country’s Constitution the Ju-
dation of the Parliament dicial Review have been borrowed?
(c) by the Governor, if the State Legisla- (a) Irish (b) Canada
ture passes a resolution to this effect (c) British (d) US
by two-thirds majority 25. Which article defines Equality before law
(d) by the President on the basis of a reso- and equal protection of law?
lution passed by the Parliament by (a) Article 16 (b) Article 14
two-thirds majority (c) Article 20 (d) Article 21
17. A judicial Review means: 26. Parliament or a State Legislature in India
(a) Powers of the courts to declare null can declare a seat vacant, if a member ab-
and void any action by any organ of sents himself without notice from the ses-
the Government if it is beyond the sion for more than
powers granted by the Constitution (a) 30 days (b) 60 days
(b) Review of the working of judiciary (c) 90 days (d) 120 days
(c) Power of the Court to try cases 27. The right to vote in election in India is
(d) Review of the judicial power of the (a) constitutional (b) natural
judges (c) statutory (d) moral
18. When the Vice-President acts as President 28. Match the following
he gets the emoluments of I. Ad Verbatim
(a) Vice-President II. Audi Alterm Partem
(b) President III. Pro Bono Publico
(c) President in addition to what he gets IV. Res Ipsa Loquitor
as Chairman of Rajya Sabha A. Facts speak for themselves
(d) Chairman of Rajya Sabha B. For the public
19. The executive power is vested in the Pres- C. To hear
ident but it is actually used by him on the D. Word to word
advice of I II III IV
(a) The Council of Ministers (a) D C B A
(b) Parliament (b) A B C D
(c) The Prime Minister (c) B A C D
(d) None of these (d) A C B D
20. What is the power of the Rajya Sabha with 29. The Contingency Fund of India has been
regard to Money Bill? placed at the disposal of the
(a) It can withhold it (a) President
(b) It has no power (b) Prime Minister
(c) It can amend it (c) Finance Minister
(d) It can reject it (d) Governor, RBI

CLAT-5-Mock Tests.indd 313 13/09/2010 10:36:30


314 MOCK TESTS

30. The grounds under which the President of (a) I and III (b) II and III
India may remove a Governor from office (c) I and IV (d) I, III and IV
are 37. The Advisory opinion tendered by the Su-
(a) laid down in the Constitution preme Court
(b) not laid down in the Constitution (a) is not made public at all
(c) governed by Conventions (b) is binding on the President only if it is
(d) governed by Supreme Court Guide- unanimously made
lines (c) is binding on the President
31. Which of the following provisions of the (d) is not binding on the President
Constitution can be amended by Parlia- 38. The phrase ‘bicameral legislature’ means
ment by a simple majority? (a) Parliamentary system of Government
(a) Provision concerning establishment (b) a legislature consisting of lower and
of High Court an upper chamber
(b) Provision concerning federal judicia- (c) a single assembly
ry (d) an elected legislature
(c) Provision concerning representation 39. The Legislative Council is expected to
of States in Parliament have a strength equal to one third of the
(d) None of the above membership of the Legislative Assembly.
32. Who, among the following, acts as the The minimum strength of the Council has
chairman of the National Integration to be
Council? (a) 50 (b) 40
(a) the Prime Minister (c) 100 (d) 30
(b) the Vice-President 40. Which one of the following does not take
(c) the Union Home Minister part in the election of the President?
(d) the Chief Justice of India (a) elected members of Lok Sabha
33. The Finance Commission submits its re- (b) members of the legislative councils
port to (c) elected members of the Rajya Sabha
(a) the President (d) None of the above
(b) the Speaker of the Lok Sabha 41. Economic Planning is a subject in
(c) the Comptroller and Auditor-General (a) the Union List
(d) None of the above (b) the State List
34. In India, new All India Services can be (c) the Concurrent List
created (d) None of the above lists
(a) through an amendment to the Consti- 42. Originally the Constitution recognized
tution ………. languages
(b) by the Union Government in consulta- (a) 2 (b) 14
tion with the UPSC (c) 15 (d) none
(c) by the Parliament 43. Which one of the following Presidents of
(d) by the President on the advice of the India is known as Philosopher-President?
Union Council of Ministers (a) Dr Rajendra Prasad
35. Who is the Chairman of the National De- (b) Dr Zakir Hussain
velopment Council? (c) Dr S. Radhakrishnan
(a) The President (d) Dr Fakhruddin Ali Ahmed
(b) The Vice-President 44. The UPSC conducts examinations for
(c) The Prime Minister I. Central Services
(d) The Planning Minister II. All India Services
36. The Chief Justice of a High Court is ap- III. State Services
pointed by the President in consultation IV. State services for two or more States
with jointly
I. Chief Justice of the Supreme Court. (a) I, II and III (b) II and IV
II. Former Chief Justice of the High (c) I only (d) I and II
court 45. The main sources of law in India are
III. Other Judges of the High court I. The Constitution
IV. Governor of the State concerned II. Statues

CLAT-5-Mock Tests.indd 314 13/09/2010 10:36:30


MOCK TESTS 315

III. Customary law (c) If Parliament decrees it by two-thirds


IV. Judicial decisions of superior courts majority
(a) I and II (b) I, II and IV (d) During a Financial Emergency
(c) II and IV (d) I, II, III and IV 55. The Chief Justice and other Judges of the
46. What is the qualification for obtaining vot- High Court are appointed by the
ing right in India? (a) Chief Minister of the concerned State
(a) Caste (b) Property (b) Governor of the concerned State
(c) Age (d) Education (c) President
47. “Equal pay for equal work for both males (d) Chief Justice of the Supreme Court
and females”. Which article has proposed 56. If the Speaker of a State Legislative As-
this? sembly in India resigns, he has to submit
(a) Article 48 (b) Article 39 his resignation to the
(c) Article 14 (d) Article 25 (a) Deputy Speaker of the Assembly
48. The first leader of opposition to be given (b) Chief Justice of the concerned High
the status and rank of a Cabinet Minister Court
was (c) President of India
(a) A. K. Gopalan (b) Y. B. Chavan (d) Governor of State
(c) C. M. Stephen (d) P. Upendra 57. The Deputy Chairman of the Council of
49. The three languages included in the Eighth States in India can be removed by
Schedule of the Constitution by the 71st (a) the Chairman of the Council of States
amendment are (b) the Parliament (both Houses)
(a) Konkani, English, Manipuri (c) a resolution of the Council of States
(b) Konkani, Nepali, Maithili by a majority of the then members of
(c) Konkani, Manipuri, Nepali the House provided, at least, 14 days’
(d) Manipuri, Nepali, Dogri notice has been given of the intention
50. Which of the following is not a condition to the resolution
for becoming a citizen of India? (d) None of the above
(a) Birth 58. The Attorney-General for India shall hold
(b) Acquiring Property office during the pleasure of the
(c) Descent (a) Prime Minister and his Cabinet
(d) Naturalization (b) Parliament
51. The age of retirement of the judges of (c) President
High Court is …..... years (d) None of the above
(a) 65 (b) 58 59. When the President of India gives assent
(c) 62 (d) 60 to a law passed by State Legislature after
52. A High Court consists of Chief Justice being reserved for his consideration and
and then Parliament passes a law on the same
(a) Such other judges as determined by subject then
Parliament (a) State law will prevail
(b) Such other judges as determined by (b) Central law will prevail
the President (c) President’s Ordinance will prevail
(c) Such other judges as specified in the (d) No law will prevail
Constitution 60. In India the historically important objects
(d) At least 5 other judges are to be protected
53. Fiduciary relationship means, the relation- (a) as it is the duty of the State
ship is based on (b) as it is the Fundamental Duty
(a) Blood relation (b) Money (c) as it is a Directive Principle of State
(c) Trust (d) Contract Policy
54. When can the salaries of High Court judg- (d) for none of the above reasons
es be reduced 61. The person appointed to the post of Judge
(a) At no time of High Court takes an oath before
(b) If the State Legislature passes a law to (a) The Chief Justice of the High Court
the effect (b) The Chief Justice of India

CLAT-5-Mock Tests.indd 315 13/09/2010 10:36:30


316 MOCK TESTS

(c) The President of India (c) Chief Minister


(d) The Governor of the State or some (d) Chief Justice
person appointed by him for that pur- 68. The right to vote in India is based on the
pose principle of
62. Which of the following features makes (a) restricted franchise
our Constitution Unitary? (b) hereditary privileges
(a) Emergency Powers of the Centre (c) proprietary qualifications
(b) Residuary Powers with the Union Par- (d) universal adult franchise
liament 69. ………. is the Commander-in-Chief of the
(c) Union Government’s power to issue Defence Forces of India
directions to the State Governments (a) Prime Minister
(d) All the above (b) President
63. Which one of the following statements (c) Defence Minister
correctly describes the Fourth Schedule of (d) Army Chief
the Constitution of India? 70. The Indian Parliament is based on the
(a) It lists the distribution of powers be- Principle of
tween the Union and the States (a) Unicameralism
(b) It contains the languages listed in the (b) Nomination
Constitution (c) Bicameralism
(c) It contains the provisions regarding (d) Hereditary principle
the administration of tribal areas 71. How long did the Constituent Assembly
(d) It allocates seats in the Council of take to finally pass the constitution?
(a) about 6 months in 1949
States
(b) exactly a year since 26 November
64. Which of the following emergencies have
1948
been declared the maximum number of
(c) about 2 years since 15 August 1947
times in India?
(d) about 3 years since 9 December 1946
(a) Emergency under Article 352 due to
72. Who was the Chairman of the Constituent
external aggression
Assembly?
(b) Emergency under Article 352 due to
(a) Pt Jawaharlal Nehru
armed rebellion
(b) Dr Rajendra Prasad
(c) Financial Emergency under Article
(c) Dr B. R. Ambedkar
360
(d) C. Rajagopalachari
(d) Emergency under Article 356 due to 73. India became a Sovereign, democratic re-
breakdown of constitutional machin- public on
ery in a State (a) 15 August 1947
65. For raising a half-an-hour discussion in a (b) 30 January 1948
House of Parliament, the notice in writing (c) 26 January 1950
has to be given to the (d) 26 November 1929
(a) Presiding Officer of the House 74. The Constitution names our country as
(b) Secretary-General of the House (a) Bharat
(c) Secretary of Department of Parlia- (b) India, that is Bharat
mentary Affairs (c) Hindustan
(d) Minister concerned (d) Aryavarta
66. A judgment passed by the Supreme Court 75. The Comptroller and Auditor-General can
of India can be reviewed only by be removed from his office before expiry
(a) President of his term by
(b) Prime Minister (a) the Prime Minister
(c) International Court of Justice (b) the President at his discretion
(d) None of the above (c) the President on the advice of the
67. The Head of a State in India is known as Council of the Ministers
(a) Governor General (d) the President on the recommendation
(b) Governor of the Parliament

CLAT-5-Mock Tests.indd 316 13/09/2010 10:36:30


MOCK TESTS 317

76. To whom the member of Lok Sabha in the (a) Political Review
Parliament of India submits his resigna- (b) Judicial Review
tion (c) Legal Review
(a) President (b) Prime Minister (d) All the above
(c) Speaker (d) Vice-President 85. Jyoti Basu stepped down from the post of
77. A Parliament of India can make any Law Chief Minister in West Bengal on 29 Oc-
for the whole or any part of India for im- tober 2000 after serving as Chief Minister
plementing international treaties for a record period of
(a) without the consent of any State (a) 21 years (b) 23 years
(b) without the consent of Rajya Sabha (c) 24 years (d) 19 years
(c) with consent of 2/3 of States 86. The Constitution of India is
(d) with consent of all States (a) flexible
78. Right to education in India is a (b) rigid
(a) Fundamental Right and Statutory (c) partly rigid and partly flexible
Right (d) constant in number of Articles and
(b) Directive Principle of State Policy flexible in Parts
(c) Legal Right 87. The Fundamental duties of Constitution
(d) None of the above of India, were added in the Constitution in
79. Power to declare an area as scheduled area year
in India depends on (a) 1976 (b) 1979
(a) President (b) Prime Minister (c) 1983 (d) 1992
(c) Governor (d) Chief Minister 88. The Vice-President of India discharges the
80. Which among the following Articles of duties of President in the event of
the Constitution of India empowers the (a) absence due to illness
President to set up the Finance Commis- (b) his death
sion? (c) his resignation
(a) Article 280 (b) Article 291 (d) All of the above
(c) Article 291 (d) Article 356 89. The Vice-President of India is elected by
81. Who was the first Chairman of the Plan- (a) members of Rajya Sabha
ning Commission? (b) members of Lok Sabha
(a) Jawaharlal Nehru (c) elected by Electoral College
(b) Gulzari Lai Nanda (d) directly elected by Lok Sabha and Ra-
(c) K. C. Neogy jya Sabha at a joint sitting
(d) Shriman Narayanan 90. In respect of citizenship the Constitution
82. The Solicitor-General of India is of India provides for
(a) a legal advisor (a) double citizenship
(b) an administrative officer (b) single citizenship
(c) legal advisor of the President of India (c) multiple citizenship
(d) an advisor to Prime Minister (d) no restriction is provided
83. Which of the following correctly repre- 91. The Vidhan Sabha is
sents the value of vote of a Member of (a) the permanent House of State Legisla-
Legislative Assembly in the Presidential ture
election? (b) indirectly elected
(a)
Population of the State
×1000 (c) subject to dissolution
Total no. of members in the State (d) has little importance in the State Gov-
Legislative Assembly
ernment
(b) Population of the State
÷1000 92. That the Prime Minister acts as a channel
Total no. of elected members in the of communication between the President
State Legisslative Assembly and the Council of Ministers is provided
Total no. of elected members in the by
(c)
State Legislative Assembly
×1000 (a) convention (b) statute
Total no. of Lok Sabha Constituencies (c) Article 78 (d) Article 75
(d) None of these 93. An ordinance can be issued by the Presi-
84. Which one of the following power is en- dent
joyed by the Supreme Court? (a) on any subject

CLAT-5-Mock Tests.indd 317 13/09/2010 10:36:30


318 MOCK TESTS

(b) subject to some constitutional limita- (b) President


tions as legislation by Parliament (c) Prime Minister
(c) in his individual judgement as well as (d) Chief of Airforce
on the advice of the Council of Minis- 97. The maximum duration of the zero hour in
ters Lok Sabha of Parliament of India can be
(d) at any time (a) One hour (b) Unspecified
94. An ‘office of profit’ which disqualifies a (c) 30 minutes (d) Two hours
person from being a member of the Union 98. Which State enjoys the distinction of be-
or State Legislature does not include of- ing the latest State of the Union of India?
fice held under (a) Jharkhand (b) Uttaranchal
(c) Chhattisgarh (d) All the above
(a) the Government of India
99. The Governor of a State in India is respon-
(b) a State Government
sible for his act to
(c) a local authority
(a) the State Cabinet
(d) All the above (b) the State Legislature
95. Can the Governor reserve certain State (c) the President
Bill for Presidential assent? (d) the Prime Minister
(a) Yes 100. Under the Constitution of India the STs
(b) No are designated as such
(c) Circumstantial (a) by an Act of Parliament
(d) None of the above (b) by President notification
96. Who is the Supreme Commander of the (c) by the Commissioner of Scheduled
Defence forces of India? Tribes
(a) Chief of Army (d) by each State Government

ANSWERS
1. (b) 2. (c) 3. (b) 4. (a) 5. (a) 6. (a) 7. (b) 8. (a) 9. (d) 10. (c)
11. (a) 12. (b) 13. (b) 14. (c) 15. (c) 16. (d) 17. (a) 18. (b) 19. (a) 20. (b)
21. (d) 22. (b) 23. (c) 24. (d) 25. (b) 26. (c) 27. (a) 28. (a) 29. (a) 30. (b)
31. (c) 32. (a) 33. (a) 34. (c) 35. (c) 36. (d) 37. (d) 38. (b) 39. (b) 40. (b)
41. (a) 42. (b) 43. (c) 44. (d) 45. (d) 46. (c) 47. (b) 48. (b) 49. (c) 50. (b)
51. (c) 52. (a) 53. (c) 54. (a) 55. (c) 56. (a) 57. (c) 58. (c) 59. (b) 60. (b)
61. (d) 62. (d) 63. (c) 64. (d) 65. (a) 66. (d) 67. (b) 68. (d) 69. (b) 70. (c)
71. (d) 72. (b) 73. (c) 74. (b) 75. (b) 76. (c) 77. (a) 78. (a) 79. (a) 80. (a)
81. (c) 82. (a) 83. (b) 84. (b) 85. (a) 86. (c) 87. (a) 88. (d) 89. (c) 90. (b)
91. (c) 92. (c) 93. (b) 94. (d) 95. (a) 96. (b) 97. (b) 98. (a) 99. (b) 100. (b)

CLAT-5-Mock Tests.indd 318 13/09/2010 10:36:30


Chapter 6
PREVIOUS YEARSÊ PAPERS

CLAT 2008
ENGLISH at the bottom of the shore without knowing
you’re doing it. In so doing you’re actually fal-
PART – A sifying the sort of seaweeds they live on and so
Instruction: Read the given passage carefully on, which may seem unimportant, but it is actu-
and attempt the questions that follow ally changing the natural surroundings to make
Example: If the appropriate answer is (a), shade them prettier. Unfortunately, many of the people
the appropriate oval on the OMR sheet. who select pictures are looking for attractive
images and, at the end of the day, whether it’s
My love of nature goes right back to my child-
truthful or not doesn’t really matter to them.
hood, to the times when I stayed on my grand-
It’s important to think about the animal
parent’s farm in Suffolk, My father was in the
first, and there are many occasions when I’ve
armed forces, so we were always moving and
didn’t have a home base for any length of time, not taken a picture because it would have been
but I lived going there. I think it was my grand- too disturbing. Nothing is so important that you
mother who encouraged me more than anyone: have to get that shot; of course, there are cases
she taught me the names of wild flowers and got when it would be very sad if you didn’t, but it’s
me interested in looking at the countryside, so it not the end of the world. There can be a lot of
seemed obvious to go on to do zoology at Uni- ignorance in people’s behaviour towards wild
versity. animals and it’s a problem that more and more
I didn’t get my first camera until after people are going to wild places: while some ani-
I’d graduated, when I was due to go diving mals may get used to cars, they won’t get used
in Norway and needed a method of recording to people suddenly rushing up to them. The
the sea creatures I would find there. My father sheer pressure of people, coupled with two fact
didn’t know anything about photography, but he that there are increasingly fewer places where
bought me an Exacta, which was really quite a no one else has photographed, means that over
good camera for the time, and I went off to take the years, life has become much more difficult
my first pictures of sea anemones and starfish. I for the professional wildlife photographer.
became keen very quickly, and learned how to Nevertheless, wildlife photographs play a
develop and print; obviously I didn’t have much very important part in educating people about
money in those days, so I did more black and what is out there and what needs conserving.
white photography than colors, but it was all still Although photography can be an enjoyable pas-
using the camera very much as a tool to record time, as it is to many people, it is also something
what I found both by diving and on the shore. I that plays a very important part in educating
had no ambition at all to be a photographer then, young and old alike. Of the qualities it takes to
or even for some years afterwards. make a good wildlife photographer, patience is
Unlike many of the wildlife photographers perhaps the most obvious—you just have to be
of the time, I trained as a scientist and therefore prepared to sit it out. I’m actually more patient
my way of expressing myself is very different. now because I write more than ever before, and
I’ve tried from the beginning to produce pictures as long as I’ve got a bit of appear and a pencil,
which are always biologically correct. There are I don’t feel I’m wasting my time. And because
people who will alter things deliberately: you I photograph such a wide range of things, even
don’t piok up sea creatures from the middle of if the main target doesn’t appear I can probably
the shore and take them down to attractive pools find something else to concentrate on instead.

CLAT-6-Previous Years Papers.indd 319 06/09/2010 17:50:06


320 PREVIOUS YEARS’ PAPERS

1. The writer decided to go to university an (c) it is harder to find suitable places


study zoology because (d) they have become frightened of cars
(a) She wanted to improve her life in the 8. Wildlife photography is important because
countryside it can make people realize that
(b) She was persuaded to do so by her (a) photography is an enjoyable hobby
grandmother (b) we learn little about wildlife at school
(c) She was keen on the natural world (c) it is worthwhile visiting the country-
(d) She wanted to stop moving around all side
the time (d) wildlife photographs educate people
2. Why did she get her first camera? about wild animals
(a) She needed to be able to look back at 9. Why is she more patient now?
what she had seen (a) she does other things while waiting
(b) She wanted to find out if she enjoyed (b) she has got used to waiting
photography (c) she can concentrate better than she
(c) Her father thought it was a good idea used to
for her to have one (d) she knows the result will be worth it
(d) She wanted to learn how to use one 10. Which of the following describes the writ-
and develop her own prints er?
3. She did more black and white photogra- (a) proud (b) sensitive
phy than colour because (c) aggressive (d) disappointed
(a) She did not like color photograph
PART – B
(b) She did not have a good camera
(c) She wanted quality photograph Instructions: Three of the four words given be-
(d) She didn’t have much money in those low are spelt wrongly. Select the word that is
days spelt correctly.
4. How is she different from some of the 11. (a) acquaintance (b) acquaintence
other wildlife photographers she meets? (c) acuaintance (d) acqaintance
(a) She tries to make her photographs as 12. (a) negligence (b) negligense
attractive as possible (c) neglegence (d) nigligence
(b) She takes photographs which record- 13. (a) grievance (b) greivance
ed accurate natural conditions (c) grievanse (d) griecence
(c) She likes to photographs plants as 14. (a) hierarchical (b) hierarchikal
well as wildlife (c) hierarchecal (d) hierarichal
(d) She knows the best places to find 15. (a) garanter (b) garantor
wildlife (c) guaranter (d) guarantor
5. Which does them refer to in the 7th line in PART – C
paragraph 3?
(a) sea creatures Instructions: Select the best option from the
(b) attractive pools four alternatives given.
(c) seweeds 16. They live on a busy road. ………… a lot
(d) natural surroundings of noise from the traffic
6. What the writer means by ignorance in (a) it must be
people’s behaviors’ is (b) it must have
(a) altering things deliberately (c) there must have
(b) people suddenly rushing up to ani- (d) there must be
mals 17. The more electricity you use, …………
(c) people taking photographs of wild (a) your bill will be higher
animals (b) will be higher your bill
(d) people not thinking about the animals (c) the higher your bill will be
in the first place (d) higher, your bill will be
7. The writer now finds it more difficult to 18. Ben lines walking ………………
photograph wild animals because (a) every morning he walks to work
(a) there are fewer of them (b) he walks to work every morning
(b) they have become more nervous of (c) he walks every morning to work
people (d) he every morning walks to work

CLAT-6-Previous Years Papers.indd 320 06/09/2010 17:50:07


PREVIOUS YEARS’ PAPERS 321

19. It’s two years ………. Sophy trolley at a glance has just been devel-
(a) that I don’t see oped.
(b) that I haven’t seen The best sequence is
(c) since I didn’t see (a) ii, i, iii, iv (b) iv, i, iii, ii
(d) since I last saw (c) iv, iii, ii, i (d) iii, i, iv, ii
20. What was the problem? Why……. leave 27. UNIT II
early? (i) Of course, modern postal services now
(a) have you to are much sophisticated and faster, re-
(b) did you have to lying as they do on moter vehicles and
(c) must you planes for delivery
(d) you had to (ii) Indeed, the ancient Egyptians had a
21. Nobody believed Arun at first, but he system for sending letters from about
…….. to be right 200 BC, as did the Zhou dynasty in
(a) worked out (b) came out China a thousand years later
(c) found out (d) turned out (iii) Letters, were and are, sent by some
22. We can’t ……….. making a decision. We form of postal service, the history of
have to decide now which goes back a long way
(a) put away (b) put over (iv) for centuries, the only form of written
(c) put off (d) put out correspondence was the letter.
23. The accident was my fault, so I had to pay The best sequence is:
for the damage ………. the other car (a) ii, i, iii, iv (b) iv, i, iii, ii
(a) of (b) for (c) iv, iii, ii, i (d) iii, i, iv, ii
(c) to (d) on 28. UNIT III
24. I really object ………. people smoking in (i) Converting money into several cu8r-
my house rencies in the course of one trip can
(a) to (b) about also be quite expensive, given that
(c) for (d) on banks and bureau de change charge
25. A contract may be ……… if the court commission on the transaction
finds there has been misinterpretation of (ii) Trying to work out the value of the
the facts various notes and coins can be quite
(a) restrained (b) rescinded a strain, particularly if you are visiting
(c) compelled (d) conferred more than one country.
(iii) Travel can be very exciting, but it can
PART – D
also be rather complicated.
Instructions: The five paragraphs given below (iv) One of these complications is, un-
have all had their constituent sentences jum- doubtedly, foreign currency
bled. Read each jumbled passage carefully and The best sequence is:
then pick the option in which the best sequence (a) ii, i, iii, iv (b) iv, i, iii, iv
is shown. (c) iv iii, ii, i (d) iii, iv, ii, i
26. UNIT 1 29. UNIT IV
(i) The super tag scanner could revolu- (i) She was right about three curiosity,
tionize the way people shop, virtually freckles, and doubt-but wrong about
eradicating supermarket queues love
(ii) The face of retailing will change even (ii) “Four of the things I’d be better with-
more rapidly when the fibre optic net- out: love, curiosity, freckles, and
works being built by cable TV compa- doubt”
nies begin to be more widely used (iii) Love is indispensable in life
(iii) The scanner would have a double (iv) So wrote Dorothy parker, the Ameri-
benefit for supermarkets—removing can writer
the bottleneck which causes frustra- The best sequence is:
tion to most customers and reducing (a) ii, iv, i, iii (b) ii, i, iii, iv
the number of checkout staff (c) ii, i, iv, iii (d) iii, iv, i, ii
(iv) An electronic scanner which can read 30. UNIT V
the entire contents of a supermarket (i) This clearly indicates that the brains of

CLAT-6-Previous Years Papers.indd 321 06/09/2010 17:50:07


322 PREVIOUS YEARS’ PAPERS

men and women are organized differ- 38. Bone fide


ently in the way they process speech (a) identification card
(ii) Difference in the way men and women (b) without doubt
process language is of special interest (c) in good faith
to brain researchers (d) indispensable condition
(iii) However, women are more likely than 39. Status quo
men to suffer aphasia when the front (a) legally valid (b) preset condition
part of the brain is damaged (c) social position (d) side remarks
(iv) It has been known the aphasia a kind 40. De jure
of speech disorder is more common in (a) here and there (b) as per law
men than in women when the left side (c) small details (d) in the same place
of the brains damaged in an accidents
or after a stroke
GENERAL KNOWLEDGE
The best sequence is: Instructions: From the four answers, select the
(a) ii, i, iv, iii (b) iv, i, iii, ii most appropriate answer.
(c) iv, iii, i, ii (d) ii, iv, iii, i 41. The Supreme Court of India upheld the
PART – E decision to implement the quota for other
Instructions: Given below are five list of words backward classes (OBCs) in higher edu-
followed by some choices. In each case, choose cational institution. The court, however,
the alternative that you can combine with every excluded the “creamy layer” from being a
word in that particular list to form a familiar beneficiary. The reason is
word phrase. (a) creamy layer is not an OBC; it is a
31. Down, aside, about, forth forward caste
(a) set (b) fly (b) creamy layer is politically powerful
(c) burn (d) take (c) it can complete with others on equal
32. Over, about, after, at footing
(a) cross (b) lay (d) the inclusion of creamy layer would
(c) here (d) go be unjust
33. Forward, across, around, upon 42. Hedge fund is a fund
(a) straight (b) come (a) used for absorbing inflation
(c) fast (d) mark (b) used for cushioning health risks
34. In, down, for, out (c) applied to minimized the risk of finan-
(a) pray (b) try cial market transactions
(c) grow (d) stand (d) applied for absorbing the risk in com-
35. Away, through, up, down modity trading
(a) stray (b) come 43. What does strong rupee as against the dol-
(c) break (d) speak lar men to India?
(a) there is a balance of payments sur-
PART – F
plus
Instructions: Given below are a few foreign lan- (b) Indian economy is globally respected
guage phrases that are commonly used. Choose (c) It is a sign of economic buoyancy
the correct meaning for each of the phrases and (d) Income from exports is falling
shade the appropriate answer in the space pro- 44. Name the latest state which declared inde-
vided for it on the OMR answer sheet. pendence in 2008
36. Prime facie (a) Serbia (b) Kosovo
(a) the mast important (c) Kurdistan (d) Tibet
(b) that which comes first 45. Name the Finance Minister who presented
(c) at first view the highest number of budgets in the Par-
(d) the face that is young liament so far
37. Sine die (a) P. Chidambaram
(a) without setting a fixed day (b) Morarji Desai
(b) by voice vote (c) Man Mohan Sigh
(c) applying mathematical concepts to (d) T. T. Krishanmachari
solve a difficult problem 46. Who is the Chairman of 13th Finance
(d) signing legal documents before death Commission constituted in 2007?

CLAT-6-Previous Years Papers.indd 322 06/09/2010 17:50:07


PREVIOUS YEARS’ PAPERS 323

(a) Vijay Kelkar (b) C. Rangarajan (c) Singapore


(c) Ashok Lahiri (d) K. C. pant (d) Japan
47. Indo-U.S. nuclear deal was opposed in 57. The current impasse in Doha Round of
Parliament mainly because Negotiations is centered around
(a) all Indian nuclear reactors would fall (a) access to cheaper drugs
under American supervision (b) access to markets of developed coun-
(b) nuclear energy sector will be domi- tries
nated by American corporations (c) agricultural subsidies provided by de-
(c) nuclear relations between Indian and veloped countries
the USA will be governed by the Hybe (d) removal of non-tariff barriers
Act 58. The phenomenon called “Equinox” is due
(d) the USA will dictate Indian policies to the
48. The Indian industrialist who bought Tipu (a) rotation of the Earth on its own axis
Sultan’s sword in an auction in London (b) revolution of the Earth on its inclined
was axis
(a) Vijay Mallya (b) Anil Ambani (c) oblate spheroid shape of the earth
(c) Amar Singh (d) Lakshami Mittal (d) gravitational pull of the planet
49. The contentious Baglihar Dam is built on 59. The Director-General of the world trade
the river organization is
(a) Indus (b) Jhelum (a) Renal to Ruggiero
(c) Chenab (d) Satluj (b) Pascal Lamy
50. Which country has its richest man as the (c) Arthur Dunkell
Head of the Government? (d) Oliver Long
(a) The USA (b) Italy
60. Capital account convertibility signifies
(c) Saudi Arabia (d) Russia
(a) guaranteeing the right to investment
51. Who is the person known as the Father of
to foreigners
Modern Indian Retail Trade?
(b) ensuring the right of buyers to make
(a) Mukesh Ambani
intonation payments
(b) Kishore Bijani
(c) ensuring free international movement
(c) Aditya Birla
of capital
(d) Anil Ambani
(d) ensuring the right of an individual to
52. The largest software service company in
invest in foreign capital markets
Asia is
(a) WIPRO 61. The purpose of Kyoto Protocol is
(b) INFOSYS (a) to promote tourism
(c) Tata Consultancy Services (b) to contribute sustainable development
(d) Satyam Computers (c) to promote renewable sources of en-
53. Taikonaut means ergy
(a) a character in coming steps (d) to put a limit on greenhouse gas emis-
(b) a character in Russian opera sions by states
(c) astronaut in China 62. What do carbon credits signify?
(d) as delicious Japanese dish (a) Credits given in the course of carbon
54. The CEO of Microsoft Corporation is products sales
(a) Bill Gates (b) Warren Buffett (b) Entitlements to emit certain quantity
(c) Steve Ballmer (d) John Wallace of greenhouse gases
55. The country which stands for Gross Na- (c) Permissible amount of Carbon diox-
tional Happiness in contradistinction to ide in the atmosphere
Gross National Product (d) the extent of carbon required to ensure
(a) Sweden (b) Switzerland sustainable development
(c) Bhutan (d) Finland 63. The practice of selling goods in a foreign
56. The highest paid Head of the Government country at a price below their domestic
in the World at present is selling price is called
(a) The USA (a) discrimination (b) dumping
(b) Russian Federation (c) double pricng (d) predatory pricing

CLAT-6-Previous Years Papers.indd 323 06/09/2010 17:50:07


324 PREVIOUS YEARS’ PAPERS

64. Which of the following is considered as (c) Hill range


bulwark of personal freedom? (d) Man-made Walls
(a) Mandamus (b) Haebus Corpus 74. A nautical mile is equal to
(c) Certiorari (d) Quo Warranto (a) 1825 metres (b) 2000 metres
65. Vanda Mata ram is composed by (c) 1575 metres (d) 2050 metres
(a) Rabindranth Tagore 75. Which of the following is concerned with
(b) Sharatchandra Chatterji land forms?
(c) Bankimchandra (a) Geology (b) Geomorphology
(d) Surendrantath Banerji (c) Ecology (d) Geography
66. How many minutes for each degree of 76. The country known as the land of mid-
longitude does the local time of any place night sun
vary from the Greenwich time? (a) Sweden (b) Norway
(a) Four minutes (b) Two minutes (c) Fialand (d) Denmark
(c) Eight minutes (d) Ten minutes 77. The monk who spread Buddhism is Tibet
67. Article 1 of Indian Constitution defines and Far-East:
India as (a) Ananda
(a) Federal State (b) Nagarajuna
(b) Unitary State (c) Padmasambava
(c) Union of State (d) Mahendra
(d) Quasi-Federal State 78. TRIPs, forming part of the world trade or-
68. Which is the highest body that approves ganization is intended
Five Year Plans in India? (a) to provide for stronger patent protec-
(a) Parliament tion
(b) Planning Commission (b) to promote transational corporate in-
(c) National Development Council terests
(d) Council of Ministers (c) to harmonize IPR regime internation-
69. The economist who for the first time sci- ally
entifically determined national income in (d) to replace World Intellectual Property
India Organization
(a) Dr D. R. Gadgil 79. Carbon dating method is used to deter-
(b) Dr V. K. R. V. Rao mine the age of
(c) Dr Manmohan Singh (a) Rocks
(d) Dr Y. V. Alagh (b) Fossils
70. Which of the following is the largest pen- (c) Trees
insula in the world? (d) Ancient monuments
(a) Indian peninsula 80. The Managing Director of Delhi Metro-
(b) Arabian peninsula politan Railway Corporation
(c) Malaysian Peninsula (a) Chairman of Indian Railway Board
(d) Chinese Peninsula (b) Lt. Governor of Dehli
71. The person responsible for economic (c) C. Sreedharan
model for Indian planning (d) Sheela Dikshit
(a) Jawaharlal Nehru 81. 18 carat gold signifies
(b) P. C. Mahalanobis (a) 18 parts of gold and 82 parts of other
(c) Tarlok Singh metal
(d) V. T. Krishnamachari (b) 18 parts of gold and 6 parts of other
72. Social forestry aims at metal
(a) Ensuring fuel and forest products to (c) 82 parts of gold and 18 parts of other
weaker sections metal
(b) Medicinal and fruit plantation (d) None of the above
(c) Large scale afforestation 82. Bali Road map adopted in December 2007
(d) Scientific forestry provides for
73. The Great Barrier Reef refers to (a) fixation of Green House Gas Emission
(a) Coral formation limits
(b) Tidal Waves (b) launching of an adaptation fund

CLAT-6-Previous Years Papers.indd 324 06/09/2010 17:50:08


PREVIOUS YEARS’ PAPERS 325

(c) amendment to UN framework con- (c) the places where industry get certain
vention on climate change tax advantages
(d) special and differential treatment for (d) the places where in the national labour
developing countries laws do not apply
83. Special uranium, used in a nuclear reactor, 91. The Space Shuttle which successfully car-
is ried Sunita Williams to space
(a) uranium freed of all impurities ura- (a) Challenger (b) Atlantis
nium treated with radiation (c) Discovery (d) Columbus
(b) uranium mixed with radiation on cli- 92. The leader who led the country in atoning
mate change of the past wrongs
(c) uranium mixed with isotopes (a) John Howard (b) Desmond Tutu
(d) uranium alloy with aluminium (c) Kevin Rudd (d) Jimmy Carter
84. The scientist responsible for developing 93. Gandhiji expounded his economic ideas
atomic energy in India in
(a) C. V. Raman (b) H. J. Bhabha (a) Hind Swaraj
(c) H. K. Sethna (d) Vikram Surabhai (b) My Experiments with Truth
85. Salwa Judum practiced in certain places in (c) Unto the Last
India refers to (d) Economies of Permanence
(a) witchcraft 94. Bio-fuels have become controversial be-
(b) arming civilians to fight militants cause
(c) training civilians in the use of fire (a) they increase environmental pollution
arms (b) they slow down industrialization
(d) training civilians to be home guards (c) they reduce food cultivation
86. Indian who won Raman Magasaysay (d) they lean to degeneration of soil
award in 2007 95. Ever greening of patents means
(a) Arun Roy (b) P. Sainath (a) granting patents in perpetuity
(c) Medha Patkar (d) Ruth Manorama (b) granting patents for 100 years
87. The person who won Jawaharlal Nehru (c) granting protection to incremental in-
award in 2007 ventions having no substantial signifi-
(a) Lula de Silva cance
(b) Hugo Chavez (d) patenting of green technology
(c) Aung Saan Suu ki
96. By signing which pact with Gandhiji did
(d) Fidel Castro
Ambedkar give up his demand for sepa-
88. Free Trade Area means
rate electorates
(a) the area where anything can be bought
(a) Poona Pact (b) Aligarh Pact
and sold
(c) Deccan pact (d) Delhi Pact
(b) countries between whom trade barri-
97. India earns maximum foreign exchange
ers have been substantially reduced
from the export of
(c) countries which have common exter-
(a) Garments
nal tariff
(b) Jute
(d) countries which have common cur-
(c) Gems and Jewelleries
rency
(d) Light engineering goods
89. Affirmative action in Indian context signi-
fies 98. Sunita Williams, renowned astronaut of
(a) providing security to weaker sections Indian origin, spent a record of ……….
(b) welfare measures to alleviate the suf- days in space
ferings of poor people (a) 195 (b) 185
(c) providing positive opportunities to (c) 200 (d) 160
deprived sections 99. The second biggest Green House Gas
(d) giving incentives to start industries emitter (after the USA) in the world is
90. Special Economic zones are (a) Russia (b) Germany
(a) the places where industries can oper- (c) China (d) Japan
ate without any control 100. The author of the management principle in
(b) the places where in any person can a hierarchy, every employee tends to rise
start any industry to his level of incompetence

CLAT-6-Previous Years Papers.indd 325 06/09/2010 17:50:08


326 PREVIOUS YEARS’ PAPERS

(a) Prof. Ducker (a) M. A. Jinnah


(b) Prof. J. Peter (b) Hakim Azmal khan
(c) Prof. C. H. Prahlad (c) Mohammad Iqbal
(d) Prof. Schmitthoff (d) Liaqusat Ali Khan
101. The World Trade Organization was earlier 112. Khilafat Movement was organized
known as (a) for getting Muslim homeland
(a) UNCTAD (b) GATT (b) as a protest against British suppres-
(c) UNIDO (d) UNCITRAL sion of Turks
102. The “Wailing Wall” is associated with (c) to preserve Turkish Empire with Kh-
(a) Christians (b) Bahais ilafat as temporal head
(c) Jews (d) Shias (d) as a protest against communal poli-
103. An Education Minister who got Bharata tics
Ratna in India 113. The pattern of Centre-State relations in In-
(a) G. B. Pant dia can be traced back to
(b) M. C. Chagla (a) The U.S. Constitution
(c) Abdul Kalam Azad (b) The Government of India Act 1935
(d) Humayun Kabir (c) Motilal Nehru Committee Report
104. Why is Ozone layer important? (d) Ambedkar’s vision
(a) It absorbs greenhouse gases 114. Indian who played a very important role in
(b) It protects earth from ultraviolet radia- world Communist Movement
tion (a) Joyti Basu (b) M. N. Roy
(c) It maintains earth’s temperature (c) A. K. Gopalan (d) Prakash Karat
(d) It is buffer against extra terrestrial 115. Who was the first recipient of Jnanapith
hazards Award?
105. The world’s largest river is (a) Amrita Pritam
(a) Brahmaputra (b) Amazon (b) Dinkar
(c) Nile (d) Mississippi (c) D. V. Gundappa
106. Tsunami is caused by (d) G. Shankara Kurup
(a) plate tectonics 116. Name the winner of 2007 Noble Prize for
(b) underwater ridges Literature
(c) underwater volcanic activity (a) Dorris Lessing (b) V. S. Naipaul
(d) pressure from Earth’s core (c) Doras Lessing (d) Salman Rushdie
107. The Chipko Movement is associated with 117. Plea bargaining is
(a) preventing the felling of trees (a) permissible in India
(b) afforestation (b) illegal in India
(c) transparency in public life (c) mandatory in India
(d) sustainable development (d) allowed subject to the permission of
108. The First Great Inidan Empire was the court
(a) Magadhan Empire 118. The person appointed by two parties to
(b) Kuru Empire settle a dispute is known as
(c) Pandava Empire (a) Judge (b) Arbitrator
(d) Gandhara Empire (c) Solicitor (d) Conciliator
109. The First President of Indian National 119. Right to travel is a Fundamental Right un-
Congress der
(a) A. O. Hume (a) Article 19 of the Constitution
(b) W. C. banerji (b) Article 21 of the Constitution
(c) Dadabhai Nauroji (c) Article 14 of the Constitution
(d) Phirozeshah Mehta (d) None of the above
110. The King who gave permission to estab- 120. Genetically modified seeds have become
lish East India Company in India controversial mainly because of
(a) Jahangir (b) Aurangzeb (a) adverse impact on human health
(c) Shahjahan (d) Shershah (b) legal right
111. The person who conceptualized the idea (c) adverse impact on ozone layer
of Pakistan (d) None of the above

CLAT-6-Previous Years Papers.indd 326 06/09/2010 17:50:08


PREVIOUS YEARS’ PAPERS 327

121. Legal aid for an accused is 130. The Chairman of Sixth Pay Commission
(a) Fundamental Right (a) Justice B. N. Srikrishan
(b) legal right (b) Justice Ratnavl Pandian
(c) Directive Principles of State Policy (c) Justice Jagannatha Shetty
(d) Discretion of State (d) Justice A. K. Majumdar
122. The Members of Constituent Assembly 131. Right to education emanates from
who framed the Constitution were (a) right to culture and education under
(a) directly elected by the people articles 29 and 30
(b) indirectly elected (b) right to equality under articles 14
(c) nominated (c) freedom of speech 7 expression under
(d) appointed by political parties article 19
123. Ambedkar acted in Constituent Assembly (d) right to life and personal liberty under
as articles 21
(a) President of the Assembly 132. International Court of Justice is
(b) Chairman of the Drafting Committee (a) an independent International Institu-
(c) the leading spokesman of weaker sec- tion
tions (b) a principal organ of the U.N.O.
(d) a strong defender of Fundamental (c) a subsidiary organ of the U.N.O.
Rights (d) an European institution
124. In India, international treaties are ratified 133. The Liberhan Commission which received
by repeated extensions has been inquiring
(a) Parliament into
(b) President (a) Godhra riots
(c) Prime Minister (b) Mumbai riots
(d) The Union Cabinet (c) Demolition of Babri Masjid
125. It is a Constitutional requirement that the (d) killing of Sikhs in Delhi
Parliament shall meet at least 134. The monopolies and restrictive trade prac-
(a) twice in a year tices act was repeated by
(b) thrice in a year (a) Completion Act
(c) once in a year (b) Consumer Protection Act
(d) none of the above (c) Foreign Trade (Development and
126. Governor of a State can be removed by
Regulation) Act
(a) impeachment by State Legislature
(d) Liberalization Policy of the Govern-
(b) the President
ment
(c) by the State Cabinet
135. Only judge against whom a motion of
(d) the Union Government at the request
impeachment was introduced into parlia-
of the Chief Minister
ment
127. Sovereignty under the Constitution be-
(a) Justice Subba Rao
longs to
(b) Justice Ramaswami
(a) The Parliament
(c) Justice Mahajan
(b) The People
(d) Justice Viraswami
(c) The Supreme Court
136. The Mallimath Committee Report deals
(d) The President along with parliament
128. The Supreme Court upheld Mandal Com- with
mission Report in (a) Judicial delays in India
(a) Bommai vs Union of India (b) Criminal Justice Administration
(b) Indra Sawhney vs Union of India (c) Stock Market reforms
(c) Unnikrishnan vs Union of India (d) Review of Constitutional System
(d) Maneka Gandhi vs Union of India 137. The First Woman Chief Justice of High
129. Under our Constitution, Right to Property Court in India
is (a) Leila Mukherji (b) Leila Seth
(a) Fundamental Right (c) Fatima Bibi (d) Ruma Pal
(b) basic structure of the Constitution 138. Lok Adalat have been created under
(c) Constitutional Right (a) Legal Services Authority Act
(d) a mere legal right (b) Arbitration and Conciliation Act

CLAT-6-Previous Years Papers.indd 327 06/09/2010 17:50:08


328 PREVIOUS YEARS’ PAPERS

(c) Administration of Justice Act hours. If all the taps can be turned on at the
(d) None of the above same time time, the tank will be full in
139. Recent Nepal elections are globally sig- (a) 1 hours (b) 40 minutes
nificant because (c) 1½ hours (d) 3 hours
(a) Communists came to power through 147. Of the three numbers, the first is one third
ballot box for the first time in the of the second and twice the third, the aver-
world age of these numbers is 27. The largest of
(b) monarchy was defeated by democratic these numbers is
forces (a) 18 (b) 36
(c) a militant movement joined the main- (c) 54 (d) 108
stream 148. The length of a square is increased by 15%
(d) secularism triumphed over theocracy and breadth decreased by 15%. The area
140. The Third World leader who has been de- of the rectangle so formed is
fying the USA. (a) neither increases nor decreases
(a) Fidel Castro (b) decreases by 2.25%
(b) Hugo Chaves (c) increases by 2.25%
(c) Robert Mughabe (d) decreases by 22.5%
(d) Hu Jintao 149. The ratio of milk and water in 60 litres of
MATHEMATICAL ABILITY adulterated milk is 2 : 1. If the ratio of milk
Instructions: From the four given answers, se- and water is to be 1 : 2, then the amount of
lect the appropriate answer. water to be added further is
141. Raju earns twice as much in march as in (a) 20 litres (b) 30 litres
each of the other months of the year. What (c) 40 litres (d) 60 litres
part of his annual earnings he earns in that 150. A piece of cloth costs ` 70 if the piece is 4
month? metre longer and each metre costs ` 2 less,
(a) 1/5 (b) 5/7 the cost remains unchanged. The length of
(c) 2/13 (d) 1/10 the piece is
142. Sanjay sold his watch for ` 1140 and (a) 8 m (b) 9 m
thereby loses 5% in order to gain 5% he (c) 10 m (d) 12 m
has to sell the watch for LOCIAL REASONING
(a) ` 1254 (b) ` 1260
Instructions: Read carefully the questions and
(c) ` 1197 (d) ` 1311
143. A mixture of 40 litres of milk and water select the appropriate answer.
contains 10% of water, how much water 151. A college received fifty applications for a
is to be added to the mixture so that water certain course. In the qualifying examina-
may be 20% in the new mixture. tion, one tenth of them secured marks in
(a) 5 litres (b) 4 litres 90-95% range. Within remaining segment,
(c) 6.5 litres (d) 7.5 litres three-fifths of them secured marks in 75-
144. A train 100 metres long running at 54 90% range. The rest secured below 75% to
km/h takes 20 seconds to pass a bridge. get admission, the following restrictions
The length of the bridge is hold good
(a) 50 m (b) 150 m (i) no student who has scored below 75%
(c) 200 m (d) 620 m can seek admission to Physics course
145. Sameer is as much younger to Mohan (ii) no student is allowed to opt Physics
as he is older to Arun. If the sum of the without opting Mathematics
ages of Mohan and Arun is 48, the age of (iii) no student is allowed to opt Physics
Sameer is and Astrophysics simultaneously
(a) 20 years (iv) to opt Mathematics or Astrophysics,
(b) 24 years a student should have scored at least
(c) 30 years 70% in the qualifying examination.
(d) cannot be determined Which one of the following alternatives is
146. A tank can be filled up by two pipes A and possible?
B in 2 hours and 3 hours respectively. A (a) Ninety percent of the applicants are
third pipe C can empty the full tank in 6 admitted to Physics course

CLAT-6-Previous Years Papers.indd 328 06/09/2010 17:50:08


PREVIOUS YEARS’ PAPERS 329

(b) Thirty-five percent of the applicants 155. Political turmoil in a country is mainly
who are other wise ineligible to join caused by widespread violence and flawed
Physics course are admitted to Math- economic policies of successive govern-
ematics and Astrophysics course ments. If at all this has to be crushed, it
(c) Students of Physics course outnumber can be achieved only by a dictatorial
those of Mathematics government which rules with iron hand.
(d) Whoever is eligible to study Math- Therefore, the need of the hour is to elect
ematics is also eligible to study Phys- a government which imposes fresh set of
ics stringent legislations.
152. A tourist can tour utmost four places out The alternatives suggested (not necessar-
of A, B, C, D, E, F and G. Out of four, ily all), if true, considerably weaken the
not more than two can come under holi- argument however, one of them is mot
day tour and at least two must come under forceful. Identify the same.
business trip. The break up is as follows: (a) It is not the imposition of new legisla-
A, B, C and D—business tour; E, F and tions which is required, but effective
G—holiday tour adherence to the existing legislations
The following restrictions hold good. (b) That government is the best govern-
(a) If A is included, then both C and are ment which governs least
excluded. (c) It is possible to overcome any evil by
(b) If neither E nor F is included, then B educating people
or G or both of them can be included (d) Only dialogue in a free society can
(c) If G is included, then D cannot be in- eradicate political turmoil
cluded. 156. Under the same fact situation as above,
Which one of the following combinations the alternatives suggested (not necessar-
is possible? ily all), if true, significantly strengthen the
(a) A, C, E and F (b) B, G and E argument. However, one of them is most
(c) A, D and G (d) A, B and D forceful. Identify the same.
153. Under the same fact situation as above, (a) Espionage activities by enemy na-
suppose that the following restrictions tions, which contribute to political
hold good turmoil, can be prevented only if the
(A) A can be included provided C is in- government is very strong
cluded (b) The philosophy behind any economic
(B) E is included provided B or G is in- policy, push from bottom, press from
cluded but not both top’ is to be followed to mitigate vio-
(C) C can be included provided at least D lence, and it is not observed
or F is (c) Political turmoil is due to corrupt es-
Which one of the following is a certainty? tablishment
(a) A, B, C and E (b) A, C, D and F (d) Man is, by nature, a beast
(c) B, C, D and E (d) A, B, C and F 157. Exploitation of poor rich can be stemmed
154. Four members have to be nominated to a only if the state exercises complete control
committee and there are six candidates: A, over agriculture and industrial production.
B, C, D, E and F. The following restric- But state control is beset by two evils; cor-
tions hold good ruption and delay. The net result is that if
(A) If A is nominated, then D does not find man tries to escape from one evil, then he
any place. is trapped by another suffering. Hence, is
(B) If B is nominated, then either E or F inescapable.
has to be nominated, but not both. The argument presented above seems to
(C) If C is nominated, then both D and E imply the following conclusions. Iden-
have to be nominated. tify the one which is least dubious. Apply
Which one of the following is an accept- common sense.
able combinations? (a) If agriculture and industrial produc-
(a) A, B, C and E (b) A, B, C and D tion can be abolished, we can free
(c) B, C, D and E (d) B, C, D and F ourselves from all forms of evil

CLAT-6-Previous Years Papers.indd 329 06/09/2010 17:50:08


330 PREVIOUS YEARS’ PAPERS

(b) To avoid evil austere life shall be en- However, he knew with the help of earlier
couraged study carried out by Petit and Nicholson
(c) The gap between poor and rich can be that it was not the case. So, he concluded
bridged by heavily taxing the rich and that the planet must rotate fairly often to
passing on the largesse to the poor keep the darker side warmer.
(d) Man is, by nature, dishonest. Which of the following is the original
158. That the human soul is immaterial is an premise?
undisputed fact. Significantly, what is not (a) Slow rotation of Venus
matter is not spatial and consequently, it (b) Temperature of Venus
is no vulnerable to motion. Evidently, no (c) Frequent rotation of Venus
motion no dissolution. What escapes from (d) Equality of the rate of rotation and
dissolution is also free from corruptibil- revolution
ity. 162. Before formulating the laws of motion,
Therefore, the human soul is immortal. In Galileo distinguished between mathemati-
this argument, one premises is missing. cal study and empirical study. He, first,
Complete the argument by choosing from theoretically derived the relation between
the following. distance and time for uniformly accelerat-
(a) Nothing is free from dissolution ing motion by letting the ball roll a quar-
(b) What is incorruptible is immortal ter, then half, then two thirds and so on
(c) There is no motion of the length of the groove and then mea-
(d) Matter does not exist sured the times on each occasion, which
159. Under the same fact situation as above, he repeated hundred times. He calculated,
which one of the following, if true, affects, based on this study, that the distance trav-
seriously the argument presented above? eled equaled the square of the time on all
(a) Matter is not bound by space occasion.
(b) Matter is indestructible Which one of the following characterizes
(c) Whatever exists is not necessarily af- Galileio’s method?
fected by motion (a) Speculation
(d) What is not matter also is vulnerable (b) Theoretical analysis
to motion (c) Generalization
(d) Statistical analysis
160. Protagonists of human rights vehemently
163. Read carefully a breif summary of one
oppose capital punishment. Their oppo-
of the investigations of Sherlock Hol-
sition stems mainly from three reasons.
mes: “While investigating the murders of
Firstly, man cannot terminate what he
Stangerson and Enoch Drebber he got into
cannot generate. Secondly, the function of
conversation with fellow deteotives which
punishment is to reform the culprit. Third-
runs as follows: “The last link ……….
ly, a culprit should be given an opportu-
My case is complete ……. could you lay
nity to repent. Admittedly, death penalty
your hands upon those pills”. After he got
fails on all three counts. A however, the
those pills, Holmes cut none of them, dis-
defenders argue that a person is punished
solved it in water and placed it in front of
because he has to pay for his deeds. Refor-
the terrier. Contrary to his expectations,
mation or repentance, according to them, the animal survived, though disappointed
is peripheral. Hence, death penalty is ad- a bit, he thought for a while and then cut
missible. the other pill, dissolved it, added milk and
Which one of the following is the focus of placed before the animal. The moment it
this debate? licked, the animal died. Those were the
(a) Man’s rights and privileges pills present at the scenes of crime.
(b) Nature and purpose of punishment Which one of the following aptly de-
(c) Prevention of crime scribes the methods which this passage
(d) Mercy and revenge indicates?
161. Since Venus rotates slowly, Fred Whipple (a) Imaginations
through that like mercury, venues keeps (b) Experiment
one face always towards the sun. If so, he (c) Observation
said that the dark side would be very cold. (d) Thought experiment

CLAT-6-Previous Years Papers.indd 330 06/09/2010 17:50:08


PREVIOUS YEARS’ PAPERS 331

164. There has been much speculation concern- The off spring were tall plants. He next
ing the origin of lunar craters. One hypoth- let the flowers of this first generation be
esis is that they are the results of heavy fertilized with their own pollen. In the fol-
meteors on the surface of the moon while lowing generation, shortness reappeared.
still soft. The most probabale explanation Tallness and shortness were distributed
is that they were produced by the gases not at random but in a definite, constant,
liberated from the rocky matter while so- and simple ratio: three dominant talls to
lidification was taking place these gases one recessive short’
and water vapors steadily escaped through Which one of the following aptly describes
viscous surface, raising giant bubbles. The the distribution of dominant and recessive
reader can easily visualize the process that characteristics?
and noticing the formation of bubbles and (a) Systematic (b) Equal interval
craters on their surface. (c) Unpredictable (d) Irregular interval
Which one of the following actually helps 169. It is said that in his strongly worded reac-
us to determine the origin of lunar cra- tion to quantum physics, Einstein remarks
ters? ‘God does not play dice’ to which Bohr,
(a) Analogy another great Physicist, reacted saying
(b) Study of foreign body ‘Do not tell God what to do’ Bohr earlier
(c) Course and effect relation had argued that we can never known what
(d) Speculation the properties of an isolated quantum sys-
165. “Perhaps the earliest work of Archimedes tem, though we can know the properties of
that we have is that on plane equilibrium. macrocosmic objects.
In this, some fundamental principles of Which one of the following is the focus of
mechanics are set forth as rigorous geo- their debate?
metric propositions. The work opens (a) The behaviour of God
with famous postulate equal weights at (b) Probabilistic interpretation of the be-
equal distances are in equilibrium; equal haviour of quantum object
weights at unequal distances are not in (c) Limits of human knowledge
equilibrium, but incline toward the weight (d) Irrelevance of microcosmic object
at the greater distance” 170. An efficient and diesel-independent pub-
lic transport system s essential to the eco-
According to this passage, which factor or
nomic development of nation, suppose
factors determine equilibrium?
that the government adopts a policy to
(a) Weight
that effect then there is another favorable
(b) Distance
result. The pollution of environment is re-
(c) Weight and distance
duced to a greater extent. But then it has
(d) Equality of weights and distances
two pronged backlash. The sale and con-
166. According to the above passage, which
sequently the production of two and four
one of the following values can be as-
wheelers reduce to the minimum which
signed to the statement inequal weights at
in turn render a large number of people
equal distance are in disequlibrium?
jobless. Cash flow to the treasury also is
(a) True
adversely affected. Such a step, therefore,
(b) False is self-defeating unless the government
(c) Highly probable evolves a counter strategy to nullify the
(d) Highly improbable adverse effects.
167. According to the above passage, which Which one of the following accurately
one of the following values can be as- projects the opinion of an imaginary
signed to the statement in equal weights at speaker or author as the case may be of
in equal distances are in disequilibrium? this passage?
(a) true (b) false (a) Abandon the idea of efficient an diesel
(c) uncertain (d) unverifiable independent public transport system
168. ‘Gregor Mendel in examining tea plants (b) Ensure sustained cash flow and create
found two sharply marked races, the tall better job opportunities by invention
and the short, he experimentally fertilized an alternate or more than one alter-
flowers of tall plant with pollen of short. nate

CLAT-6-Previous Years Papers.indd 331 06/09/2010 17:50:08


332 PREVIOUS YEARS’ PAPERS

(c) Public transport system shall be given progress without meddling with nature is a
up myth. But none van live without scientific
(d) Maintain production and sale at the and technological advance which has sin-
same level by offering incentives gularly made progress possible. Further-
171. A most question to be considered is more, environment includes not just forest
whether democratic form of Government wealth and hills, but animal wealth also.
is a boon or bane, no matter what Lincoln An honest environmentalist is obliged to
might or might not have said. Rather his address the following questions. First,
most (in?) famous adage, ‘by the people, should man in the interest of hygiene, kill
for the people and of the people’ misses any living being be it an insect purported
the most pertinent question ; which atti- to be harmful or stray dogs? After all, this
tude works behind when a person declares world does not belong to man alone.
that he is a (or the right?) candidate to Which one of the following runs counter
serve the people, and does not hesitate to the spirit of the passage?
to contest and fight toothy and nail the (a) In the interest of health and cleanli-
election, an euphemism for battle with ness, our surroundings must be from
or without bullets. Admittedly, the covert disease spreading bacteria
attitude is different from overt attitude. (b) Non-violence as a moral principle ex-
Hardly any one contests the election un- tends to all living creatures
willingly. A contestant is not persuaded by (c) Vegetarian food is ideal to all men
any one, but driven by his own passions (d) Man should protect his environment
and dubious motives. Contrast this picture because he has to live
with Socrates’ version; no honest man 174. Does our soviet need reservation in job?
willingly takes up the job of ruler. If at all Before we defend reservation, we must
he accepts, he does so for fear of being consider some issues. Why do we need
ruled by one made up of inferior mettle. It reservation? Obviously, reservation is re-
is beyond even the wildest imaginations, quired to lift the downtrodden and thereby
to expect an honest person to contest the achieve equality. How do you achieve
election. this? Every individual, without exception,
Assuming that every statement is true, has a right t receive quality education. It
is more so in the case of downtrodden
identify from among the given alterna-
people. Only a good – natured meritorious
tives the one which strictly follows from
teacher can impart quality. Suppose that
the passage.
a person who is neither good natured nor
(a) No assessment or appraisal of democ-
meritorious becomes a teacher thanks to
racy is possible
reservation suffer.
(b) Lincoln and Socrates are talking dif-
(a) Reservation is individual–centric but
ferently
not group centric
(c) Actually, Socrates, scores over Lin-
(b) Reservation in at least on field, is self
coln on this issue
defeating
(d) Rulers can be honest
(c) The argument is biased
172. According to the above passage, which
(d) Education is not required to uplift the
one of the following correctly differenti- downtrodden
ates Lincoln’s and Socrates’ analyses? 175. Under the same fact situation as above,
(a) The nature of democracy which one of the following helps you to
(b) Merits and demerits of politicians circumvent the situation?
(c) Qualities of election (a) Replace education with money and
(d) Difference in mind set of respective make poor rich
men (b) To achieve equality encourage inter
173. Many environmentalists either adopt dou- caste marriage
ble standard or do not know what they are (c) Only downtrodden people should
talking about. A protagonists of environ- form the government
ment, for talking about. A protagonist of (d) Identify good natured and meritorious
environment, for obvious reasons, ought people within downtrodden group to
not to but for any type of progress because make them teachers

CLAT-6-Previous Years Papers.indd 332 06/09/2010 17:50:08


PREVIOUS YEARS’ PAPERS 333

LEGAL REASONING 177. Principles


(1) If a person commits an act by which
PART – A
death is caused to another person and
Instructions: Each question contains some ba-
the act is done with the intention of
sic principle and fact situation in which these
causing death, that person is liable for
basic principles have to be applied. A list of
murder
probable decision and reasons are given. You
(2) A person has a right of self defence to
have to choose a decision with reason.
the extent of causing death to another
176. Principles
provided he apprehends death by the
(1) On the death of husband, the widow
act of the latter
shall inherit the property of her de-
ceased husband along with children
Facts: Shuva went to a hardware shop owned
equally.
by Anup. Bargaining on some item
(2) A widow can not claim the property of
led to altercation between the two and
the deceased if on the date when the
Shuva picked up a sharp object and hit
question of succession opens, she has
at Anup. When Anup started bleeding,
remarried.
his wife Maridula intervened and she
(3) A female acquiring property in any
was also hit by Shuva and she became
way has the absolute title to the prop-
unconscious. Finding himself totally
erty.
cornered, Anup delivered a severe
Apply the above three principles and de-
blow to Shuva with a sharp object.
cide the case of the following fact situa-
Shuva died instantly.
tion:
Possible decisions
Facts: When Sudhir died, he had 1/3rd share
(a) Anup murdered shuva.
of the family property, which the three
(b) Anup killed Shuva with the intention
brothers Rudhir, Sudhir and Yasu in-
of killing to save himself and his wife
herited from father, B.
(c) Anup killed Shuva without any inten-
Sudhir died on September 23rd 2006
tion to do so just to save himself and
without having any issue. The widow
his wife probable reasons for the deci-
of Sudhir, Ms Win remarried on Janu-
sion
ary 1, 2007.
(i) if a person kills another instantly on
Rudhir and Yasu refused ‘win’ the
the spot, the intention to kill is obvi-
share from Sudhir’s portion when Win
ous
claimed the entire property belonging
(ii) Anup used force apprehending death
to Sudhir on January 30, 2007.
of himself and his wife
Select your decision from the possible de-
(iii) Anup used disproportionate force
cisions given in list I and the appropriate
(iv) there was nothing to show that Shuva
reason from the indicated reason given in
wanted to kill Anup or his wife
list II given below:
Your decision with the reason
List I—Decisions
(a) (a) (i) (b) (a) (ii)
(a) win cannot inherit the property of Su-
(c) (a) (iii) (d) (a) (iv)
dhir
178. Principle
(b) win can inherit the property of Sudhir
(1) Consumable goods which are not fit
List II—Reasons
for consumption are not marketable
(i) Win does not belong to the family
(2) A consumer shall not suffer on ac-
(ii) Win was remarried
count of unmarketable goods
(iii) her claim was on the date of Sudhir’s
(3) A seller is liable for knowingly selling
death
unmarketable goods
(iv) her claim was subitted after she was
(4) A manufacturer shall be liable for the
remarried
quality of his products
Your decision and reason
Facts: Ram bought a Coca Cola bottle from
Shade the right decision with reason from
Shama’s shop. Back at home, the
the following.
server opened the bottle an poured the
(a) (a) (i) (b) (a) (i)
drink the into the glasses drinking. He
(c) (a) (iii) (d) (a) (iv)

CLAT-6-Previous Years Papers.indd 333 06/09/2010 17:50:08


334 PREVIOUS YEARS’ PAPERS

felt irritation in his throat, immediate- (d) Both Somu and Amar are liable for
ly, Ram and Tom took the sample to the wrong.
test and found nitric acid in the con- Suggested reasons
tent. Ram filed a suit against Shama, (i) Amar committed the wrong while act-
Coca Cola company and the bottler, ing for the benefit of Somu.
Kishan and Co. (ii) Amar cannot do while acting for
Suggested decisions Somu something which he cannot do
(a) Ram cannot get compensation while acting for himself.
(b) Tom can get compensation (iii) Both Amar and Somu are liable since
(c) Both ram and tom can get compensa- they are bound by the contract.
tion (iv) Somu has to be responsible for the act
Suggested reasons of Amar committed for Somu’s ben-
(i) Shama did not know the contents of efit.
sealed bottle. Your decision with the reason
(ii) Ram did not actually suffer though he (a) (a) (i) (b) (a) (ii)
bought the bottle. (c) (c) (iii) (d) (d) (iv)
(iii) Tom did not buy the bottle. 180. Principles
(iv) Coca Cola company is responsible 1. The owner of a land has absolute in-
since it supplied the concentrate. terest on the property including the
(v) Kishen & Co, is responsible since it contents over and under the property.
added water, sugar, etc, and sealed the 2. Water flowing below your land is not
bottle yours though you can use it.
(iv) Shama is responsible for selling the 3. Any construction on your land be-
defective product. longs to you.
Your decision with the reason 4. All mineral resources below the land
(a) (a) (ii) (b) (b) (vi) belongs to the state.
(c) (c) (v) (d) (c) (iv) Facts: There is a subterranean water flow
179. Principles under Suresh’s land surface. Suresh
1. If A is asked to do something by B, B constructed a huge reservoir and drew
is responsible for the act, not A. all subterranean water to the reservoir.
2. If A, while acting for B commits a As a result, the wells of all adjacent
wrong, A is responsible for the wrong, property owners have gone dry. They
not B. demanded that either Suresh must de-
3. If A is authorized to do something molish the reservoir or share the reser-
for B, but in the name for A without voir water with them.
disclosing B’s presence, both A and B Proposed decision
may be held liable. (a) Suresh need not demolish the reser-
Facts: Somu contracted with Amar where voir.
under Amar would buy a pumpset to (b) Suresh has to demolish the reservoir.
be used in Somu’s farm such a pump- (c) Suresh has to share the water with his
set was in short supply in the market. neighbours.
Gulab, a dealer, had such a pump- (d) The government can take over the res-
set and he refused to sell it to Amar. ervoir.
Amar threatened Gulab of serious Possible reasons
consequences if he fails to part with (i) Water cannot be captured by one per-
the pumpset. Gulab filed a complaint son for his personal use
against Amar. (ii) The government must ensure equi-
Proposed decision table distribution of water.
(a) Amar alone is liable for the wrong (iii) Whatever is under Suresh’s land may
through he acted for Somu. be used by him.
(b) Amar is not liable for the wrong. (iv) Suresh has to respect the rights of oth-
Through he is bound by the contract ers regarding water.
with Somu. Your decision with the reason
(c) Somu is bound by the contract and li- (a) (a) (iii) (b) (b) (i)
able for the wrong. (c) (c) (iv) (d) (d) (ii)

CLAT-6-Previous Years Papers.indd 334 06/09/2010 17:50:08


PREVIOUS YEARS’ PAPERS 335

181. Principle Syndicate Bank with the help of her


1. An employer shall be liable for the nephew by name Keshav show was
wrong committed by his employees in at that time working as a clerk in
the course of employment. the bank, Keshav used to deposit the
2. Third parties must exercise reason- money of Rama Bhai from time to
able care to find out whether a person time and get the entries done in the
is actually acting in the course of em- passbook. After a years or so, Keshav
ployment. was dismissed from the service by
Facts: Nandan was appointed by syndicate the bank. Being unaware of this fact.
bank to collect small savings from Rama Bhai continued to hand over her
its customers spread over in different savings to him and Keshav misappro-
places on daily basis. Nagamma, a priated them. Rama Bhai realized this
housemaid, was one of such custom- only when Keshav disappeared from
ers making use of Nadan’s service. the scene one day and she sought
Syndicate bank after a couple of years compensation from the bank.
terminated Nandan’s service. Nagam- Possible decision
ma, unaware of this fact, was handing (a) Syndicate Bank shall be liable to com-
over her savings to Nandan who mis- pensate Rama Bhai
appropriated them. Nagamma realized (b) Syndicate Bank shall not be liable to
this nearly after three months, when compensate Rama Bhai
she went to the bank to withdraw (c) Rama Bhai cannot blame others for
money. She filed a complaint against her negligence.
the bank. Possible reasons
Possible decisions (i) Keshav was not an employee of the
(a) Syndicate Bank shall be liable to com- Bank when the fraud was committed.
pensate Nagamma (ii) The Bank was not aware of the special
(b) Syndicate Bank not be liable to com- arrangement between Rama Bhai and
pensate Nagamma Keshav
(c) Nagamma ha to blame herself for her (iii) It is the Bank’s duty to take care of
negligence vulnerable customers.
(iv) Rama Bhai should have checked about
Possible reasons
Keshav in he own interest.
(i) Nandan was not acting in the course
Your decision with the reason
of employment after the termination
(a) (a) (iii) (b) (c) (iv)
of his service.
(c) (b) (ii) (d) (b) (i)
(ii) A person cannot blame others for his
183. Principle
own negligence.
1. A person is liable for negligence, if he
(iii) Nagamma was entitled to be informed
fails to take care of his neighbour’s
by the back about Nandan.
interest
(iv) The bank is entitled to expect its cus-
2. A neighbour is anyone whose inter-
tomers to know actual position.
ests should have been foreseeable by
Your decision with the reason
a reasonable man while carrying on
(a) (b) (i) (b) (c) (ii) his activities.
(c) (a) (iii) (d) (b) (iv) Facts: A cricket match was going on in a
182. Principle closed door stadium. A cricket fan
1. A master shall be liable for the fraudu- who could not get in to the stadium
lent acts of his servants committed in was watching the game by climbing
the course of employment up a nearby tree and sitting there. The
2. Whether an acts is committed in cricket ball in the course of the game
the course of employment has to be went out of the stadium and hit this
judged in the context of the case. person and injured him. He filed a suit
3. Both master and third parties must ex- against the organizers.
ercise reasonable care in this regard. Possible decisions
Facts: Rama Bhai was an uneducated widow (a) The organizers are liable to compen-
and she opened a S.B. account with sate the injured person.

CLAT-6-Previous Years Papers.indd 335 06/09/2010 17:50:08


336 PREVIOUS YEARS’ PAPERS

(b) The organizers ar not liable to com- (iii) The hospital could not have achieved
pensate the injured person. its success without that appendix be-
(c) The injured person should have avoid- longing to the patient.
ed the place where he might be hit by (iv) Everybody must care for and share
the cricket ball with others.
Possible reasons Your decision with the reason
(i) The organizers are responsible for the (a) (a) (i) (b) (a) (ii)
people inside the stadium. (c) (c) (iii) (d) (c) (iv)
(ii) The organizers could not have fore- 185. Principle
seen somebody watching the game by 1. Copying including attempt to copy in
climbing up a tree. examinations is a serious offence.
(iii) A person crazy about something must 2. One shall not take any unauthorized
pay the price for that. materials into the examinations hall.
(iv) The organizers shall be liable to ev-
erybody likely to watch the game. Facts: Rohini, an examinee in PUC, was
Your decision with the reason thoroughly checked while entering
(a) (a) (iv) (b) (a) (iii) into the examination hall. She did not
(c) (b) (ii) (d) (c) (i) have anything other than authorized
184. Principle materials such as pen, instrument box,
1. When a person unlawfully interferes etc., with her. As she was writing he
in the chattel of another person by paper an invigilator found close to
which the latter is deprived of its use, her feet a bunch of chits. The invigi-
the former commits the tort of conver- lator on scrutiny found that the chits
sion. contained answers to the paper being
2. Nobody shall himself at other’s ex- written by Rohini. Rohini’s answers
pense. tallied with the answers in the chits. A
Facts: A patient suffering from stomach ail- charge of copying was leveled against
ment approached a teaching hospital. Rohini.
He was diagnosed as suffering from (a) Rohini shall be punished for copying.
a appendicitis and his appendix was (b) Rohini cannot be punished for copy-
ing.
removed. He become alright. The
Probable reasons
hospital however found some unique
(i) Something lying near the feet does not
cells in the appendix and using the
mean that the person is in possession
cell lines thereof, it developed drugs
of that thing.
of enormous commercial value. When
(ii) The fact that she was checked thor-
the erstwhile patient came to know
oughly while getting into the hall
about it, he claimed a share in the
must be conclusive.
profit made by the hospital.
(iii) Similarities between her answers and
Possible decisions
the answers in the chit indicate that
(a) The hospital need not share its profits
she used those chits.
with the patient
(iv) After using those chits, she must have
(b) The hospital may share its profits on failed to dispose of them properly.
ex gratia basis. Your decision with the reason
(c) The hospital shall share its profits with (a) (a) (iii) (b) (a) (iv)
the patient. (c) (b) (iii) (d) (b) (i)
Possible reasons Instructions: From the four answers given,
(i) The patient, far from being deprived shade the appropriate answer in the space pro-
of the use of his appendix, actually vided for it on the OMR answer sheet.
benefited by its removal. 186. All contracts are agreements all agree-
(ii) The hospital instead of throwing ments are accepted offers.
away the appendix conducted further Which of the following derivation is cor-
research on it on its own and the de- rect?
velopment of drug was the result of its (a) All accepted offers are contracts
own effort. (b) All agreements are contracts

CLAT-6-Previous Years Papers.indd 336 06/09/2010 17:50:08


PREVIOUS YEARS’ PAPERS 337

(c) All contracts are accepted offers 189. A contract contravening public policy is
(d) None of the above void. There cannot be a genera definition
187. No minor can enter into a contract of of public policy.
work. Working in a shop can be done only Which of the following is correct deriva-
by a contract. tion for the above?
Which of the following derivation is cor- (a) There cannot be a general definition
rect? of contract
(a) A minor cannot work in a shop (b) Since public Policy is uncertain, con-
(b) A shop cannot work in a shop tract is also uncertain
(c) There cannot be a contract to which (c) The impact of Public Policy on con-
minor is a party tract is to be judged in individual cas-
(d) None of the above es
188. All motor vehicle are required to have (d) None of the above
third party insurance. Any vehicle not us- 190. International Law is the law between sov-
ing mechanical device is not a motor ve- ereign state. A sovereign is the supreme
hicle. authority not bound by legal constraints.
Which of the following if correct deriva- Which of the following is correct deriva-
tion from the above? tion from the above?
(a) All third party insurances relate to (a) International law is not law binding
motor vehicles on the sovereign states
(b) Vehicles not using mechanical device (b) International law is only a positive
need not have third party insurance morality
(c) All vehicles must have third party in- (c) International law is in the nature of
surance pact between sovereign states
(d) None of the above (d) None of the above

ANSWERS
1. (c) 2. (a) 3. (d) 4. (b) 5. (a) 6. (d) 7. (b) 8. (d) 9. (a) 10. (b)
11. (a) 12. (a) 13. (a) 14. (a) 15. (d) 16. (d) 17. (c) 18. (c) 19. (b) 20. (d)
21. (d) 22. (c) 23. (a) 24. (a) 25. (b) 26. (c) 27. (d) 28. (d) 29. (d) 30. (b)
31. (a) 32. (d) 33. (b) 34. (d) 35. (c) 36. (b) 37. (a) 38. (c) 39. (b) 40. (b)
41. (d) 42. (c) 43. (a) 44. (b) 45. (b) 46. (a) 47. (c) 48. (a) 49. (c) 50. (b)
51. (b) 52. (c) 53. (c) 54. (c) 55. (c) 56. (c) 57. (d) 58. (c) 59. (b) 60. (c)
61. (d) 62. (b) 63. (b) 64. (b) 65. (c) 66. (a) 67. (c) 68. (c) 69. (b) 70. (b)
71. (b) 72. (c) 73. (a) 74. (a) 75. (b) 76. (b) 77. (c) 78. (c) 79. (b) 80. (c)
81. (b) 82. (c) 83. (c) 84. (b) 85. (b) 86. (b) 87. (a) 88. (b) 89. (c) 90. (c)
91. (c) 92. (c) 93. (a) 94. (a) 95. (a) 96. (a) 97. (a) 98. (a) 99. (c) 100. (b)
101. (b) 102. (c) 103. (c) 104. (b) 105. (b) 106. (c) 107. (a) 108. (a) 109. (b) 110. (a)
111. (c) 112. (b) 113. (b) 114. (b) 115. (d) 116. (a) 117. (a) 118. (b) 119. (a) 120. (a)
121. (a) 122. (b) 123. (b) 124. (a) 125. (a) 126. (b) 127. (b) 128. (b) 129. (d) 130. (a)
131. (d) 132. (b) 133. (c) 134. (a) 135. (d) 136. (b) 137. (b) 138. (a) 139. (b) 140. (b)
141. (c) 142. (b) 143. (a) 144. (c) 145. (b) 146. (c) 147. (c) 148. (b) 149. (d) 150. (c)
151. (b) 152. (d) 153. (a) 154. (c) 155. (a) 156. (b) 157. (c) 158. (b) 159. (d) 160. (b)
161. (b) 162. (d) 163. (c) 164. (c) 165. (c) 166. (a) 167. (c) 168. (b) 169. (c) 170. (b)
171. (c) 172. (d) 173. (a) 174. (a) 175. (d) 176. (b) 177. (c) 178. (c) 179. (b) 180. (d)
181. (c) 182. (c) 183. (c) 184. (a) 185. (d) 186. (c) 187. (c) 188. (b) 189. (c) 190. (c)

CLAT-6-Previous Years Papers.indd 337 06/09/2010 17:50:08


338 PREVIOUS YEARS’ PAPERS

LLB ENTRANCE TEST (DU) 2008


1. Who is the present Chancellor of Germa- 9. Which of the following is not a carnivo-
ny? rous plant?
(a) Angela Merkel (b) Helmut Kohl (a) Venus Flytrap (b) Pitcher Plant
(c) Carla Bruni (d) Nicholas Sarkozy (c) Sundew (d) Poison Ivy
2. Which district of China was the worst hit 10. Which of the following is a standard in-
by the earthquake of May 2008? fantry rifle of the Indian Armed Forces?
(a) Beijing (b) Shanghai (a) FN-FAL (b) AK-47
(c) Sichuan (d) Tangshan (c) INSAS (d) Sten Gun
3. Which of the following was/is not a space 11. Becquerel is a unit of
station programme? (a) Sound/loudness
(a) Salyut (b) Soyuz (b) Radioactivity
(c) ISS (d) Mir (c) Pressure
4. Who is a genealogist? (d) Humidity
(a) A person who creates a DNA gene 12. Which was the first country to abolish
map of individuals capital punishment?
(b) A person who studies the total gene (a) United States of America
pool of an ecosystem (b) Australia
(c) A person who studies and traces fam- (c) Venezuela
ily lineages (d) Netherlands
(d) A person who locates individuals with 13. Which of the following is associated with
IQ in the range of geniuses in a sam- what is considered as one of the most
ple population devastating oil spill in terms of damage
5. What is geriatrics? caused to the environment?
(a) Branch of science focusing on the (a) Exxon Valdez
study of genes and DNA (b) Three Mile Island
(b) Branch of medicine focusing on health (c) 1991 Gulf War
care of elderly (d) Easter Island
(c) Branch of medicine dealing with care 14. What is a ‘Plug-in Hybrid Electrical
of infants and children Vehicle’?
(d) Branch of medicine dealing with (a) An automobile running on hydrogen
germs and microbes fuel
6. Who were the recipients of the 2007 No- (b) An automobile running on batteries
bel Prize for Peace? and solar cells
(a) Intergovernmental Panel on Climate (c) An automobile running alternately on
Change (IPCC) and Al Gore batteries and gasoline engine, as re-
(b) Muhammad Yunus and Grameen quired
Bank (d) An automobile running on rails with
(c) International Atomic Energy Agency overhead electric lines
(IAEA) and Mohamed ElBaradei 15. Who is the author of the book “A Brief
(d) Albert Fert and Peter Grünberg History of Time”?
7. What was the code name of the second (a) Bill Bryson
nuclear weapon used by the United States (b) Stephen Hawking
of America against Japan in the Second (c) Douglas Adams
World War? (d) Issac Assimov
(a) Fat Man (b) Manhattan 16. Which one of the following expressions is
(c) McDonald (d) Trinity correct?
8. Who is credited with the depiction of (a) The scenes of Kashmir are charming
‘The Common Man’ through his cartoon (b) The sceneries of Kashmir are charm-
strips? ing
(a) R. K. Narayan (b) K. R. Narayanan (c) The scenery of Kashmir is charming
(c) R. K. Laxman (d) Pran (d) The scene of Kashmir is charming

CLAT-6-Previous Years Papers.indd 338 06/09/2010 17:50:09


PREVIOUS YEARS’ PAPERS 339

17. Which one of the following is a funda- 27. Find out the correct spelling.
mental duty of citizens? (a) jewelery (b) jewellry
(a) To sing National Anthem (c) jwellery (d) jewellery
(b) To pay equal wages to men and wom- 28. Find out the correct spelling
en (a) prevalant (b) pravalent
(c) Guardians to provide for education (c) prevelant (d) prevalent
to children between 6 and 14 years of 29. Poets often write in a __________ mood.
age (a) disgusting (b) critical
(d) To organize village panchayats (c) sensible (d) reflective
18. Which one of the following does not relate 30. Your remark is wide ________ the mark.
to a company? (a) off (b) of
(a) Articles of association (c) around (d) upon
(b) Prospectus 31. A “pandora’s box” is so called because
(c) Dissolution (a) it belonged to the Pandavas
(d) Winding up (b) it is made at Pandara road
19. Which one of the following does not relate (c) it contains undergarments
to environmental protection? (d) it contains all the gifts
(a) Public Liability Insurance 32. Which one of the following expressions is
(b) National Environment Appellate correct?
Authority (a) One of the boy are absent
(c) National Environment Tribunal (b) One of the boys is absent
(d) National Highway Authority (c) One of the boys are absent
20. The adult suffrage has been provided (d) One of the boys is absent
under 33. Who is the present Chairperson of the
(a) Constitution of India Planning Commission?
(b) Representation of Peoples Act (a) Amartya Sen
(c) Right to Information Act (b) Montek Singh Ahluwalia
(d) Indian Majority Act (c) Sonia Gandhi
21. I can say this ___________ his face. (d) Manmohan Singh
(a) on (b) upon 34. What is an anagram?
(c) over (d) to (a) The result of rearranging the letters
22. _________ MBBS doctor is qualified to of a word or phrase to produce a new
practise medicine. word or phrase
(a) A (b) The (b) A geometrical shape with ‘n’ number
(c) An (d) Every of sides
23. Nothing can be achieved without (c) A telegraph transmitted via analogous
_________ a certain. mode
(a) getting (b) sacrificing (d) A technique for constructing holo-
(c) enduring (d) imbibing grams
24. He is so __________ of his own intelli- 35. Who is the author of the books “Three
gence that he will not entertain any sug- Men in a Boat” and “Idle Thoughts of an
gestion from others. Idle Fellow”?
(a) hopeful (b) enchanted (a) Jerome K. Jerome
(c) jealous (d) enamoured (b) Ruskin Bond
25. He was about to move his bike into the (c) Rudyard Kipling
compound of his apartment when a pass- (d) O. Henry
ersby ___________ down the motorbike. 36. Who created the popular fictional charac-
(a) Forced (b) fell ter, Jeeves?
(c) knocked (d) climbed (a) P. G. Wodehouse
26. The security of the minister has been (b) Jerome K. Jerome.
_________ up following an attack at a (c) The creators of www.askjeeves.com
public meeting. (d) Charles Dickens
(a) speeded (b) bloated 37. What is the number of States in the Union
(c) steered (d) beefed of India?

CLAT-6-Previous Years Papers.indd 339 06/09/2010 17:50:09


340 PREVIOUS YEARS’ PAPERS

(a) 28 (b) 32 (c) Battle of Gettysburg


(c) 26 (d) 27 (d) Battle of Lexington and Concord
38. The Tarapur Atomic Power Station was 46. Who is the author of the short story “The
established under an Agreement with Last Leaf”?
(a) Britain (a) O. Henry (b) Agatha Christie
(b) Soviet Union (c) Ruskin Bond (d) Jeffery Archer
(c) United States of America 47. Who is the present United States Secretary
(d) Canada of State?
39. Metals that can be easily beaten into sheets (a) Colin Powell
are known as (b) Condoleezza Rice
(a) Beatable (b) Malleable (c) Hillary Clinton
(c) Polymers (d) Ductile (d) Dick Cheney
40. Why does a freshly cut apple change color 48. Which of the following statements regard-
after some time? ing ISRO’s April 2008 rocket launch is
(a) Due to growth of sugar degrading incorrect?
bacteria in the apple (a) The launch was carried out using
(b) Due to bacterial defense mechanism PSLV-C9 rocket
of the exposed fruit tissue (b) The rocket carried 10 satellites at the
(c) Due to oxidation reaction same time
(d) All of the above (c) The rocket was launched from Anda-
41. Who is the Prime Minister of Pakistan fol- man and Nicobar islands
lowing the 2008 general elections? (d) The rocket carried Indian as well as
(a) Asif Zardari international satellites
(b) Nawaz Sharif 49. Each of the following personalities were
(c) Yousaf Raza Gillani victims of assassination except one. Mark
(d) Altaf Hussain the one.
42. Who is the present Chief of Naval Staff of (a) Indira Gandhi
India? (b) John F. Kennedy
(a) Admiral Vishnu Bhagwat (c) Zulfikar Ali Bhutto
(b) Admiral S. M. Nanda (d) Abraham Lincoln
(c) Admiral Sureesh Mehta 50. Which is the current Five-year Plan for In-
(d) Admiral Vijai Singh Shekhawat. dia?
43. What is the meaning of the phrase ‘ubi jus (a) Twelfth Five-year Plan
ibi remedium’? (b) Tenth Five-year Plan
(a) Justice for all is the remedy of social (c) Eleventh Five-year Plan
evils (d) Eighth Five-year Plan
(b) Where there is a right, there is a rem- 51. Which one of the following statements
edy concerning the Rajya Sabha is incorrect?
(c) Complete justice requires speedy rem- (a) The Vice President of India is its ex-
edy officio Chairman
(d) Justice must not only be done but (b) Sh. Somnath Chatterjee is its present
seem to be done Speaker
44. Which one of the following pairs of mili- (c) Sh. Mohammad Hamid Ansari is its
tary honors is matched incorrectly present Chairman
(a) Param Vir Chakra : India (d) Under certain circumstances, the
(b) Victoria Cross : Britain Chairman is not allowed to preside
(c) Medal of Honor : United States of over the proceedings even if he is
America present
(d) Legion of Honor : Germany 52. In whose presence does the President of
45. Which of the following is not associated India subscribe to the oath of affirmation?
with the American Revolution for inde- (a) The Chief of Army Staff of the Indian
pendence? Army
(a) No taxation without representation (b) The Chief of Staffs of all the three ser-
(b) Boston Massacre vices

CLAT-6-Previous Years Papers.indd 340 06/09/2010 17:50:09


PREVIOUS YEARS’ PAPERS 341

(c) The Speaker of Lok Sabha trary to law: provided that the thing which
(d) The Chief Justice of India intoxicated him was administered to him
53. You purchased a mobile phone handset without his knowledge or against his will.
which stopped working in its guarantee Factual Situation: ‘A’ was having a get
period. The mobile handset manufacturer together with his old friends and on
is refusing to repair or replace the same. his friends suggestions, he consumed
What would you do to get justice quick- some alcohol. On his way back to
ly? home at home at night, ‘A’ heard
(a) Approach the civil court with a suit some footsteps and turning back, he
for declaration imagined a figure moving towards
(b) File a writ petition before the Delhi him with a spear. In fact, it was only
High Court a man, ‘B’ with an umbrella who was
(c) Approach the consumer disputes re- telling ‘A’ to walk carefully since ‘A’
dressal forum appeared to be unsteady. However, ‘A’
(d) Make a complaint of cheating before proceeded to attack ‘B’ with an iron
the local police station rod leading to grave injuries to ‘B’. Is
54. Principle: No person shall be prosecuted ‘A’ guilty of causing grevious hurt to
and punished for the same offence twice. ‘B’?
Factual Situation: ‘A’, a kleptomaniac, (a) No, ‘A’ is not guilty because in his in-
was an acquaintance of ‘B’ and used toxicated state, the umbrella appeared
to visit ‘B’s’ home on weekends. One a spear to him and he exercised his
day, ‘B’ was unable to find his watch right of private defence.
after a visit from ‘A’. Ultimately, ‘B’ (b) No, ‘A’ is not guilty because ‘B’ could
informed the police about the missing have attacked ‘A’ with his umbrella.
watch and the persons who had been (c) No, ‘A’ is not guilty because he was
to his room. The police, among oth- intoxicated on the suggestions of his
ers, made a search on person of ‘A’ friends and was incapable of knowing
and found the watch in his pocket. At that he was savagely attacking a man
his trial, the Court found him guilty of who was carrying only an umbrella.
theft and sentenced him to 3 months in (d) Yes, ‘A’ is guilty because he got in-
prison. After completing his sentence, toxicated voluntarily and under the ef-
‘A’ went to ‘B’s’ home to apologize fect of this voluntary intoxication, he
for his conduct. After ‘A’ left, it was attacked and caused grievous injuries
discovered that ‘B’s’ watch was miss- to ‘B’ who posed no threat to him in
ing. ‘A’ informed the police who lo- fact.
cated the watch from ‘A’s’ home. Can 56. Principle: Nothing is an offence which is
‘A’ be prosecuted for theft? done in the exercise of the right of private
(a) No, he has already been prosecuted defence. Every person has a right to de-
and punished for theft and cannot be fend his property against any act of theft,
punished for the same again. robbery, mischief or criminal trespass.
(b) No, he is a kleptomaniac with a psy- This right of private defence of property
chological disorder of stealing and extends to causing of death of the wrong-
therefore cannot be found guilty. doer, if the person exercising the right ap-
(c) Yes, he can be prosecuted because he prehends that death or grievous hurt shall
has committed two separate acts of be the consequence is such right of private
theft. defence is not exercised.
(d) Yes, he can be prosecuted because the Factual Situation: ‘A’s cattle was being
recovery of the stolen goods in each regularly stolen and ‘A’ was unable
case has been from separate places. to apprehend the thief. One night, ‘A’
55. Principle: Nothing is an offence which is finally manages to catch ‘B’ untying
done by a person who, at the time of doing his cow from the cowshed under the
it, is, by reason of intoxication, incapable cover of darkness. ‘A’ slowly crept
of knowing the nature of the act, or that up to ‘B’ and slashed his neck with a
he is doing what is either wrong, or con- sickle leading to the death of ‘B’. Is

CLAT-6-Previous Years Papers.indd 341 06/09/2010 17:50:09


342 PREVIOUS YEARS’ PAPERS

‘A’ guilty of the offence of culpable (a) Mandamus (b) Habeas Corpus
homicide. (c) Rule Nisi (d) Certiorari
(a) No, ‘A’ was only exercising his right 65. Who was the king of France at the time of
of private defence of property. French Revolution?
(b) No, ‘B’s’ continued stealing of his (a) Louis XVI
cattle would have rendered his busi- (b) Louis XIV
ness inoperable. (c) Napoleon Bonaparte
(c) Yes, ‘A’ had no reasonable apprehen- (d) George II
sion that ‘A’ could suffer any grievous 66. Under which Article of the Constitution
hurt if he did not kill ‘B’. of India is the law laid down by Supreme
(d) Yes, ‘A’ should have first challenged Court binding on all courts?
‘B’ to surrender before taking any (a) Article 32 (b) Article 141
steps to cause ‘B’s’ death. (c) Article 226 (d) Article 124
57. Which of the following is not associated 67. Which country recently planted a flag on
with the phenomena of Bioluminescence? the sea floor at North Pole?
(a) Jack O’Lantern mushroom (a) United States of America
(b) Gulper eel (b) France
(c) Collosal Squid (c) Russia
(d) Rattle snake (d) Australia
58. Who was the first space tourist? 68. Which of the following is an easily digest-
(a) Dennis Tito (b) Josip Broz Tito ible source of protein?
(c) Yuri Gagarin (d) Gregory Olsen (a) Egg albumin (b) Soybean
59. Which one of the following is not estab- (c) Fish (d) Red meat
lished under the constitutional provisions 69. Which countries entered into the
(a) Finance Commission Panchsheel series of Agreements?
(b) Inter-state Council (a) China, Soviet Union and India
(c) Planning Commission (b) China and India
(d) Scheduled Caste Commission (c) United States of America and India
60. Who is the Chairman of the 13th Finance (d) Russia and India
Commission? 70. What was the Liberhan Commission of
Inquiry constituted for?
(a) Vijay Kelkar (b) K. C. Pant
(a) To conduct an inquiry of the riots
(c) C. Rangarajan (d) N. K. P. Salve
against the Sikh community in 1984
61. Bay of Pigs is associated with:
(b) To conduct an inquiry into the demoli-
(a) The invasion of Cuba by United States
tion of the Babri Masjid in 1992
of America marines
(c) To conduct an inquiry of the Godhra
(b) The invasion of Falklands by Argen-
riots of 2002
tina
(d) To conduct an inquiry of the Mumbai
(c) The invasion of Kuwait by Iraq
riots of 1992-93
(d) The invasion of Cuba by Cuban ex-
71. Which organization of the United Nations
iles
has child welfare as its primary goal?
62. Who was known as the ‘Desert Fox’?
(a) UNESCO
(a) Adolf Hitler (b) UNICEF
(b) Manfred von Richthofen (c) ILO
(c) Erwin Rommel (d) General Assembly
(d) George S. Patton 72. Which chemical is responsible for the
63. The ‘eco-mark’ is given to characteristic odor of garlic?
(a) high quality products (a) Sulfur
(b) environmental friendly products (b) Chlorine
(c) electrical appliances with BIS mark (c) Phosphorus
(d) high quality products which are (d) Hydrogen Sulfide
cheap 73. In which year was the Suez Canal opened
64. Which of the following is not in the na- for navigation?
ture of a writ issued by the Constitutional (a) 1888 (b) 1882
courts of India? (c) 1936 (d) 1869

CLAT-6-Previous Years Papers.indd 342 06/09/2010 17:50:09


PREVIOUS YEARS’ PAPERS 343

74. What is the water channel separating India (b) Judgment of the Delhi High Court in
from Sri Lanka known as? ‘Priyadarshini Mattoo’ case
(a) Gulf of Mannar (c) The judgment of a designated TADA
(b) Palk Strait court in 1993 Mumbai blasts case
(c) Adam’s Channel (d) Judgment of the Supreme Court of
(d) Ram Setu India upholding 27% quota for OBC
75. Which of the following is not a member of candidates in educational institutions
ASEAN? 81. Up to what age does a judge of the Su-
(a) Vietnam (b) Thailand preme Court of India hold office?
(c) Myanmar (d) India (a) 60 (b) 62
76. Which of the following countries do not (c) 65 (d) 67
fall within the collective expression, 82. Which of the following statements about
‘Scandinavian Countries’? Right to Information Act is not true?
(a) Norway (b) Iceland (a) It was enacted to promote transparen-
(c) Sweden (d) Switzerland cy and accountability in the working
77. Which of the following laws do not entitle of every public authority
a parent to maintenance (b) It protects the information available to
(a) Criminal Procedure Code, 1973 a person in his fiduciary relationship
(b) Guardians and Wards Act, 1890 (c) It repealed the Official Secrets Act,
(c) Maintenance and Welfare of Parents 1923
and Senior Citizens Act, 2007 (d) It puts every public authority under an
(d) All of them contain provisions for obligation to appoint Public Informa-
payment of maintenance to a parent tion Officer
78. In what context was the term “judicial 83. Who is known as the framer of the Indian
overreach” used recently? Penal Code, 1872?
(a) The powers of Judiciary to bring to (a) Lord Hastings
justice even the citizens of India out- (b) Lord Denning
side India, for instance through the (c) Lord Macaulay
extra territorial operation of the Indian (d) Lord Wilberforce
Penal Code 84. Who among the following judges gave
(b) The jurisdiction of Judiciary on a dissenting opinion in the case of ADM
MNCs, as seen in the Bhopal gas leak Jabalpur vs Shukla also known as the Ha-
case where the subsidiary of a com- beas Corpus case?
pany incorporated in the United States (a) Justice P. N. Bhagwati
had to submit to the jurisdiction of (b) Justice V. R. Krishna Iyer
courts in India (c) Justice H. R. Khanna
(c) The High Court in a state of the union (d) Justice A. N. Sen
hearing a matter falling in the territo- 85. Which of the following is not associ-
rial jurisdiction of another state’s High ated with the Constitution (Thirty-Ninth
Court Amendment Act)?
(d) The expanding realm of public inter- (a) It placed the Coal Mines (Nation-
est litigation and judicial activism in alisation) Act, 1973 under the Ninth
India Schedule of the Constitution of India
79. Who is the present Chairman of the Law (b) It ousted the jurisdiction of any court
Commission of India? to decide a question on the validity of
(a) Justice A. R.Lakshmanan election of a person as President of In-
(b) Justice B. P. Jeevan Reddy dia
(c) Justice K. G. Balakrishnan (c) It amended Article 217 of the Consti-
(d) Justice A. S. Anand tution of India
80. Which of the following judgments high- (d) It was enacted in the year 1975
lighted in the media was not delivered in 86. A provision in a statute pertaining to med-
the year 2007? ical institutions in India was struck down
(a) Judgment of a Delhi court in ‘Uphaar by the Supreme Court of India in the year
Fire’ case 2008 as being ultra vires the Constitution

CLAT-6-Previous Years Papers.indd 343 06/09/2010 17:50:09


344 PREVIOUS YEARS’ PAPERS

of India. Which of the following statutes (b) Slave Dynasty


contained that provision? (c) Tughlak Dynasty
(a) All India Institute of Medical Sciences (d) Lodi Dynasty
Act 94. With which country does India share the
(b) Lady Hardinge Medical College and longest border?
Hospital (Acquisition and Miscella- (a) Bangladesh (b) China
neous Provisions) Act (c) Pakistan (d) Nepal
(c) Post-Graduate Institute of Medical 95. In which police station was the FIR relat-
Education and Research, Chandigarh, ing to the assassination of Mahatma Gan-
Act dhi registered?
(d) Sree Chitra Tirunal Institute for Medi- (a) Parliament Street Police Station
cal Sciences and Technology, Trivan- (b) Daryaganj Police Station
drum Act (c) Tuglaq Road Police Station
87. Who was appointed by the NDA govern- (d) None of the above
ment as the Chairperson of the National 96. What is the maximum strength of mem-
Commission to Review the Working of the bers of the House of the People chosen by
Constitution of India? direct election under the Constitution of
(a) Justice Y. K. Sabharwal India?
(b) Justice Rangnath Mishra (a) 530 (b) 550
(c) Justice M. N. Venkatachaliah (c) 552 (d) 540
(d) Justice A. R. Lakshmanan 97. Which of the following is an Open-Source
88. Which country won Davis Cup in Decem- computer operating system?
ber, 2007 (a) MAC OS-X (b) Windows 98
(a) United Sates of America (c) Linux (d) Symbian OS
(b) Germany 98. The principle of ‘collective responsibility’
(c) Russia means
(d) United Kingdom (a) all citizens must collectively elect
89. In which town was Rajiv Gandhi assassi- their representatives
nated? (b) all members of the union council of
(a) Sriharikota ministers are collectively responsible
to the House of the People
(b) Sriperumbudur
(c) all Ministers are accountable to the
(c) Thiruanantpuram
Parliament
(d) Chembur
(d) all ministers hold office at the plea-
90. Who among the following was not in the
sure of the Prime Minister.
top 10 of the Forbes 2007 list of 100 Most
99. Who is the author of the book “Autobiog-
Powerful Women?
raphy of an Unknown Indian”
(a) Smt. Sonia Gandhi
(a) V. S. Naipaul
(b) Indra K. Nooyi
(b) Barbara Tuchman
(c) Condoleezza Rice
(c) Nirad C. Chaudhari
(d) Hillary Rodham Clinton
(d) Kuldeep Nayyar
91. What is the number of cervical vertebrae
100. Who is the author of the book “A Judge’s
in humans? Miscellany”?
(a) 24 (b) 7 (a) Justice H. R. Khanna
(c) 5 (d) 12 (b) Justice M. Hidayatullah
92. Which place in China is associated with (c) Justice Dalveer Bhandari
a series of pro-democractic protests in (d) Lord M. R. Dennings
1989? 101. Who was the judge who held that funda-
(a) Red Square mental rights were not amendable?
(b) Tiananmen Square (a) Justice P. N. Bhagwati
(c) Shanghai People’s Square (b) Justice O. Chinnappa Reddy
(d) Trafalgar Square (c) Justice K. K. Mathew
93. Which dynasty did Razia Sultan belong (d) Justice K. Subba Rao
to? 102. Under which law a minor is incapable of
(a) Khilji Dynasty entering into contract?

CLAT-6-Previous Years Papers.indd 344 06/09/2010 17:50:09


PREVIOUS YEARS’ PAPERS 345

(a) Indian Contract Act, 1872 110. Which Mughal Emperor prohibited the
(b) Majority Act, 1875 use of tobacco?
(c) Guardians and Wards Act, 1890 (a) Babur (b) Akbar
(d) Contract Labour ((Regulation and (c) Jahangir (d) Shahjahan
Abolition) Act, 1970 111. The trade practice known as “minimum
103. The prohibition against employment of resale price maintenance” is prohibited
children below 14 years of age has been by
provided (a) Monopolies and Restrictive Trade
(a) as a fundamental right against exploi- Practices Act, 1969
tation in the Constitution (b) Competition Act, 2002
(b) as a right to equality (c) Consumer Protection Act, 1986
(c) as a directive principle of state policy (d) Multi-State Co-operative Societies
(d) as a duty on the parents and guard- Act, 2002
ians 112. What is the official Chinese news agency
104. How many fundamental rights have been known as?
guaranteed under the Constitution of In- (a) ITAR-TASS (b) AFP
dia? (c) Xinhua (d) Hu-Gin Group
(a) 5 (b) 6 113. The workers’ petitions and protests ______
(c) 7 (d) 8 a change in management policies.
105. Who has power to decide disputes pertain- (a) affected (b) effected
ing to disqualification of a member of Par- (c) lead (d) altered
liament? 114. The private detective deliberately chose
(a) Speaker of Lok Sabha _______ clothes.
(b) President of India (a) decrepit (b) nondescript
(c) Chairman of Rajya Sabha (c) flashy (d) derisive
(d) President of India on the advice of 115. The ship managed to survive the nasty
Election Commission weather by casting its goods overboard,
106. The Raghvan Committee relates to but this _______ will pose a nuisance to
(a) students’ union elections the residents on the shore.
(b) ragging in educational institutions (a) traffic (b) jetsam
(c) reservations for OBC in educational (c) flotsam (d) noise
institutions 116. The police investigator tried hard to
(d) sexual harassment at workplace _________ some response from the crimi-
107. D. K. Basu vs State of West Bengal deals nal but the criminal refused to divulge any
with information of his ________ operation.
(a) safeguards for arrested persons (a) elicit, illicit
(b) safeguards against sexual exploita- (b) illicit, elicit
tion (c) illicit, illegitimate
(c) safeguards for children (d) get, elicit
(d) safeguards for unorganized workers 117. The Minister’s speech made _________ to
108. In which landmark judgment did the Su- the unpublished report.
preme Court of India lay down guidelines (a) illusions (b) allusions
against sexual harassment of women at (c) delusions (d) perusals
workplace? 118. Which is the following is not a synonym
(a) Nilabati Behera vs State of Orissa to the word ‘Parochial’?
(b) Vishakha vs State of Rajasthan (a) Provincial (b) Insular
(c) Maneka Gandhi vs Union of India. (c) Limited (d) Preferable
(d) Hussainara Khatoon vs State of Bihar 119. Which is the following is not a synonym
109. To whom does the slogan “Jai Jawan, Jai to the word ‘Parsimonious’?
Kisan, Jai Vigyan” relate? (a) Penurious (b) Miserly
(a) Shri Lal Bahadur Shastri (c) Acrimonious (d) Niggard
(b) Shri Morarji Desai 120. A drives 30 km south. Then he turns right
(c) Smt. Indira Gandhi and drives another 30 km. Turning left
(d) Shri Atal Behari Bajpayee from here, he drives 20 km. Finally turn-

CLAT-6-Previous Years Papers.indd 345 06/09/2010 17:50:09


346 PREVIOUS YEARS’ PAPERS

ing left, he drives 30 km. How far is A in respect of Basmati rice and Neem to
from his starting position? United States based firms is an example of
(a) 30 km (b) 20 km what may be termed as
(c) 80 km (d) None of these (a) Infringement of a patent
121. A is more hard working than B but he is (b) Trademark violation
not as hard working as C. D is less hard (c) Bio-piracy
working than C but more hard working (d) Passing off
than A. If C is the most hard working in- 126. Which was the first space shuttle to make
dividual, who is the least hard working of a successful flight after the Columbia di-
them? saster?
(a) B (b) A (a) Endeavor (b) Discovery
(c) C (d) D (c) Titan V (d) Mars Rover
122. In the question below are given two state- 127. The inscriptions of Asoka haven been
ments followed by four conclusions num- found in the following languages:
bered I, II, III, and IV. You have to take (a) Magadhi, Sanskrit and Greek
the two given statements to be true even if (b) Magadhi
they seem to be at variance from common- (c) Magadhi, Sanskrit, Greek and Ara-
ly known facts. Read the conclusions and maic
then decide which of the given conclusions (d) Magadhi and Greek
logically follow the two given statements, 128. Which of the following was the first to ac-
disregarding commonly known facts. quire a Superjumbo, Airbus A380?
Statements: (a) Luftwaffe
A. All statutes of legislature are law of the (b) Lufthansa
land. (c) Singapore Airlines
B. Judge made law is laid down by the (d) British Airways
Courts. 129. Who among the following is the present
Conclusions: President of the African National Con-
I. Constitution is a statute. gress?
II. Both statutes and judge made law form (a) Thabo Mbeki
the law of the land. (b) Ban ki Moon
III. Courts make statutes. (c) Nelson Mandela
IV. Judge made law is superior to statutes. (d) Jacob Zuma
(a) Both I and IV follow 130. The Archaeological Survey of India func-
(b) Only IV follows tions under the
(c) Only II follows (a) Ministry of Culture
(d) None follows (b) Department of Science and Technol-
123. The US-EU Safe Harbour Arragement re- ogy
fers to (c) Ministry of Tourism
(a) Mutual guarantees for safety and se- (d) Ministry of Human Resource Devel-
curity to the ships of one country at opment
the ports of the other country 131. Which blood group is also known as the
(b) An understanding that EU shall extra- universal donor type?
dite criminals wanted in the US (a) O Positive (b) AB Positive
(c) An method for US companies to ad- (c) O Negative (d) AB Negative
here to the privacy principles of the 132. Who is the Managing Director of the Del-
EU laws hi Metro Rail Corporation?
(d) None of the above (a) E. Sreedharan
124. Which of the following is the oldest High (b) S. Sreesanth
Court in India? (c) Kalyan C. Jena
(a) High Court of Madras (d) Sushma Chawla
(b) High Court of Calcutta 133. What does ‘time travel paradox’ in popu-
(c) High Court of Delhi lar fiction refer to?
(d) High Court of Allahabad (a) Time travel may be possible through
125. The controversy regarding grant of patent black holes, but the gravitational force

CLAT-6-Previous Years Papers.indd 346 06/09/2010 17:50:09


PREVIOUS YEARS’ PAPERS 347

of black holes is so strong that it does versity of Delhi when India attained inde-
not allow even light to escape pendence?
(b) An individual may go back in time (a) V. K. R. V. Rao
and act in such a way as to change the (b) B. N. Ganguli
course of the future (c) Maurice Gwyer
(c) Time travel requires traveling faster (d) Hari Singh Gaur
than the speed of light, which is im- 142. Which is the largest moon of Neptune?
possible under the classical laws of (a) Nereid (b) Triton
physics (c) Trinitron (d) Proteus
(d) Time travel may be possible through 143. Which ancient Roman city was destroyed
black holes, but the human body is by the eruption of Mount Vesuvius?
incapable of taking the gravitational (a) Paestum (b) Pompeii
stress of a passage through a black (c) Helike (d) Teotihuacan
hole, making time travel practically 144. Which of the following bodies is entrusted
impossible with the statutory duty of laying down the
134. Which was the first new sovereign state of standards of professional conduct and eti-
the twenty first century? quette for advocates in India?
(a) Yugoslavia (b) Belarus (a) Bar Association of India
(c) East Timor (d) Tibet (b) Bar Council of India
135. On which organism did Thomas Morgan (c) Advocates Association of India
conduct his experiments, leading to the (d) Bar Advocates Council of India
award of 1933 Nobel Prize in medicine 145. The President of India is elected by
for identifying chromosomes? (a) members of Parliament
(a) Pea plant (b) Earthworm (b) elected members of Parliament
(c) Fruit fly (d) Bacteria (c) elected members of both Houses of
136. When was the Faculty of Law of the Uni- Parliament and state legislatures
versity of Delhi established? (d) elected members of both Houses of
(a) 1915 (b) 1962 Parliament and elected members of
(c) 1950 (d) 1924 the Legislative Assemblies of the
137. Which is the highest peak of the Alps states
146. “Rule of Law” means
range?
(a) all persons are equal in the eyes of
(a) Mont Blanc (b) Monte Rosa
law
(c) Monte Carlo (d) Piz Bernina
(b) treating all unequally as equals
138. Which of the following organizations was
(c) working according to law
instrumental in bringing numerous copy-
(d) distributing state largesse to everyone
right infringement lawsuits against indi-
in equal proportion
viduals in the United States of America,
147. Point out the member of Parliament who
for sharing music online?
was held disqualified to remain as member
(a) Federal Bureau of Investigation
for having held an office of profit
(b) Hollywood Actors Association
(a) Amitabh Bachchan
(c) Recording Industry Association of
(b) Amar Singh
America (c) Jaya Bchchan
(d) Musicians Pan America Association (d) Jaya Prada
139. Which of the following is not a fundamen- 148. Which University has been ranked 254th
tal right? in “Top 500 World University Rankings
(a) Right to property for the year 2007?
(b) Right to information (a) Bombay University
(c) Right to life and liberty (b) Shantiniketan
(d) Both (a) and (b) (c) University of Delhi
140. Where was the 13th ASEAN Summit (d) Calcutta University
held? 149. Which one of the statements is correct?
(a) Bali (b) New Delhi (a) The Constitution of India contains a
(c) Singapore (d) Bangkok directive principle about uniform civil
141. Who was the Vice Chancellor of the Uni- code

CLAT-6-Previous Years Papers.indd 347 06/09/2010 17:50:09


348 PREVIOUS YEARS’ PAPERS

(b) The Constitution of India imposes a 159. 11th May is celebrated as


duty on every citizen to support uni- (a) Electronics Day
form civil code (b) Technology Day
(c) The Constitution of India contains a (c) Invention Day
fundamental right assuring the citi- (d) Environment Day
zens that the state shall bring about 160. A consumer cannot file a complaint before
uniform civil code the consumer disputes redressal forum
(d) None of the above statements is cor- after the expiry of _______ from the date
rect when the cause of action arose
150. The marriage of a 15 year old girl with a (a) one year (b) two years
45 year old man is (c) three years (d) four years
(a) valid 161. A gas engine was invented by
(b) invalid (a) Daimler (b) Davy
(c) Diesel (d) Charles
(c) voidable at the option of the girl
162. The safety fuse wire used in domestic
(d) voidable at the will of the man
electrical appliances is made of metal of
151. The nature of winter rainfall in north-
low
western India is
(a) resistance
(a) orographic (b) cyclonic
(b) melting point
(c) monsoonal (d) convectional (c) specific gravity
152. World Earth Day is celebrated on (d) None of the above
(a) 14 February (b) 11 May 163. Chemically, dry ice is
(c) 11 July (d) 22 April (a) kept at sub-zero temperatures
153. The Mediterranean type climate is charac- (b) formed from pure distilled water
terized by (c) solid carbon dioxide
(a) hot summers and wet winters (d) solid sulphur dioxide
(b) Warm winters and cold summers 164. Which one of the following is not the prin-
(c) Rains throughout the year cipal organ of the United Nations?
(d) Wet summers and dry winter (a) World Health Organization
154. Which one of the following is not relevant (b) Trusteeship Council
for determining the climate of a place? (c) Security Council
(a) Construction of pucca houses (d) General Assembly
(b) Nature of the soil 165. The resolution against international ter-
(c) Forests rorism was passed by the United Nations
(d) Distance from equator subsequent to terror attack on WTC and
155. The shape of earth is Pentagon in
(a) a circular disc (a) October 2001
(b) a sphere (b) September 2001
(c) a tetrahedron (c) December 2001
(d) an oblate spheroid (d) January 2002
156. At the equator, the duration of the day is 166. The major tribal group in Bihar consists
(a) 16 hours (b) 14 hours of
(c) 12 hours (d) 10 hours (a) Todas (b) Bhils
157. Find out incorrect pair (c) Santhals (d) None of the above
167. India’s first batsman to score double cen-
(a) cataract—eyes
tury in five test matches is
(b) jaundice—liver
(a) Kapil Dev
(c) pheumonia—lungs
(b) Sunil Gavaskar
(d) pyorrhea—bones
(c) Sachin Tendulkar
158. Which one of the following is not a symp-
(d) Rahul Dravid
tom of heart attack? 168. The first woman Chief Minister of a state
(a) Nausea and sweating (a) Mayawati
(b) Numbness and aching (b) Jayalalita
(c) Squeezy feeling in chest (c) Sucheta Kriplani
(d) Pain in legs (d) Sarojini Naidu

CLAT-6-Previous Years Papers.indd 348 06/09/2010 17:50:09


PREVIOUS YEARS’ PAPERS 349

169. The National Institute of Oceanography is 172. When Shri Atal Behari Bajpayee became
located in Prime Minister for the first time, the Presi-
(a) Andhra Pradesh dent of India was
(b) Tamil Nadu (a) R. Venkatraman
(c) Kolkata (b) S. D. Sharma
(d) Goa (c) K. R. Narayanan
170. Which one of the following statements is (d) A. P. J. Abdul Kalam
incorrect? 173. Who was the leader of Gadar Party?
(a) The Comptroller and Auditor General (a) Bhagat Singh
of India (CAG) is appointed as per the (b) Lala Haydayal
Constitution of India. (c) Balgangadhar Tilak
(b) The CAG is appointed by the Presi- (d) Abul Kalaam Azad
dent of India. 174. Pt Madan Mohan Malviya was the founder
(c) The CAG advises the President of In- of
dia as to the form in which accounts of (a) Banaras Hindu University
the Union and states are to be kept. (b) Delhi University
(d) The CAG submits a report to the Par- (c) Kashi Vidyapeeth
liament. (d) Allahabad University
171. The system of diarchy, decentralization 175. Point out the wrong match:
and bicameral legislature was introduced The Bhakti Movement in the Middle Ages
in India by in India was started by
(a) Government of India Act, 1919 (a) Tukaram—in Punjab
(b) Government of India Act, 1935 (b) Kabir—in U.P.
(c) Indian Councils Act, 1909 (c) Jaidev—Rajasthan
(d) Constitution of India, 1950 (d) Chaitanya—in Bengal

ANSWERS
1. (a) 2. (c) 3. (b) 4. (c) 5. (b) 6. (a) 7. (a) 8. (c) 9. (d) 10. (c)
11. (b) 12. (c) 13. (a) 14. (c) 15. (b) 16. (c) 17. (c) 18. (c) 19. (d) 20. (a)
21. (d) 22. (c) 23. (c) 24. (d) 25. (c) 26. (d) 27. (d) 28. (d) 29. (d) 30. (b)
31. (d) 32. (d) 33. (d) 34. (a) 35. (a) 36. (a) 37. (a) 38. (c) 39. (b) 40. (d)
41. (c) 42. (c) 43. (b) 44. (d) 45. (c) 46. (a) 47. (b) 48. (c) 49. (c) 50. (c)
51. (b) 52. (d) 53. (c) 54. (c) 55. (d) 56. (c) 57. (d) 58. (a) 59. (c) 60. (a)
61. (d) 62. (c) 63. (b) 64. (c) 65. (a) 66. (b) 67. (c) 68. (b) 69. (b) 70. (b)
71. (b) 72. (a) 73. (d) 74. (b) 75. (d) 76. (d) 77. (b) 78. (d) 79. (a) 80. (d)
81. (c) 82. (c) 83. (c) 84. (c) 85. (c) 86. (a) 87. (c) 88. (d) 89. (b) 90. (d)
91. (b) 92. (b) 93. (b) 94. (a) 95. (c) 96. (c) 97. (c) 98. (b) 99. (c) 100. (b)
101. (d) 102. (a) 103. (a) 104. (b) 105. (d) 106. (b) 107. (a) 108. (b) 109. (d) 110. (c)
111. (a) 112. (c) 113. (b) 114. (b) 115. (b) 116. (a) 117. (b) 118. (d) 119. (c) 120. (d)
121. (a) 122. (c) 123. (c) 124. (b) 125. (c) 126. (b) 127. (c) 128. (c) 129. (d) 130. (a)
131. (c) 132. (a) 133. (b) 134. (c) 135. (c) 136. (d) 137. (a) 138. (c) 139. (a) 140. (c)
141. (c) 142. (b) 143. (b) 144. (b) 145. (d) 146. (a) 147. (c) 148. (c) 149. (a) 150. (c)
151. (b) 152. (d) 153. (a) 154. (a) 155. (d) 156. (c) 157. (d) 158. (d) 159. (b) 160. (b)
161. (a) 162. (b) 163. (c) 164. (a) 165. (b) 166. (c) 167. (d) 168. (c) 169. (d) 170. (d)
171. (a) 172. (b) 173. (b) 174. (a) 175. (c)

CLAT-6-Previous Years Papers.indd 349 06/09/2010 17:50:09


350 PREVIOUS YEARS’ PAPERS

CLAT 2009
ENGLISH of the world economy. In either case the extinc-
tion of a large fraction of the earth’s animals,
PART - A plants and microorganism seems possible. The
Instructions: (Questions 1-10). Read the given population size of homo sapiens conceivably
passage carefully and answer the questions that could be reduced to prehistoric levels or below
follow. and extinction of the human species itself can-
Marks: Each question carries 1 (one) mark. (To- not be excluded.
tal 10 marks) 1. Choose the word, which is most opposite
in meaning of the word, printed in bold as
There is a fairly universal sentiment that the use
used in the passage deleterious.
of nuclear weapons is clearly contrary to moral-
(a) Beneficial (b) Harmful
ity and that its production probably so, does not
(c) Irreparable (d) Non-cognizable
go far enough. These activities are not only op-
2. The author’s most important objective of
posed to morality but also to law if the legal ob-
wiring the above passage seems to
jection can be added to the moral, the argument
(a) highlight the use of nuclear weapons
against the use and the manufacture of these
as an effective population control
weapons will considerably be reinforced. Now
measures.
the time is ripe to evaluate the responsibility of
(b) illustrate the devastating effects of use
scientists who knowingly use their expertise of of nuclear weapons on mankind.
the construction of such weapons which has del- (c) duly highlight the supremacy of the
eterious effect on mankind. nations which possess nuclear weap-
To this must be added the fact that more ons.
than 50 percent of the skilled scientific man- (d) summarize the long biological effects
power in the world is now engaged in the arma- of use of nuclear weapons.
ments industry. How appraise it is that all this 3. The scientists engaged in manufacturing
valuable skill should be devoted to the manu- destructive weapons are
facture of weapons of death in a world of pov- (a) very few in number
erty is a question that must touch the scientific (b) irresponsible and incompetent
conscience. (c) more than half of the total number
A meeting of biologists on the long-term (d) engaged in the armaments industry
worldwide biological consequences of nuclear against their desire
war added frightening dimension to those fore- 4. According to the passage, the argument on
casts. Its report suggested that the long biologi- use and manufacture of nuclear weapons
cal effects resulting from climatic changes may (a) does not stand the test of legality
at least be as serious as the immediate ones. (b) possesses legal strength although it
Sub-freezing temperatures, low light levels, and does not have moral standing
high doses of ionizing and ultraviolet radiation (c) is acceptable only on moral grounds
extending for many months after a large scale (d) becomes stronger if legal and moral
nuclear war could destroy the biological support considerations are combined
system of civilization, at least in the northern 5. The author of the passage seems to be of
hemisphere. the view that
Productivity in natural and agricultural eco- (a) utilization of scientific skills in manu-
systems could be severely restricted for a year or facture of weapons is appropriate.
more. Post was survivors would face starvation (b) manufacture of weapons of death
as w4ell as freezing conditions in the dark and would help eradication of poverty.
be exposed to near lethal doses of radiation. If, (c) spend icing money on manufacture of
as now seems possible, the southern hemisphere weapons may be justifiable subject to
were affected also, global disruption of the bio- the availability of funds.
sphere could ensue. In any event, there would be (d) utilization of valuable knowledge for
severe consequences, evening the areas not af- manufacture of lethal weapons is in-
fected directly, because of the interdependence human.

CLAT-6-Previous Years Papers.indd 350 06/09/2010 17:50:09


PREVIOUS YEARS’ PAPERS 351

6. Which of the following is one of the con- words given in these questions are spelt wrong.
sequence of nuclear war? Select the word that is spelt correct.
(a) Fertility of land will least for a year or Marks: Each question carries 1 (one) mark. (To-
so. tal 5 marks)
(b) Post-war survivors being very few
11. (a) renaissance (b) renaisance
will have abundant food.
(c) renaissence (d) renaisence
(c) Lights would be cooler and more
12. (a) malaese (b) melaize
comfortable.
(c) melaise (d) malaise
(d) Southern hemisphere would remain
13. (a) irelevant (b) irrelevant
quite safe in the post war period
(c) irrelevent (d) irrellevant
7. Which of the following best explains the
14. (a) survilance (b) surveillance
word devoted, as used in the passage?
(c) surveilance (d) surveilliance
(a) dedicated for a good cause
15. (a) gaiety (b) gaietty
(b) utilized for betterment
(c) gaeity (d) gaitty
(c) abused for destruction
(d) under utilized PART – C
8. The biological consequences of nuclear Instructions: (Question 16-20), Given below
war as given into eh passage include all are a few foreign language phrases that are
the following, except commonly used. Choose the correct meaning for
(a) fall in temperature below zero degree each of the phrases and shade the appropriate
Celsius answer in the space provided for it on the OMR
(b) ultraviolet radiation Answer sheet.
(c) high doses of ionizing Mark: Each question carries 1 (One) mark. (To-
(d) abundant food for smaller population tal 5 marks)
9. It appears from the passage that the use of
nuclear weapons I considered against mo- 16. Mala fide
rality by (a) in good faith
(a) only such of those nations who cannot (b) in bad faith
afford to manufacture and sell weap- (c) without any faith
ons. (d) with full faith
(b) almost all the nations of the world. 17. Pro rata
(c) only the superpowers who can afford (a) at the rate of
to manufacture and sell weapons. (b) at quoted rate
(d) most of the scientists devote their (c) in proportion
valuable skills to manufacture nucle- (d) beyond all proportion
ar weapons. 18. Vice versa
10. Which of the following statements I, II, III (a) in verse
(b) versatile verse
and IV is definitely true in the context of
(c) in consonance with
the passage?
(d) the other way round
(I) There is every likelihood of survival
19. Ab initio
of the human species as a consequence
(a) from thievery beginning
of nuclear war.
(b) high initiative
(II) Nuclear war risks and harmful effects
(c) things done later
are highly exaggerated.
(d) without initiative
(III) The post war survivors should be ex-
20. Alibi
posed to the benefits of non-lethal ra-
(a) everywhere
diation.
(b) else where
(IV) Living organisms in the areas which
(c) nowhere
are not directly affected by nuclear
(d) without any excuse
was would also suffer.
Instructions: (Questions 21-25), some idioms
(a) I (b) II
given below are commonly used. Choose the
(c) II (d) III
correct meaning for each of the idioms.
PART – B Marks: Each question carries 1 (One) mark.
Instructions: (Question 11-15), Three of the four (Total 4 marks)

CLAT-6-Previous Years Papers.indd 351 06/09/2010 17:50:10


352 PREVIOUS YEARS’ PAPERS

21. To give the game away (a) forward (b) about


(a) To lose the game (c) in (d) out
(b) To give a walk over in a game 33. If he …………. a horse he would fly.
(c) To reveal the secret (a) was (b) were
(d) To play the game badly (c) is (d) goes
22. To cool one’s heels 34. Mohan has a bad habit of ………… at on
(a) To close the chapter odd hour.
(b) To walk on the heels (a) turning up (b) turning in
(c) To kick someone with the heels (c) turning over (d) turning off
(d) To wait and rest for some time 35. He must refrain …… immoral conducts.
23. To bury the hatchet (a) off (b) through
(a) To fight with the hatchet (c) from (d) against
(b) To forget the enmity PART – F
(c) To bury the treasure under ground
Instructions: (Question 36-40), the constitu-
(d) To pic up enmity
ent sentences of a passage have been jumbled
24. Gift of the gab
up. Read jumbled sentences carefully and then
(a) Gift for hard work
choose the option which shows the best sequence
(b) Gift undeserved
of sentences of the passage.
(c) Gift of being a good conversationalist
(d) Gift from unknown person Marks: Each question carries 1 (One) mark.
25. To smell a rat (Total 5 marks).
(a) To suspect a trick 36. (i) The collector said that the Dams
(b) To detect a foul smell should receive
(c) To behave like a rat (ii) To ensure uninterrupted
(d) To trust blindly (iii) Wate up to a particular level
PART – E (iv) Supply of water for irrigation
The best sequence is:
Instructions: (Questions 26-35), given below (a) ii, i, iv, iii (b) ii, i, iv, iii
are sentences with a blank in each sentence. (c) iv, i, iii, ii (d) ii, iv, i, iii
Choose the right answer to fill in the blank. 37. (i) He loved to distribute them among
26. Some people believe that in emotional ma- small kids.
turity men are inferior ………… women (ii) He wore a long, loose short with many
(a) than (b) to pockets.
(c) from (d) against (iii) And in doing so his eyes brightened.
27. My father was annoyed ………….. me. (iv) The pockets of his shirt bulged with
(a) towards (b) against toffees and chocolates
(c) with (d) upon The best sequence is:
28. Some orthodox person are averse ………. (a) ii, i, iii, iv (b) i, iv, ii, iii
drinking liquor (c) iv, i, iii, ii (d) ii, iv, i, iii
(a) against (b) for 38. (i) As we all know, a legislation
(c) towards (d) to (ii) Needs the assent of the president
29. The cinema hall was on fire and the cinema (iii) Passed by the houses of parliament
owner had to send ....... the fire brigade. (iv) To become law
(a) for (b) through The best sequence is:
(c) off (d) in (a) i, iii, ii, iv (b) i, iv, ii, iii
30. He was not listening ……. I was saying. (c) iv, i, iii, ii (d) ii, iv, i, iii
(a) that (b) which 39. (i) The farmers grow food for the whole
(c) to what (d) what country.
31. Drinking country liquor at marriage is a (ii) And therefore it is our duty to improve
custom ……….. certain tribes. their lot.
(a) in (b) among (iii) Yet, these fellows are exploited by the
(c) between (d) with rich.
32. The struggle for justice brings ……… the (iv) Hence, they are the most useful mem-
best of moral qualities of men. bers of the society.

CLAT-6-Previous Years Papers.indd 352 06/09/2010 17:50:10


PREVIOUS YEARS’ PAPERS 353

The best sequence is: 48. Who got the World Food Prize?
(a) ii, i, iv, iii (b) i, iv, ii, iii (a) Kofi Annan
(c) i, iv, iii, ii (d) ii, iv, i, iii (b) Manmohan Singh
40. (i) The ripples looked enchanting the (c) Bhumibol Adulyadej
light of the sun. (d) Hillary Clinton
(ii) War went to the pond. 49. ISO 9000 is a
(iii) We flung stones to create ripples. (a) Quality Standard
(iv) We stood knee-deep in the muddy wa- (b) Space Project mark
ter of the pond. (c) Trade Technique
(d) Loan Security
The best sequence is:
50. What is ‘AGMARK’?
(a) ii, i, iv, iii (b) ii, iv, iii, i
(a) Name of Brand
(c) iv, i, iii, ii (d) iv, ii, i, iii
(b) A Marketing Research Organization
GENERAL KNOWLEDGE/ (c) Eggs supplied by Government-run co-
CURRENT AFFAIRS operative
(d) Agriculture marketing for agro prod-
Instructions: (Questions 41-90), out of the four ucts
answers, select the correct answer. 51. The headquarters of Indian space research
Marks: Each question carries 1 (One) mark (to- organization is at
tal 50 marks) (a) Trivandrum (b) New Delhi
41. Capital market means (c) Bangalore (d) Ahmadabad
(a) mutual funds 52. “Saras” is the name of the
(b) money market (a) An aircraft (b) A tank
(c) securities market (c) A missile (d) A submarine
53. First woman Prime Minister in the World
(d) banking business
was from
42. From which river would the National Riv-
(a) Sri Lanka (b) Bhutan
er Project be started? (c) India (d) Nepal
(a) Yamuna (b) Gomati 54. Who was felicitated with ‘Nishan-e-Paki-
(c) Ganga (d) Krishna stan’?
43. “The Audacity of Hope” is a book written (a) Shatrughan Sinha
by (b) Maulana Azad
(a) Bill Clinton (b) Barrack Obama (c) Dilip Kumar
(c) Gorge Bush (d) Bill Gates (d) Khan Abdul Gaffar Khan
44. ‘WPI’ is used as an acronym for 55. Which state provided separate reservation
(a) World Price Index for Muslims and Christians in the state
(b) World Price Indicators backward clases list in 2007?
(c) Wholesale Price Index (a) Andhra Pradesh
(d) Wholesale Price Indicators (b) Tamil Nadu
45. If the tax rate increases with the higher (c) Bihar
level of income, it shall be called (d) Kerala
(a) Progressive Tax 56. Which of the following dances is not a
(b) Proportional Tax classical dance?
(c) Lump Sum Tax (a) Kathakali (b) Garba
(d) Regressive Tax (c) Odissi (d) Manipuri
57. Sulabh international is an organization
46. Why is the Director of “Chak De India”?
which provides
(a) Shimit Amiro
(a) Health Services in Rural Areas
(b) Yash Chopra (b) Good Sanitation at Cheap Rates
(c) Shahrukh Khan (c) Low Cost Accommodation
(d) Ram Gopal Verma (d) Low Cost Credit
47. What is the full from of the scanning tech- 58. Who among the following was honoured
nique CAT? with ‘Officer of the legion of Honour’
(a) Complete Anatomical Trepanning award by French government in July
(b) Computerized Automatic Therapy 2008?
(c) Computerized Axial Tomography (a) Dev Anand (b) Yash Chopra
(d) Complete Axial Transmission (c) B. R. Chopra (d) Mrinal Sen

CLAT-6-Previous Years Papers.indd 353 06/09/2010 17:50:10


354 PREVIOUS YEARS’ PAPERS

59. The largest gland in the human body is 70. Which one of the following vitamins is re-
(a) Liver (b) Pancreas sponsible for blood clotting?
(c) Thyroid (d) Endocrine (a) Vitamin A (b) Vitamin E
60. The Railway Budget for 2007-208 has de- (c) Vitamin C (d) Vitamin K
clared the year 2007 as the year of 71. What is the meaning of ‘gilt edged mar-
(a) Cleanliness ket’?
(b) Passenger Comfort (a) market in government securities
(c) Staff Welfare (b) market of smuggled goods
(d) Computerization of Reservations (c) market of auctioned goods
61. In the world of Hindi Cinema, who was (d) market of gold products
affectionately called as ‘Dada Moni’? 72. Who is the Central Chief Information
(a) Balraj Sahni (b) Prithviraj Kapoor Commissioner of India?
(c) Ashok Kumar (d) Utpal Dutt (a) Prof Ansari
62. Outstanding Parliamentarian Award (b) Mrs Padma Subrmanian
(2006) was presented in 2007 to (c) Mr Wajahat Habibullah
(a) P. Chidambaram (d) Dr O. P. Kejariwal
(b) Sarad Pawar 73. The youngest recipient of Padma Shri so
(c) Mani Shankar far is
(d) Sushma Swaraj (a) Sachin Tendulkar
63. World’s longest sea bridge has taken shape (b) Shobana Chandrakumar
in 2007 in (c) Sania Mirza
(a) China (b) Japan (d) Billy Arjan Singh
(c) Singapore (d) USA 74. Who is the director of the film “Elizabeth:
64. Who among the following Indians become The Golden Age Cast”?
a Citigroup’s Investment Banking Head? (a) Rama Nand Sagar
(a) L. N. Mittal (b) Ram Gopal Verma
(b) A. K. Subramanian (c) Karan Johar
(c) Vineet Seth (d) Shekhar Kapur
(d) Vikram Pandit 75. The territorial waters of India extend up
65. The green Revolution in India has been to
identified with (a) 12 nautical miles
(a) Dr Manmohan Singh
(b) 6 km
(b) Dr Montek Singh Ahluwalia
(c) 10 nautical miles
(c) Mr Rajendra, Singh ‘waterman’
(d) 15 nautical miles
(d) Dr M. S. Swaminathan
76. ‘Samjhouta Express’ runs between the
66. Indian Judge in the UN Law of the Sea
Railway Stations of
Tribunal is
(a) New Delhi – Wagah
(a) Dr P. S. Rao
(b) New Delhi – Lahore
(b) Dr P. C. Rao
(c) Amritsar – Lahore
(c) Mr Justice Jagannath Rao
(d) New Delhi – Islamabad
(d) Mr Justice Rajendra Babu
77. Blue revolution refers to
67. The Ozone Layer thins down as a result
(a) Forest Development
of a chain chemical reaction that separates
from the layer (b) Fishing
(a) Oxygen (b) Chlorine (c) Poultry Farming
(c) Nitrogen (d) Hydrogen (d) Horticulture
68. Joint SAARC University of eight SAARC 78. Dr A. P. J. Abdul Kalam has been appoint-
Nations has been established in ed as Chancellor of
(a) Colombo (b) Dhaka (a) IIM Mumbai
(c) Kathamandu (d) New Delhi (b) IIT Kanpur
69. Which country recently produced the (c) IIM Ahmedabad
world’s first cloned rabbit using a biologi- (d) IIST Thiruvananthapuram
cal process that takes cells from a fetus? 79. In which state “Kanya Vidyadhan Yojna”
(a) UK (b) China is operational?
(c) USA (d) Germany (a) Andhra Pradesh

CLAT-6-Previous Years Papers.indd 354 06/09/2010 17:50:10


PREVIOUS YEARS’ PAPERS 355

(b) Uttar Pradesh (a) Orissa


(c) Rajasthan (b) Gujarat
(d) Haryana (c) Andhra Pradesh
80. Who emerged the fastest woman of the (d) Karnataka
world at Beijing Olympics? 90. World “No Tobacco Day” is observed on
(a) Sheron Sumpson (a) January 10 (b) June 1
(b) Kerron Stewart (c) May 31 (d) March 5
(c) Ann Fraser
(d) Elina Basiena ELEMENTARY MATHEMATICS
81. Savannath grasslands are found in (NUMERICAL ABILITY)
(a) North America (b) Africa
Instructions: (Questions 91-110), From the four
(c) Australia (d) East Asia
answers given, select the appropriate.
82. Which state has launched the “Arogya
Sri” a Health Insurance Scheme for fami- Marks: Each question carries 1 (One) total 20
lies below poverty line? Marks)
(a) Andhra Pradesh 91. The average monthly income of a person
(b) Uttar Pradesh in a certain family of 5 members is ` 1000.
(c) Maharashtra What will be the monthly average income
(d) Kerala of one person in the same family if the in-
83. The first nuclear reactor of India is named
come of one person increased by ` 12,000
(a) Rohini (b) Vaishali
per year?
(c) Apsara (d) Kamini
(a) ` 1200 (b) ` 1600
84. In May 2007 Air Sahara acquired by Jet
(c) ` 2000 (d) ` 3400
Airways is being operated as a separate
airline under the name of 92. A dishonest shopkeeper uses a weight of
(a) Jet Lite (b) Jet Sahara 800 gm for a kg and professes to sell his
(c) Air Jet Line (d) Jet Sahara Lite good at cost price. His profit is
85. Suez Canal connects (a) 20 per cent (b) 21 per cent
(a) Mediterranean Sea and Red Sea (c) 24 per cent (d) 25 per cent
(b) Mediterranean Sea and Black Sea 93. By selling 11 oranges for a rupee, a man
(c) Baltic sea and Red Sea loses 10 per cent. How many oranges for a
(d) Baltic Sea and Black Sea rupee should he sell to gain 10 per cent?
86. Government has launched E–Passport (a) 9 (b) 10
Scheme and First E–Passport was issued (c) 8 (d) 5
to 94. A person takes 3 hours to walk a certain
(a) Mr Arjun Singh distance and riding back. He could walk
(b) Mrs Sonia Gandhi both ways in 5 hours. How long could it
(c) Dr Manmohan Singh take to ride both ways.
(d) Mrs Partibha Patil (a) 1.5 h (b) 1 h
87. The Hindu outfit ‘Hindraf’ has been hand- (c) 0.5 h (d) 2 h
ed in 95. Change 1/8 into percentage
(a) Pakistan (b) Thailand (a) 12.5 per cent (b) 15 per cent
(c) Malaysia (d) Bangladesh (c) 8 per cent (d) 25 per cent
88. Which organization is headed by Indian 96. 12.5% of 80 is equal to
Environmentalist R. K. Pachauri, a Nobel
(a) 8 (b) 20
Laureate?
(c) 10 (d) 40
(a) International Environment Panel
97. Which number should fill the black space
(b) International Panel on Climate
to complete the series: 1, 2, 4, 5, 7, 8, 10,
Change
(c) International Pollution Control Panel 11, ……..
(d) International Panel on Global Warm- (a) 12 (b) 13
ing (c) 14 (d) 15
89. Kandhamal, the worst affected town by 98. The smallest of the fractions given below
sectarian violence in September-October (a) 9/10 (b) 11/12
2008 is situated in (c) 23/28 (d) 32/33

CLAT-6-Previous Years Papers.indd 355 06/09/2010 17:50:10


356 PREVIOUS YEARS’ PAPERS

99. Three friends shared the cost of a televi- 108. Ram weights 25 kg more than Shyam.
sion. If Amit, Bharat and Dinesh each Their combined weight is 325 kg. How
paid ` 3000 and ` 1800 respectively, then much does Shyam weight?
Dinesh paid what percent of the total (a) 150 kg (b) 200 kg
cost? (c) 125 kg (d) 160 kg
(a) 10 per cent (b) 20 per cent 109. Find out the wrong number in the series:
(c) 30 per cent (d) 40 per cent 3, 8, 15, 24, 48, 63
100. The average age of 29 boys of a class is (a) 24 (b) 34
equal to 14 years. When the age of the (c) 15 (d) 63
class teacher is included the average be- 110. What is the location value of 7 in the fig-
comes 15 years. Find the age of the class ure 9872590?
teacher. (a) 72590 (b) 7
(a) 44 years (b) 40 years (c) 70000 (d) 7000
(c) 52 years (d) 66 years
101. It takes 8 people working at equal rates to LEGAL APTITUDE
finish a work in 96 days. How long will 6
workers take for the same work? Instructions: (Questions 111-125), from the
(a) 28 days (b) 128 days four options given, select the appropriate cor-
(c) 111 days (d) 84 days rect option
102. Ram’s income is 20 per cent less than Shy- Marks: Each question carries1 (One) mark. (To-
am’s. How much is Shyam’s income more tal 45 marks)
than Ram’s in percentage terms?
111. Which is the oldest code of Law in India?
(a) 20 per cent (b) 25 per cent
(a) Naradasmriti (b) Manusmriti
(c) 30 per cent (d) 15 per cent
(c) Vedasmriti (d) Prasarsmriti
103. The monthly salary of A, B and C are in
112. Private international law is also called
the ratio 2 : 3 : 5. If C’s monthly salary is
(a) Civil Law
` 1,200 more than that of A, find B’s an-
(b) Local Laws
nual salary.
(c) Conflict of Laws
(a) ` 2000 (b) ` 1000
(d) Common Law
(c) ` 1500 (d) ` 1200
113. A nominal sum given as a, token for strik-
104. In a town there are 94500 people. 2/9 of
them are foreigners, 6400 are immigrants ing a sale is called
and the rest are natives. How many are na- (a) Earnest money (b) Advance
tives? (c) Interest (d) Solution
(a) 67100 (b) 27400 114. Joint heirs to a property are called
(c) 77600 (d) 88100 (a) co-heirs (b) coparceners
105. Total salary of three persons A, B and C is (c) successors (d) joint owners
` 1,44,000. They spend 80 per cent, 85 per 115. The right of party of initiate an action and
cent and 75 per cent respectively. If their be heard before a court of law is called
savings are in the ratio 8 : 9 : 20, find C’s (a) right in rem
salary. (b) Right in personam
(a) ` 48,000 (b) ` 64,000 (c) Fundamental right
(c) ` 40,000 (d) ` 32,000 (d) locus standi
106. The population of a town is 155625. For 116. Indian parliament is based on the principle
ever 1000 males there are 1075 females. If of
40 per cent of the males and 24 per cent of (a) Bicameralism
the females are literate, find the percent- (b) Universal Adult Franchise
age of literacy in the town. (c) Dyarchy
(a) 33.7 (b) 32.7 (d) Federalism
(c) 31.7 (d) 30.7 117. The Supreme Court held that evidence can
107. 10 sheep and 5 pigs were brought for be recorded by video conferencing in the
` 6,000. It the average price of a sheep is case …….
` 450, find the average price of pig. (a) State of Maharashtra vs Prafull B. De-
(a) ` 380 (b) ` 410 sai
(c) ` 340 (d) ` 300 (b) Paramjit Kaur vs State of Punjab

CLAT-6-Previous Years Papers.indd 356 06/09/2010 17:50:10


PREVIOUS YEARS’ PAPERS 357

(c) Pappu Yadav vs State of Bihar (a) Accused (b) Prosecution


(d) Bachan Singh vs State of Punjab (c) Police (d) Complainant
118. When the master is held liable for the 127. Offence which can be compromised be-
wrongful act of his servant, the liability tween the parties is known as
is called wrongful at of his servant, the li- (a) non-compoundable offence
ability is called (b) cognizable offence
(a) Strict Liability (c) compoundable offence
(b) Vicarious Liability (d) non-cognizable offence
(c) Tortuous Liability 128. Husband and wife have a right to each
(d) Absolute Liability others company. The right is called
119. The act of unlawfully entering into anoth- (a) Conjugal Right
er’s property constitutes (b) Human Right
(a) Trespass (b) Restraint (c) Civil Right
(c) Appropriation (d) Encroachment (d) Fundamental Right
120. Which Parliamentary Committee in In- 129. A person ‘dying intestate’ means he
dian System of Democracy is Chaired by (a) Died without legal heirs
a Member of Opposition Party? (b) Died without making a will
(a) Estimates Committee (c) Died without any property
(b) Joint Parliamentary Committee (d) Died without a son
(c) Public Accounts Committee 130. If a witness makes a statement in court,
(d) Finance Committee knowing it to be false, he commits the of-
121. Supreme Court held that Preamble as a fence of
basic feature of Constitution cannot be (a) Forgery (b) Falsehood
amended in the case of (c) Perjury (d) Breach of trust
(a) Golaknath vs State of Punjab 131. A child born after father’s death is
(b) Maneka Gandhi vs Union of India (a) Posthumous (b) Heir
(c) S. R. Bommai vs Union of India (c) Intestate (d) Bastard
(d) Kesavanada Bahrati vs State of Kera- 132. A formal instrument by which one per-
la son empowers another to represent him is
122. In the year 2002 the competition act was known as
enacted replacing (a) Affidavit
(a) Trademarks Act (b) Power of Attorney
(b) Copyright Act (c) Will
(c) Contract Act (d) Declaration
(d) MRTP Act 133. The temporary release of a prisoner is
123. A right to recover time barred debt is called
(a) Universal Right (a) Parole (b) Amnesty
(b) Perfect Right (c) Discharge (d) Pardon
(c) Imperfect Right 134. The offence of inciting disaffection, ha-
(d) Fundamental Right tred or contempt against government is
124. The law relating to prisoners of war has (a) Perjury (b) Forgery
been codified by (c) Sedition (d) Revolt
(a) Geneva Convention 135. India became the member of United Na-
(b) Vienna Convention tions in the year
(c) Paris Convention (a) 1956 (b) 1945
(d) None of the above (c) 1946 (d) 1950
125. Public holiday are declared under 136. A party to the suit is called
(a) Criminal Procedure Code (a) Accused (b) Plaintiff
(b) Civil Procedure Code (c) Litigant (d) Complainant
(c) Constitution of India 137. Who heads the four member committee
(d) Negotiable Instruments Act appointed to study the Centre-State rela-
126. When a person is prosecuted for com- tions especially the changes took place
mitting a criminal offence, the burden of since Sarkaria commission
proof is on (a) Justice M. M. Punchi

CLAT-6-Previous Years Papers.indd 357 06/09/2010 17:50:10


358 PREVIOUS YEARS’ PAPERS

(b) Justice Nanavati 148. The concept of judicial review has been
(c) Justice Barucha borrowed from the Constitution of
(d) Justice Kuldip Singh (a) U.S.S.R (b) UK
138. No one can be convicted twice the same (c) USA (d) Switzerland
offence. This doctrine is called 149. Every duty enforceable by law is called
(a) Burden of Proof (a) Accountability
(b) Double conviction (b) Obligation
(c) Double Jeopardy (c) Burden
(d) Corpus Delicti (d) Incidence
139. A participant in commission of crime is 150. The killing of a new born child by its par-
popularly known as ents is
(a) Respondent (a) Malfeasance (b) Infanticide
(b) Under-trial (c) Abortion (d) Foeticide
(c) Defendant 151. Offence of breaking a divine idol is
(d) Accomplice (a) Salus populi (b) Crime
140. Which of the following is not payable to (c) Sacrilege (d) Blasphemy
central government? 152. A person who goes under-ground or evades
(a) Land revenue (b) Customs duty the jurisdiction of the court is known as
(c) Income tax (d) Wealth tax (a) Offender (b) Under-ground
141. Where sit he national judicial academy lo- (c) Absentee (d) Absconder
cated? 153. What is a caveat?
(a) Kolkata (b) Bhopal (a) A warning (b) An injunction
(c) Delhi (d) Mumbai (c) Writ (d) Certiorari
142. Who have constitutional right to audience 154. Muslim religious foundation are known
in all Indian courts? as
(a) President (a) Din (b) Wakfs
(b) Chief Justice of India (c) Ulema (d) Quzat
(c) Attorney General 155. Beyond what distance from the coast, is
(d) Solicitor General the sea known as “High Sea”?
143. Which of the following is not included in (a) 20 miles (b) 300 miles
the preamble to the constitution? (c) 200 km (d) 12 miles
(a) Morality (b) Justice
(c) Sovereign (d) Socialist LOCICAL REASONING
144. ‘Court of Record’ is a court which? Marks: (Questions 15-200) Each question car-
(a) Maintains records ries 1 (One). Mark (Total 45 marks)
(b) Preserves all its records Instructions: (Question 156-165), Each ques-
(c) Can punish for its contempt tion below consists of one Assertion (A) and one
(d) Is competent to issue writs Reason (R). Examine them and select the correct
145. A Judge of the Supreme Court can be re- answers.
moved from office only on grounds of Code:
(a) Gross inefficiency (a) Both A and R are individually true and
(b) Delivering wrong judgments R sit he correct explanation of A.
(c) Senility (b) Both A and R are individually true but
(d) Proven misbehavior or incapacity R is not the correct explanation of A.
146. Fiduciary relationship means a relation- (c) A is true but R is false.
ship based no (d) A is false R is true.
(a) Trust (b) Money 156. A: Area along the Equator records the
(c) Contract (d) Blood relation highest temperature throughout the
147. The Chairman of Tehelka Enquiry Com- year.
mission is R: On the equator, days and nights are
(a) Justice Kirpal equal for the largest part of the years.
(b) Justice S. N. Phukan 157. A: Commercial fisheries have not devel-
(c) Justice Saharia oped in the tropics.
(d) Justice Liberhan R: The demand for marine food from low

CLAT-6-Previous Years Papers.indd 358 06/09/2010 17:50:10


PREVIOUS YEARS’ PAPERS 359

income population is low in the trop- 167. Statement: Detergents should be used to
ics. clean cloths.
158. A: Lightning thunder and heavy rain ac- Assumptions:
company volcanic activity. I. Detergent form more lather.
R: Volcanoes throw water vapour and II. Detergent helps to dislodge grease
charged particles in the atmosphere. and dirt.
159. A: Soils in some parts of Punjab, Harya- 168. Statement: “As there is a great demand,
na and Rajasthan are saline. every person seeking ticket of the pro-
R: Evaporation exceeds precipitation. gramme will be given only five tickets”.
160. A: The monsoons arrive suddenly in In- Assumptions:
dia in June. I. The organizers are not keen on selling
R: The monsoonal low pressure trough is the tickets.
well developed over India. II. No one is interested in getting more
161. A: India built dams and reservoirs to than five tickets.
manage water resources. 169. Statement: Double your money in five
R: India had enough experience in ca- moths an advertisement.
nals. Assumptions:
162. A: The life expectancy in European I. The assurance is not genuine
countries is very high. II. People want their money to grow.
R: European countries have low mortal- 170. Statement: Films have become indispens-
ity rate. able for the for the entertainment of peo-
163. A: The nomadic herders slaughter their ple
animals for meat. Assumptions:
R: Animals form the chief source of food I. Films are the only media of entertain-
and livelihood for nomadic herders. ment.
164. A: Exploitation of equatorial rain forest II. People enjoy films.
of Amazon basin is not easy. 171. Statement “To keep myself up-to-date, I
always listen to 9.00 p.m, news on radio”.
R: This region is very rich in several type
– A candidate tells the interview board.
of deadly animals and insects.
Assumptions:
165. A: The sea remains free from ice from
I. The candidate does not read newspa-
British Columbia to Bering sea.
per.
R: Air moving off the comparatively
II. Recent news are broadcast only on ra-
warm waters of north pacific drift
dio.
gives
172. Statement: Never before such a lucid book
The coastal area of British Columbia a
was available on the topic.
warmer climate.
Assumptions:
Instructions: (questions 166-175), is each ques-
I. Some other books were available on
tion below are given one statement and two as- this topic.
sumptions I and II. Examine the statements and II. You can write lucid books on very few
shade the correct assumption which is implicit topics.
in the statement on the OMR answer sheet using 173. Statement: In case of any difficulty about
the following code. this case, you may contact our company’s
Code: lawyer.
(a) if only assumption I is implicit. Assumptions:
(b) if only assumption II is implicit. I. Each company has a lawyer of its
(c) if either I or II is implicit. own.
(d) if neither I nor II is implicit. II. The company’s lawyer is thoroughly
166. Statement: The patient’s condition would briefed about this case.
improve after operation. 174. Statement: “Present day education is in
Assumptions: shambles and the country is going to the
I. The patient’s can be operated upon in dogs”.
this condition. Assumptions:
II. The patient cannot be operated upon I. A good education system is essential
in this condition for the well being of a Nation.

CLAT-6-Previous Years Papers.indd 359 06/09/2010 17:50:10


360 PREVIOUS YEARS’ PAPERS

II. A good education alone is sufficient (c) Implementation


for the well being of a nation. (d) Leader
175. Statement: Children are influenced more 182. UP : Uttaranchal : : Bihar : ?
by their teachers nowadays. (a) Jharkhand
Assumptions: (b) Chhatisgarh
I. The children consider teachers as their (c) Madhya Pradesh
models. (d) Manipur
II. A large amount of children’s time is 183. Gold : Silver : : Cotton : ?
spent in school. (a) Yarn (b) Silk
Instructions: (Questions 176-180), each ques- (c) Fibre (d) Synthetic
tion below contains a statement on relationship 184. Botany : Flora : : Zoology : ?
and a question regarding relationship based on (a) Fauna (b) Biology
(c) Fossils (d) Pathology
the statement. Shade the correct option on rela-
185. Cold wave : Winter : : Loo : ?
tionship on the OMR Answer sheet.
(a) Humidity (b) Frostbite
176. Pointing to a photograph a lady tells Ram,
(c) Summer (d) Storm
“I am the only daughter of this lady and 186. King : Royal : : Saint : ?
her on is your maternal uncle”. How is the (a) Religious (b) Red
speaker related to Ram’s father? (c) Priesthood (d) Blue
(a) Sister-in-law 187. Sculptor : Statue : : Poet : ?
(b) Wife (a) Painter (b) Singer
(c) Either (a) or (b) (c) Poem (d) Writer
(d) Neither (a) nor (b) 188. Laugh : Happy : : Cry : ?
177. Introducing a man, a woman said, “he (a) Sad (b) Bickering
is the only son of my mother’s mother”. (c) Frown (d) Complain
How is the woman related to the man? 189. Black : Absence : : White : ?
(a) Mother (b) Aunt (a) Red (b) Presence
(c) Sister (d) Niece (c) Rainbow (d) Crystal
178. Shyam said, “this is the wife of the grand- 190. Governor : President : : Chief Minster : ?
son of my mother”. Who is Shyam to the (a) Commission (b) Attorney-General
to the girl? (c) Justice (d) Prime Minister
(a) Father (b) Grandfather Instructions: (Questions 191-195), Each ques-
(c) Husband (d) Father-in-law tion below contains a statement and two courses
179. Pointing to a man on the stage, Sunita of action I and II. Assuming the statement to be
said, “he is the brother of the daughter of true, decide which of the two suggested course
the wife of my husband” how is the man of action logically follows and shade the answer
on the stags related to Sunita? on the OMR answer sheet, using the code given
(a) Son (b) Husband below.
(c) Cousin (d) Nephew Code:
(a) if only I follows.
180. Introducing a man to her husband, a
(b) if only II follows.
woman said, “his brother’s father is the
(c) if either I or II follow.
only son of my grandfather”. How is the
(d) if neither I nor II follow.
woman related to his man?
191. Statement: One of the problems facing the
(a) Mother (b) Aunt food processing industry is the irregular
(c) Sister (d) Daughter supply of raw material. The producers of
Instructions: (Questions 181-190), in each raw materials are not getting a reasonable
question below two words are paired which price.
have certain relationships. After the double co- Course of action:
lon (::), another word is given. Shade the cor- I. The government should regulate the
rect option on the OMR answer sheet which supply of raw material to other indus-
pairs with this word taking into account the pair tries also.
already given. II. The government should announce an
181. Legislation : Enactment : : Executive : ? attractive package to ensure regular
(a) Minister supply of raw material for food pro-
(b) Officer cessing industry.

CLAT-6-Previous Years Papers.indd 360 06/09/2010 17:50:10


PREVIOUS YEARS’ PAPERS 361

192. Statement: The officer in-charge of a com- Code:


pany had a hunch that some money was (a) if only argument I is strong.
missing from the safe. (b) if only argument II is strong.
Course of action: (c) if either argument I or II is strong.
I. He should get it recounted with the (d) in neither argument I nor II strong.
help of the staff and check it with the 196. Statement: Should a total ban be put on
balance sheet. trapping wild animals?
II. He should inform the police. Arguments:
193. Statement: If the retired professors of the I. Yes. Trappers are making a lot of
same institutes are also invited to deliber- money.
ate on restructuring of the organization, II. No bans on hunting and trapping are
their contribution may be beneficial to the not effective.
institute. 197. Statement: Should school education be
Course of action: made free in India?
I. Management may seek opinion of the Arguments:
employees before calling retired pro- I. Yes. This is the only way to improve
fessors. the level of literacy.
II. Management should involve experi- II. No. It would add to the already heavy
enced people for the systematic re- burden on the exchequer.
structuring of the organization. 198. Statement: Should government jobs in ru-
194. Statement: The sale of a particular product ral areas have more incentives?
has gone down considerably causing great Arguments:
concern to the company. I. Yes. Incentives are essential for at-
Course of action: tracting government servants there.
I. The company should make a proper II. No. Rural areas are already cheaper,
study of rival products in the market. healthier and less complex than big.
II. The price of the product should be re- So, why offer extra incentives!
duced and quality improved. 199. Statement: should luxury hotels be banned
195. Statement: Mr X, an active member of the in india?
union, often insults his superiors in the of- Arguments:
fice with his rude behavior. I. Yes. They are placed from where in-
Course of action: ternational criminals operate.
I. He should be transferred to some oth- II. No. affluent foreign tourists will have
er department. no place to stay.
II. The matter should be referred to the 200. Statement: should the political parties be
union. banned?
Instructions: (Questions 196-200), each ques- Arguments:
tion below contains a statement and two argu- I. Yes. It is necessary to teach a lesson to
ments I and II. Assume the statement to be true, the politicians.
shade the Argument which is strong on the OMR II. No. it will lead to an end of democ-
answer sheet using the code below. racy.

ANSWERS
1. (b) 2. (b) 3. (c) 4. (d) 5. (d) 6. (a) 7. (a) 8. (d) 9. (b) 10. (d)
11. (a) 12. (d) 13. (b) 14. (b) 15. (a) 16. (b) 17. (c) 18. (d) 19. (a) 20. (b)
21. (c) 22. (d) 23. (b) 24. (c) 25. (a) 26. (b) 27. (c) 28. (d) 29. (a) 30. (c)
31. (a) 32. (d) 33. (b) 34. (a) 35. (c) 36. (d) 37. (b) 38. (a) 39. (c) 40. (c)
41. (c) 42. (c) 43. (b) 44. (c) 45. (a) 46. (a) 47. (c) 48. (d) 49. (a) 50. (d)
51. (c) 52. (a) 53. (a) 54. (c) 55. (b) 56. (b) 57. (b) 58. (b) 59. (c) 60. (a)
61. (c) 62. (c) 63. (a) 64. (d) 65. (d) 66. (b) 67. (b) 68. (d) 69. (b) 70. (d)
71. (a) 72. (c) 73. (c) 74. (d) 75. (a) 76. (b) 77. (b) 78. (d) 79. (b) 80. (c)

CLAT-6-Previous Years Papers.indd 361 06/09/2010 17:50:10


362 PREVIOUS YEARS’ PAPERS

81. (b) 82. (a) 83. (c) 84. (a) 85. (a) 86. (d) 87. (c) 88. (d) 89. (a) 90. (c)
91. (a) 92. (d) 93. (a) 94. (b) 95. (a) 96. (c) 97. (b) 98. (c) 99. (d) 100. (a)
101. (b) 102. (b) 103. (d) 104. (d) 105. (b) 106. (c) 107. (d) 108. (a) 109. (b) 110. (c)
111. (b) 112. (c) 113. (a) 114. (b) 115. (d) 116. (a) 117. (a) 118. (b) 119. (a) 120. (c)
121. (d) 122. (d) 123. (c) 124. (a) 125. (d) 126. (b) 127. (c) 128. (a) 129. (b) 130. (c)
131. (a) 132. (b) 133. (a) 134. (c) 135. (b) 136. (c) 137. (a) 138. (c) 139. (d) 140. (a)
141. (b) 142. (c) 143. (a) 144. (c) 145. (b) 146. (a) 147. (b) 148. (c) 149. (b) 150. (b)
151. (c) 152. (d) 153. (a) 154. (b) 155. (c) 156. (a) 157. (b) 158. (a) 159. (b) 160. (d)
161. (b) 162. (b) 163. (b) 164. (a) 165. (a) 166. (a) 167. (b) 168. (d) 169. (b) 170. (b)
171. (d) 172. (a) 173. (c) 174. (a) 175. (a) 176. (b) 177. (d) 178. (d) 179. (a) 180. (c)
181. (c) 182. (a) 183. (d) 184. (a) 185. (c) 186. (a) 187. (c) 188. (a) 189. (b) 190. (d)
191. (b) 192. (c) 193. (b) 194. (c) 195. (b) 196. (d) 197. (c) 198. (a) 199. (d) 200. (c)

CLAT-6-Previous Years Papers.indd 362 06/09/2010 17:50:10


PREVIOUS YEARS’ PAPERS 363

PANJAB UNIVERSITY CET 2009


1. Who is the Executive Head of a State in 12. Who was the last King of Nepal?
India? (a) Gyanendra (b) Mohindra
(a) The Governor (c) Robindra (d) Paras
(b) The Chief Minister 13. Richter Scale is used to register the inten-
(c) The Prime Minister sity of ..........
(d) The Speaker (a) Volcanoes (b) Tidal Waves
2. Who wrote Arthashastra? (c) Wind Energy (d) Earthquakes
(a) Arnartya Sen 14. Amnesia means loss of ..........
(b) Kautilya (a) Hearing (b) Eyesight
(c) Manmohan Singh (c) Memory (d) Blood
(d) Montek Singh 15. Who founded Din-i-Ilahi?
3. Malaria is usually spread by .......... (a) Akbar
(a) female culex (b) Aurangzeb
(b) flies (e) Nizamuddin Chishti
(c) male Anopheles (d) Amir Khusro
(d) female Anopheles 16. Who replaced Shivraj Patil as the Home
4. The letter E in the acronym IAEA stands Minister of India in November 2008?
for: (a) P. Chidambaram
(a) Environment (b) Energy (b) Pranab Mukherjee
(c) Ecology (d) E-mail (c) R. R. Patil
5. The letter T in the acronym WTO stands (d) C. K. Antony
for .......... 17. Barack Obama is the .......... President of
(a) Trade (b) Tariff the USA.
(c) Textile (d) Taxes (a) 42nd (b) 43rd
6. With which sport is Rafael Nadal associ- (c) 44th (d) 45th
ated? 18. The Capital of Jharkhand is ..........
(a) Badminton (a) Ranchi (b) Patna
(b) Lawn Tennis (c) Dhanbad (d) Raipur
(c) Golf 19. Which is the largest Ocean?
(d) Formula 1 Racing (a) Atlantic (b) Indian
7. Who was defeated in the Battle of Water- (c) Pacific (d) Arctic
loo? 20. Which is the largest Desert in the World?
(a) Napoleon (a) Sahara (b) Gobi
(b) Hitler (c) Kalahari (d) Thar
(c) Stalin 21. Which State has two separate Capitals for
(d) Mussolini Winter and Summer?
8. World War II lasted from .......... (a) Arunachal Pradesh
(a) 1914-20 (b) 1930-36 (b) Sikkim
(c) 1939-45 (d) 1939-47 (c) J & K
9. The currency of China is .......... (d) Uttarakhand
(a) Yuan (b) Yen 22. Where are the Headquarters of World
(c) Chinese Dollar (d) Yang Health Organization located?
10. What does 3G stand for in 3G Mobile Ser- (a) Geneva (b) Paris
vices? (c) New York (d) Rome
(a) Three Gigabytes 23. Who is associated with the Theory of Rel-
(b) Third Generation ativity?
(c) Three Giga-pixel (a) Newton (b) Galileo
(d) Three Goals (c) Kepler (d) Einstein
11. Identify the Capital of Australia: 24. The planet nearest to the Sun is ..........
(a) Melbourne (b) Canberra (a) Venus (b) Mars
(c) Sydney (d) Perth (c) Earth (d) Mercury

CLAT-6-Previous Years Papers.indd 363 06/09/2010 17:50:10


364 PREVIOUS YEARS’ PAPERS

25. The letter S in the acronym AIDS stands (c) Bankim Chandra
for .......... (d) Mahatama Gandhi
(a) Society (b) Sickness 39. The Third Battle of Panipat was fought be-
(c) Syndrome (d) Status tween ............
26. In which country is Swat Valley? (a) the Afghans and the Marathas
(a) Afghanistan (b) Talibanistan (b) the French and the English
(c) Baluchistan (d) Pakistan (c) Babur and Ibrahim Lodhi
27. The new name of Spice Telecom is .......... (d) Akbar and Bairam Khan
(a) Idea Cellular (b) Hutch 40. With which religion do we associate the
(c) Aircel (d) Vodafone Jataka Tales?
28. LTTE is an organization based in .......... (a) Hinduism (b) Buddhism
(a) Pakistan (b) Sri Lanka (c) Jainism (d) Zoroasterianism
(c) Afghanistan (d) India 41. In Ramayana, Lakshman is the son of
29. On whose novel is the story of the movie ..............
Slumdog Millionaire based? (a) Kaushalya (b) Sumitra
(a) Raja Rao (b) Danny Boyle (c) Kaikeyi (d) Gandhari
(c) Aravind Adiga (d) Vikas Swarup 42. The Southernmost point of India, Indira
30. Tamasha is associated with the State of Point is in ...............
.......... (a) Tamil Nadu
(a) Assam (b) J & K (b) Kerala
(c) Maharashtra (d) Orissa (c) Lakshwadeep
31. Who is famous for his Bansuri vadan (d) Andaman and Nicobar Islands
(Flute playing)? 43. Which two countries does the Palk Strait
(a) Hari Prasad Chaurasia separate?
(b) Amjad Khan (a) India and Indonesia
(c) Ravi Shankar (b) Sri Lanka and Maldives
(d) Shiv Prasad (c) India and Sri Lanka
32. Which one of these flows into the Arabian (d) Pakistan and Iran
Sea? 44. The Kutch region is located in ..............
(a) Godavari (b) Krishna (a) J & K (b) West Bengal
(c) Narmada (d) Kaveri (c) Gujarat (d) Karnataka
33. Sunderbans is the largest ............. forest in 45. Radcliffe Line demarcates the boundary
the world. between India and ..........
(a) teak (b) deodar (a) China (b) Pakistan
(c) mangrove (d) oak (c) Burma (d) Afghanistan
34. The largest soil group found in India is 46. According to the Preamble to the Indian
.......... Constitution, who has given the Constitu-
(a) Black (b) Laterite tion to the People of India?
(c) Alluvial (d) Red (a) The President
35. In India, economic liberalization started (b) The Constituent Assembly
in: (c) The Parliament
(a) 1971 (b) 1981 (d) The People of India
(c) 1991 (d) 2001 47. In which city is Charminar located?
36. In which language has Ramcharitmanas (a) Ahmedabad (b) Lucknow
been written by Tulsidas? (c) Hyderabad (d) Gandhinagar
(a) Sanskrit (b) Pali 48. Which country is in the news for the prob-
(c) Hindi (d) Prakriti lem of Sea-pirates recently?
37. Who is the author of Geet Govinda? (a) Cuba (b) Afghanistan
(a) Kalidas (b) Bharat Muni (c) Somalia (d) Ethiopia
(c) Surdas (d) Jayadeva 49. Who has written War and Peace?
38. Who has written the National Song of In- (a) Tolstoy (b) Dostoevsky
dia? (c) Chekhov (d) Pushkin
(a) Tagore 50. The letter A in the acronym NATO stands
(b) Nazrul Islam for .............

CLAT-6-Previous Years Papers.indd 364 06/09/2010 17:50:10


PREVIOUS YEARS’ PAPERS 365

(a) Association (b) Agreement (a) Prosperous (b) Fluent


(c) Allied (d) Atlantic (c) Talkative (d) Poor
51. Kalhana from Kashmir is primarily fa- 63. What is the word for “A disease which
mous as a/an .......... spreads by contact”?
(a) novelist (b) philosopher (a) Infectious (b) Contagious
(c) historian (d) emperor (c) Contiguous (d) Contractual
52. According to the Preamble to the Indian 64. Fill in the blank in “What is the time ..........
Constitution, India is not a .......... Repub- your watch?”
lic. (a) in (b) on
(a) Sovereign (b) Democratic (c) by (d) at
(c) Socialist (d) Nationalist 65. Identify the word with the correct spell-
53. What was known as Burma is now known ings:
as ............. (a) Committee (b) Comitee
(a) Kampuchea (b) Yangoon (c) Commitee (d) Comittee
(c) Myanmar (d) New Burma 66. Identify the word that is not spelt correct-
54. UNO was established in the year .......... ly:
(a) 1914 (b) 1939 (a) Accommodation
(c) 1945 (d) 1943 (b) Success
55. Which Article of the Indian Constitution (c) Receive
guarantees “Equality before Law” to ev- (d) Concious
ery person? 67. The word ‘stoned’ in the sentence “He was
(a) Article 14 (b) Artic1e21 stoned to death” is:
(c) Article 19 (d) Article 356 (a) a Verb (b) an Adjective
(c) a Noun (d) an Adverb
56. Which Ocean separates the Americas from
68. “To be above board” means to:
Europe?
(a) survive
(a) Atlantic (b) Pacific
(b) be honest
(c) Mediterranean (d) Antarctic
(c) sail in a ship
57. Dandi (as in Gandhiji’s Dandi March) is
(d) live on the first floor
the name of a ..........
69. “An axe to grind” means to:
(a) village (b) poet
(a) be a warrior
(c) music form (d) dance form
(b) have a selfish motive
58. Who is known as the Iron Man of India? (c) sharpen weapons
(a) Sardar Vallabhbhai Patel (d) to cut trees
(b) Subhash Chander Bose 70. “Hydrophobia” means fear of
(c) Lala Lajpat Rai (a) Heights (b) Dogs
(d) Indira Gandhi (c) Water (d) Electricity
59. Who has taken the maximum number of 71. Under which Article of the Constitution
wickets in Test Cricket? special status is granted to the State of
(a) Muttiah Muralidharan Jammu and Kashmir
(b) Shane Warne (a) Article 70 (b) Article 170
(c) Anil Kumble (c) Article 270 (d) Article 370
(d) WasimAkram 72. Which day is celebrated as a “Consumer
60. Who has written the famous novels Godan day”?
and Gahan? (a) 15th March (b) 30th June
(a) Jayashankar Prasad (c) 15th October (d) 10th December
(b) Munshi Premchand 73. Which day is celebrated as “Human Rights
(c) Girish Karnad Day”?
(d) Sadat Hassan Manto (a) 26th January (b) 26th October
61. Identify the word nearest in meaning to (c) 10th March (d) 10th December
the word LETHAL: 74. Advisory jurisdiction is rested with:
(a) Lawful (b) Unlawful (a) Subordinate Courts
(c) Deadly (d) Lively (b) High Courts
62. Identify the word nearest in meaning to (c) Supreme Courts
the word AFFLUENT: (d) All the Courts

CLAT-6-Previous Years Papers.indd 365 06/09/2010 17:50:11


366 PREVIOUS YEARS’ PAPERS

75. “Life imprisonment” in India means im- 85. “Judicial review” means
prisonment for (a) Review of the order
(a) 10 years (b) 14 years (b) Review by the higher Court
(c) 20 years (d) for life (c) Review of the decision of magistrate
76. Which of the following writs can be is- (d) Power of the court to determine the
sued to release the person from illegal de- Constitutional validity of the Act of
tention? Legislature or Executive action
(a) Habeas corpus (b) Mandamus 86. “Bonafide” means
(c) Prohibition (d) Quo warranto (a) In good faith
77. How many amendments have been made (b) In bad faith
to the Constitution of India so far? (c) In good behaviour
(a) 94 (b) 97 (d) In bad behaviour
87. The President of India subscribes the oath
(c) 104 (d) 114
in the presence of
78. What is the salary of the Vice-President of
(a) Vice-President
India?
(b) Prime Minister
(a) ` 1,50,000 (b) ` 1,25,000
(c) Chief Justice of India
(c) ` 1,10,000 (d) ` 1,00,000 (d) Law Minister of India
79. How many languages are recognized in 88. “Ultra Vires” is a term used for
the eighth schedule of the Constitution? (a) minute virus in the computer
(a) 8 (b) 10 (b) an act beyond the authority
(c) 18 (d) 22 (c) an act only authorized by law
80. In which case the Supreme Court upheld (d) an act carefully undertaken
the Constitutional validity of the Central 89. Joint sitting of both the Houses of Parlia-
Educational Institutions (Reservation in ment is presided over by the
Admissions) Act, 2006? (a) President (b) Vice-President
(a) Kesavananda Bharti Case (c) Prime Minister (d) Speaker
(b) T.M.A. Pai Foundation Case 90. How many High Courts are there in In-
(c) Ashok Kumar Thakur Case dia?
(d) Minerva Mill Ltd Case (a) 18 (b) 21
81. What is a “Moot Court”? (c) 25 (d) 27
(a) Debatable question 91. Candle : Sun : : Pound :
(b) Basic fact of the case (a) Water (b) Ocean
(c) Mock court (c) Sea (d) Well
(d) Magistrate’s real court 92. Pointing to a boy in the photograph, Mad-
82. Within how much period a person is re- hu said, “his sister is the only daughter
quired to be produced before the magis- of my father.” How is the boy related to
trate? Madhu’s father?
(a) Immediately (a) Father (b) Brother
(c) Son (d) Cousin
(b) Within 12 hours
93. Radha stands with her face in the South
(c) Within 24 hours
East direction. She walks 15 m and then
(d) Within 24 hours excluding the time
turns northwards and walks 12 m. How far
taken from the place of arrest to the
is she from the starting time?
nearest magistrate. (a) 12 m (b) 10 m
83. Capital punishment means punishment (c) 9 m (d) 5 m
for 94. Find the missing number in the following
(a) 7 years series 4, 6, 3, 5, 2, ?
(b) 10 years (a) 8 (b) 4
(c) 20 years (c) 3 (d) 6
(d) Life or death sentence 95. Which one of the following does not pos-
84. Right to education has been declared as sess the common property that the other
fundamental right under article .......... three numbers do?
(a) 21 (b) 21-A (a) 64 (b) 45
(c) 31 (d) 31-A (c) 36 (d) 25

CLAT-6-Previous Years Papers.indd 366 06/09/2010 17:50:11


PREVIOUS YEARS’ PAPERS 367

96. A is taller than C but shorter than G. S is (a) Monday (b) Tuesday
shorter than C but taller than R. B is taller (c) Wednesday (d) Thursday
than R but shorter than S. H is taller than 99. 1f 5 and 3, 8 and 6, 9 and 4, and 2 and
A but shorter than G. Who stands in the 7 change themselves mutually, what will
middle? 84325 become?
(a) S (b) C (a) 65937 (b) 69753
(c) A (d) B (c) 69573 (d) 96537
97. Fill in the blank in the series 47, 32, __,11, 100. Fill in the next two blanks in the series
5, 2 : 1 + 3; 4 + 5; 9 + 7; __ ; __ :
(a) 27 (b) 20 (a) 16 + 7 ; 25 + 9
(c) 15 (d) 12 (b) 14 + 9 ; 21 + 15
98. If there are five Saturdays in a month, (c) 16 + 9 ; 25 + 11
what will be the 1st day of the month? (d) 16 + 12 ; 25 + 16

ANSWERS
1. (a) 2. (b) 3. (d) 4. (b) 5. (a) 6. (b) 7. (a) 8. (c) 9. (a) 10. (b)
11. (b) 12. (a) 13. (d) 14. (c) 15. (a) 16. (a) 17. (c) 18. (a) 19. (c) 20. (a)
21. (c) 22. (a) 23. (d) 24. (d) 25. (c) 26. (d) 27. (a) 28. (b) 29. (d) 30. (c)
31. (a) 32.. (c) 33. (c) 34. (c) 35. (c) 36. (c) 37. (d) 38. (c) 39. (a) 40. (b)
41. (b) 42. (d) 43. (c) 44. (c) 45. (b) 46. (d) 47. (c) 48. (c) 49. (a) 50. (d)
51. (c) 52. (d) 53. (c) 54. (c) 55. (a) 56. (a) 57. (a) 58. (a) 59. (a) 60. (b)
61. (c) 62. (a) 63. (b) 64. (c) 65. (a) 66. (d) 67. (a) 68. (b) 69. (b) 70. (c)
71. (d) 72. (a) 73. (d) 74. (c) 75. (d) 76. (a) 77. (a) 78. (b) 79. (d) 80. (c)
81. (c) 82. (d) 83. (d) 84. (b) 85. (d) 86. (a) 87. (c) 88. (b) 89. (d) 90. (b)
91. X 92. (c) 93. (c) 94. (b) 95. (b) 96. (b) 97. (b) 98. (d) 99. (c) 100. (c)
Note: An ‘X’ in the key indicates that either the question is ambiguous or it has printing mistake. All
candidates will be given credit for this question.

CLAT-6-Previous Years Papers.indd 367 06/09/2010 17:50:11


368 PREVIOUS YEARS’ PAPERS

LLB ENTRANCE TEST (DU) 2009


1. Name the oil tanker that was hijacked by 11. On what date did Rakesh Sharma launch
Somali pirates in November, 2008? aboard Soyuz T-11 towards its destination,
(a) MV Elizabeth the Salyut 7 space station?
(b) MV Sirius Star (a) 13th January 1984
(c) Seabourn Spirit (b) 23rd April 1984
(d) Achille Lauro (c) 2nd April 1984
2. The Great Fire of Rome is associated (d) 26th January 1982
with 12. Polio virus is primarily transmitted via
(a) Maximus Quintus (a) Contaminated food and water
(b) Tacitus (b) Mosquito bite
(c) Pliny the Elder (c) Contaminated air
(d) Lucius Domitius (d) Mother to foetus
3. Which of the following is not a wireless 13. Which road in Delhi is known as the Fleet
data transfer technology? Street of India?
(a) EDGE (b) GPRS (a) Kasturba Gandhi Marg
(c) Bluetooth (d) 3GP (b) Race Course Road
4. Human saliva contains enzymes necessary (c) Bahadur Shah Zafar Road
for breakdown of (d) Parliament Street
(a) Proteins (b) Starch 14. Which of the following is popularly known
(c) Cellulose (d) Vitamins as the ‘Siamese Fighting Fish’?
5. Which of the following substance is an an- (a) Betta (b) Pirhana
ticoagulant? (c) Tuna (d) Guppy
(a) Heparin (b) Tannin 15. Which car maker is reportedly building
(c) Fibrin (d) Thrombin the Presidential car for the President of
6. The first metal used by man was United States, Barack Obama?
(a) Iron (b) Copper (a) Ford Motor Company
(c) Aluminium (d) Bronze (b) General Motors
7. Where is the Large Hadron Collider (LHC) (c) Mercedes-Benz
particle accelerator located? (d) Rolls Royce
(a) Switzerland and France 16. Treatment of Dengue fever may require
(b) Germany (a) Platelet transfusion
(c) United States of America and Mexico (b) Antibiotics
(d) Sweden (c) Administration of propanolol
8. Which of the following is not a private (d) All of the above
space tourism venture 17. Which of the following popular fictional
(a) Virgin Galactic characters is not the public face of a super-
(b) Intercosmos hero?
(c) MirCorp (a) Peter Parker (b) Bruce Wayne
(d) Bigelow Aerospace (c) Clark Kent (d) Paul Drake
9. Where is the world’s highest waterfall lo- 18. Which of the following countries conduct-
cated? ed its first anti-satellite weapon test in the
(a) Canada and United States year 2007?
(b) Argentina (a) United State of America
(c) Sri Lanka (b) Russia
(d) Venezuela (c) Iran
10. The chemicals used in tear gas are known (d) China
as 19. ‘Bar’ is a unit of
(a) Mustard gas (a) Humidity (b) Alcohol content
(b) Lachrymatory agents (c) Pressure (d) Depth
(c) Nitrous oxide 20. Who is considered the inventor of detec-
(d) Nerve agents tive fiction?

CLAT-6-Previous Years Papers.indd 368 06/09/2010 17:50:11


PREVIOUS YEARS’ PAPERS 369

(a) Arthur Conan Doyle (b) R. K. Narayanan


(b) Agatha Christie (c) R. K. Laxman
(c) Erle Stanley Gardner (d) K. R. Narayanan
(d) Edgar Allan Poe 30. Which event has been described as “the
21. Besides Indian scientific instruments, the effective birth of economics as a separate
Chandrayan carried instruments from var- discipline”, by the New Encyclopedia Bri-
ious other space agencies. Which, among tannica?
the following, is not one of them? (a) The emergence of the 18th century
(a) Bulgarian Academy of Sciences French Physiocrats.
(b) National Aeronautics and Space Ad- (b) The publication of Adam Smith’s
ministration (NASA) “The Wealth of Nations”.
(c) European Space Agency (ESA) (c) The development of Keynesian Eco-
(d) Russian Federal Space Agency nomics.
(RFSA) (d) The codification of economic formu-
22. Who provided the earliest evidence of the lae in the city States of Sumer.
Big Bang model of the Universe? 31. Edible mushroom is a form of
(a) Einstein (b) Fred Hoyle (a) Animal
(c) Edwin Hubble (d) Newton (b) Plant
23. Which of the following is used as a tool (c) Lower life form
for tracking human ancestry through fe- (d) Fungus
males? 32. ‘Nitrogen fixation’ refers to
(a) Y-Chromosome (a) Fixing achievable targets for reducing
(b) Nuclear DNA nitrogen pollution in the atmosphere.
(c) Mitochondrial DNA (b) Developing means to synthesize and
(d) X-chromosome produce Nitrogen on a mass scale.
24. Which of the following is not a Kingdom (c) The process by which Nitrogen is
under the Five Kingdom taxonomical converted from its natural form in the
classification? atmosphere to Nitrogen compounds.
(a) Animalia (b) Plantae (d) The procedure for optimal utilization
(c) Monera (d) Archaea of Nitrogen based fertilizers in field.
25. Which telecom operator was the first to 33. Jacques Yves Cousteau is credited with
announce third-generation (3G) telecom (a) Liberating Paris in the Second World
services in India? War.
(a) Bharti Airtel (b) Developing French naval warfare
(b) Reliance Communications techniques.
(c) MTNL (c) Pioneering marine conservation.
(d) TATA Infocomm (d) Undertaking High Sea adventure
26. Which team won the Indian Premier trips.
League tournament in 2008? 34. Which, among the following, is not a liter-
(a) Chennai Super Kings ary piece by R. K. Narayan:
(b) Rajasthan Royals (a) The Man Eater of Malgudi
(c) Bangalore Royal Challengers (b) The Guide
(d) Deccan Chargers (c) Swami and Friends
27. What is the aerobatics demonstration team (d) Malgudi Junction
of Indian Navy known as? 35. Which of the following establishments
(a) Sea Hawks (b) Surya Kiran in India was modelled on the lines of the
(c) Sagar Pawan (d) Blue Angels United States WestPoint?
28. Water retaining plants adapted to arid cli- (a) The National Defence Academy
mates are known as (b) The National Academy of Sciences
(a) Phylloclades (b) Hemiepiphytes (c) The Indian Administrative Services
(c) Cactii (d) Succulents (d) The Defence Research and Develop-
29. Who created the ‘Common Man’ associ- ment Organization
ated with the Times of India? 36. Who, among the following fictional char-
(a) R. K. Narayan acters, is not a creation of Satyajit Ray?

CLAT-6-Previous Years Papers.indd 369 06/09/2010 17:50:11


370 PREVIOUS YEARS’ PAPERS

(a) Feluda 46. What is the unit for temperature in the


(b) Byomkesh Bakshi metric system?
(c) Professor Shonku (a) Fahrenheit (b) Kelvin
(d) Topshe (c) Degrees (d) Celsius
37. Which British doctrine is considered as 47. Where is the painting, Mona Lisa dis-
one of the reasons leading to the First War played?
of Independence, also known as the Sepoy (a) Louvre, Paris
Mutiny? (b) National Gallery of London
(a) Doctrine of Lapse (c) Luxembourg, Paris
(b) Doctrine of Contributory Negligence (d) Orsay, Paris
(c) Doctrine of Incompetence 48. Aristotle argued for the existence of a di-
(d) Doctrine of Frustration of Purpose vine being described as?
38. The vermiform appendix in humans is an (a) God
example of (b) Prime Mover
(a) Reflex action mechanism (c) The Being
(b) Digestive System organ (d) Spiritual Entity of Soul
(c) Vestigial Organ 49. Which art movement was led by Pablo Pi-
(d) Cognitive Behavioural Organ casso?
39. Which phyto-hormones influence plant (a) Cubism (b) Futurism
growth? (c) Fauvism (d) Impressionism
(a) Auxin (b) Gibberlin 50. Carl Jung, the Swiss psychologist is re-
(c) Pheromone (d) Lignin garded as founder of which school of psy-
40. Cell wall in plants is composed of chology?
(a) Cambium (b) Cellulose
(a) Ego psychology
(c) Lignin (d) Cytokinin
(b) Cultural-historical psychology
41. Which plant mimics the scent and other
(c) Evolutionary psychology
characteristics of a female wasp to lure a
(d) Analytical psychology
male wasp of the species for its pollina-
51. Munchausen Syndrome refers to:
tion?
(a) A mental disorder in which the affect-
(a) Central American Yucca
ed person fakes or exaggerates illness
(b) Australian Orchid
to gain attention and treatment.
(c) Indian Banyan
(b) A condition in which the affected per-
(d) Chinese Lily
42. Name the antimicrobial substance found son is excessively worried or preoccu-
in tears, saliva, sweat etc.? pied about having illness.
(a) Enzyme (b) Lysozyme (c) An eating disorder involving a com-
(c) Pepsin (d) Co-Enzyme pulsion to eat or avoid eating.
43. Root nodule rhizobia in legumes are an (d) A disorder involving a fear of being
example of negatively evaluated by others or be-
(a) Parasitism (b) Symbiosis ing embarrassed because of impulsive
(c) Predation (d) Commensalism acts.
44. Which of the following is associated with 52. Which event led to a decision to provide
Marx’s economics? the services of United States’ Global Posi-
(a) Theories of Surplus Value tioning System (GPS) to anyone, for civil-
(b) Essay on the Principle of Population ian purposes, free of charge?
(c) On the Principles of Political Econo- (a) The series of losses of merchant ships
my and Taxation and airplanes passing the Bermuda
(d) Okishio’s theorem Triangle.
45. The movie ‘A Fistful of Dollars’ was based (b) The shooting down of a Korean Air
on which movie? Lines flight by Soviet jet interceptors.
(a) Sholay (c) The Space Shuttle Challenger disas-
(b) Yojimbo ter.
(c) The Good, The Bad, and The Ugly (d) The failure of NASA’s Mars Observer
(d) Support Your Local Gunfighter mission.

CLAT-6-Previous Years Papers.indd 370 06/09/2010 17:50:11


PREVIOUS YEARS’ PAPERS 371

53. Which Company acquired a major- (b) Rajiv Gandhi and J. R. Jayewardene
ity shareholding in one of India’s largest (c) Rajiv Gandhi and Ranasighe Prema-
drugmaker, Ranbaxy? dasa
(a) Zenotech Laboratories Ltd. (d) Rajiv Gandhi and Chandrika Kumara-
(b) Cipla tunga
(c) Pfizer 64. What time zone does India fall into?
(d) Daiichi Sankyo (a) GMT+IST (b) GMT +5.5
54. Which Company was in news for the grue- (c) GMT +3.5 (d) GMT -3.5
some death of its CEO at the hands of its 65. Which of the following events is associ-
agitating employees in September, 2008? ated with ‘Smiling Buddha’?
(a) Gilera (b) Graziano (a) The nuclear weapons test conducted
(c) Bernardelli (d) Gaggia by India in 1998.
55. In which year was State of Israel admitted (b) The launch of Chandrayan.
as a member of the United Nations: (c) The launch of PSLV-II.
(a) 1950 (b) 1949 (d) The nuclear weapons test conducted
(c) 1948 (d) 1952 by India in 1974.
56. Name the author of the book, “Sea of Pop- 66. Which event led to Rabindranath Tagore’s
pies” renouncement of his knighthood?
(a) William Dalrymple (a) Civil Disobedience Movement
(b) Amitav Ghose (b) Jallianwalla Bagh massacre
(c) Chetan Bhagat (c) Morley Minto Reforms
(d) Salman Rushdie (d) Chauri Chaura
57. Which country won the UEFA European 67. Which one of the following is considered
a major strategic event of the 1857 Mu-
Football Championship in 2008?
tiny/First War of Independence?
(a) Brazil (b) Argentina
(a) Wheeler’s entrenchment
(c) Germany (d) Spain
(b) Massacre at Sati Chaura Ghat
58. Who won the 2008 Man Booker Prize?
(c) Bibighar massacre
(a) Steve Toltz (b) Aravind Adiga
(d) All of the above
(c) Chetan Bhagat (d) Amitav Ghosh
68. Which of the following is associated with
59. Bone marrow transplants are effective in
the business of retail supply and distribu-
cases of:
tion of electricity in Delhi?
(a) Thalassemia (b) Leukemia (a) Delhi Power Company Ltd
(c) Anemia (d) Diabetes (b) BSES Yamuna Power Ltd
60. The Chandrayan has detected the presence (c) Delhi Transco Ltd
of one of the following elements in the lu- (d) Delhi Electricity Regulatory Commis-
nar soil. sion
(a) Aluminum (b) Copper 69. How many Olympic Gold medals has the
(c) Iron (d) Uranium American swimmer, Michael Phelps won
61. Which party won the December 2008 gen- in his career till date?
eral elections in Bangladesh? (a) Eight (b) Fourteen
(a) Awami League coalition. (c) Nineteen (d) Twelve
(b) Bangladesh Nationalist Party (BNP) 70. What is the Super WASP?
coalition. (a) UK’s extra-solar planet detection pro-
(c) Jamaat-e-Islami Bangladesh gram
(d) Bangladesh Jatiya Party (b) Genetically engineered giant Wasp
62. Who was the Chief Guest at the 60th Re- (c) Anti ballistic missile shield
public Day celebrations? (d) Long range tracking radar
(a) Nursultan Nazarbayev 71. The policy of British East India Company
(b) Nicolas Sarkozy in India during the period of 1813–1858 is
(c) Jacques Chirac described as:
(d) Vladimir Putin (a) Policy of Ring Fence
63. The Indo-Sri Lanka Peace Accord was en- (b) Policy of Annexation by Lapse
tered into under the leadership of (c) Policy of Subordinate Isolation
(a) Indira Gandhi and J. R. Jayewardene (d) Policy of Subordinate Union

CLAT-6-Previous Years Papers.indd 371 06/09/2010 17:50:11


372 PREVIOUS YEARS’ PAPERS

72. Which modern Indian historian has de- (a) Nuclear weapons
scribed the events of 1857 as “neither first (b) Lunar landing
nor National nor War of Independence” (c) Netscape
(a) V. D. Savarkar (d) Television
(b) Dadabhai Naoroji 80. Who was crowned the 57th Miss Universe
(c) S. N. Sen at Miss Universe 2008?
(d) R. C. Majumdar (a) Riyo Mori
73. The airspace over one of the following (b) Dayana Mendoza
was declared a ‘no-fly zone’ in December (c) Rebeca Moreno
2008, thereby prohibiting all flying activ- (d) Taliana Vargas
ity up to a height of 10,000 feet over it. 81. Who among the following has not won
Choose the correct location. a medal in individual event at the Olym-
(a) Parliament of India pics?
(b) Satish Dhawan Space Centre, Sri- (a) Rajyavardhan Rathore
harikota (b) Abhinav Bindra
(c) Kalpakkam Nuclear Power Plant (c) Leander Paes
(d) Rashtrapati Bhavan (d) Anjali Bhagat
74. Who is the Governor of Reserve Bank of 82. Who among the following was the recipi-
India? ent of Padma Bhushan for distinguished
(a) Dr D. Subbarao services in sports?
(b) Dr Rakesh Mohan (a) Harbhajan Singh
(c) Dr Y. V. Reddy (b) Mahendra Singh Dhoni
(d) Dr Bimal Jalan (c) Abhinav Bindra
75. Which of the following offices has never (d) Balbir Singh Khullar
been held by Prime Minister Manmohan 83. A dysregulation of micro-RNA is linked
Singh? with
(a) Finance Minister (a) Cancer (b) Hepatitis
(b) Governor, Reserve Bank of India (c) Agoraphobia (d) Diabetes
(c) Honorary Professor, Delhi School of 84. The Global Potato Conference 2008 was
Economics held at
(d) Member, Lok Sabha (a) Lima, Peru
76. Op-Ed, with reference to a newspaper ar- (b) New Delhi, India
ticle, refers to (c) Mexico City, Mexico
(a) Opinion Editorial (d) Sao Paulo, Brazil
(b) Opposite Editorial 85. Which of the following is not a video
(c) Opinion Educational gaming console?
(d) Operative Editorial (a) Wii (b) Playstation
77. “Stability 2008” refers to: (c) X-Box (d) Macbook
(a) Russian strategic military exercise. 86. Which law was enacted in the United
(b) Economic bailout package envisaged States in 2008, for the bailout of the U.S.
by the Obama regime. financial system?
(c) United Nations endeavour to bring (a) Emergency Economic Stabilization
peace to Somalia. Act of 2008
(d) Measures to maintain ecological bal- (b) Troubled Asset Relief Act of 2008
ance in tropical rain forests. (c) Energy Improvement and Extension
78. Who, among the following Indians, pur- Act of 2008
chased the sword of Tipu Sultan at an auc- (d) Mental Health Parity and Addiction
tion in London? Equity Act of 2008
(a) Sanjay Khan (b) Anil Ambani 87. Who led the FIDE Men’s Chess ratings as
(c) Vijay Mallya (d) Ratan Tata on 1st February 2009?
79. Which of the following is one of the ten (a) Anatoli Karpov
forces that ‘flattened the world’ in the (b) Viswanathan Anand
opinion of Thomas L. Friedman in his (c) Veselin Topalov
book, “The World is Flat”? (d) Vassily Ivanchuk

CLAT-6-Previous Years Papers.indd 372 06/09/2010 17:50:11


PREVIOUS YEARS’ PAPERS 373

88. The de facto administrative capital of (b) Planning Commission


LTTE was located at (c) National Defence
(a) Jaffna (b) Kilinochchi (d) Panchayati Raj
(c) Vanni (d) Elephant Pass 97. Which was the first element to be discov-
89. Which Pakistan based group was des- ered through synthesis?
ignated by the United Nations Security (a) Neptunium (b) Plutonium
Council as a terrorist organization follow- (c) Technetium (d) Promethium
ing the November, 2008 Mumbai attacks 98. AIDS stands for
(a) Lashkar-e-Taiba (a) Acquired Immuno Deficiency Syn-
(b) Jamaat-ud-Dawah drome
(c) Jamaat-e-Islami (b) Acquired Immunity Deficiency Syn-
(d) Jaish-e-Mohammed drome
90. Which temple witnessed a stampede in (c) Acquired Immuno Disease Syndrome
September 2008? (d) Acquired Immuno Deficiency Symp-
(a) Chamunda Devi Temple, Jodhpur, toms
Rajasthan Questions 99 to 103 are based on the follow-
(b) Jaggannath Temple, Puri, Orissa ing passage:
(c) Tirupati Temple, Tirupati, Andhra “You’ve been in love, of course! If not,
Pradesh you’ve got it to come. Love is like the measles;
(d) Chamunda Devi, Kangra, Himachal we all have to go through it. Also like the mea-
Pradesh sles, we take it only once. One never need be
91. Which term is being served by Smt Sheila afraid of catching it a second time. The man who
Dixit as the Chief Minister of Delhi? has had it can go into the most dangerous places
(a) Second (b) Fourth and play the most foolhardy tricks with perfect
(c) First (d) Third safety. He can picnic in shady woods, ramble
92. Which State among the following wit- through leafy aisles, and linger on mossy seats
nessed the largest polling percentage in to watch the sunset. He fears a quiet country-
the State Elections in 2008? house no more than he would his own club. He
(a) Jammu and Kashmir can join a family party to go down the Rhine. He
(b) Meghalaya can, to see the last of a friend, venture into the
(c) Mizoram very jaws of the marriage ceremony itself. ...
(d) Delhi No, we never sicken with love twice. Cu-
93. With which of the following did the State pid spends no second arrow on the same heart.
Bank of India enter into a Joint venture Love’s handmaids are our life-long friends. Re-
Agreement for undertaking general insur- spect, and admiration, and affection, our doors
ance business? may always be left open for, but their great ce-
(a) New India Assurance Ltd lestial master, in his royal progress, pays but one
(b) Insurance Australia Group visit and departs. We like, we cherish, we are
(c) Lehman Brothers Holdings Inc very, very fond of—but we never love again. A
(d) Allianz man’s heart is a firework that once in its time
94. With which country did India sign an ex- flashes heavenward. Meteorlike, it blazes for a
tradition treaty in November, 2008? moment and lights with its glory the whole world
(a) Egypt (b) Sri Lanka beneath. Then the night of our sordid common-
(c) Portugal (d) Nepal place life closes in around it, and the burned-out
95. The 14th Conference of Parties to the case, falling back to earth, lies useless and un-
UN Framework Convention of Climate cared for, slowly smoldering into ashes”.
Change was held at 99. In the opinion of the author, love is like
(a) Bali, Indonesia measles because:
(b) Pozanan, Poland (a) Each one of us has to go through it.
(c) Montreal, Canada (b) It happens only once in life.
(d) Buenos Aires, Argentina (c) It makes a person reckless.
96. Balwant Rai Mehta Committee Report, (d) It is painful.
1957 is associated with Which of the above are correct?
(a) Tax Reforms (a) (a), (b) and (d)

CLAT-6-Previous Years Papers.indd 373 06/09/2010 17:50:11


374 PREVIOUS YEARS’ PAPERS

(b) (a), (b) and (c) (d) An inclination towards seeking recog-
(c) (a) and (b) nition.
(d) All of them are correct 111. Choose the most appropriate word to fill
100. The writing style of the author may be de- in the blanks. The General’s __________
scribed as at the battlefield paid off.
(a) Romantic (b) Melodramatic (a) Gamut (b) Gambit
(c) Satirist (d) Somber (c) Movement (d) Commands
101. Choose the word having the same mean- 112. The speaker made a __________ argu-
ing as “ramble”. ment against the proposed law.
(a) Saunter (b) Tumble (a) perspicuous (b) perspicacious
(c) Shamble (d) Rally (c) precipitous (d) conspicuous
102. What, according to the author, are the 113. A _______ by the Principal disciplined the
emotions that accompany love? rowdy schoolboys.
(a) Admiration and affection (a) Censor (b) Censure
(b) Courage and fearlessness (c) Direction (d) Suggestion
(c) Sickness 114. Since the Parliament was not in session,
(d) Foolhardiness the President promulgated a/an_______ in
103. In which of the following ways is a man’s view of the urgency.
heart been described as being similar to a (a) Ordnance (b) Order
firework (c) Ordinance (d) Statute
(a) A firework lights up the sky. 115. Studies have revealed that a female Whale
(b) A firework flashes only momentarily spends considerable time nursing its new-
and falls back to earth. born ________
(a) Whale
(c) Having lighted up once, it is of no fur-
(b) Brood
ther use.
(c) Calf
(d) All of the above
(d) Pre-Toothed whale
104. Choose the word having the same mean-
116. Aamir _________ 14 films and I think his
ing as “obsequious”.
latest is the best.
(a) Subservient (b) Obscene
(a) has made (b) made
(c) Rude (d) Observant
(c) had made (d) was making
105. Choose the word having the same mean-
117. Sorry, we’re late. We ________ the wrong
ing as “averse”. turning.
(a) Adverse (b) Unfavorable (a) were taking (b) took
(c) Opposed (d) Reversed (c) had taken (d) are taking
106. Choose the word having the same mean- 118. Next week I _______ Akash for 20 years.
ing as “apposite”. (a) will have known
(a) Inapt (b) Opposite (b) am knowing
(c) Relevant (d) Casual (c) will have been knowing
107. Choose the word having a meaning oppo- (d) know
site of “sequester”. 119. Following the earthquake, __________ in
(a) Cloister (b) Isolate the area _________ major repair work.
(c) Unify (d) Distribute (a) each buildings ... need
108. Choose the word having a meaning oppo- (b) each of buildings ... needs
site of “sagacious”. (c) every building ... needs
(a) Sanguine (b) Capricious (d) every building ... need
(c) Facetious (d) Foolish 120. The radio station is owned by the Enter-
109. Choose the word having a meaning oppo- tainment Group, __________ Chairman is
site of “erudite”. Mr Vishal Wadhwa.
(a) Calm (b) Scholarly (a) who’s (b) whose
(c) Ignorant (d) Filthy (c) that’s (d) which
110. Cognition refers to: 121. Jack _______ leave the hospital only six
(a) The ability to recognize. hours after the operation.
(b) The reflex to a condition. (a) could (b) can
(c) The faculty of perception. (c) is able to (d) was able to

CLAT-6-Previous Years Papers.indd 374 06/09/2010 17:50:11


PREVIOUS YEARS’ PAPERS 375

122. Choose the correct sentence. 131. The 44th Amendment Act of 1978 to the
(a) He described me the situation. Constitution of India:
(b) He describe the situation to me. (a) Deleted the Fundamental Right to ac-
(c) He described the situation to me. quire, hold and dispose off property.
(d) I was described the situation. (b) Introduced the Fundamental Right to
123. Who _________ was coming to see me free and compulsory education.
this morning? (c) Deleted Fundamental Right to work
(a) did you say (b) did you say that (d) Suspended the Fundamental Right to
(c) you did say (d) you said constitutional remedies.
124. I _________ our anniversary and I 132. Who declared a state of emergency in In-
________ how to make it up to her. dia under Article 352 of the Constitution
(a) completely forgot ... don’t just know of India in the year 1975?
(b) completely forgot ... just don’t know (a) Indira Gandhi
(c) forgot completely ... don’t just know (b) Giani Zail Singh
(d) forgot completely ... just don’t know (c) Fakhruddin Ali Ahmed
125. “FOB” in contracts of carriage refers to: (d) Sanjay Gandhi
(a) Fresh on Board 133. Who is the Solicitor General of India?
(b) Free on Board (a) Milon K. Bannerjee
(c) Fresh off Boat (b) Goolam E. Vahnavati
(d) Free off Board (c) Harish Salve
126. Dishonour of a cheque is an offence un- (d) Soli Sorabjee
der: 134. Following a ruling of the Delhi High
(a) Section 138 of the Negotiable Instru- Court, which among the following is not
ments Act a criteria for designating an advocate as
(b) Section 420 of the Indian Penal Code Senior Advocate.
(c) Section 73 of the Contract Act (a) Ability
(d) Section 41 of the Code of Criminal (b) Standing at Bar
Procedure (c) 12 years of uninterrupted law practice
127. A Lease Agreement is a form of: (d) Special knowledge or experience of
(a) Promissory note law
(b) Bill of Exchange (e) Money
(c) License (a) (b), (c) and (e) (b) (e)
(d) Contract (c) (b) and (e) (d) (c) and (e)
128. Which of the following is not an Alternate 135. The Bill for Welfare of Domestic Help has
Dispute Redressal Mechanism? been passed by the state legislature of
(a) Mediation (b) Lok Adalat (a) Orissa (b) Delhi
(c) Arbitration (d) Plea Bargaining (c) Bihar (d) Maharashtra
129. The owner and landlord of a flat, allotted 136. Which Bill of the Parliament seeks to give
to him by the Delhi Development Authori- effect to Article 21A of the Constitution of
ty, may recover possession of the flat from India?
his tenant by taking recourse to the law. (a) Right to Education Bill, 2008
Which of the following may not have any (b) Central Universities Bill, 2008
bearing on such recovery of possession? (c) Limited Liability Partnership Bill,
(a) Delhi Rent Control Act, 1958 2008
(b) Agreement to Lease for a fixed term (d) National Investigation Agency Bill,
(c) Indian Contract Act, 1872 2008
(d) Public Premises (Eviction of Unau- 137. The Rules restricting the display of use of
thorised Occupants) Act, 1971 tobacco products in films and television
130. Which of the following is not related to programmes was recently struck down by
protection of rights of consumers? the High Court of Delhi, being violative
(a) 24th December of
(b) Jago Grahak, Jago (a) Article 19(a)(a) of the Constitution of
(c) 15th March India.
(d) All of them are related to consumer (b) Article 21 of the Constitution of In-
protection. dia.

CLAT-6-Previous Years Papers.indd 375 06/09/2010 17:50:11


376 PREVIOUS YEARS’ PAPERS

(c) Section 5A of the Cinematograph Act, 145. Who appoints the Attorney General of In-
1952. dia?
(d) Article 14 of the Constitution of In- (a) Chief Justice of India.
dia. (b) Ministry of Law and Justice, Govern-
138. Against which of the following Indian es- ment of India.
tablishment has a class action lawsuit been (c) President of India
filed recently in the United States District (d) President of India in consultation with
Court, Southern District of New York? the Chief Justice of India.
(a) Price Waterhouse Coopers 146. Who among the following is an accused in
(b) Satyam Computer Services Ltd the Uphaar cinema fire case?
(c) Maytas Infra Ltd (a) Vikas Yadav (b) Sushil Ansal
(d) All of the above (c) Sanjeev Nanda (d) Manu Sharma
139. Which judge of the Supreme Court of In- 147. What is an extradition treaty?
dia was recently reported to have equated (a) A treaty for surrender of an alleged or
a terrorist with an animal and thus deserv- convicted criminal by one nation to
ing of animal rights instead of human another.
rights, in his speech? (b) A treaty for ensuring fair trial of an
(a) Justice Arijit Pasayat alleged or convicted criminal trans-
(b) Justice B. N. Agarwal ferred by one nation to another.
(c) Justice S. B. Sinha (c) A treaty for international trial of an al-
(d) Justice S. H. Kapadia leged or convicted criminal.
140. Who founded the doctrine of Utilitarian- (d) A treaty for trial of an alleged or con-
ism? victed criminal by a nation, for crime
(a) John Austin committed or allegedly committed in
(b) Holmes another nation.
(c) Jeremy Bentham 148. Which of the following is entrusted with
(d) Ronald Dworkin a statutory duty of laying down the stan-
141. Which of the following is not related to dards of professional conduct and eti-
legal research? quette for advocates in India?
(a) www.manupatra.com (a) Bar Association of India
(b) Black’s Law Dictionary (b) Bar Council of India
(c) Boyle’s Law (c) Delhi Bar Council
(d) Law of the Sea (d) Supreme Court of India
142. Which of the following is considered the 149. Which is the present Law Commission of
first international crime? India?
(a) Piracy on the High Seas (a) 13th (b) 16th
(b) Violation of Human Rights (c) 18th (d) 21st
(c) Slavery 150. Which University had established Faculty
(d) Software Piracy of Law was in 1924?
143. Under which Article of the Constitution (a) Allahabad University
of India is the law laid down by Supreme (b) Aligarh Muslim University
Court binding on all courts? (c) Banaras Hindu University
(a) Article 32 (b) Article 141 (d) University of Delhi
(c) Article 226 (d) Article 124 151. Who is the author of the book “A Bunch of
144. Driving without a valid “Pollution Under Old Letters”?
Control Certificate” is punishable under (a) Pt Jawarhar Lal Nehru
the provisions of (b) Mahtma Gandhi
(a) The Motor Vehicles Act, 1988 (c) Smt Indira Nehru Gandhi
(b) The Air (Prevention and Control of (d) Shri K.C. Pant
Pollution) Act, 1981 152. What is meant by the expression “Separa-
(c) The Environment (Protection) Act, tion of Powers”?
1986 (a) Division of powers and spheres of
(d) The Public Liability Insurance Act, authority between the police and the
1991. army at local and national level

CLAT-6-Previous Years Papers.indd 376 06/09/2010 17:50:11


PREVIOUS YEARS’ PAPERS 377

(b) Model of governance dividing the (a) travel abroad


powers and responsibilities of the (b) practise any profession
State between its Executive, Legisla- (c) get assistance of a legal practitioner of
ture and a Judiciary his choice if arrested under preventive
(c) Provision under the Constitution of detention law
India for separate High Courts for (d) challenge an arbitrary law
each state of the country 160. The only University in India covered in
(d) Division of rights and liabilities be- the first “Top 500 World University Rank-
tween the Central Government, State ings” for the year 2007 published by the
Governments and the citizens of India Times Higher Education Supplement
through the Constitution of India read was
with the Right to Information Act (a) University of Bombay
153. Which of the following theories of crime is (b) Punjab University
based on the works of Henry McKay and (c) Shantiniketan
Clifford R. Shaw of the Chicago School (d) University of Delhi
thought? Choose the most appropriate answer to each
(a) Social disorganization theory Factual Situation given in Question No. 161
(b) Strain theory to 163 within the scope of the Principle men-
(c) Rational choice theory tioned.
(d) Routine activity theory 161. Principle: A person, who lawfully brings
154. The Child Labour (Prohibition and Regu- on his land something which though harm-
lation) Act, as amended in 2006, prohibits less, but will do mischief if it escape, must
the employment of children under the age keep it at his peril, and if he does not, he is
answerable for all the damage.
of ___ years as domestic servants.
Factual Situation: ‘A’ was the owner of a
(a) 12 (b) 18
mill. In order to supply it with water he
(c) 16 (d) 14
constructed a reservoir upon nearby land
155. Which documentary film won an Oscar
by employing engineers and contractors.
during 81st Academy Awards 2009?
‘B’ was the owner of coal mines, under
(a) Slumdog Millllionaire
lands, close to but not adjoining the prem-
(b) Smile Pinki
ises on which the reservoir was construct-
(c) Pinki Sonkar
ed. The contractors, while excavating for
(d) Smile Train the bed of the reservoir, came upon aban-
156. Which of the following bodies is not set doned shafts and filled them with soil not
up under the Constitution of India? suspecting that that they were abandoned
(a) Finance Commission mine shafts. The reservoir was completed
(b) Union Public Service Commission and partly filled. Within days the bed of
(c) Supreme Court of India the reservoir gave way and burst, leading
(d) National Human Rights Commission to flow of water through the channels con-
157. Which of the following is not true about nected with B’s mine. Is ‘A’ liable to pay
LED? damages for loss caused to ‘B’?
(a) Its an abbreviation for Light Emitting (a) ‘A’ is not liable because there was no
Diode negligence on his part. He was not
(b) It can emit light of various colors aware that the reservoir bed was con-
(c) It is a type of superconductor nected to B’s mines.
(d) It can emit light of infrared and ultra- (b) ‘A’ is liable to pay damages to ‘B’
violet frequencies because he brought the water on his
158. How many official languages have been property which would have caused
recognized under the Constitution of In- mischief if it escaped and it did es-
dia? cape.
(a) 14 (b) 18 (c) ‘A’ is not liable because ‘B’ never
(c) 20 (d) 22 informed him the existence of B’s
159. An arrested person under the Indian Con- mines.
stitution does not have a fundamental right (d) ‘A’ is liable because he hired the ser-
to ____________ vices of unqualified engineers.

CLAT-6-Previous Years Papers.indd 377 06/09/2010 17:50:11


378 PREVIOUS YEARS’ PAPERS

162. Principle: Cheating is defined as deceiv- an electronics company through a draw


ing any person to deliver any property and of lots and installed the same in his taxi.
it includes the dishonest concealment of Some time later, the music system stopped
facts. Cheating by personation means a working. ‘A’ approached the electronics
person cheating another by pretending to company for a replacement but the com-
be some other person, or by knowingly pany kept delaying. Can ‘A’ sue the com-
substituting one person for another, or pany in a consumer forum?
representing that he or any other person is (a) Yes, ‘A’ is not using the music system
a person other than he or such other person for any commercial purpose. He is
really is. just plying a taxi for earning his liveli-
Factual Situation: ‘A’, while leaving a hood.
restaurant mistakingly picked up the um- (b) Yes, there is a deficiency in goods
brella of ‘B’ instead of his own. The next since the music system became non-
day he decided to return to the restaurant functional.
with the umbrella, hoping to find the real (c) No, ‘A’ is using his taxi for ferrying
owner. ‘C’, who had never seen ‘B’ in per- passengers which is a commercial
son and had only communicated with ‘B’ purpose. Hence he is not a consumer.
through letters, was on his way to the res- (d) No, ‘A’ is not a consumer since he has
taurant to meet ‘B’. Since ‘B’ and ‘C’ had not paid any consideration for the mu-
never met, it was agreed between them to sic system.
identify each other at the pre-appointed 164. *
spot, the restaurant door, by the clothing 165. Dr Hari Singh Gaur was the first Dean,
of ‘C’ and umbrella of ‘B’ which they had Faculty of Law and the Vice-Chancellor
of
described to each other in detail. ‘C’ saw
(a) Dr Hari Singh Gaur University
‘A’ at the door of the restaurant and identi-
(b) University of Delhi
fied the umbrella. Thinking ‘A’ to be ‘B’,
(c) University of Lucknow
‘C’ delivered a parcel labelled Mr B to ‘A’.
(d) None of the above
‘A’ received the same without protest and
166. Which of the following bodies had ap-
promptly returned home without looking
proached the Supreme Court of India seek-
for ‘B’. Is ‘A’ guilty of cheating by person-
ing permission to interrogate the deposed
ation?
Chairman of Satyam Computer Services?
(a) ‘A’ is not guilty because he was go- (a) Registrar of Companies
ing to return the umbrella to the real (b) Securities and Exchange Board of In-
owner and handing over ‘B’s’ packet dia
to ‘A’ was ‘C’s’ mistake. (c) Reserve Bank of India
(b) ‘A’ is not guilty because he was over- (d) Central Bureau of Investigation
come by a sudden temptation. 167. How many persons were nominated for
(c) ‘A’ is guilty because he was aware award of Ashok Chakra on 26th January
of the mistaken identity and that the 2009?
parcel was not meant for him but for (a) 9 (b) 10
‘B’. (c) 11 (d) 12
(d) ‘A’ is guilty because the parcel might 168. Which politician, considered responsible
have been valuable to ‘B’. for Babri Masjid demolition has apolo-
163. Principle: A consumer is a person who gised for his role in the demolition?
buys any goods or services for a consider- (a) Ms Uma Bharati
ation and uses the goods for non-commer- (b) Mr Madan Lal Khurana
cial purposes. Exclusive use of the goods (c) Mr Kalyan Singh
for the purposes of earning his livelihood, (d) None of the above
by means of self-employment is not con- 169. Which is the University whose Faculty of
sidered as commercial use. A consumer Law has been given first rank among all
can approach a consumer forum if there is Faculties having three-year LL.B. Degree
a deficiency in goods or service. course?
Factual Situation: ‘A’, a taxi driver re- (a) National Law School University, Ban-
ceived a car music system for free, from galore

CLAT-6-Previous Years Papers.indd 378 06/09/2010 17:50:11


PREVIOUS YEARS’ PAPERS 379

(b) University of Delhi (b) to fill one self with airy things
(c) University of Hyderabad (c) to affect humility
(d) W. B. National University of Juridical (d) to have a haughty opinion about one-
Sciences, Kolkata self
170. According to a survey conducted by the 173. Which one of the following is associated
Union Ministry of Labour and Employ- with mentally or emotionally disturbed
ment, ___________ workers lost their job (a) psychological
between October and December, 2008 in (b) psychopathic
India on account of recession: (c) psychoanalytical
(a) three lakh (b) four lakh (d) psychotherapic
(c) five lakh (d) seven lakh 174. The prediction by palm-reading is called
171. The phrase ‘de facto’ means: (a) chironomy (b) chirography
(a) all the facts (c) chiropidy (d) chiromancy
(b) all the facts and figures 175. Neuralgia is associated with
(c) bona fide (a) Record of heart beats
(d) for all practical purposes (b) Mental inbalance
172. The expression “to assume air on” means (c) Nerve pain
(a) to assume inferiority (d) Inflamation of the nerves

ANSWERS
1. (b) 2. (d) 3. (d) 4. (b) 5. (a) 6. (b) 7. (a) 8. (b) 9. (d) 10. (b)
11. (b) 12. (a) 13. (c) 14. (a) 15. (b) 16. (a) 17. (d) 18. (d) 19. (c) 20. (d)
21. (d) 22. (c) 23 (c) 24. (d) 25 (c) 26. (b) 27. (c) 28. (d) 29. (c) 30. (b)
31. (d) 32. (c) 33. (c) 34. (d) 35. (a) 36. (b) 37. (a) 38. (c) 39. (a) 40. (b)
41. (b) 42. (b) 43. (b) 44. (a) 45. (b) 46. (d) 47. (a) 48. (b) 49. (a) 50. (d)
51. (a) 52. (b) 53. (d) 54. (b) 55. (b) 56. (b) 57. (d) 58. (b) 59. (b) 60. (c)
61. (a) 62. (a) 63. (b) 64. (b) 65. (d) 66. (b) 67. (d) 68. (b) 69. (b) 70. (a)
71. (c) 72. (d) 73. (c) 74. (a) 75. (d) 76. (b) 77. (a) 78. (c) 79. (c) 80. (b)
81. (d) 82. (c) 83. (a) 84. (b) 85. (d) 86. (a) 87. (c) 88. (b) 89. (b) 90. (a)
91. (d) 92. (c) 93. (b) 94. (a) 95. (b) 96. (d) 97. (c) 98. (a) 99. (c) 100. (c)
101. (a) 102. (a) 103. (d) 104. (a) 105. (c) 106. (c) 107. (c) 108. (d) 109. (c) 110. (c)
111. (c) 112. (a) 113. (b) 114. (c) 115. (c) 116. (a) 117. (b) 118. (a) 119. (c) 120. (b)
121. (d) 122. (c) 123. (a) 124. (b) 125. (b) 126. (a) 127. (d) 128. (d) 129. (d) 130. (d)
131. (a) 132. (c) 133. (b) 134. (d) 135. (d) 136. (a) 137. (a) 138. (b) 139. (a) 140. (c)
141. (c) 142. (a) 143. (b) 144. (a) 145. (c) 146. (b) 147. (a) 148. (b) 149. (c) 150. (d)
151. (a) 152. (b) 153. (a) 154. (d) 155. (b) 156. (d) 157. (c) 158. (d) 159. (a) 160. (d)
161. (b) 162. (c) 163. (d) 164. ( ) 165. (b) 166. (b) 167. (c) 168. (c) 169. (b) 170. (c)
171. (d) 172. (d) 173. (b) 174. (d) 175. (c)

* This question was missing in the original paper.

CLAT-6-Previous Years Papers.indd 379 06/09/2010 17:50:12


380 PREVIOUS YEARS’ PAPERS

PANJAB UNIVERSITY CET 2010


LEGAL AND GENERAL AWARENESS (c) UK—FTSE
(d) USA—Nasdaq
1. Which of the following is not a natural 11. Which one of the following countries is
harbour in India? not a member of ASEAN?
(a) Mumbai (b) Chennai (a) Myanmar
(c) Calcutta (d) Visakhapatnam (b) Bangladesh
2. Which of the following rivers is known as (c) Brunei Darussam
India’s River of Sorrow? (d) Vietnam
(a) Periyar (b) Hooghly 12. The study of clouds is
(c) Damodar (d) Kosi (a) Neology (b) Nefology
3. Which canal shortened the distance be- (c) Nematology (d) Numerology
tween India and Europe? 13. State with lowest sex ratio in India is
(a) Buckimgam Canal (a) Punjab (b) Haryana
(b) Suez Canal (c) UP (d) MP
(c) Indira Gandhi Canal 14. Who is considered as the Father of Clon-
(d) Panama Canal ing?
4. Who wrote the book “Waiting for Mahat- (a) Ian Mendel (b) Ian Wilmutt
ma”? (c) G. Fisher (d) Ian W. S. Foster
(a) Shashi Tharoor 15. Who was appointed as the first Chief In-
(b) R. K. Naraian formation Commissioner of India?
(c) Raj Mohan Gandhi (a) Najma Heptullah
(d) Vikram Seth (b) Tahir Mahmood
5. Which of the following is not a statutory (c) Irfan Habib
body? (d) Wajahat Habibullah
(a) Election Commission 16. Who has been recently awarded Officer de
(b) Finance Commission La Legion d’ Honor, the highest award of
(c) Planning Commission France?
(d) UPSC (a) Shahrukh Khan
6. Where is the headquarters of WTO locat- (b) Lata Mangeshkar
ed? (c) Amitabh Bachchan
(a) New York (b) Geneva (d) Aishwarya Rai
(c) Hague (d) New Delhi 17. The author of ‘Arthashastra’ was a con-
7. The United Nations Organization was temporary of
formed in the year (a) Ashoka
(a) 1940 (b) 1942 (b) Chandragupta Maurya
(c) 1945 (d) 1948 (c) Chandragupta Vikramaditya
8. Who was the first woman President of UN (d) Samudra Gupta
general Assembly ? 18. What is the name of the latest book by
(a) Sarojini Naidu President APJ Abdul Kalam?
(b) Annie Besant (a) Children of India
(c) Vijayalakshmi Pandit (b) Indomitable Spirit
(d) Subblakshmi (c) India in 2020
9. Which line divides India into two halves? (d) Ignited Minds
(a) Tropic of Cancer 19. Which of the following is not a constitu-
(b) Equator tional body?
(c) Tropic of Capricorn (a) Election Commission
(d) Great circle (b) Finance Commission
10. Which one of the following pairs is not (c) Inter-state Council
correctly matched? (d) National Advisory Council
(a) Japan—Nikkei 20. Fundamental Rights in the Indian Consti-
(b) Singapore—Shcomp tution have been taken from the

CLAT-6-Previous Years Papers.indd 380 06/09/2010 17:50:12


PREVIOUS YEARS’ PAPERS 381

(a) Russian Constitution 31. The innermost layer of the Earth is known
(b) U.S. Constitution as
(c) British Constitution (a) Lithosphere
(d) Act of 1935 (b) Mesosphere
21. When the offices of both the President and (c) Asthenosphere
Vice President of India are vacant, who (d) Barysphere
will discharge their functions? 32. Which of the following States has declared
(a) Prime Minister year 2007 as ‘Year of Farmers’?
(b) Home Minister (a) Punjab (b) Meghalaya
(c) Chief Justice of India (c) Haryana (d) Bihar
(d) The Speaker 33. Clove, the commonly used spice, is ob-
22. The National Development Council is pre- tained from the
sided over by the (a) Root (b) Stem
(a) Prime Minister of India (c) Flower bud (d) Fruit
(b) Deputy Chairman of the Planning 34. ‘A man can be destroyed, but not defeated’
Commission is the words of
(c) Finance Minister (a) Napoleon
(d) Union Minister of Planning (b) Ernest Hemingway
23. Who is legally competent to declare war (c) Alexander the Great
or conclude peace treaty? (d) Julius Caesar
(a) The Prime Minister 35. Famous Michael Schumacher, the World
(b) The Council of Ministers No. 1 in motor racing (Formula 1), be-
(c) The President longs to
(d) The Parliament (a) Italy (b) Spain
24. The term ‘Fourth Estate’ is used for (c) Germany (d) Brazil
(a) The Press and Newspapers 36. Standard Cricket Bats are made of
(b) Parliament (a) Pine wood (b) Rose wood
(c) Judiciary (c) Teak wood (d) Willow wood
(d) The Executive 37. ‘Come Out and Play’ is the anthem of
25. Who is the author of the book “My Life”? (a) Olympic Games 2008
(a) Bill Clinton (b) Tony Blair (b) Commonwealth Games 2010
(c) Dalai Lama (d) Sonia Gandhi (c) National Games 2008
26. The organization which got Nobel Prize (d) International Children’s Games 2009
for peace three times 38. ‘The Universe in a Single Atom’ is a re-
(a) UNO (b) ILO cently published book written by
(c) Red Cross (d) UNICEF (a) George H. W. Bush
27. Which one of the following is correctly (b) Dalai Lama
matched? (c) Hillary Clinton
(a) Eskimo : Canada (d) Vladimir Putin
(b) Oran : Japan 39. Who was the author of ‘Geet Govind’?
(c) Lapps : India (a) Vidyapati (b) Surdas
(d) Gonds : Africa (c) Jayadeva (d) Mirabai
28. Which one of the following is not cor- 40. The book ‘A Tale of Two Cities’ was writ-
rectly matched? ten by
(a) Fiji : Suva (a) John Lennon (b) Mark Twain
(b) Finland : Oslo (c) Shakespeare (d) Charles Dickens
(c) Guyana : George Town 41. ‘Life Divine’ is a book written by
(d) Lebanon : Beirut (a) Gandhiji (b) Tagore
29. When the Sun is nearest to the Earth, the (c) Radhakrishnan (d) Sri Aurobindo
Earth is said to be in? 42. The World Bank’s Headquarters is in
(a) Aphelion (b) Perihelion (a) Geneva (b) New York
(c) Apogee (d) Perigee (c) Paris (d) Washington D.C.
30. Which is the land of morning calm? 43. The Constitution of UNO is called
(a) Finland (b) Korea (a) Magna Carta (b) Peace Agreement
(c) Japan (d) Thailand (c) Charter (d) Declaration

CLAT-6-Previous Years Papers.indd 381 06/09/2010 17:50:12


382 PREVIOUS YEARS’ PAPERS

44. The Headquarters of UNESCO is in 55. First Space University is established at


(a) Germany (b) Paris (a) Thiruvananthapuram
(c) USA (d) Italy (b) Mysore
45. Which one of the following awards is giv- (c) Nasik
en for Agricultural Research? (d) Hyderabad
(a) Dr Homi Bhabha Award 56. Biological Death of a patient means death
(b) Borlaug Award of tissues of the
(c) Ramon Magsaysay Award (a) Kidney (b) Heart
(d) Shanti Swarup Bhatnagar Award (c) Lungs (d) Brain
46. The Chipko Movement is associated with 57. The world famous rock cut Kailasa Tem-
(a) Felling of trees ple at Ellora was built by the
(b) Water harvesting (a) Mauryas (b) Pallavas
(c) Save the Tiger (c) Chalukyas (d) Rashtrakutas
(d) Saving dams 58. The largest cell in the human body is
47. ‘Men and the City’ is the autobiography (a) Nerve cell (b) Muscle cell
of (c) Liver cell (d) Kidney cell
(a) Martin Luther King 59. The area of Union of India is nearest to
(b) Saddam Hussain (a) 39 lakh km2 (b) 33 lakh km2
(c) Julfikar Ali Bhutto (c) 30 lakh km2 (d) 36 lakh km2
(d) Liyaqat Ali Khan 60. Abhijit Sen Committee was constituted to
48. The first airline to allow flyers to surf the study
net was (a) Electoral reforms
(a) United Airlines (b) Future Trading
(c) Educational reforms
(b) Singapore Airlines
(d) Administrative reforms
(c) Emirates Airlines
61. MISA is named as
(d) Air Canada
(a) Maintenance of Internal Security Act
49. Which Planet is named after the Roman
(b) Maintenance of Industrial Security
god of agriculture?
Act
(a) Uranus (b) Mercury
(c) Maharashtra Insolvency Security Act
(c) Saturn (d) Jupiter
(d) Media Institute of Southern Africa
50. The motto of CNBC India is
62. Right to travel is a Fundamental right un-
(a) Profit from it der
(b) With you all the way (a) Article 19 of the Constitution
(c) Leading the way (b) Article 21 of the Constitution
(d) Makes life simple (c) Article 15 of the Constitution
51. Which one of the following is correctly (d) Article 14 of the Constitution
matched? 63. Family Courts Act was passed in
(a) Dr Kurian : Space Science (a) 1980 (b) 1984
(b) Dr Malcom : Cooperative Movement (c) 1986 (d) 1975
Adiseshiah 64. Member of State Public Service Commis-
(c) Dr Abdul Kalam : Development Eco- sion is removed from his office by
nomics (a) President
(d) Dr Pramod Karan Sethi : Jaipur Foot (b) Governor
52. Cosmic Noise is caused by (c) Parliament
(a) Lighting discharges (d) Legislative Assembly
(b) Solar eruptions 65. The Competition Commission replaced
(c) Distant stars (a) Consumer Commission
(d) Industrial discharges (b) NHRC
53. Heavy water is (c) MRTP Commission
(a) Tritium oxide (b) Deuterium oxide (d) Human Rights Commission
(c) Rain water (d) Water at –4°C 66. What is minimum age to get elected as a
54. Marsh gas is member of council of states?
(a) Hydrogen (b) Nitrogen (a) 25 years (b) 30 years
(c) Methane (d) Ethane (c) 32 years (d) 35 years

CLAT-6-Previous Years Papers.indd 382 06/09/2010 17:50:12


PREVIOUS YEARS’ PAPERS 383

67. Impeachment of President of India can be 78. How many Fundamental Duties are men-
initiated in tioned in Indian Constitution?
(a) Either house of Parliament (a) 10 (b) 11
(b) In a joint sitting of both houses of Par- (c) 9 (d) 6
liament 79. ‘A man is presumed to be innocent until he
(c) Lok Sabha is proved guilty’ it is a maxim of
(d) Rajya Sabha (a) Natural Law (b) Criminal Law
68. Which Article of Constitution makes men- (c) Common Law (d) Universal Law
tion of reservation in public employment
80. Writs are issued by
in favour of backward classes?
(a) Supreme Court
(a) 16 (1) (b) 16 (2)
(c) 16 (3) (d) 16 (4) (b) High Courts
69. Mens rea means (c) Supreme Court and High Courts
(a) Aggressive mind (d) Central Government
(b) Offensive mind 81. aku fqz ? Pzj ueo zjt :
(c) Guilty mind (a) Kue (b) Jtd
(d) Naughty mind (c) Jue (d) Kve
70. The Constitution of India contains 82. 1, 3, 7, 13, 21, ?
(a) 395 Articles and 8 Schedules (a) 25 (b) 27
(b) 395 Articles and 12 Schedules (c) 29 (d) 31
(c) 391 Articles and 14 Schedules 83. ? b a a b ? b ? a ? b b :
(d) 359 Articles and 10 Schedules (a) a b b b (b) b a b a
71. By which one of the following amend- (c) a a b b (d) b b b a
ments the Fundamental Right to property 84. ? : JKHI : : TURS : OPMN :
was removed from the Constitution (a) MNOP (b) LMNO
(a) first (b) twenty-fourth (c) OPMN (d) NOLM
(c) forty-second (d) forty-fourth
85. POND is written as FXRI, ROTE may be
72. Which one of the following is not a Nego-
written as
tiable Instrument?
(a) JKIZ (b) GHIZ
(a) Will (b) Promissory Note
(c) Cheque (d) Bill of Exchange (c) GHIJ (d) JIGZ
73. Under Muslim Law, marriage is regarded 86. Rita’s grades in four tests are 82, 80, 76
as a and 70. What grade must she get on the
(a) Sacrament (b) Contract fifth test to raise her average to 80?
(c) Social need (d) Tradition (a) 86 (b) 88
74. Human Rights Day is observed on (c) 92 (d) 96
(a) 20th December 87. X and Y are children of A. A is the father
(b) 4th July of X and Y is not the son of A. What is Y
(c) 8th August to A ?
(d) 10th December (a) Sister (b) Brother
75. Under which of the following Articles, (c) Daughter (d) Son
laws inconsistent or abridging Fundamen- 88. If a clinical thermometer reads 100°. What
tal Rights are declared void? will the centigrade thermometer read?
(a) Article 12 (b) Article 13 Tick the correct one.
(c) Article 14 (d) Article 21 (a) 40° (b) 37.7°
76. Which committee is responsible for Tax
(c) 110° (d) 20°
Reforms?
89. Five students are sitting in a row. T is on
(a) Jankiraman Committee
(b) Narasimhan Committee the right of Z. M is on the left of Z. But M
(c) Malhotra Committee is on the right of L. T is on the left of Q.
(d) Chelliah Committee Who is sitting first from left ?
77. Manu Smriti is related with (a) Z (b) T
(a) Economics (c) Q (d) L
(b) Political Science 90. Find the odd one
(c) Medical Jurisprudence (a) Teacher (b) Professor
(d) Law (c) Student (d) Head Master

CLAT-6-Previous Years Papers.indd 383 06/09/2010 17:50:12


384 PREVIOUS YEARS’ PAPERS

ENGLISH LANGUAGE dividual and socio-cultural 97 ............... that can


be further broken down into many other sub-fac-
91. Financial status is no ................... to edu-
tors including social, cognitive, behavioral, eco-
cation in a government school.
nomic, and political. Language is not politically
(a) hinderance (b) hindransse
innocent but is 98 .......... for certain given ends
(c) hindrance (d) hendrance
at the individual as well as national level. Many
92. Write ........... pencil and not ........... ink.
scholars have pointed out this 99 ...............l di-
(a) in, with (b) with, in
mension of language and have looked at its vari-
(c) in , in (d) with, with
ability from different 100 .................... .
93. He buys milk from the local .............. .
95. (a) some (b) multiple
(a) dairy (b) diary
(c) all (d) three
(c) dairi (d) deary
94. ................. killed the ............... . 96. (a) coming (b) reaching
(a) curiosity, rat (c) emerging (d) concluding
(b) curiousity, cat 97. (a) factors (b) trends
(c) curiosity, mouse (c) causes (d) reasons
(d) curiosity, cat 98. (a) utilized (b) analyzed
Read the passage and choose the correct words (c) learnt (d) copied
to fill in the numbered blanks : 99. (a) social (b) cultural
The concept of language can not be confined (c) elitist (d) political
to a single dimension because language has 95 100. (a) areas (b) views
................ interfaces 96.................. from both in- (c) perspectives (d) angles

ANSWERS
1. (b) 2. (d) 3. (b) 4. (b) 5. (c) 6. (b) 7. (c) 8. (c) 9. (a) 10. (b)
11. (b) 12. (b) 13. (b) 14. (b) 15. (d) 16. (b) 17. (b) 18. (b) 19. (d) 20. (b)
21. (c) 22. (a) 23. (c) 24. (a) 25. (a) 26. (c) 27. (a) 28. (b) 29. (b) 30. (b)
31. (d) 32. (b) 33. (c) 34. (b) 35. (c) 36. (d) 37. X 38. (b) 39. (c) 40. (d)
41. (d) 42. (d) 43. (c) 44. (b) 45. (b) 46. (a) 47. (b) 48. (b) 49. (c) 50. X
51. (d) 52. (c) 53. (b) 54. (c) 55. (a) 56. (d) 57. (d) 58. (a) 59. (b) 60. (b)
61. (a) 62. X 63. (b) 64. (a) 65. (c) 66. (b) 67. (a) 68. (d) 69. (c) 70. (b)
71. (d) 72. (a) 73. (b) 74. (d) 75. (b) 76. (d) 77. (d) 78. (b) 79. X 80. (c)
81. (a) 82. (d) 83. (d) 84. (c) 85. X 86. (c) 87. (c) 88. (b) 89. (d) 90. (c)
91. (c) 92. (b) 93. (a) 94. (d) 95. (b) 96. (c) 97. (a) 98. (a) 99. (d) 100. (c)
Note: An ‘X’ in the key indicates that either the question is ambiguous or it has printing mistake. All
candidates will be given credit for this question.

CLAT-6-Previous Years Papers.indd 384 06/09/2010 17:50:12


PREVIOUS YEARS’ PAPERS 385

SYMBIOSIS INTERNATIONAL UNIVERSITY 2010


LOGICAL REASONING Directions for questions 10–11: Choose the
correct option
1. If FISH is written as EHRG in a certain
10. Statements: All young scientists are open-
code, how would JUNGLE be written in
ing minded. No open-minded men are su-
that code?
perstitious.
(a) ITMFKD (b) ITNFKD
Conclusion:
(c) KVOHMF (d) TIMFKD
I. No scientist is superstitious.
2. The average of Ravi and his son’s age is
II. No young people are superstitious.
16 years. The ratio of their ages is 15 : 1
(a) Only conclusion I follow
respectively. What is the son’s age?
(b) Only conclusion II follows
(a) 30 years (b) 32 years
(c) 2 years (d) 4 years (c) Either conclusion II or I follow
3. P, Q, R, S, and T are sitting in a straight (d) Neither conclusion II nor I follow
line facing north. P sits next to S but not 11. In a class of 35 students, Kunal is placed
to T. Q is sitting next to R who is on the seventh from the bottom whereas Sonali is
extreme left corner. Who sits to the left of placed ninth from the top. Pulkit is placed
S if T does not sit next to Q? exactly in between the two. What is Ku-
(a) P (b) Q nal’s position from Pulkit?
(c) R (d) T (a) 9 (b) 10
4. Three of the following four are alike in a (c) 11 (d) 13
certain way and so form a group. Which Directions for questions 12-14:
one of the following does not belong to the Fifty books belonging to different subjects, viz.,
group? History (8), Geography (7), Literature (13), Psy-
(a) Tulip (b) Rose chology (8), and Science (14), are placed on a
(c) Jasmine (d) Lotus shelf. They are arranged in an alphabetical or-
5. The average of four consecutive even um- der subject to the condition that no two books of
bers is 27. The largest of these numbers is the same subject are placed together to so long
(a) 36 (b) 32 as books of other subjects are available. Unless
(c) 30 (d) 28 otherwise mentioned, all counting is done form
6. A and B know Punjabi, Sindhi and Guja- the left.
rati. B and C know Punjabi, Gujarati and 12. Which subjects does the 40th book belong
Bengali. A and E know Tamil, Sindhi, and to?
Gujarati. Who knows Punjabi, Tamil, Sin- (a) Science (b) Psychology
dhi and Gujarati? (c) History (d) Literature
(a) E (b) D 13. What is the position of the last book in
(c) A (d) C Psychology?
7. Which of the following alternatives repre- (a) 36th (b) 37th
sents a meaningful order of words given? (c) 38th (d) 39th
(1) Printing (2) Manuscript 14. Counting from the right to which subject
(3) Editing (4) Composing does the 39th book belong?
(5) Binding (a) History (b) Psychology
(a) 32415 (b) 24351 (c) Geography (d) Science
(c) 23145 (d) 23415 15. Mr A, Miss B, Mr C and Miss D are sitting
8. If a person is standing at the sixth number around a table and discussing their trades.
in the queue from both ends, then the total (1) Mr A sits opposite to cook
number of persons in the queue is (2) Miss B sits right to the barber
(a) 9 (b) 11 (3) The washer man is on the left of the
(c) 12 (d) 13 tailor
9. ‘FI’ related to ‘LO’ in the same way as (4) Miss D sits opposite Mr C
‘PS’ is related to What are the trades of A and B?
(a) VY (b) UZ (a) Tailor and barber
(c) WZ (d) UX (b) Tailor and Cook

CLAT-6-Previous Years Papers.indd 385 06/09/2010 17:50:12


386 PREVIOUS YEARS’ PAPERS

(c) Barber and Cook (c) Physics


(d) Washerman and Cook (d) Electronics
16. A waiter’s salary consists of his salary and 24. 4, 20, 35, 49, 62, 74, ?
tips. During one week his tips were 5/4th (a) 84 (b) 85
of his salary. What fraction of his income (c) 88 (d) 94
came from tips? Directions for questions 25-26:
(a) 4/9 (b) 5/4 There are two words and four alternative an-
(c) 5/8 (d) 5/9 swers. In each of the alternative answers there
Directions for questions 17-18: are two words. Find out the alternative whose
Given below are the two statements. One is As- two words have the same relation as in the two
sertion (A) and the other is Reason (R). Choose words given in the beginning.
the correct alternative from the following: 25. Mistake : Void
17. Assertion (A): The British Sovereignty (a) Misrepresentation : Voidable
continued to exits in free India. (b) Contract : Enforceable
Reason (R): The British Sovereign ap- (c) Counter offer : Offer
pointed the last Governor-General of free (d) Agreement : Voidable
India. 26. Law of Torts : Damages
(a) Both A and R is true and R is the cor- (a) Indian Penal Code : Punishment
rect explanation of A (b) Compensation : Law of Contracts
(b) Both A and R is true but R is not the (c) Civil Wrongs : Compensation
correct explanation of A (d) Constitutional Law : Fundamental
(c) A is true, but R is false Rights
(d) A is false, but R is true Direction for question 27-29: Choose the cor-
18. Assertion (A): Carbon monoxide when in- rect option
haled causes death. 27. Statement:
Reason (R): Carbon monoxide combines “Why don’t you go to the court if the em-
with haemoglobin. ployer does not pay you the pension and
(a) Both A and R is true and R is the cor- gratuity?”
rect explanation of A Assumptions:
(b) Both A and R true but R is not the cor- I. Courts can intervene in the maters of
rect explanation of A dispute between employers and em-
(c) A is true, but R is false ployees.
(d) A is false, but R is true II. It is obligatory for employer to pay
19. 11, 14, 12, 15, 13, 16 ? the pension and gratuity to the em-
(a) 14 (b) 15 ployees.
(c) 16 (d) 17 Which of the following can be validly in-
20. Which one is the same as Arid, Parched, ferred from the above statements?
and Droughty? (a) Both I and II
(a) Rock (b) Crow (b) Only assumption I
(c) Hay (d) Dry (c) Only assumption II
Directions for questions 21-23: (d) Neither assumption I nor II
For each of the following question four words 28. Statement:
are given, out of which three are same in certain The management of Bonn Co. Pvt. Ltd.,
way while one is different. Find out the differ- asked the workers union to call off strike
ent one. immediately otherwise the management
21. Find the odd one out will be forced to close the production as
(a) Saturn (b) Jupiter well as factory.
(c) Moon (d) Venus Assumption:
22. Find the odd one out I. No alternative other than closing down
(a) Carrot (b) Potato the production as well as factory is left
(c) Sweet Potato (d) Radish for the management of Bonn Co.
23. Find the odd one out II. By giving such threat, management
(a) Thermodynamics thinks that there would be some effect
(b) Optics on the workers union.

CLAT-6-Previous Years Papers.indd 386 06/09/2010 17:50:12


PREVIOUS YEARS’ PAPERS 387

Which of the following can be validly in- Factual Situation: Madhav and Anil had
ferred from the above statements? an argument during a meeting. After the
(a) I and II argument, Anil in order to humiliate Mad-
(b) Only assumption I hav in front of the assembled members,
(c) Only assumption II pulls out his chair when Madhav is about
(d) Neither assumption I nor II to sit. Though Madhav falls, he is not hurt.
29. Statement: He institutes a case against Anil for bat-
Beware of dogs. Our dogs do not bark but tery.
they are trained to distinguish between a (a) Anil is not liable as Madhav is not
genuine guest and the intruders. hurt
Assumptions: (b) Anil is not liable as it did not require
I. Barking dogs bite rarely. any force to pull the chair
II. Our dogs could be dangerous for in- (c) Anil is not liable as their argument
truders. justified his action
Which of the following can be validly in- (d) Anil is liable as his action was not jus-
ferred from the above statement? tified
(a) Only assumption I 33. Legal Principle: A minor is not competent
(b) Only assumption II to enter into a contract and the law will as-
(c) Either assumption I or assumption II sume that the contract never happened and
(d) Neither assumption I nor assumption everything is returned to as it was.
II
Factual Situation: A minor fraudulently
30. P, Q, R, S, and T are sitting in a straight
misrepresenting her age, enters into a con-
line facing north. P sits next to S but not
tract with another of the purchase of a pia-
to T. Q is sitting next to R who is on the
no. After the piano has been delivered she
extreme left corner. Who sits to the left of
refused to pay price for the same contend-
S if T does not sit next to Q?
ing the non existence of a legally binding
(a) P (b) Q
(c) R (d) T contract.
(a) The contract is valid and binding
LEGAL REASONING against the minor as the minor has
misrepresented her age knowingly.
31. According to article 20 of the Constitution
(b) The contract is voidable at the option
of India, ____.
of the other party and thus the minor
(a) No person shall be convicted of any
is liable to pay if the other party wants
offence except for violation of a law
in force at the time of the commission to carry out the obligation.
of the act charged as an offence nor (c) The contract is void and the court can
be subjected to a penalty greater than direct the minor to restore the piano to
that which might have been inflicted the other party.
under the law in force at the time of (d) The contract is valid and the guardian
the commission of the offence of the minor will be liable to pay for
(b) No person shall be prosecuted and the piano.
punished for the same offence more 34. Legal Principle: Theft occurs when a per-
than twice son dishonestly takes any movable prop-
(c) Any person may be compelled in any erty out of the possession of any person
criminal case to be a witness against without that person’s consent.
himself Factual Situation: Keshav finds Manu’s
(d) No person shall be compelled in any cattle roaming the streets and takes them
civil case to produce evidences home to prevent them from being run over
Directions for questions 32-34: by vehicles. Manu does not find his cattle
Given below are legal principles and a factual and makes a complaint to the police re-
situations. Apply the given legal principle to the porting them to be missing.
factual situation and select the most appropriate (a) Keshav is guilty of theft as he did not
answer to the question given below: inform Manu as the whereabouts of
32. Legal Principle: Whoever uses force his cattle.
without any lawful justification commits (b) Keshav is not guilty of theft since the
battery cattle were not in Manu’s possession.

CLAT-6-Previous Years Papers.indd 387 06/09/2010 17:50:12


388 PREVIOUS YEARS’ PAPERS

(c) Keshav is not guilty of theft since 41. Which one of the following features of
cattle are not property. the Constitution of India is borrowed from
(d) Keshav is not guilty of theft as there United Kingdom?
was no dishonest intention on his (a) Fundamental Rights
part. (b) Parliamentary type of the Govern-
35. Which one of the following writs can be ment
issued by the Supreme Court to order (c) Directive Principles of State Policy
Governmental Authority to perform its (d) Emergency Provisions
statutory duty? 42. A money Bill can be introduced in
(a) Quo Warranto (b) Prohibition ………
(c) Habeas Corpus (d) Mandamus (a) Lok Sabha only
36. Who among the following enunciated the (b) Rajya Sabha only
‘Doctrine of Rule of Law’? (c) Either Lok Sabha or Rajya Sabha
(a) Ronald Dworkin (d) Joint Session of both the Houses of
(b) Salmond Parliament
(c) A. V. Dicey Directions for question 43:
(d) John Austin 43. Read the following situation and choose
37. ‘Veto’ power in the United Nations is ex- the correct option given below.
ercised by …… Situation: Mr A has stolen Mr B’s bicycle,
(a) permanent members of the Security which he had parked at college common
Council parking. Mr B wants to sue Mr A, what
(b) non-permanent members of the Secu- would be the liability of Mr A?
rity Council (a) A is liable to B for the tort of conver-
(c) members of the general assembly
sion, only
(d) judges of the international courts of
(b) A is guilty of offence of theft, only
justice
(c) A could be tried for both, the tort of
38. Which among the following maxims liter-
conversion and offence of theft, con-
ally means that ‘welfare of the people is
currently
the paramount law’?
(d) A could be tired for both (Civil and
(a) Scienti violent non fit injuria
Criminal), alternatively
(b) Salus populi suprema lax
44. ‘A’ by letter, offers to sell his house to
(c) Sacramenta puberum sunt sarvansa
‘B’ for ` 2,00,000. Without knowing of
(d) Ubi jus ibi remedium
39. ‘A’ agrees to pay ` 5000 to ‘B’ if it rains, ‘A’s offer, ‘B’, by a letter, offers to buy
and ‘B’ promises to pay a like amount to the same house from ‘A’ for ` 2,00,000.
‘A’ if it does not rain, this agreement is Here
called (a) no binding contract comes into exis-
(a) Quasi contract tence as ‘B’s letter is merely a cross
(b) Contingent contract offer
(c) Wagering agreement (b) no binding contract comes into exis-
(d) Voidable contract tence as consideration in this contract
40. Preamble to the constitution of India reads is insufficient
as (c) binding contract comes into existence
(a) We, the members of the Constituent as ‘B’s letter is equivalent to accep-
Assembly adopt, enact and give our- tance of ‘A’s offer
selves this constitution (d) the Indian Contract Act is silent on
(b) We, the citizens of India adopt, enact such issues
and give to ourselves this Constitu- 45. The main object of Fundamental Rights is
tion to ensure ..…..
(c) We, the people of India in our Constit- (a) independence of Judiciary
uent Assembly adopt enact and give to (b) individual rights absolutely
ourselves this constitution (c) individual rights subject to reasonable
(d) We, the people of India adopt, enact restrictions
and give to India this Constitution. (d) a socialistic pattern of society

CLAT-6-Previous Years Papers.indd 388 06/09/2010 17:50:12


PREVIOUS YEARS’ PAPERS 389

Directions for questions 46-47: 52. How many regional languages are listed
Given below are the two statements. One is As- under the VIII Schedule of the Constitu-
sertion (A) and the other is Reason (R). Choose tion of India?
the correct alternative from the following. (a) 24 (b) 22
46. Assertion (A): A breach of contract gives (c) 18 (d) 15
rise to a right of the injured party to re- 53. Human Rights Day is observed on
cover the damages. _______.
Reason (R): The purpose of payment of (a) 4 June (b) 10 December
damages is to restore the position of both (c) 24 October (d) 6August
the parties in the same status in which they 54. _______ means ‘outside the power of
would have been, had they not entered into law’.
the contract. (a) Ultra valorem (b) Ultra vires
(a) Both A and R are true and R is the cor- (c) Ut supra (d) Ut infra
rect explanation of A 55. Wrongful restraint is __________ of the
(b) Both A and R are true and R is not the personal liberty of a man.
correct explanation of A (a) total restraint
(c) A is true but R is false (b) partial restraint
(d) A is false but R is true (c) voluntary restraint
47. Assertion (A): Every contract must be (d) involuntary restraint
supported with a consideration. 56. Where one party is in a position to domi-
Reason (R): Every contract without a nate the will of another and uses his su-
consideration is void. perior position to obtain the consent of
(a) Both A and R are true and R is the cor- a weaker party, the consent is said to be
rect explanation of A obtained by
(b) Both A and R are true and R is not the (a) undue influence
correct explanation of A (b) fraud
(c) A is true but R is false (c) coercion
(d) A is false but R is true (d) misrepresentation
48. The Parliament of India consists of 57. Finance Commission is a _______.
_______. (a) Statutory Authority
(a) House of People (Lok Sabha), Coun- (b) Non-Statutory Authority
cil of States (Rajya Sabha) and Presi- (c) Constitutional Authority
dent of India. (d) Judicial Authority
(b) House of People (Lok Sabha) and 58. A Hindu married woman, who has a Hin-
Council of States (Rajya Sabha) du husband living and marries a Muslim
(c) President of India and House of Peo- man, is guilty of the offence of _______.
ple (a) adultery (b) polygamy
(d) Only House of People (Lok Sabha) (c) mischief (d) bigamy
and various officials. 59. The new States can be created in the Union
49. President of India can make laws during of India by the _______.
recess of parliament through _______. (a) President of India
(a) States (b) Ordinances (b) Parliament by Law
(c) Bills (d) Notifications (c) Parliament by Amending the Consti-
50. Which one of the following rights is a tution
Constitutional Rights and not a Funda- (d) Inter-State Council
mental Right? 60. Tort is a species of _______.
(a) Right to freedom of expression (a) Criminal wrong
(b) Right to freedom of religion (b) Breach of contract
(c) Right to property (c) Civil wrong other than breach of con-
(d) Right to move freely tract
51. The concept of crime may be defined as, (d) Moral wrong
‘an act committed or omitted in _______
of public law.’
ANALYTICAL REASONING
(a) adherence (b) violation 61. Excessive amounts of lead in drinking
(c) respect (d) support water, associated with certain types of

CLAT-6-Previous Years Papers.indd 389 06/09/2010 17:50:12


390 PREVIOUS YEARS’ PAPERS

industrial pollution, have been shown to 63. The ideal candidate to occupy the office
cause lung cancer. Ajanta Island has an farthest from Mr Boman would be
economy based completely on subsistence (a) Miss Hema (b) Mr Mohan
level agriculture. There are no industries (c) Mr Tony (d) Mr Dhiraj
in the island. The inhabitants of Ajanta Is- 64. The three employees who are smokers
land have in the last 50 years shown high should be seated in the offices
incidence of lung cancer. (a) 1, 2 and 4 (b) 2, 3 and 6
Which of the following can be validly in- (c) 1, 2 and 3 (d) 1, 2 and 3
ferred from the above statements? 65. The ideal office for Mr Mohan would be
I. Lead in drinking water is absolutely (a) 2 (b) 6
safe. (c) 1 (d) 3
II. Lung cancer must have causes other 66. In the event of what occurs, within a pe-
than lead in drinking water. riod of one month since the assignment of
III. Lead in drinking water must have the offices, would a request for a change in
sources other than industrial pollu- office be put forth by one or more employ-
tion. ees?
(a) II only (b) III only (a) Mr Dhiraj quitting smoking
(c) II or III (d) II or III or both (b) Miss Rani’s needing silence in the
62. Meera: Mohan has become a better boxer office(s) next to her own
since he started meditating. (c) Mr Boman suffering from laryngitis
Sohan: Impossible, a boxer’s most impor- (d) Mr Tony taking over the duties for-
tant assets is his aggressiveness. merly taken care of by Miss Rani
Sohan’s statement implies that he believes Directions for questions 67-69:
A bus has exactly six stops on its route. The bus
that:
first stops at stop one and then at stop two, three,
(a) Meditation tends to make a person
four, five, and six respectively. After the bus
less aggressive
leaves stop six, the bus turns and returns to stop
(b) Meditation has little or no effect on
one and repeat the cycle. There are six stops in
the person who practices it
alphabetical order – L, M, N, O, P, and Q.
(c) Mohan was previously a poor box-
P is the third stop.
er because he was not aggressive
M is the sixth stop.
enough
The stop O is the stop immediately before
(d) Mohan has not really been meditating Q.
Directions for questions 63-66: N is the stop immediately before L.
An employee has been assigned the task of allot- 67. In case N is the fourth stop, which among
ting offices to six of the staff members. The of- the following must be the stop immedi-
fices are numbered 1-6. The offices are arranged ately before P?
in a row and they are separated from each other (a) O (b) Q
by six foot high dividers. Hence, voices, sounds (c) N (d) L
and cigarette smoke flow easily from one office 68. In case L is the second stop, which of the
to another. following must be the stop immediately
Miss Rani needs to use the telephone quite before M?
often throughout the day. Mr Mohan and Mr Bo- (a) N (b) L
man need adjacent offices as they need to consult (c) P (d) Q
each other often while working. Miss Hema, is a 69. In case a passenger gets on the bus at Q,
senior employee and has to be allotted the office rides past one of the stops, and gets off at
number 5, having the biggest window. P, which of the following must be true?
Mr Dhiraj requires silence in the offices (a) O is stop one
next to his. Mr Tony, Mr Mohan and Mr Dhiraj (b) Q is stop three
are all smokers. Miss Rani finds tobacco smoke (c) P is stop four
allergic and consecutively the offices next to (d) N is stop five
hers to be occupied by non-smokers. 70. If by arranging the letters of the word
Unless specifically stated all the employees ‘NESTIN’ the name of a game is formed,
maintain an atmosphere of silence during office then what are the first and the last letters of
hours. the word so formed?

CLAT-6-Previous Years Papers.indd 390 06/09/2010 17:50:12


PREVIOUS YEARS’ PAPERS 391

(a) T, S (b) T, N (a) ORNASAONTILAGA


(c) I, E (d) E, S (b) ARGONATISLONAA
71. If English alphabets are written in reverse (c) GRAONISTLOINAA
order what will be seventh letter to the (d) ROAGINASITNOLA
right of the 12th letter from the left? Directions for questions 78:
(a) H (b) F There are four alterative answers for the follow-
(c) G (d) S ing questions. Find out the correct answer.
72. If the letters of the words S, A, R, B, S 78. A, B, C, D, and E are five rivers. A is
are rearranged so that the name of a metal shorter than B but longer than E. C is the
is obtained. What will be the first letter of longest and D is a little shorter than B and
that word? a little longer than (a) Which is the short-
(a) B (b) A est river?
(c) S (d) R (a) B (b) C
Directions for questions 73-76: (c) D (d) E
Six students A, B, C, D, E, and F are to make a Directions for questions 79-81:
presentation each on a day. Three of them will Complete the series of the following:
make their presentations in the morning session 79. 14, 2, 12, 4, 10, 6, ?
before the lunch break whereas, the other three (a) 9 (b) 8
will be presenting it in the afternoon session. (c) 7 (d) 5
The presentations have to be scheduled in such 80. 40, 42, 39, 44, 38, 46, ?
a way that they comply with the following re- (a) 33 (b) 35
strictions: (c) 37 (d) 33
B should make his presentation immedi- 81. 9, 18, 21, 24, 27, ?
ately before C’s presentation. (a) 27 (b) 32
Their presentations cannot be separated by (c) 30 (d) 29
the lunch break. 82. Pointing at a photo, Rajesh said, “His
D must be either the first or the last student father is the only son of my mother. The
to make the presentation. photo belongs to whom?
73. In case C is to be the fifth student to pres- (a) Rajesh (b) Rajesh’s bother
ent, then B must be (c) Rajesh’s father (d) Rajesh’s Son
(a) first (b) second Directions for questions 83-85:
(c) third (d) fourth For each of the following questions there exists
74. B could be placed for any of the following a certain relation between the first two words.
places in the order of presenters except The same relation also exists in the third and
(a) first (b) second fourth word. Out of these four words, one word
(c) third (d) fifth is missing. The place of the missing word is
75. In case F is to make his presentation im- shown by question mark. Find out the missing
mediately after D makes his presentation, word from the five words given below in each
C could be scheduled for which of the fol- question.
lowing places in the order of presenters? 83. Parrot : Cage : : Man : ?
(a) first (b) second (a) Prison (b) Home
(c) third (d) fifth (c) Life (d) Body
76. In case F and E are the fifth and sixth pre- 84. Cardboard : Opaque : : Glass : ?
senters respectively then which of the fol- (a) Brittle (b) Transparent
lowing must be true? (c) Sparkle (d) Lustrous
(a) A is first in the order of presenters 85. Which of the following is same as uncle,
(b) A is third in the order of presenters aunt, and brother?
(c) A is fourth in the order of presenters (a) Child (b) Boy
(d) B is first in the order of presenters (c) Adult (d) Nephew
77. In the word ORGANISATIONAL, if the Directions for questions 86-87:
first and second, third and fourth, fifth and For each of the following questions, there are
sixth words are likewise interchanged up two words and four alternative answers. In each
to the last letter, what would be the out- of the alternative answers there are two words.
come? There candidate has to find out the alternative

CLAT-6-Previous Years Papers.indd 391 06/09/2010 17:50:12


392 PREVIOUS YEARS’ PAPERS

whose two words have the same relation as in Jayasheela’s father, was one of the examiners
the two words given in beginning. for law exams. Once, while he was correcting
86. Sapling : Tree paper, he asked Kishore to have a look at one
(a) Horse : Mare (b) Student : Teacher paper, and commented that it was extremely well
(c) Bud : Flower (d) Tree : Furniture written, well focused, accurate and in proper
87. Malaria : Mosquito order. He said he thought the student deserved
(a) Rat : Plague to be ranked number one and that he would time
(b) Epidemic : Bacteria him full marks for the paper. Since the answer
(c) Cholera : Contaminated water book did not bear the name of the examinee,
(d) Poison : Death Kishore, a curious youngster, memorized the
Directions for questions 88-89: seat number to check it up at a later date. When
In each of the following questions, three words the results were published, Kishore looked for
are given. They are followed by four words, one the name matching the seat number. He found
of which stands for the class to which these three the name Nani Palkhivala.
words belong. Identify that word. On completing his BA, Nani’s desire was
88. Wheat, Barley, Rice __________ to be a college lecturer. It went well with his
(a) Food (b) Kilo taste for literature, and his professors had given
(c) Agriculture (d) Cereals him all the encouragement he needed. He even
89. Honesty, Credibility, Reliability, _______ applied for a lecturer’s post, but another lady
(a) Dependability (b) Responsibility candidate was equally well-qualified and had
(c) Quality (d) Chastity the teaching experience too, which Nani did not
90. Light : Sun Heat : ? have. So he did not get selected.
(a) Electricity (b) Fire After his MA, Nani wanted to join the Indian
(c) Moon (d) Mars Civil Service (ICS). This was around 1942,
when the war was on and written examinations
READING COMPREHENSION were being held in Delhi. In the 1940s, the ICS
Directions for questions 91-100: Read the held a fascination for bright young students for
following passage and answer the questions whom it was the ultimate career. Though Nani
given below by selecting the most appropriate seldom spoke about it in his later years, the
alternative. apparently treated the Civil Service Examination
as something of a challenge. Nargesh, too,
Passage 1 (Nani A. Palkhiwala: A life—M. was very keen that he take his ICS papers.
V. Kamath) Accustomed to standing first in whatever studies
The years passed too quickly. In 1940 Nani he undertook, and winning prizes, medals and
finished his BA with honors in English, and in scholarships almost as a habit, Nani no doubt felt
1942 his MA with honors in the same subject. that he could top the list of successful candidates
In these exams he could not complete his papers evening the Civil Service Examination, if only
because the writer’s cramp that was to haunt him he had a chance to appear for it. Nani made two
for the rest of his life had begun to appear. For attempts, says Behram. The first time, he was
the first LLB examination he was given writer. disqualified in the preliminary physical test.
The person sent for the morning paper was Thereupon he regularly did a lot of exercise to
so incompetent that Nani refused to continue build up his body. Next time, he was declared
with him and came out of the examination hall physically fit, but an epidemic broke out in
exasperated. Nargesh immediately contacted the Delhi and, considering the anxiety of his dear
Vice-Chancellor of the University, who directed ones, he did not send his application form for the
the supervisor at the examination centre to allow written examination, for which a time limit had
Nargesh herself to substitute. Another writer been set. It did indeed seem that limit expired,
was sent for the afternoon paper and those to the venue for the examination was moved from
follow. He continued to be with Nani during Delhi to Bombay on account of the epidemic.
all the subsequent examinations—second LLB, The net result was that Nani could not
Advocate AS and Advocate OS. make it to the ICS. It did not bother Nani’s
An interesting anecdote is recounted by father, Ardeshir, who wanted his son to be a
Jayasheela Badami who was told about it by her lawyer. Ardeshir used to say: ‘Become a lawyer
cousin Kishore Nanavaty. Justice P.S. Badami, , my son, you are cut out for law’. The father

CLAT-6-Previous Years Papers.indd 392 06/09/2010 17:50:12


PREVIOUS YEARS’ PAPERS 393

saw that Nani had a sharp, analytical, incisive (a) once (b) twice
mind that put him a cut above ordinary mortals. (c) many (d) None of the above
He could see that a as a lawyer Nani would grow 96. This passage gives an account of Nani’s
in status and reputation. Nani was later to say: (a) experience of appearing for examina-
‘My father was of the view that I would not tions
enjoy being anybody’s servant—not even the (b) establishment of a career
Government’s—and he though that I would do (c) personality and character
well in the legal profession.’ The father knew (d) None of the above
his son’s mind better than the son himself. In 97. Ardeshir wanted his one to be a lawyer be-
the end what the Civil Service lost, the legal cause
profession gained. (a) Nani had failed in the ICS examina-
91. Nargesh had to write Nani’s remaining pa- tion
per because (b) he knew his some would be able to get
(a) for the first LLB examination Nani eminence as a lawyer
was given a writer. (c) the legal profession, according to him,
(b) Nani refused the rewriter’s help as per was better than the civil service
the instructions of the Vice-Chancel- (d) Nani was entirely comfortable with
lor the opportunities in the field of law
(c) the writer was not efficient enough to
98. In this passage, the phrase ‘writer’s cramp’
meet the requirements of Nani
means
(d) writer’s cramp started to appear in the
(a) A trophy given to somebody
exam
(b) A habit
92. Kishore got off the seat number by heart
(c) Sudden inability to write
because
(d) Inability to sit and speak
(a) Justice P. S. Badami commented that
99. Find a word in the passage which means—
it was an extremely well written pa-
per ‘greatly irritated’
(b) Kishore was interested to know who (a) Exasperated (b) Subsequent
the candidate was who deserved to be (c) Fascination (d) None of the above
ranked number one 100. Find a word in the passage which means—
(c) Jayasheela Badami was told about it ‘a sudden, widespread occurrence of
by Kishore Nanavaty something undesirable.’
(d) the answer book was well written, (a) Epidemic (b) Anecdote
well focused, accurate and in proper (c) Mortals (d) Incisive
order Directions for questions 101-110: Read the
93. Nani could not get into the field of teach- following passage and answer the questions
ing because given below by selecting the most appropriate
(a) Nani’s desire went well with his taste alternative.
for literature. Passage 2 (Lord Atkins–Geoffrey Lewis)
(b) his professors had given him all the
encouragement. TO THE EDITOR OF THE TIMES
(c) he had applied for the post of lecturer Sir, I venture to think that the division of
with another lady candidate. war criminals in Professor Goodhart’s letter
(d) he did not have any experience of published on December 22 is possibly fallacious.
teaching that the lady had. He refers to those who commit acts “in violation
94. Nani did not become a civil servant be- of existing State that those laws recognize
cause war as a justification for some acts, those that
(a) he always found it very challenging are not so justified by the laws of the invaded
(b) a time limit was set for the writing of State remaining criminal. I would not venture a
exams decided opinion upon this statement of the law;
(c) Nani appeared for the exam twice but I doubt whether it could be maintained.
(d) None of the above Invading armies come not under the law of
95. How many times did Nani give a physical the invaded country, but against it. They owe it
test? no allegiance and they receive no corresponding

CLAT-6-Previous Years Papers.indd 393 06/09/2010 17:50:13


394 PREVIOUS YEARS’ PAPERS

protection, tests which have been stated by legal should remain under the control of the Allied
authorities to have weight in such matters. Are Powers. There is a danger lest we approach
they subject to all the laws—police laws, for the subject in too legalistic mood. The crimes
instance or to the laws as to civil obligations? of which somehow the barbarian enemy have
Could a member of the invading forces be sued been guilty transcend all domestic laws. They
for trespass or conversion of goods or use and are offences against the conscience of civilized
occupation of land in a tribunal of the invaded humanity. What is desired is not revenge, but
country either if functioning during the invasion a vindication of civilities to be achieved by
or after the peace? I hesitate to affirm that either imposing retribution on the criminals so as to
British or American soldiers can be made liable ensure so far as possible that in no war in the
criminally or civilly by existing Fascist courts in future shall like horrors be perpetrated. I hold
Italy in respect of such acts as those courts may that the Allied Powers should name the black
deem not justified by war. crimes for which they demand punishment—the
But even if the proposition stated could wholesale murder of men, women, and children,
be supported, it is a matter for the grave whiter hostages or not; the deportation of whole
consideration of the Allied Powers whether war races of people; the carrying off into brothels of
criminals should be tried in accordance with it. captive women. The matter to be determined is
It involves that the punishment of these crimes whether particular persons accused are guilty of
depends upon the separate provisions of the penal those crimes.
laws of each invaded State. Procedure, evidence, This should be determined by tribunals,
an appeal introduce varying conditions as to courts—call them what you will—consisting of
delay and possibly conviction; while in some just person who, on the material before them,
countries, as I understand, capital punishment will say whether they ware honestly satisfied that
has been abolished. Moreover, if there is to be those persons are guilty or not. I would have no
a fixed venue, there may be real difficulties in technical rules of procedure or evidence, and the
administering justice for the crime may have extreme penalty should be death, to be mitigated
taken place in one country while the whiteness as the tribunal might decide. By all means try
may be, and often will be, in another, and there them if possible, as the Powers seem to have
will be no power of compelling attendance. The decided in Moscow, in the country of the crime;
accused may have a genuine plea of alibi or but the tribunals will have to be what the allied
mistaken identity dependent on the production decide, which does not exclude making national
of witnesses abroad. It must be possible in tribunals international ad hoc.
proper cases to change the venue. Another very Contrary to the Professor’s view, I think that
important factor in the trial of these crimes is the there is very little to be said for his suggesting
plea of superior orders. The effect to be given to that the British and Americans should concern
it may very it the existing laws of the countries themselves only with the trials of those charged
in question. It seems probable that it must be with having committed crimes against their
treated uniformly by a rule laid down by the own nationals. The conscience of the whole
Allied Powers. civilized world has been aroused by these
A further difficulty in the way of trying war barbarities, an surely we all concerned in seeing
criminals in accordance with the existing laws of that the criminals should be brought to justice.
the country in which the crimes are committed I do not believe that even a small minority of
is that it provides no remedy for crimes British or American people would wish to stand
committed in the country of the enemy, and aside at the trial of German, Japanese, or other
possibly lawful by the law of that country. We barbarians for crimes against, for instance, the
hear of terrible iniquities practiced in Germany, Jews in Europe. They would not if they could;
Bulgaria, and elsewhere; the crimes against but they could not if they would.
Jews in the various enemy countries would Your, & (c)
pass unpunished. The offences I mention do not ATKIN
appear to be those mentioned by the Professor 101. What is the gist of Lord Atkins’ letter?
in his first category, which seems to be confined (a) War criminals should be brought to
to those whose policy has brought on this war. justice
If I may express a personal opinion, it is that (b) War criminals should be treated with
the trial and punishment of these war criminal compassion

CLAT-6-Previous Years Papers.indd 394 06/09/2010 17:50:13


PREVIOUS YEARS’ PAPERS 395

(c) To contradict the law against war (a) increased in intensity


criminals (b) approved by higher authorities
(d) War criminals should be set free (c) disapproved by higher authorities
102. There may be difficulties in administering (d) decreased in intensity
justice considering that Directions for questions 111-120: Read the
(a) there may be no possibility of a fixed following passage and answer the question
venue given below by selecting the most appropriate
(b) witnesses are at great distances from alternative.
the accused
(c) international procedures are cumber- Passage 4 (Lord Denning: A biography—
some Edmund Heward)
(d) no power of compelling attendance A busy barrister has little time for writing.
exists with all parties Dennign’s first efforts were editing legal text-
103. Who is the prominent figure in the pas- books. In 1929, six years after his call to the
sage? Bar, he was one of the two assistant editors of
(a) Professor Goodhart Smith’s Leading Cases. His job was to rewrite
(b) Lord Acton and bring top to date the notes on certain
(c) Lord Atkin specified commercial cases. Editing a text-
(d) Mr Scrulton book is no easy matter. In one as old as ‘Smith’
104. How does Lord Atkin define ‘crime’? it was particularly difficult. The editor has to
(a) Violation of a state’s existing laws decide where the law has been changed since
(b) Violation of proposed state laws the last edition and a good deal of re-writing
(c) Transgressions is necessary. But editing a legal text-book does
(d) Acts of violence give the editor the opportunity of restating
105. The author wants a vindication of civiliza- the law. In 1935, it was good experience for a
tion so as common law barrister. He ales wrote six articles
(a) to ensure stoppage of all horrors on legal subjects for the Law Quarterly Review
(b) by foisting an act of just punishment between 1925 and 1949.
on criminals When at the Bar Denning made no speeches,
(c) to arouse the consciences of human save in court, nor did he give any lectures.
being Even after he become a judge it was for years
(d) All of the above before he gave a lecture. Then he was invited
106. Which are the enemy countries stated in to speak to law students at Queen’s University,
the passage? Belfast, it was only after he became a member
(a) Bulgaria and Germany of the Court of Appeal that he was in demand
(b) Germany and England as a lecturer. The first important public lecture
(c) England and Holland was given under the auspices of ‘Hamlyn Trust”
(d) Bulgaria and England established under the will of Miss Hamlyn of
107. Does Lord Atkin agree with Professor Torquay. She died in 1941 and came from an
Goodhart’s letter? old Devon family, her father having practiced as
(a) Yes a solicitor in Torque for many years. She gave
(b) No the residue of her estate to establish a lecture to
(c) To a certain extent enable the common people of England to realize
(d) To a large extent the privileges they enjoy in law in comparison
108. Find a word from the passage which means with other European peoples and to recognize
‘to be brave enough to say something that the obligations attaching thereto. This lecture
might be criticized’. was given at Senate House, London University,
(a) Endeavour (b) Sue in October and November 1949 and was entitled
(c) Venture (d) Hesitate ‘Freedom under the Law’. Denning sets the
109. The word ‘fallacious’ means stage by saying:
(a) not relevant (b) besides the point Whenever one of the King’s judges takes
(c) not the truth (d) without malice his seat, there is one application which by long
110. By the word ‘mitigated’ we mean a pen- tradition has priority over all others. Counsel
alty can be has but to say ‘My Lord, I have an application

CLAT-6-Previous Years Papers.indd 395 06/09/2010 17:50:13


396 PREVIOUS YEARS’ PAPERS

which concerns the liberty of the subject.’ and should be honest. He must be honest with his
forthwith the judge will put all matters aside and clients. He must be honest with his opponents.
hear it first. He must be honest with the court. Above all he
Denning raises a number of questions that must be honest with himself.’ Denning thought
form a constant theme in all his writing; personal that the administration of justice depended
freedom; freedom of mind and conscience; on the quality of the men who are ready to
justice between man the State; the power of the undertake it. He took very seriously the duty of
Executive. This lecture, published by the Trust, a judge to assist in legal education. These visits
ran into many impressions and proved far the to universities at home and abroad were part of
most popular of any of the Hamlyn lectures. his work for legal education – to teach the young
Lord Jowitt was Lord Chancellor at the time the basic principles. In the preface he says; ‘It is
and wrote reprimanding him, saying that judges to those who are about to enter the profession
should not write books. Denning said that he was that I would address these lectures’.
induced to do it because one of the government In 1959 there was a change of emphasis.
departments did not like what he had said. He was invited to deliver to ‘Romanes’ Lecture
After this lecture Denning found himself in Sheldonin Theatre, Oxford, on 21 May
much in demand at universities and his lectures 1959, and he chose as his title ‘From Precedent
and addresses to students were brought together to Precedent’. He used this lecture as an
and published in 1953 in a book entitled The opportunity to attack the too strict adherence
Changing Law. The reason given by Denning to precedent, especially in the House of Lords.
for the title was that many people think that He used examples from history to show that in
the law is certain and can only be changed by the past the house had not been bound by its
Parliament. He said: own precedents and urged that if the law was to
The truth is that the law is often uncertain developed and not to stagnate it must recapture
and it is continually being changed, or perhaps I the principle of growth. Denning was at the time
should say developed by the judges …… If the a law lord and knew what is was like in the House
common law is to retain its place as the greatest of Lords. It was the first step in his campaign to
system of law the world has ever seen, it cannot get English lawyers to treat precedent with less
stand still while everything else moves on. It reverence and to think in terms of principle. This
must develop too. It must adapt itself to new crusade went on throughout his judicial life and
conditions. was only partially successful.
The titles for these lectures were: ‘The When Denning was master of the Rolls
Spirit of British Constitution’; ‘The Rule of he was too busy to do any writing although he
Law in the Welfare State’; ‘The Changing Civil delivered great number of speeches. It was nearly
Law’; ‘The Rights of Women’; ‘The Influence twenty years before another book The Discipline
of Religions’. These subject were of general of Law published in 1979 by Butterworths.
concern at the time; the encroachment of the In the preface Denning says that his object
State on the rights of the individual, the rights is to impart in instruction in the principles of
of women and changes in the civil law. Young law, as they are, and as they should be. It is a
audiences heard his views on how the law review of the cases in which he has taken part,
should develop. indicating where changes in the law put forward
In 1954 Denning went as Nuffield visitor by him had been made, and where they had been
to the Universities of the Union of South Africa rejected. He was eighty years old when the book
under the auspices of the British Council to give was published and it was the summing up of his
his first overseas lecture. In 1955 he visited success and failures in the law in the thirty five
Canada and the United States of America. In years that he had been on the Bench. It was a
1955 these addresses were collected together book for lawyers about the law. Its theme was
and published under the title The Road to that the law laid down for the social conditions
Justice. The individual lecture were; ‘The in the nineteenth century needed moulding and
Road to Justice’, ‘The Just Judge’, ‘The Honest chanting to the needs of the twentieth century.
Lawyer’, ‘The Free Press’, ‘Eternal Vigilance’. Dennings eightieth birthday on 23 January 1979
The opening words of ‘The Honest Lawyer’ are was marked by family celebration and a reception
worthy of quotation: ‘If there is one thing more at Butterworths to marks the publication of
important than any other in a lawyer it is that he The Discipline of Law. The next day, between

CLAT-6-Previous Years Papers.indd 396 06/09/2010 17:50:13


PREVIOUS YEARS’ PAPERS 397

4.30 p.m. and 8 p.m., he signed four hundred 114. In “The Discipline of Law” (1979), Den-
copies of the book in Butterworths bookshop in ning says that it is important
Bell Yard. Many people waited for three hours (a) to impart instructions in principles as
in a queue which stretched two hundred yards they are
up Bell Yard and along Chancery Lane. (b) to show lawyers how to write books
After 1979 books by Denning came out (c) to impart instructions on how laws
each year. In 1980 The Due Process of Law was should not be
published, designed as a companion volume (d) to talk about how laws lad down in the
to The Discipline of law. This book was about 19th century needed to keep in mind
the practical working of the law and discussed needs of the 20th century
matters and the law of husband and wife. In the 115. When Denning compares himself to the
preface he wrote: ‘cobbler’s last’ he means that
In the choice of topics, I have tried to do— (a) a cobbler cannot live for long
what the cobbler should do—stick to his last— (b) the personalization of laws is impor-
to those topics which I have most experience. I tant
have chosen them also for their general interest. (c) laws cannot last longer than human
Not bookish law depends on the interpretation beings
of Statutes and Rules of Court. But the law in (d) what a lawyers wants to last is very
which persons count. important
In the late 1970s Denning had given some 116. Miss Hamlyn of Torquay bequeathed the
judgements which were not very popular with residue to her estate so that
trade unionists and when it was proposed that he (a) her family’s name would continue
should sign copies of the books at Blackwell’s (b) her interest from the bank was neces-
Bookshop in Oxford, posters appeared saying sary
‘Mass picket Denning’. On the advice of the (c) the English common man would be-
police and the proctors the event was cancelled. come aware of his privileges
In his epilogue to The Discipline of Law (d) her relatives would be impressed
Denning wrote: 117. One of the following was not a title of
It is something to have lived through this Denning’s lectures
century—the most dangerous century in the (a) The Influence of Religions on Law
(b) The Changing Civil Law
history of the English people …...
(c) The Rule of Law in the Welfare State
111. While editing a legal text-book, the editor
(d) The Reasons of Juvenile Delinquency
has to:
118. The word precedent means:
(a) rewrite the law
(a) an accident prevented from happen-
(b) decide where the law has been changed
ing
since the last edition
(b) an incident which has taken place in
(c) redraft certain complex legal sections
the past
in simple language
(c) an office before someone becomes
(d) interpret laws in new ways.
president
112. A demand for Lord Denning’s lectures
(d) an impressive position
grew when he
119. ‘The Discipline of Law’ was a book
(a) was a judge for 4 years (a) by lawyers how not to deal with law
(b) became a member of the Court of Ap- (b) about the due process of law
peal (c) which was a summary
(c) was at the Bar (d) which was a history of English law
(d) None of the above 120. The word ‘epilogue’ in any book comes
113. In 1959, the change of emphasis in ‘From (a) before the main book begins
Precedent to Precedent’ meant that Den- (b) in the middle of the book
ning wanted (c) at the end of the book
(a) legal traditions to be reinforced (d) nowhere in the book
(b) to crusade against judicial life
(c) laws to regain growth as a value GENERAL KNOWLEDGE
(d) to treat case histories with more re- 121. Which one among the following is a water
spect soluble vitamin?

CLAT-6-Previous Years Papers.indd 397 06/09/2010 17:50:13


398 PREVIOUS YEARS’ PAPERS

(a) Vitamin C (b) Vitamin C 131. ‘Let a hundred flowers bloom and let a
(c) Vitamin E (d) Vitamin K thousand schools of thought contend’ was
122. Which one of the following countries was said by
hit by the cyclone ‘Nargis’ recently, caus- (a) Lenin (b) Karl Marx
ing a great destruction and loss of life? (c) Tolstoy (d) Mao Tse-Tung
(a) Bangladesh (b) Myanmar 132. What is the full form of USB in case of
(c) Sri Lanka (d) Pakistan computers?
123. Who among the following was a contem- (a) Universal Security Block
porary of Chingiz Khan? (b) Universal Serial Bus
(a) Mahmud of Ghazni (c) Universal Software Barrier
(b) Iltutmish (d) Universal Stage Base
(c) Alauddin Khilji 133. Which of the following is not a Direct
(d) Mohammad-bin-Tughlaq tax?
124. In a dry cell battery, which of the follow- (a) Income tax (b) Wealth tax
ing are used as electrolytes? (c) Estate tax (d) Sales tax
(a) Ammonium chloride and Zinc chlo- 134. Sundra Strait separates
ride (a) Burma – Celebes
(b) Sodium chloride and Calcium chlo- (b) Java – Sumatra
ride (c) Sicily – Italy
(c) Magnesium Chloride and Zinc chlo- (d) Japan – Korea
ride 135. Which of the following Commissions of
(d) Ammonium chloride and calcium Inquiry probed into Indira Gandhi assas-
chloride sination?
125. ‘Vyas Samman’ is given for excellence in (a) Justice Shaha Commission of inquiry
the field of (b) Justice Thakkar Commission of in-
(a) Literature (b) Science quiry
(c) Dance (d) Economics (c) Justice Ranganath Mishra Commis-
126. The term ‘gene’ was coined by sion of inquiry
(a) Wilhelm Johannsen (d) Justice Sarkaria Commission
(b) G. Mendel 136. Which country has become the world’s
second biggest military spender after
(c) Louis Pasteur
United States of America?
(d) D. Vries
(a) China (b) Russia
127. Who was the President of India at the time
(c) India (d) France
of proclamation of Emergency in 1975?
137. Who among the following is known as
(a) V. V. Giri
‘Father of Ecology in India?
(b) Zakir Hussain
(a) Mr Ramesh Kaswalkar
(c) Fakhruddin Ali Ahmed
(b) Mr Rajendra Prasad
(d) Neelam Sanjeva Reddy
(c) Mr Ramdeo Mishra
128. Who was the first amongst the following
(d) Mr Radhakrishnan
women to receive the Jnanpith Award?
138. Which was the country that facilitated
(a) Mahashweta Devi
peace talks between the LTTE and Sri
(b) Mahadevi Verma Lankan Government in the year 2009?
(c) Amrita Pritam (a) USA (b) Norway
(d) Ashapurna Devi (c) India (d) Pakistan
129. ‘Mohiniattam’ is a dance form of: 139. The Province of Bengal was partitioned
(a) Kerala (b) Tamil Nadu into two parts in 1905 by
(c) Karnataka (d) Tanjavur (a) Lord Cornwallis
130. Who among the following had founded (b) Lord Dalhousie
the Theosophical Society in the United (c) Lord Warren Hasting
States of America? (d) Lord Curzon
(a) Madame Blavatsky 140. Which one of the following States has
(b) Madame Cama become the first State in India to offer
(c) Swami Dayanand Saraswati Government job to an HIV positive candi-
(d) Lala Hardayal dates?

CLAT-6-Previous Years Papers.indd 398 06/09/2010 17:50:13


PREVIOUS YEARS’ PAPERS 399

(a) Maharashtra (c) Lata Mangeshkar


(b) Uttar Pradesh (d) Sir M. Visvesvaraya
(c) Chhattisgarh 146. What is the approximate mean velocity
(d) Kerala with which the Earth moves around the
141. Which party was founded by Subhash Sun in its orbit?
Chandra Bose in 1939 after his break from (a) 15 km/s (b) 30 km/s
the Congress? (c) 50 km/s (d) 75 km/s
(a) Abhinav Bharat 147. Bhilai Steel Plant was built with the col-
(b) Forward Bloc laboration of
(c) Swaraj Party (a) Germany
(d) Congress (I) (b) Soviet Union
142. The three medals that Indians won in the (c) United Kingdom
Beijing Olympics were in (d) Japan
(a) Shooting, Boxing and Wrestling
148. The Home Rule League was started by
(b) Shooting, Boxing and Archery
(a) Maulana Azad
(c) Shooting, Wrestling and Tennis
(b) Nana Deshmukh
(d) Shooting, Boxing and Hockey
(c) B. G. Tilak
143. Which country has the largest rail network
(d) Mahatma Gandhi
in the world?
(a) India (b) U.K. 149. Who set music to Mohammad Iqbal’s
(c) China (d) USA ‘Sare Jahan Se Achcha’?
144. From which country did Kosovo declare (a) Pandit Ravi Shankar
its independence? (b) Hariharan Prasad Chaurasi and Shiv
(a) Bulgaria (b) Croatia Kumar Sharma
(c) Macedonia (d) Serbia (c) Shankar Jai Kishan
145. Who is the only India to be conferred with (d) C. Ramachandra
a Bharat Ratna Award and a Knighthood? 150. Where is the city of ‘Yangon’?
(a) Dr B. R. Ambedkar (a) Thailand (b) Myanmar
(b) Maulana Azad (c) Ethiopia (d) Estonia

ANSWERS
1. (a) 2. (c) 3. (a) 4. (c) 5. (a) 6. (c) 7. (b) 8. (b) 9. (a) 10. (d)
11. (b) 12. (a) 13. (a) 14. (a) 15. (d) 16. (d) 17. (b) 18. (a) 19. (a) 20. (d)
21. (c) 22. (b) 23. (c) 24. (b) 25. (c) 26. (d) 27. (a) 28. (c) 29. (b) 30. (a)
31. (a) 32. (d) 33. (b) 34. (a) 35. (d) 36. (a) 37. (a) 38. (b) 39. (b) 40. (c)
41. (b) 42. (a) 43. (b) 44. (a) 45. (c) 46. (a) 47. (a) 48. (a) 49. (b) 50. (c)
51. (b) 52. (b) 53. (b) 54. (b) 55. (a) 56. (a) 57. (c) 58. (d) 59. (b) 60. (c)
61. (a) 62. (a) 63. (d) 64. (d) 65. (d) 66. (d) 67. (b) 68. (d) 69. (c) 70. (a)
71. (b) 72. (a) 73. (d) 74. (c) 75. (d) 76. (c) 77. (d) 78. (d) 79. (b) 80. (c)
81. (c) 82. (d) 83. (b) 84. (b) 85. (d) 86. (c) 87. (c) 88. (d) 89. (a) 90. (d)
91. (c) 92. (d) 93. (d) 94. (d) 95. (a) 96. (b) 97. (b) 98. (d) 99. (a) 100. (a)
101. (a) 102 (d) 103 (a) 104 (a) 105. (d) 106. (b) 107. (a) 108. (c) 109. (c) 110. (d)
111. (b) 112 (b) 113 (c) 114 (d) 115. (d) 116. (c) 117. (d) 118. (b) 119. (b) 120. (c)
121. (d) 122 (c) 123 (b) 124 (a) 125. (c) 126. (b) 127. (d) 128. (a) 129. (a) 130. (a)
131. (c) 132 (a) 133 (d) 134 (b) 135. (a) 136. (b) 137. (d) 138. (c) 139. (d) 140. (b)
141. (b) 142 (a) 143 (d) 144 (d) 145. (c) 146. (b) 147. (b) 148. (c) 149. (c) 150. (b)

CLAT-6-Previous Years Papers.indd 399 06/09/2010 17:50:13


400 PREVIOUS YEARS’ PAPERS

CLAT 2010
ENGLISH make an example of the brother who had fought
against him, Polynices, by refusing he right of
Directions (Q. 1 to 3): Substitute the bold honourable burial. The penalty of death was
phrases with any of the given choices to express promulgated against any who should defy this
the opposite meaning in the sentences in follow- order.
ing questions: Antigone was distraught. Polynices had
1. She always praises everything I say been left unburied, unwept, a feast of flesh for
(a) picks holes in (b) dislikes keen eyed carrion birds. Antigone asks her sister
(c) rebukes (d) picks holes to Ismene, for it was a challenge to her royal blood.
2. He often says how wonderful his school “Now it is time to show whether or not you are
is? worthy of your royal blood. Is he not my brother
(a) says he is unworthy and yours? Whether you like it or not? I shall
(b) appreciates never desert him never!” But Ismene responds,
(c) runs up “How could you dare-when Creon has express-
(d) runs down ly forbidden it? Antigone, we are women, it is
3. She said I was the best boss they’d ever not for us to fight against men.” With a touch
had. It was obvious she was praising me of bitterness, Antigone releases her sister from
sincerely. the obligation to help her, but argues she cannot
(a) not appreciating me shrug off the burden. “If I die for it what happi-
(b) befooling me ness Live, if you will live, and defy the holiest
(c) buttering me up of laws of heaven.”
(d) disliking me 7. What is the main theme of the story of An-
Directions (Q. 4 to 6): Identify the part of speech tigone?
of the Bold words in the given sentences from (a) One must be truthful and honest
the following questions: (b) There is a conflict between the laws of
4. I must perfect the operation to make the men and heavenly laws
perfect robot. (c) One must be true to one’s kins
(a) verb (b) adverb (d) War is an evil
(c) adjective (d) noun 8. Why did Antigone decide to defy the or-
5. A kindly person is one who behaves kind- ders of Creon?
ly. (a) She loved her brother
(a) noun (b) preposition (b) She was to give an honorable burial to
(c) adverb (d) verb her brother
6. He is not normally a very fast runner, but (c) She felt she was bound by her heav-
he runs fast in major events. enly obligation
(a) adverb (b) adjective (d) To teach Creon a lesson
(c) verb (d) noun 9. What, in your opinion, would have been
Directions (Q. 7 to 16): Read the following pas- the logical end of the story?
sage carefully and answer the following ques- (a) Antigone might have agreed with
tion: her sister and refrained from giving a
Antigone was one of the daughters of Oedipus, burial to Polynices
that tragic figure of male power who had been (b) Antigone might have been allowed by
cursed by Gods for mistakenly killing his father Creon to give a decent burial to her
and subsequently marrying his mother and as- brother
suming the throne of Thebes. After the death (c) Antigone might have defied the order
of Oedipus civil war broke out and a battle was of Creon but forgiven by him
waged in front of the seventh gate of Thebes-his (d) Antigone might have been executed
two sons led opposing factions and at the height for defying the order of the king
of the battle fought and killed each other. Oedi- 10. What was the status of women in the con-
pus’ brother, Creon, uncle of Antigone, was now temporary society? They
undisputed master of the city. Creon resolved to (a) were liberated

CLAT-6-Previous Years Papers.indd 400 06/09/2010 17:50:13


PREVIOUS YEARS’ PAPERS 401

(b) could have taken their own decisions 17. I have hit upon a good plan to get rid of
(c) considered themselves inferior and him.
subordinate to men (a) found
(d) claimed equality with men (b) chanced upon
11. Why did a civil war break out in Thebes? (c) decided to beat him
The war broke out because (d) borrowed
(a) of the curse of the Gods 18. He is sticking out for better terms
(b) the brothers of Antigone were greedy (a) threatens to take action
(c) there was a fight among sons of Oe- (b) insists on using the force
dipus for the inheritance of the king- (c) decides to give concessions
dom (d) persists in demanding
(d) there was a conflict between a son of 19. He broke off in the middle of the story
Oedipus and Creon (a) failed
12. A carrion bird is a bird (b) began crying
(a) of prey (c) stopped suddenly
(b) which eats human flesh (d) felt uneasy
(c) which eats dead bodies 20. He refused to be led by the nose.
(d) which eats only grain (a) to follow like an animal
13. Why did Creon deny decent burial to (b) to be treated as a fool
Polynices? He did so because (c) to follow submissively
(a) he did not love Polynices (d) to be treated violently
(b) Polynices fought against Creon 21. The new cotton mill is mortgaged up to
(c) Polynices was disobedient to Creon the eye.
(d) Polynices did not show bravery (a) apparently (b) completely
14. Why did Ismene not support Antigone? (c) deceptively (d) actually
Ismene 22. When they embraced a new religion, it
(a) was weak and did not have the cour- is safe to say they did it for loaves and
age to defy orders of the powerful fishes.
king (a) selflessly
(b) did not consider it right to defy the (b) honest reasons
(c) material benefits
king
(d) because of fear
(c) did not think it fit to defy her uncle
Directions (Q. 23 to 27): Choose the correct
especially after the death of her father
spelling out of four choices in following ques-
(d) did not believe that Polynices de-
tions:
served better treatment
23. (a) Misogynist (b) Mysogynists
15. Why did the Gods curse Oedipus? Be-
(c) Mysoginists (d) Mysagynists
cause Oedipus
24. (a) Aracnophobia
(a) killed his father and married his moth-
(b) Aranchophobia
er
(c) Arachnophobia
(b) killed his father
(d) Arachnophobia
(c) married his mother
25. (a) Cinamon (b) Cinnamon
(d) committed an unknown sin (c) Cinnaman (d) Cinaman
16. Does the story approve the principle of vi- 26. (a) Alcohol (b) Alchohol
carious liability? If so how? (c) Alchohal (d) Alchohel
(a) No, it does not 27. (a) Bioclymatalogy
(b) Yes, it does, because of the acts of Oe- (b) Bioclimatalogy
dipus his children suffered (c) Bioclimatology
(c) Yes, it does, because his father was (d) Bioclimatelogy
killed by Oedipus Directions (Q. 28 to 32): Select the correct
(d) Yes, it does, because he married his meanings of the given words in the following
mother questions:
Directions (Q. 17 to 22): Select the meaning of 28. Lexicon
the Bold idioms and phrases in sentences in fol- (a) number (b) legal document
lowing questions: (c) dictionary (d) captain’s dog

CLAT-6-Previous Years Papers.indd 401 06/09/2010 17:50:13


402 PREVIOUS YEARS’ PAPERS

29. Hex (iii) and no law containing a declaration


(a) crude person (b) herb that it is for giving effect to such pol-
(c) parrot (d) evil spell icy shall be called in question in any
30. Seminary court on the ground that it does not
(a) chapel give effect to such policy
(b) college (iv) shall be deemed to be void on the
(c) convocation hall ground that it is inconsistent with
(d) hostel or takes away or abridges any of the
31. Liturgy rights conferred by Article 14 to 19
(a) prayer (b) priest (a) (ii), (i), (iii), (iv)
(c) ritual (d) church (b) (iv), (i), (ii), (iii)
32. Laity (c) (ii), (i), (iv), (iii)
(a) church members not baptized (d) (i), (ii), (iii), (iv)
(b) church members baptized 40. (i) neither House shall proceed further
(c) priests with the Bill
(d) church members who are not ordained (ii) if he does so, the houses shall meet ac-
priests cordingly
Directions (Q. 33 to 37): Fill in the blanks in the (iii) but the President may at any time after
following questions: the date of his notification, summon
33. Slavery was not done away ………. until the Houses to meet in a joint sitting
the last century. for the purpose specified in the notifi-
(a) with (b) for cation and
(c) to (d) off (iv) where the President has under clause
34. Does he not take ………. his father? (1) notified his intention of summon-
(a) before (b) for ing the Houses to meet in a joint sit-
(c) after (d) like ting
35. We will have to take ………. more staff if (a) (iv), (i), (iii), (ii)
we’re to take on more work. (b) (iv), (i), (ii), (iii)
(a) up (b) onto (c) (iv), (ii), (iii), (i)
(c) into (d) on (d) (i), (ii), (iii), (iv)
36. Mother takes everything in her ……….
(a) steps (b) face GENERAL KNOWLEDGE
(c) stride (d) work
37. Sale have really taken ………. now. 41. Lumbini is the place where the Buddha
(a) up (b) on (a) attained nirvana
(c) of (d) off (b) attained enlightenment
Directions (Q. 38 to 40): The constituent phras- (c) was born
es of a sentence are jumbled up in the following (d) was married
questions. Select the most appropriate sequence 42. Galvanometer is an instrument to mea-
to make the sentence meaningful: sure
38. (i) built on the site of a church destroyed (a) relative density of liquids
(ii) in the hilly area of the city is the fa- (b) electric currents
mous Shandon Steeple (c) pressure of gases
(iii) the bell tower of St. Anne’s Church (d) distances
(iv) when the city was besieged by the 43. Who wrote ‘Mudra Rakshasa’?
Duke of Marlborough (a) Vishakhadatta (b) Bana Bhatta
(a) (ii), (iii), (i), (iv) (c) Kalidasa (d) Jaya Dev
(b) (ii), (i), (iii), (iv) 44. The venue of Asian Games in 1970 was
(c) (iv), (iii), (i), (ii) (a) Beijing (b) Jakarta
(d) (iii), (ii), (i), (iv) (c) Bangkok (d) New Delhi
39. (i) no law giving effect to the policy of 45. 1929 is known for
the state towards securing all or any (a) visit of Simon Commission to India
of the principles laid in part IV (b) congress resolution for complete in-
(ii) not withstanding anything contained dependence of India
in Article 13 (c) coming of Indian National Congress

CLAT-6-Previous Years Papers.indd 402 06/09/2010 17:50:13


PREVIOUS YEARS’ PAPERS 403

under the leadership of Mahatma 56. Alberto Fujimori is the former president
Gandhi of
(d) Burma was made a part of India (a) Japan (b) Peru
46. I.N.S. Airavat is India’s (c) South Korea (d) Vietnam
(a) amphibious ship 57. Which of the following is the national
(b) submarine river of India?
(c) destroyer (a) Brahmaputra (b) Narmada
(d) frigate (c) Ganga (d) Kaveri
47. Astra-Missile is 58. The highest number of telephone users are
(a) surface to surface missile in
(b) surface to air missile (a) China (b) USA
(c) air to surface missile (c) Canada (d) India
(d) air to air missile 59. The president of Maldives is
48. Nehru Institute of Mountaineering is situ- (a) Maumoom Abdul Gayoom
ated at (b) Mohammad Ashraf
(a) Nainital (b) Darjeeling (c) Mahmood Alam
(c) Shimla (d) Uttarkashi (d) Mohammed Nasheed
49. The chief guest on the 60th Republic day 60. Arabinda Rajkhowa is a member of
of India was the President of (a) B.J.P.
(a) Uzbekistan (b) Tajikistan (b) Assam Gana Parishad
(c) Kazakastan (d) South-Africa (c) Maoist Centre
50. Which of the following was not the base (d) Ulfa
of L.T.T.E. before being captured by Sri- 61. The breakthrough in the ideas of Darwin
lankan Forces? on the evolution of species came after his
(a) Elephant Pass (b) Kilinochchi visit to a cluster of islands and where he
(c) Mullaithivu (d) Murid ke saw that each island supported its own
51. Who was given Col C. K. Nayudu Life form of finch. Name the islands
Time Achievement Award for 2007-08 for (a) Iceland (b) Greenland
exemplary contribution to cricket? (c) Galapagos (d) Christian Islands
(a) Kapil Dev 62. Mahatma Gandhi never became a Nobel
(b) Gundappa Vishwanath Laureate, but he was nominated five times.
(c) Sunil Gavaskar In which of the following years he was not
(d) Vijay Hazare nominated?
52. Who is the foreign minister of European (a) 1937 (b) 1939
Union? (c) 1948 (d) 1940
(a) Baroness Ashlon 63. Which of the following countries has not,
(b) Massimo D’ Alema till the end of 2009, decoded the entire ge-
(c) David Miliband nome of a human being?
(d) Carl Bildt (a) India (b) Russia
53. Who is the author of ‘Godan’? (c) China (d) Canada
(a) Bhishma Sahani 64. When was the University of Bombay (now
(b) Premchand is Mumbai) established?
(c) Manohar Shyam Joshi (a) 1861 (b) 1857
(d) Sharad Chandra Chattopadhyay (c) 1909 (d) 1890
54. Which political party does Raj Babbar be- 65. Harare is the capital of
long to? (a) Zambia (b) Zaire Republic
(a) Samajwadi (c) Zimbabwe (d) Yemen
(b) Bharatiya Janata Party 66. Before the Indian team left for Conference
(c) Congress on Climate Change at Copenhagen, Jairam
(d) Bahujan Samaj Party Ramesh announced that India would work
55. In which year Bastille fell on 14th of for voluntary reduction of
July? (a) 40 to 45 per cent
(a) 1879 (b) 1789 (b) 20 to 25 per cent
(c) 1787 (d) None of them (c) 30 to 35 per cent

CLAT-6-Previous Years Papers.indd 403 06/09/2010 17:50:13


404 PREVIOUS YEARS’ PAPERS

(d) 10 to 15 per cent (a) Italy (b) Latvia


in energy intensity in 2020 compared to (c) USA (d) Sweden
2005 79. Which of the following is the national an-
67. Which is the largest island in the world (if them of India?
Australia is not considered an island)? (a) Vande Mataram ……….
(a) Iceland (b) Borneo (b) Jana gana mana adhinayak ……….
(c) Sumatra (d) Greenland (c) Ye mera chaman, ye mera chaman
68. Indo-Pak Summit between Parvez Mush- ……….
arraf and Atal Bihari Bajpayee was held in (d) Sare jahan se achchha ……….
2001 at 80. Which country was known as sick man of
(a) Delhi (b) Shimla Europe?
(c) Agra (d) Mumbai (a) Greece (b) Latvia
69. In May 2009, in the final of Sultan Azlan (c) Turkey (d) Austria
Shah Championship; Indian men’s Hock- 81. In Malaysia, the word ‘bhumiputra’ refers
ey team was defeated by to
(a) Pakistan (b) Argentina (a) Malayas (b) Chinese
(c) New Zealand (d) Canada (c) Indians (d) Buddhists
70. Which of the following is not a union ter- 82. What was the real name of Munshi Prem-
ritory? chand?
(a) Tripura (b) Daman and Diu (a) Nabab Rai
(c) Lakshadweep (d) Puducherry (b) Dhanpat Rai
71. Who was the founder editor of ‘Kesari’? (c) Ram Chandra Srivastava
(a) Lala Lajpat Rai (d) Hari Shankar
(b) Bal Gangadhar Tilak 83. Who is the author of ‘Old Man and the
(c) Gopal Krishna Gokhle Sea’?
(d) Dadabhai Naoroji (a) John Ruskin
72. With whose permission did the English set (b) Raja Rao
up their first factory at Surat? (c) Gunter Grass
(a) Akbar (b) Shahjahan (d) Ernest Hemingway
(c) Jahangir (d) Aurangzeb 84. Prophet Mohammed was born in
73. The group of nations known as G-8 started (a) 570 A.D. (b) 720 A.D.
as G-7. Which among the following was
(c) 620 A.D. (d) 510 A.D.
not one of them?
85. When was the First World War declared?
(a) Canada (b) Italy
(a) 1914 (b) 1915
(c) Russia (d) Japan
(c) 1918 (d) 1913
74. Emperor Akbar the Great, died in the year
86. Deodhar Trophy is given for the me of
(a) 1505 (b) 1605
(a) Cricket (b) Football
(c) 1606 (d) 1590
(c) Hockey (d) Golf
75. What was the name of Emperor Shahjahan
87. Pariyar Wild-Life sanctuary is situated in
before he became Emperor?
the state of
(a) Kusrau (b) Khurram
(a) Tamil Nadu (b) Kamataka
(c) Parvez (d) Shaheryar
(c) Kerala (d) Andhra Pradesh
76. Which of the following trees has medici-
nal value? 88. Nepanagar in Madhya Pradesh is known
(a) Pine (b) Teak for
(c) Oak (d) Neem (a) steel mills
77. Identify the main principle on which the (b) sugar mills
Parliamentary system operates? (c) potteries
(a) Responsibility of executive to legisla- (d) news print factory
ture 89. Who destroyed the Somnath temple in
(b) Supremacy of democracy Gujarat?
(c) Rule of law (a) Mohammed Ghouri
(d) Supremacy of the Constitution (b) Mahmud Ghaznavi
78. Great tennis player Bjorn Borg belongs to (c) Changhez Khan
which country? (d) Taimur Lang

CLAT-6-Previous Years Papers.indd 404 06/09/2010 17:50:13


PREVIOUS YEARS’ PAPERS 405

90. In Cape Traflagar, the famous title of Tra- when framed did not provide for judicial
falgar was fought in 1805. Where is Cape review?
Trafalgar situated? (a) Indian (b) Pakistani
(a) Italy (b) Greece (c) USA (d) Australian
(c) Spain (d) Portugal 100. Ratio decidendi means
(a) a judicial decision
LEGAL APTITUDE (b) part of the judgement which possesses
91. The manager of waqf is known as authority
(a) Sajjadanashin (b) Khadim (c) any observation made by the court
(c) Mutawalli (d) Mujawar which goes beyond the requirement
92. ‘Ipso facto’ means of the case
(a) in place of (d) an observation made by a judge
(b) by reason of that fact 101. ‘Dyarchy’ under the government of India
(c) by the same source Act 1919 meant
(d) by the way (a) division of powers between the cen-
93. ‘Requisition’ means tral and provincial government
(a) permanent transfer of the e of the (b) separation of judiciary from execu-
property tive
(b) supervision of property (c) division of executive departments un-
(c) taking control of property temporar- der elected ministers and the members
ily of the governor’s executive council
(d) taking possession permanently (d) separation between legislature and ex-
94. ‘Corroborative evidence’ means ecutive
(a) main evidence in a case 102. Fringe benefit tax is a tax
(b) evidence which supports other evi- (a) paid by an employer in respect of the
dence fringe benefits provided or deemed to
(c) evidence that proves the guilt of an have been provided by an employer to
accused person his employee
(d) evidence of a person who supports the (b) paid by an employer for the benefits
accused which he enjoys
95. Ex-parte decision means a decision given (c) paid by a person for the benefits which
(a) after hearing both the parties he gets from his employer
(b) without proper procedure (d) paid by a member of scheduled castes
(c) after observing proper procedure and scheduled tribes for benefits they
(d) without hearing the opponent receive from the government
96. Which of the following Constitutions is a 103. Which of the following is not true about a
Unitary Constitution? criminal proceeding?
(a) USA (b) British (a) the court may ask to pay a fine
(c) Indian (d) Australian (b) the court may order the transfer of the
97. Which of the following is not a fundamen- ownership of the property
tal right in India? (c) there is prosecution
(a) right to form association (d) the court may discharge an accused
(b) freedom of religion 104. In Ram vs Shyam, Ram cannot be a
(c) right to property (a) plaintiff (b) appellants
(d) right to move throughout the territory (c) defendant (d) prosecutor
of India 105. Cr.P.C. stands for
98. Which of the following marriages is ap- (a) Criminal Proceedings Code
proved by Islamic law? Between a Mus- (b) Criminal Proceedings Court
lim (c) Crime Prevention Code
(a) male and a Christian female (d) Criminal Procedure Code
(b) female and a Hindu male 106. Medical Science used for investigating
(c) female and a Christian male crimes is known as
(d) female and a Jew male (a) Criminal Medicine
99. Which of the following Constitutions (b) Epistemological Science

CLAT-6-Previous Years Papers.indd 405 06/09/2010 17:50:13


406 PREVIOUS YEARS’ PAPERS

(c) Forensic Science the person against whom the suit is filed
(d) Ontological Science are called?
107. A puisne judge of a High Court is (a) accused, prosecutor
(a) a judge other than a Chief Justice (b) accuser, defendant
(b) the Chief Justice (c) appellant, respondent
(c) a temporary judge (d) plaintiff, defendant
(d) a retired judge 115. In a criminal case, an accused person, who
108. Intra vires means in consideration of his non-prosecution
(a) within the powers offers to give evidence against other ac-
(b) outside the powers cused, is called
(c) within the scope of fundamental (a) accomplice
rights (b) hostile witness
(d) regular (c) approver
109. X, the servant of Y, takes a hundred rupee (d) hostile accomplice
note from Y’s pocket and hides it under 116. The President of India is elected by an
the carpet in the house of Y. X tells Z an- electoral college consisting of
other servant of Y, about the currency note (a) all the members of both the Houses of
and both agree to share the money when Parliament and all the members of all
the currency note is taken by X from the the Legislative Assemblies
hiding place. Before X could recover the (b) all the elective members of both the
note, it was found by Y. Decide if an of- Houses of Parliament and all the mem-
fence was committed and if so who com- bers of all the Legislative Assemblies
mitted the offence? (c) all the members of both the Houses of
(a) No offence was committed Parliament and all the elected mem-
(b) Only X committed the offence bers of all the Legislative Assemblies
(c) Both X and Z committed the offence (d) all the elected members of both the
(d) Only Z committed the offence Houses of Parliament and all the
110. Moots, in law schools, are elected members of all the Legislative
(a) exercise of law teaching Assemblies
(b) legal problems in the form of imagi- 117. Which of the following is not a fundamen-
nary cases, argued by two opposing tal right?
students before a bench pretending to (a) freedom of speech
be a real court (b) right to life
(c) imaginary class room where a student (c) right to equality
acts as a teacher (d) right to work
(d) a debate on a legal problem 118. International Labour Organization has its
111. Scheduled Tribe status is headquarters at
(a) restricted to Hindus (a) The Hague (b) Geneva
(b) religiously neutral (c) New York (d) London
(c) restricted to Hindus and Christians 119. The Child Marriage Restraint Act 2006 is
(d) restricted to Hindus and Muslims applicable to
112. Which of the following has not been a (a) only Hindus
woman judge of the Supreme Court of In- (b) all Indians except Muslims as the
dia, till 2009? minimum age of marriage among
(a) Justice Gyan Sudha Mishra Muslim girls is puberty (beginning
(b) Justice Sujata Manohar of menstruation) in Muslim personal
(c) Justice Ruma Pal law
(d) Justice Fathima Beevi (c) all irrespective of religion
113. What is the meaning of chattel? (d) all except Muslim, Christians and
(a) any property Jews
(b) immovable property 120. X, a shopkeeper, leaves a sealed 5 kilo-
(c) movable property gram bag of a branded wheat flour at the
(d) cattle door of Y with a note “you will like this
114. In a civil suit, the person who files suit and quality wheat flour and pay Rupees 100

CLAT-6-Previous Years Papers.indd 406 06/09/2010 17:50:13


PREVIOUS YEARS’ PAPERS 407

for this bag” without being asked to do (a) the multinational corporation
so. Y on coming back, collects the bag (b) the Indian companies
from his door, opens the seal of the bag, (c) the consumers
and uses a quarter of kilogram for mak- (d) the market
ing chapattis (unleavened bread). But next 126. Which of the following judges had never
day returns the bag. Is he bound to pay for been the Chairman of the Law Commis-
the bag? He is sion of India?
(a) not bound to pay as he did not ask the (a) Justice R. C. Lahoti
shopkeeper to deliver the bag (b) Justice A. R. Lakshamanan
(b) bound to pay as he has opened the (c) Justice Jeevan Reddy
bag (d) Justice Jagannadha Rao
(c) bound to pay only for the quantity 127. Who among the following was the first
used chief Information Commissioner of In-
(d) neither bound to pay nor return the dia?
bag (a) Wajahat Habibullah
121. Within the jurisdiction of which High (b) Irfan Habib
Court does Lakshadweep fall (c) Tahir Mahmood
(a) Bombay (now is Mumbai) High (d) Najma Heptullah
Court 128. R.T.I. stands for
(b) Kerala High Court (a) Revenue Transactions in India
(c) Madras (now is Chennai) High Court (b) Research and Technology institute
(d) Delhi High Court (c) Rural and Transparency infrastructure
122. Which of the following is not the function (d) Right To Information
of the International Court of Justice? It 129. Fiduciary relationship is relationship
(a) gives advisory opinion at the request based on
of general Assembly (a) contract
(b) gives advisory opinion at the request (b) trust
of Security Council (c) blood relationship
(c) interprets treaties when considering (d) money
legal disputes brought before it by na- 130. Human rights day is observed on
tions (a) February 14 (b) November 26
(d) decides international crimes (c) October 2 (d) December 10
123. Bank nationalization case relates to the 131. ‘No-fault liability’ means
nationalization of (a) liability for damage caused through
(a) some banks by the government of negligence
India after economic liberalization in (b) liability for damage caused through
1991 fault
(b) some banks under a law during the (c) absolute liability even without any
Prime Ministership of Mrs Indira negligence or fault
Gandhi (d) freedom from liability
(c) all the private Indian Banks during 132. An ‘encumbrance’ in legal parlance is a
the Prime Ministership of Narasirnha (a) liability on property
Rao (b) grant of property
(d) all the private Indian Banks during (c) gift of property
the Prime Ministership of Mrs Indira (d) restriction on property
Gandhi 133. A husband and wife have a right to each
124. Which of the following is not included other’s company. This right is called
within the meaning of intellectual prop- (a) matrimonial right
erty? (b) consortium right
(a) Patents (c) marital right
(b) Copyrights (d) conjugal right
(c) Trade mark 134. Release of prisoner before completion of
(d) Property of an intellectual his sentence is called
125. The main aim of the competition Act 2002 (a) release (b) parole
is to protect the interests of (c) acquittal (d) lease

CLAT-6-Previous Years Papers.indd 407 06/09/2010 17:50:14


408 PREVIOUS YEARS’ PAPERS

135. Result of successful prosecution is 142. Principles:


(a) acquittal (b) discharge Freedom consists in making choices out of
(c) conviction (d) charge sheeting two or more alternatives.
Everyone has freedom to speak.
LOGICAL REASONING Facts:
136. Six students A, B, C, D, E and F are sit- X says his freedom to speech includes
ting. A and B are from Mumbai, rest are freedom not to speak. X’s assertion is
from Delhi. D and F are tall but others (a) wrong
are short. A, C and D are girls, others are (b) right
boys. Which is the tall girl from Delhi? (c) wrong because the freedom to speak
(a) F (b) D cannot mean freedom not to speak
(c) E (d) C (d) right because X may opt to speak or
137. P is the brother of Q. R is the sister of Q. S not to speak
is the Sister of R. How is Q related to S? 143. Principles:
(a) brother A master is liable for the wrongful acts of
(b) sister his servant.
(c) brother or sister A person can be called a servant only if
(d) son there is a relation of employment and he
acts under the order and on behalf of his
Directions (Q. 138 to 140): In the following
master.
questions two sets of words have certain rela-
Facts:
tion. Select a word to replace the question-mark
X bank launched a saving scheme for poor
so as to make a similar relational pair with the
sections of the society and the customer
other word in the third set.
can deposit Rs 10 per day. Y an unem-
138. Cat-Kitten; Goat-kid; Sheep-?
ployed youth collected money from sev-
(a) colt (b) filly
eral customers, and on behalf of them de-
(c) lamb (d) wool
posited the money at the Bank every day.
139. Cataract-eye; Pyorrhea-?
The bank gave to Y a small commission.
(a) breath (b) tongue After sometime, Y disappeared without
(c) ears (d) teeth depositing the money given by the cus-
140. Blue-moon; Blue-black; Black-? tomers. The customers bring a suit alleg-
(a) sheep (b) goat ing that the Bank is liable. Decide
(c) sky (d) star (a) the Bank is liable because it paid com-
Directions (Q. 141 to 145): A principle/princi- mission to Y
ples and a fact situation are given in the follow- (b) the Bank is liable because Y was their
ing questions. Decide only on the basis of the servant
principle(s). (c) the Bank is not liable because Y was
141. Principles: not their servant
Neighbour principle—A person is liable (d) No one is liable
if he harms his neighbour. A neighbour is 144. Principles:
one whose action affects another. X propounds the principle that everyone
(ii) One is liable only for contractual rela- in this world always speaks lies.
tions. Facts:
Facts: X wants to know whether this principle is
X manufacturers a food item and sells his logically true or false
food item to Y, a whole seller. Y appoints (a) logically the principle may be true
Z, a retailer to retail these items. Z sells the (b) everyone in the whole of this world
food item to a consumer who after eating does not always speak lies
them falls ill. X is liable to the consumer (c) logically X is also speaking lies
because (d) everyone is basically an honest per-
(a) of contractual relations son
(b) of the Food Adulteration Act 145. Principles:
(c) the consumer is the neighbour of X Whosoever enters into or upon the prop-
(d) of the consumer protection law erty in the possession of another, with in-

CLAT-6-Previous Years Papers.indd 408 06/09/2010 17:50:14


PREVIOUS YEARS’ PAPERS 409

tent to commit an offence or to intimidate Directions (Q. 152 to 156): In the following
or annoy any person in possession of the questions a question and two arguments are
property, and remains there with intent given. Arguments in relation to the question are
thereby to intimidate or annoy another either weak or strong. Use this key to given your
person or with intent to commit an offence responses.
is guilty of criminal trespass. (a) Argument I is strong
Facts: (b) Argument II is strong
The accused entered at night into a house (c) Both I and II are strong
to carryon intimate relations with an un- (d) Both I and II are weak
married major girl on her invitation and 152. Question: Should there be complete ban
information that her family members are on manufacture of fire crackers in India?
absent. However, he was caught by her Argument I: No, This will render thou-
uncle before he could get away. Is the ac- sands of workers jobless.
cused guilty of criminal trespass? He is Argument II: Yes, the fire cracker manu-
(a) guilty of criminal trespass as he an- facturers use child labour.
noyed the uncle 153. Question: Should private operators be al-
(b) guilty because he entered the house to lowed to operate passenger train service in
commit a crime against the girl India?
(c) guilty because no one should enter Argument I: No, private operators do not
into the house of another at night agree to operate on non profitable sectors.
(d) not guilty of criminal trespass Argument II: Yes, it will improve the
146. In a code every letter of the alphabet is re- quality of Indian Railway Service.
placed by some other letter. The code for 154. Question: Should the system of reserva-
tion of posts for scheduled castes be intro-
the name Ram Kumar is
duced in private sector?
(a) Ten Ronet (b) Len Finol
Argument I: Yes, this would give more
(c) Pen Sinet (d) Elephant
opportunity of development to these
Directions (Q. 147 and 148): Four of the given
groups.
five options are alike in a certain way and form
Argument II: No, this would effect mer-
a group. Which is the one that does not belong to
it.
that group in the following questions?
155. Question: Would the problem of old par-
147. (1) 217 (2) 143
ents be solved if children are made legally
(3) 214 (4) 171
responsible to take care of their parents in
(5) 131 old age?
(a) 1 (b) 5 Argument I: Yes, such problems can be
(c) 2 (d) 3 solved only through law.
148. (1) gourd (2) radish Argument II: Yes, this will bring relief to
(3) spinach (4) cucumber old parents.
(5) beetroot 156. Question: Should right to primary educa-
(a) radish (b) beetroot tion be made a fundamental right?
(c) potato (d) spinach Argument I: We should first complete
Directions (Q. 149 to 151): Point out the entry other developmental project, education of
which does not form a class with the other en- children may wait.
tries in the following questions. Argument II: Yes, without primary edu-
149. (a) house cation for all, there cannot be inclusive
(b) mortgage development.
(c) hypothecation 157. Pramesh is heavier than Jairam but lighter
(d) immovable property than Gulab. Anand is heavier than Gulab.
150. (a) Law (b) Court Mohan is lighter than Jairam. Who among
(c) Morality (d) Judge them is the heaviest?
151. (a) Freedom of speech (a) Jairam (b) Anand
(b) Right to equality (c) Gulab (d) Pramesh
(c) Freedom of religion 158. Ravi is the brother of Amit’s son. How is
(d) Right to make contract Amit related to Ravi?

CLAT-6-Previous Years Papers.indd 409 06/09/2010 17:50:14


410 PREVIOUS YEARS’ PAPERS

(a) Cousin (b) Father (b) International Commission of Jurists


(c) Son (d) Grandfather (c) League of Nations
159. If CABLE is coded ZCDAY, then STA- (d) International Court of Justice
BLE will be coded as Directions (Q. 171 to 175): In each of the fol-
(a) TPADAY (b) TPCDCY lowing questions, two statements are given.
(c) TPCDKY (d) TPCYAY There may or may not be cause and effect rela-
160. If CHARTER is coded UMOEPYE, then tionship between the two statements. Mark your
PARTNER will coded as answer using this code.
(a) AONPCYE (b) AEEPCYE (a) Statement I is the cause and statement
(c) AOEACYE (d) AOEPCYE II is the effect
Directions (Q. 161 to 170): In each of the fol- (b) Statement II is the cause and state-
lowing questions two words are paired which ment I is the effect
have a certain relation. Select a correct option (c) Both the statements are independent
to substitute question-mark so as to make a causes
similar relational pair with the word given after (d) Both the statements are independent
double colon (: :). effects
161. Constituent Assembly : Constitution : : 171. Statement I: School education has been
Parliament : ? made free for children of poor families.
(a) Statute (b) Legislative bills Statement II: Literacy rate among the
(c) Speaker (d) Prime Minister poor is steadily growing.
162. Right : Duty : : Power : ? 172. Statement I: Hallmarking of gold jewel-
(a) Wrong (b) Weak lery has been made compulsory.
(c) Powerless (d) Liability Statement II: Many persons do not prefer
163. Elephant : Calf : Tiger : ? to buy Hallmarked jewellery.
(a) Pup (b) Tigress 173. Statement I: Many vegetarians are suffer-
(c) Cub (d) Baby Tiger ing stomach ailments.
164. Patient : Doctor : : Litigant : ? Statement II: Many dead fish were found
(a) Advisor (b) Help near the lake shore.
(c) Legal aid (d) Lawyer 174. Statement I: Ahmed is a healthy boy.
165. Prosecutor : Accused : : Plaintiff : ? Statement II: His mother is very particu-
(a) Appellant (b) Defendant lar about the food he eats.
(c) Plaint (d) Suit 175. Statement I: Rate of crime is very low in
166. Lok Sabha : Meira Kumar : : Rajya Sabha : ? this city.
(a) Hameed Ansari Statement II: The police is efficient in
(b) Najma Heptullah this city.
(c) Sushma Swaraj Directions (Q. 176 to 180): In each of the fol-
(d) Arun Jaitely lowing questions a statement is followed by two
167. President of India : 35 : : Member Parlia- assumptions. These assumptions may or may
ment (Rajya Sabha) : ? not be implicit in the statement. Select your re-
(a) 18 (b) 21 sponse in accordance with the following codes.
(c) 25 (d) 30 (a) Only assumption I is implicit
168. India : Parliamentary System : : USA : ? (b) Only assumption II is implicit
(a) Democratic System (c) Both assumptions I and II are implicit
(b) Presidential System (d) Neither of the assumptions I and II is
(c) Federal System implicit
(d) Republican System 176. Statement: If Ram has finished reading
169. Executive : President : : Judiciary : ? the instructions, let him begin activities
(a) Supreme Court accordingly.
(b) Chief Justice Assumption I: Ram has understood the
(c) Constitution instructions.
(d) Government of India Assumption II: Ram would be able to act
170. World War II : United Nations : : World accordingly.
War I : ? 177. Statement: Children below the age of sev-
(a) Treaty of Versailles en should not be prosecuted for crimes.

CLAT-6-Previous Years Papers.indd 410 06/09/2010 17:50:14


PREVIOUS YEARS’ PAPERS 411

Assumption I: Generally children below (a) 6 km (b) 3 km


seven cannot distinguish between right (c) 0 km (d) 9 km
and wrong. Directions (Q. 187 and 188): Select appropriate
Assumption II: Children below the age of numbers to fill in the blanks in the series given
seven are generally mentally unsound. in the following questions.
178. Statement: The employer has a right to 187. 3, 9, ………., 6561.
reject the application of any candidate for (a) 18 (b) 27
employment without assigning any reason (c) 81 (d) 61
while short listing candidates for inter- 188. 100, 50, 33.33, ………., 20.
view. (a) 25 (b) 30
Assumption I: The employer is impartial (c) 22 (d) 21
and believes in transparency in employ- 189. Which of the following fractions has the
ment practices. highest value?
Assumption II: The employer wants to 3/5, 4/3, 2/5, 1/2
call only those candidates for interview, (a) 3/5 (b) 4/3
who in his opinion are eligible. (c) 2/5 (d) 1/2
179. Statement: The government has decided 190. Four gardeners with four grass mowers
to reduce custom duty on computers. mows 400 m2 of ground in four hours.
Assumption I: The government wants to How long would it take for eight garden-
make computer accessible to larger num- ers with eight grass mowers to mow 800
ber of people. m2 of ground?
Assumption II: Prices in domestic market (a) 8 hours (b) 6 hours
may go up in near future. (c) 12 hours (d) 4 hours
180. Statement: You can win over new friends Directions (Q. 191 to 193): Select one of the
by your warm smile. given numbers to replace question-mark in the
Assumption I: It is necessary to win over series in the following questions.
new friends. 191. 2, 3, 4, 6, 7, 8, 9, 11, 12, 13, 14, 15, 17, 18,
Assumption II: It is always better to smile 19, 20, 21, ?
warmly to new persons. (a) 22 (b) 23
ELEMENTARY MATHEMATICS (c) 24 (d) 25
192. 117, 104, 91, 78, ?
(Numerical Ability) (a) 39 (b) 60
181. Average age of ten persons learning yoga (c) 65 (d) 36
is 32 years. When the age of their instruc- 193. 88, 96, 104, ?
tor is added, the average age becomes 34 (a) 100 (b) 110
years. The age of their instructor is (c) 120 (d) 112
(a) 54 (b) 52 194. Ram borrows ` 520 from Govind at a
(c) 46 (d) 56 simple interest of 13% per annum. What
182. Find 12% of 5000 amount of money should Ram pay to
(a) 620 (b) 600 Govind after six months to be absolved of
(c) 680 (d) 720 the debt?
183. Square root of 400 is (a) ` 552.80 (b) ` 553.80
(a) 40 (b) 25 (c) ` 453.80 (d) ` 353.80
(c) 20 (d) 100 195. Which of the following is a prime num-
184. What is the place (location) value of 5 in ber?
3254710? (a) 19 (b) 20
(a) 10000 (b) 5 (c) 21 (d) 22
(c) 54710 (d) 50000 196. The square root of 289 is
185. 6 × 3 (3 – 1) is equal to (a) 13 (b) 17
(a) 53 (b) 36 (c) 27 (d) 23
(c) 20 (d) 19 197. Find the factors of 330
186. Y travels 15 km due south, then 5 km due (a) 2 × 4 × 5 × 11
west, then 18 km due north, then 3 km due (b) 2 × 3 × 7 × 13
south, then 5 km due east. How far is he (c) 2 × 3 × 5 × 13
from the starting point? (d) 2 × 3 × 5 × 11

CLAT-6-Previous Years Papers.indd 411 06/09/2010 17:50:14


412 PREVIOUS YEARS’ PAPERS

198. Find the factors of 1122 (a) 23 (b) 29


(a) 3 × 9 × 17 × 2 (c) 43 (d) 21
(b) 3 × 11 × 17 × 2 200. If the numbers from 1 to 24, which are di-
(c) 9 × 9 × 17 × 2 visible by 2 are arranged in descending or-
der, which number will be at the 8th place
(d) 3 × 11 × 17 × 3
from the bottom?
199. Which of the following is not a prime
(a) 10 (b) 12
number? (c) 16 (d) 18

ANSWERS
1. (b) 2. (d) 3. (c) 4. (a) 5. (c) 6. (a) 7. (a) 8. (b) 9. (d) 10. (c)
11. (c) 12. (c) 13. (b) 14. (a) 15. (a) 16. (b) 17. (a) 18. (d) 19. (c) 20. (c)
21. (b) 22. (c) 23. (a) 24. (d) 25. (b) 26. (a) 27. (c) 28. (c) 29. (d) 30. (b)
31. (c) 32. (d) 33. (a) 34. (c) 35. (d) 36. (c) 37. (d) 38. (a) 39. (a) 40. (b)
41. (c) 42. (b) 43. (a) 44. (c) 45. (b) 46. (a) 47. (d) 48. (d) 49. (c) 50. (d)
51. (b) 52. (a) 53. (b) 54. (c) 55. (b) 56. (b) 57. (c) 58. (a) 59. (d) 60. (d)
61. (c) 62. (c) 63. (b) 64. (b) 65. (c) 66. (b) 67. (d) 68. (c) 69. (c) 70. (a)
71. (b) 72. (c) 73. (c) 74. (b) 75. (b) 76. (d) 77. (c) 78. (d) 79. (b) 80. (c)
81. (a) 82. (b) 83. (d) 84. (a) 85. (a) 86. (a) 87. (c) 88. (d) 89. (b) 90. (c)
91. (c) 92. (c) 93. (c) 94. (d) 95. (d) 96. (b) 97. (c) 98. (a) 99. (c) 100. (b)
101. (a) 102. (a) 103. (b) 104. (d) 105. (d) 106. (c) 107. (a) 108. (a) 109. (b) 110. (b)
111. (b) 112. (a) 113. (c) 114. (d) 115. (c) 116. (d) 117. (d) 118. (b) 119. (c) 120. (a)
121. (b) 122. (d) 123. (b) 124. (d) 125. (a) 126. (a) 127. (a) 128. (d) 129. (b) 130. (d)
131. (c) 132. (a) 133. (d) 134. (b) 135. (c) 136. (b) 137. (c) 138. (c) 139. (d) 140. (a)
141. (c) 142. (d) 143. (c) 144. (b) 145. (d) 146. (a) 147. (b) 148. (d) 149. (c) 150. (c)
151. (d) 152. (a) 153. (b) 154. (a) 155. (a) 156. (b) 157. (b) 158. (b) 159. (c) 160. (d)
161. (b) 162. (d) 163. (c) 164. (d) 165. (b) 166. (a) 167. (d) 168. (b) 169. (b) 170. (c)
171. (a) 172. (b) 173. (d) 174. (b) 175. (b) 176. (b) 177. (c) 178. (c) 179. (a) 180. (b)
181. (a) 182. (b) 183. (c) 184. (d) 185. (b) 186. (c) 187. (c) 188. (a) 189. (b) 190. (d)
191. (a) 192. (c) 193. (d) 194. (b) 195. (a) 196. (b) 197. (d) 198. (b) 199. (d) 200. (c)

CLAT-6-Previous Years Papers.indd 412 06/09/2010 17:50:14


PREVIOUS YEARS’ PAPERS 413

LLB ENTRANCE TEST (DU) 2010


1. Which organization shifted some of its (d) it controls the pricing and promotion
operations from mainland China to Hong policy of the market.
Kong in March 2010, due to censorship is- 10. The title of “Vikramaditya” was assumed
sues? by:
(a) Microsoft (b) Google (a) Chandragupta II
(c) Dell (d) Wipro (b) Kanishka
2. The word “economics” is derived from (c) Samudragupta
which Greek word? (d) Harshavardhana
(a) Oikos 11. Which of the following rulers is consid-
(b) Oeconomicus ered as a contemporary of the poet, Tulsi-
(c) Ekklesiastikos das?
(d) Oikonomia (a) Sher Shah Suri (b) Shahjahan
3. Which of the following is primarily a digi- (c) Babur (d) Akbar
tal book reader? 12. Which of the following movie was based
(a) Apple I-Pod on a video game?
(b) Sandisk Sansa (a) Chronicles of Riddick
(c) Amazon Kindle (b) Max Payne
(d) Nokia N95 (c) Spiderman 2
4. Who is the author of “The General The- (d) James Bond 007: All or Nothing
ory of Employment, Interest and Money” 13. The British East India Company was for-
(1936)? mally dissolved by an Act of the Parlia-
(a) Milton Friedman ment of the United Kingdom, called:
(b) John Maynard Keynes (a) East India Merger & Acquisition Act
(c) Adam Smith
(b) Government of India Act
(d) Thomas Friedman
(c) East India Stock Dividend Redemp-
5. Goods that are used in production of other
tion Act
goods and services are called:
(d) Companies Act
(a) Capital goods
14. What is the latest client operating system
(b) Consumer goods
for PC from Microsoft as of March, 2010
(c) Homogenous goods
called?
(d) Consumable goods
(a) Windows 7
6. Which among the following storage media
has the maximum storage capacity? (b) Windows CE
(a) Digital Video Disc (DVD) (c) Windows Server 2008
(b) Compact Disc (CD) (d) Windows Vista
(c) Vinyl Record 15. ‘Bay of Pigs’ refers to:
(d) Blu Ray Disc (a) Slaughter of pigs in Egypt, in an at-
7. Around which year did the Persian scholar tempt to prevent outbreak of swine
Al-Beruni visited India? flu
(a) 1220 AD (b) 1000 AD (b) Argentine invasion and occupation
(c) 1030 AD (d) 1347 AD of the Falkland Islands leading to the
8. Who was the last ruler of the Aztec Em- Falklands War
pire of Mexico? (c) Unsuccessful attempt by United
(a) Montezuma II (b) Cuauhtemoc States’ backed Cuban exiles to over-
(c) Hernan Cortes (d) Quetzalcoatl throw the government of the Cuban
9. A firm is an oligopolistic industry if: premier, Fidel Castro
(a) it is run by members of the same fam- (d) The combined territorial waters of
ily. four countries of southern Europe:
(b) it considers the actions of competitors Portugal, Italy, Greece and Spain
when determining its pricing policy. 16. What does a phillumenist collect?
(c) it produces a product identical to the (a) Candles (b) Matchboxes
products produced by its rivals. (c) Newspapers (d) Light bulbs

CLAT-6-Previous Years Papers.indd 413 06/09/2010 17:50:14


414 PREVIOUS YEARS’ PAPERS

17. In Roman Catholic Church, what is the (c) studies of chemical processes on solid
title given to a person who is appointed to surfaces
oppose a candidate’s claim for canoniza- (d) studies of the structure and function of
tion? DNA
(a) Church Defendant 27. Which ad-agency was associated with the
(b) Hell’s Angel creation of animated creatures, ‘Zoozoo’,
(c) Church Prosecutor for promotion of Vodafone’s services?
(d) Devil’s Advocate (a) FCB-Ulka Advertising Ltd.
18. What is the Commonwealth Games, 2010 (b) Ogilvy & Mather Ltd.
mascot called? (c) Mudra Communications Ltd.
(a) Sheru (b) Shera (d) Percept Advertising Ltd.
(c) Cheetah (d) Jigrr 28. By which controversial instrument did the
19. Which of the following is not considered a (then) President of Pakistan, Pervez Mush-
component of microeconomics? arraf, grant powers to the Government to
(a) Demand withdraw prosecution against politicians
(b) Supply accused of various crimes, ostensibly to
(c) Market equilibrium end their political victimization?
(d) Rate of inflation (a) National Reconciliation Ordinance
20. Which of the following has developed the (b) National Coalition to Protect Political
software platform, Maemo? Freedom
(a) Nokia (b) Dell (c) President’s power to grant pardon,
(c) Microsoft (d) Sony Article 45 of the Constitution of the
21. What is the title of the film documenting Islamic Republic of Pakistan
Michael Jackson’s preparations for his (d) Universal Declaration of Human
scheduled 2009-10 concert series? Rights
(a) I’ll Be There (b) Thriller 29. Which instrument aboard the Chan-
(c) This is It (d) Heal the World drayaan-1 detected the presence of water
22. Who, among the following, was referred on the moon’s surface?
to as the ‘Mozart of Madras’ by the Time (a) Hydrophilic Receptor
magazine? (b) Moon Mineralogy Mapper
(a) Ilayaraja (c) Radiation Dose Monitor
(b) A. R. Rahman (d) X-ray Spectrometer
(c) Santha P. Nair 30. What is Satyam Computer Services Ltd.
(d) S. P. Balasubrahmanyam known as, since its takeover?
23. What is the name of the web browser de- (a) TCS Satyam
veloped by Google Inc? (b) Infosys Satyam
(a) Chrome (b) Safari (c) Wipro Satyam
(c) Firefox (d) Mozilla (d) Mahindra Satyam
24. Who is the present Chief of Army Staff? 31. Where was Christ born, according to the
(a) Deepak Kapoor Gospels of Mathew and Luke?
(b) P. V. Naik (a) Jordan (b) Nazareth
(c) V. K. Singh (c) Bethlehem (d) Jerusalem
(d) Nirmal Kumar Verma 32. A campaign launched in April, 2010, with
25. At which position did the Indian hockey a view to clean the river Ganges has been
team finish, in the 2010 Men’s Hockey named as:
World Cup held at Delhi? (a) Sparsh Ganga
(a) Ninth (b) Tenth (b) Niramal Ganga
(c) Seventh (d) Eighth (c) Pavitra Ganga
26. Venkatraman Ramakrishnan was awarded (d) Swargashram Ganga
the 2009 Nobel Prize in Chemistry for: 33. Who created the cartoon character Tintin?
(a) studies of the structure and function of (a) Bill Watterson
the ribosome (b) John Calvin
(b) studies of the structure and function of (c) Charles M. Schulz
the chromosome (d) Herge

CLAT-6-Previous Years Papers.indd 414 06/09/2010 17:50:14


PREVIOUS YEARS’ PAPERS 415

34. Which two water bodies does the Panama 44. The Abel Prize is awarded in the field of:
Canal link? (a) Biology (b) Economics
(a) Mediterranean Sea and Red Sea (c) Geography (d) Mathematics
(b) Atlantic Ocean and Indian Ocean 45. Which of the following aircraft’s primary
(c) Atlantic Ocean and Pacific Ocean role is very high-altitude reconnaissance?
(d) Pacific Ocean and Arctic Ocean (a) Lockheed U-2
35. Which of the following has NASA pro- (b) F-117 Nighthawk
posed to ‘retire from service’ by the end of (c) F-22 Raptor
the year, 2010? (d) F-35 Lightning II
(a) Ares I and Ares V 46. The Census of India, 2011, commenced on
(b) Discovery, Atlantis and Endeavor the 1st April 2010 is:
(c) International Space Center (a) 30th (b) 25th
(d) All of the above (c) 20th (d) 15th
36. The speed of light in vacuum may be 47. Which music band recently won a court
expressed in Mach numbers as, approxi- battle against EMI that may force the lat-
mately: ter to stop selling the band’s individual
(a) Mach 90 songs as downloads?
(b) Mach 9,000 (a) Queen (b) Abba
(c) Mach 9,00,000 (c) Pink Floyd (d) Guns and Roses
(d) Mach 90,000,000 48. How many public sector enterprises have
37. Where would be zero degrees latitude and been conferred the status of Navratna by
zero degrees longitude located? the Government of India, as of November,
(a) Atlantic Ocean (b) Sahara desert 2009?
(c) North Pole (d) South Pole
(a) Nine (b) Twelve
38. In the Rime of the Ancient Mariner, who
(c) Twenty Seven (d) Eighteen
does the mariner stop to recite his story?
49. Which of the following awards is focused
(a) Doctor hastening to a medical emer-
towards television?
gency
(a) Academy Awards
(b) Nobleman going to the House of
(b) Emmy Awards
Lords
(c) Golden Globe Awards
(c) Guest on his way to a wedding
(d) Grammy Awards
(d) Sailor returning to his ship
39. Which part of human body is completely 50. In which movie did Toby Stephens play
devoid of blood supply? the role of Captain William Gordon?
(a) Brain (b) Cornea (a) The Rising—Mangal Pandey
(c) Intestine (d) Thyroid (b) Rang De Basanti
40. Under what capacity is Hillary Clinton (c) Veer
serving in the administration of President (d) Shatranj Ke Khiladi
Barack Obama? 51. Who wrote the the poem, Gunga Din?
(a) Secretary of State (a) Rabindra Nath Tagore
(b) Senator for New York (b) T. S. Eliot
(c) Defense Secretary (c) Rudyard Kipling
(d) Secretary for External Affairs (d) Mulk Raj Anand
41. Safdar Hashmi was associated with: 52. What was the name of the rocket powered
(a) Painting (b) Journalism aircraft which completed first privately
(c) Street theatre (d) Higher education funded human spaceflight?
42. Which country is estimated to have the (a) X-15
third largest population in the world? (b) Space Ship One
(a) Pakistan (c) Space Ship Alpha
(b) Bangladesh (d) Space Odyssey One
(c) Indonesia 53. Which Indian scientist’s article was trans-
(d) United States of America lated to German by Albert Einstein and
43. In which sport is a ‘sand-iron’ used? submitted on his behalf to the Zeitschrift
(a) Hockey (b) Wrestling fur Physik?
(c) Golf (d) Beach Volleyball (a) Satyendra Nath Bose

CLAT-6-Previous Years Papers.indd 415 06/09/2010 17:50:14


416 PREVIOUS YEARS’ PAPERS

(b) Homi Jehangir Bhabha (b) United States of America


(c) Jagdish Chandra Bose (c) Israel
(d) Vikram Sarabhai (d) United Arab Emirates
54. What is the main activity of the ‘Institute 63. ‘Durand Line’ is the border separating:
for Works of Religion’ at Vatican City? (a) India and Pakistan
(a) Restoring old religious relics (b) Pakistan and Afghanistan
(b) Banking (c) Pakistan and Nepal
(c) Spreading religious information (d) India and Nepal
(d) Religious education 64. Which country is known as the ‘Land of
55. Who was referred to as the ‘Godfather Cakes’?
of World Music’ by the Beatles guitarist (a) England (b) Belgium
George Harrison? (c) Scotland (d) France
(a) Ustad Amjad Ali Khan 65. Who, along with Shane Bond, was the
(b) Ustad Ghulam Ali Khan highest bid player in the auction leading to
(c) Pandit Kumar Gandharva 2010 edition of the Indian Premier League
(d) Pandit Ravi Shankar cricket tournament?
56. Which is the only United States’ Naval (a) Kieron Pollard
Base that is located in a country with (b) Kevin Pietersen
which the United States has no diplomatic (c) Sachin Tendulkar
relations? (d) Wayne Parnell
(a) Guantanamo Bay Naval Base 66. Aspirin is a common name for:
(b) U.S. Fleet Activities Yokosuka (a) Propanolol
(c) Naval Air Station Sigonella (b) Acetylsalicylic Acid
(d) Naval Station Rota (c) Neomycin Sulfate
57. Which state is known as the Spice Garden (d) A combination of all the above
of India? 67. Who is the Chief Justice of India?
(a) Jammu and Kashmir (a) Justice K. G. Balakrishnan
(b) Punjab (b) Justice Altamas Kabir
(c) Kerala (c) Justice P. Sathasivam
(d) Uttar Pradesh (d) Justice S. H. Kapadia
58. The mascot adopted in a campaign to raise 68. According to latest surveys, the popula-
civic awareness in Delhi, in the run up to tion of tigers in India has been reported to
the 2010 Commonwealth games has been have dwindled to approximately:
named: (a) 3642 (b) 1411
(a) Delhi Ki Beti (c) 1165 (d) 1657
(b) Shera 69. Which Taliban leader, implicated in the
(c) Delhi Ki Shaan assassination of Benazir Bhutto, was re-
(d) None of the above portedly killed in a 2009 strike by a United
59. Which annual fair in Rajasthan is famous States’ Predator drone:
for its camel trading? (a) Baitullah Mehsud
(a) Surajkund Mela (b) Omar Saeed Sheikh
(b) Kumbh Mela (c) Wali-ur-Rehman
(c) Urs (d) Mullah Mohammed Omar
(d) Pushkar Mela 70. Who is the present Union Minister for Hu-
60. Which city was described by Jawahar Lal man Resource Development?
Nehru as the ‘Oxford of the East’? (a) Dr Veerappa Moily
(a) Colombo (b) Shanghai (b) Shri Kapil Sibal
(c) Pune (d) Banaras (c) Shri Arjun Singh
61. Chilly pepper and tobacco were intro- (d) Dr M. M. Joshi
duced in India by the: 71. Who was awarded the 19th Saraswati
(a) Portuguese (b) British Samman for 2009?
(c) Spanish (d) French (a) Vijay Tendulkar
62. Which of the following countries is to host (b) Amitabh Bacchan
the 2011 World Robot Olympiad? (c) Sunil Gangopadhyay
(a) China (d) Surjit Patar

CLAT-6-Previous Years Papers.indd 416 06/09/2010 17:50:14


PREVIOUS YEARS’ PAPERS 417

72. Who is the Chairman of the Bar Council 82. Which car maker announced the recall
of India? of nearly 100,000 units of its recently
(a) Shri Gopal Subramanium launched hatchback models citing fuel
(b) Shri Ram Jethmalani tank problems?
(c) Shri P. P. Rao (a) Maruti Suzuki India Ltd
(d) Shri N. R. Madhava Menon (b) Hyundai Motors India Ltd
73. In March this year, a city/town declared (c) General Motors Corporation
spitting of Paan (Betel leaf) as illegal, in- (d) Chevrolet
viting fines for criminal damage. Where is 83. The body of a shark has no:
this city located? (a) Gills (b) Liver
(a) Uttar Pradesh (b) Maharashtra (c) Bones (d) Fins
(c) Ottawa (d) London 84. Who is associated with the remark, “the
74. The Delhi Technological University was law, in its majestic equality, forbids rich
formerly known as the: and poor alike to sleep under bridges,
(a) I.I.T., Delhi to beg in the streets, and to steal their
(b) Aryabhat Polytechnic, Delhi bread”?
(c) Delhi College of Engineering (a) Anatole France
(d) Indraprastha University (b) Glanville Williams
75. The Gulf separating Finland and Sweden (c) Charles Dickens
is known as the: (d) Victor Hugo
(a) Gulf of Bothnia 85. Which Bill, reportedly laying down a code
(b) Gulf of Lions of conduct for judges of the higher judi-
(c) Gulf of Genoa ciary was referred to a Group of Ministers
(d) Gulf of Venice for scrutiny in March 2010?
76. Groundwater confined between two im- (a) The Supreme Court (Number of Judg-
permeable layers is called: es) Amendment Bill
(a) Artesian well (b) Artesian (b) Judges (Inquiry) Bill
(c) Trapper water (d) Artisan (c) Judges (Declaration of Assets and Li-
77. Which of the following is a country lo- abilities) Bill
cated in the continent of Europe? (d) Judicial Standards and Accountability
(a) Libya (b) Venezuela Bill
(c) Algeria (d) Albania 86. An observer in a leading national daily re-
78. Which is the oldest mountain range in In- marked the passage of Women’s Reserva-
dia? tion Bill through Rajya Sabha as a biparti-
(a) Satpura (b) Aravali san consensus and an extraordinary act of
(c) Vindhya (d) Himalaya political selflessness whereby a particular
79. Nephrons are a part of human: party put the interests of the nation above
(a) Nervous System its own. In terms of numbers, how may
(b) Respiratory System members voted in favour and against the
(c) Excretory System Bill, respectively?
(d) Endocrine System (a) 245 to None
80. Which of the following about BT Brinjal (b) 186 to 1
developed by Monsanto and Mahyco is in- (c) 232 to 1
correct? (d) The entire strength of 233 members
(a) It requires more fertilizer than ordi- voted in favour
nary Brinjal plant. 87. Interpreting Articles 72 and 161 of the
(b) It is a transgenic variety of Brinjal. Constitution of India in relation to a crimi-
(c) It contains genes from the soil bacte- nal statute, a Bench of the Supreme Court
rium Bacillus thuringiensis. of India, headed by the Chief Justice of
(d) It is claimed to have higher resistance India, has, in a judgment dated 22.03.2010
against pests. held that:
81. In a eukaryotic cell, DNA may be stored (a) The clemency power of the Executive
in: is absolute, remains unfettered and
(a) Nucleus (b) Mitochondria cannot be restricted by the provisions
(c) Chloroplast (d) All of the above of a Statute.

CLAT-6-Previous Years Papers.indd 417 06/09/2010 17:50:14


418 PREVIOUS YEARS’ PAPERS

(b) Section 377 of the Indian Penal Code, (a) The Act operates prospectively
insofar as it criminalizes consensual (b) The Act operates retrospectively
sexual acts of adults in private is vio- (c) The relief under this Act may be
lative of the Constitution of India. sought even for acts committed prior
(c) Once possession is established, the to October 2006
person who claims that it was not a (d) Both (b) and (c)
conscious possession has to establish 93. Which of the following is a well known
it. law firm in India?
(d) Both (a) and (c) (a) AZB & Partners
88. In India, making a direct marketing tele- (b) TATA Consultancy
phone call, which a person has opted not (c) Leo Burnett India Pvt. Ltd.
to receive, is prohibited under:
(d) Goldman Sachs Group, Inc.
(a) Unsolicited Direct Marketing Act
94. Which of the following most closely de-
(b) Unsolicited Communications Regula-
scribes the act of downloading Bollywood
tions
film songs for free, from websites over the
(c) Telecom Unsolicited Commercial
Communications Regulations internet?
(d) Telecom Unsolicited Commercial (a) Attempt to hack a computer system
Calls Regulations (b) Exercise of freedom of speech and ex-
89. Under the Indian Penal Code, an act may pression
be an offence under the Code, but for the (c) Breach of implied contract
existence of a justification which may (d) Violation of intellectual property
form a defence. Which of the following is rights
not such a defence? 95. Which of the following is not a feature of
(a) Act done by a child under seven years the criminal justice system in India?
of age (a) Innocent until proven guilty
(b) Act done by a person of unsound (b) Right against self-incrimination
mind (c) Preponderance of probability
(c) Act done by a child of immature un- (d) Proof beyond reasonable doubt
derstanding, above seven years and 96. Which High Court exercised jurisdiction
under twelve in age. over Delhi, prior to the establishment of
(d) Act done outside the territorial limits the High Court of Delhi?
of India. (a) High Court of Gujarat
90. The Penal Code in force in the territory of (b) High Court of Punjab
Jammu and Kashmir is known as the: (c) High Court of Allahabad
(a) Indian Penal Code (d) High Court of Himachal Pradesh
(b) Macaulay Penal Code
97. Which of the following bodies was in-
(c) Ranbir Penal Code
strumental in setting-up permanent and
(d) Jammu and Kashmir Penal Code
continuous Lok Adalat in Delhi, to resolve
91. Which of the following measure was pro-
posed in December 2009 for speedy reso- disputes pertaining to bodies such as the
lution of commercial disputes valuing 5 M.C.D., N.D.M.C., B.S.E.S., N.D.P.L.,
Crores or above, by an enactment of the M.T.N.L. etc.?
Parliament of India? (a) Delhi Legal Services Authority
(a) Setting up commercial Lok Adalats (b) Bar Council of India
(b) Creation of a commercial division in (c) Bar Association of Delhi.
the High Courts (d) Indian Council of Arbitration
(c) Mandatory reference to arbitration, 98. Under which provision of the Code of
where commercial contract contains Criminal Procedure may a Court make
an arbitration clause a proclamation requiring a person to ap-
(d) All of the above pear at a specified place, if the Court has
92. A judgment relating to the Protection of a reason to believe that the person has ab-
Women against Domestic Violence Act, sconded?
2005, delivered on 22nd March 2010 by (a) Section 40 (b) Section 42
the High Court of Delhi has held that: (c) Section 73 (d) Section 82

CLAT-6-Previous Years Papers.indd 418 06/09/2010 17:50:14


PREVIOUS YEARS’ PAPERS 419

99. Any proceedings for removal of a judge of to remain present during the court
any High Court in India may be initiated proceedings
only by: (c) A court order to compel the presence
(a) an order of the President of India of a witness at a specified place, if the
(b) a notice of motion for presenting an Court has a reason to believe that the
address to the President, praying for person has absconded
the removal of a Judge, signed by req- (d) Both (a) and (b)
uisite number of members of the Par- 104. Which of the following developments is
liament associated with a March 2010 judgment
(c) address by each house of the Parlia- of the High Court of Delhi, relating to the
ment, supported by the requisite num- role of women in the Indian armed forc-
ber of Members of Parliament es?
(d) constitution of a Committee for the (a) Opportunities to women officers to
purpose of making an investigation operate in combat areas
into the grounds on which the removal (b) Eligibility of women officers for Per-
of the Judge is prayed for manent Commission
100. In a recent case involving Toyota and Pri- (c) Reservation for women in Indian
us Auto Industries, Toyota was seeking an armed forces
interim order to restrain Prius Auto Indus- (d) Both (a) and (b)
tries from using the word Prius, claiming 105. In criminal proceedings, acquittal refers
its exclusive right to use it in view of its to:
car named Prius. While denying this relief, (a) Being cleared of a charge or accusa-
a single Judge of the High Court of Delhi, tion
by a judgment delivered on 19.03.2010, (b) Pronounced guilty of a charge or ac-
held that: cusation
(a) Prius was not an invented word (c) Conviction for a lesser offence, as part
(b) Toyota had approached the court too of a plea bargain
late (d) Taking a lenient view, considering the
(c) Prius Auto Industries was the regis- honest past record of an accused
tered owner of the trademark ‘Prius’, 106. The legal branch of the Indian Army is
since 2002 known as:
(d) All of the above (a) Army Legal Corps
101. In India, “identity theft” has been made (b) Judge Advocate General branch
punishable under the: (c) Court Martial branch
(a) Identity Theft Assumption and Deter- (d) Armed Forces Legal Corps
rence Act 107. A clause in the World Anti-Doping Code,
(b) Information Technology Act which was reportedly a cause of concern
(c) Indian Penal Code for Indian cricketers, is popularly referred
(d) Prize Chits and Money Circulation to as:
Schemes (Banning) Act (a) Endorsements Clause
102. Which of the following enactments is pro- (b) Roundabout Clause
posed to ensure supply of at least a mini- (c) Whereabouts Clause
mum quantity of food grains to families (d) No-notice Drugs Test Clause
living Below Poverty Line in India? 108. To provide affordable access to justice to
(a) Food Safety Enhancement Act the citizens of India at the grass roots level,
(b) Food Safety Act at their doorsteps, a body was established
(c) Essential Services Maintenance Act by an Act of the Parliament that came into
(d) National Food Security Act force on 2nd October 2009. What this stat-
103. In a Civil Writ Petition, a ‘stay’ may refer utory body is called?
to: (a) Nyaya Panchayat
(a) A court order restraining an action un- (b) Gram Nyayalaya
til a future event occurs, or the order is (c) Gram Panchayat
lifted (d) Lok Adalat
(b) A court order directing an individual 109. The Right of Children to Free and Com-

CLAT-6-Previous Years Papers.indd 419 06/09/2010 17:50:14


420 PREVIOUS YEARS’ PAPERS

pulsory Education Bill was enacted to (b) Advice him to convince his wife for a
give effect to: mutual, uncontested divorce.
(a) Article 14 of the Constitution of India (c) Refuse to be engaged in the case,
(b) Article 45 of the Constitution of India since it is against professional conduct
(c) Article 21A of the Constitution of In- of a lawyer set by the Bar Council of
dia India.
(d) Article 21 of the Constitution of India (d) Advice him to approach for concilia-
110. What is the closest meaning of the expres- tion proceedings.
sion, ‘ex aquo et bono’? 115. F.I.R. stands for First Information Report.
(a) In equity and good conscience As per the High Court of Delhi, if at the
(b) For the public good time of registration of F.I.R. by the police,
(c) The thing speaks for itself it appears that crime was committed out-
(d) In good faith side the jurisdiction of that Police Station,
111. Consider the following statement: “Read- the Police after registration of such F.I.R.
ing is an activity involving the use of vi- should transfer the F.I.R. to the Police Sta-
sual apparatus by means of which printed tion having jurisdiction. Such F.I.R. made
letters are recognized”. This definition at the first instance has been termed as:
would be weakened most by pointing out (a) Preliminary F.I.R.
that: (b) Zero F.I.R.
(a) ‘Skimming’ is a form of reading. (c) Daily Diary (D.D.) Number
(b) An illiterate may recognize letters. (d) Investigatory F.I.R.
(c) Some printed letters are difficult to in- 116. A student of the University of Delhi has
terpret. to pass in _____________ Papers with at
(d) Seeing is necessary for reading. least _____________ in each paper for the
112. History abounds with examples where award of LL.B. Degree:
chieftains have refused to transfer their (a) 30, 45 (b) 25, 40
military units to other chieftains who re-
(c) 32, 48 (d) 35, 50
quested such transfer. The refusals oc-
117. Point out the correct sentence:
curred even when the chieftain who re-
(a) Before joining as a Lecturer, I used to
ceived the request for assistance was in
work as a Lab Assistant for about five
a relatively safe place and even when the
years.
requesting chieftain’s clan was under at-
(b) Before joining as a Lecturer, I have
tack. Each of the reasons below, if true,
working as a Lab Assistant for about
supports the decision of the chieftain who
five years.
refuses to share his forces, EXCEPT one.
Mark the one. (c) Before joining as a Lecturer, I worked
(a) Greatest chieftains have emerged as as a Lab Assistant for about five
victors in battles where they were out- years.
numbered by the enemy. (d) Before joining as a Lecturer, I have
(b) War and peace are unpredictable, and been working as a Lab Assistant for
situations may change in a short time. about five years.
(c) The chieftain with a larger army re- 118. Read the following sentences carefully:
tains greater influence in the region. a. Most days my father __________ get
(d) Armies on the move are unnecessarily up first and make breakfast.
exposed to danger. b. When I was training for the marathon,
113. Which of the following is an anagram for I __________ run over 100 kms.a
the word “dormitory”? week.
(a) Hostel (b) Dirty Room c. We went to Agra to see the Taj where
(c) Dorm (d) My Door we ___________ regularly.
114. You are a qualified lawyer and a husband Which one of the following verbs can
approaches you, seeking your professional complete all three sentences?
services for divorce proceedings. Ideally, (a) Will (b) Would
what should you do? (c) Shall (d) used to
(a) Explain the difficulties involved in the 119. Fill in the blank with correct option: I
proceedings for divorce. _____ at 6.a.m. but ____ to be up by 5.00

CLAT-6-Previous Years Papers.indd 420 06/09/2010 17:50:15


PREVIOUS YEARS’ PAPERS 421

(a) normally get up …… I have some (c) M has committed the offence of cul-
times pable homicide since he caused death
(b) normally get up .…… sometimes by doing an act with the intention of
have causing such bodily injury as is likely
(c) get normally up ……. sometimes I to cause death.
have (d) M is guilty of committing murder
(d) get normally up …… I sometimes since he did an act by which the death
have was caused and which was done with
120. ______________ people trying to get into the intention of causing death.
the football stadium: 123. Principle: The word ‘offence’ denotes a
(a) There were too much thing made punishable by the Indian Penal
(b) It was too many Code. A prevents B from proceeding in
(c) There was too many any direction and keeps him/her confined
(d) There were too many to a place. A has committed an offence of
121. Principle: Justice must not only be done, criminal confinement. According to the
but also be seem to be done. Indian Penal Code, whoever instigates a
Facts: A judge is a shareholder in a Com- person to do a thing, is said to abet such a
pany, which is a party to a case before him. thing.
The integrity of the judge is above suspi- Facts: L, in an interview to a magazine,
cion. Both the opposing counsels affirm articulated her views in favour of living
their faith in the impartiality of the judge. together without a nuptial knot. Aggrieved
The case is at the stage of final arguments. by the same, X filed a criminal complaint
The judge, however, recuses himself from against L, for abetting criminal confine-
the case, on the ground of his sharehold- ment. Can L be prosecuted and convicted
ing. Which of the following is the most for the same?
appropriate statement in the light of the (a) Yes, because the comments of L could
Principle cited here? have encouraged people to live to-
(a) The judge should have continued gether without entering into a nuptial
hearing the case since he was an hon- bond.
est judge. (b) Yes, because the comments of L in
(b) The judge should have continued fact encouraged certain individuals to
hearing since the counsels of both live together without entering into a
the parties had requested him to do so nuptial bond.
and, had thus agreed to adjudication (c) No, because two adult people wanting
by him. to live together is not an offence.
(c) Had the judge continued to hear the (d) No, because the comments of L were
case, its outcome might have appeared not directed towards any particular
to be prejudiced, even though it would person, and anybody acting on them,
not have been so. did so at his/her own risk.
(d) The judge could have lost objectivity 124. On 1st April, P and Q, friends of X, de-
in the case and therefore recused from cided to pull a prank on her. They wore
it. black masks to cover their faces and pro-
122. M was driving his fancy sports car at the ceeded in an un-numbered car, towards
speed of 120 kmph, after consuming al- the road that X took her regular morning
cohol. He lost control of the car and ran walk. As X was taking her morning walk,
over 3 persons leading to their death. M they brought the car to a halt next to her
escaped the accident with injuries. What and proceeded to forcefully get her in it. A
offence, if any, has M committed under bystander, Y, watching this opened fire on
the Indian Penal Code? P and Q with his licensed gun and caused
(a) M is not guilty of any offence since grievous injuries to them.
he was incapable of judgment by rea- (a) Y is guilty of causing grievous hurt
son of intoxication caused against his as he had no right to intermeddle in
will. the private affairs of the friends and
(b) M is guilty of causing death by doing he was neither a public authority nor a
a rash or negligent act. police official.

CLAT-6-Previous Years Papers.indd 421 06/09/2010 17:50:15


422 PREVIOUS YEARS’ PAPERS

(b) Y is not guilty of any offence because 129. The first woman President of India hails
he had a right to protect the body of from which city of India?
any other person, besides his own. (a) Jalna (b) Jalgaon
(c) Y is not guilty of any offence because (c) Amravati (d) Sangli
he had a reasonable apprehension that 130. The rank of Colonel in the Indian Army
X was being assaulted with an inten- is equivalent to which rank in the Indian
tion of abduction. Navy?
(d) Both (b) and (c) (a) Captain (b) Commodore
125. Principle I: According to Section 494 of (c) Commander (d) Lieutenant
the Indian Penal Code, whoever, having a 131. In which city are the headquarters of Zoo-
husband or wife living, marries in any case logical Survey of India situated?
in which such marriage is void by reason (a) Delhi
of its taking place during the life of such (b) Thiruvanatpuram
husband or wife, shall be punished with (c) Kolkata
imprisonment of either description for a (d) Guhawati
term which may extend to seven years, 132. Mount Harriat National Park is located
and shall also be liable to fine. in:
Principle II: Hindu law, as contained in (a) Andaman Nicobar Islands
the Hindu Marriage Act, enforces mo- (b) Madhya Pradesh
nogamy. Polygamy is permitted under the (c) Andhra Pradesh
Muslim law. (d) Arunachal Pradesh
Facts: Jitendar, a Hindu, was married to 133. Who is the present Prime Minister of Ma-
Meena, also a Hindu. Jitendar and Sunita, laysia?
another Hindu girl, converted to Islam and (a) Mohamed Nasheed
got married. (b) Ahmadinejad
Question: Is Jitendar guilty of the offence (c) Najib Rajak
under Section 494 of the Indian Penal (d) Saad Hariri
134. Which of the following is the National
Code?
aquatic animal of India?
(a) No, because Jitender converted to
(a) Ganges crocodile
Islam which permits more than one
(b) River dolphin
wife.
(c) Blue whale
(b) No, because Jitender and Sunita con-
(d) Olive Ridley turtle
verted to Islam and married, with mu-
135. Which team won the Sultan Azlan Shah
tual consent.
Cup in 2009 held in Kuala Lumpur?
(c) Yes, because Jitender had a wife liv-
(a) Pakistan (b) New Zealand
ing. He married again. The said mar-
(c) India (d) Australia
riage was void by reason of its taking 136. Which of the following is the capital of
place during the life of the first wife, Croatia?
in terms of Section 494, IPC. (a) Zagreb (b) Nicosia
(d) Both (a) and (b) (c) Nuuk (d) Roseau
126. In which city is Khuda Baksh Oriental 137. Point out the incorrect match:
Public Library situated? (a) Mary Kom-Rajiv Gandhi Khel Ratna
(a) Delhi (b) Kolkata Award
(c) Lucknow (d) Patna (b) Sinimol Paulose-Arjuna Award
127. Kuchipudi is a dance form owing its ori- (c) P. Gopichand-Dronacharya Award
gin to: (d) Jaydev Bisht-Dhyan Chand Award
(a) Kerala (b) Andhra Pradesh 138. Mark the correct pair.
(c) Orissa (d) Tamil Nadu (a) Yamo wind-Uganda
128. When was the National Anthem of India (b) Zonda wind-Japan
first sung? (c) Austru wind-Austria
(a) 25th December 1909 (d) Laventer wind-Canada
(b) 26th December 1910 139. Who wrote the Seva Sadan?
(c) 27th December 1911 (a) Rabindra Nath Tagore
(d) 28th December 1912 (b) Munshi Premchand

CLAT-6-Previous Years Papers.indd 422 06/09/2010 17:50:15


PREVIOUS YEARS’ PAPERS 423

(c) Rama Krishna Paramhansa Questions 148 to 150 are based on the follow-
(d) Sister Nivedita ing facts:
140. Match the correct pair: A law firm comprises of 6 cubicles, sepa-
(a) Syphilis-Treponema pallidum rated from each other by 6 feet high partitions;
(b) Chancroid-Orthomyxoviridae sound and smells therefore permeate to adja-
(c) Influenza-Haemophilus ducreyi cent cubicles. You have been assigned the task
(d) Polio-Legionella of allotting the cubicles to six employees of the
141. Name the officer of Indian army who initi- firm.
ated an inquiry into the Sukna land case? Ramesh is the secretary and telephone op-
(a) Gen. V. K. Singh erator and needs to use the telephone quite often
(b) Gen. Deepak Kapoor during the course of the day. All the employees
(c) Gen. Padmanabhan generally maintain silence.
(d) Gen. B. C. Joshi Manish and Brij need adjacent cubicles
142. Who is the head of the National Advisory since they need to consult often.
Council? Hemant is the senior partner and needs to
(a) Manmohan Singh be allotted cubicle 5 which faces a window.
(b) Montek Singh Dinesh requires silence in cubicle next to
(c) Sonia Gandhi his.
(d) Pranab Mukherji Tarun, Manish and Dinesh are smokers.
143. Which of the following country is not the Hemant is allergic to cigarette smoke.
member of North Atlantic Treaty Organi- 148. The three employees who are smokers
sation (NATO)? should be allotted cubicles numbered:
(a) Argentina (b) Croatia (a) 1, 2, 6
(c) Albania (d) Canada (b) 4, 6, 1
144. Name the book for which Aravind Adiga (c) 1, 2, 3
won the booker prize in 2008? (d) Cannot be accommodated in the same
(a) The Crouching Tiger, Hidden Dragon office.
(b) Red Tide Rising 149. Ideally, Manish should be allotted cubicle
(c) The Remains of the Day number:
(d) The White Tiger (a) 1 (b) 6
145. At a given time of the day, a 9 feet high (c) 4 (d) 3
pole casts a shadow of 15 feet. At the same 150. All other things remaining same, which of
time, an adjacent tree casts a shadow of 40 the following events may lead to a request
feet. How tall is the tree? for change of cubicle?
(a) 12 feet (b) 64 feet (a) Hemant operating a portable radio-set
(c) 24 feet (d) 20 feet in his cubicle.
146. A’s initial salary was increased by 40%. (b) Ramesh needing silence in cubicles
However, during the period of recession, adjacent to his.
his salary was decreased by 20%. On the (c) Tarun taking over the duties of
whole: Ramesh.
(a) A’s salary increased over his initial (d) None of the above
salary by 12%. 151. How is 6/8 expressed as a percentage?
(b) A’s salary increased over his initial (a) 0.75% (b) 75%
salary by 20%. (c) 7.5% (d) 48%
(c) A’s salary decreased over his initial 152. A man is facing west. He turns 45 degrees
salary by 12%. in clockwise direction and then another
(d) A’s salary returned to the same level 180 degrees in the same direction and then
as his initial salary. 270 degrees in the anti-clockwise direc-
147. A man is 5 years elder to his wife and she tion. What direction is he facing now?
is now three times the age of their daugh- (a) South (b) South East
ter. If the daughter’s age is 10 years, what (c) West (d) South West
was the age of the man when the daughter 153. What is the missing number in the series,
was born? 7, 15, 32, __, 138, 281?
(a) 30 (b) 25 (a) 64 (b) 67
(c) 45 (d) 40 (c) 35 (d) 45

CLAT-6-Previous Years Papers.indd 423 06/09/2010 17:50:15


424 PREVIOUS YEARS’ PAPERS

154. It takes a moving train 30 seconds to cross 162. A joint team of scientists from Russia and
a stationary engine. To compute the speed U.S. was recently reported to have created
of the train, which of the following should the element to fill the gap in the periodic
be known? table of chemical elements. The element
(a) The length of the stationary engine. was called:
(b) The length of the moving train. (a) Element 117 (b) Element 118
(c) The length of the moving engine. (c) Ununoctium (d) Eka-radon
(d) Both the length of the stationary en- Passage:
gine as well as the length of the mov- I entered the room where the corpse lay,
ing train. and was led up to the coffin. How can I describe
155. A pole has been affixed in a riverbed. One- my sensations on beholding it? I feel yet parched
half of the pole is buried in the riverbed. with horror, nor can I reflect on that terrible mo-
One-third of it is covered by water. 8 feet ment without shuddering and agony, that faintly
of it projects out of the water. What is the reminds me of the anguish of the recognition.
total length of the pole in feet? The trial, the presence of the magistrate and wit-
(a) 48 (b) 13 nesses, passed like a dream from my memory,
(c) 18 (d) 12 when I saw the lifeless form of Henry Clerval
156. Which word would complete the follow- stretched before me. I gasped for breath; and,
ing sequence: DIRECT, ROLE, OCCI- throwing myself on the body, I exclaimed, “Have
DENTAL, DARE, ______? my murderous machinations deprived you also,
(a) DRAGON (b) ARMOUR
my dearest Henry of life? Two I have already
(c) ARMY (d) PROVOKE
destroyed; other victims await their destiny: but
157. Since October 2009, which is the highest
you, Clerval, my friend, my benefactor –”
Court of appeal for England and Wales?
The human frame could no longer support
(a) The Supreme Court of the United
the agonizing suffering that I endured, and I
Kingdom.
was carried out of the room in strong convul-
(b) The House of Lords.
sions. A fever succeeded to this. I lay for two
(c) The High Court of Justiciary.
months on the point of death; my ravings, as I
(d) The Privy Council.
afterwards heard, were frightful; I called myself
158. Which odorant is commonly added to
household Liquefied Petroleum Gas, to the murderer of William, of Justine, and of Cler-
easily detect its leakage? val. Sometimes I entreated my attendants to as-
(a) Methane (b) Propane sist me in the destruction of the fiend by whom
(c) Benzinol (d) Mercaptan I was tormented; and, at others, I felt the fingers
159. Which football club did Lionel Messi of the monster already grasping my neck, and
play for in the 2009-10 UEFA Champions screamed aloud with agony and terror. Fortu-
League? nately, as I spoke my native language, Mr. Kir-
(a) Barcelona win alone understood me; but my gestures and
(b) Lyon bitter cries were sufficient to affright the other
(c) Arsenal witnesses. Why did I not die? More miserable
(d) Manchester United than man ever was before, why did I not sink
160. Which Indian Company acquired the Me- into forgetfulness and rest? Death snatches away
gasari Makmur Group of Indonesia in many blooming children, the only hopes of their
2010? doting parents; how many brides and youthful
(a) Reliance Industries Ltd lovers have been one day in the bloom of health
(b) Godrej Consumer Products Ltd and hope, and the next a prey for worms and
(c) TATA Motors Ltd the decay of the tomb! Of what materials was
(d) Mittal Steel Company N.V. I made, that I could thus resist so many shocks,
161. Which of the following spots reportedly which, like the turning of the wheel, continually
witnesses the largest sale of ice-cream in renewed the torture?
India? Questions:
(a) Juhu Beach, Mumbai 163. Describe the emotions of the narrator as
(b) Gateway of India, Mumbai can be sensed from the above passage:
(c) Marina Beach, Chennai (a) Humiliation and pain
(d) India Gate, Delhi (b) Grief and loss

CLAT-6-Previous Years Papers.indd 424 06/09/2010 17:50:15


PREVIOUS YEARS’ PAPERS 425

(c) Guilt and despair mum of about 10,000 a year, set in


(d) Both (b) and (c) 1979, when use increased 250 percent
164. What does the use of phrases such as in just three seasons. Each year since,
‘parched with horror’, ‘shuddering and the actual number of people down the
agony’, ‘I gasped for breath’ suggest to river has hovered close to this ceiling,
the reader: which the agency believes is the riv-
(a) The mental torture that the narrator is er’s capacity for a “quality wilderness
undergoing. experience.” Socially, if not physical-
(b) The violent body language of the nar- ly, however, “wilderness experience”
rator. seems to have become an illusion if
(c) The mental torments of the narra- not irrelevant. Right now I’m the vol-
tor expressed by a violent body lan- untary ranger managing both the illu-
guage. sion and the irrelevance.
(d) The hallucinations in the mind of the (III) Most people accept the permit system
narrator, arising out of his grief. as a panacea for the explosion in num-
165. The narrator’s ravings could have been bers of river runners and the conse-
taken as an admission of guilt of being re- quences for a fragile riparian corridor.
sponsible for the death Henry Clerval, but Others find regulation about as pain-
for the fact that: less as an IRS audit.
(a) He was speaking in his native lan- (IV) They see the Southwest as a region
guage. of federally neutered rivers where a
(b) He was suffering immensely. person is no longer free to kill himself
(c) He was physically ill. in a four-foot rubber ducky pulling an
(d) He was mentally unsound. inner tube piled with beans, testoster-
166. The word ‘machination’ means: one, and a small machete. …
(a) Inventing a new machine. (V) The boat ramp is swarming with
(b) A crafty scheme. people and vehicles to be shuttled to
(c) Foul intention. the takeout. Someone’s dog is throw-
(d) Understanding the working of some- ing up what appears to be rabbit parts.
thing or someone. I’m approached by a pickup driven by
167. The narrator appears to make several refer- a man waving a spray nozzle and hose
ences to his own suffering, but ignores the hooked to a large barrel of allegedly
suffering undergone by those he claimed lethal chemicals. He’s from county
to have murdered, or others bereaved by weed control, he says. Have I seen the
the deaths. Considering this, the style of loathsome pepperweed? Not a leaf, I
the passage may best be described as: lie.
(a) Ironical (b) Comical (VI) Cheerfully I sign the permit of the
(c) Satirical (d) Self Centered outfitter who specializes in theme
Passage: river trips—stress management semi-
(I) I savor the remoteness and the rare nars, outings for the crystal fondlers
times I’m alone on this muscular river or fingernail technicians of East Je-
in southern Utah, a precious ribbon sus, New Jersey, overcoming, at last,
of wild water between reservoirs and their irrational fear of Nature. Today’s
the suck holes of industry and agricul- load is priests troubled by a lapsed
ture. faith—pale, anxious, overweight fel-
(II) Officially, I’m here to have my peace lows in the early stages of heatstroke.
disturbed. Floaters must have a permit I also check gear and answer ques-
to run this stretch of river. During the tions about bugs, snakes, scorpions,
peak season a ranger checks lottery- camps, rapids and Indians (one side of
drawn launch dates and a short list the river is reservation land). Do I live
of gear related to safety and environ- here fulltime? they ask. No, I respond,
mental protection. The permit system except for an occasional shift at the
allows the federal agency in charge to put-in, I’m on the river eight days out
hold numbers of floaters to a maxi- of sixteen, six months a year.

CLAT-6-Previous Years Papers.indd 425 06/09/2010 17:50:15


426 PREVIOUS YEARS’ PAPERS

(VII) … Some rafts look as if they barely those paragraphs?


survived World War II. (a) Physical appearance
(VIII) Others are outfitted with turbo-dy- (b) Response to nature
namic chrome-plated throw lines, (c) Occupations
heatwelded vinyl dry-bags, cargo nets (d) Conversation
spun from the fibers of dew-fed arach- 171. The description, “androgynous progeny
nids from Borneo, horseshoes, vol- who prefer to be at home fulfilling their
leyball sets, sauna tents, coffin-sized needs electronically” refers to:
coolers stuffed with sushi, a small (a) Pets not accustomed to outdoor life.
fleet of squirt boats, whining packs (b) Rafters’ preference to rafts with mod-
of androgynous progeny who prefer ern electronic gadgetry.
to be at home fulfilling their needs (c) Rafters with battered rafts envying the
electronically. All of this gear is color- ones with modern equipment.
coordinated with SPF 14 sunscreen (d) Children who would rather be playing
and owned by business majors in Sty- video games.
rofoam pith helmets and Lycra body 172. The effect of the last sentence of the pas-
gloves, in which they were placed at sage may be described as:
birth. Once loaded, their boats are (a) Comic anti-climax
pieces of personal architecture, stun- (b) Allegory
ning but nevertheless stuck on the (c) Symbolic conclusion
sandbar six feet out from the boat (d) Relief
ramp after a dramatic send-off. 173. Paragraph VI makes a reference to:
Questions: (a) Native Americans
168. The author of the aforesaid passage may (b) “Non-Resident Indians”
be best described as: (c) Immigrants
(a) Narrator of the events of the past. (d) Mexicans
(b) An ecologist concerned at the exploi- 174. The narrator uses all the following tones
tation of the river. of voice except:
(c) A ranger making an official report of (a) Amused
the events. (b) Satirical
(d) A commentator on contemporary (c) Cheerful
American customs. (d) Arrogant
169. Which of the following is NOT employed 175. Which of the following best describes the
in the Paragraph III of the above passage primary purpose of the passage?
(“Most people accept permit … IRS au- (a) To describe everyday life around
dit”)? American ‘adventure camps’?
(a) Pun (b) Simile (b) To protest against exploitation of nat-
(c) Metaphor (d) Hyperbole ural resources.
170. Paragraphs V and VI describe the charac- (c) To satirize aspects of the ‘back to the
teristics of the rafters. Which of the fol- nature’ movement.
lowing characteristics is NOT described in (d) To judge the vanity in people.

ANSWERS
1. (b) 2. (d) 3. (c) 4. (b) 5. (a) 6. (d) 7. (c) 8. (b) 9. (b) 10. (a)
11. (d) 12. (b) 13. (c) 14. (a) 15. (c) 16. (b) 17. (d) 18. (b) 19. (d) 20. (a)
21. (c) 22. (b) 23. (a) 24. (c) 25. (d) 26. (a) 27. (b) 28. (a) 29. (b) 30. (d)
31. (c) 32. (a) 33. (d) 34. (c) 35. (b) 36. (c) 37. (a) 38. (c) 39. (b) 40. (a)
41. (c) 42. (d) 43. (c) 44. (d) 45. (a) 46. (d) 47. (c) 48. (d) 49. (b) 50. (a)
51. (c) 52. (b) 53. (a) 54. (b) 55. (d) 56. (a) 57. (c) 58. (a) 59. (d) 60. (c)
61. (a) 62. (d) 63. (b) 64. (c) 65. (a) 66. (b) 67. (d) 68. (b) 69. (a) 70. (b)
71. (d) 72. (a) 73. (d) 74. (c) 75. (a) 76. (b) 77. (d) 78. (b) 79. (c) 80. (a)
81. (d) 82. (a) 83. (c) 84. (a) 85. (d) 86. (b) 87. (a) 88. (c) 89. (d) 90. (c)

CLAT-6-Previous Years Papers.indd 426 06/09/2010 17:50:15


PREVIOUS YEARS’ PAPERS 427

91. (b) 92. (d) 93. (a) 94. (d) 95. (c) 96. (b) 97. (a) 98. (d) 99. (b) 100. (d)
101. (b) 102. (d) 103. (a) 104. (b) 105. (a) 106. (b) 107. (c) 108. (b) 109. (c) 110. (a)
111. (b) 112. (a) 113. (b) 114. (d) 115. (b) 116. (a) 117. (c) 118. (d) 119. (b) 120. (d)
121. (c) 122. (b) 123. (c) 124. (d) 125. (c) 126. (d) 127. (b) 128. (c) 129. (b) 130. (a)
131. (c) 132. (a) 133. (c) 134. (b) 135. (c) 136. (a) 137. (d) 138. (a) 139. (b) 140. (a)
141. (b) 142. (c) 143. (a) 144. (d) 145. (c) 146. (a) 147. (b) 148. (c) 149. (d) 150. (c)
151. (b) 152. (d) 153. (b) 154. (d) 155. (a) 156. (b) 157. (a) 158. (d) 159. (a) 160. (b)
161. (d) 162. (a) 163. (d) 164. (c) 165. (a) 166. (b) 167. (a) 168. (d) 169. (a) 170. (b)
171. (d) 172. (a) 173. (a) 174. (d) 175. (c)

CLAT-6-Previous Years Papers.indd 427 06/09/2010 17:50:15

You might also like